Oswaal CBSE Chapterwise & Topicwise Question Bank Class 10 English Language & Literature Book (For 2023 Exam) [18 ed.] 9355951698, 9789355951694

573 61 60MB

English Pages [411] Year 2022

Report DMCA / Copyright

DOWNLOAD FILE

Polecaj historie

Oswaal CBSE Chapterwise & Topicwise Question Bank Class 10 English Language & Literature Book (For 2023 Exam) [18 ed.]
 9355951698, 9789355951694

  • Commentary
  • decrypted from 7E1A5CDDD612D77E5E2871B967C87C8E source file

Table of contents :
Cover
Title Cover
Copyright Page
Contents
Syllabus
Solved Paper, 2021-22 (Term-II)
Section A : Reading Skills
1. Unseen Passages
> SELF ASSESSMENT PAPER – 01
Section B : Writing Skills With Grammar
2. Grammar
3. Gap Filling
4. Editing
5. Sentence Transformation
> SELF ASSESSMENT PAPER – 02
CREATIVE WRITING SKILLS
6. Letter Writing & Analytical Paragraph
> SELF ASSESSMENT PAPER – 03
Section B : Literature: Textbooks and Supplementary Reading Text (First Flight) Prose
1. A Letter to God
2. Nelson Mandela : Long Walk to Freedom
3. Two Stories About Flying
4. From the Diary of Anne Frank
5. Glimpses of India
6. Mijbil, the Otter
7. Madam Rides the Bus
8. The Sermon at Benares
9. The Proposal
(First Flight) Poetry
1. Dust of Snow
2. Fire and Ice
3. A Tiger in the Zoo
4. How to tell Wild Animals
5. The Ball Poem
6. Amanda !
7. The Trees
8. Fog
9. The Tale of Custard, The Dragon
10. For Anne Gregory
SUPPLEMENTARY READER
1. A Triumph of Surgery
2. The Thief’s Story
3. The Midnight Visitor
4. A Question of Trust
5. Footprints Without Feet
6. The Making of a Scientist
7. The Necklace
8. Bholi
9. The Book that saved the earth
> SELF ASSESSMENT PAPER – 04
WORDS AND EXPRESSIONS-II [Workbook in English]
1. A Letter To God
2. Nelson Mandela : Long Walk To Freedom
3. Two Stories About Flying
4. From The Diary of Anne Frank
7. Glimpses of India
8. Mijbil The Otter
9. Madam Rides The Bus
10. The Sermon At Benaras
11. The Proposal
> PRACTICE PAPER – 01
> PRACTICE PAPER – 02

Citation preview

VISIT @PROCBSE IN TELEGRAM TO GET FREE BOOKS

@PROCBSE

@PROCBSE

ENGLISH LANGUAGE & LITERATURE (First Flight, Footprints without Feet) (NCERT Workbook — Words & Expressions-II)

PROCBSE THANK YOU FOR DOWNLOADING BOOK. WE HERE AT PROCBSE MAKE SURE THAT EVERY PIECE OF MATERIAL REACHES YOU WITHOUT DIVING DEEP INTO THE TABS OF YOUR BROWSER. THIS E-BOOK HAS BEEN DOWNLOADED TOTALLY FREE FROM PROCBSE ON TELEGRAM. UPI ID - PROCBSE.FAMC@IDFCBANK

CLICK TO VIEW/OPEN

@ PROCBSE JOIN US FOR QUESTIONS BANK , SAMPLE PAPER , PYQ , NOTES , MIND MAPS AND MANY MOTE THINGS ⚡

@PROCBSE

You can contribute a little to our efforts. Even a small amount can do wonders.

DONATE US PLEASE HELP US

OR

CONTRIBUTE TO OUR UPI ID PROCBSE.FAMC@IDFCBANK

@PROCBSE

th

18 EDITION



“978-93-5595-169-4”

I SBN

YEAR 2022-23

SYLLABUS COVERED

CENTRAL BOARD OF SECONDARY EDUCATION DELHI

PUBLISHED BY

COPYRIG HT

RESERVED BY THE PUBLISHERS

All rights reserved. No part of this book may be reproduced, stored in a retrieval system, or transmitted, in any form or by any means, without written permission from the publishers. The author and publisher will gladly receive information enabling them to rectify any error or omission in subsequent editions.

OSWAAL BOOKS & LEARNING PVT. LTD. 1/11, Sahitya Kunj, M.G. Road, Agra - 282002, (UP) India

1/1, Cambourne Business Centre Cambridge, Cambridgeshire CB 236DP, United kingdom

0562-2857671

[email protected]

www.OswaalBooks.com

D I SC L A IMER

Oswaal Books has exercised due care and caution in collecting all the data before publishing this book. In spite of this, if any omission, inaccuracy or printing error occurs with regard to the data contained in this book, Oswaal Books will not be held responsible or liable. Oswaal Books will be grateful if you could point out any such error or offer your suggestions which will be of great help for other readers. Kindle Edition

@PROCBSE

TABLE OF CONTENTS Latest CBSE Circular released on 20th May for Academic Year 2022-2023 (CBSE Cir. No. Acad 57/2022) l Latest CBSE Syllabus released on 21st April 2022 for Academic Year 2022-2023





l



(CBSE Cir. No. Acad 48/2022)



given on Page 24)

l CBSE Solved Board Papers, 2022 Term-II Examination (To download Solved paper for Term-I 2021-22 & Latest Topper's Answers 2020, scan the QR Code

Section A : Reading Skills 1. Unseen Passages 1 - 36 Topic : 1 - Discursive Passages Topic : 2 - Case-based Factual Passages SELF ASSESSMENT PAPER – 01 37 - 40

Section B : Writing Skills With Grammar

2. Grammar 41 - 53 54 - 60 l Grammar Charts 3. Gap Filling 61 - 70 4. Editing 71 - 79 5. Sentence Transformation 80 - 87 SELF ASSESSMENT PAPER – 02 88 - 90 6. Letter Writing & Analytical Paragraph 91 - 113 Topic : 1    - Formal Letter Writing Topic : 2    - Analytical Paragraph Writing SELF ASSESSMENT PAPER – 03 114 - 115

Literature: Textbooks and Supplementary Reading Text (First Flight) Prose

7 - 7 8 - 11

15 - 24

6. Mijbil, the Otter 158 - 163 7. Madam Rides the Bus 164 - 170 8. The Sermon at Benares 171 - 177 9. The Proposal 178 - 185

(First Flight) Poetry 1. Dust of Snow 186 - 191 2. Fire and Ice 192 - 197 3. A Tiger in the Zoo 198 - 204 4. How to tell Wild Animals 205 - 210 5. The Ball Poem 211 - 216 6. Amanda ! 217 - 222 7. The Trees 223 - 228 8. Fog 229 - 231 9. The Tale of Custard, The Dragon

232 - 238

10. For Anne Gregory 239 - 243

Supplementary Reader:

Footprints Without Feet

1. 2. 3. 4. 5.

A Letter to God 116 - 122 Nelson Mandela : Long Walk to Freedom 123 - 127 Two Stories About Flying 128 - 138 (A) His First Flight (A) Black Aeroplane From the Diary of Anne Frank 139 - 144 Glimpses of India 145 - 157 I. A Baker From Goa

1. A Triumph of Surgery 244 - 250

9. The Book that saved the earth

291 - 295



III. Tea From Assam

SELF ASSESSMENT PAPER – 04

296 - 296



II. Coorg

2. The Thief’s Story 251 - 255 3. The Midnight Visitor 256 - 260 4. A Question of Trust 261 - 265 5. Footprints Without Feet 266 - 270 6. The Making of a Scientist 271 - 276 7. The Necklace 277 - 283 8. Bholi 284 - 290

(3)

@PROCBSE

TABLE OF CONTENTS NCERT Workbook - Words & Expressions-II 1. A Letter To God 297 - 306

8. Mijbil The Otter 333 - 340



10. The Sermon At Benaras 347 - 352

2. Nelson Mandela :

Long Walk To Freedom 307 - 313

3. Two Stories About Flying 314 - 320 4. From The Diary of Anne Frank

321 - 325

7. Glimpses of India 326 - 332

9. Madam Rides The Bus 341 - 346 11. The Proposal 353 - 359 PRACTICE PAPER – 01 360 - 367 PRACTICE PAPER – 02

368 - 374

qq

(4)

@PROCBSE

(5)

@PROCBSE

PREFACE Who is this book for? We are delighted to present the updated edition of the CBSE Questions Banks for the 2022-23 academic session for Class 10. This edition is based on the latest CBSE Syllabus and Scheme of Assessment and Evaluation Practices of the Board for the Session 2022-23. National Education Policy 2020 has affirmed the need to move from rote to competency based learning. The Board has taken multiple steps towards the implementation of Competency Based Education (CBE) to equip the learning to meet 21st Century challanges proactively.

Why is this book different? The structural changes in Education that are being brought in by the latest Syllabus and the new National Educational Policy (NEP) resonate with our approach of focusing on deeper understanding instead of rote learning. Available for all subjects, these books have been designed to help students achieve high scores in their Board Examinations. Prepared by the Editorial Board of Oswaal Books, comprising leading subject matter experts, it consists of CBE pedagogical features such as Multiple Choice Questions, Case based Questions, Source based Integrated Questions alongwith Objective Types, Short & Long Answer Types Questions - all of which reinforce learning and improve conceptual understanding. To make these books 100% exam-centric, we've also includedl



l l l l l l l

Latest Board Exam Papers (all sets of Delhi & Outside Delhi) of CBSE Term-II 2022 Topic-wise/Concept wise segregation of chapters Important Keywords for quick recall of concepts Fundamental Facts to enhance knowledge Practice Questions within the chapters for better practice Reflection to ask about your learnings Self-Assessment Tests & Practice Papers for Self-Evaluation Art Integration for Experiential learning

The new way of learning; Blended Learning

The pandemic introduced us all to a phenomenon Blended Learning. In no time e-learning has become mainstream. Oswaal Books identified this like an opportunity and thus, we decided to introduce Oswaal360 A platform that simplifies learning with Chapter-wise e-assessments articulated for students to crack Board Exams.

Other Important features of this book such as-Art Integrated activities, Mind Maps, Concept Videos and Academically Important (AI) Questions make it an indispensable resource for students desiring to secure top marks in the examination.

How to use this book? You should use this book to supplement your learning and test your conceptual understanding. The rich instructive pedagogy encourages critical thinking and reflection and acts as a handy resource for students to learn, course correct, and test their knowledge. Making this book a part of your day-to-day exam preparation routine will help you sharpen your conceptual understanding, enhance your application to exam questions, and help you improve your test-taking abilities.

We wish you great success for your Board Examinations in 2023 and hope this book helps you achieve exemplary success. Team Oswaal Books (6)

@PROCBSE

CBSE CIRCULAR 2022-23 dsUæh; ek/;fed f'k{kk cksMZ

CENTRAL BOARD OF SECONDARY EDUCATION Dated: 20.05.2022



CBSE/DIR(ACAD)/2022/

Circular No. ACAD-57/2022

All Heads of schools affiliated to CBSE







Subject : Assessment and Evaluation Practices of the Board for the Session 2022-23 National Education Policy 2020 has affirmed the need to move from rote to competency-based learning. This will equip the learners with key competencies to meet the challenges of the 21st century proactively. Accordingly, the Board has taken multiple steps towards the implementation of Competency Based Education (CBE) in schools. These range from aligning assessment to CBE, development of exemplar resources for teachers and students on CBE pedagogy and assessment and continued teacher capacity building. In this context the Board has released Circular No. Acad-05/2019 dated 18.01.2019; Circular No. Acad-11/2019 dated 06.03.2019; Circular No. Acad-18/2020 dated 16.03.2020; Circular No. Acad-32/2020 dated 14.05.2020 and Circular No. Acad-31/2020 dated 22.04.2021. In continuation to these circulars, the Board is initiating further corresponding changes in the Examination and Assessment practices for the year 2022-23 to align assessment to Competency Based Education. Therefore, in the forthcoming sessions a greater number of Competency Based Questions or questions that assess application of concepts in real-life/ unfamiliar situations will be part of the question paper. The changes for classes IX-X (2022-23) internal year-end/Board Examination are as detailed: (Classes IX-X) Year End Examination/ Board Examination (Theory)

(2021-22) Existing (As per Special Scheme of Assessment for Board Examination – Circular No. Acad51/2021 dated 05.07.2021)

(2022-23) Modified (Annual Scheme)

Composition

• Term I – Multiple Choice Question including case based and assertion reasoning type MCQs – 100% (30% questions competency based) • Term II – Case based/ Situation based, Open Ended- short answer/long answer questions (30% questions competency based)

• Competency Based Questions would be minimum 40% These can be in the form of Multiple Choice Questions, Case based Questions, Source based Integrated Questions or any other types. • Objective Type Questions will be 20% • Remaining 40% short

Composition

• Term I – Multiple Choice Question including case based and assertion reasoning type MCQs – 100% (30% questions competency based) • Term II – Case based/ Situation based, Open Ended- short answer/long answer questions (30% questions competency based)

• Competency Based Questions would be minimum 40% These can be in the form of Multiple Choice Questions, Case based Questions, Source based Integrated Questions or any other types. • Objective Type Questions will be 20% • Remaining 40% short answer/long answer questions (as per existing pattern)

Internal Assessment : No change Internal Assessment: End of year examination = 20:80 Curriculum document released by the Board vide circular No.Acad-50/2022 dated 28th April, 2022 and the forthcoming Sample Question Papers may be referred for the details of changes in the QP design of individual subjects.









Year End Examination/ Board Examination (Theory)

(7)

(Dr. Joseph Emmanuel) Director (Academics)

@PROCBSE

SYLLABUS English Language and Literature Class - X (Code No. 184)

Latest Syllabus issued by CBSE dated 21st April 2022 for Academic Year 2022-23 Section - Wise Weightage







Sections

Unit

A

Reading Skills

(40 periods)

B

Writing Skills with Grammar

(40 periods)

C

Literature Textbooks and Supplementary Reading Text

(50 periods)

Total

PART - A

Section A Reading Skills Reading Comprehension through Unseen Passage

20 Marks











I. Discursive passage of 400-450 words. (10 marks) II. Case-based passage (with visual input- statistical data, chart etc.) of 200-250 words. (Total length of two passages to be 600-700 words). (10 marks) Multiple Choice Questions / Objective Type Questions will be asked to assess inference, analysis, interpretation, evaluation and vocabulary.

Section B Writing Skills with Grammar

























Grammar (10 marks) • Tenses • Modals • Subject – verb concord • Reported speech o Commands and requests o Statements o Questions o Determiners The courses at the secondary level seek to cement high professional grasp of grammatical items and levels of accuracy. Accurate use of spelling, punctuation and grammar in context will be assessed through Gap Filling/ Editing/Transformation exercises. Ten out of 12 questions will have to be attempted. IV. Creative Writing Skills (10 marks) This section will have short as well as long writing tasks including compositions. I. Formal Letter based on a given situation in 100-120 words. One out of two questions is to be answered. 5 marks II. Writing an Analytical Paragraph (100-120 words) on a given Map / Chart / Graph / Cue/ s . One out of two questions is to be answered. 5 marks

III.

Section C Literature Textbooks and Supplementary Reading Text









V. Reference to the Context I. One extract out of two from Drama / Prose. II. One extract out of two from poetry. (5+5 = 10 Marks) Multiple Choice Questions / Objective Type Questions will be asked to assess inference, analysis, interpretation, evaluation and vocabulary.

(8)

@PROCBSE

SYLLABUS 30 Marks

Four out of Five Short Answer Type Questions to be answered in 40-50 words from the book FIRST FLIGHT



I.





VI. Short & Very Long Answer Questions

4×3=12 marks





II. Two out of Three Short Answer Type Questions to be answered in 40-50 words each from FOOTPRINTS WITHOUT FEET. 2×3=6 marks



III. One out of two Long Answer Type Questions from FIRST FLIGHT to be answered in about 100-120 words each to assess creativity, imagination and extrapolation beyond the text and across the texts. This can be a passage-based question taken from a situation/plot from the texts. 6 marks



IV. One out of two Long Answer Type Questions from FOOTPRINTS WITHOUT FEET on theme or plot involving interpretation, extrapolation beyond the text and inference or character sketch to be answered in about 100-120 words. 6 marks Prescribed Books: Published by NCERT, New Delhi FIRST FLIGHT Prose

9. The Proposal (Play)







8. The Sermon at Benares











7. Madam Rides the Bus



6. Mijbil the Otter



3. Two Stories About Flying

5. Glimpses of India



2. Nelson Mandela - Long Walk to Freedom

4. From the Diary of Anne Frank



1. A letter to God









9.



6.

8. Fog



5. The Ball Poem



3.



2. Fire and Ice





7. The Trees





4. How to Tell Wild Animals





1. Dust of Snow



Poems A Tiger in the Zoo Amanda! The Tale of Custard the Dragon



10. For Anne Gregory



9.







8. Bholi



6.



3.

5. Footprints Without Feet



2. The Thief's Story





7. The Necklace





4. A Question of Trust





1. A Triumph of Surgery



FOOTPRINTS WITHOUT FEET The Midnight Visitor The Making of a Scientist The Book That Saved the Earth



Note: Teachers are advised to: encourage interaction among peers, students and teachers through activities such as role play, discussions, group work etc.



(i)



(ii) reduce teacher-talking time and keep it to the minimum,

(iii) take up questions for discussion to encourage pupils to participate and to marshal their ideas and express and defend their views, and (iv) follow the Speaking and Listening activities given in the NCERT books.











3. WORDS AND EXPRESSIONS – II (WORKBOOK FOR CLASS X) – Units 1 to 4 and Units 7 to 11

Besides measuring learning outcome, texts serve the dual purpose of diagnosing mistakes and areas of nonlearning. To make evaluation a true index of learners’ knowledge, each language skill is to be assessed through a judicious mixture of different types of questions. INTERNAL ASSESSMENT Listening and Speaking Competencies

30 Periods

Assessment of Listening and Speaking Skills will be for 05 marks. It is recommended that listening and speaking skills should be regularly practiced . Art-integrated projects based on activities like Role Play, Skit, Dramatization etc. must be used. Please refer to th the Circular no. Acad-33/2020 dated 14 May 2020 at the http://cbseacademic.nic.in/web_material/Circulars/2020/33_Circular_2020.pdf for details Guidelines for the Assessment of Listening and Speaking Skills are given at Annexure I.

(9)

@PROCBSE

SYLLABUS ENGLISH LANGUAGE AND LITERATURE (Code No. 184)

Marks : 80





CLASS-X

Sections

Competencies

Total marks

Reading Comprehension

Conceptual understanding, decoding, inferring, interpreting and vocabulary

Writing Skill and Grammar

Creative expression of an opinion, reasoning, justifying, illustrating, appropriacy of style and tone, using appropriate format and fluency. Applying conventions, using integrated structures with accuracy and fluency

20

Recalling, reasoning, appreciating, applying literary conventions illustrating and justifying etc. Extract relevant information, identifying the central theme and subtheme, understanding the writers’ message and writing fluently.

40

Literature Textbook and Supplementary Reading Text

analyzing,

20

Total

80

For the details of Internal Assessment of 20 marks, please refer to the circular no. Acad-11/2019, dated March 06, 2019.

Annexure I i. Activities



Guidelines for Assessment of Listening and Speaking Skills (ALS)

• Subject teachers must refer to books prescribed in the syllabus.



• In addition to the above, teachers may plan their own activities and create their own material for assessing the listening and speaking skills.



a. Interactive competence (Initiation & turn taking, relevance to the topic)

b. Fluency (cohesion, coherence and speed of delivery)

c. Pronunciation d. Language (grammar and vocabulary)











ii. Parameters for Assessment: The listening and speaking skills are to be assessed on the following parameters:

A suggestive rubric is given below:



( 10 )



5. • Can initiate & logically develop simple conversation on familiar topics • Can take turns appropriately



4. • Interaction is adequately initiated and develop • Can take turn but needs little prompting









1. 2. 3. • Contributions • Contributions • Develops are mainly unreare often unreinteraction lated to those of lated to those of adequately, other speakers the other speaker makes how• Shows hardly • Generally passive ever minimal any initiative in the developeffort to initiin the development of converate conversament of conversation tion sation • Needs con• Very limited stant promptinteraction ing to take turns

Interaction

@PROCBSE

SYLLABUS

• Usually fluent; produces simple speech fluently, but loses coherence in complex communication • Often hesitates and/or resorts to slow speech • Topics partly developed; not always concluded logically

• Is willing to speak at length, however repetition is noticeable • Hesitates and/ or self corrects; occasionally loses coherence • Topics mainly developed, but usually not logically concluded

Vocabulary & Grammar

• Demonstrates almost no flexibility, and mostly struggles for appropriate words • Uses very basic vocabulary to express viewpoints.

• Communicates • Communi• Can express • Can express with limited cates with with some with some flexibility and limited flexflexibility and flexibility appropriacy on ibility and appropriacy and approsome of the topappropriacy on most of the priacy on a ics on most of the topics variety of • Complex forms topics • Demonstrates topics such and sentence • Sometimes ability to as famstructures are uses complex use complex ily, hobbies, rare; exhibits limforms and forms and work, travel ited vocabulary sentence sentence and current to express new structures; structures events ideas has limited most of the • Frequently vocabulary time; exuses complex to describe/ presses with forms and express new adequate sentence points vocabulary structures; has enough vocabulary to express himself/ herself



















• Speaks with• Speaks fluout noticeable ently almost effort, with a with no little repetirepetition tion & minimal • Demonstrates hesitation hesitation to • Develops find words topic fully & or use correct coherently grammatical structures and/or selfcorrection • Topics not fully developed to merit











• Noticeably/ long pauses; rate of Speech is slow • Frequent repetition and/or self- correction • Links only basic sentences; breakdown of coherence evident



Fluency & Coherence



• Largely cor• Mostly correct • Can prorect pronunpronunciation nounce ciation & clear & clear articucorrectly & articulation lation articulate except occa• Can be clearly clearly sional errors understood • Is always • Some expresmost of the comprehensions cause time; very few sible ; uses stress without phonological appropriate compromiserrors intonation ing with understanding of spoken discourse.



• Insufficient • Frequently accuracy in unintelligible pronunciation; articulation many grammati- • Frequent phonocal errors logical errors • Communication • Major communiis severely afcation problems fected

Pronunciation









iii. Schedule: The practice of listening and speaking skills should be done throughout the academic year. The final assessment of the skills is to be done as per the convenience and schedule of the school. qqq

( 11 )

@PROCBSE

Professional skills, do I need to get better at them?

Is engineering meant for me?

Humanities

or Commerce?

Which stream should I go for?

Am I good at this skill?

Oswaal Books brings you a bunch of tests that will help you find answers to the above questions

To test yourself visit- www.OswaalBooks.com or Scan the QR code below

( 12 )

@PROCBSE

What is ART INTEGRATION? “When I see the universe through my songs, I recognize it, then only I know it.” - ”Gitabitan”by Ravindranath Tagore

A rt has always been a great part of our lives. It is usually defined as the creative

expression of an individuals but more often, Art defines us. From the care paintings to great artifacts found in Harappan Civilisation to the latest animation software, Art has always been there surrounding every aspect of our lives from the beginning of time. Art, not only help us understand things around us but at times we understand through it. As Rabindranath Tagore once said, "…. only man knows……. objects". Over the centuries, the ways of learning has gone through many refinements and as a result it has become a great deal easier for students to obtain knowledge dividing the ways of learning. Remember when the visuals of rips apples helped us learn about the color 'red' and the 'twinkle-twinkle' melody first introduced us to the Celestial bodies in our space. Even in the primary education, schools promote Art, Dance, Crafts, Theatre. But as students progress the higher classes and core academic subjects take the stage, they are somehow left with scribbled words and symbole and Art is either forgotten or relegated to insignificant background. It is with hopes of connecting "ever-growing" minds once again with their artistic expression and ability to not only understand and learn them but truly recognise them, a cross-circulation approach to teaching and learning is introduced by CBSE, known as Art Integration.

Art Integration is an art-based approach to make learning joyful alongwith allowing students to have greater appreciation and understanding of the subject and its application. Based on the collaboration between the teaching of subject and subject of Arts, it aims at better understanding of subject with arts and its multiple forms being the primary pathway of learning it. We at Oswaal, have always valued the improtance of simplifying the learning process and have welcomed this creative approach of learning in our books, with open arms. We have incorporated Art Integration in our Question Banks in forms of subject related fun-case based studies and various interactive visual videos to make learning not only fun but allowing deeper understanding of subjects in students mind. Now, that's learning truly made simple.

 

For experiential learning through Art Integrated Activities, scan the code below-

OR SCAN

Scan the QR code

( 13 )

( 14 )

What are Associations?

To Make clearer and better notes To Concentrate and save time To Plan with ease and ace exams

To Unlock the imagination and come up with ideas To Remember facts and

Associations are one powerful memory aid connecting seemingly unrelated concepts, hence strengthening memory.

move to the subsequent levels of association. This is exactly how the brain functions, therefore these Mind Maps.

level and the chronology continues. The thickest line is the First Level of Association and the lines keep getting thinner as we

from the core concept are the First Level of Association. Then we have a Second Level of Association emitting from the first

It’s a technique connecting the core concept at the Centre to related concepts or ideas. Associations spreading out straight

Learning made simple ‘a winning combination’

Why?

with a blank sheet of paper coloured pens and your creative imagination!

How?

MIND MAP

When?

AN INTERACTIVE MAGICAL TOOL

Result

What?

presenting words and concepts as pictures!!

anytime, as frequency as you like till it becomes a habit!

Learning MaDE SimpLE

MIND MAPS

@PROCBSE

@PROCBSE

Solved Paper, 2021-22 ENGLISH LANGUAGE & LITERATURE Term-II, Outside Delhi Set-1 Question Paper Code No. 2/4/1

Series : DPQRS

Max. Marks : 40



Time allowed : 90 Minutes

General Instructions :























(i) (ii) (iii) (iv) (v) (vi) (vii)

The Question paper contains three sections READING, WRITING & GRAMMAR AND LITERATURE. Attempt question based on specific instructions for each Part/Section. Section–B has 12 questions. Attempt any 10 questions. Section–C has 30 questions. Attempt any 26 questions. All questions carry equal marks. There is no negative marking. Attempt the number of questions as given in the direction of main heads.







































1. Read the passage given below: (1) Milkha Singh, also known as The Flying Sikh, was an Indian track and field sprinter who was introduced to the sport while serving in the Indian Army. He is the only athlete to win gold in 400 metres at the Asian Games as well as the Commonwealth Games. He also won gold medals in the 1958 and 1962 Asian Games. He represented India in the 1956 Summer Olympics in Melbourne, the 1960 Summer Olympics in Rome and the 1964 Summer Olympics in Tokyo. He was awarded the Padma Shri, India's fourth-highest civilian honour, in recognition of his sporting achievements. (2) The race for which Singh is best remembered is his fourth-place finish in the 400 metres final at the 1960 Olympic Games. He led the race till the 200 m mark before easing off, allowing others to pass him. Singh's fourth-place time of 45.73 seconds was the Indian national record for almost 40 years. (3) From beginning that saw him orphaned and displaced during the partition of India, Singh became a sporting icon in the country. In 2008, journalist Rohit Brijnath described Singh as "the finest athlete India has ever produced". (4) He was disappointed with his debut performance at the 1956 Melbourne Olympics. "I returned to India, chastened by my poor performance in Melbourne. I had been so excited by the prospects of being part of the Indian Olympics team, but, hadn't realized how strong and professional the competition would be. My success in India had filled me with a false sense of pride and it was only when I was on the track that I saw how inconsequential my talents were when pitted against superbly fit and seasoned athletes. It was then that I understood what competition actually meant, and that if I wanted to succeed on the international arena, I must be prepared to test my mettle against the best athletes in the world.” (5) Then he decided to make sprinting the sole focus of his life. “Running had thus become my God, my religion and my beloved”. "My life during those two years was governed by strict rules and regulations and a self-imposed penance. Every morning I would rise at the crack of dawn, get into my sports kit and dash off to the track, where I would run two or three miles cross-country in the company of my coach.” (6) On how he pushed himself through the tough days of vigorous training. “I practiced so strenuously that often I was drained of all energy, and there were times when I would increase my speed to such an extent that after my rounds, I would vomit blood or drop down unconscious through sheer exercise. My doctors and coaches warned me, asked me to slow down to maintain my health and equilibrium but my determination was too strong to give up. My only focus was to become the best athlete in the world. But then images of a packed stadium filled with cheering spectators, wildly applauding me as I crossed the finishing line, would flash across my mind and I would start again, encouraged by visions of victory." Based on your reading answer any five questions from the six given below: 1×5=5 (i) What is Milkha Singh known as ? What realization did Milkha Singh have when he was on the track during the Melbourne Olympics?



SECTION-A : READING

@PROCBSE











On the basis of your understanding of the passage answer any five of the six questions given below. (i) Explain J.K. Rowling's 'near magical rise to fame'. (ii) What reason did the publishers give for rejecting Rowling's book ? (iii) What was the drawback of achieving fame? (iv) Why was Rowling outraged with the Italian dust jacket? (v) Find a word in the last para that means the same as 'insecure/helpless'. (vi) According to the graph, how many years did it take Rowling to become very successful ?









1×5=5



















(ii) List any two of Milkha Singh's achievements. (iii) What strict rules and regulations did Milkha Singh follow ? (iv) State two consequences of his hard and strenuous practice. (v) What motivated Milkha Singh to become the best athlete in the world? (vi) Explain the phrase 'I would start again' in the last sentence. 2. Read the following excerpt from a Case Study. J.K. Rowling - A Journey The story of Joanne Kathleen Rowling's near magical rise to fame is almost as well known as the characters she creates. Rowling was constantly writing and telling stories to her younger sister Dianne. “The first story I ever wrote down was about a rabbit called Rabbit.” Rowling said in an interview. "He got the measles and was visited by his friends including a giant bee called Miss Bee. And ever since Rabbit and Miss Bee, I have always wanted to be a writer, though I rarely told anyone so. However, my parents, both of whom come from impoverished backgrounds and neither of whom had been to college, took the view that my overactive imagination was an amusing personal quirk that would never pay a mortgage or secure a pension. A writer from the age of six, with two unpublished novels in the drawer, she was stuck on a train when Harry walked into her mind fully formed. She spent the next five years constructing the plots of seven books, one for every year of his secondary school life. Rowling says she started writing the first book, Harry Potter and the Sorcerer's Stone, in Portugal, where she was teaching English. At first nobody wanted to publish Harry Potter. She was told that plot was too complex. Refusing to compromise, she found a publisher. In 1997 Rowling received her first royalty cheque. By book three, she had sky rocketed to the top of the publishing world. A row of zeroes appeared on the author's bank balance and her life was turned upside down. Day and night she had journalists knocking on the unanswered door of her flat. Rowling's quality control has become legendary, as her obsession with accuracy. She's thrilled with Stephen Fry's taped version of the books and outraged that an Italian dust jacket showed Harry minus his glasses. "Don't they understand that the glasses are the clue to his vulnerability.” Annual earnings of J.K. Rowling from 2010 to 2019





Oswaal CBSE Question Bank Chapterwise & Topicwise, ENGLISH LANGUAGE & LITERATURE, Class-X





16











3. Attempt any one from (i) and (ii). (i) A group of enterprising young adults conducted a survey on reopening of educational institutions in the National Capital 5





SECTION-B : GRAMMAR & WRITING

Write a paragraph in about 120 words analysing the listed responses.

@PROCBSE SOLVED PAPER – 2022 (Term-II)



(ii) You are Vikram/Vanya, Librarian, B.S. Public School, Delhi. Write a letter to the Manager of Amar Publication House, Mumbai, placing an order for 4 sets of Social Studies books for grades 6-8 published by N.C.E.R.T. in not more than 120 words. Mention school discount, mode of payment and date of delivery. 4. The following paragraph has an error in each line. Identify the error and write its correction against the correct blank number. The first one has been done as an example. 1×3=3 Incorrect Correct Air pollution is emerging like a eg like as global problem among the world. (a) ______ ______ Much of our cities have (b) ______ ______ industries which is responsible for 20% of the (c) ______ ______ pollutants. 5. Read the following conversation and complete the passage that follows: 2 Neha: I'm really looking forward to the class picnic tomorrow. Nanita: Yes, after a long time we will be meeting our friends and teachers. Neha told Namita (1) ..................... looking forward to the class picnic ....................., Namita agreed that after a long time (2) ..................... friends and teachers.

















































17































































6. Answer any six questions in about 30-40 words each. 2 × 6 = 12 (i) What does the writer recall about his childhood in Goa ? (Glimpses of India) (ii) "These sights so moved him that he at once went out into the world to seek enlightenment." What were the sights that moved ‘him’? (The Sermon at Benaras) (iii) Mention any two issues over which Amanda was nagged and scolded ? (iv) Why does the poet wish to 'turn' and live with animals ? (Animals) (v) Describe Custard the dragon. (vi) What did Ebright learn from his failure at the Science fair ? (The making of a scientist) (vii) How did Lutkins impress the lawyer? (The Hack Driver) 7. Answer any two of the following in about 120 words each. 4×2 (i) As Valli, make a diary entry sharing your joys and disappointments during the bus ride. 4 (ii) Natalya and her father Chubukov ‘turn around’ when Lomov puts his proposal in the open. As a reader briefly describe what you think of Natalaya and Chubukov from the play “the Proposal”. 4 (iii) The credit of Bholi's transformation to Sulekha goes to her teacher. Elaborate with reference to the text. 4



SECTION-C : LITERATURE

Term-II, Outside Delhi Set-II Question Paper Code No. 2/4/2

Series : DPQRS Note : Except these all other Questions are from Set - I

































6. Answer any six questions in 30-40 words each. 2 × 6 = 12 (i) What impressed Mr. Barua about Rajiv ? (Glimpses of India) (ii) Kisa Gotami admitted that she was being selfish in grief. Do you agree? Why/Why not? (The Sermon at Benaras). (iii) Amanda did not agree with the restrictions put on her. Give two reasons to support the statement. (iv) Mention any two reasons why Walt Whitman appreciates animals more than humans. (Animals) (v) Describe the pirate in the ‘Tale of Custard the Dragon.’ (vi) Why did Ebright start a project of tagging butterflies? (vii) Why did the young lawyer dislike his job in the Law firm ? (The Hack Driver)



SECTION-C : LITERATURE

Term-II, Outside Delhi Set-III Question Paper Code No. 2/4/3

Series : DPQRS Note : Except these all other Questions are from Set - I & Set - II













6. Answer any six questions in about 30-40 words each. (i) Describe the climb to the Brahmagiri hills. (Glimpses of India) (ii) Why did Kisa Gotami become weary and hopeless ? (The Sermon at Benaras).



SECTION-C : LITERATURE 2 × 6 = 12

@PROCBSE Oswaal CBSE Question Bank Chapterwise & Topicwise, ENGLISH LANGUAGE & LITERATURE, Class-X









(iii) ‘I am an orphan, roaming the street’. Why does Amanda visualize this ? (iv) List any two things that animals do and humans don't. (Animals) (v) Why did the pirate 'gape' at Belinda's dragon ? (Tale of Custard the Dragon) (vi) How was ‘The Travels of Monarch X’ a turning point in Ebright's life ? (The Making of a Scientist) (vii) Describe the young lawyer's first encounter with the hack driver.

















18

Term-II, Delhi Set-I Question Paper Code No. 2/2/1

Series : CPQRS/2

SECTION-B : GRAMMAR & WRITING

















































































For Q1 & Q2 refer Outside Delhi Set - I Q 1 & Q 2 3. Attempt any one from (i) and (ii). 10 (i) A survey was conducted in the National Capital in over twenty schools about the different activities that interest the children in the age group of 13 years to 15 years. Write a paragraph in not more than 120 words, analyzing the following information. 5 OR (ii) You are Niharika, Incharge of the Neighbourhood Library for children. Write a letter to Manautal Publishers placing an order for books 2 Sets each of Panchatantra and Amar Chitra Katha in about 120 words. Mention mode of Payment, library discount and date of delivery. 4. The following paragraph has an error in each line with a blank. Write the correct and the incorrect word in the blanks provided. The first one has been done for you. 1×3=3 Incorrect Correct A first Indian woman physician eg (a) __A___ _The_ Anandibai Joshi graduated at 1886. (b) ______ ______ About 125 years later, Indian women had start to outnumber men in (c) ______ ______ admissions of medical colleges. (d) ______ ______ 5. Read the conversation and complete the passage that follows: 3 Ritika: Can I borrow your Math book for a couple of days? Mohit: Yes certainly, I have already studied for the test tomorrow. Ritika asked Mohit (1)_____________for a couple of days. Mohit agreed and said that (2)___________.





























































6. Answer any six questions in about 30-40 words each. 2 × 6 = 12 (i) Why is Coorg called the land of the rolling hills? (Glimpses of India). (ii) What was the objective of Lomov's visit to Chubukov's house? (The Proposal) (iii) Ananda was blamed for a number of things. Mention any two. (iv) Walt Whitman speaks of the human race with a tinge of sarcasm. Which are the two reasons that you agree with? (Animals) (v) How did Custard prove ‘the cowardly dragon’ tag to be untrue? (vi) What role did Ebright's mother play in his success? (The Making of a Scientist) (vii) How was the hack driver recognised? 7. Answer any two of the following in about 120 words each. 4×2 (i) How does Buddha bring about a different perspective in Kisa Gotami's understanding of life? (The Sermon at Benaras) 4 (ii) As a reader do you sympathise with Matilda? Give reasons from the text to support your answer. (The Necklace) 4 (iii) As Valli, make a diary entry your experience of riding the bus alone for the first time. (Madam Rides the Bus) 4



SECTION-C : LITERATURE

Term-II, Delhi Set-II Question Paper Code No. 2/2/2

Series : CPQRS/2 Note : Except these all other Questions are from Delhi Set - I













6. Answer any six questions in about 30-40 words each. (i) Why are Kodavus permitted to carry fire arms without licence? (Glimpses of India) (ii) Why did Chubukov misunderstand the purpose of Lomov's visit? (The Proposal)



SECTION-C : LITERATURE 2 × 6 = 12

@PROCBSE SOLVED PAPER – 2022 (Term-II)

19



(vi) How did Ebright get the idea of his new theory about cell life? (The Making of a Scientist) (vii) Why was the narrator happy to go to New Mullion? (The Hack Driver) 4















(iii) As a reader do you identify with Amanda? How? (iv) What two attributes of the Animals please the poet? (v) What did everyone laugh at the realio, trulio dragon? (The Tale of Custard the Dragon)













Term-II, Delhi Set-III Question Paper Code No. 2/2/3



































Proposal) (iii) Why does Amanda wish to be a mermaid? (iv) ‘They do not sweat and whine about their condition.’ What human traits does the poet condemn in these lines? (Animals) (v) How was Belinda's behaviour with the dragon? (The Tale of Custard the Dragon) (vi) Why did Viceroy butterflies copy the Monarchs? (The Making of a Scientist) (vii) How was Lutkins described by the hack driver?



6. Answer any six questions in about 30-40 words each. 2 × 6 = 12 (i) Kodavus have contradictory traits. Support the statement with reference to the text. (Glimpses of India) (ii) Why does Natalaya quarrel with Lomov? (The



SECTION-C : LITERATURE



Note : Except these all other Questions are from Delhi Set - I & II



Series : CPQRS/2

ANSWERS Term-II, (Outside Delhi Set-I) Question Paper Code No. 2/4/1

Series : DPQRS





(ii)





(iii)





(iv)





(v)





(vi)





strong determination. J.K. Rowling's ‘near magical rise to fame’ mentions her incredible journey of becoming a famous writer. Publishers rejected her book, saying that the plot of her story was too complex. After achieving fame, her life was turned upside down. Journalists used to visit and disturb her personal life at the odd hours. Rowling was outraged with the Italian dust jacket because it showed Harry without his peculiar feature, i.e., his glasses. ‘Vulnerability’ means the same as insecure/ helpless. Rowling took 6 years to become very successful.

SECTION-B Ans. 3. (i)





(iv)

(vi)





(iii)

(v)

Milkha Singh is known as ‘The Flying Sikh’. When he was on the track during the Melbourne Olympics, he realised the actual meaning of competition. He felt that he must be well prepared to test his mettle against the best athletes of the world in order to succeed on an international plateform. (a) Milkha Singh is the only Indian to win gold in 400 metres in Asian Games as well as in the Commonwealth Games. (b) He was awarded with the Padma Shri, India's fourth highest civilian honour. Milkha Singh practised hard to achieve his goal of becoming the best athlete of the world. He used to run 2-3 miles a day early in the morning. His hard and strenuous practice drained out his energy leading him to vomit blood and sometimes dropping him down unconscious. The view of packed stadium, full of cheering spectators, who applauded Milkha Singh as he crossed the finishing line, motivated him to become the best athlete in the world. Very often, vigorous training drained Milkha Singh's energy, resulting in poor health, even to the extent of sometimes making him unconscious. But he never gave up. The sight of cheering spectators encouraged him to start again, running towards his goal with









(ii)

















Ans. 1. (i)



Ans. 2. (i)

SECTION-A

In light of current education crises due to COVID-19, a survey on reopening of educational institutions in the National Capital was conducted by a group of enterprising young adults. 28% people opined that the educational institutions should allow the students and other staff only after complete vaccination, i.e. they should be allowed to enter the campus only after they are vaccinated. As many as 24% people believed that schools should reopen only for exams/practical work; whereas 20% people were of view that it would be wise

@PROCBSE 20

Oswaal CBSE Question Bank Chapterwise & Topicwise, ENGLISH LANGUAGE & LITERATURE, Class-X

1.

A complete set of books of Social Studies (for grade 6)

N.C.E.R.T.

4

2.

A complete set of books of Social Studies (for grade 7)

N.C.E.R.T.

4

3.

A complete of books of Social Studies (for grade 8)

N.C.E.R.T.

4































































We shall not settle for less than 15% discount on the purchase of the above mentioned books. Besides, we request you to give another 5% special discount that is usually granted to the institutional buyers. It goes without saying that you will have to bear the mail and other expenses involved in the dispatch and delivery of books. The above books must reach us in proper th condition and order not later than 6 May if our terms and conditions are acceptable to you. Payment will be made through Googlepay. Thanking you Yours sincerely Vanya (Librarian) Ans. 4. Incorrect Correct (a) among in/all over (b) Much Most (c) is are Ans. 5. (1) that she was really ................ the next day



No. of Sets

The writer recalls about his childhood days in Goa when the baker used to visit every house of the Goan people. He also tells us that in Goa, bread was a part of not only everyday life but also of festive occasions and events. For each occasion there was a special kind of bread. He also tells us that the baker had leading role in the society. He was so important in the life of the Goan’s that they got up with the jingling sound of his bamboo. (ii) Once Prince Siddhartha had gone for hunting where he came across a sick man, an aged man, a monk asking for alms and also witnessed a funeral procession. Unable to understand those sufferings, he at once went out into the world to seek enlightenment, concerning the sorrows he had witnessed. (iii) Amanda was nagged and scolded on do's and don'ts. She was instructed, most probably by her mother, for not biting her nails and for sitting lazily with her shoulders bent. She was nagged by her mother for eating a chocolate as previously it caused her acne. (iv) The poet Walt Whitman expresses his wish that he can identify himself better with the animals, for their qualities of serenity and calm composure. So he intends turning towards them, away from his human companions, since they have better values than those of the humans. (v) Custard, the dragon had a formidable appearance, with sharp teeth, toes like daggers, spikes on back and scales on the belly. Its mouth was a fireplace and the nose was always fuming like a chimney. However, in sharp contrast to the appearance, his temperament was extremely timid. (vi) When Ebright could not win a prize at the science fair, he learnt that winners do real experiment, they don't simply display slides. Then he started conducting experiments. It was definitely a stepping stone towards his success. His competitive nature, his extra efforts and the will to win for the right reasons made him a successful scientist. (vii) To impress the lawyer Oliver Lutkins befriended him when he went to New Mullion for locating Lutkins against whom he had a summon. They went to Bieneke, Gustaffson's, Gray barber's shop and finally to Lutkin's house. In this way Lutkins, the hack driver took the lawyer all around the village to search for Lutkins. Ans. 7. (i) 27 April, 2022 Wednesday Dear Diary, Today, I am feeling very happy as my deepest desire to undertake a bus ride has been fulfilled.

Authors/ Publishers

Ans. 6. (i)



Name of Books/Subject

SECTION-C



S. No.

(2) they would be meeting their

















The Manager Amar Publication House Andheri West Mumbai Subject : Supply of books for School Library Sir, Through this letter of mine, we'd like to place an order for the following books:

























to open these institutions only after the cases are less than 0.3%. A few people (15%) seemed to be in support of the reopening th of schools, coachings etc., only for class 9 th to 12 whereas 8% people were in favour of reopening the same for all classes. Only 5% of persons couldn't say anything in this matter. Hence, we see that approx. 75% people wanted that it’s time for educational institutions to move on with full precautions whereas 20% are still waiting for the situations to become better and 5% of them are perhaps on ‘wait and watch’ stand. (ii) B.S. Public School Rani Bagh Delhi April 28, 2022

@PROCBSE SOLVED PAPER – 2022 (Term-II)

















daughter and plays the role of a good father. He is in search of a good match for his daughter. He is a good selector of words. He calls Lomov my angle, treasure etc. when he comes to know about his proposal. But when Chubukov finds Lomov fighting with Natalya, he immediately changes and starts to support his daughter. Later, Lomov wants to resolve the issue and again visits Chubukov with his marriageproposal. Then the proposal is accepted by Chubukov and allowed Natalya to marry him. (iii) There is no doubt that the credit of Bholi's transformation to Sulekha goes to her teacher. Bholi's teacher played a very important role in transforming the shy and dumb Bholi into a bold and confident girl. The teacher's kind and encouraging words helped her to overcome her issue of stammering. It was the teacher's untiring efforts which helped her to become so bold that she had the courage to finally refuse to marry Bishamber Nath who was much older to her age. Also he was a very greedy and contemptible person. The metamorphosis of the innocent Bholi to a bold and confident person was because of her teacher. The teacher had definitely created a masterpiece. Under her teacher's guidance, she bloomed as a bold and confident person. The shy and reticent child like Bholi had transformed into a bold and confident girl. She was a true masterpiece because she had the courage to revolt against a social evil of dowry and refused to marry the greedy Bishamber Nath.

Term-II, (Outside Delhi Set-II)



















Listening to people who had travelled by the bus, I gathered little details about the trip. I decided to gather enough money to travel by it. When I had the required money, I went on the bus. The conductor was surprised to see me, a young girl, travelling by herself but jokingly called me ‘Madam’ and offered me the front seat. I observed everything very carefully e.g. the trees that seemed to be running, a cow that suddenly moved in front of the bus and so on. When we reached the town, I gaped at everything in amazement—the big shops and so on. The conductor asked me to get down and have a look at the sights, but I refused. I told him that I had come only for the bus ride. The conductor offered to buy a cold drink for me but I refused that too. On the journey back, I saw a young cow dead by the roadside. The conductor told me that it was the same cow we had seen earlier. I felt sad. The memory of the cow haunted me. When we reached the village, the conductor told me to come again, but to remember to bring the fare. In all, my first excursion on the bus was a great success. (ii) Natalya is the only daughter of the landowner Chubukov. She is very possessive about her land and was determined not to part with it. She is a short tempered lady who used to quarrel with anyone very often. She always pinpoints the amount of help she use to offer to her neighbour. Stephen Stepanovitch Chubukov loves his

21

Question Paper Code No. 2/4/2

Series : DPQRS

SECTION-C







unhappy. She felt that she was not free to do anything. She wanted freedom from all restrictions. She dreamt to lead a life of a mermaid in a Languid, emerald sea with her being the only inhabitant there. She also wished to lead a calm and quiet life in a tower like that of Rapunzel with nobody to disturb her. (iv) The poet appreciates animals more than humans because he feels that animals are satisfied and self-contained. They have better values than those of the humans. The poet finds animals more comfortable than humans. (v) The pirate looked very dangerous as he was carrying many weapons with him. He had pistol in both his hands. He also held a sharp blade between his teeth. He had black beard and a wooden leg. His looks and gesture made it quite clear that he meant to harm the house and the people living in it. (vi) Guided by Dr. Urguhart Hart, Ebright









Indeed, Rajvir possessed qualities like time management, positivity and humility which impressed Mr. Barua. His research before the start of his journey to Assam helped him to explore more about the place. Knowing about the destination and being familiar with the real place is of utmost importance. Rajvir also surprised Mr. Barua, Pranjol's father with his in-depth knowledge of the subject. (ii) ‘Selfishness’ means when a person does something only for his own benefit or thinks only about himself/herself. Kisa Gotami was selfish in wanting her dead son to be alive. It was a mother's love which had blinded her. She was wrong in wanting that. She could not see the reality of life but it is natural for anyone to be selfish at such times. (iii) Amanda's parents forever nagged her about what to do and what not to do. These frequent restrictions made her very







Ans. 6. (i)

@PROCBSE 22

(vii) The young lawyer disliked his job because he was made to deliver summons, instead of preparing legal briefs. For that task, he had to go to dirty and shadowy corners of the city. He also feared of being beaten up by the witness.

started tagging Monarch butterflies which made him realise the importance of cells. That project showed how a cell would divide and develop into normal butterfly wing-scales. Later, it formed the blue print of DNA.



Oswaal CBSE Question Bank Chapterwise & Topicwise, ENGLISH LANGUAGE & LITERATURE, Class-X

Term-II, (Outside Delhi Set-III) Question Paper Code No. 2/4/3

Series : DPQRS







(iv) Animals are contented and serene. They are wise. They are always satisfied and do not need to possess things. They remain indifferent in their pleasure and pain. (v) The pirate, who terrified Belinda and all others pets, gaped at Belinda's dragon because he was amazed to find that the dragon didn't fear even a bit. (vi) The book ‘The Travels of Monarch X’ opened the world of science for Richard. After reading that book, he became interested in tracking the migration of butterflies. This interest led him to his other projects and experiments. Finally, he became a great scientist. (vii) When the young lawyer reached New Mull to serve summons on Oliver Lutkins, he came across a man of forty who was redfaced and cheerful. He introduced himself as Bill, the hack driver. The narrator trusted him and told him about his purpose of visiting the place.











The climb to the Brahmagiri hills not only offers the panoramic view of the entire misty landscape of Coorg but also it is a gateway to many hidden surprises. Besides, a walk across the rope bridge leads to the 64 acres island of Nisarg dhama. Then, there is the India's largest Tibetan settlement, Bylakuppe. (ii) Kisa Gotami was sad because her only son had died. In her hour of grief, she went from house to house in search of a medicine to cure him. She had become selfish in wanting her son back. (iii) Amanda often escapes from reality and desires herself to be an orphan as she wants to live a life of her own without any disturbance. She feels troubled by her parents. She is nagged most of the time. and wants freedom from her parents and their intrusions.







Ans. 6. (i)



SECTION-C

Term-II, (Delhi Set-I) Question Paper Code No. 2/2/1





Sir, Thank you for the catalogue you had sent, with the latest price list. After going through the same we'd like to place an order for the following books:

Amar Chitra Katha (Hindi)

2 Sets













Quality 2 Sets





Book's Name





The Sales Manager Manautal Publishers 12, Mall Road Indore Sub : Placing Order for books.

Panchtantra (Hindi)











Ans. 3. A survey was conducted in more than twenty schools of New Delhi about the different activities that interest the children in the age group of 13 year to 15 years. It was amazing to know that it was not playing cricket in which maximum students showed their interest. Rather music interests the maximum number of school children. Then, with a great margin, cricket becomes their second choice. It may be due to the reason that during corona period, children had to shift their preference to solo and indoor activities. Again, just with a little difference children seem to be interested in theatre. Unexpectedly, dance as well as art has been placed as their last but not the least choice of these budding artists. Above all, the survey brought out this ‘healthy’ fact in notice that growing students prefer real activities to virtual games/activities. OR (ii) Neighbourhood Library Aurobindo Road Indore April 27, 2022





SECTION-B



Series : CPQRS / 2

We've transferred 50% advance payment through Paytm. The balance payment will be made on delivery. We expect a 15% discount, as promised by you. Please note th that delivery should be made latest by 10 May. Thanking you Yours sincerely Niharika (Library incharge)

@PROCBSE SOLVED PAPER – 2022 (Term-II)































Coorg is called the land of rolling hills because the city is situated on the gentle sloping hills. The entire area is covered with these hills. Brahmagiri hills has a panoramic view. (ii) Lomov was a rich landowner. He was neighbour of Chubukov. He was a thirtyfive year old rich bachelor who wanted to marry Natalya (Chubukov's daughter). So he came in evening, dressed well in a jacket and white gloves, to ask for Natalya's hand. (iii) Amanda was blamed by her mother for making such mistakes which her mother used to think, are not part of code of good conduct, laid out by the society. She was continuously blamed for biting her nails and for not sitting in the right posture. She was also blamed for not cleaning her room and shoes. (iv) Speaking about the human race, the poet says that there was a time when both humans and animals possessed noble virtues of love, peace, contentment and simplicity. With passage of time, humans gave up all such token of virtues. The road of noble virtues is still followed by animals. Instead human being suffer from the mania of possessing things which animals do not. (v) The dragon, Custard, initially commanded no amount of respect from Belinda and the other pets, on account of his cowardice; but when the house was attacked by an armed pirate and all others panicked and ran for shelter, the dragon savagely attacked the pirate and ate him up. Thus, the other animals realised his worth and tag of ‘the cowardly dragon’ proved to be untrue. (vi) Ebright's mother encouraged his interest in learning. She took him on trips, bought him telescopes, microscopes, cameras, mounting materials and other equipment and helped him in many other ways. By this, we can say that it was Ebright's mother who led him to become a great scientist. (vii) Oliver Lutkins disguised himself as Bill, the hack driver and deceived the young lawyer about his identity. When the lawyer was not able to find Lutkins, he went to his boss who sent him back with a person who knew















Ans. 6. (i)



SECTION-C

Lutkins. On reaching there, they found ‘Bill’ who his companion recognised as Lutkins himself. Ans. 7. (i) When Kisa's only son had died, she went to Buddha. Being plunged into deep grief, first she went to every neighbour, asking for medicine for her dead son. She had lost all her senses and forgot that no medicine could bring back the dead. Then she went to Buddha in hope of bringing her dead son back to life. Buddha asked her to bring some mustard seeds from a house where no death had occurred. But she couldn't find any such house. Buddha made her realise that death is common to all. One must accept the truth and should have farsightedness. Weeping and grieving over the death of near and dear ones cannot bring peace of mind. Blessed is the person who overcomes sorrow. She learnt that a person who remains composed after the death of his kinsmen obtains peace of mind. In such a way, Buddha brought about a different but meaningful perspective in Kisa Gotami's understanding of life. (ii) No, I don't sympathise with Matilda. Infact, it was her ostentation and vanity which landed her in trouble. It is very essential for a person to live a contented life. He should believe in simple living and high thinking. Not everyone is born with a golden spoon in his mouth. With honesty and hard work we can always achieve our best. A person will never be happy if he keeps craving for what he doesn't have. In this way, he may ruin even his present. Matilda had the same craving for a life of luxury and wealth. She immersed herself in glamour when her husband got an invitation to attend a ball. She didn't care for her loving middle-class husband, rather she was running after her fantasies. She always thought beyond her limits. For the minister's party she could have gone in her simple clothes, but she insisted on buying new dress for four hundred francs which her husband gave her. Then she longed for jewellery, which she borrowed from her friend for the party. She borrowed a diamond necklace and lost it. For repaying the diamond necklace to the friend, she had to sacrifice many years of her life, as well as her husband's whole income. Both of them lived in utter poverty to repay for it. Matilda lost all her charm and looked old and pale. Her ostentation and vanity had put her in trouble. It is better to be satisfied with what one has. One should not believe in false show off. (iii) Same as (Outside Delhi Set I, II, III Q. 7. (i))































Ans. 4. Incorrect Correct (1) A The (2) at in (3) start started (4) of to Ans. 5. (1) if she could borrow his Math book (2) he had already studied for the test the next day.

23

@PROCBSE 24

Oswaal CBSE Question Bank Chapterwise & Topicwise, ENGLISH LANGUAGE & LITERATURE, Class-X

Term-II, (Delhi Set-III) Question Paper Code No. 2/2/2

Series : CPQRS/2









(iv) The poet is pleased to observe that the animals never bewail of their lot and condition; they remain indifferent in their pleasure and pain. Also, they don't have any kind of falsehood. (v) Belinda, the little girl tickled the dragon quite cruelly to make it laugh. Seeing the dragon, reacting to the tickling, made all of them laugh. (vi) Richard Ebright conducted an experiment on a flock of butterflies and raised the eggs of female monarchs. In addition to this, he also studied the cause of a viral disease that killed all monarch caterpillars. He also tried to find out similarities between Viceroy butterflies and Monarch. That study of insect hormones helped him in giving a new theory on the life of cells. (vii) The narrator didn't like the dirty and dark sides of the city life. He thought that he would find some pleasant sights in New Mullion. So he was happy to go there. He went there to serve summons on Oliver Lutkins.









Kodavs, being a martial race, have been bearing arms for thousands of years. Firearms have actually become a part of their tradition over the years. That's why they are permitted to carry them without licence. (ii) Chubukov misunderstood that Lomov had come to borrow money because Lomov himself didn't reveal clearly his purpose of that visit. Moreover, he confused Chubukov by saying that he had come to trouble him with a request for help. (iii) Yes, I do identify myself with Amanda. The greatest wealth that a child wants to possess his/her freedom. And the thing that he/she never likes is the constast nagging, unwanted instructions and even over protection of the parents. Every child, like me (in such family environment) wants to postpone growing up, and wants to remain a child for just a bit longer.







Ans. 6. (i)



SECTION-C

Term-II, (Delhi Set-III)

Question Paper Code No. 2/2/3

Series : CPQRS/2









(iv) The poet is quite impressed by the animals that they do not complain and cry about their conditions in the same way as humans do. They eat and sleep peacefully and they have nothing to worry about. The animals are pure and they do not pray to God for forgiveness or ask God to fulfil their desires. (v) Belinda teased the cowardly dragon. She used to tickle the dragon very hard. She used to make fun to him and laughed at him so loudly that it appeared that the house was shaking due to her laughter. (vi) Birds eat Viceroy butterflies because they taste good to them; whereas Monarch butterflies do not taste good to the birds. So the Viceroys try to copy the Monarchs to protect themselves from the birds. (vii) The hack driver told the lawyer that Lutkins was a hard fellow to catch and he was good at deceiving people. He was always up to something or the other. He owed money to many people, including Bill and had never even paid a cent to anybody.









People of Coorg are harmony cherishing individuals. They are known for their customs and their love for neighbours. On the other side, they are also known for their bravery and boldness. They are only martial race in India who are allowed to possess guns without a permit. (ii) Lomov and Natalya are next-door neighbours. One day, Lomov comes to Natalya to propose her. Natalya, who doesn't know that he has come to propose her. She thinks that he had come to claim Oxen Meadows as his own. On this very issue, she quarrels with Lomov. (iii) Amanda, being constantly nagged by her parents for her laziness, thinks herself to be a mermaid one who lives a relaxed and calm life without any confines on the soft waves in the green sea. Amanda uses such imagination as an escape from her day to day nagging and scolding of her parents.







Ans. 6. (i)



SECTION-C

Don't Stop Reading !



You never know what might be asked in the exam.

To download Term-I (2021-22) Solved Paper scan the code below

SCAN

To download Topper's Answer Sheet-2020 scan the code below

@PROCBSE









O R P

‰





@

E S CB

@PROCBSE 2

Oswaal CBSE Question Bank Chapterwise & Topicwise, ENGLISH LANG. & LIT., Class-X

@PROCBSE

READING SKILLS

3

Latest Board Marking Scheme















: To comprehend the passage. To identify the main points of the text. Marking : 1 × 10 = 10 marks Objective : To test inference, analysis, interpretation, evaluation and vocabulary. Only 1 answer for each question will be accepted. Two answers chosen will result in zero credit of marks.



Objective

7.



6.



1. 2. 3. 4. 5.



Some Useful Tips Read the passage carefully and thoroughly. Reading should be done with speed and quickness. Focus on the relevant details and underline them. Read the questions carefully and go back to the passage to find the answers. The answers are generally in a logical sequence. To find answers to the vocabulary-based questions like synonyms, etc., replace the word with the meaning. If you find that it is the same in meaning, the answer is correct. To find the correct option in Multiple Choice Questions, go through all the options. Re-read the passage and then tick the correct option. The incorrect options are called distractors. These distractors appear to be so convincing that in their hurry or carelessness students often mark them as correct answers.

Topic-1 Discursive passage



(10 marks each)











with plenty of textbooks visible. The computer was not sufficient by itself. IV. The next big change began with ARPANET in the 1960s. This foundation led to the Internet, and its growth has been phenomenal. But a widespread communication medium alone is not sufficient to dislocate textbooks either. Inexpensive computers, easy-to-use interfaces (GUIs) and ready access to the Internet all contribute. V. "Fine," say many. "But the textbook publishers are putting their books online. They are creating e-textbooks. So, textbooks live!" VI. This argument misses the essential difference between education technology in the earlier times and education technology in the present, other than mere cost. Computer software can be interactive. Previous education technology was, like textbooks, rather passive. Active learning is far superior to passive learning, so much so that it’s not a step, not a leap, but a rocket-propelled launch past old-fashioned learning. VII. It’s the mental engagement with something that determines whether you learn and how well. The really big change will come when students can jump into online situations that are more like real life, and are able to use a variety of devices to reach the web and become part of the learning community. They will be guided by a combination of human mentors and machines. Furthermore, diagnostic analysis of their work will









I. Read the passage given below and answer the question complete the sentences that follow: (10×1=10) I. Despite plenty of nay-sayers, the textbook is dead. It just doesn’t know it yet and continues on walking about as though alive. I even have one, A TextBook of Physics, on my bookshelf beside me that was printed in 1891. It has some line drawings and no colour. Today, textbooks have lots of colourful images and engaging questions sprinkled about them. They also have tons of advice for teachers on how to use them effectively. II. The word “textbook” originated in the 1720s, almost 300 years ago. It’s had a good run and is ready to retire. Those who argue that you cannot learn well without a textbook ignore the centuries prior to 1720 when lots of people learned and learned well with no textbooks anywhere. You can learn without textbooks. That’s certain. But why should we bother to change something that’s worked for 300 years? Many deliver the verdict in a single word: technology. That’s way too simplistic. III. We’ve seen film loops, overhead projectors, motion picture projectors, machine scoring of tests and a myriad of other technological innovations in classrooms, but the textbook remains. No, technology alone will not end the reign of the textbook. It takes something more, and that became available in the 1940s — the programmable electronic computer. But here we are, 70 years later,



OBJECTIVE TYPE QUESTIONS

@PROCBSE 4

Oswaal CBSE Question Bank Chapterwise & Topicwise, ENGLISH LANG. & LIT., Class-X

















Q. 7. Select the option from the passage that does NOT qualify as a fact. (A) Today, textbooks have lots of colorful images and engaging questions sprinkled about. (paragraph I) (B) The word “textbook” originated in the 1720s, almost 300 years ago. (paragraph II) (C) But here we are, 70 years later, with plenty of textbooks visible. (paragraph III) (D) The really big change will come when students can jump into online situations that are more like real life... (paragraph VII) Ans. Option (D) is correct. Q. 8. Suppose the students have to learn about the water cycle. According to the author, a student will BEST learn about the water cycle while __________. (A) creating a miniature model of it (B) listening to a teacher explain it in detail (C) reading about it through colourful textbooks (D) watching an informative video about it on the internet Ans. Option (A) is correct. Q. 9. Based on the text given, we can say that the author is __________. (A) documenting the history of learning tools (B) suggesting ways to limit the use of technology (C) proposing an argument and making a case for it (D) highlighting the cause of a problem and its effects Ans. Option (C) is correct.



























































































































Q. 2. Based on the passage, the following developments took place in the education field. (i) The process of learning took place without the use of textbooks. (ii) The invention of computers and digital books reduced the dependency on physical books. (iii) The creation of adaptive and interactive learning tools contributed to the way learning took place. (iv) The introduction and circulation of textbooks turned them into a commonly used medium of learning. Which of these arranges the developments in the order of their occurrence? (A) i-ii-iii-iv (B) i-iv-ii-iii (C) ii-i-iii-iv (D) iii-i-ii-iv Ans. Option (B) is correct. Q. 3. Select the option that can weaken the author's case for a complete shift to computer learning in paragraph VI. (A) a sharp increase in the creation of e-textbooks (B) a rising dependency on using technology as a medium of learning (C) scholarly articles comparing the effectiveness of online learning with that of textbooks (D) the development of e-textbooks with activities that require active participation from readers Ans. Option (D) is correct. Q. 4. Which of these best conveys the meaning of 'diagnostic analysis' as used in paragraph VII? (A) a depiction of the learners' interests (B) a display of the learners' achievements

(C) an evaluation of the learners' capabilities (D) an appreciation of the learners' initiatives Ans. Option (C) is correct. Q. 5. Which of these sentences CORRECTLY uses the phrase 'rocket-propelled' as used in paragraph VI? (A) Sean is lucky to have a friend who talks to him in a rocket-propelled way. (B) Anika screamed out of fear and ran out of her house in a rocket-propelled manner. (C) Regular practice and a good coach gave the batsman a rocket-propelled rise in his career. (D) The TV was buzzing with the news of severe damage caused by a rocket-propelled explosion. Ans. Option (C) is correct. Q. 6. Select the option that BEST describes the passage. (A) Harry Keller's piece gives an account of the origin of textbooks. (B) Harry Keller's essay provides a boost to the field of education technology. (C) Harry Keller's commentary on the popularity of computers is a must-read. (D) Harry Keller's observations on the changes in learning patterns are brilliant. Ans. Option (B) is correct. Explanation: According to the passage, technology has made education more easy-to-use and ready to access.











tell mentors when students are ready for certain learning and, more importantly, when they’re not. VIII. Textbooks do not tap into our brains to realise this learning potential. Hence, today, the textbook is a zombie. It’s just waiting for that wooden stake or silver bullet to put it to a well-deserved rest. - Harry Keller, July 22, 2013 (482 words) [CBSE Additional Practice Question, 2021-22] On the basis of your understanding of the above passage, answer the questions that follow: (1 × 10 = 10) Q. 1. In the passage, the author talks about a welldeserved rest for the textbooks. Which of these describes the author's attitude towards textbooks? (A) He criticises the value added by textbooks in society. (B) He recommends that textbooks are the sole medium for effective learning. (C) He thinks that textbooks are far more superior than other mediums of knowledge. (D) He is concerned about the continued use of textbooks as a primary source of learning. Ans. Option (D) is correct. Explanation: Because textbooks, today, are zombie. It does not our tap into our brains to realise the learning potential.

@PROCBSE

READING SKILLS





















































































































 





machines. Nearly two decades ago, there were around 65 lakh artisans in the country. Three years ago, when the government started the process of granting a unique number to the artisans based on the Aadhaar card, 25 lakhs were identified. Loss of traditional crafts is clearly a worrying issue, but it stands to reason that forcing any artisan to follow old ways when concerns of livelihood overrule other considerations, is unfair. [CBSE-SQP, 2021-22] On the basis of your understanding of the above passage, answer the questions that follow: (1 × 10 = 10) Q. 1. What does the writer mean by calling handicrafts a ‘valued tradition’? (A) It showcase talents assosiated with artisans ‘lifestyle’. (B) It represents our rich artistic culture. (C) It is very costly. (D) Both (A) and (B) are correct. Ans. Option (D) is correct. Q. 2. What does the phrase ‘failing to make progress’ means? (A) To make progress. (B) Languishing (C) Succeeding (D) None of the above Ans. Option (B) is correct. Q. 3. Why are artisans choosing to work via machines rathes than handcrafted tooks? (A) Because it saves labour. (B) Because the artisans are lazy. (C) Because it increases production and efficiency (D) Both (A) and (C) are correct. Ans. Option (D) is correct. Q. 4. The artisans need to be 'lured with incentives’ to impart training because_________________. (A) they run the risk of losing their traditional livelihood to them. (B) need to be tempted. (C) they are few in number. (D) they work whole heartedly. Ans. Option (A) is correct. Q. 5. The impact of the support of government and private sectors towards the culture of making handicraft has resulted in ____________________. (A) more artisans to be incentivized to continue their tradition. (B) creation of awareness and public support. (C) artisans wouldn't change their professions seeking better livelihood. (D) All the above are correct. Ans. Option (A) is correct. Q. 6. What argument does the writer give to justify the artisan's act of abandoning his/her traditional craft for a more lucrative option ? (A) That the artisans cannot be expected to continue their profession if it is not profitable (B) That continuation is very difficult.









Q. 10. We’ve seen film loops, overhead projectors...and a myriad of other technological innovations in classrooms... In other words, we can say that the author has seen film loops, projectors and __________ technological innovations in classrooms. (A) various (B) definite (C) distinct (D) complex Ans. Option (A) is correct. II. Read the following extract and answer the questions/complete the sentences that follow : (10×1=5) Technology is making advancements at a rapid rate but at the cost of a valued tradition—the crafts industry. The traditional crafts industry is losing a lot of its trained and skilled craftsmen. With that, the art of embellishing brass and copper utensils with fine engravings is also disappearing. The government has identified around 35 crafts as languishing craft. The speciality of handcrafted items is its design, an association with long traditions belonging to a specific region. The word ‘handcrafted’ does not imply the involvement of dexterous human fingers or an agile mind with a moving spirit anymore. Lessening drudgery, increasing production and promoting efficiency have taken precedence. The labour-saving devices are taking the place of handcrafted tools and this has jeopardized the skills of these artisans. Mechanisation has made its way into everything - cutting, polishing, edging, designing etc. Ideally, the use of machinery should be negligible and the handicrafts should be made purely by hand with a distinguishable artistic appeal. However, with the exception of small-scale industries, the export units are mostly operated by machines. The heavily computerised designs contribute to a faster production at lower costs. Although mechanization of crafts poses a challenge to safeguarding traditional crafts, the artisans are lured with incentives in order to impart handicrafts training. Some makers do see machines as a time-saving blessing since they are now able to accomplish difficult and demanding tasks with relative ease. These machines might give a better finesse to these products but they don’t stand out as handcrafted. Quantity has overtaken quality in this industry. A need to highlight the importance of the handmade aspect is required by both the government and private sectors, in order to amplify awareness and also support the culture of making handicrafts. A few artisans are still trying their best to rejuvenate and revive their culture and heritage but it’s an uphill task competing with the machine-made goods. A multitude of artisans have changed their professions and are encouraging their progeny to follow suit. There are others who have stayed their ground but are clearly inclined towards buying

5

@PROCBSE 6

Oswaal CBSE Question Bank Chapterwise & Topicwise, ENGLISH LANG. & LIT., Class-X



































































Q. 4. Match the following: A. Romans (i) A priest to swear for the accused B. Saxons (ii) Highly paid lawyers can win a case C. Modern (iii) Educated judges and lawyers





Q. 3. Study the following statements: (A) Romans were proud of their judicial system. (B) There is not much difference between the Norman and modern system of justice. (A) (A) is right and (B) is wrong. (B) (B) is right and (A) is wrong. (C) Both (A) and (B) are right. (D) Both (A) and (B) are wrong. Ans. Option (C) is correct. Explanation: The Romans had a highly sophisticated system of courts and their members were well educated. The modern system of justice, too, has both the qualities. Hence, both 'A' and 'B' are correct.

















III. Read the following extract and answer the questions/complete the sentences that follow : (5×1=5) In most societies that have any glimmer of civilization, a person accused of wrong doing is given at least a nominal chance of proving his innocence. The Romans had a highly sophisticated comprehensive system of courts and the members of their legal profession were well educated, but the Saxons who followed them to rule Britain used rougher methods. From about the sixth century A.D. to the eleventh the majority of the trials were in the form of cruel physical torture (carrying a piece of red hot iron, stepping barefoot and blindfold across a floor covered with red hot coals) or sometimes by a gentler method of oath - swearing. The accused was ordered to bring to the Saxon authorities. A police officer or a priest could be persuaded to swear on oath or still a number of persons who would say that the accused was of good character and thus, innocent. The number of persons who swore depended on the crime. A noble/ a landlord or a priest counted for up to half a dozen ordinary peasants. As almost everyone lived in small villages, where almost everyone knew everyone else and very few would risk telling a lie on oath (the people were mostly religious), the truth was generally told. If the accused could not produce enough oath helpers, he was found guilty and punished. In the eleventh century, the Normans introduced trial by battle in certain cases. The accused and the accuser fought with special weapons until one was dead or surrendered. It was believed that God would know the guilty and give the 

















Q. 9. The synonym of ‘rejuvenate' is __________. (A) Vigour (B) Culture (C) Revitalize (D) Art Ans. Option (C) is correct. Q. 10. Machines have jeopardized the skills of the artisans. (True / False) Ans. True Explanation: As computerised designs contribute to a faster production at lower costs.

innocent the power to win. The whole idea became ridiculous when both the parties were allowed to hire champions who would fight on their behalf. It seemed likely whoever could pay the more for a stronger professional fighter stood a good chance of winning and being judged innocent. This may sound unfair to us but there is a parallel with a wealthy person today who can hire a costly and brilliant barrister to defend him. In the early middle ages when England was a land of small villages remote from each other, crime tended to be basic and direct : beating up, theft, sex and murder being the main offences. But as towns and manufacturing and commerce grew, the possibilities for cheating and fraud soared. The whole organisation of society became more complex and opened the door to a world of more sophisticated wickedness. With no regular police force, spies and informers were offered rewards when they brought in criminals. [CBSE-SQP, 2021-22] On the basis of your understanding of the above passage, answer the questions that follow: (1 × 10 = 10) Q. 1. England (or Britain) turn by turn came under the rule of: (A) Saxons; Romans; Normans (B) Normans; Saxons; Romans (C) Romans; Saxons; Normans (D) Normans; Romans; Saxons Ans. Option (C) is correct. Q. 2. The article describes: (A) The development of the system of justice in England (B) Civilized societies and justice (C) Justice v/s Civilisation (D) Rule of Justice in England Ans. Option (A) is correct. Explanation: The system of justice in England developed with the passage of time which has been described in the given article.































(C) That the loss is perturbing. (D) All the above are correct. Ans. Option (A) is correct. Q. 7. What is the benefit of heavily computerised designs ? (A) It Contribute to a faster production. (B) The production is at lower costs (C) The designs are outdated. (D) Both (A) and (B) are correct. Ans. Option (D) is correct. Q. 8. The mechanization of crafts & safeguard traditional crafts. (True / False) Ans. False Explanation: Rather mechanization of crafts poses a challenge to safeguarding traditional crafts.

@PROCBSE

READING SKILLS













































































Q. 8. Which of the following statements are true? The rich have always enjoyed an advantage in the judicial system because: (A) they were physically strong, so would win the trial by fighting. (B) they could hire the strongest champion. (C) they can hire the best lawyers. (D) they were mostly religious and innocent people.



IV. Read the following passage carefully and answer the questions/complete the sentences that follow : (10×1=5) The choices we make on a daily basis—wearing a seat belt, lifting heavy objects correctly or purposely staying out of any dangerous situation—can either ensure our safety or bring about potentially harmful circumstances. You and I need to make a decision that we are going to get our lives in order. Exercising self-control, selfdiscipline and establishing boundaries and borders in our lives are some of the most important things we can do. A life without discipline is one that’s filled with carelessness. We can think it’s kind of exciting to live life on the edge. We like the image of “Yeah! That’s me! Living on the edge! Woo-hoo!” It’s become a popular way to look at life. But if you see, even highways have lines, which provide margins for our safety while we’re driving. If we go over one side, we’ll go into the ditch. If we cross over the line in the middle, we could get killed. And we like those lines because they help to keep us safe. Sometimes we don’t even realize how lines help to keep us safe. I’m not proud of this, but for the first 20 years of my life at work, I ignored my limits. I felt horrible, physically, most of the time. I used to tell myself “I know I have limits and that I’ve reached them, but I’m going to ignore them and see if or how long I can get by with it.” I ran to doctors, trying to make myself feel better through pills, vitamins, natural stuff and anything I could get my hands on. Some 





















Q. 7. Study the following statements: (A) Earlier England comprised small villages each with a small population. (B) Crimes like cheating and fraud were rare. (A) (A) is an assertion and (B) is the response (B) (B) is an assertion and (A) is the response (C) Both (A) and (B) are unrelated assertions. (D) Both (A) and (B) are responses to some other assertions. Ans. Option (A) is correct. Explanation: Since the population was small, the cases of cheating or fraud were rare.

































Q. 6. Study the following statements: (A) Saxon system of trial was nobler than that of the Romans. (B) Saxon system had two aspects – rough and noble. (C) The rich Saxons could hire champions to argue their case. (D) Even an innocent person would be held guilty if enough people did not swear for him. Which of the following is correct? (A) (A) and (B) (B) (B) and (C) (C) (C) and (A) (D) (B) and (D) Ans. Option (d) is correct. Explanation: Saxon system had two aspects – rough and noble. Evan an innocent person would be held guilty if enough people did not swear for him.

Q. 9. Which of the following statements are not true? (A) In the quest for justice the guilty often went unpunished. (B) For seven hundred years from the sixth century trial was mostly rough. (C) Swearing value of a priest was equal to a dozen ordinary peasants. (D) Use of champions in a trial by battle finds an equivalent in the modern times. (A) (A) and (B) (B) (B) and (C) (C) (C) and (D) (D) (A) and (B) Ans. Option (B) is correct. Q. 10. ‘any glimmering of civilisation’ ‘Glimmering’ in the above expression has been used as a metaphor. Glimmering stands for: (A) a slight suggestion (B) a great hope (C) some fear (D) a little confidence Ans. Option (B) is correct. Explanation: The word 'glimmer' mean 'shine'. There was a shining great hope of a civilisation.





















(A) (A) and (B) (B) (B) and (C) (C) (C) and (D) (D) (A) and (D) Ans. Option (B) is correct. Explanation: With the help of their money, rich have always been able to hire the best champion for a battle or the best lawyers for court. Hence, they have always enjoyed an advantage in the judicial system.



D. Normans (iv) The winner in a battle declared innocent (A) (A)-(iv); (B)-(ii); (C)-(i); (D)-(iii) (B) (A)-(iii); (B)-(i); (C)-(ii); (D)-(iv) (C) (A)-(i); (B)-(iii); (C)-(iv); (D)-(ii) (D) (A)-(ii); (B)-(i); (C)-(iii); (D)-(iv) Ans. Option (B) is correct. Q. 5. Study the following statements: (A) In a trial by battle, money played a main role. (B) God helped the innocent win the battle. (A) (A) is right and (B) is wrong. (B) (B) is right and (A) is wrong. (C) Both (A) and (B) are right and (A) was the conclusion. (D) Both (A) and (B) are right and (A) was not the conclusion. Ans. Option (A) is correct. Explanation: Para 4 clearly conveys that whoever could pay more for a stronger professional fighter stood a good chance of winning and being judged innocent.

7

@PROCBSE Oswaal CBSE Question Bank Chapterwise & Topicwise, ENGLISH LANG. & LIT., Class-X









  































































(4)

























































































(A) Option (1) (B) Option (2) (C) Option (3) (D) Option (4) Ans. Option (D) is correct. Q. 3. Which of the characteristics are apt about the writer in the following context: “I know I have limits and that I’ve reached them, but I’m going to ignore them and see if or how long I can get by with it.” ? 1. negligent 2. indecisive 3. spontaneous 4. reckless 5. purposeless 6. patient (A) 2 and 5 (B) 3 and 6 (C) 1 and 4 (D) 2 and 3 Ans. Option (C) is correct. Q. 4. Which of the following will be the most appropriate title for the passage?















(3)



(2)

Balance is not something you find, it’s something you create. — Jana Kingsford











(1)

Life is like riding a bicycle. To keep your balance you must keep moving. — Albert Einstein











To go beyond is as wrong as to fall short. —Confucius











It's all about quality of life and finding a happy balance between work and friends. —Philip Green

(A) Much too soon (B) Enough is enough (C) How much is too much? (D) Have enough to do? Ans. Option (C) is correct. Q. 5. The phrase “potentially harmful circumstances” refers to circumstances that can (A) certainly be dangerous. (B) be fairly dangerous. (C) be possibly dangerous. (D) seldom be dangerous. Ans. Option (C) is correct. Q. 6. Select the option that makes the correct use of “unsustainable”, as used in the passage, to fill in the blank space. (A) In the long run, the ______ officials followed emergency procedures. (B) Emergency procedures were _______ by the officials. (C) Officials reported an ________ set of events during the emergency. (D) Officials admit that the emergency system is _______ in the longer run. Ans. Option (D) is correct. Q. 7. The author attempts to ___________ the readers through this write-up. (A) rebuke (B) question (C) offer aid to (D) offer advice to Ans. Option (D) is correct. Q. 8. The author uses colloquial words such as “yeah” and “Woo-hoo!”. Which of the following is NOT a colloquial word? (A) hooked (B) guy (C) stuff (D) stress Ans. Option (D) is correct. Q. 9. What does the author mean when he says, “to get our lives in order”? (A) To resume our lives. (B) To organize our lives. (C) To rebuild our lives. (D) To control our lives. Ans. Option (C) is correct. Q. 10. Choose the option that correctly states the two meanings of ‘outlook’, as used in the passage. 1. A person’s evaluation of life 2. A person’s experiences in life 3. A person’s point of view towards life 4. A person’s regrets in life 5. A person’s general attitude to life (A) (1) and (4) (B) (2) and (3) (C) (3) and (5) (D) (4) and (5) Ans. Option (C) is correct. V. Read the following passage carefully : 1. Caged behind thick glass, the most famous dancer in the world can easily be missed in the National Museum, Delhi. The Dancing girl of Mohenjodaro is that rare artefact that even school children are familiar with. Our school textbooks also communicate the wealth of our 5000 year heritage



of the doctors would tell me, “It’s just stress.” That just made me mad. I thought stress meant you don’t like what you do or can’t handle life, and I love what I do. But I kept pushing myself, travelling, doing speaking engagements and so on— simply exhausting myself. Finally, I understood I was living an unsustainable life and needed to make some changes in my outlook and lifestyle. You and I don’t have to be like everyone else or keep up with anyone else. Each of us needs to be exactly the way we are, and we don’t have to apologize for it. We’re not all alike and we need to find a comfort zone in which we can enjoy our lives instead of making ourselves sick with an overload of stress and pressure. [CBSE SQP, 2020-21] On the basis of your understanding of the above passage, answer the questions that follow: (1 × 10 = 10) Q. 1. The reason why living on the edge has become popular, is because of the (A) constant need for something different. (B) population being much younger. (C) exhausting effort to make changes. (D) strong tendency to stay within our limits. Ans. Option (A) is correct. Q. 2. Choose the option that best captures the central idea of the passage from the given quotes. 





8

@PROCBSE READING SKILLS







  























































































































































9

(B) why museums in our country are exciting. (C) why she will make us come into money. (D) of dancing figures. Ans. Option (B) is correct. Q. 7. The dancing girl made of _________. (A) terracotta (B) glass (C) bronze (D) Both (a) and (c) Ans. Option (C) is correct. Q. 8. Which of these is FALSE : (A) The dancing girls has no feet. (B) She is small. (C) She is little bigger than the human palm. (D) She is a little over 10 cm tall. Ans. Option (C) is correct. Q. 9. The ornament that the dancing girl wears are made of _______ . (A) shells (B) ivory (C) thin metal (D) All the above are correct. Ans. Option (D) is correct. Q. 10. The dancing girl is a perfect example of _____. (A) rare artefact (B) hopeful human spirit (C) modern art (D) Both (a) and (b) Ans. Option (D) is correct. VI. Read the following passage carefully : (1) Few guessed that this quiet, orphan girl growing up in New York City would one day become the First Lady of the United States. Even fewer thought she would become an author and lecturer and a woman much admired and loved by people throughout the world. (2) Born Anna Eleanor Roosevelt in 1884, to wealthy but troubled parents who both died when she was young, Roosevelt was cared for by her grandmother and sent to school in England. In 1905, she married her distant cousin, Franklin Delano Roosevelt. She and her husband had six children. Although, they were wealthy, her life was not easy and she suffered several personal tragedies. Her second son died when he was a baby. In 1921, her strong athletic husband was stricken with polio, which left him physically disabled for life. (3) Eleanor Roosevelt was a remarkable woman who had great intelligence and tremendous strength of character. She never let things get her down. She nursed her husband back to good health and encouraged him to remain in politics. She then helped him to become the Governor of New York, and in 1933, the President of the United States. (4) While her husband was the President, she took a great interest in all the affairs of the country.



  





of art. You have to be alert to her existence there, amid terracotta animals to rediscover this bronze image. 2. Most of us have seen her only in photographs or sketches, therefore, the impact of actually holding her is magnified a million times over. One discovers that the dancing girl has no feet. She is small, a little over 10 cm tall – the length of a human palm – but she surprises us with the power of great art – the ability to communicate across centuries. 3. A series of bangles – of shell or ivory or thin metal – clothe her left upper arm all the way down to her fingers. A necklace with three pendants bunched together and a few bangles above the elbow and wrist on the right hand display an almost modern art. 4. She speaks of the undaunted ever hopeful human spirit. She reminds us that it is important to visit museums in our country to experience the impact that a work of art leaves on our senses, to find among all the riches one particular vision of beauty that speaks to us alone. [CBSE Delhi Set-1, 2020 (Modified)] On the basis of your understanding of the above passage, answer the questions that follow: (1 × 10 = 10) Q. 1. The dancing girl belongs to (A) Mohenjodaro (B) Greek culture (C) Homosapiens (D) Tibet Ans. Option (A) is correct. Q. 2. In the museum she’s kept among (A) dancing figures (B) bronze statues (C) terracotta animals (D) books Ans. Option (C) is correct. Q. 3. Which information is not given in the passage? (A) The girl is caged behind glass. (B) She is a rare artefact. (C) School books communicate the wealth of our heritage. (D) She cannot be rediscovered as she’s bronze. Ans. Option (D) is correct. Q. 4. ‘Great Art’ has power because (A) it appeals to us despite the passage of time. (B) it is small and can be understood. (C) it’s seen in pictures and sketches. (D) it’s magnified a million times. Ans. Option (A) is correct. Q. 5. The jewellery she wears: (A) consists of bangles of shell, ivory or thin metal. (B) is a necklace with two pendants. (C) both (i) and (ii) are correct. (D) neither (i) nor (ii) is correct. Ans. Option (C) is correct. Q. 6. She reminds us (A) of the never say-die attitude of humans.









@PROCBSE Oswaal CBSE Question Bank Chapterwise & Topicwise, ENGLISH LANG. & LIT., Class-X

















  











































































Q. 9. The word scaffolding in para-4 stands for ______. (A) a wooden structure outside the building (B) invalidate (C) factories (D) mis-management Ans. Option (A) is correct. Q. 10. Roosevelt kept in touch with the American people through ________ . (A) My day (B) City life (C) My life (D) Day life Ans. Option (A) is correct. VII. Read the following passage carefully : (1) Keep your watch accurate. For some people, moving up the time on their watch will help them get up earlier. For others, they will remember that the time on the watch is wrong and will disregard it altogether. It may be helpful to set your watch just two minutes ahead instead of five or ten. (2) Keep a clock, phone, computer or anything that displays time in each room of your house. One of the easiest ways to run late is simply by not realising that the time is passing as quickly as it is. (3) Set all your clocks and watches to the same time. Don’t be an optimist. Things usually take longer than what you’d expect, even without major delays. If you have a dinner appointment at 7:30 p.m., don’t think you can work till 7 p.m., then take a bath, dress and reach on time. Realistically, calculate the time you will take at each step and then add 10 minutes more to allow for unexpected delays, or you cannot get to your job done in time. (4) Wake up when you are supposed to wake up. Don’t hit the snooze button, keep on lying in bed, and watch TV at the very start of your day. May be you can try even setting your clock 10 minutes earlier than you need. If you have difficulty with this, move your alarm clock to somewhere away from your bed; that way, you will have to get up to turn it off. Commit yourself to being 15 minutes early for everything. If you have to reach your place of work at 8:00, don’t even tell yourself this. Just tell yourself (and everyone else who listens































Q. 8. The appropriate sub-heading to para -4 is ___. (A) Mrs Roosevelt as a devoted wife (B) Mrs. Roosevelt as a volunteer (C) Roosevelt ‘s political career (D) Roosevelt as a frustrated lady Ans. Option (C) is correct. Explanation: Part - 4 discusses about Mrs. Roosevelt Political career.





















Q. 7. The passage is about Mrs. Roosevelt’s ______. (A) courage, determination (B) married life

(C) retarded husband (D) life as a mother Ans. Option (A) is correct.





































She became her husband’s legs and eyes; she visited prisons and hospitals; she went down into mines, in scaffoldings and into factories. Roosevelt was tireless and daring. During the period of depression, she travelled all over the country bringing goodwill, reassurance and helped people without food and jobs. During World War II she visited American soldiers in camps all over the world. The United States had never known a First Lady like her. (5) Roosevelt also kept in touch with the American people through a daily newspaper column called ‘My Day’. She broadcast on the radio and delivered lectures, all first for a First Lady. [Delhi Set, 2019 (Modified)] On the basis of your understanding of the above passage, answer the questions that follow: (1 × 10 = 10) Q. 1. What do you mean by the First Lady of the United State? (A) The wife of the President of the US (B) The lady President of the US (C) The highest ranked lady of the US (D) None of the above Ans. Option (A) is correct. Q. 2. Franklin Delano Roosevelt was her _______ . (A) relative (B) cousin (C) cousin husband (D) distant cousin Ans. Option (D) is correct. Q. 3. Her life was quite ______. (A) troubled (B) comfortable (C) wealthy (D) easy Ans. Option (A) is correct. Q. 4. Her conduct shows that she was ______ . (A) strong willed (B) timed (C) weak willed (D) frustrated Ans. Option (A) is correct. Q. 5. The most appropriate title to para-2 the passage is ________ . (A) The early life of Anna Eleanor Roosevelt (B) Childhood days of miss Roosevelt (C) Mrs. Roosevelt (D) The married life of Anna Eleanor Roosevelt Ans. Option (D) is correct. Q. 6. President Roosevelt was ________ . (A) physically disabled (B) a weak husband (C) mentally related (D) a diseased person Ans. Option (A) is correct. Explanation: As he was striken with polio.



10

@PROCBSE

READING SKILLS











(B) after 5 a.m. (C) between 5 a.m. to 6 a.m. (D) at 7 a.m. Ans. Option (C) is correct.



























 























































































































On the basis of your understanding of the above passage, answer the questions that follow: (1 × 10 = 10) Q. 1. Keep ______ in each room. (A) clock (B) phone (C) computer (D) any one of the above Ans. Option (D) is correct. Q. 2. It is good to commit yourself ______ for everything. (A) to be late (B) to delay (C) to being 15 minutes early (D) to search to the last moment Ans. Option (C) is correct. Q. 3. To be punctual we should ______. (A) hit the snooze button of the alarm clock (B) get up at the right time (C) start watching TV in the morning (D) keep on lying in bed Ans. Option (B) is correct. Q. 4. It is a good habit to ______. (A) reach your work place in time. (B) calculate the time before starting any work. (C) keep lying in bed after the alarm clock buzzes. (D) both (a) and (b). Ans. Option (D) is correct. Q. 5. According to the passage the watch should be set _________ ahead. (A) five minutes (B) ten minutes (C) two minutes (D) fifteen minutes Ans. Option (C) is correct. Q. 6. The writer suggests to keep the watch ______. (A) ahead of time (B) accurate (C) inaccurate (D) none of these Ans. Option (B) is correct. Q. 7. The given chart shows that students should get up ____________. (A) before 5 a.m.





[CBSE Outside Delhi Set-3, 2019 (Modified)]







































Q. 8. A student spends maximum time in __________ during the day time. (A) taking meals (B) taking rest (C) school (D) sleeping Ans. Option (C) is correct. Q. 9. A student ____________ devote time to studying on his own. (A) can (B) should not (C) should (D) will Ans. Option (C) is correct. Q. 10. Proper time ___________ helps everyone lead a stress free life. (A) organisation (B) management (C) planner (D) devotion Ans. Option (B) is correct. VIII. Read the following passage carefully : (1) Tourists to Jammu and Kashmir have another attraction – a floating post office on the Dal Lake in Srinagar, the first in the country. ‘Floating Post Office, Dal Lake – claimed to be the only one such post office in the world – is built on an intricately carved maroon houseboat, fastened on the western edge of the Dal Lake. (2) This post office lets you avail of all the regular postal services available in the country while being afloat. The seal used on everything posted from Floating Post Office is unique – along with the date and address. It bears the design of a boatman rowing a Shikara on the Dal Lake. The special feature of this post office is that letters posted from here carry a special design which has the picturesque scenery of Dal Lake and Srinagar city. These pictures reach wherever these letters are posted to and hence promote Kashmir as a tourist destination across the world. (3) This is actually a heritage post office that has existed since British times. It was called Nehru Park Post Office before 2011. But after that the chief postmaster John Samuel renamed it as ‘Floating Post Office’. (4) The post office’s houseboat has two small rooms – one serves as the office and the other a small museum that traces the philatelic history of the state postal department. It has a shop that sells postage stamps and other products. (5) But for the locals, Floating Post Office is more than an object of fascination. Rupees 1-2 crore is deposited per month in Floating Post Office by communities living in and around the Dal Lake. The lake has several islets that are home to more than 50,000 people. (6) The greatest difficulty is the recurrence of 2014 like floods in which the houseboat had gone for a toss uncontrollably pushed by the flood. Rescue teams had to anchor it using special mechanism at



— but don’t annoy them or make them think that they are late or early!) “I have to be at work at 7:45.” If you do this, you will be on time even with little unforeseen delays. You will be on time even with a traffic jam.

11

@PROCBSE Oswaal CBSE Question Bank Chapterwise & Topicwise, ENGLISH LANG. & LIT., Class-X

post-office! [Delhi & Outside Delhi Set 1, 2, 3 2018 Modified]

On the basis of your understanding of the above passage, answer the questions that follow: (1 × 10 = 10) Q. 1. Before 2011, this post office was called ______. (A) Jawahar Park Post Office (B) Nehru Park Post Office (C) British Post Office (D) Floating Post Office Ans. Option (B) is correct. Q. 2. The seal used in the floating Post Office is unique because (A) it bears the design of a boatman rowing a Shikara. (B) it is used by the local people. (C) there is no date and address on it. (D) it is available throughout the country. Ans. Option (A) is correct. Q. 3. The seal bears the design of (A) Aeroplane (B) Bus (C) Ship (D) Shikara Ans. Option (D) is correct. Q. 4. Floating Post Office is fastened on which edge of the Dal Lake? (A) Eastern (B) Northern (C) Western (D) Southern Ans. Option (C) is correct. Q. 5. In which year flood struck floating Post Office? (A) 2014 (B) 2004 (C) 1994 (D) 1894 Ans. Option (A) is correct. Q. 6. The monthly deposits per month in Floating Post Office is ` ____________. (A) 4-5 crore (B) 3-4 crore (C) 2-3 crore (D) 1-2 crore Ans. Option (D) is correct. Q. 7. _____________ is claimed to be the only one such in the world. (A) the museum of the state postal department (B) the shop that sells postage stamps

(C) Floating Post Office situated on Dal Lake (D) all of the above Ans. Option (C) is correct. Q. 8. John Samuel, who renamed it floating Post Office, was a postman. (True / False) Ans. False Q. 9. The special feature of this post office is that letters posted from here _____________. (A) have the design a houseboat (B) have the design of snow-capped mountains (C) have the design of Dal Lake (D) have the design of local handicrafts Ans. Option (C) is correct. Q. 10. In the post office’s houseboat there is ______. (A) an office, museum and a shop (B) a museum, library and a prayer room (C) an office, gift shop and a library (D) none of the above Ans. Option (A) is correct. IX. Read the following passage carefully : Then all the windows of the grey wooden house (Miss Hilton used to live here. She expired last week) were thrown open, a thing I had never seen before. At the end of the day a sign was nailed on the mango tree : FOR SALE Nobody in the street knew Miss Hilton. While she lived, her front gate was always locked and no one ever saw her leave or saw anybody go in. So even if you wanted to, you couldn’t feel sorry and say that you missed Miss Hilton. When I think of her house, I see just two colours— grey and green. The green of the mango tree, the grey of the house and the grey of the high iron fence that prevented you from getting at the mangoes. If your cricket ball fell in Miss Hilton’s courtyard you never got it back. It wasn’t the mango season when Miss Hilton died. But we got back about ten or twelve of our cricket, balls. The house was sold and we were prepared to











































   















































































































a nearby highland. Then it was brought back on the Dal after the water receded. The biggest boon is that at no time of the year, you need a fan in this





12

@PROCBSE READING SKILLS

















































































Q. 4. You would always get your cricket ball if they fell in Miss Hilton’s courtyard. (True/False) Ans. False Q. 5. The iron fence did not let the boys get at the ________ on the tree. (A) Mango (B) Oak (C) Guava (D) Banyan Ans. Option (A) is correct. Q. 6. Which word means the same as ‘arranged’. (A) settled (B) craved (C) adjusted (D) None of the above Ans. Option (A) is correct. Q. 7. Nobody went into Miss. Hilton’s house because her front _______ . (A) gate was always locked (B) gate was isolated (C) gate was broken

13



















For millions of people in India, river Ganga is the most sacred river. It is considered as mother and goddess. It is also a lifeline to millions of Indians who live on the banks of its course and depend on it for their daily needs. River Ganga is the third largest river in the world by the amount of water that flows through it. It is the longest river in India. The river water of Ganga is used for irrigation, transportation and fishing. The Gangetic plain formed by river Ganga is one of the most fertile lands on earth. This is why almost 10% of the world population lives here and earns its livelihood. The Ganga, in India is the most worshipped body of water. The irony here is that in spite of being the most worshipped river, it is also the dirtiest one. It carries metals thrown out by tanneries, waste produced by industries and urban waste from different cities. All this has made river Ganga the fifth most polluted river in the world. Another major reason that adds to the Ganga river pollution is the coal based power plants on its banks which burn tons of coal every year and produce a lot of fly ash. This ash mixed with domestic waste water is released into the river. This bad situation calls for an urgent need to make efforts to reduce pollution and revive river Ganga. To achieve these objectives, the Government of India has started a programme named “Namami Ganga Programme”. The main pillars of this programme are sewage treatment, river surface cleaning, afforestation, river front development and public awareness. The importance of the success of “Namami Ganga 











































(D) None of the above Ans. Option (A) is correct. Q. 8. What happen to Miss. Hilton? (A) She died (B) She fell ill (C) She slipped (D) None of the above Ans. Option (A) is correct. Q. 9. The house of Miss. Hilton was then ________. (A) sold (B) broken (C) destroyed (D) None of the above Ans. Option (A) is correct. Q. 10. What do the grey and green of Miss. Hilton house denote? (A) House & Mango tree (B) Floor & Grass (C) Garden & Guava (D) None of the above Ans. Option (A) is correct. X. Read the following passage carefully :









dislike the new owners ever before they came. I think we were a little worried. Already we had one resident of the street who kept on complaining about us to our parents. He complained that we played cricket on the pavement and if we were not playing cricket he complained that we were making too much noise anyway. One afternoon when I came back from school Pal said, “Is a man and a woman. She pretty pretty, but he ugly like hell”. I didn’t see much. The front gate was open but the windows were shut again. I heard a dog barking in an angry way. One thing was settled pretty quickly. Whoever these people were, they would never be the sort of people to complain that we were making noise and disturbing their sleep. A lot of noise came from the house that night. The radio was going at full volume until midnight when the radio station closed down. The dog was barking and the man was shouting. I didn’t hear the woman. On the basis of your understanding of the above passage, answer the questions that follow: (1 × 10 = 10) Q. 1. Miss Hilton’s house had just 2 colours _____. (A) Grey & Black (B) Grey & Blue (C) Grey & Pink (D) Grey & Green Ans. Option (D) is correct. Q. 2. The sign ‘For sale’ was hung on a ______ . (A) Banyan tree (B) Mango tree (C) Oak tree (D) Guava tree Ans. Option (B) is correct. Q. 3. Miss. Hilton was a very friendly lady. (True/False) Ans. False Explanation: Miss. Hilton never want anywhere nor did anyone come to her.











@PROCBSE 14

Oswaal CBSE Question Bank Chapterwise & Topicwise, ENGLISH LANG. & LIT., Class-X













































































































































































(C) 10% (D) 20% Ans. Option (C) is correct. Q. 7. The Ganga is the ____________ most polluted river in the world (A) second (B) third (C) first (D) fifth Ans. Option (D) is correct. Q. 8. The Ganga is the ____________ largest river in the world by the amount of water that flows through it. (A) first (B) fourth (C) third (D) second Ans. Option (C) is correct. Q. 9. In which year was river Ganga cleaned the maximum? (A) 2016-17 (B) 2018-19 (C) 2017-18 (D) 2015-16 Ans. Option (C) is correct. Q. 10. ` 147.6 crores were spent on cleaning Ganga in __________. (A) Uttarakhand (B) Bihar (C) Jharkhand (D) Uttar Pradesh Ans. Option (D) is correct. XI. Read the following passage carefully : (1) Necessity is indeed the mother of invention. When areas in and around Leh began to experience water shortages, life didn’t grind to a halt. Why? Because Chewang Norphel, a retired civil engineer in the Jammu and Kashmir government came up with the idea of artificial glaciers. (2) Ladakh, a cold desert at an altitude of 3,000-3,500 metres above sea level, has a low average annual rainfall rate of 50 mm. Glaciers have always been the only source of water. Agriculture is completely dependent on glacier melt unlike the rest of



On the basis of your understanding of the above passage, answer the questions that follow: (1 × 10 = 10) Q. 1. The Ganga water is used for ______________. (A) irrigation (B) transportation (C) fishing (D) all of the above Ans. Option (D) is correct. Q. 2. The pollutants that make river Ganga very dirty are: (A) effluents from industries (B) urban waste from cities (C) metals discharged from tanneries (D) all of these. Ans. Option (D) is correct. Q. 3. The Namami Ganga project has been started by ______________. (A) an NGO (B) people of India (C) Government of India (D) local bodies. Ans. Option (C) is correct. Q. 4. Most people in India consider the Ganga as ______________. (A) Mother (B) River (C) Goddess (D) Both (a) and (c) Ans. Option (D) is correct. Q. 5. River __________ is a lifeline for millions of Indians. (A) Ganga (B) Krishna (C) Narmada (D) Yamuna Ans. Option (A) is correct. Q. 6. __________ of the world population lives near river Ganga. (A) 15% (B) 30%







Programme” can be seen through the following India thrives. No Ganga, No India”. lines: “ If Ganga dies, India dies. If Ganga thrives, [Delhi Comptt., 2017 (Modified)] Money spent on Cleaning Ganga 2015-16 to 2018-19

@PROCBSE

READING SKILLS

July

August

Sept.

2010

3410

16518

22889

22415

5575

3041

2011

3001

26310

33908

46843

37756

20833

































































































































[CBSE SQP, 2017 (Modified)] (A) August 2010 (B) August 2011 (C) July 2010 (D) July 2011 Ans. Option (D) is correct. Q. 8. The lowest number of tourists who visited Leh was in the month of ____________ . (A) April 2010 (B) April 2011 (C) Sept. 2010 (D) August 2011 Ans. Option (B) is correct. Q. 9. The number of visitors to Leh in July 2011 was _________ than those who visited in July 2010. (A) one third (B) double (C) more than double (D) half Ans. Option (C) is correct. Q. 10. _________ visitors visited Leh in May 2010. (A) 16815 (B) 16518 (C) 15618 (D) 16158 Ans. Option (B) is correct. XII. Read the following passage carefully : If you are addicted to coffee and doctors warn you to quit the habit, don’t worry and just keep relishing the beverage, because it is not that bad after all! In fact, according to a new study, the steaming cup of Java can beat fruits and vegetables as the primary source of antioxidants. Some studies state that coffee is the number one source of antioxidants in the American diet, and both caffeinated and decaf versions appear to provide similar antioxidant levels. Antioxidants, in general, have been linked to a number of potential health benefits, including protection against heart diseases and cancer, but Vinson, a dietician said that their benefits ultimately depend on how they are absorbed and utilised in the body. The research says that coffee





























June







May







April

On the basis of your understanding of the above passage, answer the questions that follow: (1 × 10 = 10) Q. 1. For how many months does the ice formation continue? (A) 2 months (B) 3-4 months (C) 5 months (D) none of these Ans. Option (B) is correct. Q. 2. Mr. Norphel designed 15 artificial glaciers in _____________. (A) Kashmir (B) Ladakh (C) Leh (D) a village Ans. Option (C) is correct. Q. 3. What kind of landform is Ladakh? (A) Hot desert (B) Cold desert (C) Dry desert (D) Hot and Dry desert Ans. Option (B) is correct. Q. 4. Who was Chewang Norphel? (A) a retired principal (B) a retired activist (C) a retired social worker (D) a retired civil engineer Ans. Option (D) is correct. Q. 5. In which year did Mr. Norphel get Padma Shri award? (A) 2007 (B) 2015 (C) 2009 (D) 2005 Ans. Option (B) is correct. Q. 6. Ladakh is at an altitude of ___________ metres above sea level. (A) 2000-2500 (B) 1500-2000 (C) 3000-3500 (D) 2500-3000 Ans. Option (C) is correct. Q. 7. The tabular graph given above depicts that maximum tourists visited Leh in __________ .



Month/ Year















river/monsoon fed India. But over the years with Leh since 1987. In recognition of his pioneering increasing effects of climate change, rainfall and effort, he was conferred the Padma Shri by snowfall patterns have been changing, resulting in President Pranab Mukherjee, in 2015. severe shortage and drought situations. Given the (5) There are a few basic steps followed in creating the severe winter conditions, the window for farming artificial glacier. is usually limited to one harvest season. (6) River or stream water at higher altitude is (3) It is located between the natural glacier above and diverted to a shaded area of the hill, facing north, the village below. The one closer to the village where the winter sun is blocked by a ridge or a and lowest in altitude melts first, providing water mountain range. At the start of winter/November, during April/May, the crucial sowing season. the diverted water is made to flow onto sloping Further layers of ice above melt with increasing hill face through distribution channels. Stone temperature thus ensuring continuous supply to embankments are built at regular intervals which the fields. Thus, farmers have been able to manage impede the flow of water, making shallow pools two crops instead of one. It costs about ` 1,50,000 and freeze, forming a cascade of ice along the and above to create one. slope. Ice formation continues for 3-4 months (4) Fondly called the “glacier man”, Mr. Norphel has resulting in a large accumulation of ice which is designed over 15 artificial glaciers in and around referred to as an “artificial glacier”. Number of Tourists who visited Leh





15

@PROCBSE 16

Oswaal CBSE Question Bank Chapterwise & Topicwise, ENGLISH LANG. & LIT., Class-X







































































































Q. 5. The synonym of the ‘chief’ as given in para 1 is: (A) new (B) primary (C) version (D) similar Ans. Option (B) is correct. Q. 6. Consuming __________ coffee can make a person jittery: (A) less (B) MORE (C) moderate (D) NO Ans. Option (B) is correct. Q. 7. The graph above depicts that maximum caffeine is















Kausani is situated at a height of 6,075 feet in the central Himalayas. It is an unusually attractive little town. It covers just about 5.2 sq. km. It lies to the north of Almora in Uttarakhand’s picturesque Kumaon region. Kausani provides the 300 km. wide breath-taking view of the Himalayas. It is the most striking aspect of this place. Snow-capped peaks are spread in a stately row. They stare at you in silvery-white majesty. The most famous peak on view is Nanda Devi, the second highest mountain in India. It is situated at a height of 25,645 feet and is 36 miles away as the crow flies. The other famous peaks on view are Choukhamba (23,420 feet) and Trishul (23,360 feet). Then there are also Neelkanth, Nandaghunti, Nandaghat and Nandakot. On a clear day, the blue of the sky makes a splendid background to these peaks. At sunrise and at sunset, when the colour of the sky changes to a golden orange, the scene gets etched in your memory. 



















































On the basis of your understanding of the above passage, answer the questions that follow: (1 × 10 = 10) Q. 1. Who is Vinson in the paragraph? (A) A Doctor (B) A Health consultant (C) A Nutritionist (D) A Dietician Ans. Option (D) is correct. Q. 2. Of all the foods and beverages studied, __________ actually have most antioxidants. (A) dates (B) tea (C) cranberries (D) coffee Ans. Option (A) is correct. Q. 3. Which beverage helps in preventing against liver and colon cancer? (A) Tea (B) Milk (C) Coffee (D) Hot chocolate Ans. Option (C) is correct. Q. 4. It is a type of diabetes (A) Type 5 (B) Type 4 (C) Type 3 (D) Type 2 Ans. Option (D) is correct.















in ___________ coffee. (A) instant (B) brewed (C) filter (D) espresso Ans. Option (B) is correct. Q. 8. How much caffeine is found in two cups of brewed coffee? (A) 200 mg (B) 100 mg (C) 300 mg (D) 50 mg Ans. Option (C) is correct. Q. 9. Coffee, which has minimum caffeine per cup is __________. (A) Espresso (B) decalf brewed (C) instant (D) decalf instant Ans. Option (D) is correct. Q. 10. Brewed coffee has more caffeine than _______. (A) filter (B) instant (C) espresso (D) none Ans. Option (B) is correct. XIII. Read the following passage carefully :











outranks popular antioxidant sources like tea, milk, chocolate and cranberries. Of all the foods and beverages studied, dates actually have the most antioxidants based solely on the serving size, but since dates are not consumed anywhere near the level of coffee, the drink comes as the top source of antioxidants, Vinson said. Besides keeping you alert and awake, coffee has been linked to an increasing number of potential health benefits, including protection against liver and colon cancer, type 2 diabetes and Parkinson’s disease, according to some recently published studies. The researchers, however, advise that one should consume coffee in moderation, because it can make you jittery and cause stomach pains. [Delhi, 2016 (Modified)]

When Gandhi ji visited this place in 1929, its scenic beauty held him spellbound. He named it the ‘Switzerland of India’. He prolonged his two day stay to fourteen days, making time to write a book, ‘Anashakti Yoga’. The place where he was staying was originally a guest house of a tea estate. It was renamed ‘Anashakti Ashram’ after the book. Kausani is the birthplace of Sumitra Nandan Pant, India’s poet laureate. Its natural surroundings inspired many of his poems. Its tea gardens mingle with dense pine forests and fruit orchards. The area is also host to many fairs and religious ceremonies. If Uttarakhand is the abode of Gods, Kausani is God’s own backyard. There is no traffic,

@PROCBSE

READING SKILLS



































































There are others too, whose purpose in life has made a difference to society. Mother Teresa had a comfortable life at Loretto Convent, Kolkata. On a train journey to Darjeeling, she felt her ‘calling’ and wanted to give up her comfortable life in the convent and serve the poor and the needy.























One’s purpose is shaped over a period of time, but every noble purpose is based on a feeling for the suffering of others and our desire to alleviate that condition. Some years ago, in Kerala, a young man saw his father join Gandhiji and fight for opening temples to the Harijans. When he grew up, the boy decided to take up agricultural studies, because he had seen the havoc caused by the Bengal famine. He went to Europe and studied agriculture. On his return to India, he spearheaded the Green Revolution. Dr. M.S Swami Nathan, thus demonstrated how purpose intensifies with time. [Board Term, 2015-16 (Modified)] 

















After that, she took a three month nursing course and returned to Kolkata. She devoted her time to the sick and the homeless. Touched by her sacrifice, some of her students joined her and work began. Not everyone has such a strong calling. When I asked her how one could be sure of one’s calling, she replied.” Deep down in our hearts we know exactly what our calling is, if we are sincere.”

























































He sent an educationist to Europe and the United States to report on what should be the best model for India. The John Hopkins University in Baltimore was recommended as the model. But, Tata made it a point to see that his name was not associated with the university. After his death, his dream came true with the establishment of the Indian Institute of Science in Bengaluru. It was to be the fountain head of science in the decades to come. 

On the basis of your understanding of the above passage, answer the questions that follow: (1 × 10 = 10) Q. 1. Kausani is situated at a height of __________ feet in the Central Himalayas. (A) 7125 (B) 6075 (C) 6175 (D) 4275 Ans. Option (B) is correct. Q. 2. In which year Gandhi ji visited Kausani? (A) 1914 (B) 1925 (C) 1935 (D) 1929 Ans. Option (D) is correct. Q. 3. At sunrise and at sunset the colour of the sky changes to (A) silverish grey (B) golden orange (C) blue (D) white Ans. Option (B) is correct. Q. 4. Gandhi ji prolonged his two day stay to (A) One week (B) 10 days (C) 14 days (D) 15 days Ans. Option (C) is correct. Q. 5. __________ is the abode of Gods. (A) Uttarakhand (B) Kausani (C) Switzerland (D) Almora Ans. Option (A) is correct. Q. 6. Kausani is situated in __________ Himalayas. (A) Central (B) Northern (C) Southern (D) Eastern Ans. Option (A) is correct. Q. 7. Which month received the lowest rainfall in Kausani? (A) January (B) March (C) April (D) February Ans. Option (C) is correct. Q. 8. The month which received almost half the rainfall of June is ________ . (A) February (B) May (C) March (D) January Ans. Option (B) is correct. Q. 9. The average rainfall in the month of February in Kausani is approx ________ .

























(A) 70 mm (B) 90 mm (C) 50 mm (D) 60 mm Ans. Option (A) is correct. Q. 10. Gandhiji named Kausani as _________ . (A) Switzerland (B) India’s Switzerland (C) Switzerland of India (D) Switzerland in India Ans. Option (C) is correct. XIV. Read the following passage carefully : A PURPOSE IN LIFE A noble purpose gives meaning to one’s own life and is of benefit to others. I realised this while researching on and writing about the life of Jamshed ji Tata. What gave one man living in a colonised country the vision to advance it in the community of nations? He not only gave India the power of steel and hydroelectric energy, but also an institution of research.







no one is in a hurry. If serenity could be put on a canvas, the picture would resemble Kausani. [Outside Delhi, 2015 (Modified)]

17

@PROCBSE 18

Oswaal CBSE Question Bank Chapterwise & Topicwise, ENGLISH LANG. & LIT., Class-X





























































































































Answering Tip While choosing the correct answer from the given options, do not hurry and get easily carried away by the distractors (the incorrect options) that appear to be plausible. In vocabulary based questions, generally search for the word in same part of speech. Such as: attraction fascination and not fascinated.





Many students do not read the passage thoroughly and jump to conclusions to find the correct options.





Commonly Made Error











On the basis of your understanding of the above passage, answer the questions that follow: (1 × 10 = 10) Q. 1. Indian Institute of Science does not bear the title of whom? (A) Ambani (B) Tata (C) Birla (D) Reliance Ans. Option (B) is correct. Q. 2. Who bought Green Revolution to India? (A) Dr. B.R. Ambedkar (B) Vinoba Bhave (C) Dr. Radha krishnan (D) Dr. M.S. Swami Nathan Ans. Option (D) is correct. Q. 3. Where is Indian Institute of Science situated? (A) Hyderabad (B) Pune (C) Delhi (D) Bengaluru Ans. Option (D) is correct. Q. 4. Name the Indian city where Mother Teresa settled and served the poor? (A) Delhi (B) Chennai (C) Mumbai (D) Kolkata Ans. Option (D) is correct. Q. 5. The above passage talks about (A) being selfish (B) having no purpose in life (C) giving a meaning to life (D) None of the above Ans. Option (C) is correct.































Q. 6. Which of the following is true about Jamshed ji Tata? (A) Established Indian Institute of Science in Bengaluru. (B) Gave his name as the model to the John Hopkins University. (C) Gave India the power of steel and hydroelectric energy. (D) Opened temples to the Harijans. Ans. Option (C) is correct. Q. 7. Mother Teresa wanted to give up __________. (A) teaching (B) social work (C) her comfortable life in the convent and serve the poor and the needy (D) none of these Ans. Option (C) is correct. Q. 8. Jamshed ji Tata sent his educationist to Europe and the United States to report. (True/False) Ans. True Q. 9. Jamshed ji Tata gave India the _____________. (A) power of education (B) power of technology (C) power of steel (D) all of these Ans. Option (C) is correct. Q. 10. Mother Teresa did a course in ____________. (A) nursing (B) teaching (C) social service (D) all of these Ans. Option (A) is correct.

Topic-2 Case-based passage



(10 marks each)



I Read the following excerpt from a case study titled Impacts of Festivities on Ecology. Festivals are synonymous with celebration, ceremony and joy. However, festivals bring to fore the flip side of celebrations – pollution – air, water,



OBJECTIVE TYPE QUESTIONS soil and noise. This led to the need of assessing the awareness level among people about ecological pollution during festivals. So, a study was conducted by scholars of an esteemed university in India. This study was titled ‘Awareness Towards

@PROCBSE

19

READING SKILLS

containing 6 questions, shared with 50 respondents across four selected districts of a state in the southern region of India. The research began by understanding the socioeconomic conditions of the respondents before sharing the questionnaire. Once the responses were received, the data collected was tabulated (Table 1), for analysis.





Impact of Festivals on Ecology’. There were two main objectives of the study. The first one was to assess the awareness level among people about ecological protection during festivities. Exploring solutions to bring awareness about celebrating festivals without harming ecology was the second objective. The method used to collect data was a simple questionnaire

Table-1: Awareness level among respondents Can’t Say%

54

0

2. Do you think most people abuse environmental resources during celebration of festivals?

72

28

0

3. Do you think that celebrations & festivities result in uniting people?

64

32

4

4. Do you enjoy bursting crackers for amusement?

68

32

0

5. Do you feel pressured to burst crackers during festivals as an expectation of your social status?

82

12

6









No%

46











Questions



Yes%

1. Do you feel that bursting crackers is a must during festivities?























































































































6. Are you aware of waste segregation & disposal guidelines for 56 40 4 better ecology? Q. 3. The significance of the second objective of the The study recommended the imposition of strict study is __________________. rules and regulations as opposed to a total ban on (A) based on objective one. all festive activities which have a drastic impact (B) to focus on what needs to be corrected and on our environment. The researchers believed what needs to be avoided to solve problem. that such measures would help in harnessing (C) to explore solutions. some ill-effects that add to the growing pollution and suggested further studies be taken up across (D) None is correct. Ans. Option (B) is correct. the country to assess awareness about ecological degradation. Q. 4. The researchers recommend for limiting the drastic impact of festival pollution on environment. It is The observations made in the study pointed to justified because _________________________. the environmental groups and eco-clubs fighting (A) it make the environment pollution free. a losing battle due to city traffic issues, disposal (B) strict rules are better than a total ban. of plastics, garbage dumping and all sorts of (C) strict rules pose some restrictions. ecological degradation. The researchers stressed (D) Both (B) and (C) are correct. that the need of the hour is increasing awareness Ans. Option (D) is correct. among people to reduce ecological pollution Q. 5. Why do the researchers feet that environmental which can be facilitated by celebrating all festivals groups and eco-clubs are fighting a losing battle in in an eco-friendly manner. [CBSE SQP, 2020-21] the given scenario ? On the basis of your understanding of the above (A) As festivals cause pollution along with other passage, answer the questions that follow: issues. (1 × 10 = 10) (B) As awareness is the only solution but lack of it Q. 1. Why is pollution called the ‘flip side’ of festivals? only adds to the problem. (A) Because people fail to see the other side, which (C) Bursting crackers cannot be stopped. is Pollution. (D) Both (A) and (B) are correct. (B) Festivals and pollution are polar opposites. Ans. Option (D) is correct. (C) Because people enjoy festivals. Q. 6. Which of the following support the below (D) Both (A) and (B) are correct. statement ? Ans. Option (D) is correct. ‘Even though a larger number of people say ‘no’ to bursting crackers than those who say 'yes', festival Q. 2. What is the first main objective of the study? pollution persists. (A) To pollute the environment. (A) 72% of people abuse environmental resources (B) To assess awareness among people about to celebrate festivals. ecological and protection during festivals. (B) 6% are still confused. (C) Both (A) and (B) are correct. (C) 82% use crackers to celebrate festivals. (D) Both (A) and (B) are incorrect. (D) Both (A) and (C) support the statement. Ans. Option (B) is correct. Ans. Option (D) is correct.

@PROCBSE 20



















































































































The study also used the road-kill surveys to rank the bird and mammal species whose long-term survival was most threatened by road-kill. The hazel grouse and ground squirrel were found to be the most at risk of local extinction. Both are common in Europe but are classified as species of Least Concern Red List of Threatened Species. The most vulnerable animals classified as threatened by IUCN were the red-knobbed Coot, Balcan mole and Podolian mole. The study revealed that road-kill hotspots were not the areas with the highest population of endangered species. For example, house sparrows had a high road-kill rate (2.7 per km/year) but were ranked 420th of 423 bird species for vulnerability. Conversely, the hazel grouse had a low predicted road kill-rate (0.2 per km/yr) but was most vulnerable of all birds studied. On the basis of your understanding of the above passage, answer the questions that follow: (1 × 10 = 10) Q. 1. Study the following statements: (i) Roads are killers for animals. (ii) Both birds and mammals are killed on roads. (iii) Species most killed are necessarily the endangered ones. (iv) (i) is correct and (ii) is false. (B) (ii) is correct and (iii) is false. (C) Both (i) and (ii) are correct. (D) (iii) is correct and (i) is false. Ans. Option (C) is correct. Explanation: The first paragraph clearly conveys that around 194 million birds and 29 million mammals are thought to be killed on roads every year in Europe. Q. 2. Study the following statements: (i) Roads have covered 50% of land in Europe. (ii) Road traffic causes a great risk to wild life. (iii) Some species can survive all kinds of traffic on roads. (A) (i) is correct and (ii) is false. (B) (ii) is correct and (iii) is false. (C) (iii) is correct and (i) is false. (D) Both (i) and (ii) are false. Ans. Option (B) is correct. Q. 3. Which of the following points clearly depicts the message of the given paragraph? (A) While planning roads we should see which species to protect. (B) We are doing a lot to protect those most at risk. (C) 50% of Europe is covered only with roads. (D) Small animals even with low population density are most at risk. Ans. Option (D) is correct. Q. 4. Which of the following given points is the correct statement? (A) More mammals than birds are killed on the roads. (B) Small animals generally keep away from roads. (C) Number of road-kills depends upon the population density of small animals.























Q. 7. What purpose does the ‘can't say’ column serve in the questionnaire (table-1)? (A) gives provision to choose not to express (B) allows an option to those who lack clarity. (C) allows an option to those who are unwilling to respond. (D) All are correct. Ans. Option (D) is correct. Q. 8. What do the researchers suggest towards the end of the passage. (A) Not to use crackers. (B) Not to celebrate festivals. (C) need to increase awareness to reduce ecological pollution. (D) All are correct. Ans. Option (C) is correct. Q. 9. ‘Controlling’ is as same the meaning as ___________ in the passage. (A) awareness. (B) imposition (C) control (D) harnessing Ans. Option (D) is correct. Q. 10. Which is the example of ecological degradation according to the passage? (A) disposal of plastic (B) pollution (C) garbage dumping (D) All the above are correct. Ans. Option (D) is correct. II. Read the passage given below and answer the questions/complete the statements that follow by choosing the most appropriate options out of the given ones. (Any six) Around 194 million birds and 29 million mammals are thought to be killed each year on European roads, according to a new study that has ranked the most vulnerable species. The research has found that the species killed most often were not necessarily the endangered species. This means action to preserve wildlife when new roads are built risks being targeted at the wrong species based on current methods. Road densities in Europe are among the world’s highest, with 50% of the continent within 1.5 km of a paved road or railway. Roads are therefore a significant threat to wildlife and evidence shows deaths on them could even cause some species to disappear completely. Despite this, the long-term protection of species is not currently considered when assessing the impact of new roads on wildlife, meaning we risk giving support to only the endangered species, doing nothing to help those most at risk. A better understanding of which species are most vulnerable to roads is therefore important if we are to take a more effective action of protection. A research team based in Lisbon calculated roadkill rates for 423 bird species and 212 mammal species. They found that small animals with high population densities and which mature at an early age were most likely to be killed on roads. Nocturnal mammals and birds with a diet of plants and seeds were also shown to have higher death rates.



Oswaal CBSE Question Bank Chapterwise & Topicwise, ENGLISH LANG. & LIT., Class-X

@PROCBSE

READING SKILLS



















































Q. 2 Based on the passage, what efforts are being made by the Royal Swedish Academy of Sciences to address the low representation of female scientists? (i) The Academy is consciously awarding female scientists from the 2000s. (ii) The Academy is inviting sociologists to lecture about the gender gap. (iii) The Academy is hosting group discussions to boost the intelligence of female scientists. (A) only (i) (B) only (ii) (C) (i) and (iii) (D) (ii) and (iii) Ans. Option (A) is correct. Q. 3. According to the passage, Goran Hansson says that there are __________. (A) more female scientists in East Asia than in



























On the basis of your understanding of the above passage, answer the questions that follow: (1 × 10 = 10) Q. 1. According to the passage, the gender disparity among Nobel Prize recipients is due to __________ factors. (A) historical (B) economical (C) sociological (D) psychological Ans. Option (A) is correct. Explanation: As per paragraph-II, only 57 women have received the award compared to 873 male recipients Reasons being historical.





















































































more women are being recognised now compared with previous decades, only about 10% of the professors in natural sciences in western Europe or North America are women, and even lower if you go to east Asia." All over the world, the number of men working in STEM fields is much higher than the number of women. 4. Hansson emphasised that they have made sure to identify the problem and also learn about subconscious bias in the prize-awarding committees and academies. "We’ve had lectures by sociologists, we’ve had group discussions, we have put quite a lot of effort into it,” he said. 5. “In the end, we will give the prize to those who are found the most worthy, those who have made the most important contributions," he added.

















(D) Animals that come out only at night are saved. Ans. Option (C) is correct. Q. 5. (i) The surveys ranked the road-kill rate of birds and mammals. (ii) The finding puts grouse and squirrel at great risk. (A) (i) is an assertion and (ii) is the response. (B) (ii) is an assertion and (i) is the response. (C) Both (i) and (ii) are false. (D) Both (i) and (ii) are unrelated to each other. Ans. Option (A) is correct. Q. 6. Hazel grouse and ground squirrel are classified as species of least concern. The statement is: (A) a logical conclusion. (B) a piece of good news. (C) ironical. (D) a pleasant surprise. Ans. Option (C) is correct. Q. 7. The title of the study should be: (A) A Birds and Mammals survey (B) Road-kills (C) Road density in Europe (D) Need for conservation Ans. Option (B) is correct. Q. 8. The purpose of the study is: (A) how to prevent road-kills. (B) to see who is more at risk on the roads. (C) how to plan better roads. (D) to estimate the number of road accident victims. Ans. Option (B) is correct. Q. 9. ‘A place of significant activity’ is called _______________. (Para-4) (A) Vulnerable (B) Hotspot (C) Hazel (D) Reveal Ans. Option (B) is correct. Q. 10. Synonym of ‘ Vulnerable’ is __________________. (A) protected (B) safe (C) unprotected (D) homeless, Ans. Option (C) is correct. III. Read the following passage carefully: 10 marks 1. The Nobel Prize is an international award administered by the Nobel Foundation and is based on the fortune of Alfred Nobel, a Swedish inventor and entrepreneur. The Nobel prize is awarded for the best contribution in the fields of Physics, Chemistry, Medicine, Literature and to the person who has contributed by and large to bring Peace on Earth. In 1968, Sweden’s central bank established the Sveriges Riksbank's Prize in Economic Sciences in memory of Alfred Nobel. 2. A look into the history of the recipients of the award reveals that only 57 women have received the award compared to 873 male recipients. There are various historical reasons for why this is the case, but the trend appears to be improving. 3. Goran Hansson, the permanent secretary of the Royal Swedish Academy of Sciences said, "While

21

@PROCBSE 22

Oswaal CBSE Question Bank Chapterwise & Topicwise, ENGLISH LANG. & LIT., Class-X























































Q. 8. The Nobel Prize is an international award because it recognises the achievements made__________. (A) in all subjects (B) by all genders (C) in all countries (D) by all institutions Ans. Option (C) is correct. Q. 9. Identify the opinion from these statements about the information shared in the passage. (A) The Nobel prizes have been awarded to more men than women. (B) Alfred Nobel's wealth is used to award the Nobel prizes to the winners. (C) The Nobel prize was awarded for the best contribution in only four fields initially. (D) Women in general are more inclined to working on world peace than on other subjects. Ans. Option (D) is correct. Q. 10. The maximum number of Women Nobel Laureates was recorded in_________________. (A) 1991 -2000 (B) 2001-2010 (C) 2010-2019 (D) 2020 Ans. Option (C) is correct.































IV. Read the following passage carefully : The UN’s 2017 International Year tells that sustainable tourism is an important tool for development, most importantly in poor communities and countries. Today sustainability – environmental, social, and economic – is increasingly recognised as the benchmark for all tourism business. As noted by the UN World Tourism Organisation, 57% of international tourist arrivals will be in emerging economies, by 2030. The various ‘Tourism Terms’ are defined as follows:

Geo tourism Pro-Poor Tourism



Ethical Tourism



Eco tourism



CATEGORY





















Q. 5. According to Goran Hansson, the Royal Swedish Academy of Sciences __________. (A) researches the cause of gender discrepancy in Nobel prize nominations (B) nominates fewer men for prestigious awards like the Nobel prize (C) awards the Nobel prize to the most famous contributors (D) selects individuals who are not prejudiced in any way Ans. Option (A) is correct. Q. 6. Which of these pieces of information is NOT present in the passage? (A) number of Nobel prizes given in the year 2009 (B) number of men who have received Nobel prizes till date (C) percentage of male scientists belonging to

eastern countries (D) percentage of female scientists belonging to western countries Ans. Option (C) is correct. Q. 7. Select the sentence that CORRECTLY uses the phrase 'by and large' as used in paragraph I. (A) Paula prepared a by and large cake for dinner. (B) We bought by and large clothes for the campaign. (C) Most of the lions by and large eat deer when hungry. (D) She was so excited that she screamed by and large on the road. Ans. Option (C) is correct. Explanation: Phrase ‘by and large’ means ‘on the whole or everything considered'.



















Europe (B) fewer male scientists than female scientists in Europe (C) fewer male scientists being considered for recognition now (D) more female scientists being recognised for their work now Ans. Option (D) is correct. Q. 4. Which of these can be inferred from the graph showing the number of women Nobel laureates by decade? (A) No woman received the Nobel prize before the year 1910. (B) More women have received the Nobel prize since 1971 than men. (C) At least one woman received the Nobel prize every year since 1901. (D) Since 1961, women Nobel laureates have been increasing steadily every decade. Ans. Option (D) is correct. Explanation: The number has gradually increased from 4 to 13 since 1991 to 2019.

DEFINITION Responsible travel to natural areas that conserves the environment, socially and economically sustains the well-being of local people, and creates knowledge and understanding through interpretation and education of all involved (including staff, travellers, and community residents). Tourism in a destination where ethical issues are the key driver, e.g. social injustice, human rights, animal welfare, or the environment. Tourism that sustains or enhances the geographical character of a place - its environment, heritage, aesthetics, culture, and well-being of its residents. Tourism that results in increased net benefit for the poor people in a destination.

@PROCBSE READING SKILLS

Responsible Tourism

23

Tourism that maximizes the benefits to local communities, minimizes negative social or environmental impacts, and helps local people conserve fragile cultures and habitats or species.



Tourism that leads to the management of all resources in such a way that economic, social, and aesthetic needs can be fulfilled while maintaining cultural integrity, essential ecological processes, biological diversity, and life-support systems. 57% of their annual profits from International Based on data collected by a survey by Travel tourists. Bureau, the following market profile of an eco(B) countries with upcoming economies shall see tourist was constructed: maximum tourist footfall from all over the Age: 35 - 54 years old, although age varied with world in the next decade. activity and other factors such as cost. (C) a large number of international tourists in 2030 will be from developing countries. Gender: 50% female and 50% male, although clear (D) barely any tourist in the next decade shall differences based on activity were found. travel from an economically strong nation to a Education: 82% were college graduates, a shift in weak one. interest in eco-tourism from those who have high Ans. Option (B) is correct. Q. 3. One of the elements that is important to ecolevels of education to those with less education was tourists on trip is: also found, indicating an expansion into mainstream (A) wild and untouched surroundings. markets. (B) cultural exchange. Household composition: No major differences were (C) car and bus rides. found between general tourists and experienced eco(D) fully furnished flats. tourists** Ans. Option (A) is correct. Party composition: A majority (60%) of experienced Q. 4. Choose the option that lists the correct answers for the following: eco-tourism respondents stated they prefer to travel 1. Asha Mathew, an NRI, loves animals and as a couple, with only 15% stating they preferred to wishes to travel to places that safeguard their travel with their families, and 13% preferring to travel rights and inculcate awareness of their rights. alone. (** experienced eco-tourists = Tourists that What kind of tourist is she? had been on at least one “eco-tourism” oriented trip.) 2. Gurdeep Singh from UK is an environmental scientist and has always chosen to travel Trip duration: The largest group of experienced ecoto places that are examples of a symbiotic tourists- (50%) preferred trips lasting 8-14 days. relationship between man and nature. What Expenditure: Experienced eco-tourists were willing kind of tourist is he? to spend more than general tourists, the largest group (A) (1) is an eco-tourist and (2) is a geo tourist (26%). (B) (1) is an ethical tourist and (2) is a geo tourist (C) (1) is a sustainable tourist and (2) is a pro-poor Important elements of trip: Experienced eco-tourists tourist top three responses were: (a) wilderness setting, (b) (D) (1) is a geo tourist and (2) is a responsible tourist wildlife viewing, (c) hiking/trekking. Ans. Option (B) is correct. Motivations for taking next trip: Experienced ecoQ. 5. Based on your understanding of the passage, tourists top two responses were (a) enjoy scenery/ choose the option that lists the inherent qualities of geo tourism. nature, (b) new experiences/places.





































































Sustainable Tourism















3. promises luxurious travel























6. initiates 5. believes in commercializing donations for the forests underprivileged (A) 1 & 2 (B) 5 & 6 (C) 2 & 4 (D) 3 & 5 Ans. Option (C) is correct. Q. 6. In the market profile of an eco-tourist, the information on gender indicates that: (A) female eco-tourists were more than the male















4. includes being environmentally responsible 







1. showcases 2. promotes landscape adventure sports appreciations



[CBSE SQP, 2020-21] On the basis of your understanding of the above passage, answer the questions that follow: (1 × 10 = 10) Q. 1. In the line “……recognised as the benchmark”, the word “benchmark” DOES NOT refer to: (A) a basis for something. (B) the criterion required. (C) the ability to launch something new. (D) a standard point of reference. Ans. Option (C) is correct. Q. 2. The World Tourism Organisation of the UN, in an observation, shared that: (A) emerging economies of the world will gain

@PROCBSE 24

Oswaal CBSE Question Bank Chapterwise & Topicwise, ENGLISH LANG. & LIT., Class-X



































































































































decision was made by the head of the family and the others fell in line. Today every member of the family has a say in every decision which also promotes a sense of togetherness and bonding. (4) We empower our kids to take their own decisions from a very early age. We ask them the cuisine they prefer, the movie they want to see, the holiday they wish to go on and the subjects they wish to study. (5) It’s a closely connected world out there, where children consult and guide each other. A parent’s well meaning advice can sound like nothing more than unnecessary preaching. How then do we reach our children through all the conflicting views and make the voice of reason be heard? Children today question choices and prefer to go with the flow. (6) What then is the best path to take? I would say the most important thing one can do is to listen. Listen to your children and their silences. Ensure that you keep some time aside for them, insist that they share their stories with you. Step into their world. It is not as complicated as it sounds; just a daily half an hour of the quality time would do the trick. [CBSE Delhi Set - I, 2020 (Modified)] On the basis of your understanding of the above passage, answer the questions that follow: (1 × 10 = 10) Q. 1. According to the passage, who said, “so, where are we going for dinner now?” (A) narrator (B) one at the steering (C) one of the passengers (D) one of the guests Ans. Option (B) is correct. Q. 2. Twenty years ago, according to the passage, decisions were __________ . (A) unanimous ,with no argument (B) made by the children (C) taken by the head of the family (D) not taken by elders Ans. Option (C) is correct. Q. 3. In modern times, children are _____ to take their own decision from a very early age. (A) forbidden (B) trained (C) not given permission (D) allowed Ans. Option (D) is correct. Q. 4. Generally, a parent’s advice is often taken as ______ by the children. (A) dictatorship (B) unnecessary preaching (C) obligation (D) distraction Ans. Option (B) is correct.

















eco-tourists. (B) the activity preferences were varied in females and males. (C) the choice of things to do on a trip were quite similar for both the genders. (D) male eco-tourists were frequent travellers. Ans. Option (B) is correct. Q. 7. The education aspect in the market profile of the eco-tourist revealed that: (A) mainstream market trends were popular with undergraduates. (B) eco-tourists were only those who had basic education. (C) mainstream markets were popular tourist destinations for educated eco-tourists. (D) eco-tourism was no more limited to the small group of highly educated travellers. Ans. Option (D) is correct. Q. 8. According to the survey conducted by the Travel Bureau, the total percentage of experienced ecotourists who DID NOT prefer to travel alone was: (A) 60%. (B) 75%. (C) 15%. (D) 13%. Ans. Option (B) is correct. Q. 9. According to the survey, one of the most powerful driving forces leading experienced eco-tourism to invest in new trips was: (A) setting up work stations in new places. (B) the chance to go camping in the wild. (C) competing with other eco-tourists as frequent travellers. (D) the opportunity to travel to new places. Ans. Option (D) is correct. Q. 10. Choose the option that lists statement that is NOT TRUE. (A) Economically backward countries will benefit from sustainable tourism. (B) The tourism business currently recognizes sustainability as an important factor. (C) Emerging economies will receive negligible international tourists in the near future. (D) The sustainability factor in tourism is a significant means for development. Ans. Option (C) is correct. V. Read the following passage carefully : (1) As the family finally sets off from home after many arguments, there is a moment of lull as the car takes off. “Alright, so where are we going for dinner now?” asks the one at the driving wheel. What follows is a chaos, as multiple voices makes as many suggestions. (2) By the time, order is restored and a decision is arrived at, tempers have risen, feelings injured and there is at least one person grumbling. (3) Twenty years ago, you would step out of home, decision of meal and venue already made with no arguments or opposition and everybody looked forward to the meal with equal enthusiasm. The

@PROCBSE

READING SKILLS



















































































































































































really cannot happen without the other. Any improvements that have not been generated by improving yourself are superficial and short-lived. If you want to have a better future, you have to change yourself. Stop worrying about the various challenges that will come your way. Just believe in yourself and use them as stepping-stones. (4) “Success” is getting whatever you want out of life without violating the rights of others.” It is not an accident. Success is the result of our attitude and our attitude is a choice. Hence success is a matter of choice and not chance. (5) Today success represents a holistic and positive attitude to life. Attitude is everything. Success is not restricted within some kind of brick and mortar premises. It assumes the individuality of a complete act executed with perfection; material achievements do not define life. We don’t remember sports stars for the products they endorse but the spirit of achievement that they represent. In the abundance of positive attitude, lie the grandeur of a truly rewarding rich life. (6) Success can be measured from two standpoints— the external and the internal. Externally it is the measure of a job well done and the recognition. Internally it is the feeling of achievement and wholeness derived from the completion of a task or fulfilment of desire. [CBSE SQP, 2020] On the basis of your understanding of the above passage, answer the questions that follow: (1 × 10 = 10) Q. 1. What qualities lead one to the place called success? (A) determination (B) perseverance (C) faith (D) all of these Ans. Option (D) is correct. Q. 2. The Road to success is__________. (A) straight (B) curved (C) slanting (D) not straight Ans. Option (D) is correct. Q. 3. The difference between a disaster and triumph lies in ____________. (A) whether you are prepared to weather the storm (B) willing to work hard (C) ready to fail (D) your efforts Ans. Option (A) is correct. Q. 4. According to the text, which of the following statement is true? (A) Working harder is the same as working smarter. (B) Hard work is the only key to success. (C) Working smarter is the only way to achieve your goal. (D) Hard work and luck are important to succeed in life. Ans. Option (A) is correct. Q. 5. To work smarter you need ___________. (A) the right qualifications





















Q. 5. These days, children want to _____. (A) rule their own life (B) go with the flow (C) be looked after by others (D) live their life under the guidance of everyone Ans. Option (B) is correct. Q. 6. The message conveyed by this passage is that parents should _____. (A) give children some space (B) listen to each need of theirs (C) let them solve their problems by helping them from the back (D) all of these Ans. Option (D) is correct. Q. 7. After arriving at a decision almost everyone is ____ in the family mentioned above. (A) happy (B) excited (C) hurt (D) elated Ans. Option (C) is correct. Q. 8. In modern times, the say each family member in decision making points towards ______ in the family. (A) grudges, (B) perfect bonding (C) individual opinions (D) none of these Ans. Option (B) is correct. Q. 9. The synonyms of ‘hurt’ given in para 2 is ____. (A) injured (B) restored (C) arrived (D) decision Ans. Option (A) is correct. Q. 10. The word which means the same as ‘a style or method of cooking’ in para 4 is _____. (A) gourmet (B) cuisine (C) gastric (D) science Ans. Option (B) is correct. VI. Read the following passage carefully : THE ROAD TO SUCCESS (1) The Road to success is not straight; there is a curve called failure, a loop called confusion, speed bumps called friends, caution–lights called family, and you will have flats called jobs. But, if you have a spark called determination, an engine called perseverance, insurance called faith, and a driver called God, you will eventually make it to a place called success! (2) In spite of all our planning and preparation, unexpected challenges are certain to arise. The bumps in the road will be unavoidable, but you’ll never get to your destination if you aren’t willing to drive over them. The difference between a disaster and triumph lies in whether or not you’re prepared and willing to weather the storm. If you’re ready, those bumps will become the stepping-stones. (3) The key thing to realise is that working harder is the same as working smarter. How do you work smarter? You just need to have the right skills. People want improved circumstances and better life without an improved self. They want victory without paying the price or making efforts. It

25

@PROCBSE 26

Oswaal CBSE Question Bank Chapterwise & Topicwise, ENGLISH LANG. & LIT., Class-X























































































































































 























felt to go through one and the urge to get to the stage and perform overcomes even a little desire to learn, if any. If at all, somewhere along the way one feels the need to gain some knowledge and training, it leads to hurried shortcuts and half-hearted attempts, best described as ‘Crash Courses’. (5) It is observed that those who have attained the so called partial success, suddenly feel that they lack the required knowledge and are not learned enough. But it is too late by then. It should be understood here that the stage or a performance brings in a different mind-set within the artist. It is always recommended and rightly said that while on stage, cover the mistakes and weaknesses if any, and get along. But the contrary is true when it comes to acquiring knowledge and during the learning process. While under training, the student is expected to make mistakes but then rectify those mistakes under the supervision and guidance of the teacher. Therefore, it is good to make mistakes and then be corrected during the process of learning as this subsequently makes one flawless and educated. This is a different mind-set. And these two mind-set discussed above, (those of a performer and that of a student) cannot coexist. [Outside Delhi, 2019 (Modified)] On the basis of your understanding of the above passage, answer the questions that follow: (1 × 10 = 10) Q. 1. Music is perhaps the most popular and widely practised form of ____________. (A) performing arts (B) modern arts (C) linguistic arts (D) fine arts Ans. Option (D) is correct. Q. 2. Nature has blessed almost two-thirds of the human race with ____________. (A) singing ability (B) musical ability (C) dancing ability (D) all of these Ans. Option (B) is correct. Q. 3. _____ are brought out by the power of music. (A) Hidden talents (B) Feelings of joy and contentment (C) Deepest emotions (D) Hidden desires Ans. Option (C) is correct. Q. 4. How can we say that music is a magic medicine? (A) It can make one cry or bring a smile on one’s face. (B) It can heal people. (C) It relieves people of their pain. (D) People forget their problems. Ans. Option (B) is correct. Q. 5. What is best described as ‘Crash Courses’? (A) Detailed courses (B) Dedicated learning (C) Years of learning and practice (D) Hurried shortcuts Ans. Option (D) is correct.















(B) the right skills (C) the right contacts (D) all of these Ans. Option (B) is correct. Q. 6. How can one have a better future? (A) By changing the circumstances (B) By changing the surroundings (C) By changing yourself (D) By putting down others Ans. Option (C) is correct. Q. 7. Externally success is measured by _________. (A) money (B) position (C) recognition (D) all of these Ans. Option (C) is correct. Q. 8. Internally it is measured by ___________. (A) feeling of achievement (B) feeling happy (C) feeling satisfied (D) feeling complete Ans. Option (A) is correct. Q. 9. The synonym of ‘victory’ as given in paragraph 2 is (A) planning (B) preparation (C) triumph (D) challenges Ans. Option (C) is correct. Q. 10. The synonym of ‘surplus’ as given in paragraph 5 is _____________. (A) holistic (B) endorse (C) assumes (D) abundance Ans. Option (D) is correct. VII. Read the following passage carefully : (1) Music is perhaps the most popular and widely practised form of fine arts, transcending all kinds of cultural and linguistic barriers. Any form of fine art is difficult to master and almost impossible to perfect and music is no exception. (2) Nature, it is learnt, has blessed almost two-thirds of the human race with musical ability of some sort. Music has the power to bring out the deepest emotions. In fact, it is a magic medicine and many seek refuge in it when they are depressed or stressed. It is this intimacy that makes us listen to music or even hum or sing sometimes. This singing, or realistically speaking, expressing one’s emotion musically sometimes takes a serious turn. (3) The desire to sing before an audience is innocent and beautiful and indeed it is perfectly alright to have such a genuine desire. But it is also important to understand that singing is an intricate art — a highly refined one at that, which requires systematic, prolonged and strict training, to be acceptable. This is an aspect we forget in our keen desire to reach the stage and perform. It is almost like preparing a formal meal for some specially invited guests, without even having learnt and trained in the basics of cooking. (4) These days almost everyone sings and it does not stop here. Most of us want to become professional singers. Result: a complete disregard for and ignorance of the training part, as the need is never

@PROCBSE READING SKILLS

































































































































































 





























27

(6) Some snake killers are prone to have partial immunity to poison. Famously, mongooses are sure; highly resistant to the poison of cobra, and with their speed and agility, they can kill snakes fearlessly and are relatively unharmed . It would be mean definite death of cobras as a species if they did not evolve a more deadly poison to stop and resist the mongooses. (7) Poison has another very important role. It has an extreme meat softening property; their specific enzymes break up the insides of the prey. Normally, a reptile depends on the sun’s warm rays to aid it in its digestion. (8) But I wonder if we cannot use the venom in our favour. In remote parts of India, local hospitality there often involves serving leather-tough meat. I chew and chew until my jaws start aching. If I spit it out or refuse, our hosts would be offended, I swallow like a python stuffing a deer, down its throat and hope I don’t choke. If only I had poison. [Delhi Set, 2019 (Modified)] On the basis of your understanding of the above passage, answer the questions that follow: (1 × 10 = 10) Q. 1. How do Russell vipers attack their prey? (A) Inject poison (B) Crush their prey (C) Push their prey against the ground (D) Bite their prey Ans. Option (A) is correct. Q. 2. Rat snakes attack their prey by ___________. (A) injecting poison (B) crushing their prey (C) pushing their prey against the ground (D) biting their prey Ans. Option (C) is correct. Q. 3. The Sand Boa attacks its prey by __________. (A) injecting poison (B) crushing its prey (C) both (a) and (b) (D) only (a) Ans. Option (C) is correct. Q. 4. How many rats’ vipers consume at a time? (A) one or two at a time. (B) three to four at a time. (C) five to six at a time. (D) seven to eight at a time. Ans. Option (A) is correct. Q. 5. Why do animals try to kill efficiently? (A) as their prey will use any trick to avoid becoming a meal. (B) as their prey might attack them. (C) as they are testing their skill. (D) as they are trying to learn new tricks. Ans. Option (A) is correct. Q. 6. What immunity does Californian ground squirrels have? (A) They have grown bigger in size. (B) They inject poison. (C) They have become resistant to Northern Pacific rattle snake poison.





















Q. 6. The mind-set of a student of music should be ____________. (A) Never to make mistakes. (B) Always be perfect. (C) Cover up the mistakes. (D) Make mistakes but then rectify those mistakes. Ans. Option (D) is correct. Q. 7. In our keen desire to reach the stage and perform we forget _______________. (A) It requires prolonged and strict training. (B) It requires confidence. (C) It requires hard work. (D) It requires talent. Ans. Option (A) is correct. Q. 8. While performing on stage an artist must ___________. (A) make mistakes and seek guidance (B) cover the mistake and get along (C) stop singing (D) none of these Ans. Option (B) is correct. Q. 9. Any form of ________ is difficult to master. (A) fine arts (B) dance (C) cooking (D) all of these Ans. Option (A) is correct. Q. 10. ‘Transcending’ in para 1 means : (A) drown under (B) rise above (C) surrender (D) fail Ans. Option (B) is correct. VIII. Read the following passage carefully : (1) Overpowering prey is a challenge for creatures that do not have any limbs. Some species like Russell’s viper, inject poison into their prey. Some others opt for an alternative non-chemical method - rat snakes, for instance, catch and push their prey against the ground to immobilize them, while pythons use their muscle power to crush their prey to death. But snakes can’t be neatly divided into poisonous and non-poisonous categories. (2) Even species listed as non-poisonous are not completely free of poison. The common Sand Boa, for instance, produces secretions particularly poisonous and dangerous to birds. So, the species does not prefer to take any chance – it crushes its prey and injects poison as an extra step. (3) Do vipers need poison powerful enough to kill hundreds of rats with just one drop? After all, they eat only one or two at a time. (4) While hunting, animals try their best to kill as efficiently as possible while their prey will put in all the efforts, use any trick to avoid getting caught and becoming a meal, such as developing immunity to the poison. For instance, Californian ground squirrels are resistant to Northern Pacific rattle snake getting caught and poison. (5) Competition with prey is not the only thing driving snakes to evolve more and more deadly poison. Snakes also have to struggle to avoid becoming prey themselves.





@PROCBSE 28

Oswaal CBSE Question Bank Chapterwise & Topicwise, ENGLISH LANG. & LIT., Class-X





























































































































































open schools for girls and even outlined suitable curriculum to educate them. He went from door to door, requesting heads of the families to allow their daughters to be enrol them in schools. (4) Ishwar Chandra was a man who defined his own path of action. He never listened to what other people said about his work and made decisions based on his own judgement. He was famous for all these qualities. He was also known for his great courage. Vidyasagar had a very soft heart that sympathised with those in trouble. He was easily moved to tears when he saw someone in pain and was always the first one to offer his help to his colleagues and friends in distress. After his death, Rabindranath Tagore said, “One wonders how God, in the process of producing forty million Bengalis, produced a man!” (CBSE Comptt. 2018 Set 1, 2, 3) On the basis of your understanding of the above passage, answer the questions that follow: (1 × 10 = 10) Q. 1. Why are great people born? (A) to influence the society (B) to bring peace to the world (C) to unite the people (D) to heal the society Ans. Option (A) is correct. Q. 2. Ishwar Chandra Vidyasagar was a __________. (A) painter, dancer, singer (B) social reformer, writer, educator (C) economist, freedom fighter, social reformer (D) all of these Ans. Option (B) is correct. Q. 3. ‘Borno Porichoy’ is a book on____________ (A) Bengali history (B) freedom fighters of Bengal (C) introductory text to teaching Bengali alphabet (D) Bengali writers Ans. Option (C) is correct. Q. 4. What changes did Vidyasagar bring about in the method of teaching at Sanskrit college? (A) started teaching with songs (B) included Bengali and English as learning mediums (C) taught through stories (D) brought modern outlook into the method of teaching Ans. Option (D) is correct. Q. 5. How did Ishwar Chandra Vidyasagar promote women education? (A) wrote books for girls (B) opened schools for girls (C) appointed women teachers (D) all of these Ans. Option (B) is correct. Q. 6. How can we say that Vidyasagar was a soft-hearted man? (A) sympathised with those in trouble (B) built hospitals for people











(D) They can run extremely fast. Ans. Option (C) is correct. Q. 7. Poison has one important role, it ___________. (A) makes the meat sweet (B) reduces the size of the meat (C) it acts as a preservative (D) makes the meat soft Ans. Option (D) is correct. Q. 8. The writer wished he had poison because ____________. (A) he wanted to hurt someone. (B) he wanted to know how the poison works. (C) he wanted to make the meat soft. (D) none of these. Ans. Option (C) is correct. Q. 9. The synonym of the ‘another’ in para (1) is (A) opt (B) alternative (C) a creature (D) against Ans. Option (B) is correct. Q. 10. One word for ‘liquid substances released from glands’: (A) secretions (B) species (C) injects (D) poisonous Ans. Option (A) is correct. IX. Read the following passage carefully : (1) Great people are born with the objective to produce an influence on the society. One such personality, who was so humble and lived his complete life with determination and purpose to achieve certain aims was Ishwar Chandra Vidyasagar. He was not only a great social reformer but also a writer, educator and he worked endlessly to transform the society. He strongly protested against social evils like polygamy and child marriage. He favoured widow remarriage and women’s education in India. The Widow Remarriage Act was passed in 1856 making the marriage of widows legal because of his continuous efforts to solve these problems. (2) Ishwar Chandra Vidyasagar refined the way Bengali language was written and taught. His book ‘Borno Porichoy’ (Introduction to the Alphabet) is still used as the introductory text to teach the Bengali alphabet. Thus, he brought about a revolution in the Bengali Education System. Vidyasagar is credited with the role of completely changing the old method of teaching which prevailed in Sanskrit College. As a professor in Sanskrit College, he bought modern outlook into the method of teaching. He included English and Bengali as mediums of learning, besides Sanskrit. He, also, introduced courses in European History, Philosophy and Science along with Vedic scriptures. He encouraged the students to study these subjects and make the best of both the worlds. (3) Ishwar Chandra Vidyasagar was a strong advocate of women education. He rightly viewed education as the primary way for women to achieve emancipation from the society’s oppression they had to face at that time. He exercised his power to

@PROCBSE

READING SKILLS



















































































































































































succeeded. When asked why he kept going despite his hundreds of failures, he merely stated that they were not failures. They were hundreds of ways not to create a light bulb. This statement not only revealed his grit but also his optimism for looking at the bright side. (4) Grit can be learnt to help you become more successful. One of the techniques that help is mindfulness. Mindfulness is a practice that helps the individual stay in the moment by bringing awareness of his or her experience without judgement. This practice has been used to quiet the noise of their fears and doubts. Through the simple practice of mindfulness, individuals will have the ability to stop the self-sabotaging downward spiral of hopelessness, despair and frustration. (5) What did you do to overcome the negative and self-sabotaging feelings of failure? Reflect on what you did, and try to use those same powerful resources to help you today. On the basis of your understanding of the above passage, answer the questions that follow: (1 × 10 = 10) Q. 1. If we are scared of failure, we are not a robot. (True / False) Ans. True Q. 2. How are humans different from robots? (A) We have feelings and emotions. (B) We are living beings. (C) We are more intelligent. (D) All of these Ans. Option (A) is correct. Q. 3. What is perceived as grit? (A) To be courageous. (B) To persevere and keep going. (C) To be honest. (D) To be determined. Ans. Option (B) is correct. Q. 4. ________ of the seven qualities that have been described as the keys to personal success. (A) Failure, dejection and dishonesty (B) Curiosity, gratitude and optimism (C) Fear, greed and jealousy (D) Envy, defensive and cruelty Ans. Option (B) is correct. Q. 5. Thomas Edison is a perfect example for _______________. (A) courage (B) determination (C) optimism (D) grit Ans. Option (D) is correct. Q. 6. Grit helps you to become______________. (A) fearless (B) successful (C) brave (D) content Ans. Option (B) is correct. Q. 7. What is mindfulness? (A) To stay in the moment. (B) To be aware of your surroundings.













(C) engaged in charity (D) looked after the old and sick Ans. Option (A) is correct. Q. 7. “One wonders how God, in the process of producing forty million Bengalis, produced a man!”. These words were said for _________. (A) Pandit Jawaharlal Nehru (B) Rabindranath Tagore (C) Ishwar Chandra Vidyasagar (D) Bankim Chandra Chatterjee Ans. Option (B) is correct. Q. 8. Which of the following statement is true? (A) Ishwar Chandra Vidyasagar always paid attention to what people had to say about his work. (B) Ishwar Chandra Vidyasagar made decisions based on his own judgement. (C) Ishwar Chandra Vidyasagar was an idealist. (D) Ishwar Chandra Vidyasagar was a great economist. Ans. Option (B) is correct. Q. 9. The synonym of the word ‘impression’ in para 1 is _____________. (A) humble (B) reformer (C) polygamy (D) influence Ans. Option (D) is correct. Q. 10. The word which means the same as ‘change’ in para 1 is _____________. (A) transform (B) favoured (C) determination (D) protested Ans. Option (A) is correct. X. Read the following passage carefully : (1) Have you ever failed at something so miserably that the thought of attempting to do it again was the last thing you ever wanted to do? (2) If your answer is yes, then you are “not a robot.” Unlike robots, we human beings have feelings, emotions, and dreams. We are all meant to grow and stretch despite our circumstances and our limitations. Flourishing and trying to make our dreams come true is great when life is going our way. But what happens when it is not? What happens when you fail despite all of your hard work? Do you stay down and accept the defeat or do you get up again and again until you are satisfied? If you have a tendency to preserve and keep going then you have what the experts call, grit. (3) Falling down or failing is one of the most agonizing, embarrassing and scariest human experiences. But it is also one of the most educational, empowering and essential parts of living a successful and fulfilling life. Did you know that perseverance (grit) is one of the seven qualities that have been described as the keys to personal success and betterment in society? The other six are: curiosity, gratitude, optimism, self-control, social intelligence, and zest. Thomas Edison is a model for grit for trying 1,000 plus times to invent the light bulb. If you are reading this with the lights on in your room, you know well that he

29

@PROCBSE 30

Oswaal CBSE Question Bank Chapterwise & Topicwise, ENGLISH LANG. & LIT., Class-X











































































































































 

























to their handsets by the use of their credit cards, debit cards or by simply buying a prepaid card from the network they subscribe to. This plan also doesn’t commit a particular customer to a contract. If prepaid card is not that appealing to you, then you can opt to subscribe using the pay by month plan. [Outside Delhi, 2018 (Modified)] On the basis of your understanding of the above passage, answer the questions that follow: (1 × 10 = 10) Q. 1. How has the world advanced? (A) introduction of new technologies (B) strengthening the political ties (C) helping each other (D) maintaining world peace Ans. Option (A) is correct. Q. 2. What are the benefits of mobile phones for the young generation? (A) help in online learning. (B) help look up new word they come across. (C) help complete assignments on time. (D) all of these Ans. Option (D) is correct. Q. 3. How can mobile phones be considered ‘lifesavers’? (A) They automatically connect you to a doctor. (B) They warn you of an impending crime. (C) They prevent crimes by providing information to security forces. (D) They give life saving tips. Ans. Option (C) is correct. Q. 4. Mention any one demerit of mobile phones. (A) very expensive (B) take a lot of time to charge (C) affect academic performance (D) difficult to avail services Ans. Option (C) is correct. Q. 5. Through mobile, internet access is _________ times hurried and short. (A) ten (B) fifty (C) hundred (D) more than ten Ans. Option (A) is correct. Q. 6. Which of the following statement is true? (A) Smart phones can connect people with their loved ones anywhere in the world at cheaper rates. (B) Smart phones have distanced the people from one another. (C) Smart phones have increased the expenses of people. (D) Smart phones have made life easy. Ans. Option (A) is correct. Q. 7. How can young people be kept away from the ill effects of mobile phones? (A) They should not be given mobile phones. (B) They should be strictly monitored. (C) They should be mentored. (D) They should be allowed to use the mobile phones sparingly. Ans. Option (C) is correct.















(C) To be cautious. (D) To be self aware. Ans. Option (A) is correct. Q. 8. Mindfulness helps the individual _________. (A) to be scared (B) to face the challenges (C) to be bold (D) from self-sabotaging Ans. Option (D) is correct. Q. 9. Failure is a part of ___________ life. (A) human (B) daily (C) regular (D) all of these Ans. Option (A) is correct. Q. 10. In para 2, ___________ means ‘to continue’. (A) robots (B) satisfied (C) persevere (D) flourishing Ans. Option (C) is correct. XI. Read the following passage carefully : (1) It is an indisputable fact that the world has gone too far with the advancement of new technologies such as mobile phones, the internet and so on, due to which people are able to tour the cosmos virtually sitting at one place using their smart devices or other technological gadgets. Through mobile, internet access is ten times hurried and short, it can still provide common internet features like alerts, weather data, e-mails, search engines, instant messages and game and music downloading. (2) Due to the easy access of smart phones, communication has become very effective, efficient and faster. People are able to convey their message all around the globe to their loved ones without spending hefty sums of money. Adults are always fond of such gadgets and they always welcome and adopt such new technology readily. Further, it has helped young people as they have been able to broaden their minds and improve their skill by doing research on the Internet. For instance, they use smart phones to look up any new word they come across. As we know that most of the universities have an online teaching provision, smart phones assist the students to complete their assignments on time and avoid delay. (3) The mobile phone has been a lifesaver for a lot of people in case of an emergency. Likewise, use of smart phones can be of vital importance in preventing crimes in the society by providing information to the security forces in time. (4) Nonetheless, for the young, the use of mobile phone can be like an addiction and they can also misuse it. Young people are also prone to getting involved in undesirable activities on the Internet. This might have adverse effect on their academic performance. Therefore, young people should always be mentored and made aware of its bad outcomes. (5) Also a major contributor to its popularity is the easy availability of prepaid or pay as you go services from a phone shop or an online store. This allows subscribers to load text or airtime credits

@PROCBSE

READING SKILLS











































































































































 

































many such examples in nature, but we are not ready to learn a lesson yet. Overcome with greed, we are destroying the nature. As a result, we face natural disasters like drought, flood and landslides. We don’t know that nature is angry with us. However, it is never too late to learn. If we learn to respect nature, the quality of our life will definitely improve. [Outside Delhi, 2017 (Modified)] On the basis of your understanding of the above passage, answer the questions that follow: (1 × 10 = 10) Q. 1. When we are children, we are taught to __________. (A) go to school every day (B) help your parents (C) study regularly (D) to worship the photos and idols of the Gods of our respective religions Ans. Option (D) is correct. Q. 2. What do we learn from the holy books? (A) life lessons (B) good manners (C) about nature (D) about our past Ans. Option (A) is correct. Q. 3. Why should we respect our parents? (A) They teach us good values. (B) They work hard to make our life comfortable. (C) It is our duty. (D) All of these Ans. Option (B) is correct. Q. 4. The narrator learnt that we learn a lot from our surroundings when he learnt to _____________. (A) ask questions (B) enquire and explore (C) analyse the scripture (D) decipher the scriptures Ans. Option (B) is correct. Q. 5. In many cultures besides scriptures, ________ is also worshipped. (A) Idols (B) planets (C) nature (D) water Ans. Option (C) is correct. Q. 6. According to the author, where do we learn more than our holy books? (A) parents (B) grandparents (C) surroundings (D) family Ans. Option (C) is correct. Q. 7. What message do we get from the scriptures? (A) to save our environment and maintain ecological balance (B) to respect our parents (C) to be kind (D) to be virtuous Ans. Option (A) is correct. Q. 8. What does the river teach us? (A) to be progressive in life (B) to face the obstacles (C) to never give up (D) to face the challenges 























Q. 8. What has contributed to the popularity of mobile phones? (A) cheap in cost (B) easy availability of services (C) various apps and its features (D) to show as status symbol Ans. Option (B) is correct. Q. 9. In para 1, synonym of ‘innovation’ is _______. (A) production (B) sincerity (C) invention (D) prevention Ans. Option (C) is correct. Q. 10. Mobile internet can provide access to _______. (A) emails (B) letters (C) stories (D) none of these Ans. Option (A) is correct. XII. Read the following passage carefully : (1) During our growing up years, we as children have been taught–both at home and also school—to worship the photos and idols of the Gods of our respective religions. When we grew a little older, we were told to read holy books like the Bhagwad Gita, the Bible and the Quran; we were told that there are a lot of life lessons to be learnt from these holy books. We were then introduced to stories from our mythologies which have taught us about ethics and morality—what is good and what is bad. I also learnt to be respectful towards my parents who have made my life comfortable with their hard work and love and care, and my teachers who have guided me to become a good student and also a responsible citizen. (2) Much later in life, I realised that though we have learnt much from our holy books, there is a lot to learn from our surroundings. This realisation dawned upon me when I learnt to enquire and to explore. Everything around us—the sun, the moon, the stars, rain, rivers, stones, rocks, birds, plants and animals—teach us many valuable life lessons. (3) No wonder that besides the scriptures in many cultures, nature is also actively worshipped. The message that we get is to save our environment and to maintain ecological balance. People are taught to live in harmony with nature and recognise that there is God in all aspects of nature. (4) Nature is a great teacher. A river never stops flowing. If it finds an obstacle in its way in the form of a heavy rock, the river water fights to remove it from its path or finds an alternative path to move ahead. This teaches us to be progressive in life, and to keep the fighting spirit alive. (5) Snakes are worshipped as they eat insects in the field that can hurt our crops, thus protecting the grains for us. In fact, whatever we worship is our helper and makes our lives easy for us. There are

31

@PROCBSE 32

























































































































Q. 9. The word which means the same as ‘trained’ in para 1 is ____________. (A) tamed (B) volunteer (C) obedience (D) therapy Ans. Option (A) is correct. Q. 10. What is meant by the word, ‘obedience’? (para 1) (A) faithfulness (B) servility (C) calmed (D) respect























































Ans. Option (D) is correct. Q. 2. She was called ____________ when she was little. (A) The Demon (B) The Devil (C) Naughty (D) Doodle Ans. Option (A) is correct. Q. 3. Maggie _______ when she was small. (A) loved to bite (B) loved to go for walks (C) loved to play (D) loved to sleep Ans. Option (A) is correct. Q. 4. Why did the author want a dog? (A) to entertain (B) to keep him walking (C) to keep him company (D) all of these Ans. Option (B) is correct. Q. 5. The author is grateful to the Canine Academy because ________________. (A) it taught her to respond to commands. (B) it groomed her. (C) it transformed the unruly dog (D) none of these Ans. Option (C) is correct. Q. 6. The author had difficulty in taking her for walks earlier because ______________. (A) her weight was too much for the author to manage (B) she was too fast (C) the author had difficulty walking (D) the author was not used to taking dogs for walks Ans. Option (A) is correct. Q. 7. Which of the following statement is true? (A) We go for walks daily. (B) We go for a walk twice every day. (C) We go for a walk on alternate days. (D) We never go for walks. Ans. Option (B) is correct. Q. 8. How did Lisa and her trainers help Maggie? (A) taught her to walk slowly (B) taught her to fetch things (C) groomed her (D) gave her the tools to be a wonderful dog Ans. Option (D) is correct.





























Ans. Option (A) is correct. Q. 9. The synonym of ‘advised’ is ______________. (A) ethics (B) respective (C) guided (D) morality Ans. Option (C) is correct. Q. 10. The synonym of ‘search’ in para 2 is ______________. (A) dawned (B) enquire (C) valuable (D) explore Ans. Option (D) is correct. XIII. Read the following passage carefully : (1) Maggie is a Golden Doodle. However, she has turned out to be more Golden than just Doodle. She has lots of energy, is very loving and a people pleasure, soft and loves every minute of life. But when she was small, we called her “The Demon”! We have our reasons for naming her so. She lived to jump on me, bite me, and steal anything and everything. My goal had been to get a dog that would keep me walking every day. But more than that, I am in a period of my life where I was looking forward for a way to give back to the society. Seeing Maggie’s loving personality and high energy, I realised that Maggie needed a job and I needed to volunteer, and if we worked together, we could both have what we seek. But the little “demon” needed to be tamed first ! After going through a beginner and intermediate obedience at Canine Academy, I knew that Maggie would benefit from the Off-Leash programme and that would help her to get ready to become a therapy dog, so that she could be kept busy and I could give back. (2) She is home from the Academy now, and what a great dog she has become! She loved her training and did very well at the Canine Academy. Walks that used to be difficult earlier because at 75 lbs, her pulling on the leash was hard on me, has now turned into a pleasurable experience. We walk twice every day and she has many friends in the neighbourhood, that have watched her grow from an unruly little puppy to a beautiful companion, all thanks to the Canine Academy. We train every day and she loves to attend the weekly training sessions that the Off-Leash graduates take part in. Lisa and her trainers gave Maggie the tools she needed to be an absolutely wonderful and loved dog. Now we work together to fine tune those tools and everyone that knew her before, is quite impressed by how well-behaved she is now as compared to before. My life is so much better with Maggie in it, but part of that is thanks to the wonderful people at the Canine Academy and the great work they do with dogs and transform them. If I ever add to my dog family, we will surely be calling them again! [Board Term 1, 2016-17 (Modified)] On the basis of your understanding of the above passage, answer the questions that follow: (1 × 10 = 10) Q. 1. Maggie is a Golden ______________. (A) Retriever (B) Pug (C) Labrador (D) Doodle



Oswaal CBSE Question Bank Chapterwise & Topicwise, ENGLISH LANG. & LIT., Class-X

@PROCBSE

































































































































33

On the basis of your understanding of the above passage, answer the questions that follow: (1 × 10 = 10) Q. 1. Generation gap is seen between ___________. (A) grandparents and grandchildren (B) two siblings (C) parents and children (D) children and rest of the adult members in the family Ans. Option (C) is correct. Q. 2. Parents have to empathise with the changing values and ___________ of the modern world. (A) thinking pattern (B) technological advancement (C) peer pressure (D) challenges Ans. Option (A) is correct. Q. 3. Communication gap is caused because of ___________. (A) unavailability of mobiles (B) faulty telephone lines (C) lack of good services (D) generation gap Ans. Option (D) is correct. Q. 4. Parents should ______________. (A) monitor the children (B) put restrictions on the children (C) talk to the children openly (D) neglect the children Ans. Option (C) is correct. Q. 5. When do children become mentally isolated? (A) When they don’t have friends. (B) When they don’t perform well in exams. (C) When the parents seem authoritative. (D) When their demands are not fulfilled. Ans. Option (C) is correct. Q. 6. What is the result of children spending more time on digital devices? (A) They avoid their parents. (B) They become aggressive. (C) They become demanding. (D) They don’t discuss their problems with their parents. Ans. Option (D) is correct. Q. 7. Who provides solutions to most worldly problems? (A) internet (B) friends (C) parents (D) teachers Ans. Option (A) is correct. Q. 8. Parents should avoid _____________the children. (A) neglecting (B) scolding (C) beating (D) pressurising Ans. Option (B) is correct. Q. 9. What is meant by the word, ‘particular’? (para 2) (A) specific (B) usual (C) normal (D) instant Ans. Option (A) is correct.









 





 











Ans. Option (A) is correct. XIV. Read the following passage carefully : (1) The difference in the ways of thinking and perception in the people of two different generations, which results in behavioural differences and sometimes, conflict among them is usually called generation gap. (2) Generation gap is generally seen in the family between parents and their children. It is not only because of the gap in age, but also because of the way parents react to a particular situation. Children being very young and immature do not understand the way of thinking of their parents. In many cases, the parents, even if they are matured, do not empathise with the changing values and thinking pattern of the modern world. This creates a communication gap between these two generations. (3) Generation gap between parents and children is mostly caused by parents themselves. They do not talk openly to their children and do not take part in solving their problems. This behaviour gives the impression about parents being authoritative persons, because they only dictate without understanding the problems faced by their children. (4) As a result, the children become mentally isolated from their parents. Some parents become so busy with their work that they don’t spend quality time with their family, which makes them completely unaware of how their children are growing, what kind of mentality is being developed in them, and so on. This ultimately creates a gap between them. They only realise it when it is too late. (5) Changes in technology have led to the generation gap in this modern world. Children tend to spend most of their time with their digital devices and are so busy with social media, that they do not discuss their problems with their parents. The internet provides solutions to most of the worldly problems. So children, instead of contacting their own parents, seek the help of internet in case of any problem, thus again reducing the communication, leading to a generation gap between them. (6) The generation gap has greatly increased these days because the time is changing fast and people find it difficult to cope up with this change. To overcome this problem, parents should show interest in all matters of their children and deal with them positively instead of just scolding them and leaving them alone to deal with their problems. Giving time to the children, having open communication, and allowing the child to feel free and not in any kind of pressure, will eventually reduce the generation gap to a tolerable extent. [Board Term 1, 2016-17 (Modified)]



READING SKILLS





















@PROCBSE 34

Oswaal CBSE Question Bank Chapterwise & Topicwise, ENGLISH LANG. & LIT., Class-X





































































































































































a major bone of contention throughout history. [Board Term 1, 2016-17 (Modified)] On the basis of your understanding of the above passage, answer the questions that follow: (1 × 10 = 10) Q. 1. Which convention is considered to be boring? (A) accountants (B) lawyer (C) librarians (D) teachers Ans. Option (A) is correct. Q. 2. ___________ is the most interesting thing on earth. (A) Nature (B) Human beings (C) Animals (D) Money Ans. Option (D) is correct. Q. 3. What is fascinating and magical? (A) how money appeared on earth. (B) how the planets were formed. (C) study of the solar system. (D) how life appeared on earth. Ans. Option (A) is correct. Q. 4. West Africans traded in ______________. (A) ivory (B) manillas (C) quartz (D) shells Ans. Option (B) is correct. Q. 5. Which of the following was considered an accepted currency? (A) whale’s teeth (B) elephant tusks (C) tiger skin (D) shark teeth Ans. Option (A) is correct. Q. 6. What is charming and child like about our primitive society? (A) Their way of life. (B) The clothes they wore. (C) The colourful money they used. (D) The jobs they did. Ans. Option (C) is correct. Q. 7. Who are in a strong position? (A) The government. (B) The politicians. (C) The accountants. (D) Those who produce and loan out money. Ans. Option (D) is correct. Q. 8. People who manipulate money are called ____________. (A) money changers (B) money makers (C) game changers (D) money planners Ans. Option (A) is correct. Q. 9. What is meant by the word, ‘convention’? (para 1) (A) conference (B) fair (C) wave (D) competition Ans. Option (A) is correct. Q. 10. The word, ‘fascinating’ in para 2 means (A) attractive (B) uninteresting (C) appalling (D) revolting











 



  

































Q. 10. What is meant by the word, ‘isolated’? (Para 3) (A) bound (B) grown (C) separated (D) unconcerned Ans. Option (C) is correct. XV. Read the following passage carefully : (1) Tell someone that you are going to a convention of accountants and you might get a few uninterested yawns, yet money and how it works is probably one of the most interesting and intriguing things on the earth. It is fascinating and almost seems magical, how money appeared on our planet. Unlike most developments we enjoy, which can be traced back to a source, civilisation or inventor, money appeared in places then unconnected all over the world in a more or less remarkably similar way. (2) Consider the American Indians using Wampum, West Africans trading in a decorative metallic objects called Manillas and the Fijians economy was based on whale’s teeth, some of which are even now considered legal tender; add to that shells, amber, ivory, decorative feathers, cattle including oxen and pigs and a large number of stones including jade and quartz, which have all been used for trade across the world, and we will get a taste of the variety of accepted around the world. (3) There is something charming and child- like imagining our primitive societies, our ancestors, using all these colourful forms of money. As long as everyone concerned can agree on a suitable value, this is a very sensible thing for a community to do. (4) After all, the person who has what you need might not need what you have to trade. Money solves that problem very neatly. Real value with each and every exchange, and everyone gaining from the convenience. The idea is really inspired, which might explain why so many diverse minds had come up with it. Once money has been accepted as a form of exchange, those who produce, loan out and manipulate the quantity of money are obviously in a very strong position. They are the “Money Changers”. (5) “History records that the money changers have used every form of abuse, intrigue, deceit, and violent means possible to maintain their control over the governments by controlling money and its issuance,” said President James Madison. (6) Money, money, money, we feel it has always just been there, right? Wrong. Obviously according to us it is issued by the government to make it easy for us to exchange things. Wrong again! (7) Truth is, most people do not realise that the issuing of money is essentially a very private business, and that the privilege of issuing money has been

@PROCBSE READING SKILLS





















































































































(4) Brain Drain means the professionals going abroad to work and settle. This is not good for the country of departure because nowadays all the good brains are going out to work for other countries in the world. If they had efficiently utilised their brains in their own country, India could have been a far better and developed country. America has become a developed country, because maximum number of employees consists of skilled Indians. So if those employees had worked for India instead of going to America, India would surely have become equivalent to America if the Indians who left had decided to stay back. So Brain Drain is not good from the economic point of view. [Board Term I, 2015-16 (Modified)] 





(2) The urge to go to the West has become more compelling in India in the post-independence years. Both the world wars and political upheavals like civil wars and revolutions had spawned large scale migrations around the world during the first half of the 20th century. The migrations in the latter half has stemmed from the economic motivation. People in power like scientists and skilled persons have, from the very beginning, played a key role in the international migration. The conscious policy to encourage the migration of high-quality manpower from other regions and countries is related to the positive contribution and inputs made by these immigrants to their country of adoption, resulting in a net advantage resulting in increased profit to.

(3) The earnings and remittances from the Indians who are settled abroad or their eventual return, are a poor compensation for the lasting and serious loss inflicted by their outflow which is irreparable. The specific objectives of the anti-brain drain policies, within the specific framework of independent development, are in whatever way possible trying to bring back to a limited extent in whatever way possible, the lost talents and skills from foreign countries and start reducing the outflow in the short run and finally end it in whatever way possible in the future, except that necessitated by genuine, multilateral international dependence.







(1) Give me your tired, your poor, your huddled masses yearning to breathe free. These are the words which are inscribed on the famous Statue of Liberty in New York. But ironically, those who go there are not the poor and the wretched but the skilled and the highly qualified doctors, engineers, scientists, nurses, technicians from other countries around the world including India.

35

On the basis of your understanding of the above passage, answer the questions that follow: (1 × 10 = 10) Q. 1. “Give me your tired, your poor, your huddled masses yearning to breathe free“are inscribed on ______________ . (A) Gateway of India (B) Statue of Liberty (C) Taj Mahal (D) Lincoln Memorial Ans. Option (B) is correct. Q. 2. Who goes to America? (A) Poor (B) Wretched (C) Unemployed (D) Qualified people Ans. Option (D) is correct. Q. 3. The urge to go to the _____________ has been compelling in India. (A) West (B) North (C) South (D) East Ans. Option (A) is correct. Q. 4. _____________ has spawned large migrations during the first half of the 20th century. (A) Natural disasters (B) Unemployment (C) World wars (D) Economic depression Ans. Option (C) is correct. Q. 5. Migration in the later half was a result of ______________. (A) better lifestyle (B) good climatic conditions (C) better healthcare (D) economic motivation Ans. Option (D) is correct. Q. 6. Migration has given ____________ to their adopted country. (A) more labour (B) positive contribution (C) financial benefits (D) economic strain Ans. Option (B) is correct. Q. 7. Returning India provide ____________ for the lasting and serious loss inflicted by their outflow. (A) poor compensation (B) economic dependence (C) financial boost (D) increased compensation Ans. Option (A) is correct. Q. 8. What is Brain Drain? (A) Inflow of people from other countries. (B) People going to the other counties for studies. (C) People going abroad to work and settle. (D) All of these. 

VI. Read the following passage carefully :







Ans. Option (A) is correct.













@PROCBSE 36

Oswaal CBSE Question Bank Chapterwise & Topicwise, ENGLISH LANG. & LIT., Class-X

Commonly Made Errors



















Most candidates merely copied the options without reading the whole passage properly. Very few students could guess the correct answer for vocabulary based question.

Answering Tips







































Ans. Option (C) is correct. Q. 9. What is meant by the word ‘inscribed’? (para 1) (A) designed (B) written (C) read (D) painted Ans. Option (B) is correct. Q. 10. Find the word from the passage which means ‘great changes or disturbances’. (para 2) (A) wars (B) immigrants (C) migrations (D) upheavals Ans. Option (D) is correct.

Candidates can guess the antonyms and synonyms from the context. Regular use of dictionary is advisable. Practice unseen passage on regular basis.







REFLECTIONS Were you able to increase your reading speed, after practising these passages? Did you feel like that your grammar and vocabulary skills has improved?

@PROCBSE

READING SKILLS

37

SELF ASSESSMENT PAPER - 1

Max. Marks: 30 

  















I. Read the following passage carefully : (10 marks) (1) The white tiger is an offspring of Bengal tiger, which has the necessary gene for white colouring. It has beautiful black stripes on a white coat. This species of animal has blue eyes and a pink nose. (2) A pure white tiger is totally white without any stripe, which is due to the presence of double recessive allele in the genetic code. It happens only in the Bengal tiger subspecies, and only one in 10,000 births can have it naturally. White tigers are rarely seen in the wild, and only twelve of them have been spotted in India since last one-hundred years. Tigers have a life-span of 10-15 years. They are solitary animals, and are fond of hunting at night. Their diet consists of any animals that they can catch, and can eat up to 40 pounds of meat at one time. After such a big meal, they usually remain away for several days from food. (3) They are slow runners, but good swimmers. They are poor climbers and cannot climb a tree easily. They live in an area ranging from 10 to 30 square miles depending upon the availability of prey, water and shelter. Habitat loss, poaching and expansion of human population have together contributed to their becoming endangered. They are widely hunted for their body-parts, which are used in traditional Chinese medicines. (4) Genetic inbreeding is another reason for their endangerment. Due to shortage in their number, this species of animal has been subjected to inbreeding. It has resulted in high mortality rates and severe disabilities. About 80 percent of the white tiger cubs are born with serious birth-defects related to inbreeding, and they soon die-off. (5) The critical condition of this animal has attracted the attention of conservationists worldwide. The World Wildlife Federation (WWF), defenders of wildlife, and many other wildlife organizations are working with the support of the government for the cause of protecting them from becoming extinct. Once they become extinct, it will not be possible to bring them back to life.



Max. Time: 1 hour

1411

1706

2226

2006

2010

2014

2967 2018

Increasing Tiger Number 











































































































On the basis of your understanding of the above passage, answer the questions that follow: Q. 1. The factors that have put tiger species in the category of endangered species are: (A) habitat loss (B) poaching (C) genetic inbreeding (D) all the above Q. 2. Which of the following statements is not true about tigers? I. The tigers are social animals. II. The tigers are slow runners. III. The tigers can climb a tree easily. IV. The tigers have become endangered. (A) both I & II (B) both I & III (C) both II & III (D) both II & IV Q. 3. What is the life span of a tiger? (A) 5-10 years (B) 10-15 years (C) 15-20 years (D) 20-25 years Q. 4. The White tiger is an offspring of (A) Rajasthan tiger (B) Chennai tiger (C) Punjab tiger (D) Bengal tiger Q. 5. Tigers are ___________ climbers and cannot climb a tree easily. (A) expert (B) goat (C) poor (D) worst Q. 6. A pure ___________ tiger is totally white without any stripes. (A) white (B) yellow (C) golden (D) brown

(1 × 10 = 10)

@PROCBSE 38

Oswaal CBSE Question Bank Chapterwise & Topicwise, ENGLISH LANG. & LIT., Class-X

































Read the following passage carefully :

(10 marks)









(1) I know poverty and misery and I quite appreciate by personal experience what it is to be poor, what it is to have no clothes, what it is to have no books, what it is to struggle through life, what it is to walk through the streets without an umbrella, without conveyance along miles on dusty roads. I have been through it all and I can understand the difficulties that most of you graduates have to face today. I am speaking from a long experience of 60 years. Please do not imagine that all the 60 years were milk and roses. To be able to accomplish something, I want to tell you that you have to go through such experiences. (2) I admit, success in life is not always to the intelligent or the strong, and it is to some extent a bit of a gamble, but nonetheless, those who got their minds right and those who know their job sooner or later, will sooner perhaps than later make their way in life. But they should not be disappointed if they do not, they have to face up life and take it as they find it. (3) What I say is that the great things in life are not really great things in life. What I love is to enjoy the common things of life. I am happy that I am still able to sleep at night provided I have three miles walk in the evening. I am still able to enjoy a good lunch or a good dinner. I am still able to look at the blue sky and like it. Still like to walk in the open fields and like the smell of the Ragi or the Jowar. (4) We think that happiness consists in going to pictures and seeing thrilling films and techni colour dramas. Not at all, the great things in life are the God-given things which cost nothing. What you need is the desire to appreciate them. If you have your minds and hearts open, you have around you, things which give you joy. There is the butterfly jumping about in flourishing colours on all sides. Look at the wonderful things that God has given for our enjoyment. (5) We have to love nature and appreciate nature and appreciate her wonderful gifts, of nature’s marvellous ingenuity, its resourcefulness, and infinite variety. It is the same thing that has inspired me all my life. On the basis of your understanding of the above passage, answer the questions that follow: (1 × 10 = 10) What does C.V. Raman know about being poor? (A) what it is to have no books. (B) what it is to have no clothes. (C) what it is to struggle through life. (D) all of these. How many years of experience does C.V. Raman have? (A) 60 years (B) 50 years (C) 70 years (D) 40 years In what way does tough experiences help? (A) teaches to value education. (B) teaches to value money. (C) helps to accomplish something. (D) helps in facing challenges. Success in life is not always to the __________. (A) intelligent and strong (B) rich and wealthy (C) hardworking and patient (D) strong and powerful Which of the following statement is true? (A) People should not be disappointed if they do not succeed. (B) People should give up trying if they do not succeed. (C) People should be disappointed if they do not succeed. (D) People should accept defeat if they do not succeed. What does the author love? (A) To spend time with his family. (B) To go to work. (C) To enjoy common things of life. (D) To learn new things. 



































Q. 6.















Q. 5.











Q. 4.











Q. 3.











Q. 2.











Q. 1.





















II.





































Q. 7. ___________ affects the tiger from remaining away for several days from food. (A) A big meal (B) Sleep (C) Rest (D) Thirst Q. 8. The number of tigers in 2018 shows an increase by almost ____________ than in 2014. (A) one fifth (B) one third (C) half (D) one tenth Q. 9. The report of the tiger census given above from 2014 to 2018 is ___________ . (A) depressing (B) disgusting (C) encouraging (D) decreasing Q. 10. The year 2006 showed the _____________ number of tigers. (A) highest (B) average (C) lowest (D) no change

@PROCBSE

READING SKILLS

39













































































































































Q. 7. The great things of life are ____________. (A) expensive (B) free (C) difficult to find (D) God given Q. 8. According to C.V. Raman, what should we love? (A) Family (B) Nature (C) Friends (D) God Q. 9. The synonym of ‘despair’. (A) disappointment (B) hopelessness (C) misery (D) distress Q. 10. The word which means the same as ‘endless’ in the last para is __________. (A) appreciate (B) resourcefulness (C) infinite (D) inspired III Read the following passage. THE ULTIMATE PRODUCTIVITY HACK IS SAYING NO -by James Clear 1. Not doing something will always be faster than doing it. This statement reminds me of the old computer programming saying, “Remember that there is no code faster than no code.” The same philosophy applies in other areas of life. For example, there is no meeting that goes faster than not having a meeting at all. This is not to say you should never attend another meeting, but the truth is that we say too many things we don’t actually want to do. There are many meetings held that don’t need to be held. 2. But if the benefits of saying no are so obvious, then why do we say yes so often? We agree too many requests not because we want to do them, but because we don’t want to be seen as rude, arrogant, or unhelpful. Often, you have to consider saying no to someone you will interact with again in the future—your co-worker, your spouse, your family and friends. Saying no to these people can be particularly difficult because we like them and want to support them. (Not to mention, we often need their help too.) Collaborating with others is an important element of life. The thought of straining the relationship outweighs the commitment of our time and energy. 3. But even after we have accounted for these social considerations, many of us still seem to do a poor job of managing the trade-off between yes and no. We find ourselves over-committed to things that don’t meaningfully improve or support those around us, and certainly don’t improve our own lives. 4. Perhaps one issue is how we think about the meaning of yes and no. The words “yes” and “no” get used in comparison to each other so often that it feels like they carry equal weight in conversation. In reality, they are not just opposite in meaning, but of entirely different magnitudes in commitment. When you say no, you are only saying no to one option. When you say yes, you are saying no to every other option. In other words, saying no saves your time in the future. Saying yes costs your time in the future. No is a decision. Yes is a responsibility. 5. Saying no is sometimes seen as a luxury that only those in power can afford. And it is true: turning down opportunities is easier when you can fall back on the safety not provided by power, money, and authority. 6. Nobody embodied this idea better than Steve Jobs, who said, “People think focus means saying yes to the thing you’ve got to focus on. But that’s not what it means at all. It means saying no to the hundreds other good ideas that there are. You have to pick carefully.” 7. There is an important balance to strike here. Saying no doesn’t mean you’ll never do anything interesting or innovative or spontaneous. It just means that you say yes in a focused way. Once you have knocked out the distractions, it can make sense to say yes to any opportunity that could potentially move you in the right direction. You may have to try many things to discover what works and what you enjoy. 8. What is true about health is also true about productivity: an ounce of prevention is worth a pound of cure. More effort is wasted doing things that don’t matter than is wasted doing things inefficiently. And if that is the case, elimination is a more useful skill than optimization. I am reminded of the famous Peter Drucker quote, “There is nothing as useless as doing efficiently that which should not be done at all.” On the basis of your understanding of the above passage, answer the questions that follow: (1 × 10 = 10) Q. 1. According to the passage, select the option that best describes what the author intends to propagate through this article. (A) You should never do anything interesting or innovative or spontaneous. (B) Say ‘NO’ to all the meetings that your boss has scheduled without your consent. (C) Saying ‘NO’ is a process of elimination of distractions to improve productivity. (D) Saying ‘NO’ is sometimes seen as a luxury that only those in power can afford. Q. 2. What are the reasons cited by James Clear for people’s inability to say ‘NO’ in the passage? I. People don’t want to be seen as rude, arrogant, or unhelpful. II. People don’t want to ruin their chances of promotion. III. People don’t want to strain their relationships with their co-worker, spouse, family or friend. IV. People who are not in power cannot afford to say ‘NO’. V. People have been trained and programmed to only say ‘YES’. (A) All of the above (B) I, III, IV (C) I, II, V (D) I, II, IV Q. 3. The word that means the opposite of ‘inefficiently’ in Para 8: (A) competently (B) prevention (C) elimination (D) cure

@PROCBSE 40

Oswaal CBSE Question Bank Chapterwise & Topicwise, ENGLISH LANG. & LIT., Class-X

























(A) Image 1 (B) Image 2 (C) Image 3 (D) Image 4 7. Select the option that best explains: ‘Elimination is a more useful skill than optimization’. (A) Eliminate people who give you more work and you’ll become efficient. (B) Eliminate tasks that are not focused towards your goal to increase productivity. (C) Skill to say ‘NO’ can outweigh being most efficient. (D) Skill to eliminate redundant tasks increases productivity. 8. After reading the passage, when should one say ‘YES’? (A) Once you have knocked out all the distractions and useless activities. (B) To any opportunity that could potentially move you in the right direction. (C) To someone you will interact with again in the future. (D) All of the above. 9. Not doing something will always be faster than doing it. Which of the following statements elucidate the point? (A) Remember that there is no code faster than no code. (B) There is no meeting that goes faster than not having a meeting at all. (C) There is nothing as useless as doing efficiently that which shouldn’t be done at all. (D) All of the above 10. Which of the following words best suit as meaning of ‘embodied’ in Para 6? (A) concretize (B) manifest (C) incorporate (D) All of these













Q.













Q.













Q.















Q.























Q. 4. The word that means the same as ‘exceed in value or importance’ in Para 2: (A) straining (B) outweighs (C) collaborate (D) commitment Q. 5. Select the suitable title for Para 4. (A) Difference between YES & NO (B) Reasons for saying YES (C) The role of NO (D) Why we say YES Q. 6. Which of the following images do not match with ‘an ounce of prevention is worth a pound of cure’?

@PROCBSE

































@PROCBSE 42

Oswaal CBSE Question Bank Chapterwise & Topicwise, ENGLISH LANG. & LIT., Class-X

@PROCBSE

GRAMMAR













reached her house. Keywords : since, for Verb : MV- I form + ing HV- had been







1.







l

















3.



4.



Future Tense :

Future Indefinite (Simple) Tense Usage : Refers to simple action which is yet to take place. I will go to Surat tomorrow. Keywords : tomorrow, next, coming etc. Verb : MV- I form HV- will, shall (Modals) NB- 1- 1st Person (I, We) - shall 2nd & 3rd Person (You, He/ She, It, They)-will Future Continuous Tense Usage : A progressive future action. At this time tomorrow, she will be doing her homework. Keywords : tomorrow at this time etc. Verb : MV- I form + ing HV- will, shall (Modals) + be Future Perfect Tense Usage : Refers to an action which will be completed at the said time in future. e.g., I will have prepared supporting material by Saturday. Keywords : after, by, etc. Verb : MV- III form HV- will, shall (Modals) + have Future Perfect Continuous Tense Usage : An action which will begin before a definite time in the future and will just end up at that time or may continue even after that. When I reach my school at 08:30 a.m. tomorrow, the teacher will have been giving remedial classes. Keywords : - Since, for, after three years, etc. Verb : MV- I form + ing HV- will, shall (modals) + have been

Future Time Reference By using – will/shall I‘ll go to Agra tomorrow. By using – Simple Present tense What time does the match begin? By using – Present Continuous tense The PM is leaving for America next week. By using – Going to : I am going to take bath. By using – about to : The train is about to come. l

5.



l



4.





l

3.





l

2.





l



 1.









4.













l





3.











l



l













2.





Past Indefinite (Simple) Tense Usage : Action done in remote past. Example I went to Mathura yesterday. I did not go to Mathura yesterday. Keywords : - yesterday, last, etc. Verb : MV- II form (Affirmative sentences) HV- Did NB- 1-HV ‘did‘ is used in negative & interrogative sentences. 2. With HV ‘did‘, I form of MV. Past Continuous Tense Usage : An action done progressively in the past. Example He was working on a project, when I last visited him. Keywords : those days, etc. Verb : MV-I form + ing HV- was, were Past Perfect Tense Usage : An action completed in the past before a said time or another action. The patient had died before the doctor came. Keywords : before, when, as soon as etc. Verb : MV- III form (earlier past) & II form (Past) HV- had (earlier past) Past Perfect Continuous Tense Usage : An action being done continuously for the said period in the past. She had been reading for two hours when I





Past Tense :



 1.

2.

















l



l







4.











Keywords : Recently, Just now Verb : MV- III form HV- Has (Singular Subject)/ Have (Plural Subject) Perfect Continuous Tense Usage : Action started somewhere in the past but being done now. Example I have been living in Baroda for the last two years. She has been studying in this school for two years. Keywords : Since, for Verb : MV- I form + ing HV- Has been (Singular Subject)/ have been (Plural Subject) Since- Point of time For- Period of time

43

@PROCBSE 44

Oswaal CBSE Question Bank Chapterwise & Topicwise, ENGLISH LANG. & LIT., Class-X

OBJECTIVE TYPE QUESTIONS









































































































































































Topic-2

Modals Concepts Covered



Modal Auxiliaries - can, could, May, might, will, shall, should, ought to, must have to used to.





Q. 5. Raman always _______ excuses for not doing the work. (A) makes (B) have made (C) make (D) are making Ans. Option (A) is correct. Q. 6. I_______ imagine how you _______ into such a mess. (A) won‘t; got (B) can‘t; got (C) can; get (D) will; get Ans. Option (B) is correct. Q. 7. The patient _______ before the ambulance ______ the hospital. (A) died; had reached (B) died; has reached (C) died; reached (D) had died; reached Ans. Option (D) is correct. Q. 8. I _______ you are new to this place. (A) thinks (B) have thought (C) think (D) thinking Ans. Option (C) is correct. Q. 9. The votes _______ in the Multipurpose Hall. (A) are counting (B) are being counted (C) is counted (D) count Ans. Option (B) is correct. Q. 10. I _______ teaching in this school _______ ten years. (A) have been; since (B) has been; for (C) have been; for (D) had; from Ans. Option (C) is correct.





Q. 1. Choose the option that completes the sentence below correctly: She had already begun eating her dinner by the time I ____ there. (A) reached (B) had reached (C) was reaching (D) had been reaching [CBSE QB, 2021] Ans. Option (A) is correct. Q. 2. In which of these does the underlined phrase indicate that the subject has completed the action in the very recent past? (A) Kelly and I would study together each evening. (B) Sarah has been playing in the rain since 2 o'clock. (C) Jack has informed her that he would not be available. (D) The plane had left by the time Mandy reached the airport. [CBSE QB, 2021] Ans. Option (C) is correct. Q. 3. The train _______ Delhi by now. (A) have reached (B) had reached (C) must have reached (D) reached Ans. Option (C) is correct. Q. 4. She _______ the Student of the Year award last year. (A) win (B) won (C) has won (D) winning Ans. Option (B) is correct.





FILL IN THE BLANKS GIVEN BELOW BY CHOOSING THE CORRECT OPTION:

Revision Notes









(a) Can …………… Ability, Permission (informal)

(e) Will & Shall …………… To express future time, to express determination, promise and threatening, to make requests (f) Should, Ought to ……… Moral obligation, to express advice (g) Must, have to …………… Social obligation (compulsion), duty, necessity (h) Used to …………… To express past habit.







Modals are the auxiliaries that are used to convey special idea. Modal Verbs Function/Idea conveyed







(c) May ………… Possibility, Permission (Formal)







(b) Could Past of …………… Can









(d) Might Past of …………… May’

OBJECTIVE TYPE QUESTIONS (D) would













(C) can Ans. Option (C) is correct.

Q. 1. Raj has fractured his arm, still he _______ write. (A) may (B) should



FILL IN THE BLANKS GIVEN BELOW BY CHOOSING THE CORRECT OPTION:

@PROCBSE GRAMMAR





















































































































































(CBSE SQP, 2020-21) (A) must (B) will (C) could (D) may Ans. Option (B) is correct. Q. 5. A form _______ filled in order to open an account in a bank. (A) can be (B) would be (C) might be (D) must be Ans. Option (D) is correct. Q. 6. _______ you please give me glass of water?

45

(A) Would (B) Can (C) Must (D) Should Ans. Option (A) is correct. Q. 7. Madhu _______ win an air ticket to U.S.A. (A) dare (B) must (C) need to (D) might Ans. Option (D) is correct. Q. 8. If I were rich, I _______ help the needy. (A) could (B) should (C) would (D) might Ans. Option (C) is correct. Q. 9. All the vehicles _______ stop at a red traffic light. (A) can (B) may (C) might (D) must Ans. Option (D) is correct. Q. 10. The court warned the representative that he _______ tamper with the evidence. (A) need not (B) should not (C) could not (D) would not Ans. Option (B) is correct.



Q. 2. According to your doctor’s advice, you _______ not take any sugar. (A) can (B) must (C) shall (D) dare Ans. Option (B) is correct. Q. 3. Thick clouds have gathered in the sky. It _______ rain this evening. (A) may (B) could (C) should (D) need to Ans. Option (A) is correct. Q. 4. The Komodo dragon _______ follow its prey till it eventually dies due to its venomous bite.





Topic-3

Subject-verb Agreement Concepts Covered  The relationship between the subject and verb; The subject and verb must agree in two ways; Tense and Number.







5.





6.





7.





8.







9.

















































the subject should be placed near the verb;  John or his brothers are to blame. If two subjects joined by, ‘or, nor’ are of different persons, the verb agrees in person with the subject nearest to it;  Either you or he is telling a lie.; If two nouns are joined by ‘with or as well as’ the verb agrees with the first noun, i.e. if the first noun is singular, the verb must be singular, even if the second noun is plural;  Raman as well as his friend has won the prize. A collective noun takes a singular or in plural verb according to the sense;  The jury [=men of the jury] were divided in their opinions.  The jury [=one body] has elected its leader. ‘Either, neither, each, every one, many a’ must be followed by a verb in the singular;  Either of the two candidates is suitable.  Each one of these men is reliable. When a plural noun denotes some specific quantity or amount considered as a whole, the verb is generally singular;  One lakh rupees is a good sum (not are) Similarly, with titles of books and names of sciences  The Arabian Nights is an interesting book.



4.



A verb must agree with its subject in number and person, i.e., the verb should be of the same number and person as the subject.  Mohan likes sweets.  Children like sweets. 1. Two or more singular subjects joined by ‘and’ take a plural verb;  He and his brother were absent.  Note:  When two subjects are regarded as representing one idea, then the verb is singular;  Slow and steady wins the race.  If two singular nouns refer to the same person or thing, the verb must be singular;  The poet and philosopher is dead. [Here ‘poet’ and ‘philosopher’ refer to the same person.] 2. If two singular nouns joined by ‘and’ are preceded by ‘each’ or ‘every’, the verb is singular;  Every man and every woman in the village was terrified. 3. Two or more singular subjects connected by ‘or, nor’, either .............. or, neither .............. nor, take a singular verb;  Either Ashok or Ranjit is the culprit.  Note: when one of the nouns or pronouns joined by ‘or, nor’ is in the singular and the other on the plural, 































Revision Notes

@PROCBSE



[Here the verb will agree with ‘behaviour’ not children]  The quality of the mangoes was (not were good. Some sentences begin with ‘there’ or ‘here’. In these sentences the subject comes after the verb and the verb agrees with the subject;.  Here is a rabbit.  There is one girl behind the door.









12.













11.





A relative pronoun always agrees in number and person with its antecedent;  I am a woman who works (not work) for children’s welfare. Errors of Proximity (nearness): Often the verb is made to agree in number with a noun near to it instead of its proper subject. This should be avoided:  The behaviour of the children was excellent

10.



Oswaal CBSE Question Bank Chapterwise & Topicwise, ENGLISH LANG. & LIT., Class-X





46

OBJECTIVE TYPE QUESTIONS













































































































































































Topic-4

Q. 6. The manger and the orator _______ been invited to the function. (A) was (B) has (C) were (D) have Ans. Option (D) is correct. Q. 7. No scholarship or reward _______ given to the student who stood first in the examination. (A) are (B) were (C) was (D) had Ans. Option (C) is correct. Q. 8. Bread and butter _______ what the poor want. (A) are (B) has (C) is (D) has been Ans. Option (C) is correct. Q. 9. Everybody _______ keen to participate in the upcoming nukkad natak. (CBSE SQP, 2020-21) (A) are (B) has (C) is (D) were Ans. Option (C) is correct. Q. 10. Many of my friends _______ motorbikes. (A) ride (B) riding (C) rides (D) has ride Ans. Option (A) is correct.



Q. 1. Mathematics _______ a compulsory subject for under-graduation in most of the science and commerce streams. (A) are (B) were (C) is (D) has Ans. Option (C) is correct. Q. 2. One of my books _______ missing from my bag. (A) is (B) has (C) are (D) have Ans. Option (A) is correct. Q. 3. Oil and water _______ not mix. (A) does (B) had (C) do (D) is Ans. Option (C) is correct. Q. 4. Slow and steady _______ the race. (A) win (B) wins (C) winning (D) had win Ans. Option (B) is correct. Q. 5. The little boy who is wearing brown shoes _______ my cousin. (A) is (B) am (C) are (D) was Ans. Option (A) is correct.



FILL IN THE BLANKS GIVEN BELOW BY CHOOSING THE CORRECT OPTION:

Determiners Concepts Covered  Types of determiners: 1. Articles; (i) indefinite Article; (ii) Definite Article; 2. Demonstratives.; 3. Possessives; 4. Quantifiers; 5. Others;















(d) Adjectives (Quantity and Number) : some, any, much, many, all, both, little, few,several, less, one, two, etc. (e) Others : each, every, next, another, either or neither ....... nor, first, second, etc.







What are determiners? Determiners are the words which are used before nouns to determine or fix their meaning. Types : (a) Articles : a, an and the (b) Demonstrative adjectives : this, that, these, those (c) Possessives : my, our, your, his, her, its, their



Revision Notes

@PROCBSE

GRAMMAR

47

Articles

Indefinite (a/an)

Definite (The)

An - used before singular countable nouns beginning with vowel sound. [Example- an Indian, an hour, an umbrella, an honest boy (mute _ ‘h‘) (ii) Definite article (the) – used before both countable and uncountable nouns. Usage : 1. When we talk about something for the second time in the same context or anything qualified by a phrase.

































(i) Indefinite articles (a/an) 1. Used before singular countable nouns. 2. Represents a class or kind in general. e.g., - A cow gives us milk. ↓ (Every Cow) A - used before singular countable nouns beginning with consonant sound. [Example- a one-eyed man, a unique place, a European, a university, a mango, etc.



(a)

(i) I met a girl at the gate of the school. The girl was weeping. (Second time reference) (First time reference) (ii) The book which is on the table is mine. (Phrase)

Used before the names of scriptures. e.g., The Ramayan, The Quran, The Guru Granth Sahib, The Bible. Used before the names of newspapers, magazines. e.g., The Times of India, The Competition Success Review, The Reader‘s Digest. Used before an adjective when the noun is understood. e.g., The poor, The rich (The rich become richer, the poor become poorer). When a proper noun used as a common noun. Kalidas is the Shakespeare of India. (a great dramatist) Demonstrative adjectives This- to demonstrate nearby things That- to demonstrate far off things These- plural of ‘this‘ Those- plural of ‘that‘ Possessives These are used to show belongingness/ ownership. e.g., This is my book. That is his book.









11. (a)









10.









9.









8.













(b)



l



l







(c)











7.































2. Used before superlatives She is the tallest girl of the class. 3. Used before the names of water bodies i.e. - rivers, seas, oceans The Ganges, The Arabian Sea, The Pacific Ocean, etc. Note : not used before the names of lakes. e.g., Dal Lake. 4. Used before the names of mountain ranges. e.g. The Himalayas. Note : not used before the names of peaks. e.g., Mount Everest. 5. Used before the names of satellites, planets, stars. e.g., The moon, the earth, the sun. 6. Used before the names of monuments and memorials. e.g., The Red Fort, The Taj Mahal. Used before the names of states/countries that have a common noun in their name. e.g., the USA, the UAE, the UNO, the UK.

OBJECTIVE TYPE QUESTIONS



These questions are for practice and their solutions are available at the end of the chapter

























(D) each; Many; those (CBSE SQP, 2020-21)

Ans. Option (B) is correct. Q. 2. ___________ fruit seller does not earn much during the rainy season. (A) Am (B) The (C) A (D) Many









(C) every; Few; those



Q. 1. My work requires me to spread awareness about the kinds of bullying prevalent in a student's life. Did you know that one out of __________ five students is bullied by another youth? __________ students are scared of opening up to others about instances of bullying __________ may be happening to or around them. (A) each; Few; that (B) every; Many; that



FILL IN THE BLANKS GIVEN BELOW BY CHOOSING THE CORRECT OPTION:

@PROCBSE Oswaal CBSE Question Bank Chapterwise & Topicwise, ENGLISH LANG. & LIT., Class-X

























































































































































Topic-5

(A) a; an (B) an; the (C) the; a (D) an; a Ans. Option (A) is correct. Q. 8. There are ______ bananas in the fruit basket, but there are ______ mangoes. (A) some; any (B) some; no (C) any; some (D) all; many Q. 9. Harish sent his story to ___________ magazines for publishing. (A) much (B) any (C) many (D) no Ans. Option (C) is correct. Q. 10. How many boys are there in ___________ class? (A) a (B) your (C) both (D) all Ans. Option (B) is correct. Q. 11. You can’t have ___________ mangoes as they are rotten. (A) this (B) that (C) these (D) your Ans. Option (C) is correct.



Q. 3. He drove with ___________ care and hence was fined by the traffic police. (A) many (B) less (C) much (D) little Ans. Option (D) is correct. Q. 4. I spent ___________ money I had. (A) a little (B) few (C) the little (D) little Ans. Option (C) is correct. Q. 5. You may have ___________ of the four watches. (A) some (B) any (C) no (D) both Ans. Option (B) is correct. Q. 6. The good news is that _________ volunteers dropped out this month than the last two. (CBSE SQP, 2020-21) (A) fewer (B) less (C) few (D) a little Ans. Option (A) is correct. Q. 7. It was _______ historic day for the organisation when _____ honour was bestowed upon its employees.



48

Reported Speech Concepts Covered  Transformation rules from direct speech to indirect speech while reporting: (i) statements; (ii) Commands & requests; (iii) Questions









(b) We have removed the inverted commas of the reported speech. (c) The pronoun ‘I’ of the reported speech has been changed to ‘he’. (d) The verb am is changed to was. (e) The adverb tomorrow has been changed to the next day. Rules for changing Direct Speech into Indirect 1. When the reporting or principle verb is in the Past Tense, all Present tense verbs of the Direct are changed into the corresponding Past Tense verbs. Thus, (a) A simple present becomes a simple past. Direct : He said, “ I am quite well.” Indirect : He said that he was quite well. (b) A present continuous becomes a past continuous. Direct : She said, “My brother is writing a letter.” Indirect : She said that her brother was writing a letter. (c) A present perfect becomes a past perfect. Direct : Lakshmi said, “I have passed the examination.” Indirect : Lakshmi said that she had passed the examination.



l



l



l





























There are two ways of reporting a speech whether ours or someone else’s. (i) We may quote the actual words, that is, to report the speech exactly as we had heard or said it without making any change. This is called Direct Speech. (ii) We may report what is said, without quoting the exact words or changing their meaning. This is called Indirect (or Reported) Speech. Examples : Direct — His friend said, “I am going tomorrow.” Indirect — His friend said that he was going the next day. It will be seen that in the first example the first part of the sentence which is before the comma, is referred to as reporting verb and the part which is under inverted commas is called the reported speech. It will be further noticed that in changing the above Direct Speech into Indirect, certain changes have been made. Thus, (a) We have removed the comma in the indirect sentence and put the conjunction that in its place.

























Revision Notes

@PROCBSE

l



l



l



l



l



l











l























l l



l



l







l



l







49

night becomes that Direct : He said, “I am glad to be here this evening.” Indirect : He said that he was glad to be there that evening. The changes do not occur if the speech is reported during the same period or at the place; e.g., Direct : He says, “I am glad to be here this evening.” Indirect : He says that he is glad to be here this evening. 4. Change of person : The persons of the pronouns and the verbs in the reported speech undergo changes when converted into indirect form. Thus, (a) First person pronouns in the direct speech change according to the subject of the reporting verb. Direct : He says, “I am going to Delhi.” Indirect : He says that he is going to Delhi. (b) Second person pronouns change according to the noun or pronoun coming after the reporting verb. Direct : I said to Mohan, “You will have to read.” Indirect : I said to Mohan that he would have to read. (c) Third person pronouns of the direct speech remain unchanged when converted to indirect. Direct : Rita said to me, “He will be defeated.” Indirect : Rita said to me that he would be defeated. 5. Questions : In reporting questions the indirect speech is introduced by verbs such as, asked, inquired, etc. When the question is not introduced by an interrogative word, the reporting verb is followed by whether or if. Direct : He said to me, “What are you doing”? Indirect : He asked me what I was doing. Direct : “Where do you live?”, asked the stranger. Indirect : The stranger enquired where I lived. Direct : He said, “Will you listen to such a man?” Indirect : He asked them whether they would listen to such a man. This







(d) The will/shall of the Future Tense is changed into would/should Direct : The girl said to her friend, “I will write a letter.” Indirect : The girl said to her friend that she would write a letter. (e) The simple past in the Direct becomes the Past perfect in the Indirect. Direct : Doctor said, “The patient died in the night.” Indirect : Doctor said that the patient had died in the night. 2. If the reporting verb i.e. the main verb in the reporting speech is in the Present or the Future Tense, the tense of the verbs in the reported speech will not change. Direct : The girl says, “I may not sing today.” Indirect : The girl says that she may not sing that day. Direct : Rahim will say, “I have not done my work.” Indirect : Rahim will say that he has not done his work. Exception : 1. If the reported speech contains some universal truth or habitual fact, the tense of the verb does not change even if the reporting verb is in the past tense. Direct : The teacher said to the students, “The earth is round.” Indirect : The teacher said to the students that the earth is round. 2. When two actions occur at the same time in the past, no change in the tense of the verb in the reported speech is made. Direct : He said, “I cried out when I saw the thief coming.” Indirect : He said that he cried out when he saw the thief coming. 3. Words expressing nearness in time or place are generally changed into words expressing distance. Thus:Today becomes that day Tomorrow becomes the next day Yesterday becomes the previous day /the day before Now becomes then Here becomes there Ago becomes before Last night becomes the previous



GRAMMAR

l







l



In reporting commands and requests, the Indirect Speech is introduced by some verb expressing command or request, and the imperative mood is changed into Infinitive. Direct : Ganguly said to Mahmood. “Go l







6. Commands and Requests : away.” Indirect : Ganguly ordered Mahmood to go away. Direct : The Principal said to the visitor,

@PROCBSE 50



l







l









l



l l







l



l



l

question or request. When transforming statements, check whether you have to change: pronouns present tense verbs (3rd person singular) place and time expressions tenses (backshift)

l





If we report what another person has said, we usually do not use the speaker’s exact words (direct speech), but reported (indirect) speech. Therefore, you need to learn how to transform direct speech into reported speech. The structure is a little different depending on whether you want to transform a statement,



Statements





7.

Indirect : He told me that there was no more water. Direct : He said to me, “You are getting lazy.” Indirect : He told me that I was getting lazy. ‘Said to’ is not always changed into ‘told’. It can also be changed into replied, stated, informed, remarked, etc., depending on the nature of the sentence. Must and need not : (a) In some cases these two are used in place of have to. Direct : He said, “I must go now.” Indirect : He said that he had to go then. Direct : He said, “I need not go.” Indirect : He said that he would not have to go. (b) ‘Must’ is used in place of ‘should/would, have to’ when it expresses necessity or compulsion. Direct : He said, “I must finish this book on Monday.” Indirect : He said that he would have to finish that book on Monday. Note : But in some sentences when ‘must’ indicates some kind of advice or suggestion, in those sentences ‘must’ remains ‘must’. l





9.

l











8.























7.











l







“Please wait here till I return.” Indirect : The Principal requested the visitor to wait there till he returned. Direct : He said to the students, “Don’t make a noise.” Indirect : He forbade the students to make noise. Exclamation and Wishes : In reporting exclamations and wishes, the indirect speech is introduced by some verb expressing exclamation or wish. Direct : He said, “Alas! I am ruined.” Indirect : He exclaimed sadly that he was ruined. Direct : He said, “Bravo! You have done well.” Indirect : He applauded him saying that he had done well. Said to : If the reporting verb ‘said to’ is followed by an object, it is changed into ‘told’ while converting it into indirect speech. Direct : He said to me, “ There is no more water.”



Oswaal CBSE Question Bank Chapterwise & Topicwise, ENGLISH LANG. & LIT., Class-X

Type Direct speech

“I speak English.”

Reported speech (no backshift)

He says that he speaks English.

Reported speech (backshift)

He says that he spoke English.





l

Also note that you have to:



When transforming questions, check whether you have to change: pronouns present tense verbs (3rd person singular) place and time expressions tenses (backshift)



Questions





transform the question into an indirect question use the interrogative or if / whether





l





l

l



l



l







8.

Example

Example

Type With interrogative Without interrogative

Direct speech

“Why don’t you speak English?”

Reported speech He asked me why I didn’t speak English. Direct speech

“Do you speak English?”

Reported speech He asked me whether/ if I spoke English.

These questions are for practice and their solutions are available at the end of the chapter

@PROCBSE

GRAMMAR

51

OBJECTIVE TYPE QUESTIONS Reported Speech



















































































































dinner today. (B) My brother told me that he would be late for dinner that day. (C) My brother said that she would be late for dinner that day. (D) My brother told me that he will be late for dinner today. Ans. Option (B) is correct. Q. 5. Rohit said to Mala, “You can do it.” (A) Rohit told Mala that he could do it. (B) Rohit told Mala that she can do it. (C) Rohit told Mala that she could do it. (D) Rohit told Mala that you can do that. Ans. Option (C) is correct. Q. 6. The peon said, “I rang the bell on time.” (A) The peon said that he had rung the bell on time. (B) The peon told that he rang the bell on time. (C) The peon said that I rung the bell on time. (D) The peon said that he rang the bell on time. Ans. Option (A) is correct. Q. 7. The teacher said to the students, “The peacock is a beautiful bird.” (A) The teacher told the students that the peacock was a beautiful bird. (B) The teacher said that the peacock was a beautiful bird. (C) The teacher told the students that the peacock is a beautiful bird. (D) The teacher told the students that the peacock had been a beautiful bird. Ans. Option (C) is correct. Q. 8. I said, “I have completed my work.” (A) I said that I have completed my work. (B) I said that I have complete your work. (C) I said that I had completed my work. (D) I told that I have completed my work.



Q. 1. Read the following dialogue between Kashika and Sam. The underlined parts of this dialogue may or may not have an error. Choose the option that best replaces the underlined part(s). You may choose 'no error' if no change is required in the underlined parts. Kashika: Sam, will we take a look at the rules of the poetry competition conveyed by Ms Natasha last Tuesday? We have very little time to prepare. Sam: You're right! I completely forgot about it. We should not delay it any longer. (A) no error; shall not (B) shall we; no error (C) may we; may not (D) can we; could not (CBSE SQP, 2020-21) Ans. Option (B) is correct. Q. 2. Given below is a statement made by politician Jason Bailey: "I don't believe in giving second chances to these socalled allies who went behind my back." Imagine you are a newspaper reporter. While reporting this statement, you will NOT __________. (A) change 'my' to 'his' (B) change 'went' to 'gone' (C) change 'don't' to 'didn't' (D) change 'these' to 'those' (CBSE SQP, 2020-21) Ans. Option (B) is correct. Q. 3. Mohini said, “I have decided to rent the shop.” (A) Mohini said that she has decided to rent the shop. (B) Mohini said that I had decided to rent the shop. (C) Mohini said that she had decided to rent the shop. (D) Mohini said that she decided to rent the shop. Q. 4. My brother said to me, “I will be late for dinner today.” (A) My brother said to me that I would be late for







FILL IN THE BLANKS GIVEN BELOW BY CHOOSING THE CORRECT OPTION:

Commands and Requests

These questions are for practice and their solutions are available at the end of the chapter





























































(iii) ordered (iv) warned (A) (i) & (ii) (B) (i) & (iii) (C) (ii) & (iv) (D) (ii) & (iii) Ans. Option (D) is correct. Q. 3. Sonali _______ her mother to allow her to go out with her friends. (A) ordered (B) asked (C) requested (D) told

Q. 1. Sunil said to his brother, “Please give me a glass of water.” Sunil _____ his brother to give him a glass of water. (A) request (B) ordered (C) requested (D) advised Q. 2. The officer ________ the soldiers to move ahead. (i) requested (ii) commanded



FILL IN THE BLANKS GIVEN BELOW BY CHOOSING THE CORRECT OPTION:

@PROCBSE Oswaal CBSE Question Bank Chapterwise & Topicwise, ENGLISH LANG. & LIT., Class-X











































(B) The principal commanded the peon to ring the bell. (C) The principal ordered the peon to ring the bell. (D) The principal commanded the peon to bring the papers. Ans. Option (A) is correct. Q. 6. Which of the following is a request? (A) Sit down and do your work. (B) Please open the window. (C) Do not disturb me. (D) Give up smoking. Ans. Option (B) is correct.





Ans. Option (C) is correct. Q. 4. The teacher said to the students, “Do not make a noise.” The teacher ________ the students ______ make a noise. (A) advised; do not (B) ordered; not to (C) requested; not to (D) ordered; do not Ans. Option (B) is correct. Q. 5. Which of the following sentences is INCORRECT? (A) The principal requested the peon to ring the bell.







52

Statements









































































(C) Rajeev told that he was not well today. (D) Rajeev said that he has not well that day. Ans. Option (A) is correct. Q. 4. He said, “I live in Model Town.” (A) He said that I live in Model Town. (B) He said that he lived in Model Town. (C) He said that he lives in Model Town. (D) He said that I live in Model Town. Q. 5. Mohit said, “I have decided to change my job.” (A) Mohit said that I have decided to change my job. (B) Mohit said that he has decided to change his job. (C) Mohit said that he had decided to change my job. (D) Mohit said that he had decided to change his job. Ans. Option (D) is correct.

Q. 1. Ridhima said, “I do not like examinations.” (A) Ridhima said that she do not like examinations. (B) Ridhima said that she does not like examinations. (C) Ridhima said that, do not liked examinations. (D) Ridhima said that she did not like examinations. Ans. Option (D) is correct. Q. 2. I said to my friend, “I missed my school bus.” (A) I told my friend that I missed your school bus. (B) I told my friend that I had missed my school bus. (C) I said to my friend that I missed my school bus. (D) I said to my friend that I have missed my school bus. Ans. Option (B) is correct. Q. 3. Rajeev said, “I am not well today.” (A) Rajeev said that he was not well that day. (B) Rajeev said that he is not well today.



CHOOSE THE CORRECT OPTION:

Questions 





























These questions are for practice and their solutions are available at the end of the chapter





















(D) All I, II and III are correct.(CBSE SQP, 2020-21) Ans. Option (A) is correct. Q. 2. The man said to the beggar, “What do you want?” (A) The man asked the beggar what did he want. (B) The man said to the beggar what he wanted. (C) The man asked the beggar what he did want. (D) The man asked the beggar what he wanted. Ans. Option (D) is correct. Q. 3. My friend said to me, “Can you help me?” (A) My friend asked me if I could help her. (B) My friend asked me can I help her. (C) My friend asked me if I can help her. (D) My friend asked if you can help me. Q. 4. The doctor said, “ Do you like sweets?” (A) The doctor asked do you like sweets.









Q. 1. Gina was asked to convert sentence A to reported speech. Given below are three options that she came up with. Sentence A: Larissa said, "Why don't you join me for dinner tonight?" I. Larissa suggested I join her for dinner that night. II. Larissa asked me to join her for dinner that night. III. Larissa insisted that I should join her for dinner that night. Which of the following statements about her options is TRUE? (A) Only I is correct. (B) Only III is correct. (C) Only II and III are correct.





CHOOSE THE CORRECT INDIRECT SPEECH FROM THE GIVEN OPTIONS:

@PROCBSE

GRAMMAR











































Solutions for Practice Questions (Topic-5)

These questions are for practice and their solutions are available at the end of the chapter











Statements 4. Option (B) is correct. Questions 3. Option (A) is correct. 7. Option (D) is correct.











Reported Speech 3. Option (C) is correct. 8. Option (C) is correct. Commands and Requests 1. Option (C) is correct.









2. Option (C) is correct. 8. Option (B) is correct.





Solutions for Practice Questions (Topic-4)





(B) He asked whether anyone is inside the house. (C) He asked whether no one is inside the house. (D) He said that anyone is inside the house. Ans. Option (A) is correct. Q. 7. Kiran said, “How are you?” (A) Kiran said how I am. (B) Kiran asked how are you. (C) Kiran asked how am I. (D) Kiran asked how I was. Q. 8. The man said, “When did the train leave?” (A) The man enquired when the train left. (B) The man wanted to know when the train had left. (C) The man asked when the train leave. (D) The man asked when did the train leave. Ans. Option (B) is correct.

















(B) The doctor asked if he liked sweets. (C) The doctor enquired if he did liked sweets. (D) The doctor said that he liked sweets. Ans. Option (B) is correct. Q. 5. The students said to the teacher, “When will the vacations begin?” (A) The students asked the teacher when the vacation will begin. (B) The students asked the teacher that when will the vacations begin. (C) The students asked the teacher when the vacation would begin. (D) The students asked the teacher when would vacation begin. Ans. Option (C) is correct. Q. 6. He said, “ Is anyone inside the house?” (A) He asked if anyone was inside the house.

53

@PROCBSE 54

Oswaal CBSE Question Bank Chapterwise & Topicwise, ENGLISH LANG. & LIT., Class-X

@PROCBSE

GRAMMAR

55

@PROCBSE 56

Oswaal CBSE Question Bank Chapterwise & Topicwise, ENGLISH LANG. & LIT., Class-X

@PROCBSE

GRAMMAR

57

@PROCBSE 58

Oswaal CBSE Question Bank Chapterwise & Topicwise, ENGLISH LANG. & LIT., Class-X

@PROCBSE

GRAMMAR

59

@PROCBSE 60

Oswaal CBSE Question Bank Chapterwise & Topicwise, ENGLISH LANG. & LIT., Class-X

@PROCBSE

CHAPTER

3

GAP FILLING

Revision Notes Introduction

Gap filling / Cloze passage is an exercise in which some words are missing from the text and they. are to be filled. The words and to be filled are generally items of grammar like preposition, conjunction, verb forms etc. the word / words filled should be filled in such a way that the entire paragraph makes complete sense and is grammatically sound.

Tips to remember



Read the paragraph carefully before attempting to fill in the blanks. Make sure that the verb agrees with the subjects.. number and person as well as the aspect of the tense tense.

OBJECTIVE TYPE QUESTIONS I.



Fill in any four of the following blanks given below choosing the most appropriate options from the ones that follow. Write the answers in your answer-sheet against the correct blank numbers. [CBSE Delhi/Outside Delhi Set I, 2020]







Communication 1. become very effective 2. instant due to smart phones. People are able 3. convey their messages all around the globe to 4. loved ones 5. spending hefty sums of money. [1 mark each] (A) is

(B)

has

(C)

have

(D)

had

2.

(A) but

(B)

as

(C)

or

(D)

and

3.

(A) for

(B)

in

(C)

to

(D)

of

4.

(A) his

(B)

her

(C)

their

(D)

your

5.

(A) with

(B)

without

(C)

and

(D)

to

(C) to

(D) and

4.

(C) their



2.



(B) has



3.





5. (B) without [CBSE Marking Scheme, 2020] Explanation: 1. The Conjunction ‘and’ is used to define that communication has become effective as well as instant. 2. The Preposition ‘to’ is used to define an activity that is to be done. 3. ‘Their’ is used in Plural to define the belongingness of Plural ‘people’. 4. ‘Without’ is used when part of the sentence defines work that has been done and the other part of the connected work has not been done. 







Ans. 1.



1.

@PROCBSE 62

Oswaal CBSE Question Bank Chapterwise & Topicwise, ENGLISH LANG. & LIT., Class-X

II. Fill in the blanks in the paragraph given below by choosing the most appropriate options from the ones that follow. Write the answers in your answer-sheet against the correct blank numbers. [CBSE Outside Delhi Set I, 2020] 2.

Vast New World to us. They increase





Reading books 1. a good hobby. Books open and change our outlook 4. the World. 1.

(A)

has

(B)

was

(C)

are

(D)

is

2.

(A)

the

(B)

a

(C)

an

(D)

some

3.

(A)

his

(B)

one's

(C)

our

(D)

your

4.

(A)

towards

(B)

for

(C)

by

(D)

from

2.

knowledge [1 mark each]

(B) a



(D) is







Ans. 1.

3.













3. (C) our 4. (A) towards [CBSE Marking Scheme, 2020] Explanation: 1. ‘Reading’ here is an activity, a noun shown in simple present. So, 'is' is the correct verb to be used. 2. The world of books is referred to a new thing but it is not unique. So, the article ‘a’ is used for without vowel sound word. 3. The later use of ‘our;’ in the sentence indicates similar use of ‘our’ here. 4. ‘Towards’ here is used to indicate ‘in the direction of ’.

III. Choose the correct options to fill in the blanks to complete the note about the Wangala Festival of Meghalaya. [CBSE SQP, 2020-21]





The Wangala 1. festival for the Garo in Meghalaya, Assam and Nagaland. It is a post harvest festival 2. the end of the Agricultural Year. It is popularly known as ‘The Hundred Drums’ Festival. During the Signature dance, the Leading Warrior 3. with Synchronised dance 4. and specific Hand-head Movements. [1 mark each] 1.

(A)

is important

(B) are an important (C)

was the important (D)

is an important

2.

(A)

being celebrated for marking

(B) celebrated to mark

(C)

celebrated to marking

(D)

being celebrated for mark

3.

(A)

leads the youngsters

(B) is lead the youngsters

(C)

was leading the youngsters

(D)

had leads the youngsters

4.

(A)

step

(B) steps

(C)

foot

(D)

feet

2.



(D) is an important







Ans. 1.

(B) celebrated to mark 















3. (A) leads the youngsters 4. (B) steps [CBSE Marking Scheme, 2020] Explanation: 1. Wangala is singular and article ‘an’ is used before ‘important’ (vowel sound). The passage is in present tense, so, ‘is an important’ is the correct option. 2. As it is an Annual event which takes place regularly, so, ‘celebrated to mark’ as simple present and gerund is correct option. 3. Singular lead warrior – so ‘leads’ is used, Simple present tense as it is a statement of fact. 4. ‘Dance’ is a synchronisation of the ‘steps’ taken. In dance, even hand and facial movements are referred to as ‘steps’.







IV. Choose the correct options to fill in the blanks to complete the note about Delhi, the Capital of India. [CBSE Delhi Set I, 2019 (Modified)] Delhi 1. the Capital of India. People from all parts 2. the Country and World come to visit Delhi. There 3. many Historical Buildings here. Last year I, 4. Delhi. I also watched 5. Commonwealth Games 2010. [1 mark each] 1.

(A) was

(B) is

(C) are

(D)

being

2.

(A) of

(B) from

(C) for

(D)

at

3.

(A) being

(B) are

(C) been

(D)

were

4.

(A) visit

(B) visiting

(C) visited

(D)

will visit

5.

(A) a

(B) an

(C) the

(D)

some

@PROCBSE





2. 4.

63

(A) of (C) visited (Any Four) [CBSE Marking Scheme, 2019]













Ans. 1. (B) is 3. (B) are 5. (C) the Explanation:



GRAMMAR









1. Delhi is singular and is a statement of fact. So, simple present tense verb – ‘is’ – is the correct option. 2. Preposition 'of' tells the relationship between the Country and its parts. 3. Buildings is plural and is a statement of fact. So, simple present tense verb – ‘are’ – is the correct option. 4. Last year indicates, it is past tense, so ‘visited’ is the correct option. Fill in any four of the following blanks choosing the most appropriate options from the ones given below. Write the answers in your answer sheet against the correct blank numbers. [CBSE Delhi Set II, 2019 (Modified)] Birds and animals live in the lap 1. nature and can predict the likely changes accurately. Swallows usually fly high 2. the sky. But during a Storm, they come down 3. fly close to the ground. If they fly low you 4. be sure of strong winds. Even a Toad can be 5. reliable Weatherman. [1 mark each] 









V.

(A)

in

(B) of

(C) on

(D)

for

2. 3. 4. 5.

(A) (A) (A) (A)

in nor would a

(B) (B) (B) (B)

(C) (C) (C) (C)

(D) (D) (D) (D)

to and could some





2. 4.

of or should an

on but might the

(A) in (C) could (Any Four) [CBSE Marking Scheme, 2019]













Ans. 1. (B) of 3. (B) and 5. (C) a Explanation:



1.













1. Preposition ‘of ’ defines the relationship between lap and nature – that the lap belongs to the nature. 3. Conjunction ‘and’ is used to state that two events – ‘come down’ as well as ‘fly close to the ground’ happen simultaneously. 4. It is a general fact about swallows, so, ‘could’ is the correct choice. 5. Singular form, not unique, no vowel sound, so, use of article ‘a’ is correct. VI. Fill in any four of the following blanks choosing the most appropriate options from the ones given below. Write the answers in your answer sheet against the correct blank numbers. [CBSE Delhi Set III, 2019 (Modified)] 2.

7th December. The plane

3.

reach Madras

4.

11.00 am. He [1 mark each]





He will fly 1. Madras 5. stay at Hotel Plaza. (A)

in

(B)

on

(C)

to

(D)

after

2.

(A)

on

(B)

in

(C)

for

(D)

from

3.

(A)

may

(B)

can

(C)

will

(D)

could

4.

(A)

for

(B)

at

(C)

on

(D)

by

5.

(A)

could

(B)

will

(C)

shall

(D)

must





2. 4.

(A) on (B) at (Any Four) [CBSE Marking Scheme, 2019]









1. 2. 3. 4.









Ans. 1. (C) to 3. (C) will 5. (B) will Explanation:



1.

Preposition ‘to’ is used to define destination. Preposition ‘on’ is used to express date. As the event is scheduled for future, so, simple future tense verb – ‘will’ – is the correct option. Preposition ‘at’ is used to express time.



VII. Fill in the blanks in the paragraph given below by choosing the most appropriate options from the ones that follow. Attempt any four. Write the answers in your answer sheet against the correct blank numbers. The Caves of Ajanta and Ellora 3. spellbound. Thousands Houses. 





[CBSE Outside Delhi Set I, 2019 (Modified)] 1. 4.

the Magnificent Works of Sculpture. Whoever Tourists visit these places every year. One can stay

2.

5.

there the Guest [1 mark each]

@PROCBSE 64

Oswaal CBSE Question Bank Chapterwise & Topicwise, ENGLISH LANG. & LIT., Class-X

1.

(A)

is

(B) are

(C) was

(D)

were

2.

(A)

go

(B) going

(C) goes

(D)

went

3.

(A)

becomes

(B) become

(C) becoming

(D)

has become

4.

(A)

for

(B) of

(C) from

(D)

at

5.

(A)

on

(B) over

(C) upon

(D)

in



(C) goes (B) of



2. 4.



(B) are (A) becomes







Ans. 1. 3.

(Any Four) [CBSE Marking Scheme, 2019]







5. (D) in Explanation:







1. Caves are in plural and is a statement of fact, so simple present tense verb – ‘are’ – is the correct option. 2. Whoever refers to each individual that goes – singular – and simple present tense fact – so, ‘goes’ is correct choice. 3. Whoever refers to each individual that goes – singular – and simple present tense fact – so, ‘becomes’ is correct choice.

5. Relation between the person and nature of stay at Guest House is expressed using the preposition ‘in’.

VIII. Fill in the blanks in the paragraph given below by choosing the most appropriate options from the ones that follow. Attempt any four. Write the answers in your answer sheet against the correct blank numbers. [CBSE Outside Delhi Set II, 2019 (Modified)]





Television 1. become very popular in our Country. People spend a lot 2. time watching a variety of Programmes. Some of the Programmes 3. are telecast these days are of a very poor quality. We should remember that television is 4. effective in Educating 5. Nation. [1 mark each] 1.

(A)

is

(B)

was

(C)

has

(D)

had

2.

(A)

of

(B)

for

(C)

at

(D)

by

3.

(A)

those

(B)

when

(C)

here

(D)

that

4.

(A)

so

(B)

many

(C)

very

(D)

some

5.

(A)

a

(B)

an

(C)

the

(D)

some

4.

(A) of



2.

(D) that



(C) has

(C) very







3.







Ans. 1.





(Any Four) [CBSE Marking Scheme, 2019]







IX.



2. 3. 4. 5.





5. (A) a Explanation:

Preposition ‘of ’ is used as a phrasal adjective with ‘a lot’. ‘That’ can be used to refer to Programmes being telecasted. ‘Very’ can be used with ‘effective’ to show that it is more than effective than what is thought about in general. Nation is singular and no particular nation is referred to. So, article ‘a’ is the correct choice.

Fill in the blanks in the paragraph given below by choosing the most appropriate options from the ones that follows. Attempt any four. Write the answers in your answer sheet against the correct blank numbers. Belgium is considered to 1. a Country 2. Chocolates. It is a big Business 3. Employs nearly 72,000 people. Most Belgian Chocolate Makers 4. their Craft at Vocational Schools. Then they set up 5. Own Business. [1 mark each] 



[CBSE Outside Delhi Set III, 2019 (Modified)]

1.

(A)

been

(B) be

(C) as

(D)

for

2.

(A)

of

(B) for

(C) from

(D)

to

3.

(A)

also

(B) or

(C) but

(D)

and

4.

(A)

are learning

(B) have learnt

(C) will learn

(D)

learn

5.

(A)

that

(B) those

(C) their

(D)

our

@PROCBSE 65

GRAMMAR

(C) their



(D) learn (Any Four) [CBSE Marking Scheme, 2019]









5.

(A) of



4.



2.

(D) and



(B) be

3.



Ans. 1.

Explanation:





Fill in the blanks in the paragraph given below by choosing the most appropriate options from the ones that follow. Attempt any four. Write the answers in your answer sheet against the correct blank numbers. [CBSE Delhi, 2019] Karthik has penned 1. book Titled, ‘The Ashes of the Prey’, a thriller 4. a lot 5. trouble after an accident.

2.

a lawyer





X.

Preposition ‘of ’ is used when something is deemed to be made up of. Conjunction ‘and’ is used to connect two or more sentences which are both positive and correct. Simple statement of fact makes its use as plural simple present tense, so, ‘learn’ is the correct choice. ‘Their’ is used to define the relationship between chocolate makers and the businesses started by them.



2. 3. 4. 5.

(D)

one

2.

(A) about

(B) with

(C) as

(D)

or

3.

(A) which

(B) whom

(C) who

(D)

whose

4.

(A) on

(B) into

(C) under

(D)

upon

5.

(A) to

(B) from

(C) for

(D)

of

4.

(B)



(A)









(D) of



5.



(C)

2.

(Any Four) [Topper’s Answer, 2019]



3.



(C)



Ans. 1.



(C) a



(B) an



(A) the



1.

3. runs [1 mark each]

Explanation:

XI.







It is a single book and not unique. The word ‘book’ does not start with a vowel sound. So, article ‘a’ is used. To express what is covered in the novel, preposition ‘about’ is the correct choice. Preposition ‘into’ is used as a phrasal verb with ‘runs’. Preposition ‘of ’ is used as a phrasal adjective with ‘a lot’.



1. 2. 4. 5.

Fill in any four of the blanks in the paragraph given below with the help of options that follow : [CBSE Delhi/Outside Delhi Set I, 2018]







The Modern Student 1. the importance 2. physical exercise. He spends one to two hours in open air 3. he takes part in different Sports. However, care should 4. not to overstrain 5. body. [1 mark each] 1.

(A)

understood

(B)

understand

(C)

have understand

(D)

understands

2.

(A)

of

(B)

by

(C)

from

(D)

with

3.

(A)

how

(B)

which

(C)

where

(D)

why

4.

(A)

be taken

(B)

took

(C)

takes

(D)

has taken

5.

(A)

a

(B)

an

(C)

the

(D)

some

(A) be taken

5.

(C) the









(Any Four) [CBSE Marking Scheme, 2018]

Explanation:







1. 2. 3. 5.







(A) of

4.



2.

(C) where



(D) understands

3.



Ans. 1.

Student is singular and is a statement of fact – simple present tense is used. Relationship between importance and exercise is explained using preposition ‘of ’. Adjective of place ‘where’ is used to tell the place at which the modern student takes part in different sports. Every individual has got a unique body, so, article ‘the’ is used here.

@PROCBSE 66

Oswaal CBSE Question Bank Chapterwise & Topicwise, ENGLISH LANG. & LIT., Class-X





XII. Complete the paragraph given below by filling in the blanks with the help of options that follow : [CBSE Delhi Set I, 2017 (Modified)] 1.

2.

wanted a ‘Selfie’

me. So we

3.

4.

on top

a boundary wall. [1 mark each]





I met a smart young person 1.

(A)

which

(B)

who

(C)

whose

(D)

whom

2.

(A)

with

(B)

on

(C)

for

(D)

to

3.

(A)

sit

(B)

sits

(C)

sat

(D)

sitting

4.

(A)

on

(B)

in

(C)

of

(D)

at



XIII. Complete the paragraph given below by filling in the blanks with the help of options that follows : [CBSE Delhi Set II, 2017 (Modified)] 1.

the Family and start living in a

2.

Room

3.





Everyone is not in a position to 4. the Ganga. 1.

(A)

leave

(B)

leaver

(C)

tell

(D)

leaving

2.

(A)

small

(B)

smaller

(C)

smallest

(D)

little

3.

(A)

on

(B)

in

(C)

of

(D)

off

4.

(A)

in

(B)

of

(C)

on

(D)

off

the Banks [1 mark each]



Ans. Objective: To use grammatical items accurately and appropriately. (A) leave

2.







1.





Marking : 1 mark for each correct answer. (A) small 

























3. (A) on 4. (B) of [CBSE Marking Scheme, 2017] Explanation: 1. First form of the verb is used with gerund ‘to’. 2. No comparisons are there, so, smaller and smallest cannot be used. Room is referred to as small if used as a place where someone can live, but referred to as little when referred to in context of scope. 3. Preposition ‘on’ is used when referring to the Banks of a River. 4. Preposition ‘of ’ is used to explain relationship between the Banks and River Ganga. XIV. Complete the paragraph given below by filling in the blanks with the help of options that follows : [CBSE Outside Delhi Set I, 2017 (Modified)] Very few people 1. loneliness. It seems to me that most people 2. scared of 3. left on their 4. . [1 mark each] 1. 2. 3. 4.

XV.

(A) (A) (A) (A)

enjoy is be kids

(B) (B) (B) (B)

enjoys am being relatives

(C) (C) (C) (C)

enjoyed are been home

(D) (D) (D) (D)

enjoying was having own 







Complete the paragraph given below by filling in the blanks with the help of options that follows : [CBSE Outside Delhi Set II, 2017 (Modified)] Last year I visited the same spot 1. I was not 2. a cycle. I saw that many buildings 3. been built on 4. Banks of the Ganga. [1 mark each] 1.

(A)

as

(B)

for

(C)

even if

(D)

but

2.

(A)

ride

(B)

rides

(C)

rode

(D)

riding

3.

(A)

has

(B)

have

(C)

had

(D)

having

4.

(A)

a

(B)

an

(C)

the

(D)

some



[1 × 4 = 4 Marks]

(D) riding (C) the



4.



(C) had

2.



3.



(C) even if



1.



Marking : 1 mark for each correct answer.











Ans. Objective: To use grammatical items accurately and appropriately.

These questions are for practice and their solutions are available at the end of the chapter

[CBSE Marking Scheme, 2017]

@PROCBSE 67

GRAMMAR

Explanation:









1. The second part of the sentence is negative (use of ‘not’), so contradicting preposition ‘even if ’ is the correct choice. 2. ‘Was not’ indicates use of past continuous tense. So, riding is the correct form of verb to be used. 3. Buildings are plural, so, ‘have’ is the correct option. 4. River Ganga is unique, so, use of article ‘The’ with Banks of Ganga is the correct option.

XVI. Complete the paragraph given below by filling in the blanks with the help of options that follows : [CBSE Outside Delhi Set III, 2017 (Modified)] 3.

both of us with ‘Selfie’. I could

say ‘no’

4.





2.

a picture

1.

(A)

want

(B)

wants

(C)

wanted

(D)

wanting

2.

(A)

on

(B)

of

(C)

in

(D)

for

3.

(A)

hard

(B)

harder

(C)

hardest

(D)

hardly

4.

(A)

for

(B)

with

(C)

to

(D)

at

him. [1 mark each] 

1.

A young person



Ans. Objective: To use Grammatical items accurately and appropriately.

4.

(C) to



[1 × 4 = 4 Marks] [CBSE Marking Scheme, 2017 (Modified)]



(B) of



2.







3. (D) hardly Explanation:



(C) wanted



1.





Marking : 1 mark for each correct answer.







1. The event appears to happen in the past. So, past form of the verb – wanted – is suitable. 2. Relationship between ‘picture’ and ‘both’ is explained using preposition ‘of ’. 3. The sentence indicates that neither found difficulty in saying no to the person, so, preposition ‘hardly’ is used here.





XVII. Read the sentence given below and fill in the blanks in it by choosing the most appropriate options from the ones that follow: [CBSE Delhi Set I, 2016 (Modified)] 2.

the success of a Poor Candidate

3.

got the highest number of Votes [1 mark each]





He was 1. at the News 4. the Bihar Election. surprising

(B)

surprise

(C)

surprised

(D)

having surprised

2.

(A)

of

(B)

for

(C)

to

(D)

by

3.

(A)

which

(B)

whose

(C)

whom

(D)

who

4.

(A)

on

(B)

in

(C)

at

(D)

for

4.

(B) in



(A) of [CBSE Marking Scheme, 2016]







3. (D) who Explanation:

2.



(C) surprised



Ans. 1.



(A)



1.









1. Past form of the verb is used with ‘was’ in simple past tense. 2. Preposition ‘of ’ explains about whom the News is. 3. ‘Which’ is used with non-living things. Whose and Whom are used to express belongingness of someone or something is to be conveyed. So, ‘Who’ is the correct option. 4. Elections is an event where activity of casting votes is done. So, preposition ‘in’ is used. 

The Vichitra Veena as 1. name suggests can Play 4. Instrument.

2.

a rare instrument and there are not

3.









XVIII.Complete the paragraph given below with the help of options that follow : [CBSE Delhi Set II, 2016 (Modified)]

1.

(A)

a

(B)

an

(C)

the

(D)

some

2.

(A)

is

(B)

am

(C)

are

(D)

have been

3.

(A)

much

(B)

many

(C)

more

(D)

most

4.

(A)

his

(B)

the

(C)

this

(D)

that

Artists who [1 mark each]

@PROCBSE 68

2.

(A) is



(C) the







Ans. 1.



Oswaal CBSE Question Bank Chapterwise & Topicwise, ENGLISH LANG. & LIT., Class-X



















3. (B) many 4. (C) this [CBSE Marking Scheme, 2016] Explanation: 1. Here, a particular name is talked about. So, article ‘the’ is to be used. 2. As name is singular and in third person, so, ‘is’ is the correct form of ‘be’ verb here. 3. Artists refers to countable plural noun, so, ‘many’ is to be used here. 4. Earlier reference to the musical instrument is invoked using ‘this’. 





XIX. Complete the paragraph given below with the help of options that follow : [CBSE Delhi Set III, 2016 (Modified)]





Kanishka, the ruler of the Kushanas embraced Buddhism. He 1. 2. Kashmir during 3. reign. It was during his time that Buddhism Java.

invited to a Buddhist Meeting 4. to Sri Lanka, Myanmar and [1 mark each]

1.

(A) is

(B)

was

(C)

had

(D)

has

2.

(A) at

(B)

of

(C)

in

(D)

on

3.

(A) her

(B)

his

(C)

their

(D)

that

4.

(A) spread

(B)

spreads

(C)

spreading

(D)

is spread





XX. Read the paragraph given below. Fill in the blanks by choosing the most appropriate options from the ones that follow. Write the answers in your answer sheet against the correct blank numbers. Do not copy the whole passage. [CBSE Delhi Set III, 2016 (Modified)] 2.

found near lakes, rivers, canals [1 mark each]





Kingfisher is 1. beautiful bird with attractive colours. It 3. ponds. Kingfisher 4. omnivores. (B)

an

(C)

the

(D)

such a

2.

(A)

are

(B)

am

(C)

is

(D)

has

3.

(A)

and

(B)

but

(C)

so

(D)

or

4.

(A)

are

(B)

is

(C)

was

(D)

have

Ans. 1.

(A) a

(C) is (A) are

[CBSE Marking Scheme, 2016]









4.





2.

3. (A) and Explanation:



a



(A)



1.





















1. Reference to any singular Kingfisher bird is made using article ‘a’. 2. Simple statement of facts in present tense using the ‘is’ form of verb ‘be’. 3. Conjunction ‘and’ is used indicate multiple nouns. 4. Reference to all Kingfisher birds is made as a general statement of facts – use of ‘are’ form of verb. XXI. Complete the paragraph given below with the help of options that follow : [CBSE Outside Delhi Set II, 2016 (Modified)] Recent heavy rains lashed Chennai City throwing normal life out of gear. The City 1. severe water logging and people wading through knee-deep water. Many Volunteers in their respective areas 2. out the Relief activities. In Annai Satya Nagar, Food packets 3. distributed to the 4. . [1 mark each] (A)

seen

(B)

saw

(C)

see

(D)

sees

2.

(A)

carry

(B)

carrying

(C)

carried

(D)

had carried

3.

(A)

were

(B)

was

(C)

did

(D)

have

4.

(A)

poors

(B)

people

(C)

victims

(D)

needy

2.





(B) saw

(C) carried 









3. (A) were 4. (C) victims [CBSE Marking Scheme, 2016] Explanation: 1. A past is narrated in the paragraph. So, ‘saw’ is used to explain the incident seen by people living in the City at that time. 2. A past is narrated in the paragraph. So, ‘carried’ is used to explain the rescue work done by the Volunteers at that time.





Ans. 1.



1.

These questions are for practice and their solutions are available at the end of the chapter

@PROCBSE GRAMMAR

69















3. Food packets indicate to plural noun and the paragraph is about a past event. So, ‘were’ is to be used while referring to a plural noun. 4. ‘People’ is used to refer to any group of people. ‘Poor’ refers to those who are in need of funds or are unfortunate and ‘Needy’ is used to those who are in need of something but all of these may or may not be affected by some disaster or calamity. The people who are affected by a calamity are referred to as ‘Victims’. XXII.Complete the paragraph given below with the help of options that follow : [CBSE Outside Delhi Set III, 2016 (Modified)] Most Indian schools fail to ensure their students adequate Playtime and Fitness Regime. Two out of every five School Going Children 1. have a healthy Body Mass Index (BMI) and 50% of Children 2. adequate lower body strength. Some Schools 3. found to offer three or 4. Physical Education Periods Per Week. [1 mark each] 1.

(A)

does

(B)

does not

(C)

don’t

(D)

do

2.

(A)

lack

(B)

lacked

(C)

have lacked

(D)

had lacked

3.

(A)

was

(B)

are

(C)

is

(D)

have

4.

(A)

much

(B)

many

(C)

more

(D)

less

(A) lack



2.



(C) don’t





Ans. 1.

















3. (B) are 4. (C) more [CBSE Marking Scheme, 2016] Explanation: 1. As the paragraph starts with a negative tone and is in plural about the students, so ‘do not’ or ‘don’t’ is to be used here. 2. Being a statement of fact, first form of verb – lack – is to be used. 4. As the paragraph is in negative tone, three or ‘less’ periods are referred to indicate the poor condition of Physical Education in the Country.

Commonly Made Errors









Sometimes students choose more than one options as answer. Some students copy incorrect question numbers from the question paper.

Answering Tips









Students must choose only one option as the answer. In no case should more than one answer be given. Students must ensure that they have written the correct question number and option number in their answer.

Solutions for Practice Questions (Topic-1) Objective: To use grammatical items accurately and appropriately.



XII.

(B) who

2.







1.

(A) with





Marking : 1 mark for each correct answer.









Objective: To use grammatical items accurately and appropriately. 

Marking : 1 mark for each correct answer. (D) own







(C) are

4.



2.

(B) being



(A) enjoy

3.



1.









XIV.













3. (C) sat 4. (C) of [CBSE Marking Scheme, 2017] Explanation: 1. ‘Which’ is used with non-living things. Whose and Whom are used to express belongingness of someone or something is to be conveyed. So, ‘Who’ is the correct option. 2. Preposition ‘with’ is used to define relation between the narrator (I), the young persona and the Selfie. 3. The event already took place in the past, so, past tense form of verb – sat – is used. 4. Preposition ‘of ’ is used to express position of the wall where the selfie was taken. [1 × 4 = 4 Marks] [CBSE Marking Scheme, 2017]

@PROCBSE

@PROCBSE

CHAPTER

4

EDITING

Revision Notes Introduction

An editing is a type of exercise in which one word in each line is incorrect. The students have to find the error and write the correct word. In this exercise, the word replaced is generally of the same part of speech as that of the incorrect verb form.

PASSAGE BASED MCQs  



The following paragraph has not been edited. There is one error in each line. Identify the error and write its correction against the correct blank number. Remember to underline the correction. The first one has been done for you. [CBSE SQP, 2021-22] [1 mark each] e.g.

Error learn

Have you ever learn from a mistake you have made?

Correction learnt

Many shouldn’t admit doing so. For those who do,

(a)

___

___

there was no need for guilt. We often make mistakes

(b)

___

___

while taking risks, but all brush them aside and learn.

(c)

___

___

With that, they may not make mistakes the next time.









Ans.

Award 1 mark for each correct answer ½ mark for identification of the error ½ mark for the writing of the correction Editing is incomplete if either aspect (identification or rectification) is missing. Error

Correction

(a)

shouldn’t

won’t/don’t

(b)

was

is

(c)

all

some/many









[CBSE SQP Marking Scheme, 2021] Explanation: 1. Height above sea level or Altitude is expressed using preposition ‘at’. 2. Western Ghats have a single Southern arm and is unique. So, article ‘the’ is used. 3. The arm of the Western Ghats is ‘in’ the deep Bay. 4. The Beauty of Forest Scenery is expressed using the adjective ‘delightful’. Lighted and Delighted are verb forms.



I.

@PROCBSE 72

Oswaal CBSE Question Bank Chapterwise & Topicwise, ENGLISH LANG. & LIT., Class-X







II. The following paragraph has not been edited. There is an error in each line. Write the error along with its correction in the space provided. [CBSE Delhi Set - I, 2020 (Modified)] [1 mark each] Error

Correction

Tallam is situated in an

1.

_______

_______

altitude of about 550 feet on a Southern

2.

_______

_______

arm of a deep Bay of the Western Ghats.

3.

_______

_______

Tallam boasts of delight Forest Scenery.

4.

_______

_______

1.

(A)

at

(B)

on

(C)

of

(D)

onto

2.

(A)

an

(B)

the

(C)

a

(D)

that

3.

(A)

of

(B)

off

(C)

in

(D)

on

4.

(A)

delighted

(B)

lighted

(C)

delightful

(D)

None of the these









Ans. Guidance Award 1 mark for each correct answer • ½ mark for identification of the error • ½ mark for the writing of the correction Editing is incomplete if either aspect (identification or rectification) is missing. Error

Correction

1.

in

(A)

at

2.

a

(B)

the

3.

of

(C)

in

4.

delight

(C)

delightful











[CBSE SQP Marking Scheme, 2020] Explanation: 1. Height above sea level or Altitude is expressed using preposition ‘at’. 2. Western Ghats have a single Southern arm and is unique. So, article ‘the’ is used. 3. The arm of the Western Ghats is ‘in’ the deep Bay. 4. The Beauty of Forest Scenery is expressed using the adjective ‘delightful’. Lighted and Delighted are verb forms.







III. The following paragraph has not been edited. There is an error in each line. Write the error along with its correction in the space provided. [CBSE Outside Delhi Set - I, 2020 (Modified)] [1 mark each] Error

Correction

Spending time for their Kids bring

1.

_______

_______

immediate as well as long-last

2.

_______

_______

gain for a Parents. According to a

3.

_______

_______

research, children with concerned

4.

_______

_______

parents are more efficient. 1.

(A)

for

(B)

with

(C)

along

(D)

of

2.

(A)

latest

(B)

lately

(C)

lasting

(D)

lasted

3.

(A)

all

(B)

the

(C)

an

(D)

one

4.

(A)

of

(B)

off

(C)

among

(D)

between

Error

Correction

1.

for

(B)

with

2.

last

(C)

lasting

3.

a

(B)

the

4.

with

(A)

of 

Ans.

[CBSE SQP Marking Scheme, 2020]

@PROCBSE

GRAMMAR

73









Explanation: 1. Time is spent in company of the Kids. So, preposition ‘with’ is the correct choice. 2. As the gains continue for a long period, so, ‘last’ needs to be replaced with ‘lasting’ as phrasal adjective. 3. Parents is plural. So, both – one and an – cannot be used. 4. Preposition ‘of ’ explains the relationship between children and concerned Parents.







IV. The following paragraph has not been edited. There is an error in each line. Write the error along with its correction in the space provided. [CBSE Delhi Set - I, 2019 (Modified)] [1 mark each] e.g.,

Error

Correction

a

the

Sachin Tendulkar is a best cricketer in India. He has play for

1.

___

___

more than 20 years on the country.

2.

___

___

He retired from the games

3.

___

___

4.

___

___

last year. He is known for his skill in batting or fielding. 1.

(A)

plays

(B)

played

(C)

playing

(D)

play

2.

(A)

on

(B)

of

(C)

for

(D)

with

3.

(A)

game

(B)

gaming

(C)

gambling

(D)

an game

4.

(A)

and

(B)

both

(C)

but

(D)

of

Ans.

Error

Correction

1.

play

(B)

2.

on

(C)

for

3.

games

(A)

game

4.

or

(A)

and

played



The following paragraph has not been edited. There is an error in each line. Write the error along with its correction in the space provided. [CBSE Delhi Set - II, 2019 (Modified)] [1 mark each] 

V.













[CBSE SQP Marking Scheme, 2019] Explanation: 1. Present perfect tense in singular is written as ‘has’ + . So, ‘play’ must be replaced with ‘played’. 2. As he has played on behalf of the Country or representing the Country, so, ‘on’ must be replaced with ‘for. 3. Here, cricket is referred to as a singular game. If it would have been matches, then it would have been correct but with game, it should be singular. 4. He was both a batsman ‘and’ a bowler. ‘Or’ is used in case if only one is correct.

e.g.,

Error in

of

is fundamental for human

1.

___

___

happiness. There are so much

2.

___

___

The pleasure in being outdoors

Correction

to learn and it is an constant

3.

___

___

source of delight to know more on the

4.

___

___

flora and fauna around. 1.

(A) of

(B)

to

(C)

with

(D)

about

2.

(A) is

(B)

was

(C)

were

(D)

are

3.

(A) a

(B)

the

(C)

few

(D)

none

4.

(A) into

(B)

about

(C)

of

(D)

onto

@PROCBSE 74

Oswaal CBSE Question Bank Chapterwise & Topicwise, ENGLISH LANG. & LIT., Class-X

Ans.

Error

Correction

1.

for

(B)

to

2.

are

(A)

is

3.

an

(A)

a

4.

on

(B)

about











[CBSE Marking Scheme, 2019] Explanation: 1. Fundamental as verb is always followed with ‘to’ preposition. 2. ‘Much’ is used with uncountable things, so, singular forms are used with much. As the paragraph is in present tense, so ‘is’ is correct option. 3. ‘An’ is used with words beginning with vowel sound. Constant does not start with vowel sound, so ‘an’ must be replaced with ‘a’. 4. When information is to be collected, ‘about’ preposition is a correct choice.







VI. The following paragraph has not been edited. There is an error in each line. Write the error along with its correction in the space provided. [CBSE Delhi Set - III, 2019 (Modified)] [1 mark each] e.g.,

Error in

of

large cities are on areas sensitive

1.

___

___

over Global warming and rising Sea level.

2.

___

___

Millions of people face a risk

3.

___

___

of being swamp by Floods

4.

___

___

More than two-thirds in the World’s

Correction

and Storms according to a study. 1.

(A)

on

(B)

of

(C)

for

(D)

in

2.

(A)

too

(B)

to

(C)

with

(D)

off

3.

(A)

an

(B)

the

(C)

related

(D)

to

4.

(A)

swamps

(B)

swamping

(C)

swamped

(D)

swamp

Ans.

Error

Correction

1.

on

(D)

in

2.

over

(B)

to

3.

a

(B)

the











4. swamp (C) swamped [CBSE Marking Scheme, 2019] Explanation: 1. Cities are situated ‘in’ the areas or regions. 2. When sensitivity about some issue is to be expressed, ‘to’ preposition is used. 3. Risk can not be counted. ‘A’ is used with countable nouns. So, ‘the’ article must replace ‘a’ in the sentence. 4. Third form of verb is used with ‘being’. So, ‘swamped’ is the correct word to be used in the sentence.









VII. The following paragraph has not been edited. There is one error in each line. Identify the error and write down its correction against the correct blank number. The first one has been done for you. [CBSE Outside Delhi Set - III, 2019 (Modified)] [1 mark each] e.g.,

Error

Correction

is

are

Some days is good and some days are bad. Today, I got on very late. I

1.

___

___

ran to bathe. Then I had to got ready

2.

___

___

for my school. I ran by the bus stop to

3.

___

___

4.

___

___

catch my school bus. It had already left but I came back home. 1.

(A) in

(B)

into

(C)

up

(D)

upon

2.

(A) gets

(B)

getting

(C)

get

(D)

got

@PROCBSE

GRAMMAR

3.

(A) to

(B)

too

(C)

for

(D)

of

4.

(A) as

(B)

when

(C)

so

(D)

as if

Ans.

Error

75

Correction

1.

on

(C)

up

2.

got

(C)

get

3.

by

(A)

to

4.

but

(C)

so











[CBSE Marking Scheme, 2019] Explanation: 1. Waking is also referred to as ‘getting up’. So, ‘on’ must be replaced with ‘up’ here. 2. First form of the verb is used with determiner ‘to’, so, ‘get’ is the correct option. 3. ‘By’ is used to indicate alongside. ‘To’ is used to indicate towards. 4. As a result of missing the school bus, I came back home. ‘So’ is the correct choice.









VIII.The following paragraph has not been edited. There is one error in each line. Identify the error and write down its correction against the correct blank number. The first one has been done for you. [CBSE Outside Delhi Set - III, 2019 (Modified)] [1 mark each] The modern student understand the importance for physical exercise.

Error

Correction

e.g.,

understand

understands

1.

___

___

He spend one or two hours daily

2.

___

___

on the open air where he takes

3.

___

___

part on exercises when work

4.

___

___

wonders for his health. (A) at

(B)

of

(C)

off

(D)

on

(A) spent

(B)

spends

(C)

spending

(D)

spend

3.

(A) into

(B)

in

(C)

by

(D)

of



4. (A) which (B) where (C) whether (D) why The following paragraph has not been edited. There is an error in each line. Write the error along with its correction in the space provided. [CBSE Delhi Set - I, 2018 (Modified)] [1 mark each] 



IX.

1. 2.

Error

Correction

In the prisoner’s room a candle is

e.g.,

is

was

burning dimly. A prisoner himself

1.

___

___

sat by the table Only him back,

2.

___

___

the hair by his head, and his

3.

___

___

hands are visible from outside

4.

___

___

through the window. 1.

(A)

an

(B)

The

(C)

two

(D)

None of these

2.

(A)

his

(B)

her

(C)

their

(D)

he

3.

(A)

of

(B)

on

(C)

near

(D)

off

4.

(A)

were

(B)

was

(C)

is

(D)

have

Ans.

Error

Correction

A

(B)

The

2.

him

(A)

his

3.

by

(B)

on

4.

are

(A)

were 

1.

These questions are for practice and their solutions are available at the end of the chapter

[CBSE Marking Scheme, 2018]

@PROCBSE 76

Oswaal CBSE Question Bank Chapterwise & Topicwise, ENGLISH LANG. & LIT., Class-X





X.











Explanation: 1. The paragraph is about a particular prisoner. So, article ‘the’ must replace the article ‘a’. 2. Belongingness of back is to be determined using possessive Pronoun ‘his’ in place of Personal Pronoun ‘him’. 3. The hairs are on top of the head, so preposition ‘on’ must be used. 4. The paragraph is in past tense. So, ‘are’ must be replaced by ‘were’. The following paragraph has not been edited. There is one error in each line. Write the error and its correction as shown in the example. [CBSE Delhi Set - I, 2017 (Modified)] [1 mark each] Error

Correction

A saint walks the streets of Kolkata It can happen only at India. It is time that us realised our

e.g.,

walks

walked

1. 2.

___ ___

___ ___

strength. We are greater people. We have so much religious leaders.

3. 4.

___ ___

___ ___

1.

(A)

on

(B)

in

(C)

at

(D)

near

2. 3. 4.

(A) (A) (A)

ours great more

(B) (B) (B)

they greatest few

(C) (C) (C)

he greater many

(D) (D) (D)

we greatful all

Ans.

Error

Correction

1.

at

(B)

in

2.

us

(D)

we

3.

greater

(A)

great

4.

much

(C)

many

















[CBSE Marking Scheme, 2017] Explanation: 1. The event can happen ‘in’ a Country, ‘at’ is used to determine a small place within a big place. 2. Possessive Pronoun ‘us’ is not to be used here. It must be replaced with Personal Pronoun ‘we’. 3. There is no comparison being made with someone else, so comparative degree is not needed. 4. Religious Leaders are a countable noun. ‘Much’ is used with uncountable nouns. For countable nouns, we use ‘many’ as adjective. XI. The following paragraph has not been edited. There is one error in each line. Write the error and its correction as shown in the example. [CBSE Delhi Set - II, 2017 (Modified)] [1 mark each] Error

Correction

We, there in India have so many

e.g.,

there

here

religions, we ate so many different

1.

___

___

foods, we wear so much different

2.

___

___

kinds of dresses. Still we are much

3.

___

___

united than any other country on the world.

4.

___

___

1.

(A)

eaten

(B)

eating

(C)

eat

(D)

eats

2.

(A)

most

(B)

more

(C)

many

(D)

few

3.

(A)

most

(B)

more

(C)

many

(D)

few

I met her by chance then I







4. (A) of (B) at (C) in (D) up XII. The following paragraph has not been edited. There is one error in each line. Write the error and its correction as shown in the example. [CBSE Delhi Set - II, 2017 (Modified)] [1 mark each] e.g.,

Error

Correction

then

when

went to getting some Medicine

1.

___

___

which Sister Amy use to give.

2.

___

___

It was amazing for meet mother.

3.

___

___

I start loving just her presence.

4.

___

___

These questions are for practice and their solutions are available at the end of the chapter

@PROCBSE

GRAMMAR

(A) gets

(B)

got

(C)

getting

(D)

get

2. 3. 4.

(A) uses (A) to (A) starting

(B) (B) (B)

used as started

(C) (C) (C)

to use on start

(D) (D) (D)

useful from starts

1.

getting

(D)

get

2.

use

(B)

used

3.

for

(A)

to

4.

start

(B)

started



4



Objective: To use Grammatical items accurately and appropriately. Marking : 1 mark for each correct answer. Error Correction



Ans.

1.

77















[CBSE Marking Scheme, 2017] Explanation: 1. Only first form of verb is used after ‘to’. So, ‘getting’ must be replaced with ‘get’. 2. As Sister Amy did something in the past, so, ‘use’ must be replaced with ‘used’. 3. Preposition ‘to’ explains how the narrator felt upon meeting Mother. XIII.The following paragraph has not been edited. There is an error in each line. Write the error and its correction as shown in the example. [CBSE Outside Delhi Set - II, 2017 (Modified)] [1 mark each] Being with her give One a

Error

Correction

give

gave

new kind on energy. When she

1.

___

___

put his hand on your head

2.

___

___

to blessed you or you touched

3.

___

___

her you can feel her love.

4.

___

___

1.

(A)

in

(B)

off

(C)

of

(D)

as

2.

(A)

him

(B)

her

(C)

she

(D)

your

3.

(A)

bless

(B)

blessing

(C)

blest

(D)

blesses

4.

(A)

would

(B)

may

(C)

might

(D)

could



4







Ans. Objective: To use Grammatical items accurately and appropriately. Marking : 1 mark for each correct answer.

My meetings of mother had always been a finer experience. I would go to him whenever I was sad, happy and angry. Sometimes I would told her what somebody. had said to me.

















Error Correction 1. on (C) of 2. his (B) her 3. blessed (A) bless 4. can (D) could [CBSE Marking Scheme, 2017] Explanation: 1. The correct preposition is ‘of ’ when types are being discussed. 2. ‘His’ is masculine possessive Pronoun. As ‘she’ is Feminine gender and is used in the paragraph previously, the correct possessive Pronoun to be used here is ‘her’. 3. Only first form of verb is used after ‘to’. So, ‘blessed’ must be replaced with ‘bless’. 4. The paragraph is in past tense, 'could' is appropriate. XIV. The following paragraph has not been edited. There is an error in each line. Write the error and its correction as shown in the example. [CBSE Outside Delhi Set - III, 2017 (Modified)] [1 mark each] e.g. 1. 2. 3. 4.

Error of ___ ___ ___ ___

Correction with ___ ___ ___ ___

@PROCBSE 78

Oswaal CBSE Question Bank Chapterwise & Topicwise, ENGLISH LANG. & LIT., Class-X

(A) fines

(B)

fine

(C)

find

(D)

finest

2.

(A) he

(B)

her

(C)

she

(D)

them

3.

(A) as

(B)

or

(C)

nor

(D)

both

4.

(A) told

(B)

tell

(C)

tells

(D)

telling

Error finer him and told

Correction fine her or tell

(B) (B) (B) (B)

[CBSE Marking Scheme, 2017]





1. 2. 3. 4.

[1×4 = 4 Marks]



Objective: To use Grammatical items accurately and appropriately. Marking : 1 mark for each correct answer.



Ans.

1.









Explanation: 1. There is no comparison with anyone. So, comparative degree is not required. The correct word is ‘fine’. 2. ‘Mother’ is feminine gender, so, the correct possessive Pronoun is ‘her’. 3. One cannot be both happy ‘and’ angry at the same time. The correct conjunction to be used here is ‘or’. 4. Simple present tense and Third person use is ‘tell’ and not ‘told’. The following paragraph has not been edited. There is one error in each line against which a blank has been given. Write the error and the correction in your answer sheet against the correct blank number as given in the example. Remember to underline the word that you have supplied. [CBSE Delhi Set - I, 2016 (Modified)] [1 mark each] Error Correction e.g. an a Research is an detailed study of a ___ ___ 1. subject undertaking on a systematic basis in order to increase a Stock of 2. ___ ___ Knowledge, including Knowledge for man, 3. ___ ___ ___ ___ Culture and Society that the use of this Stock 4.  







XV.

of knowledge to devise new applications.



(A) undertake

(B)

undertakes

(C)

undertaken

(D)

undertook

2.

(A) an

(B)

some

(C)

the

(D)

only

3.

(A) in

(B)

of

(C)

near

(D)

on

4.

(A) and

(B)

both

(C)

as

(D)

whether

Objective: To use Grammatical items accurately and appropriately. Marking : 1 mark for each correct answer. ERROR





Ans.

1.

[1×4 = 4 Marks]

CORRECTION

1.

undertaking

(C)

undertaken

2.

a

(C)

the

3.

for

(B)

of

4.

that

(A)

and









[CBSE Marking Scheme, 2016] Explanation: 1. ‘Undertaking’ is a noun. The work / research is ‘undertaken’ which is the correct verb form. 2. Stock is a collective noun. With collective nouns, article ‘the’ is used. 3. The preposition ‘of ’ is used to denote that the Knowledge belongs to the Man, Culture and Society.













Commonly Made Errors Some students read the lines in hurry as a result they fail to detect any error in the lines. Students are unable to find the incorrect word due to lack of practice. Students do not read the whole paragraph in one context.

@PROCBSE

GRAMMAR

79





















Answering Tips Lay emphasis on each word and ask self editing questions to your self such as, – Do subject and verb agree in number? – Have the prepositions been used relevantly? Adequate practice in tenses and verb forms will help in solving ‘editing’ exercises. Read the sentences thoroughly before answering. Do not jump to conclusions. Think logically.

Solutions for Practice Questions (Topic-1) 8.

Error

Correction

1.

for

(B)

of

2.

spend

(B)

spends

3.

on

(B)

in

4.

when

(A)

which

4



Ans. 11 Objective: To use Grammatical items accurately and appropriately. Marking : 1 mark for each correct answer.













[CBSE Marking Scheme, 2019] Explanation: 1. Relationship between importance and exercise is explained using preposition ‘of ’. 2. Student is singular and is a statement of fact – simple present tense is used. 3. The student cannot sit on top of air, so ‘in’ is the correct preposition here. 4. Adjective of place ‘which’ is used to tell the place at which the Modern Student takes Exercise.

Error

Correction

1.

ate

(C)

eat

2.

much

(C)

many

3.

much

(B)

more

4.

on

(A)

of











[CBSE Marking Scheme, 2017] Explanation: 1. As it is a general statement of facts, simple present tense is to be used. With ‘we’, the correct form of the verb is ‘eat’. 2. ‘Much’ is used with uncountable nouns. For countable nouns, like Dresses, we use ‘many’ as adjective. 3. Comparative degree is to be used as we are comparing ourselves with the World – the use of ‘than’. So, more is the comparative degree to be used here. 4. Preposition ‘of ’ clarifies that all the countries belong to the World or are part of the World.

@PROCBSE

CHAPTER

5

SENTENCE TRANSFORMATION



















Transformation of Sentences is done in various ways. The nature of the sentences can be changed without changing the meaning of the sentences. Sentences containing the adverb ‘too’: Example 1 :  My friend is too rich to be my consort. You can see how the Transformation of Sentences, containing the adverb ‘too’, takes the place without changing the meaning of the sentence.  My friend is so rich that he cannot be my consort. Example 2 :  The news is too good to be true. You can see how the Transformation of Sentences takes place into the following version without changing the meaning of the sentence.  The news is so good that it cannot be true. Example 3 :  He drove too fast for the police to catch. This sentence can be changed into the following version without changing the meaning of the sentence.  He drove so fast that the police could not catch him. The transformation of sentence takes place by removing the adverb ‘too’ and by adding a conjunction ‘so…that’. In this way, the following sentences have been changed for your revision. Examples :  He is too proud to beg. He is so proud that he will not beg.  It is never too late to mend. It is not so late that it cannot be mended.  He is too ignorant for the post of a postman.

He is so ignorant that he is not suitable for the post of a postman. This shirt is small for me. This shirt is so small that it is not suitable for me. He speaks too fast to be understood. He speaks so fast that he cannot be understood.





Interchange of Degrees of Comparison: : The Transformation of Sentences, containing comparatives, can be done as follows without changing the meaning of the sentences. Example 1 :  I am as strong as him. This sentence is in positive degree. This sentence can be changed into a sentence of comparative one.  I am not stronger than him.













This sentence conveys the same meaning as the above sentence. Example 2 :  Positive : This razor is not as sharp as that one.  Comparative : That razor is sharper than this one. Example 3 :  Positive : Very few cities in India are as rich as Mumbai.  Comparative : Mumbai is richer that most other cities in India.  Superlative : Mumbai is one of the richest cities in India. Example 4 :  Superlative : Upshaw is not the tallest girl in the class.















































2.











































1.







SENTENCE TRANSFORMATION

@PROCBSE































1.



81

sentences: The affirmative sentence can be changed into a negative sentence by using ‘not’. Example 1 :  Affirmative : I was doubtful whether it was you. Negative : I was not sure that it was you. Example 2 :  Affirmative : Everybody is present. Negative : Nobody is absent. Example 3 :  Affirmative : All cheered. Negative : There was no one present who did not cheer. In this way, the negative sentences can be changed into affirmative sentences as follows. Example 4 :  Negative : God will not forget the cry of the humble. Affirmative : God will heed the cry of the humble. The ‘not’ in the negative sentences should be removed to convert them into affirmative sentences. Example 5 :  Negative : No one could deny that she is pretty. Affirmative : Everyone accepts that she is pretty. As seen above, the Transformation of Sentences can be done in many ways without changing the meaning of the sentences. There are few more ways in which the Transformation of Sentences can be done. Let us see here few more methods for such a transformation. To transform an interrogative sentence into an assertive sentence: An interrogative sentence can be transformed into an assertive sentence and vice-versa. Example 1 :  When can their glory fade? This is an interrogative sentence. This sentence can be transformed into an assertive sentence as follows.  Their glory can never fade. Example 2 :  Was he not a villain to do such a deed? This interrogative sentence can be transformed into an assertive sentence as follows.  He was a villain to do such a deed. Example 3 :  Who can touch ditch without being defiled? (Interrogative Sentence)





























































4.

Comparative : Upshaw is not taller than many girls in the class. The Transformation of Sentences, according to the nature of the sentences, takes place into either negative or affirmative sentences. Interchange of Active and Passive voice: A sentence in the Active form can be changed into Passive form. Example 1 :  Active : Brutus stabbed Caesar. Passive : Caesar was stabbed by Brutus. The proper Auxiliary verb and ‘by’ are used in the Passive form. Example 2 :  Active : The members will make him the President of this organization. Passive : He will be made the President of this organization by its members. Example 3 :  Active : The audience loudly cheered the Mayor’s speech. Passive : The Mayor’s speech was loudly cheered. Whenever it is evident who the agent (doer of the action) is, it is not necessary to mention it in the passive voice and this omission gives the sentence a beauty. In example, it is evident that only the audience would have cheered the speaker. So, it has been avoided and such an omission adds only a touch of beauty to the sentence. Example 4 : The same way, the proper Pronoun form and the proper verb should be added in the active sentence when the passive sentence is changed into an active sentence.  Passive : She is known to me. Active : I know her. Example 5 :  Passive : Promises should be kept. Active : One should keep one’s promises. The Active Voice is used to make the agent prominent. The Passive Voice is used to make ‘the action of the verb’ prominent. The Passive Voice can be used when the agent is unknown. Example 6 :  Active : We admire the brave.  Passive : The brave are admired. In this sentence, the agent ‘we’ has been omitted to generalize the statement that the brave are admired. Although, the Active-sentence has used the pronoun ‘we’. Interchange of affirmative and negative 



3.

SENTENCE TRANSFORMATION

@PROCBSE Oswaal CBSE Question Bank Chapterwise & Topicwise, ENGLISH LANG. & LIT., Class-X







































































To change one part of a sentence for another part : The verb of a sentence itself can be changed into another verb without (changing) the meaning of the sentence. Example 1 :  This kind of jokes never amuses me. This kind of joke never gives me any amusement. In this sentence the verb has been changed into its noun form. Example 2 :  It costs twelve dollars. Its cost is twelve dollars. Here also the verb has been changed into its noun form. You can see how the Transformation of Sentences takes place in the following examples without changing the meaning of the sentence. Example 3 :  He has disgraced his family. He is a disgrace to his family. Example 4 :  He gave a curt reply. He replied curtly. Here the adjective has been changed into an adverb. Example 5 :  I see him every day. I see him daily. Example 6 :  This scene is surpassingly beautiful. The beauty of this scene is surpassing. Here the adjective has been changed into its noun form. You can see how the Transformation of Sentences takes place in the following examples without changing the meaning of the sentence. Example 7 :  He is admittedly the greatest general of this country. It has been admitted that he is the greatest general of this country. Here, the adverb has been changed into a verb form. And the simple sentence itself has been changed into a compound sentence. So far we have seen the nature of Transformations of Sentence. At the same time, a simple sentence can be changed into a complex sentence and viceversa. At the same time, a compound sentence can be changed into a complex sentence and vice













2.

(Assertive Sentence)





3.



You can see How the Transformation-ofSentence takes place into the following version without changing the meaning of the sentence.  No one can touch ditch without being defiled. (Assertive Sentence) Example 4 :  Who does not know him? (Interrogative Sentence)  Everyone knows him. (Assertive Sentence) Example 5 :  Is this the kind of dress to be worn for a school function? (Interrogative Sentence)  This is not the kind of dress to be worn for a school function. (Assertive Sentence) In this same way an assertive sentence can be transformed into an interrogative sentence. Example 6 :  We were not sent to this world simply to make money. (Assertive Sentence)  Were we sent to this world simply to make money? (Interrogative Sentence) Example 7 :  I never forget those happy days. (Assertive Sentence)  Shall I ever forget those happy days? (Interrogative Sentence) To transform an exclamatory sentence into an assertive sentence:  How sweet the moonlight sleeps upon the riverbank! (Exclamatory Sentence)  The moonlight sweetly sleeps upon the river bank. (Assertive Sentence) Although such an exclamatory sentence can be transformed into an assertive sentence, an exclamatory sentence is preferred on many occasions to an assertive sentence for the emotional effect that an exclamatory sentence is carrying. Example 1 :  If only I were young again! (Exclamatory Sentence)  I wish I were young again. (Assertive Sentence) Example 2 :  How beautiful is this night! (Exclamatory Sentence) This night is very beautiful. (Assertive Sentence) You can see how the Transformation of Sentence takes place in the following examples without changing the meaning of the sentence. Example 3 :  What a delicious meal! (Exclamatory Sentence) This meal is delicious one.















82

@PROCBSE

SENTENCE TRANSFORMATION













































































4.









3.

































2.

































1.

At the same time, a complex sentence can be changed into a compound sentence and viceversa. All such changes are possible to make the ideas of the sentences understandable. This is in addition to the previous chapters on Transformation of Sentences. To transform the simple sentence into a compound sentence: A simple sentence can be transformed into a compound sentence by enlarging a phrase or word into a co-ordinate clause. Example 1 :  He must work hard to make up for the lost time. This sentence can be made into two parts and those two parts can be joined by a conjunction ‘and’.  He must work hard and make up the lost time. Example 2 :  To his eternal disgrace, he betrayed his country.  He betrayed his country and this was to his eternal disgrace. Example 3 :  Besides robbing the poor child, he also murdered the child.  He not only robbed the poor child but he also murdered the child. Example 4 :  The teacher punished the children for disobedience.  The children were disobedient so the teacher punished them You can see how the Transformations of Sentence take place without changing the meaning of the sentence. To transform a compound sentence into a simple sentence: The following examples illustrate the chief ways in which the compound sentences are transformed into simple sentences. Example 1 :  We must eat or we cannot live. We must eat to live. Example 2 :  You must either pay the bill at once or return the goods. Failing prompt payment, the goods must be returned by you. Example 3 :  He must not be late or he will be returned. In the event of his being late, he will be denied entry. Example 4 :  He is rich, yet he is not content. In spite of being rich, he is not content. Example 5 :

This coat cannot be mine, for it is too big. Due to its big size, it cannot be mine. Example 6 :  He is very poor, but he does not complain. In spite of being poor, he does not complain. These examples are enough to make clear how the compound sentences are transformed into simple sentences. You can see how the Transformation of Sentences take place without changing the meaning of the sentence. To transform a simple sentence into a complex sentence: A simple sentence can be transformed into a complex sentence by enlarging a phrase into a subordinate clause. The clause maybe Noun, Adjective or Adverb. Example 1 :  He confessed his crime. Here the noun (his crime) has been changed into a subordinate clause.  He confessed that he was guilty of the crime. Example 2 :  On the arrival of the mails, the steamer will leave. Here the adverbial phrase has been changed into a subordinate clause.  The steamer will leave as soon as the mails arrive. Example 3 :  I saw a wounded bird. Here the adjective phrase has been changed into a subordinate clause.  I saw a bird that was wounded. Example 4 :  On being punished, he wept.  When he was punished, he wept. You can see how the Transformation of Sentences take place without changing the meaning of the sentence. To transform a Complex Sentence into a Simple Sentence: The following sentences will make it clear how to transform the complex sentences into simple sentences. Example 1 :  He said that he was an innocent. This Complex Sentence has been changed into a Simple Sentence as follows.  He declared his innocence. Example 2 :  How long I will stay is doubtful. Here, the Subordinate Clause has been changed into a Noun Clause.  The duration of my stay is doubtful. Example 3 :  Tell me where you live. Here also, the Subordinate Clause has been changed into a Noun Clause.  Tell me your address. Example 4 :  He died in the village where he lived. 



versa.

83

@PROCBSE 84

Oswaal CBSE Question Bank Chapterwise & Topicwise, ENGLISH LANG. & LIT., Class-X





















changed into an Adverb Clause.  He was too tired to stand. Example 7 :  He will not pay unless he is compelled. Here also, the Subordinate Clause has been changed into an Adverb Clause.  He will pay only under compulsion. You can see how the 'Transformations of Sentence' take place without changing the meaning of the sentence.





















Here the Subordinate Clause has been changed into an Adjective Clause.  He died in his native place. Example 5 :  The moment that is lost is lost forever. Here also the Subordinate Clause has been changed into an Adjective Clause.  The lost moment is lost forever. Example 6 :  He was so tired that he could not stand. Here the Subordinate Clause has been

REPORTED SPEECH

:





Ranbir



The car has been towed away as you parked it in the ‘no parking zone’.





Policemen :



Where is my car ? I remember parking it here. Sir, can I be excused this once ?







Policeman :

I hope you’ll remember the lesson you learnt today. OK ! This should be the last time. Don’t repeat



:





Ranbir







Q. 1. Read the dialogue given below and then complete the report that follows. Write your answers in your answer sheet against the correct blank numbers. (Any four) (4)



the mistake again.





Ranbir asked the policeman standing there (a) __________ . The policeman replied that the car (b) __________ parked it in the ‘no parking zone’. Ranbir requested the policeman (c) __________. The policeman hoped he’d remember (d) __________ . But finally he (e) __________.



Ans. (a) where his car was and added that he remembered parking it there.



(b) had been towed away as he had





(c) to excuse him that once



(d) the lesson he had learnt that day



(e) advised him not to repeat the mistake again



Ram



:



:



:

Shyam



Ram











Q. 2. Read the dialogue given below and then complete the report that follows. Write your answers in your answer sheet against the correct blank numbers. (Any four) (4) What a cold day it is ! I think this must be the coldest day of the season. I don’t mind the cold. It is the summer that bothers me. Well ! I am only worried about my mother.

























Ram explained that (a) __________ and was positive that (b) __________ Shyam replied that (c) __________ and further added that (d) __________. Ram stated that (e) __________. Ans. (a) it was a very cold day (b) it must be the coldest day of the season (c) he didn’t mind the cold (d) it was the summer that bothered him (e) he was worried about his mother

What are you going to do ?

:

I will phone the warden.

:

What’s her name ?

:

Jenny, she is a very helpful person.







:





Son





Father







Son





Father







Q. 3. Read the dialogue given below and then complete the report that follows. Write your answers in your answer sheet against the correct blank numbers. (Any four) (4)





The father asked his son (a) __________ to do. The son (b) __________ phone the warden. On this the father asked (c) __________. The son answered that it (d) __________ . The son also said that she was (e) __________ .



(b) replied that he would what her name was



(c)

(d) was Jenny









Ans. (a) what he was going

@PROCBSE

SENTENCE TRANSFORMATION

85





(e) a very helpful person







































Q. 4. Read the dialogue given below and then complete the report that follows. Write your answers in your answer sheet against the correct blank numbers. (Any four) (4) Ram : I want to meet the Principal. Peon : Sir, he is not in his office. Ram : When will he come back to the office ? Peon : I think in another ten minutes. Ram : What are the meeting hours ? Ram went to the Principal’s office and told the peon (a) __________ the Principal. Respectfully, the peon told him that (b) __________. So Ram asked him (c) __________. The peon replied that (d) __________ minutes. Ram again enquired from the peon that (e) __________.





(b) he was not in his office





Ans. (a) that he wanted to meet





(c) when he would come back to the office







(d) he would be back in another ten (e) what the meeting hours were





Q. 5. Read the following dialogue and then complete the paragraph that follows. Write your answers in your answer sheet against the correct blank numbers. (Any four) (4)



: Maitryee ! My new dress is very pretty.

: Will you show it to me ? : I will show it to you tomorrow when you come to my house. My father gave it to me as a birthday gift.



Kokila





Maitryee





Kokila























She replied that (c) ....................... to her (d) ....................... house. Her father had given it (e) ................ . Ans. (a) her new dress was (b) she would show it to her (c) she would show it (d) the next day when she came to her (e) to her as a birthday gift





Kokila told her friend Maitryee that (a) ....................... very pretty. Maitryee asked her if (b) ................





: How did the locals react to it ? : Many assisted us during the making of the film. I am thankful to the local residents.





Interviewer Santosh

: Was shooting in Kashmir challenging ? : Filming in any place is difficult, but in Kashmir it was even more difficult.





Santosh





Interviewer







Q. 6. Read the following dialogue and then complete the paragraph that follows. Write your answers in your answer sheet against the correct blank numbers. (Any four) (4)

























The interviewer asked Santosh (a) ...................... challenging in Kashmir. Santosh replied that (b) ............................... difficult but in Kashmir (c) ...................... more difficult. The interviewer wanted to know (d) ........................ to it. Santosh told him that many had assisted them. He was very thankful (e) ........................ . Ans. (a) if shooting had been (b) filming in any place was (c) it had been even (d) how the locals had reacted (e) to the local residents



Ravi

: Where ?

Rani

: He is lying under the umbrella near the coconut tree.

Ravi

: Oh yes, I see him. He looks very comfortable. Let us leave him alone. Let’s go for a sea-ride.









: There’s Mohan !



Rani















Q. 7. Read the following dialogue and then complete the paragraph that follows. Write your answers in your answer sheet against the correct blank numbers. (Any four) (4)

@PROCBSE 86

Oswaal CBSE Question Bank Chapterwise & Topicwise, ENGLISH LANG. & LIT., Class-X



Rani exclaimed with surprise that Mohan (a) ............................ Ravi enquired where he was. Rani told Ravi that he (b) ............................ near the coconut tree. Ravi agreed that he could see Mohan who (c) ............................ . He suggested that they (d) ............................ Mohan alone. Ravi further suggested to (e) ............................ .



Ans. (a) was there



(c)

looked very comfortable





(b) was lying under the umbrella





(d) should leave



(e) go for a sea-ride (4)





Q. 8. Read the following dialogues and complete the paragraph that follows. (Any four)



: Not now, Wait till your class 12 board results.



Dad





Rajeev : Dad, will you gift me a laptop ?





: Yes, but my long cherished desire is to see you top class 12 too. You should prepare for JEE-MAINS also



Dad





Rajeev : But, Dad, didn’t you promise me one ?





Rajeev asked his dad (a) ...................... Negating, Dad asked Rajeev (b) ...................... A little disappointed Rajeev (c) ...................... Motivating Rajeev, Dad said that his long cherished desire (d) ...................... top class 12 too. He also wished that Rajeev should prepare (e) ...................... .



Ans. (a) if he would gift him a laptop.



(c)

reminded dad if he hadn’t promised him one.





(b) to wait till his class 12 board results.





(d) was to see him



(e) For JEE-MAINS too.

(4)

: Well, you will have to pay only six lacs for her treatment.





Dr. Gautam



: Dr. my wife is unwell. May I know what will be the total expenses on her treatment ?



Ganesh

: Is there any concession ?

Dr. Gautam

: No, not a penny. I want the complete amount together.





Ganesh













Q. 9. Read the conversation given below and complete the paragraph that follows. (Any four)



Ganesh told Dr. Gautam that his wife was unwell and asked if (a) ...................... on her treatment. The doctor replied that (b) ...................... for her treatment. Ganesh asked (c) ...................... The doctor refused (d) ...................... concession, and further stated that he wanted (e) ...................... .



Ans. (a) he might know what would be the total expenses



(c)

if there was any concession.





(b) he would have to pay only six lakhs





(d) to give any





(e) the complete amount together.



Son

: I am going to my friend’s house.



: No, I’ll do it when I will come back.

: This is not the right way to answer.



Father





: Have you completed your homework ?

Son



Father







(b) that he was going







Father asked his son (a) ...................... . The son replied (b) ...................... to his friend’s house. The father further questioned (c) ...................... his homework. The son said that he would do it when (d) ...................... . The father (4) disapproved of (e) ...................... .

Ans. (a) where he was going.

(4)



: Where are you going ?



Father













Q. 10. Read the dialogue given below and complete the paragraph that follows. (Any four)

@PROCBSE

SENTENCE TRANSFORMATION

if he had completed





(c)





(d) he would come back.



(e) the son’s way to answer





REFLECTIONS Were you able to convert the nature of the sentences without changing the meaning of the sentences ?

87

@PROCBSE 88

Oswaal CBSE Question Bank Chapterwise & Topicwise, ENGLISH LANG. & LIT., Class-X

SELF ASSESSMENT PAPER - 02 Max. Marks: 30



Max. Time: 1 hour





Q. 1. Read the passage given below. Fill in the blanks by choosing the most appropriate words from the given options: (4)

Rosemary School, Meerut organized Cleaning Day Drive in neighbouring areas (A) . Participants raised slogans (B) Hindi and English while visiting the areas. The drive (C) generous praise and appreciation from shopkeepers (D) residents of neighboring colonies. The students actively (E) . (A) (i) recent

(ii)

recipient

(iii) recently

(iv)

recentness

(B)

(ii)

about

(iii) by

(iv)

in

(C) (i) earns

(i) on

(ii)

earning

(iii) to earn

(iv)

earned.

(D) (i) and

(ii)

or

(iii) but

(iv)

also

(E)

(ii)

participated

(iii) sang

(iv)

laughed.

(i) danced



(6)

Son

: I am going to my friend’s house.





: Where are you going ?



Father

Son

: No, I’ll do it when I will come back.





: Have you completed your homework ?



Father

Son

: Sorry, I will not repeat it.



: This is not the right way to answer.



Father















Q. 2. Read the dialogue given below and complete the paragraph that follows.









































































































Father asked his son (A) __________ . The son replied (B) __________ to his friend’s house. The father further questioned (C) __________ his homework. The son said that he would do it when (D) __________. The father disapproved saying that it (E) __________ to answer. The son apologised and said that (F) __________. (A) (i) where he is going (ii) where he will be going (iii) where he can be going (iv) where he was going (B) (i) that he was going (ii) that he is going (iii) that he has been going (iv) that he had been going (C) (i) if he has completed (ii) if he will complete (iii) if he had completed (iv) if he had been completing (D) (i) he would come back (ii) he should come back (iii) he will come back (iv) he is coming back (E) (i) is not the right way to answer (ii) was not the right way to answer (iii) will not have the right way to answer (iv) has not the right way to answer (F) (i) he would not repeat it again (ii) he cannot not repeat it again (iii) he could not repeat it again (iv) he must not repeat it again





Q. 3. The following passage has not been edited. There is an error in the underlined words. Write the incorrect word and choose the correct word from the given options. (4)

Correction .................

fit and being our best. A simple rules

(C) .................

.................

(D) .................

.................

enjoyable activity. Wake up at 5 a.m., do five stretches and five yoga asanas.















of ‘five’ helps make exercising a most



.................

(B) .................



(A) .................

undisputed connection among being



Health precedes action. There are an

















Error

@PROCBSE SELF ASSESSMENT PAPER









(iv) rulers

(iv) of







(iii) in

(iv) about







(iii) ruler

(iv) are







(iii) between



(ii) on

















(i) the

(iii) were





(ii) ruling



(i) rule



(C) rules



(ii) besides



(i) beside

(D) a



(ii) was







(i) is

(B) among









(A) are

Options





Error

(6)







Dr. my wife is unwell. May I know what will be the total expenses on her treatment ? Well, you will have to pay only six lakhs for her treatment. Is there any concession ? No, not a penny. I want the complete amount together. Doctor, I can’t pay this big amount.  

: : : : :















Ganesh Dr. Gautam Ganesh Dr. Gautam Ganesh





Q. 4. Read the dialogue given below and complete the paragraph that follows:



89











































































































Ganesh told Dr. Gautam that (A) __________ and asked if (B) __________ on her treatment. The doctor replied that (C) __________ for her treatment. Ganesh asked (D) __________. The doctor refused to give any concession, and further stated that he wanted (E) __________ . Ganesh told the doctor (F) __________. (A) (i) his wife is unwell (ii) his wife will be unwell (iii) his wife was unwell (iv) his wife were unwell (B) (i) he might know what would be the total expenses (ii) he must know what would be the total expenses (iii) he might knows what would be the total expenses (iv) he will know what would be the total expenses (C) (i) he will have to pay only six lakhs (ii) he should have to pay only six lakhs (iii) he would have to pay only six lakhs (iv) he must have to pay only six lakhs (D) (i) if there is any concession (ii) if there will be any concession (iii) if there was any concession (iv) if there can be any concession (E) (i) the complete amounts together (ii) the complete amount together (iii) the completes amount together (iv) the completing amount together (F) (i) that he shouldn’t pay that big amount (ii) that he couldn’t pay that big amount (iii) that he can pay that big amount (iv) that he mustn’t pay that big amount





Q. 5. Read the passage given below and fill in the blanks by choosing the most appropriate words from the given options. (5)

We are living in a world (A) __________ every girl wants to be a diva and every boy (B) __________ to be called a dude. Today, your social acceptance is measured by your friends’ list (C) __________ a social networking site. But true friend is one (D) __________ helps you in (E) __________ . (A) (i) by

(ii)

that

(iii) where

(iv)

the

(B)

(ii)

desires

(iii) desiring

(iv)

desired

(C) (i) in

(i) desire

(ii)

on

(iii) at

(iv)

for

(D) (i) which

(ii)

that

(iii) who

(iv)

some

(E)

(ii)

office

(iii) need

(iv)

ground

(i) work





Q. 6. The following passage has not been edited. There is an error in the underlined words. Write the incorrect word and choose the correct word from the given options. (5)



................. ................. .................

.................

.................





(E)



(D)

it is a major contributor on climate change.







economies of much countries. But



.................

Correction

.................



(C)

.................

.................



(B)

288 million jobs. It is vital for an



in the world generating on least



(A)



Tourism is one in the biggest businesses















Error

.................

@PROCBSE Oswaal CBSE Question Bank Chapterwise & Topicwise, ENGLISH LANG. & LIT., Class-X













(iii) upon

(iv) must





(iv) am (iv) to



(iv) by





(iv) was

(iii) many



(iii) the



(ii) into







(iii) at

(ii) mostly



(iii) is





(ii) is

(i) in





(E) on



(ii) on



(i) most



(D) much

(ii) the











(i) in



(C) an



(i) in





(i) of

(B) on





(A) a

Options



Error















90

@PROCBSE

CREATIVE WRITING SKILLS CHAPTER

6

I. Formal Letter based on a given situation 100-120 words. One out of two questions is to be answered. 5 marks II .Writing an Analytical Paragraph (100-120 words) on a given Map/ Chart/Graph / Cues. One out of two questions is to be answered. 5 marks 

List of Topics

In this chapter you will study

Page No. 61



Topic-2 : Analytical Paragraph Writing Page No. 74











 Identification of formal letters  To use an appropriate style and format to write a formal letter.  To compose a formal letter as per the demand of the question: Formal Letter to the Editor, Formal Letter of inquiry, Formal Letter to place an order, Formal Letter to file a complaint, Formal Letter to cancel the order.  Learn to write Analytical Paragraph Writing.  Include all the statistics to analyse the data.

Topic-1: Formal Letter Writing 





Syllabus

LETTER WRITING & ANALYTICAL PARAGRAPH

Topic-1

Formal Letter Concepts Covered 

Letter to the Editor



Letter of complaint



Letter of Inquiry



Letter to place order

Revision Notes









l l l l



Format 1. Complimentary close An organised format structure is expected. Absence of even one aspect shall result in credit of ½ marks only. Scan to know Full credit of 1 mark more about requires all listed this topic aspects of format. Zero credit if just format is listed without letter content.











Writing a Formal Letter is a great way to engage with a topic you are passionate about and to influence public opinion. Tips to write a Formal Letter : 1. Properly write your postal address, e-mail address, phone number or any other contact information. 2. Write a simple salutation. 3. State the argument you are responding to, provide evidence and say what should be done. 4. Give a simple closing. 5. Keep the tone ‘formal’ or ‘business like’ but always polite and courteous. 6. Keep the language formal, straight forward and clear. There should not be any ambiguity. 

















Introduction

How to write a Formal Letter

@PROCBSE 92

Oswaal CBSE Question Bank Chapterwise & Topicwise, ENGLISH LANG. & LIT., Class-X

@PROCBSE

CREATIVE WRITING SKILLS

93

Format of a Formal Letter Sample Letter :

You are Kavita Khanna, a resident of 50-C, Pushp Vihar, New Delhi. Write a letter to the editor of ‘India Today’ magazine based on the article on ‘Ban on Poly Bags’, published in the latest edition.

Sender’s Address

50-C, Pushp Vihar, New Delhi

Date

26 August 20××

Receiver’s Designation and address;

The Editor, India Today, New Delhi

Subject Statement should not be very lengthy

Leave one line (Spacing) Leave one line (Spacing)

Subject : Ban on Poly Bags (Spacing)

Sir, Through the columns of your reputed newspaper, I wish to share my views on poly bags which are largely used these days. I totally agree with the views, published in the article ‘Ban on Poly Bags’. It is a fact that poly bags are harmful due to their non-biodegradable nature. They don’t decompose, instead they choke the drainage system. Most animals eat them and die in the long run. If they are burnt, their obnoxious smell pollutes the atmosphere. I wish that people at large, in their own interest and in the interest of mother earth Change in paragraph with the wake up to this problem. Everyone should stop using poly bags. Jute bags, paper bags, etc., should take their place. I request you to publish this letter of mine in change in point. your newspaper so as to make everyone aware about the harm of poly bags. Salutation

Never use apostrophe in Yours Write full name if it is given

Details

Why ban poly bags ? Measures that should be taken

Thank you Yours truly Kavita Khanna





















Q. You are Varun/ Varuna, a resident of Raj Nagar colony, Rajkot, Gujarat. You have noticed that some resident of your colony repeatedly Flouting quarantine rules laid out during the outbreak of Covid-19 Pandemic. Write a letter to the SHO of local police Station drawing attention to the same. Explain how such acts impact the health of the community and request immediate intervention and strict action.



Example 1

Satya Nagar Colony Bhubaneswar 24 May, 20XX The Station House officer Raj Nagar Police Station Rajkot, Gujarat Subject: Complaint Against Flouters of Quarantine Rules Sir, Through this letter of mine, I would like to draw your attention towards some residents of our colony who are repeatedly flouting quarantine rules which have been laid out during the outbreak of COVID 19 pandemic.

@PROCBSE 94

Oswaal CBSE Question Bank Chapterwise & Topicwise, ENGLISH LANG. & LIT., Class-X



There is an evident disregard to the rules of social distancing. Many people are roaming without masks in the colony and do not stay indoors. Despite repeated requests, they are turning a deaf ear to all the appeals. The RWA is also not taking any steps to prevent such behaviour. It is a matter of great concern as they are putting everyone’s life at great risk through their casual behaviour. It aggravates mental stress also.



I request you to look into the matter at the earliest. I earnestly hope that you will take immediate action against these people. I hope for a positive response from your end.



Thank you.



Yours sincerely



Varun

Example 2 





Q. Your are Vayanthi / Vijay from Prakasham Nagar / Secundrabad, Andhra Pradesh, Write a letter to Book Haven Store, requesting home delivery of the books, stationery and art materials you had ordered telephonically. Share the reason for being unable to pick up the goods in person, confirm your address details and a convenient time slot.





204 Prakasham Nagar Secunderabad



14 October 20XX







The Sales Manager Book Haven Store Secunderabad



Subject: Request for home delivery of books.







Sir This is with reference to the order placed telephonically on 9 October ‘20 by me for some reference books, registers, oil pastels, and brushes, I had informed you that I would pick up the goods of the placed order in person. But through this letter, I request you to make arrangements for the home delivery of ordered goods. Actually, an emergency meeting has been called by our organisation and I have to leave for Mumbai for one week tonight only. I would really be grateful if you deliver the ordered goods at the address stated above.



Thanking you.





Yours sincerely Vaijanthi

SUBJECTIVE TYPE QUESTIONS 

Short Answer Type Questions

(5 marks each)

1. Letter of Complaint



















Q. 1. You are Kapil/Komal, living at C-424, Sector-18, Green Park, New Delhi. You bought a washing machine from Balaji Electricals Private Limited, Lajpat Nagar, New Delhi. It started giving trouble within a few days. Write a complaint letter (100-120 words) to the Sales Manager seeking immediate repair or replacement. Invent the details of defects. [CBSE Delhi Set-II 2020 (Modified)] [5] Ans. C-424, Sector 18, Green Park New Delhi 27th May, 20×× The Sales Manager Balaji Electricals Pvt. Ltd.

@PROCBSE

CREATIVE WRITING SKILLS





Lajpat Nagar New Delhi



Subject : Complaint about defective Washing Machine

Yours sincerely Kapil/Komal Q. 2. You are Shammi/Sapna, resident of 12 Mall Road, Agra. You have noticed that majority of the water taps in your colony keep on leaking. Also, water drips from various joints in the supply pipes. All these lead to a big wastage of potable water. Write a letter of complaint to the Municipal Commissioner, Agra Municipal Corporation requesting him to get the necessary repairs done. (100-120 words) [CBSE Outside Delhi Set-III 2020 (Modified)] [5]

















Sir, Through this letter of mine, I wish to inform you that I had purchased an “LG Washing Machine” from your outlet on the receipt no. 10579 on 24th May, 20××. Its been just 3-4 days and it has started giving trouble. It makes a loud noise and stops on its own all of a sudden. The machine does not wash the clothes properly. I am sure it has some manufacturing defect. Since it is within the guarantee period, I request you to get it repaired or better replace it at the earliest. I have been your regular customer and purchase articles in good faith. I will be thankful if the matter is attended to at the earliest. Thanking you

Topper's Answer, 2020





95

@PROCBSE 96



Q. 3. You are Tabassum/Tarun, a resident of Satya Nagar Colony, Bhubaneswar, Odisha. You have noticed that some residents of your colony are repeatedly flouting quarantine rules laid out during the outbreak of the COVID–19 pandemic. Write a letter to the SHO of the local Police Station, drawing attention towards the same. Explain how such acts impact the health of the community and request immediate intervention and strict action. [CBSE SQP, 2020][5] 









Oswaal CBSE Question Bank Chapterwise & Topicwise, ENGLISH LANG. & LIT., Class-X

Q. 1. You are Samina Zaveri, Class X, Vadodara, Gujarat. You come across the following information on a local library’s notice board.



2. Letter of Inquiry

You wish to participate but require more information. Write a letter to Teen-Toggle Games Pvt. Ltd. in about 120 words, enquiring about rules, scholarship details and deadlines. Also enquire about specifications for solo or group entries. [CBSE Board SQP, 2021-22] 



Ans. Value Points–Contents Reference to the source of information Conveying interest Seeking information about rules Enquiring about scholarship details Asking about defined articles Confirm type of entries allowed











• • • • • •





Create Your Own Board Game Competition! Create an educational board game, and send it to us at Teen-Toggle Games Pvt. Ltd, 307, Satija building, Colaba, Mumbai by July 2022. The top 10 winning board games will be featured on our international portal. Attractive scholarships for the winners!

Any other relevant information

Descriptors for Content NOTE—Dedicated marks at a level are to be awarded only if ALL descriptors match. If one or more descriptors do not match, the marks are awarded at a level lower.

@PROCBSE

CREATIVE WRITING SKILLS

97

Ans.

















2 marks All points included Well-developed with sustained clarity 1½ marks Almost all points incorporated Reasonably well-developed 1 mark Some points incorporated Fair attempt at developing ideas with some impact on clarity of response ½ mark Most of the points of the given task not incorporated Limited awareness of task development Expression—2 Marks Marks

Descriptors for Expression NOTE–Dedicated marks at a level are to be awarded only if ALL descriptors match. If one or more descriptors do not match, the marks are awarded at a level lower. •



2





















• •



1







• •



½









Highly effective style capable of conveying the ideas convincingly with appropriate layout of a formal letter viz. addresses, salutation, subscription, and ending. Carefully structured content with organised paragraphing presented cohesively. Highly effective register (formal tone and vocabulary), relevant and appropriate sentences for conveying the ideas precisely and effectively. Frequent clarity of expression most of the times, layout of a formal letter largely accurate. Ideas generally well sequenced and related to the given topic maintaining overall cohesion of ideas. Range of vocabulary is mostly relevant and conveys the overall meaning and the purpose of the writing. Inconsistent style, expression sometimes awkward, layout of a formal letter basically accurate. Sequencing of ideas is somewhat clear and related to the given topic attempting to maintain a general overall cohesion. Range of vocabulary is limited but manages to convey the overall meaning and the purpose of the writing. Expression unclear, layout partially followed affecting the format of the letter. Poor sequencing of ideas but ideas are related to the given topic in a disjointed manner exhibiting a lack of coherence of ideas. Very limited vocabulary or copying from the question. Accuracy -1 mark Descriptors for Accuracy







1 mark Spelling, punctuation and grammar consistently/largely accurate, with occasional minor errors, that do not impede communication. ½ mark Spelling, punctuation and grammar display some errors spread across, causing minor impediments to the message communicated. No Credit Frequent errors in spelling, punctuation and grammar, impeding communication.

@PROCBSE 98





















Oswaal CBSE Question Bank Chapterwise & Topicwise, ENGLISH LANG. & LIT., Class-X

Detailed Answer: 53-B, Ring Road Vadodara Gujarat January 12, 20XX The Manager, Teen Toggle Games Pvt. Ltd. 307, Satya Building Colaba, Mumbai Subject : Enquiry regarding Board Game Competition





























Dear Sir, Referring to the information flashed on the library’s notice board, I want to state that I wish to participate in the Board Game Competition. So, I require some more information regarding the same. Kindly provide the details of the following : 1. Rules of the competition for all the rounds 2. Scholarship details 3. Type of entry – solo or group entries 4. Mode of conduction – online or offline 5. Consolation Prizes [if any] Kindly specify the deadline. so that I can register myself before the last date of the competition. Thanking you in anticipation. Yours faithfully, Samina Zaveri Q. 2. You are Vipul/Apporva, living at D-424, Island Avenue, Ernakulam. You have seen an advertisement about a diploma course in French language soon to be organized by Maxwell Institute of Languages, Fort Road, Kochi. Write a letter in 100-120 words to the advertiser seeking all the relevant information like admission procedure, fee structure, duration of the course, timing of the class, transport facilities etc. 

[CBSE Outside Delhi Set-III 2020] (5 marks)





Ans. D-424, Island Avenue Ernakulam

02nd June, 2022









The Centre Head Maxwell Institute of Languages Fort Road Kochi



Subject : Enquiry about diploma course in French







Sir, With reference to your advertisement dated 24th May, I had come to know that you are providing diploma courses in various languages. I would like to join for the same in French language. I am waiting for class X result, so my am free for next two months. Kindly let me know the admission procedure, fee structure, duration of the course and timings for the same. Kindly inform at the earliest so that I can join the course without wasting time.



Thanking you





Yours sincerely Apoorva











Q. 3. You have lost your original certificates of class X and XII. You want to get their duplicates issued but you do not know the procedure. Write a letter to the Chairman, CBSE, Preet Vihar, enquiring about the fee to be deposited, mode of payment, time taken by the board for issuing duplicate certificates and any other formalities. You are Tarun / Taruna, 7/9, Kunj Apartments, Shimla (100-120 words). [CBSE Delhi Set-II 2019] (5 marks) 7/9, Kunj Apartments Shimla 3 April 20××

@PROCBSE

CREATIVE WRITING SKILLS

99



The Chairman



CBSE



Preet Vihar



Delhi



Subject: Query about issuing the Duplicate Certificates

Respected Sir,



Through this letter of mine, I wish to inform you that while travelling in a bus, I left my bag in a bus. The bag contained my original certificates of X and XII. Now I have to apply for a job in a government department. Please let me know the procedure of issuing the duplicate certificates, time taken by the board and the fees to be deposited. My registration no. is 4567/897/HP and I have filed an FIR regarding the loss of my bag.



Thanking you in anticipation





Please do the needful as early as possible as it is a question of my career.



Yours faithfully











Tarun Kumar Q. 4. As a health conscious person, you have noticed an advertisement in a newspaper on yoga classes in your neighbourhood. Write a letter in 100 – 120 words to the Organiser, Yoga for Public, R.K. Puram, New Delhi requesting him/her to send you information about the duration of the course and other relevant details. You are Shweta/Srikar of 15, R.K. Puram, New Delhi. [CBSE Outside Delhi, Set-I 2019] (5 marks) Q. 5. Your school has decided to take part in half marathon organised by an NGO in New Delhi. Write a letter in 100-120 words to the Area Manager of the NGO requesting permission to participate in the half marathon on 28 April 2016 starting from National Stadium. Request the NGO to provide your school with 200 T-shirts for boys and girls aged 15-17 years. You are School Captain, Manish Mehta, Sarvodaya Vidyalaya, Saket. [CBSE Outside Delhi, Set-II 2016] (5 marks)







l

[CBSE Marking Scheme, 2016]



































Other relevant information. Detailed Answer :





l



l

l









Ans. Suggested Value Points : State the purpose of the marathon. Seek permission politely. Request for 200 T shirts

Sarvodaya Vidhyalaya Saket April 18, 2016 The Area Manager XYZ NGO Karol Bagh New Delhi Subject : Permission to Participate in Half-Marathon Sir, It is matter of great pleasure that authorities of our school has decided to take part in the half-marathon organised by your reputed NGO. As the noble purpose of this marathon is to create awareness for ‘Clean City, Green City’, it will be our honour if you grant us permission to be a part of this noble cause. Further I’d make a request to provide us with 200 T-shirts for the boys and girls of our school, who are aged 15-17 years. I hope, we’ll receive the above mentioned consignment, along with the permission letter within this week so that the students may reach the National Stadium on April 25, 2016, at 5:30 a.m. sharp to join the marathon. We shall be highly obliged. Thanking you Yours sincerely Manish Mehta (School Captain)

@PROCBSE 100

Oswaal CBSE Question Bank Chapterwise & Topicwise, ENGLISH LANG. & LIT., Class-X

Q. 1. You are Vaibhavi Sinha, examination incharge, Goodway Public School, Aurobindo Road, Indore. You require 4 reams of white paper, 2 packets of carbon paper, one dozen registers, blue and red ball point pens (50 each). Place an order with Sunrise Stationary Mart, 12 Mall Road, Indore mentioning terms of payment, discount asked by you and delivery date. (100-120 words) [CBSE Delhi Set-I 2020] (5 marks) Ans. Goodway Public School Aurobindo Road Indore



























3. Letter for Placing Order

2nd April, 20×× The Sales Manager Sunrise Stationary Mart 12, Mall Road Indore Subject: Placing Order for Stationary Sir Thank you for the catalogue you had sent along with the latest price list. After going through, we would like to place an order for the following items. Product

Quantity

White Paper

4 reams

Carbon paper

2 packets

Registers

One dozen

Ball point pens (Red and Blue)





We attach herewith a draft of 50% advance payment. The balance payment will be made on delivery. We expect a 10% discount, which you have already promised. Please note that there are no damages during transportation. We expect delivery within 15 days as discussed earlier. Thanking You





Yours sincerely Vaibhavi Sinha Examining incharge



Q. 3. You are the librarian of Delhi Public School, Meerut. Write a letter to Oxford University Press, Ghaziabad, placing an order for books. Mention at least 4 titles of the books to be supplied immediately. Address yourself as Kalpana, (Word limit 100 – 120 words) (5 marks) 





50 each



























Ans. Delhi Public School, Bye-Pass Road, Meerut 2nd October, 20×× The Manager M/s Oxford University Press Civil Lines, Ghaziabad Subject : Supply of books for the School Library Sir, Through this letter we wish to place a bulk order for the supply of books to our school library. The list of books with their particulars is attached herewith. List of Books: S.No.

Name of Books

Authors or Publishers

Copies required

1.

History of India

L. Prashad

20

2.

Modern Trigonometry

Bansi Lal

30

@PROCBSE

CREATIVE WRITING SKILLS

3.

Modern Algebra

S.K. Sen

25

4.

The New Atlas of the World

Frank Bros.

30

5.

Wonders of India

Bhisham

15

6.

India Wins Freedom

M.A.Q. Azad

10

101



We shall not settle for less than 20% discount on the purchase of the books. Besides, we request you to give another 5% special discount that is usually granted to institutional buyers. It goes without saying that you will have to bear the mail and other expenses involved in the dispatch and delivery of books.





The above books must reach us in proper condition and order by 25th November. Hope our terms and conditions are acceptable to you. Thanking you















Yours faithfully Kalpana Q. 3. You are Ambica / Mohit, Librarian, High Scope Public School, Hauz Khas, Delhi. Write a letter to Jindal Publisher, Pratap Vihar, Delhi to place an order for English to Hindi dictionaries, illustrated children’s encyclopedia, fiction books, etc., for your school library. Request them for a catalogue, discount offered, mode of payment and time taken for delivery. (150-200 words) [CBSE Delhi Set - III, 2019] (10 marks) Ans. High Scope Public School Hauz Khas Delhi

3 April 20xx









The Manager Jindal Publishers Pratap Vihar Delhi



Subject: Order for Books

























Dear Sir We want to purchase the following books for our school library: (a) Oxford English to Hindi Dictionary 10 copies (b) Illustrated Children’s Encyclopedia 10 copies (c) Short Fiction Stories by Toby 10 copies Please let us know when you will be able to supply the books, the normal discount offered to the educational institutions, and mode of payment. Also please send us the catalogue of books available with you, so that we can place more orders accordingly.



Thanking you





Yours faithfully, Ambica Lamba (Librarian)

Q. 1. You are Akshay/Akshita, 23, Rampur Road, Delhi, a responsible citizen of your country. When you go on the road, you see that the people are becoming more and more intolerant of others, and drive their vehicles recklessly which sometimes leads to road rage. Write a letter to the editor of a local newspaper advising people

















to follow rules while driving vehicles. (100-120 words) Ans. 23, Rampur Road Delhi







4. Letter to the Editor

26th March, 20×× The Editor i-Next Delhi Subject : Concern over growing accidents of Road Rage in Delhi Sir,

[CBSE Delhi Set-III, 2020] (5 marks)

@PROCBSE 102

Oswaal CBSE Question Bank Chapterwise & Topicwise, ENGLISH LANG. & LIT., Class-X

Through the columns of your esteemed newspaper, I would like to draw the attention of the readers towards the serious issue of growing incidents of road rage in Delhi. Now and then, we come across the news of road rage where people start fighting on the roads on trivial issues. Everyone seems to be in great hurry on the road and moreover people are over stressed these days and their frustration is often displayed through such incidents. They start quarrelling on a matter of small collision or honking or even on the issue of parking. Generally, the fight starts verbally which turns into a physical fight. Many incidents of serious injuries and death of people have been reported so far and hence it needs immediate attention of everyone. The government should come forward with strict legal provisions to stop these and people should also learn to be patient and tolerant in order to make Delhi roads safe to travel.





Thanking you



Yours truly Akshay Q. 2. Slums are spreading more than the planned townships. The life in the slums is miserable. Crowded living, lack of drinking water, insanitary conditions, poor power supply etc. are causing a lot of suffering to the people. Write a letter in 100-120 words to the editor of a national newspaper drawing attention of the concerned authorities to this problem and asking for a proper solution. You are Udit/Udita, 26, Latouche Road, Kanpur. [CBSE Outside Delhi, Set-II 2020] (5 marks) Ans. 26, Latouche Road Kanpur













28th April, 20××



The Editor The Times of India Tughlaq Road New Delhi









Subject : Need to improve miserable living conditions in slums



Sir,



Through the columns of your reported newspaper, I would like to draw the kind attention of the concerned authorities towards the miserable living conditions of people in slums.



A country’s growth depends on the growth of its people. The people living in slums do not have basic amenities like clean drinking water, proper power supply and healthy food. Many can be seen ravaging through dustbins to get food. Lack of proper sanitation and living among such dreadful conditions lead to spread of many diseases. They are illiterate and thus lack job opportunities. They live in congested spaces where proper breathing is also unavailable.



Only if the slum dwellers get the basic amenities and education, the nation can be said to have truly progressed.



Thanking you



Yours sincerely



Udit Q. 3. You are Tanvi/Shirish of Safdarjung Development Area, New Delhi. You are extremely disturbed about the attacks on old people living alone. Write a letter in 100-120 words to the Editor, ‘The New Indian News’ making people sensitive to the problem. Make suggestions to the people living alone and the authorities to ensure that [Delhi Set-I, 2019] (5 marks) such attacks do not take place.





Q. 4. You are Hema/Hemant, 14/9, Hill Apartments, Jaipur. You are not satisfied with the facilities for tourists in Jaipur. Write a letter in 150 – 200 words to the editor of a local daily giving suggestions on ways to improve the facilities for tourists. [CBSE Outside Delhi Set-II, 2019] (5 marks)



Ans. 14/9, Hill Apartments Jaipur











16 July 20xx The Editor The Times of India Jaipur These questions are for practice and their solutions are available at the end of the chapter

@PROCBSE

CREATIVE WRITING SKILLS

103



Subject : Insufficient facilities for Tourists



Respected Sir,







Through the columns of your reputed newspaper, I would like to raise my voice against the insufficient facilities provided to the tourists of Jaipur. Everyone is well aware that Jaipur has many historical places which attract tourists not only from India but abroad as well. But when they reach here, they are not properly guided. They are mishandled by crooks who rob them of their valuables, thus leaving a bad name to our country. The hotel rooms are not available at reasonable rates. They have to pay a hefty amount for it. Most of them come on package tours but they are refused the facilities mentioned in their package. It is our prime duty to look into the matter as it may hit the tourism industry badly. The tourism industry should be well-managed so that the tourists leave Jaipur with a happy note and return again. It is my humble request to the concerned authorities to look into the matter and do the needful.



Thanking you





Yours sincerely Hemant







Q. 5. Increase in the number of private vehicles coupled with lack of civic sense has led to several road accidents in your city. Write a letter in 150-200 words to the editor of a local daily urging people to use public transport and to have patience while driving. You are Ram/Rama, 4 Raja Road, Karnal. [CBSE Outside Delhi Set-III 2016] (5 marks)











l l l l











Ans. Suggested Value Points : Too many vehicles on the road/ Roads are congested/Traffic Jams. Too many accidents. Use Public transport –more comfortable. It saves energy and money.



l

[CBSE Marking Scheme, 2016]

































Any other relevant information. Detailed Answer : 4, Raja Road, Karnal July 5, 20XX The Editor The Hindustan Karnal Subject : Use of Public Transport Sir,

Kindly allow me to make an appeal to public through the columns of your esteemed newspaper about the advantage of public transport usage. Everybody can observe easily that the increase in the number of private vehicles on the road is the major cause of air as well as noise pollution. More vehicles mean more congestion and traffic jams because public has become devoid of civic sense. Everybody tries to move first and fast which not only increases the chance of accidents, but also causes a lot of noise pollution because people continuously blow horn during the waiting moments. An easy and quick solution of the problem is that we should make more and more use of public transport. It’ll prove more comfortable as it will save both our energy and time and we can use that energy at our work place. Moreover, I want to make an appeal to the vehicle-owner to practice patience while driving because our life is very precious. It is better to reach late than never. This is a matter of great concern and I hope you’ll publish my opinion in your esteemed newspaper. Yours truly Ram

@PROCBSE 104

Oswaal CBSE Question Bank Chapterwise & Topicwise, ENGLISH LANG. & LIT., Class-X



















Commonly Made Errors Sometimes students make spelling errors and errors related to tenses and subject-verb-concord. In many cases, the format of the letter is incorrect and incomplete. Many students lack adequate analysis of the topic.















     



Answering Tips Learn the correct format for writing formal letters. Remember an editor can only publish your letter in the newspaper. Never ask him to help you in person. Discourage ‘Examination Hall’ as the address and ‘xyz’ as signature. Insist on writing in paragraph. Students should learn to address the issue directly instead of rambling. Firstly, read the question and analyze the specific points pertaining to the content.

Topic-2

Analytical Paragraph Writing Concepts Covered



Chart,



Graph,



Cue,



Map

Ø

Ø

Revision Notes Analytical paragraphs are the paragraphs where a comparative study of two or more things is done. The data is given in the form of pie charts, line-graphs, bar graphs, maps etc. and it is interpreted to show how the things or the trend is different or has become different over the time. They show choices and preferences also. The following expressions can be used while writing the analytical paragraphs:

all......

as many as......

lesser......

almost all......

some......

than

a majority......

almost no......

sharp increase......

one third (etc.)

fewer......

remain unchanged......

hit a high of......

an increase/a decrease

overall......

Vocabulary for Line Graph : 1. Upward trend

2. Downward trend

3. Time Phrases

4. Other vocabulary

• rise/a rise

• decrease/ a decrease

• over the next three days

• fluctuate/ a fluctuate

• increase/ an increase

• drop/a drop

• three days later

• dip/a dip

• climb/a climb

• fall/ a fall

• in following three days

• remain steady

• grow/ a growth

• decline/ a decline

• the next three days show • remain stable • over the period

• remain unchanged



• from____To___ / between • level off ____ and ____ • the last final year

• peak at

• at the beginning/ end of • reach a peak of the period 

• go up

• hit a high/low of • bottom out • plateau

@PROCBSE CREATIVE WRITING SKILLS

Points to remember:

Ø

Ø





 Read the question carefully

 Summarise the information by selecting the main features.

Scan to know more about this topic



 You can make relevant comparisons (only if needed)

105



 Do not add your own information or data.

 Avoid repetitions.

Format of an Analytical paragraph

Ø

Ø



 Your analysis should support the given figure or data.

Analytical Pragraph Writing



 Heading

 [explain introduction in one or two lines the subject of the given data]

 [Body: explain in detail what it is about, make comparisons, divide into sub- paragraphs, if required]

 [Conclusion: conclude the paragraph giving the overall view or summary of the given data].

Q. A survey was conducted in a town where the views of people were collected on cigarette smoking. Giving below is a pie-chart showing the results of the survey. Write a paragraph in about 100-120 words interpreting the data. (5 marks)

Ans.











Example 3

Heading introduction

Conclusion

Cigarette smoking The above pie chart shows the result of the views of people where a survey was conducted whether smoking should be banned or not. There were only a few people i.e., 5% who were of the view that no restrictions should be imposed on cigarette smoking. Almost one fifth, 20% of the people opined that it should be banned at all public places so that passive smoking can also be taken care of. As many as 30% people were of the view that cigarette smoking should be banned but only at shopping malls or big stores. But there was another 45% of opinion i.e., majority of the people showed an interest in imposing blanket ban on cigarette, as it is hazardous to health. Overall, people are aware that smoking is injurious to health so it should be banned but everyone has different views.

Body

@PROCBSE 106

Oswaal CBSE Question Bank Chapterwise & Topicwise, ENGLISH LANG. & LIT., Class-X

Q. Study the following bar graph depicting major exports from Medland in the current year. Write a paragraph interpreting the data in 100-120 words. (5 marks)

Ans. The bar graph given above depicts major exports from Medland. The export trends convey that Medland exported slightly less this year as compared to last year. Concerning foodstuffs bananas and rice showed a little less units than the previous year. On the other hand, the units of palm oil and peanuts exported were much more than last year. Almost the same amount of cocos were exported in both the years. As regards non-edible goods, the number of guns and iron ore was much more than last year. However, the graph of cotton goods shows the same number in both the years.









Example 4

Q. A survey was conducted on the population of Rhinoceros unicorn is between 1910 to 2005. Interpret the line graph given below in about 100-120 words. (5 marks)

Ans. A survey was conducted to find out of the population of Rhinoceros Unicornis between 1910 to 2005. The result showed only 125 rhinoceros in early 1900’s i.e., 1910 which reached almost 740 in 1959. Then the number plummeted down and touched 600 in 1964. It kept fluctuation and ranged between 600-750. The number again rose in 1966 but did not cross 750. By 1983 it has again reached 750. But after that there was a considerable increase in the number which touches 1750 in the year 1990. The graph then shows a plateau till 2000 and the number again rose to 2100 by the year 1995. There is hardly any increase in the next three years. The graph shows a sharp rise in the number of rhinoceros in 2000, when it touched 2500. It remains stable for the next two years, but again the number again decreased by almost one hundred in 2002, when it was an estimated 2400.









Example 5

@PROCBSE

CREATIVE WRITING SKILLS

107





Q. 1. Study the concept chart from the self-help magazine section of a monthly publication.

(5 marks)

Write a paragraph in not more than 120 words, analysing the listed responses to the situation when one faces setbacks. [CBSE SQP, 2021-22] 



The question tests the following writing LOs:









▪ ▪ ▪ ▪

convey ideas convincingly using appropriate language organize the content and structure the ideas logically, sequentially, cohesively use a range of vocabulary and sentence structure appropriate to the content and context use of functional language to show comparison, contrast, emphasis, conclusion etc.

Guidance



Award 3 marks for content— Topic sentence identifying the two responses to setbacks in the concept map – ½ mark ▪ For Instance: Setbacks often leave one with a feeling of disappointment and sometimes even worse. / The concept chart given below displays two ways to handle setbacks, for the better or for the worse/ the two ways of processing setbacks – healthy and unhealthy. ▪ Any 2 points of contrast/ comparison with evidence – 2 marks For instance: Healthy processing helps one learn from and let go the emotions inside. On the contrary, inability to process leads to a block, self-criticism and self-doubt. Or An optimistic way of seeing a setback processes the ability to feel, reflect, learn and align. On the other hand, negative perspective of a failure can lead to listlessness, diverted attention, excessive rumination and nonacceptance of failures. ▪ Concluding sentence, tied to the content of the topic sentence, showcasing a perspective/ rationalising the importance of healthy processing of setbacks encountered. – ½ mark For instance, one might want to point to the fact that the responding to setbacks negatively leads to a vicious cycle of undesirable feelings: It is evident that experiencing a setback leads to feelings of inadequacy and incompetency along with an unwillingness to accept situations. Responding to setbacks the positive way is empowering as it leads to success. Note—Just listing concept map matter without evidence of analysis carries no credit.

Ans.







SUBJECTIVE TYPE QUESTIONS

@PROCBSE 108

Oswaal CBSE Question Bank Chapterwise & Topicwise, ENGLISH LANG. & LIT., Class-X







[CBSE Marking Scheme, 2021-22]

Detailed Answer:

Setbacks Setbacks often leave one with a feeling of disappointment and sometimes even worse. The given concept chart displays two ways to handle setbacks, for the better or for the worse — healthy and unhealthy. Healthy processing helps one learn from and let go the emotions inside. One should believe in oneself. In all of your jobs, expect that you will do well at your job. Your co-workers enjoy working with people with a ‘can-do’ attitude. Think about what you learned or gained from each new situation. An optimistic way of seeing a setback processes the ability to feel, reflect, learn and align. On the contrary, inability to process leads to a block, self-criticism and self-doubt often times, we tend to wait for the setback to pass before we can move on. Very few times do we actually take up the deal with the failure head on. Acknowledging it and taking proactive mental and physical action to minimize the effect is what ‘handing’ a failure gracefully really means. Hence, responding to setbacks the positive way is empowering as it leads to success. Q. 2. The chart below displays data about the number of digital devices purchased in Rishu nagar across the years 2015-2019. Write a paragraph analyzing the given data. (5 marks) 





[CBSE SQP, 2020-21] Q. 3. Read the following excerpt from an article that appeared in the magazine section of a local daily: The ban on single-use plastic is impractical. The purpose of articles like bags and packaging is ultimately to make human life easier. Plastic articles do this well, so they shouldn’t be banned. Write a paragraph to analyze the given argument. You could think about what alternative explanations might weaken the given conclusion and include rationale / evidence that would strengthen / counter the given argument. (5 marks) 



























































Award 2 marks for organisation & expression— ▪ ½ mark— Inclusion of a single paragraph organisation with a suitable topic sentence supporting sentences and a suitable concluding sentence. No requirement of a title (because the purpose is analysis, not publication). No penalty if title is written. ▪ ½ mark— use of appropriate functional language to show comparison/contrast & emphasis: Comparison/ Contrast: in contrast with, in comparison to, on the contrary, however, whereas, as opposed to, while, a striking difference, a noticeable difference, despite etc. Emphasis: in other words, /especially/ specifically/ to emphasise/ to demonstrate/such as/in particular etc. full credit 1 mark to be allotted if the functional language has been used consistently partial credit ½ mark to be allotted if the functional language has been used occasionally/sparingly No credit of marks if functional language is missing (not used at all) ▪ ½ mark— Unity of ideas in the complete paragraph with ideas arranged logically –sentences within paragraph follow expected organizational frameworks* *[Categorical – in order of importance; Evaluative - a problem is introduced, and the pros and cons are weighed; Comparative – similarities and differences; Cause and Effect; Descriptions-from general to specific attributes] Accuracy— Deduct from the overall score if the error density is high as this impacts the communicative function. ½ mark for a total of 2-3 spelling and grammatical errors 1 mark for a total of more than 3 spelling and grammatical errors

These questions are for practice and their solutions are available at the end of the chapter

@PROCBSE

CREATIVE WRITING SKILLS











Argument for excerpt --Ban on Plastic Competency applied for overall expression Single paragraph Use of appropriate functional language required to show comparison/ contrast/ emphasis etc. to achieve cohesion • Accuracy – Introductory sentence Competency applied for content • Stating the given assumption in the question as the introduction. – Arguments rationalizing against/in favour of assumption • At least 2 clearly listed arguments countering/ favouring the premise. Note – focusing and extending one particular trend would be considered as one trend only. – Concluding line • Reiterating main inference/position. [CBSE Marking Scheme, 2020]



Ans. (A) – • •

109































Detailed Answer:

Ban On Plastic













Single use plastic causes more harm to the environment than benefitting it. Its toxic contents are harmful for all whether they are human beings, animals or marine creatures. In fact they cause a decline in the healthy state of those also who produce it as it leads to many illnesses. The government has decided to put a blanket ban on single-use plastic articles which is a wise step. It is going to be helpful for our environment which is in a declining state everyone must join in this campaign to save our planet and to make it pollution free. More of environment friendly products should be used. The ban on these products is not impractical. There is no denying the fact that these products make our life easier but we should come up for the welfare of all and not be selfish. The ban will be useful in the long run. Q. 4. Look at the data given below which shows the changes that have occurred between 1995 to 2010 in Bilaspur regarding the women education. Write a paragraph in about 100-120 words to record the changes by interpreting this data. (5 marks) Female Education in Bilaspur

2010

1.

No. of Girl’s school

10

15

2.

No. of Women Colleges

3

4

3.

No. of students in schools

10,000

16,000

4.

No. of students in colleges

6000

10,000

5.

Literacy rate

30%

55%

6.

Distance Education Centres

Nil

03





Women Education Women education can play an important role in the development of the country. A survey was conducted in Bilaspur to find out the changes that have occurred between 1995 to 2010 regarding the women education. It was found that there was a rise in the number of girl’s school. Earlier, in 1995 there were only 10 schools. A slight increase was seen in the number which rose to 15 in 2010. Not much growth was recorded in the number of women colleges which rose from three in 1995 to four in 2010. However, a significant growth was recorded in the number of students in school which increases from 10,000 in 1995 to 16,000 in 2010. There was a sharp increase of 4000 students in colleges also. The percentage of literacy rate increased steadily from 30 to 55% in these years. There were no Distance Education Centres at the beginning of the period. However, three such centres were found by the end of 2010. We can conclude by saying that an overall increasing trend was seen in the women education. Q. 5. Every four seconds a new baby is born in India while the death ratio is very low. If the present trend continues serious consequences will be seen. Pressure is growing on the civic amenities. Over population has become a menace. Being an alert citizen, you feel strong about it and intend to spread awareness. In this regard you decide to write a paragraph on, “Over Population - A Hazard” in about 150-200 words, taking help from the verbal input and visual stimulus given below : (10 marks) 













Ans.

1995

@PROCBSE 110

Oswaal CBSE Question Bank Chapterwise & Topicwise, ENGLISH LANG. & LIT., Class-X

Slums have sprung up everywhere - pressure is growing on basic amenities - public transport ply overcrowded - long queues in hospitals, at railway stations, shops, gas agencies, bus stands - problem of unemployment solution is family planning, youth need to be made aware of dire consequences. Over-Population – A Hazard



Ans.

Over population has become a menace in present times. Every four seconds, a new baby is born in India while the death ratio is very low. The rise in population is taking its toll on the normal life. Pressure is growing on the basic amenities. The public transport services ply overcrowded with no space to breathe fresh air. Wherever you go, there are long queues. People can be seen waiting whether it is a hospital, a railway station, shops, gasagencies, bus-stands and in fact almost everywhere. Over population has led to unemployment also which further leads to social unrest among the youth. The cases of robbery, violence, eve-teasing are on a rise. The only solution to this problem is family-planning. People must be made aware of the dire consequences of over-population. Awareness should be spread among the masses through campaigning or street plays so that people may understand that over-population must be checked. Impart family planning, empowering women, making education entertaining and attractive, government incentives are some of the suggestions that control over population.







Commonly Made Error Students sometimes misinterpret the graphs or the visuals given and analyze the paragraph in a wrong manner













Answering Tips Students should study the graph well and go through the minute details. Then they should cover them in the paragraph Students must not add any information from their end

Solutions for Practice Questions (Topic-1)







































3 Letter to SHO requesting action against flouters of quarantine rules Format = 1 mark • An organised format structure is expected. • Absence of even one aspect shall result in credit of ½ marks only. • Full credit of 1 mark requires all listed aspects of format. • Zero credit if just format is listed without letter content. Suggestive Points: Complaint about some residents not following the quarantine rules. – There is an evident disregard to the rules of social distancing. – People roaming without masks in the colony. – No staying indoors. – The Resident Welfare Association is not taking any steps to prevent such behaviour. – Any other relevant. Impact – Putting everyone’s life at risk through their casual behaviour. – Aggravates mental stress. – Any other relevant. Appeal – Request to look into the matter at the earliest. – Take immediate action. – Any other relevant point. [CBSE Marking Scheme, 2020] 





1. Letter Of Complaint

@PROCBSE

CREATIVE WRITING SKILLS

111



Ans.3. Satya Nagar Colony

Bhubaneswar



24 May, 20XX



The Station House officer



Satya Nagar Police Station



Bhubaneswar, Odisha



Subject: Complaint Against Flouters of Quarantine Rules



Sir,



Through this letter of mine, I would like to draw your attention towards some residents of our colony who are repeatedly flouting quarantine rules which have been laid out during the outbreak of COVID 19 pandemic.



There is an evident disregard to the rules of social distancing. Many people are roaming without masks in the colony and do not stay indoors. Despite repeated requests, they are turning a deaf ear to all the appeals. The RWA is also not taking any steps to prevent such behavior. It is a matter of great concern as they are putting everyone’s life at great risk through their casual behavior. It aggravates mental stress also.



I request you to look into the matter at the earliest. I earnestly hope that you will take immediate action against these people. I hope for a positive response from your end.



Thank you.



Yours sincerely



Tarun

2. Letter Of Inquiry





















Ans.4. R.K Puram New Delhi 7 August 20×× The Organiser Yoga For Public R.K. Puram New Delhi Subject : Enquiry About the Courses Respected Sir, With reference to your advertisement in ‘The Times of India’ dated 5 August 20xx, I would like to tell you that I wish to join the yoga classes organised by you. Kindly furnish the following details :



















• • • • •

Duration of the course Timings of the classes No. of persons in each batch Any special class for girls Fees structure

I am awaiting an early reply from your end so that I may join as early as possible. Thanking you Yours sincerely Shweta

@PROCBSE 112

Oswaal CBSE Question Bank Chapterwise & Topicwise, ENGLISH LANG. & LIT., Class-X

4. Letter to the Editor 















































































Ans 3. Objective: To use an appropriate style and format to write a formal/an informal letter Note: No marks are to be awarded if only the format is given. Credit should be given for the candidate’s creativity in presenting his/her own ideas. Format (i) Sender’s address (ii) Date (iii) Receiver’s address (iv) Subject/Heading (v) Salutation (vi) Closing Value points : • old people live alone • vulnerable to attack • increase in number of attacks on the elderly • live in joint families • active Resident Welfare Associations • install alarms and cameras at home and locality • police verification of domestic help • law and order to be tightened • senior citizens to have contact with area SHO • regular police patrolling in the area [CBSE Marking Scheme, 2019] Detailed Answer: 13, Safdarjung Development Area New Delhi

2 April 20××







The Editor The New India News New Delhi



Sir Through the column of your esteemed newspaper, I want to sensitize people about the increase in number of attacks on older folks. Old people are generally physically weak. They are quite vulnerable to attacks. The result is that they can’t overpower the rogues in the war of muscles, who generally come to loot them. Any resistance from the old people can prove detrimental to their lives. In the locality of Safdarganj Area many incidents of theft, snatching of jewels and brutal attacks on the old people have already happened. It is the duty of our government to protect the lives of their senior citizens. Provision should be made to install alarms and cameras in homes as well as in localities. Police verification of domestic help is a must. The regular police patrolling can also be of great help. Thus, all measures for their safety should be adopted to save old people. Though repeated requests in this regard have been made to the concerned authorities but no headway has still been done. Perhaps the lines in your esteemed paper may shake the authorities from their deep slumber. Thank you Yours truly Tanvi Bansal 
















Subject : Concern over Attacks on Old People

@PROCBSE

CREATIVE WRITING SKILLS

113

Solutions for Practice Questions (Topic-2) Analytical Paragraph Writing (10 marks)



Ans. Analytical Paragraph writing

4 marks for content + 1 mark for format



5 marks for expression (1 mark-grammatical accuracy, appropriate words and spellings + 1 mark-cohesion via connecting ideas, logical progression & coherence through relevance of ideas and style)



(A) – Data about purchase of digital devices



Competency applied for overall expression



• Single paragraph 



• Use of appropriate functional language required to show trend progression and comparison (more/ less/ increase/ decrease/ stable etc.) to achieve cohesion



• Accuracy



– Introductory sentence



Competency applied for content



• Paraphrase Q. information



– Identification of main trends



• At least 4 clearly identified trends



Note – focusing and extending one particular trend would be considered as one trend only.



– Concluding line



Stating main inference from the data provided. [CBSE Marking Scheme, 2020-21]





Detailed Answer: Purchase of Digital Devices According to the given data, a large number of digital devices were purchased in Rishu nagar across the years 2015-2019. The data indicates that more smart phones were purchased in 2015 as compared to the computers which numbered approx-35,000 in 2015. There was a gradual size in the purchase in 2016 when the number of smart phones rose to 60,000 and computers to 59,000. The trend suddenly changed and a sharp rise was seen us. The purchase of computers in 2017 which again dropped to approx. 45,000 in 2018. The year showed a decline in the purchase of smart phones as well. But the year 2019 brought hope and cheer for the sellers when there was again an upward trend in both smart phones and computers. To sum up it can be said that the purchase of digital devices across these fire years showed the incline of people towards the hi-tech devices.



Ø

REFLECTIONS



Ø



Ø

Ø





‘’Through the Columns of your esteemed newspaper, I would like to draw the attention of the readers towards…’’ Were you able to identify the format of letter to the editor based on the above mentioned lines? Were you able to use the appropriate style to write a formal letter? Were you able to use the given Map/Chart/Cues to write a comprehension paragraph? Were you able to compose a crisp, concise paragraph after interpreting the data?

qqq

@PROCBSE 114

Oswaal CBSE Question Bank Chapterwise & Topicwise, ENGLISH LANG. & LIT., Class-X

SELF ASSESSMENT PAPER - 03 Max. Marks: 30



Max. Time: 1 hour



Q. 1. The condition of the public park in your locality is miserable. Write a letter to the editor of a local newspaper with the help of the points given below in 100-120 words. You are Ranjit/Reshma.

l



l



l



l

Poorly managed parks Heaps of garbage everywhere Boundary wall broken Very dirty animals resting there













(5 marks) OR You are Aryan/Amrita of C/43, Hauz Khas, New Delhi. You have read a report in a magazine mentioning that there are 12 million child workers in your country. It is a national problem. Write a letter in about 100-120 words to the editor of ‘The Indian Express‘, giving your views on why child labour exists and how it can be abolished. (10 marks) Q. 2. You came across the following advertisement published in the public interest by the Ministry of Health. You feel that despite rising taxes and restrictions against smoking in India, there are still 260 million smokers in the country and the number is still rising. Taking ideas from the advertisement with your own ideas, write a paragraph for your school magazine in not more than 100-120 words. (5 marks)

WHEN YOU SMOKE YOU BURN YOUR LUNGS !!! Take deep breath and think FACTS : SMOKERS HAVE • 20-25 higher risk of developing lung cancer • 2-3 times higher risk of having a heart attack • 3 times higher risk of sudden death PASSIVE SMOKERS • Risk of having bronchitis, pneumonia, asthma and reduced rate of lung growth. If current patterns continue, Smoking will cause some 10 million deaths every year SMOKING BRINGS YOU Brain strokes, heart attack, lung cancer, sperm deformities and impotence in males. Cancer of cervix and infertility in females. MINISTRY OF HEALTH AND FAMILY WELFARE







Q. 3. Write a letter to the editor of ‘The Hindustan Times’, Kasturba Gandhi Marg, New Delhi advocating the need for people to obey traffic rules for safety, to be law-abiding citizens, to wear helmets and to put on seat belts when driving on road. (5 marks)

@PROCBSE CREATIVE WRITING SKILLS

115

Q. 4. Look at the data given below which shows the changes that have occurred between 1995 to 2010 in Bilaspur regarding the women education. Write a paragraph in about 150-200 words to record the changes by interpreting this data. (5 marks) 







1995

2010

1.

No. of Girl’s school

10

15

2.

No. of Women Colleges

3

4

3.

No. of students in schools

10,000

16,000

4.

No. of students in colleges

6000

10,000

5.

Literacy rate

30%

55%

6.

Distance Education Centres

Nil

03

Q. 5. You are Somya/Shekhar of 3B, Sector II, Rajendra Nagar, New Delhi. You have observed that the subways in New Delhi are seldom used by the pedestrians. You have decided to write a letter to the editor of ‘The India Daily Times’ highlighting the need of creating awareness about the importance of such ways and their proper maintenance. Based on the points given below write the letter in about 100 words. (5 marks) Q. 6. The graph below proves that even in the 21st century the status of women is not equal to that of men when it comes to occupying high positions in various occupations. Read the graph carefully and write its interpretation in 150-200 words. (5 marks)

10 0

(13.8) World

(3.6) Arab States

20

(11.0) Sub-Saharan African

30

WOMEN IN NATIONAL PARLIAMENT (% by region, both houses combined)

(13.5) Pacific

40

(14.1) Europe (excluding western countries)

50

(15.3) Asia

60

(15.3) America

70

(38.1) Western countries













Female Education in Bilaspur

@PROCBSE 116

Oswaal CBSE Question Bank Chapterwise & Topicwise, ENGLISH LANG. & LIT., Class-X

@PROCBSE

Syllabus





Section C LITERATURE





Literature Textbooks and Supplementary Reading Text V. Reference to the Context I. One extract out of two from Drama / Prose. (5+5 = 10 Marks) II. One extract out of two from poetry. Multiple Choice Questions / Objective Type Questions will be asked to assess inference, analysis, interpretation, evaluation and vocabulary. VI. Short & Very Long Answer Questions 30 Marks I. Four out of Five Short Answer Type Questions to be answered in 40-50 words from the book FIRST FLIGHT (4×3=12 marks) II. Two out of Three Short Answer Type Questions to be answered in 40-50 words each from FOOTPRINTS WITHOUT (2×3=6 marks) FEET.) III. One out of two Long Answer Type Questions from FIRST FLIGHT to be answered in about 100-120 words each to assess creativity, imagination and extrapolation beyond the text and across the texts. This can be a passage-based question taken from a situation/plot from the texts. 6 marks IV. One out of two Long Answer Type Questions from FOOTPRINTS WITHOUT FEET on theme or plot involving interpretation, extrapolation beyond the text and inference or character sketch to be answered in about 100-120 words. 6 marks

CHAPTER

1

A LETTER TO GOD

– By G. L. Fuentes

Revision Notes



Introduction

Scan to know more about this topic

‘A Letter to God’ is a story of extreme faith in God. A natural calamity leaves Lencho, a hardworking farmer, in distress. The farmer writes a letter to God asking him to send money. The chapter concludes in a very ironic situation.

Lencho was a poor but hardworking farmer. His house was on the top of a hill and that was the only house in the valley. He hoped for a good crop but his fields needed rain or at least a shower. He waited for the rain and it did come. One day, it started drizzling. In the beginning, Lencho felt the falling rain drops as ‘new silver coins’ falling from the sky. But gradually, the rain transformed into a hail storm. The hailstones fell for an hour. It destroyed his crops completely. Lencho’s soul was filled with sadness. He thought that his family would go hungry for the whole year unless they





Summary

found someone who could help them. Suddenly his heart was filled with hope. He had firm faith in God. Lencho knew how to read and write. The following Sunday, he wrote a letter to God mentioning his destroyed crops and asked A Letter to God him for money so that he could sow his field again. Then he put the letter in an envelope, addressed it to ‘God’ and went to the post office. He affixed a stamp on it and dropped it in the mail-box. The postman, who took the letter out of the mailbox saw the letter and showed it to the postmaster. Everyone in the post office had a hearty laugh. But immediately the postmaster realised the man’s predicament and unshakable faith in God. He decided to help the man. He discussed with his

@PROCBSE 118

Oswaal CBSE Question Bank Chapterwise & Topicwise, ENGLISH LANG. & LIT., Class-X

The postmaster took out the letter from the mailbox and opened it. Lencho had requested God to send the rest of the money i.e., thirty pesos as he had received only seventy pesos. Lencho had a feeling that the people at the post office had cheated him by taking out some money from God’s envelope. So, he wrote that God should not send money through mail as according to him, the post office employees were a bunch of crooks.

Key Words





1. Peso: currency of several Latin American Countries 2. Amiable: friendly and pleasant 3. Contentment: satisfaction



colleagues and all of them decided to part with some money for an act of charity. They collected money, put it in an envelope and addressed it to Lencho. This letter contained a single word in the form of a signature: God. The next Sunday, Lencho went to the post office. The postmaster handed him the letter. Lencho was not at all surprised on receiving a letter from God. He had unshakable faith that God would reply in the form of money and he did receive it. He opened the envelope but became angry on counting the money. There were only seventy pesos in the envelope, whereas he had asked for one hundred. He knew God could not have made a mistake. Immediately, he wrote another letter to God, put it in the mail-box and went out.

OBJECTIVE TYPE QUESTIONS Extract Based Questions



(5 marks each) (1)

heavy rain

(i)

light rain that falls in very fine drops

(2)

thunderstorm

(ii)

very heavy tropical rain

(3)

drizzle

(iii)

it's coming down quite strong and you get very wet very quickly

(4)

torrential rain

(iv)

really heavy rain that comes very suddenly

(5)

downpour

(v)

is a violent, short-lived weather disturbance associated with lightning, thunder and strong, gusty winds.







I. Read the following extract and answer the questions/ complete the sentences that follow: (5×1=5)

Q. 1. Based on the details of the house's location, how can it best be described?





(B) Imposing

(A) (1)-(ii), (2)-(iv), (3)-(v), (4)-(i), 5-(iii)

(D) Unique

(B) (1)-(iv), (2)-(i), (3)-(iii), (4)-(v), 5-(ii)

Ans. Option (C) is correct.

(C) (1)-(v), (2)-(iii), (3)-(iv), (4)-(ii), 5-(i)









(A) (B) (C) (D)



Ans. Option (B) is correct.









Explanation: When seen from a height, the corn field flowers appeared like dots spread or scattered all across the field. Q. 3. Lencho wished for a downpour or a heavy shower. Pick the option that correctly lists the correct match for different kinds of rain.

(D) (1)-(iii), (2)-(v), (3)-(i), (4)-(ii), 5-(iv) Ans. Option (D) is correct. Explanation: The correct sequence from light to heavy rainfall is – drizzle (meaning very few and fine drops of rain), heavy rain (meaning one can get easily wet in that rain), downpour (meaning heavy rain that comes suddenly and one does not have chance to reach safe dry place), torrential rain (meaning very heavy and continuous rain for a long time) and thunderstorm (meaning violet rainfall accompanied with lightning, thunder and forceful winds).

not a single flower was bigger than a dot. the flowers were scattered across. the flowers were shaped like dots. the flowers had shrunk in size.







Q. 2. The field of corn dotted with flowers means that:



Explanation: The words “only one in the entire valley” mean solitary.

Q. 4. Based on the given extract, what is Lencho NOT likely to think while looking at his field?





(C) Solitary

rain,





(A) Majestic











The house – the only one in the entire valley – sat on the crest of a low hill. From this height one could see the river and the field of ripe corn dotted with the flowers that always promised a good harvest. The only thing the Earth needed was a downpour or at least a shower. Throughout the morning, Lencho, who knew his fields intimately-had done nothing else but see the sky towards the north-east. [CBSE-QB, 2021]

@PROCBSE

A LETTER TO GOD



















Q. 3. Pick the option that lists the option corresponding to—'with a blow of his fist.'





(C) Option (ii) and (iv) (D) Option (iii) and (v) Ans. Option (C) is correct. Explanation: As Lencho had got some money, supposedly from God, the postmaster expected him to be thankful (full of gratitude) and delighted (full of elation). He had no reasons to be sorrowful (sad), disappointed (disheartened) and shocked (as he himself had written to God to send some money).

Q. 5. Which quote supports the idea in the given extract? (A) "Farming is a profession of hope." (B) "I would rather be on my farm than be emperor of the world." (C) "Farming looks mighty easy when your plough is a pencil and you're a thousand miles from the corn field." (D) "Those too lazy to plough in the right season will have no food at the harvest." Ans. Option (A) is correct. II. Read the following extract and answer the questions/ complete the sentences that follow: (5×1=5) When he finished, he went to the window to buy a stamp which he licked and then affixed to the envelope with a blow of his fist. The moment the letter fell into the mailbox the postmaster went to open it. It said: "God: Of the money that I asked for, only seventy pesos reached me. Send me the rest, since I need it very much. But don't send it to me through the mail because the post office employees are a bunch of crooks. Lencho." [CBSE-QB, 2021] Q. 1. '...bunch of crooks.' Pick the option that DOES NOT collate with 'bunch of, correctly.













(A) Option (i) (B) Option (ii) (C) Option (iii) (D) Option (iv) Ans. Option (D) is correct. Explanation: Lencho is hopeful that after the rains, he will have a good harvest which will be more than enough than his personal requirement of food. So, he will surely not be hoping to have a scarce harvest which will leave him arranging for more food.

119

















































Q. 2. What was the most likely response that the postmaster expected in Lencho's second letter? (i) sorrowful (ii) gratitude (iii) disappointment (iv) elation (v) shock (A) Option (ii) and (v) (B) Option (i) and (iii)



















Q. 5. Pick the most suitable quote for this extract. (A) "It is easier to fool people than to convince them that they have been fooled." - Mark Twain (B) "Real knowledge is to know the extent of one's ignorance."- Confucius (C) "You see a person's true colours when you are no longer beneficial to their life."- Anonymous (D) "True generosity means accepting ingratitude." - Coco Channel Ans. Option (D) is correct. Explanation: Generosity means doing good without expecting good in return. So accepting ingratitude or thanklessness is also a part of true generosity.







(A) Option (i) (B) Option (ii) (C) Option (iii) (D) Option (iv) Ans. Option (D) is correct. Explanation: A chain of islands is called archipelago.













Q. 4. Lencho's letter included: (A) details of his problems. (B) description of the post office. (C) belief of being looted. (D) List of further demands. Ans. Option (D) is correct. Explanation: He thought that God is already aware of his problems, so he need not bother him with the details. As he thought that God has fulfilled some of his demands (or rather all demand of 100 pesos out of which 30 had been retained by post office staff, he told God about what more he needed.

















(A) Option (i) (B) Option (ii) (C) Option (iii) (D) Option (iv) Ans. Option (D) is correct. Explanation: Option (i) is Good Luck or Good Job Done. Option (ii) is to show fist for punching out of anger. Option (iii) is to hit fists or getting ready to compete. Option (iv) shows punching something by blowing fist over it.

@PROCBSE 120

Oswaal CBSE Question Bank Chapterwise & Topicwise, ENGLISH LANG. & LIT., Class-X

SUBJECTIVE TYPE QUESTIONS Short Answer Type Questions





























Ans. The Employees were kind and helpful. [CBSE Marking Scheme, 2018]



Commonly Made Error

Some candidates failed to mention the qualities of the post office employees rather they talked about the post master only.









Detailed Answer: The post office employees represented the people who believed in helping others. They were kind and helpful. They cooperated with each other and contributed to their best of the capability just to keep Lencho’s faith in God alive. They had love, concern, compassion and empathy for Lencho.

Answering Tip



Read the question → reflect → then answer it.

Q. 7. Why would you not agree with Lencho calling them ‘a bunch of crooks’? A [CBSE SQP, 2018-19] 

















Ans. Lencho was filled with utter sadness when the hailstorm stopped. He was troubled to see the corn which was totally destroyed. Nothing was left in the fields. He and his family had to go hungry without corn as he suffered huge losses in the hailstorm. Q. 5. Why did Lencho write a letter to God? R [O.D. Set-III 2019] Q. 6. Explain the qualities of the post office employees. A [CBSE SQP, 2018-19]











Q. 1. 'The field was white, as if covered with salt.' This is how the field is described after the hailstorm. The pelting hailstones could have been easily seen as sugar cubes. Do you think comparing it with sugar would have been more appropriate? Why/ U [CBSE-QB, 2021] Why not? Ans. According to me, comparing salt to the hailstones is more appropriate because the hailstorm destroyed Lencho’s fields. Comparing the hailstones with sugar would mean that it benefited him. On the contrary, the hailstorm caused a lot of damage and all his harvest was destroyed. Q. 2. Lencho and his family knew the implications the hailstorm would have on their lives. Write a conversation between Lencho and his wife as they watched the downpour turn to a hailstorm. C [CBSE-QB, 2021] Ans. Lencho: The weather is getting worst now. I have an intuition something bad is going to happen. Wife: Yes, it has been raining hard for quite a long time. It will destroy our crop. Lencho: Look! Good Lord! It is hailing now. God save my corns! Wife : Don’t worry. God won’t let his children starve. Lencho: I see no hope! We are ruined! Q. 3. Lencho did not bother exploring any other means to resolve his situation but just turned to God. Do you feel that his approach was justified? Why/ U [CBSE-QB, 2021] Why not? Q. 4. What were Lencho’s feelings when the hailstorm U [O.D. Set-I 2020] stopped?





(40-50 words & 3 marks each)

Long Answer Type Questions



















Ans. (a) The main reason behind their help was to keep Lencho’s faith in God intact. Yes, I would also like to help someone who has such an immense faith in God by giving a part of my earning just like the postmaster and the post office

employees did. (b) Exactly, saying ‘no’ to a cause initiated by my boss won’t be a wise decision. Lencho, who was a poor farmer, was in a fear of going hungry this year. His ripe corn got destroyed due to an unexpected hailstorm. Lending a helping hand is a true spirit of a true human being. I, too, won’t hesitate while helping Lencho. Moreover, to find people like Lencho is very rare in this materialistic world. He was a true gem of his own type. And helping such a man was infact, just like helping God himself. Q. 2. Read the given anecdote and analyse the similarities and differences with reference to 'A Letter to God'.

Q. 1. The people at the post-office contribute to put together whatever they can manage, for Lencho. (a) What do you think was the main reason behind their help? Would you have done the same if you were one of the employees? (b) As an employee, write about your perspective and reason(s) for contributing to the fund. The fact that you don't want to say 'no' to a cause initiated by your boss, can also be an equally C [CBSE-QB, 2021] compelling reason!





(100-120 words & 6 marks each)

@PROCBSE A LETTER TO GOD



Ans. Objective: To test local and global comprehension, themes and ideas in the text. 6 Marks Marking: Content: 4 marks Expression : 2 marks Value Point (Minimum four points) • tried to keep Lencho’s faith in God intact 







• empathetic, softhearted, caring, good leadership qualities, sensitive • collects money from his employees to help Lencho and also gives a part of his salary • generous person, encourages to perceive others in a more positive light • fosters a sense of community building • spreads positivity which creates a healthy 











  

society.

[CBSE Marking Scheme, 2018]





Detailed Answer: The Postmaster first laughed looking at the letter which had a strange address. But soon he became serious. He was surprised at the faith that Lencho had in God. He wanted his faith to be intact in God. The postmaster was a kind and compassionate human being. He tried to keep Lencho’s faith in God intact by collecting the amount for him. He was an empathetic and softhearted person who cared for Lencho. He displayed good leadership qualities by asking each Post office employee to contribute to his extent. The postmaster himself gave a part of his salary and also requested his employees and a few friends to contribute for the good cause as a charity. He encouraged to perceive others in a more positive light. He was a man who fostered a sense of community building and wished to spread positivity which helps in creating a healthy society.





Commonly Made Error Students generally read the summary of the chapters and thus, do not know the in-depth details of the chapters.

Answering Tip

















 Started correspondence with God, immense faith, asked for money.  Received a lesser amount, blamed the post office employees.  Didn’t realise the irony- they were the real helpers, who had contributed parts of their salary.  Didn’t act practically. [Board Marking Scheme, 2020]

Detailed Answer: Lencho was a simple and hard working farmer. His corn crops were destroyed in a hailstorm and he was very sad. He felt that he and his family would go hungry the whole year unless they found someone who could help them. He had a firm faith in God, so he started correspondence with God and asked for a hundred pesos. He was confident of receiving the amount. The letter, on the other hand, was read by the postmaster of the post office. To ensure that Lencho does not lose faith in God, the postmaster collected money from post office employees and sent the seventy pesos that he had collected, to Lencho. When Lencho opened the envelope, he found a lesser amount in it. He was not happy. He thought that the rest of the money had been taken by the post office employees. He































Ans. These questions have been set to test the student’s understanding of the text and their ability to interpret, evaluate and respond to the themes / issues raised. Therefore, there can’t be just one correct answer. All presentations logically supported by textual evidence must be considered correct. Content = 4 marks Expression = 2 marks Suggested Value Points:  Lencho- a simple person.





E [CBSE-QB, 2021] Q. 3. The postmaster was a representative of God. Evaluate this statement in the context of your understanding of 'A Letter to God'. [CBSE-QB, 2021] Q. 4. Lencho had faith in God but lacked faith in humanity. Elaborate with reference to ‘A Letter to A [Board SQP, 2020] God’.

121

called them a bunch of crooks and wrote another letter to God asking for the remaining money but telling Him(God) not to send through the Post Office employees. He didn’t realise the irony that it was in fact, those people only who were the real helpers. Calling them ‘a bunch of crooks’ was not at all justified. Q. 5. In the lesson ‘A Letter to God’, what moral values does the postmaster display in his behaviour? A [Comptt. Set-1, 2 & 3, 2018] Modified















A very poor woman called-in a radio station asking for help from God. A non-believer, also listening to this radio program, decided to make fun of the woman. He got her address, called his secretary and ordered her to buy food and take it to the woman. However, the instruction was: "When the woman asks who sent the food, tell her that it's from the devil." When the secretary arrived at the woman's house, the woman was very happy and grateful for the help. The Secretary then asked her, "Don't you want to know who sent the food?" The woman replied, "No, I don't even care because when God orders, even the devil obeys!





The value based questions like Q4 and Q5 should be answered beyond the text, instead of simply providing the summary.

@PROCBSE 122

Oswaal CBSE Question Bank Chapterwise & Topicwise, ENGLISH LANG. & LIT., Class-X

Solutions for Practice Questions Him to send hundred pesos to sow his field again / support his family. [CBSE Marking Scheme, 2019]

Detailed Answer: Lencho’s entire crop was destroyed in the hailstorm, but he had a deep faith in God and he was his only hope. Therefore, he wrote a letter to God asking for monetary help so that he could sow his field again and support his family.







Long Answer Type Questions









Ans.2. There are many similarities in the anecdote like: The poor woman too had immense faith in God just like Lencho. She too asked help from God himself instead of asking aid from people around her. Just like Lencho she too believed the devils bunch of crooks exist in this world. The man made fun of her in the same way as the post office employees laughed on Lencho when they read his first letter. But both the non-believers the post office employees rendered a helping hand to the needy. The differences are as follows: The poor woman considered a devil as help from God. She became happy when she saw the food. On the contrary, Lencho became angry because the amount of money was less than desired one.

The man who helped the old woman called himself as a devil whereas the postmaster himself signed as the ‘God’. Ans.3. Yes, the postmaster was a representative of God for at least three reasons. First, he was the one who actually read the letter, on behalf of God. Also, he made effort to fulfill Lencho's prayers, which were addressed to God. So, the postmaster acted in God’s place and tried to collect funds for the poor farmer. Thirdly, Lencho himself believed that God had answered his prayers and sent him the money. The postmaster represents a bunch of people like Lencho who have faith in God. There are people like the Postmaster and the Post Office employees, who help others without any self interest. To keep Lencho's faith in God intact, the postmaster cooperated with his employees and collected the amount. He had love and empathy towards Lencho when he knew about his situation. He was a compassionate man and even hid his identity in helping Lencho. He sent the amount to Lencho in the name of God. Even though none of the Post Office employees or Post Master were related to Lencho in any manner, yet it was a pure act of humanity, kindness and selflessness on their part. Thus, the postmaster was a fellow human being and not God, but his actions were very kind. It can be concluded that he was indeed a representative of God.















Ans.3. Lencho did not bother exploring any other means to resolve his situation because he had a deep faith in God. He believed that only God could help him to come out of this disaster. He was the only one whom he could trust. All others were a bunch of crooks and they would take undue advantage of his situation. Hence his approach was justified. Ans.5. Crops completely destroyed / wrote a letter to God to seek help because he was the only hope / asked Ans.7. Lencho was not at all justified in calling them ‘a bunch of crooks’ because they helped him by collecting money during his difficult time of his crop being spoilt by hail. [CBSE Marking Scheme, 2018] Detailed Answer: Lencho was not at all justified in calling them ‘a bunch of crooks’ because they helped him by collecting money and understanding Lencho’s unshakable faith in God. Each one of them parted with some money for the act of charity which ‘a bunch of crooks’ would never do.



Short Answer Type Questions





REFLECTION Were you able to lean the importance of being generous without expecting gratitude in return, as demonstrated by Post office employees towards Lencho?

@PROCBSE

CHAPTER

2

NELSON MANDELA: LONG WALK TO FREEDOM – By Nelson Rolihlahla Mandela

Revision Notes Scan to know more about this topic



Introduction The chapter is an extract from Mandela’s autobiography ‘Long Walk to Freedom’. It provides us a glimpse of the early life of Nelson Mandela, his education, thirty years in prison and the pains he had suffered in his young age. It also recounts his fight for the freedom of his own people who were tortured by the whites.



The oath taking ceremony of Nelson Mandela, the first black President of South Africa and his colleagues took place on 10th May 1994. It was a historic occasion. Dignitaries and representatives of 140 countries came to attend it. The ceremony took place in the lovely sandstone amphitheater, formed by the Union Buildings in Pretoria. First, Mr. De Klerk, the 2nd Deputy President and then Thabo Mbeki, the 1st Deputy President were sworn in. Nelson Mandela took oath as the President. He pledged to obey and uphold the Constitution and devote himself to the well-being of the republic and its people. Then President Mandela addressed the guests. He welcomed and thanked them for having come to take possession with the people of his country for a common victory of justice, peace and human dignity. After getting political freedom, his government pledged to liberate people from the bondage of poverty, deprivation, suffering, gender and other discriminations. He wished the sun of freedom to shine on his country forever. After the ceremony, the display of military force was carried out. Finally, the jets left off smoke trails of different colours, e.g., black, red, green, blue and golden colour of the new South African flag. In the end, two National Anthems were sung by the whites and the blacks. Later on, that day, Mandela reformed history. In the first decade of the 20th Century, a few years













Summary

after Anglo-Boer War before his birth, the white skinned patched up their differences and erected a system of racial domination against the dark skinned people of South Africa. It was the birth Nelson of Apartheid, the harshest Mandela: Long in human creation. Now, in the last decade of the 20th walk to freedom century, the system has been overturned forever, recognising the rights of all people irrespective of the colour of their skin or religion. He remembered the suffering and courage of thousands of patriots who participated in the long struggles but were not there to witness the fruit of their achievement. It was a reign of oppression and cruelty that created a deep wound in African people. But deep oppression produced the Oliver Tambos, the Walter Sisulus, the Yusuf Dadoos. The Chief Luthulis, the Bram Fischers, the Robert Sobukwes, etc.— men of unparallel courage, wisdom and generosity. Mandela thinks South Africa’s real wealth is her people who are finer, truer than the purest diamonds. His comrades taught him what courage meant. It is not the absence of fear but victory over it. No one is born to hate another on the basis of colour of skin or religion. If they can learn to hate, then why not learn to love which comes naturally. He believed in the goodness of man that never dies. Nelson Mandela was of the opinion that every man has twin obligations, one towards his family and the other towards his people and his country. In the reign of Apartheid, if one tried to fulfill his duty towards his people, he was ripped off his family and home. Mandela said that he was born free. He had the freedom to run in the fields, swim in the stream and

@PROCBSE 124

Oswaal CBSE Question Bank Chapterwise & Topicwise, ENGLISH LANG. & LIT., Class-X

ride on a bull. Boyhood freedom was an illusion. As a student he wanted transitory freedom— freedom to stay out at night, to read books of his choice. As a young man, he yearned for basic honourable freedoms of achieving his potential, of earning, of marrying and having a family. When he became a young man and joined the African National Congress Party, he first wanted freedom only for himself and then for all his people and his country. Both need to be liberated. The oppressor is a prisoner of hatred, prejudice and narrow

mindedness. The oppressor and the oppressed, both are robbed of their humanity.

Key Words









1. Ampitheatre: a building without a roof, with many rows of seats rising in steps 2. Emancipation: freedom from restriction 3. Discrimination: Being treated differently or unfavourably 3. Spectacular array: An impressive display

Extract Based Questions

I. Read the following extract and answer the questions/ complete the sentences that follow: (5×1=5) "We, who were outlaws not so long ago, have today been given the rare privilege to be host to the nations of the world on our own soil. We thank all of our distinguished international guests for having come to take possession with the people of our country of what is, after all, a common victory for justice, for peace, for human dignity." [CBSE-QB, 2021] (5 × 1 = 5) Q. 1. The guests at the spectacular ceremony are being called distinguished because: (A) they have been invited as guests to attend it. (B) they are eminent world leaders witnessing it. (C) they are visiting the country for this purpose. (D) they have resumed diplomatic relations with the country. Ans. Option (B) is correct. Explanation: ‘Distinguished’ here means ‘someone different from the common’. The guest list consisted of some prominent and well-known leaders of the world.

Blacks have got equal rights as the Whites. It was the first time that those generations were experiencing this.

Q. 4. How do you think the speaker feels? Choose the option that best fits his state of mind.

(A) (i) emotional

(ii) elated

(B) (i) elated

(ii) unmindful (iii) overwhelmed

(iii) unmindful

(C) (i) overwhelmed (ii) elated

(iii) indecency

(B) (i) liberty

(ii) indecency

(iii) self-respect



Q. 5. Pick the option that showcases the usage of 'host' as in the extract. (A) He was praised for his hospitality as the host of the party. (B) She was able to host the event without any hindrance. (C) She met the host and apologised for her friend's misbehaviour. (D) He is the best host that one can ever come across. Ans. Option (B) is correct. Explanation: In options, (A), (C) and (D), ‘host’ is used as a noun and referring to a person. In the extract, ‘host’ is used as a verb meaning organising an event and inviting others to join which is the same as used in option (B).

(ii) liberty





(A) (i) equality

(iii) honoured

(D) (i) elated (ii) honoured (iii) unmindful Ans. Option (C) is correct. Explanation: Unmindful means careless or forgetful. The speaker is recalling years of struggle and is asking his fellow citizens to maintain this freedom carefully. So, all the options (A), (B) and (D) express unmindfulness as feeling of the speaker which is not correct.





Q. 2. It is a victory for 'human dignity'. Pick the option that lists the correct answer for what 'human dignity' would include.



































(5 marks each)













II. Read the following extract and answer the questions/ complete the sentences that follow: (5×1=5) "It was only when I began to learn that my boyhood freedom was an illusion, when I discovered as a young man that my freedom had already been taken from me, that I began to hunger for it. At first 













Q. 3. Why does the speaker say that it is a 'rare privilege'? He says this as they have (A) been deprived of this honour. (B) seldom been given this honour. (C) experienced it for the first time. (D) been chosen over other countries, for this honour. Ans. Option (C) is correct. Explanation: After decades of resentment, the





(D) (i) equality (ii) liberty (iii) self-respect Ans. Option (D) is correct. Explanation: In Options (A) and (B), indecency means not decent, which is against human dignity. In Option (C) immorality or not being moral is also against human dignity. The virtues of equality, liberty and self-respect best define human dignity.











(C) (i) immorality (ii) self-respect (iii) equality

@PROCBSE

NELSON MANDELA: LONG WALK TO FREEDOM

Explanation: In the passage, the narrator had been living a life under a deception or misapprehension - an illusion. This similar meaning is conveyed in option (D). Q. 4 The speaker says, 'at first as a student I wanted freedom only for myself.' Why do you think he only thought about himself? (A) He didn't want to think about the freedom denied to others. (B) He was being selfish and was only bothered about himself. (C) He didn't think that freedom denied to him was important for others. (D) He was too young to realise that freedom was denied to others as well. Ans. Option (D) is correct. Explanation: When he was a young boy, he did not have a wider vision – he was more concerned about his personal freedom. He could not look from other people’s perspective that they too were deprived of freedom in much more important aspects than his boyish perspective.









Q. 2. Why do you think the speaker mention some freedoms as 'transitory'? (A) The freedoms are momentary and keep changing with time. (B) The definition of freedom is constant but perspectives differ. (C) Freedom means different things to different people. (D) Freedom is not that important after a certain age. Ans. Option (A) is correct. Explanation: ‘Transitory’ means ‘temporary’ or ‘passing’. So, option (A) that states freedoms as momentary or temporary and changing with time is correct.































as a student, I wanted freedom only for myself, the transitory freedoms of being able to stay out at night, read what I pleased and go where I chose. Later, as a young man in Johannesburg, I yearned for the basic and honourable freedoms..." [CBSE-QB, 2021] Q. 1. The title that best suits this extract is: (A) Freedom for Everything (B) Knowledge about Freedom (C) Significance of Freedom (D) Realisation of Freedom Ans. Option (C) is correct. Explanation: The extract explains why freedom is important and what it means to those who are not free. So, significance of freedom is the correct title for the extract.













Q. 5 A part of the extract has been paraphrased. Choose the option that includes the most appropriate solution to the blanks in the given paraphrase of the extract. The speaker's belief about freedom, since childhood proved false. It was not until the speaker grew up to be a young man when it (i) ______on him that he was (ii)_______of freedom. Then he began (iii)_________ it. (A) (i) desired (ii) dawned (iii) depriving (B) (i) dawned (ii) deprived (iii) desiring (C) (i) dawned (ii) arrived (iii) desiring (D) (i) arrived (ii) deprived (iii) dawned









Q. 3 Choose the option that best fits the usage of the word 'illusion' as used in the extract. (A) He was never able to get past the illusion. (B) The illusion I experienced was quite intriguing. (C) A large mirror in the room creates an illusion. (D) I was living under the illusion that this is possible. Ans. Option (D) is correct.

125













Ans. Option (B) is correct. Explanation: Here, Dawned means emergence of a thought, Deprived means denied and Desiring means wanting to have. These meanings best fit the given part of the extract.

SUBJECTIVE TYPE QUESTIONS Short Answer Type Questions













These questions are for practice and their solutions are available at the end of the chapter











(ii) At which age, did you decide to study law? What exactly piqued your interest in the subject? Q. 3. Your teacher organised a mini-debate competition in class on the topic: Courage, Wisdom and Generosity are the ONLY attributes of a remarkable leader. Write the debate script with two points to supplement your stand, either as a proposition speaker or as an opposition one. C [CBSE-QB, 2021] 

Q. 1. Mandela says that his country's greatest wealth is its people and not the purest diamonds. Justify his U statement. Q. 2. Create a questionnaire of two most important questions that you would have liked to ask Nelson Mandela during an interview. Give your reasons for choosing those questions. C [CBSE-QB, 2021] Ans. I would like to ask the following two questions to Nelson Mandela during an interview: (i) How did your education help you in contributing to the removal of Apartheid?





(40-50 words & 3 marks each)

@PROCBSE 126

Oswaal CBSE Question Bank Chapterwise & Topicwise, ENGLISH LANG. & LIT., Class-X 













(b) My friend, true freedom isn’t realized by the lack of physical boundaries but is born through the revelations of the society people around us, our understanding of others and the outward expression of our heart and soul. Q. 5. Nelson Mandela speaks of ‘Twin Obligations’. R [Board SQP 2020] Elucidate.











Ans. One towards the society, one towards the family. [Board Marking Scheme, 2020]











Detailed Answer: According to Nelson Mandela, every man has two obligations—one is towards his family and the other is towards his people and his country. But in the reign of apartheid, if one tried to fulfill his duty towards his people, he was ripped off with his family and home. Q. 6. What did ‘being free’ mean to Mandela as a boy U and as a student? [Board Term-1 2016-17] [NCERT]



Ans. (a) I believe that freedom isn’t selfish individually. It is the generosity of self with the world.



















Ans. As a proposition speaker: It is rightly said that Courage, Wisdom and Generosity are the only attributes of a remarkable leader. A leader should be ready to face any problem. It may be internal or external. To solve the matter, he should analyse before passing the judgment where he has to use his wisdom. He should not be partial in his decision, here he needs courage when necessary. He should be generous in helping others without hesitation. Q. 4. You recently read a blog by your teen friend, on 'Freedom-My Perception'. You feel that your perception of freedom has seen a transformation after having read 'The Long walk to Freedom'. Complete the dialogue in 50 words, with your friend, explaining your new understanding of freedom. You may begin like this: Friend: I think the freedom to watch T.V. for extended hours or choose the kind of programmes I'd like to view is important. You: (a) _____________________________________. Friend: Hmmm. I still feel that the restrictions are not required. I want my freedom. I'm human too, am I not? You: (b) ____________________________________ . C [CBSE-QB, 2021]

Long Answer Type Questions





Commonly Made Errors















Most of the students talked about Nelson Mandela in general. They did not correlate the answer with the content in the chapter. A lot of spelling mistakes were found in words like colleagues, Apartheid, Sobukwes, etc..

Answering Tip

These questions are for practice and their solutions are available at the end of the chapter

























human dignity. In the same way, the Indians were exploited by the British. They worked in the lands like animals who were only supposed to obey. Their lands, resources and approaches were captured by the British. The struggle of Indian under British rule resembles with the struggles of the people of South Africa under the apartheid regime. The struggle created a deep and lasting wound on them. But they never gave up their cause, ultimately, their sacrifices led to their victory- common victory of humanity, for peace, for justice and for human dignity. Q. 3. Why was Nelson Mandela overwhelmed with a sense of history? How did he succeed in ending the apartheid regime in South Africa? U [Delhi Board Set- II 2020] 

Q. 1. Freedom is inconsequential if it is behind bars of prejudice and narrow mindedness. How would you explain this statement? Support your answer with suitable examples from the real world. A [CBSE-QB, 2021] Q. 2. After having read the lesson on the oppression that communities faced in South Africa, you were deeply hurt. You could also relate to the struggles and hardships of millions of Indians who fought against the oppressive British rule. Write a diary entry expressing your feelings about oppression faced by people in their homeland. You may begin like this: 24 August 20XX, Monday 9:00 pm My heart is filled with sadness as I think of the oppressed who had to tolerate the inhuman attitude of the oppressors in their own land.......... C [CBSE-QB, 2021] ..................... Ans. 24 August, 20XX, Monday, 9:00 pm My heart is filled with sadness as I think of the oppressed who had to tolerate the inhuman attitude of the oppressors in their own land. The oppression that the communities faced in South Africa under the apartheid regime reminds me the struggles and hardships of millions of Indians who fought against the oppressive British rule. The white regime in South Africa was based on racial discrimination. It symbolised exploitation and extraordinary human disaster. The blacks were deprived of their rights to equality and





(100-120 words & 6 marks each)

A thorough reading of the chapter along with writing practise is advised.

@PROCBSE

NELSON MANDELA: LONG WALK TO FREEDOM

127

Solutions for Practice Questions





Ans.1. Mandela feels that the people of his nation are the greatest wealth compared to any other form of wealth. He lays great emphasis on people and their power to change the system and bring a new change in the society. Ans.6. As a boy ‘being free’, meant to Nelson Mandela to wander free in fields, to swim freely and to run through the village. As a student-to stay out at night, to read what he pleased and to go wherever he chose was ‘being free’.

Long Answer Type Questions

Ans.1. Nelson Mandela believed that freedom is indivisible. His hunger for his own freedom became the greater hunger for the people. He couldn’t live his life with dignity and self-respect if his own people were bound in chains. The chains on any one of his people were the chains on all of them. Mandela realised that the oppressor must be liberated as surely as the oppressed. A man who takes away another’s freedom is a prisoner of hatred prejudice and narrow mindedness. He is not truly free if he is taking away someone else freedom. A person’s freedom is of no use if he doesn’t expand the horizons of his thoughts and

think like a narrow minded person. A person’s freedom is insignificant if he takes the freedom of another man, thus, the oppressed and the oppressor alike are robbed of their humanity. Ans.3. The oath taking ceremony of Nelson Mandela, the first black President of South Africa and his colleagues was a historic occasion. Dignitaries of 140 countries came to attend it. In the first decade of the 20th century, the white-skinned erected a system of racial discrimination against the dark-skinned people of South Africa. This discrimination was called Apartheid, the harshest in human creation. Nelson Mandela, when became a young man and joined the African National Congress party, he first wanted freedom only for himself and then for all his people. He carried on years of struggle with the desire to give himself and his people a life of dignity and self-respect. Finally, when he successfully became the President of South Africa, he promised to give his country freedom from poverty and discrimination. No country man would be oppressed on the basis of colour and creed. He wished the sun of freedom to shine on his country forever.

Short Answer Type Questions





REFLECTION Were you able to understand the underlying message "The oppressor and the oppressed both lose their humanity in Nelson Mandela's speech?

@PROCBSE

CHAPTER

3

TWO STORIES ABOUT FLYING

Topic-1 (A) His First Flight – By Liam O'Flaherty

Revision Notes Scan to know more about this topic



Introduction This imaginary story conveys the message that one learns by taking courage and not by sitting idle. A young seagull is fed lovingly by his parents. But when the time comes for him to fly and feed himself, he feels afraid. At last, his mother hits upon a plan by persuading him with food in her beak. And the seagull finally learns how to fly.





The young seagull was alone on his ledge. His two brothers and his sister had already flown away. When he wanted to fly, he could not do so. He was afraid of flying. When his brothers and sister flew, he failed to gather the courage to fly. He was really very sad. His father and mother came to him. They asked him to fly. They even rebuked him. They threatened if he did not fly, he would die of hunger there. But he could not move. That was twenty-four hours ago. Since then, no one had come near him. He had watched his parents and brothers and sister flying. His parents had been perfecting his brothers and sister how to dive for fish. They called him to fly. But he could not. The sun was rising. It was getting very hot. He felt the heat. He had not eaten for long. He had found a dried piece of a fish tail. But it was not enough. There was not even a single piece of food anywhere for him. He had gone forward and backward on the ledge. But he couldn’t fly. He wanted to reach his parents. There was a deep sea below. The ridge was high. The seagull closed his eyes standing on one leg. He had the other leg hidden under his wing. He pretended to sleep. He thought that his parents didn’t take notice of him. However, mother was looking at him. He saw her tearing at a piece of









Summary

fish at her feet. He was mad to see the food. He gave out a sound to get it. His mother replied in a sound and looked at him. She picked a piece of the fish. She flew across to him with it. He leaned out tapping the rock with his feet. His His First Flight mother flew across trying to get nearer to him. The seagull had the food within the reach of his beak. But he could not get at it. The seagull waited a moment in surprise. He wondered why his mother did not come nearer. He was so mad with hunger that he dived at the fish. He fell outwards with a scream and downwards into space. His mother had swooped upwards. As he passed beneath her, he heard the swish of her wings. Then a great fear caught him. His heart stood still. He could hear nothing. But it only lasted for a moment. The next moment, he felt his wings spread outwards. He felt the wind rushing against his breast feathers. It rushed under his stomach and against his wings. He was not falling headlong now. He was soaring slowly downwards and upwards. He was no longer afraid. The seagull gave out a joyous scream. He soared higher calling ‘ga, ga, ga’. His mother gave out ‘gaw-col-ah’. His father flew over him screaming. Then he saw his brothers and sister flying around him. They were roaring and diving. Then he completely forgot that he had not always been able to fly. He commended himself to dive and soar. He was near the sea now, flying straight over it. He saw a vast green sea beneath him. His parents and his brothers and sister had landed on

@PROCBSE

TWO STORIES ABOUT FLYING

He had made his first flight.



Key Words









1. 2. 3. 4. 5.



the green surface of water ahead of him. They were calling him to do the same thing. He dropped his leg to stand on the green sea. But his legs sank into it. He cried with fear. He tried to rise again flapping his wings. But he was tired and weak with hunger. He could not rise. His feet sank into the green sea. His belly touched it but he sank no further. He was floating on it. Around him was his family, crying and praising him. It was offering him scraps of dog-fish.

Rebuked: To criticise harshly Threatened: Feeling insecured Ledge: A narrow cliff Joyous: Full of joy Scraps: Pieces, fragments

Extract Based Questions



(5 marks each)

(iv)





























(iii)





(ii)



(i)











Q. 5. Select the most appropriate option for the following: devour: guzzle:: nibble: (A) chew (B) savour (C) peck (D) gulp Ans. Option (C) is correct. Explanation: Devour and Guzzle refer to eating in big bites and quick swallowing. Nibble and Peck refer to eating in small tit bits and lazily eating. Chew refers to biting and grinding with teeth the bite taken. Savour refers to enjoying food. Gulp also refers to eating big bites and quick swallowing, so does not match with Nibble.





































Q. 4. Which of the following feelings did the young gull, NOT feel according to the given context? "...all day long, he had watched his parents flying about with his brothers and sister..." (i) sad (ii) incompetent (iii) excluded (iv) ungrateful (v) inspired (vi) jealous (vii) anxious (A) (i), (iii), (vi) (B) (ii), (v), (vii) (C) (ii), (iii), (vii) (D) (iv), (v), (vi) Ans. Option (D) is correct. Explanation: The young bord was full of feelings of sadness (for not able to enjoy flying from one place to another), incompetence (as he thought that he cannot fly), excluded (as others enjoyed in water and he used to sit alone outside) and anxious (as he also wanted to fly). However, he never felt ungrateful, inspired and jealous of his parents or siblings.































(A) (i) and (ii) (B) (ii) and (iii) (C) (iii) and (iv) (D) (i) and (iv) Ans. Option (C) is correct. Explanation: The young bird knew that birds are meant to fly, so he already knew about himself. It also is not about how early or late he should start flying so as to benefit. The extract is about the young bird whose siblings had already learnt flying, but he was having the first flight, so he need not bother himself about what others were doing, it was important for the young bird to focus on his First flight. Also, he was earlier afraid of flying, but when he overcame his fear, he learnt the art of flying.





I. Read the following extract and answer the questions/ complete the sentences that follow: (5×1=5) The day before, all day long, he had watched his parents flying about with his brothers and sister, perfecting them in the art of flight, teaching them how to skim the waves and how to dive for fish. He had, in fact, seen his older brother catch his first herring and devour it, standing on a rock, while his parents circled around raising a proud cackle. And all the morning the whole family had walked about on the big plateau midway down the opposite cliff taunting him with his cowardice. [CBSE-QB, 2021] Q. 1. Based on the given sentence, pick the option that corresponds to what human parents would say. 'While his parents circled around raising a proud cackle.' (A) Well done! (B) Oh no! (C) Ready? (D) Really! Ans. Option (A) is correct. Explanation: ‘Well done!’ is said to encourage the child after successfully doing something. ‘Oh No!’ will be said if the child is not able to do something. ‘Ready?’ will be said before starting of the event and not after the completion. ‘Really!’ will be said when the parents do not expect or trust that their child will be able to do this. Q. 2. The write of the story is (A) Frederic Forsyth (B) Liam O'Flaherty (C) Anne Frank (D) Walt Whitman Ans. Option (B) is correct. Q. 3. Imagine that the young gull attended a workshop on inspiration and confidence building and received a couple of pieces of advice. Choose the option that reflects these pieces of advice, most relevant to his situation.

129

@PROCBSE Oswaal CBSE Question Bank Chapterwise & Topicwise, ENGLISH LANG. & LIT., Class-X















































































(B) naturalness of the act of flying for the young gull. (C) satisfaction and joy of flying together as a family. (D) desire of the young gull to leave his fears behind. Ans. Option (B) is correct. Explanation: Flying is natural to birds. It was just that the young bird learnt it later that the birds are made for flying. Q. 4. The extract refers to the many movements of the young gull's brothers and sister. Choose the option that correctly sequences these movements. (A) The young gull's brothers and sister flew by tilting their wings, rose high, made darting movements and plunged headfirst. (B) The young gull's brothers and sister flew by plunging headfirst, making darting movements, titled their wings and rose high. (C) The young gull's brothers and sister flew with darting movements, titled their wings, rose high and plunged headfirst. (D) The young gull's brothers and sister flew by rising high, plunging headfirst, making darting movements and tilting their wings. Ans. Option (C) is correct. Explanation: Curveting – flying with tilted wings, Banking – quick darting movements, Soaring – rising high in sky, Diving – plunging headfirst into water (to catch fish). Q. 5. Which of the following mirrors the use of the literary device in "shrieking shrilly"? (A) Sparkling saga (B) Singing soft (C) Slippery sloppily (D) Sneeze silently Ans. Option (D) is correct. Explanation: ‘Shrieking shrilly’ and ‘Sneeze silently’ are use of alliterations.

II. Read the following extract and answer the questions/ complete the sentences that follow: (5×1=5) He just felt a bit dizzy. Then he flapped his wings once and he soared upwards. "Ga, ga, ga, Ga, ga, ga, Gaw-col-ah," his mother swooped past him, her wings making a loud noise. He answered her with another scream. Then his father flew over him screaming. He saw his two brothers and his sister flying around him curveting and banking and soaring and diving. Then he completely forgot that he had not always been able to fly and commended himself to dive and soar and curve, shrieking shrilly. [CBSE-QB, 2021] Q. 1. Pick the most appropriate reason why the young gull felt dizzy. (A) He hadn't eaten anything for a day. (B) He was dizzy with excitement. (C) He was wary of heights. (D) He was flying for the first time. Ans. Option (D) is correct. Explanation: Dizzy here means giddy, a feeling which one experiences when someone is doing a new thing for the first time. Here, the young bird was flying for the first time. Q. 2. How would you describe the screams of the gulls in the given extract? (A) Elation (B) Bewilderment (C) Shock (D) Protection Ans. Option (A) is correct. Explanation: His parents were delighted and overjoyed, that is elated, when the young bird started flying. They were not bewildered or shocked (as they knew that he can fly). They were not trying to protect him by flying over him, they were just expressing their joy. Q. 3. The line "he completely forgot that he had not always been able to fly" implies the: (A) great confidence the young gull had in his skills. 



130

SUBJECTIVE TYPE QUESTIONS Short Answer Type Questions

























the 'plunge' here be both literal and metaphorical? Explain. [CBSE-QB, 2021] Q. 4. The 'fight or flight response', that is, to stay and face a situation or run from it- is an automatic reaction to an event perceived as stressful or harmful. How would you evaluate the young gull's response on finding himself off the ledge? [CBSE-QB, 2021] Ans. No doubt the young gull faced the situation with the ‘fight response’ success can’t be taken to be granted. One has to struggle and finally win it. Hunger made the young seagull take his final plunge. This led him to make his first flight and soar into space after satisfying his hunger.

These questions are for practice and their solutions are available at the end of the chapter



Q. 1. Describe the young seagull's emotions when he U [CBSE-QB, 2021] flew over the sea. Q. 2. If you were the family member of the young seagull, would you also decide not to go near him? U [CBSE-QB, 2021] Elaborate with reason. Ans. Yes, even I would have done the same. The seagull’s family members encouraged him to fly. But he was too afraid to fly. Then they kept him hungry. Then they thought that experience would teach him, so they made him fall from the ledge. Ultimately, all their efforts were fruitful. Q. 3. The young seagull "failed to muster up courage to take that plunge". In what way can the reference to





(40-50 words & 3 marks each)

@PROCBSE TWO STORIES ABOUT FLYING 



U [CBSE Board Set 1, 2020]

Ans. The young seagull was afraid to fly because the sight of the sea filled him with fear. What motivated him to fly was his hunger. For past twenty four hours, he had not eaten anything. When his mother tempted him with a piece of fish in her beak, he was maddened to see the food. His hunger compelled him to dive. To save himself, he spread his wings and was able to fly. (Board Marking Scheme, 2020) Q. 6. How did the seagull’s parents try to make him fly? [CBSE Board Set 1, 2019] 











131

Q. 5. "The sight of the food maddened him." What does this suggest? (His First Flight) 













Topper Answer, 2019











Ans. Dropped to land on sea, legs sank, screamed with fright, tried to rise again, feet sank, belly touched water, started floating. (Board Marking Scheme, 2016)

Detailed Answer: When the young seagull landed on the sea, his legs sank into it. He screamed with fright. He tried to rise again flapping his wings. But being tired and weak with hunger, his feet sank into the green sea. His belly touched water but he sank no further. He started floating on it. Q. 8. What happened when the young seagull dived at the fish picked up by his mother? [Board Term-1 2016-17 Set-W1W2XLL] Ans. When the young seagull dived at the fish, he fell outwards and downwards with a scream. His monstrous terror seized him and his heart stood still. Soon his wings spread outwards and he could feel the tips of his wings cutting through the air.

Long Answer Type Questions Q. 1. The young seagull's parents wanted him to fly, like his brothers and sister. How could the young seagull's newly flying siblings (Brother 1, Brother 2 and Sister) have motivated him? Write a brief C [CBSE-QB, 2021] conversation among them. Ans. Young seagull : Brother 1, I’m so hungry, kindly give me some food. Brother 1 : I’m holding the food in my beak for you. But you will have to jump the ledge and take it. Young seagull : I am scared! The sea below is so deep. I’ll die. Brother 2 : No, you won’t. We birds learn flying only after jumping. See, how well I’m flying. Young seagull : Are you sure, I won’t fall into the sea? Sister : Yes, Brother 2 is right. Would a sister ask her brother to suicide? I am your sister. I am just speechless at your fear. Young seagull : OK! I trust you.







These questions are for practice and their solutions are available at the end of the chapter















Sister : would fly? Young seagull : (feeling elated) WOW! Success is just behind the daring. Thanks sister! Q. 2. A noted author, Richard Bach wrote - "For most gulls it was not flying that mattered, but eating. For this gull, though, it was not eating that mattered, but flight." Evaluate this statement with respect to A [CBSE-QB, 2021] the young seagull. Q. 3. Look at the image of Maslow's hierarchy of needs.



(The young seagull jumped off the ledge. He fell for hundred metres. Soon he began flying.) See! Didn't I tell you, you













































(100-120 words & 6 marks each)















Detailed Answer: The little seagull's parents tried every possible way to make him to fly. They came to him and asked him and asked him to fly. The rebuked him and threatened him, that if he didn't fly, the will leave him to die of hunger. They even pretended to ignore him. Q. 7. How did the young seagull react when he landed R [Board Term-1, 2016] on the sea?

Abraham Maslow devised a theory of motivation that shows the basic needs at the bottom and more advanced needs as you move up. It includes the theory that fundamental needs must be met before an individual can be motivated to achieve higher order needs.

@PROCBSE Oswaal CBSE Question Bank Chapterwise & Topicwise, ENGLISH LANG. & LIT., Class-X



Where in this hierarchy of needs, would you locate the young seagull's first flight? Justify your response. [CBSE-QB, 2021] Q. 4. Look at the given image of different parenting styles and what they signify.

Which of the given parenting styles would you attribute to the young seagull's parents? Do you think there was a difference between the mother's and father's attitude? Support your answer with reference to the text. U [CBSE-QB, 2021]













132





Commonly Made Error Many students failed to describe the efforts made by the mother of the little seagull.

While writing long answers, students should focus on the feelings and emotions of the seagull and his mother.

Solutions for Practice Questions Short Answer Type Questions

Ans.1. When the young seagull touched the green sea for the first time, he was frightened. So, he tried to flap his wings to fly higher. Since he was too weak to fly, he found himself floating on the sea with his family. Ans.3. The young seagull didn’t have the courage to fly. Hence, he made excuses for not flying. He had





Answering Tip

These questions are for practice and their solutions are available at the end of the chapter

no courage to flap his wings and literally failed to muster up courage to take dive. Metaphorically he couldn’t muster up courage to take the risk (plunge) when compared to his siblings. All his brothers and sisters had far shorter wings than his wings but they had already learnt the art of flying.

@PROCBSE

TWO STORIES ABOUT FLYING

The seagull was very hungry. He hadn’t eaten anything for last twenty four hours. When he saw his mother bringing food in her beak, he dived towards her. But fell from the brink of the ledge. He screamed with fear. But his fear lasted only for a moment. The next moment, he felt that his wings spread outwards instinctively. He was flying now. Thus, hunger motivated him.



Long Answer Type Questions





Ans.2. Food is the most essential ingredient that sustains all life on earth. But the young seagull, it was not eating that mattered, but flight. Whenever he tried to flap his wings to fly, he was seized with fear. He felt certainly that his wings would never support him. He had seen his elder brother catch his first Herring and devour it. It compounded his helplessness even more. He uttered a joyful scream when he saw his mother holding a piece of fish in her beak and flying quite near him. Hunger became motivation for the young seagull. Maddened by hunger, he fell outwards and downward into space. Then a monstrous terror seagull him but only for a minute. The next moment, he felt his wings spread outwards. The fear left him. Soon, he was floating on the green sea. Ans.3. The young seagull’s first flight can be located in the category of ‘Basic Needs’ in this hierarchy of needs. These needs are most essential for the very survival and maintenance of self. These include food, warmth and shelter. These needs must be met repeatedly within short time period such as hunger.

133





Ans.4. According to me, the young seagull’s parents were authoritative parents. As in this case, parents have clear standards for their children. While the parents set limits, they also give the children an opportunity to explore and learn. Parent also explain why they punish the child when the punishment is given and give positive encouragement at the right time. Yes, there was a great difference between the mother’s and father’s attitude. The mother used sensitive and responsive approach. She is actually portrayed as a tough but motivating character. She understood that her child needed a lot of motivation to achieve its fad of flying. Initially, she tried to threaten the young seagull in many ways but to no avail. She knew very well that fear was the only thing that actually had to be feared. So, she used food to arouse young seagull’s need.

Were you able to identify the underlying message 'Never give up' based on your reading of this chapter?





REFLECTION

Topic-2 (B) Black Aeroplane – By Frederick Forsyth

Revision Notes This is a mysterious story. The author was flying his Dakota aeroplane. Suddenly, he was caught in a big cloud and his compass, radio and other instruments failed. There was not much fuel in his plane. Suddenly he saw a black aeroplane near him. The pilot in that plane guided him. As a result, he was able to land safely. But he was greatly surprised when the woman at the control centre told him that there had been no other plane except the writer’s Dakota in the sky that night.

Summary



Introduction The author says that he was flying an aeroplane at 1 o’clock at night. The moon was coming up in the east behind him. Stars were shining in the clear sky above him. He was flying his old Dakota aeroplane over France, back to England. He was dreaming of his holiday.

Scan to know more about this topic

Black Aeroplane

@PROCBSE 134

Oswaal CBSE Question Bank Chapterwise & Topicwise, ENGLISH LANG. & LIT., Class-X





So, he followed him. He flew for half an hour. He had fuel to last for five or ten minutes. He began to feel frightened. Then he started to go down. Suddenly, he was on the runway. He looked for his friend in the black aeroplane. He wanted to thank him. But the sky was empty. He went into the control tower. He asked the woman sitting there where he was. She looked at him strangely and then she laughed. She said that there was no other aeroplane except that of his. The author was wonder struck. He wondered who had helped him to arrive there. He was without a compass or a radio and without any more fuel in his tanks, who was the pilot on the strange black aeroplane, flying in the storm without lights.

Key Words

1. Twisted: Coiled, wound spirally 2. Compass: an electronic device used to determine the cardinal directions.









He was looking forward to being with his family. He decided to call Paris control. He then radioed to it. He told it that he was on his way to England. Paris control acknowledged. It advised him to turn twelve degrees west. He duly turned to that direction. Everything was going well. Paris was about 150 kilometers behind him. Then he saw the storm clouds. They looked like black mountains in front of him. He knew he could not fly up and over them. He decided to go back to Paris. But he also wanted to get home. So, he took the risk and flew into the storm. He could see nothing outside his aeroplane. Everything was black. The old aeroplane jumped and twisted in the air. He looked at the compass. It was gone. He tried to contact Paris control. He was lost in the storm. Then in the black clouds, he saw another aeroplane. It had no lights on its wings. He could see it flying next to him in the storm. He was glad to see another person. He was signaling to follow him. The author thought he was trying to help him.

Extract Based Questions















(iv) (B) Option (ii) (D) Option (iv)











(iii) (A) Option (i) (C) Option (iii)













(ii)



(i)





























Ans. Option (C) is correct. Explanation: Before completing the flight, the narrator mentioned it as ‘an easy flight’. The later incidents show that the flight was full of adventure and suspense. So, ‘You spoke too soon’ is the best response to the comment. Q. 2. How would you describe the "risk" the narrator took? (A) Calculated (B) Impetuous (C) Unavoidable (D) Navigable Ans. Option (B) is correct. Explanation: Impetuous means impulsive or hasty. Flying the Dakota into the storm was an impulsive decision. He could not calculate in advance the risk that he might have to face, it was avoidable as he could return to Paris and it was not navigable as he could not see far due to thick and heavy clouds. Q. 3. In what way might the reference to the Dakota as "old" be relevant? (A) Its antique value made it expensive and precious to the narrator. (B) It is employed by the narrator as a term of endearment. (C) It did not have enough fuel to fly around the storm clouds. (D) Its ability to negotiate the storm clouds might have been suspected. Ans. Option (D) is correct. Explanation: ‘Old’ here has been used to refer to its inability to stand heavy storms during flight as estimated by the narrator. Q. 4. Read the statements given below and then select the option that best describes the given statements. Statement I - The narrator's desire to reach home and see his family made him complacent.

I. Read the following extract and answer the questions/ complete the sentences that follow: (5×1=5) Everything was going well — it was an easy flight. Paris was about 150 kilometres behind me when I saw the clouds. Storm clouds. They were huge. They looked like black mountains standing in front of me across the sky. I knew I could not fly up and over them and I did not have enough fuel to fly around them to the north or south. "I ought to go back to Paris," I thought, but I wanted to get home. I wanted that breakfast. 'I'll take the risk,' I thought and flew that old Dakota straight into the storm. [CBSE-QB, 2021] Q. 1. Based on the given extract, choose the option that lists the name which would be the most appropriate response to "...it was an easy flight"? 





(5 marks each)

@PROCBSE TWO STORIES ABOUT FLYING

Explanation: The correct track of expressions is excited (to come out of the storm), surprised (to see the empty sky and at absence of the black aeroplane), relieved (to land safely in old Dakota), grateful (towards the other pilot who guided him to safety) and perplexed (to know that there were no other aeroplanes flying that night).





















Ans. Option (A) is correct.





  



  



Q. 4. The narrator exclaimed that he was "safe". Which of the following represented the most immediate threat to the narrator's safety?





































Ans. Option (B) is correct. Explanation: She thought that the narrator was an experienced pilot and yet, he wanted to meet the other pilot, scheduled to land at the same time when there was no aeroplane in sight. So, she thought that he was trying to be funny. Preposterous means absurd, she had no sympathy with safe landing of narrator as she was not aware of the danger he had faced and he was surely not trying to tease her as indicated in the story. Q. 3. Filled with questions, the narrator decides to place an advertisement in the local newspaper to look for his "friend". Read the advertisement given below and select the option that includes the most appropriate solutions for the blanks: Looking for a pilot of a black aeroplane who (i)______ an old Dakota out of storm clouds late last night, but (ii)________before the Dakota pilot could express his gratitude after landing. Though control centre and radar did not (iii)________ its presence, the Dakota pilot would really appreciate if his friend reached out. Please contact the Dakota pilot at 5200100110. In deep gratitude and eager (iv)________, XXX. (A) (i) guided; (ii) disappeared; (iii) register; (iv) anticipation (B) (i) took; (ii) landed; (iii) acknowledge; (iv) appreciation (C) (i) brought; (ii) went away; (iii) confirm; (iv) expectation (D) (i) helped; (ii) vanish; (iii) make note; (iv) excitement Ans. Option (A) is correct. Explanation: The black aeroplane helped and showed way – guided Later on, the narrator was not able to see the aeroplane because it – disappeared The radar and control centre did not know about its presence, they did not – register The narrator expected to hear about the pilot and the black aeroplane - anticipation











II. Read the following extract and answer the questions/ complete the sentences that follow: (5×1=5) I was safe! I turned to look for my friend in the black aeroplane, but the sky was empty. There was nothing there. The black aeroplane was gone. I could not see it anywhere. I landed and was not sorry to walk away from the old Dakota near the control tower. I went and asked a woman in the control centre where I was and who the other pilot was. I wanted to say 'Thank you'. She looked at me very strangely and then laughed. "Another aeroplane? Up there in this storm? No other aeroplanes were flying tonight. Yours was the only one I could see on the radar." So, who helped me... [CBSE-QB, 2021] Q. 1. Select the option that correctly tracks the progression of emotions experienced by the narrator in the given extract. (A) excited - surprised - relieved - grateful perplexed (B) relieved - confused - curious - dejected - panicstricken (C) optimistic - lonely - calm - elated - appreciative (D) triumphant - reassured- inquisitive - thankful uncertain 



Ans. Option (B) is correct. Explanation: Following the pattern of risk: risky, the correct patterns would be danger: dangerous, hazard: hazardous, peril: perilous and caution: cautious. So, option (B) is correct.

Q. 2. Why do you think the woman in the control centre laughed? (A) She found the narrator funny. (B) She thought his question preposterous. (C) She thought he was teasing her. (D) She was relieved the narrator was safe.













Q. 5. Select the correct option to fill in the blanks below: risk: risky::__________:__________ (A) danger: dangerously (B) hazard: hazardous (C) peril: imperilled (D) caution: precaution

135



Statement II - The narrator was unaware of the threat that the adversarial storm clouds presented. Statement III - The narrator's decision making was quick, but irresponsible as well as dangerous. (A) Statement I is False, Statement II is True, Statement III cannot be inferred. (B) Statement I and III are True, Statement II cannot be inferred. (C) Statement I cannot be inferred, Statement II is False, Statement III is True. (D) Statement I and II are False, Statement III is True. Ans. Option (C) is correct. Explanation: The passage does not mention that the narrator was content / satisfied, that is complacent or not before reaching and meeting his family, so, Statement I cannot be inferred. The narrator was aware of the threat he might have to face. The phrases like ‘old Dakota’ and ‘I will take the risk’ indicate the fact. So, statement II is false. The narrator’s decision was impulsive, so statement III is true.







@PROCBSE 136

Oswaal CBSE Question Bank Chapterwise & Topicwise, ENGLISH LANG. & LIT., Class-X

(A) The black mountain-like storm cloud (B) The depletion of fuel in the last fuel tank (C) Being lost due to non-functioning equipment (D) The old rattling Dakota aeroplane Ans. Option (B) is correct. Explanation: The pilots are used to flying in bad

Q. 5. Choose the option that correctly matches the idioms in Column A to the story's events in Column B:













weather conditions also, so storm cloud was not a major threat. The non-functioning equipment was also not a threat. The narrator was comfortable in flying the Dakota, so he was not afraid of that too. The near empty fuel tank was a major threat to the safety of the narrator.

Column A - Idioms

Column B - Story events

1.

Every cloud has a silver (i) lining.

2.

To be on cloud nine.

3.

To have your head in the (iii) clouds.

The compass and other instruments stopped working. The radio was dead too.

4.

Gathering clouds.

(iv)

And there it was - the well-lit runway. An airport. The narrator could safely land.

(ii)

The narrator really wanted to have a hearty English breakfast, even though he really ought to have turned back. The man in the other plane waved at the narrator and asked him to follow closely, drawing him out.

(B) 1-(iii); 2-(i); 3-(iv); 4-(ii)







(C) 1-(i); 2-(ii); 3-(iii); 4-(iv) (D) 1-(ii); 2-(iv); 3-(i); 4-(iii) Ans. Option (D) is correct. Explanation: Every cloud has a silver lining means – every difficult situation has some hope – the narrator was lost in storm clouds and the black aeroplane came to guide him. To being on cloud nine means – being elated or feeling too happy – the narrator saw the airport and runway and landed safely. To have your head in clouds means – being absent minded – the narrator should have turned back but absentmindedly moved forward to have a hearty breakfast at home. Gathering clouds means – upcoming dangers – the equipment of narrator’s plane have stopped working.







(A) 1-(iv); 2-(iii); 3-(ii); 4-(i)

SUBJECTIVE TYPE QUESTIONS Short Answer Type Questions





Ans. The narrator seemed to lack his professional ethics and responsibility. He was about 150 km away from Paris when the clouds appeared. He didn’t have enough fuel to fly. Actually, he ignored his professional wisdom. The narrator was a grateful man. He went to the Paris control to ask who was the other pilot who saved his life. Q. 4. Why did the lady in the control room give the pilot U [Board Term I, 2016] a puzzled look? 



















ns. • No sensible man flies in such weather. A • Not seen any other plane on the radar. (CBSE Marking Scheme, 2016) Detailed Answer: The lady in the control room gave the pilot a puzzled look when he asked her whereabouts of black aeroplane because she knew that it was not possible for any sensible man to fly in such a bad weather. Moreover, no other plane was seen on the radar.



















Q. 1. As a reader, do you feel betrayed or let down in the way the story ends? Validate your opinion. A [CBSE-QB, 2021] Q. 2. Based on your reading, would you call the narrator a family man? Justify your answer. U [CBSE-QB, 2021] Ans. Indeed, the narrator is a family man. He was dreaming of his holidays with his family. He was looking forward to be with his family. He wanted to be in time for breakfast for which he took the risk of moving ahead in the storm. The narrator was a grateful man. He went to the Paris control to ask who was the other pilot who saved his life. Q. 3. James Lane Allen says, "Adversity reveals character". What do you learn about the narrator from the experience? Mention any two traits supported by the textual evidence. [CBSE-QB, 2021]





(40-50 words & 3 marks each)

These questions are for practice and their solutions are available at the end of the chapter

@PROCBSE

Long Answer Type Questions

(100-120 words & 6 marks each)





































Long Answer Type Questions

The old Dakota …….. I waved the pilot of the old Dakota and gestured him to follow me. Like an obedient child, he started to do so. He followed me until he could see the runaway. I was tensed whether he would land safely or not. Because he had already made me understand that he was running short of fuel. I saw him landing safely on the runway. I was relaxed. I felt there was no need to wait. The good deed of the day was done. Once again, I had rescued a plane stuck in storm. So, I decided to fly away from there in search of someone else who may be in need of my help. I, as the Marshal of the Air Force, had been using my skills of flying Air planes in the harshest



Ans.2.







weather since past ten years. Surprising the rescued ones by my mysterious entry and exists. Ans.3. February 28, 20XX France, 10.00 pm Who saved me when I had lost all the contacts on the radio with Paris control? Who was the pilot of the black aeroplane who helped me to land safely without a compass, without the radio and without much fuel? These questions had been haunting me since past few days but they still remain unsolved puzzle. Even the lady in the control room couldn’t detect any other plane on the stormy night other than my old Dakota on the radar. The mysterious pilot of the black aeroplane was like God’s help sent to me to land safely. I was in real trouble before the mysterious black aeroplane appeared before me. I was lost and to make things worse my compass became dead. Gods knows what would have been my condition, had the pilot of the black aeroplane not guided me out of storm clouds to safety.



Ans.1. ‘Black Aeroplane’ is a mysterious story with an open end. The readers are left free to guess the end. But it does convey a strong message of ‘Never say ‘die’ till the end. Never lose hope in the most hopeless situation.



Short Answer Type Questions









Solutions for Practice Questions





Compose the said excerpt of the story employing relevant details from 'Black Aeroplane'. [CBSE-QB, 2021] Q. 3. A few days after the experience, the narrator sat with his wife and recalled a long conversation he'd had with his young son the day before. The narrator's son had spoken to him about his close friend, Freddie and the many ways Freddie helped him. When the narrator suggested inviting Freddie home for lunch, his wife casually said, "Freddie is like your friend in the black aeroplane!" That night the narrator decided to record his thoughts in his diary to clear his head. As the [CBSE-QB, 2021] narrator, write the diary entry. Q. 4. Why and when did the narrator say that 'everything' was going well, it was an easy flight? (Black Aeroplane) [CBSE Board OD Set-II, 2020] Q. 5. Read the extracts given below and comment on the difference in the nature of help both the characters, of these two stories, received. (a) Suddenly I came out of the clouds and saw two straight lines of lights in front of me. It was a runway! An airport! I was safe! I turned to look for my friend in the black aeroplane but the sky was empty. [The Black Aeroplane] (b) All through the night Lencho thought only of his one hope: the help of God…The following Sunday, at daybreak he began to write a letter…It was nothing less than a letter to God...The following Sunday Lencho came a bit earlier than usual to ask if there was a letter for him…. Lencho showed not the slightest surprise on seeing the money, such was his confidence. [A Letter to God] [CBSE SQP, 2021] 









Q. 1. Imagine that the narrator shared his story with friends and family once he got home. One of them, a spiritual leader, decided to incorporate the story as part of his weekly sermon to the congregation. Another friend, a psychologist, worked the story into his next lecture on survival instinct and crisis management. Do you think the two interpretations of the story would be different? If so, how? What insights might the narrator get about his mysterious experience, if he were to attend both the sessions? [CBSE-QB, 2021] Ans. Yes, two interpretations of the story would be different. Spiritually, a person has deep faith in God and he recalls Him in the time of distress he will definitely be answered. God will surely send help through his messengers as done in chapter 1. A letter to god. The postmaster became the messenger of God. Whereas psychologically, it interprets ‘Never say die’ till the end. The narrator might also have got the insight of never to lose hope in the most hopeless situation. No one know who may come to save you. The narrator was hopeless when he was caught in the storm. He lost all contact with Paris control and all his instruments were dead. Then came a help from unknown source in the form of a mysterious pilot of the black aeroplane to guide the narrator to safety. Q. 2. In a parallel world, the pilot of the black aeroplane narrates his adventure in a story titled 'The Old Dakota'. You come across an excerpt from the story, which responds to the ending questions of 'Black Aeroplane' and describes the black aeroplane pilot's decision to fly away after watching the Dakota land.





137

TWO STORIES ABOUT FLYING

These questions are for practice and their solutions are available at the end of the chapter

@PROCBSE 138

Oswaal CBSE Question Bank Chapterwise & Topicwise, ENGLISH LANG. & LIT., Class-X

























Detailed Answer: Both Lencho and the pilot of the Dakota plane needed help and there is no doubt that both of them received the help too. But still there is a lot of difference between the help received by both of them. In 'The Black Aeroplane' the pilot was caught in the storm and it was difficult to come out of it. But suddenly, out of nowhere, a black aeroplane appeared and helped him come out of the storm. The black aeroplane was without lights but it steered the pilot through the dark clouds. He landed at an airport . He was so happy. He decided to thank the pilot of that mysterious aeroplane but was told by the lady in the control room that there was no other aeroplane in the air other than his. It is a mystery which was not resolved. The only thing was that he received an unexpected help and was thankful for it. On the other hand, in 'A letter to God', Lencho needed help in the form of pesos to look after his family. He wrote a letter to God and asked for hundred pesos. His conviction touched the postmaster who raised money from his staff and sent it to Lencho. In this case, Lencho received help as he had asked for it. The source of his aid has clearly been revealed whereas it was not revealed in the case of pilot unlike the pilot in The Black Aeroplane', Lencho is not happy and satisfied with the help that he had received. 

























Ans.5. Competency applied for: content • Required to display understanding of the variation of a common aspect, across the texts. (Movement of response from common element to the differences.) • Support of / reference to textual evidence expression • Applying logical progression, using appropriate linking devices (words/phrases showing comparison/contrast are expected to be a part of this answer- whereas, on the other hand, as opposed to etc. as they impact expression) cohesion • Response relevance and an appropriate conclusion to achieve coherence • Accuracy in use of grammatical structures, vocabulary and spellings NOTE—Paragraphing recommended. Just listing of points shall result in deduction of 1 mark allotted for cohesion & coherence. – Both need help – In The Black Aeroplane, he is helped by a stranger in a black aeroplane in the storm, without lights – steered through the dark clouds; was told there was no other aeroplane in the air other than his – wasn’t clear who helped him – open to interpretation – help is not asked for but he receives it. – is thankful – In A Letter to God-different nature of aid – Lencho writes a letter to God asking for money; his conviction in God touches the postmaster

instructions. After receiving the instructions, the pilot geared up and followed them while putting the last fuel tank into operation. He was calm as everything was going as planned. So, he says that 'Everything' was going well, it was an easy flight. – Postmaster raises money and sends it to him. – He asks for help – the source of his aid is clearly revealed – not satisfied (5 marks) [CBSE Marking Scheme, 2020]

















Ans.4. The story 'Black Aeroplane' is about a pilot who feels happy and contended to fly over a city during night time. He is flying from Paris to London. He is looking forward to enjoy his holiday with his family. As soon as he crosses Paris, he gets a look of dark clouds indicating an upcoming storm. But still his desire to reach his family and have a scrumptious breakfast with them, made him risk his and his passenger's life. Then the pilot thought of informing the Paris Control Personnel about its presence and for





REFLECTION Were you able to learn the underlying message "One should never lose hope" through the reading of the lesson?

@PROCBSE

CHAPTER

4

FROM THE DIARY OF ANNE FRANK – By Anne Frank

Revision Notes



Introduction This is a story of a young girl named Anne Frank. The story is based on her diary. Anne is a Jewish girl who has to go into hiding during the World War to avoid the Nazis. She shares her experiences in the story when she is depressed. The chapter is an excerpt from the ‘Diary of a Young Girl’ by Anne Frank.





A thirteen-year old school girl, Anne Frank was under some depression and despair. She thought of the saying, “Paper has more patience than people.” Then she started writing a diary but she was in need of a real friend and who could be more than a diary. The writer explains that no one believed that the girl was alone in the world because she was actually not alone. She had her loving parents, a sister and thirty other people. She had a decent family except her one true friend. With friends one can have a good time. We can talk of ordinary things everyday but we won’t get closer. Even we cannot confide in one another. Since the written facts cannot be changed, Anne started writing the diary. That was her everlasting friend. She called that friend ‘Kitty’. Anne wrote that her father was the most adorable person. At the age of thirty six, he got married to Anne’s mother, Edith. In 1926, her sister Margot was born. Then she was born on 12th June, 1929. They lived in Germany. In 1933, her father emigrated to Holland. She along with Margot, went to Aachen to stay with their grandmother. By December, both the sisters went to Holland. There, she started studying at the Montessori Nursery School. When she was in sixth class, Mrs. Kuperus was her headmistress. At the end of the year, there was a farewell function. The separation from head mistress was full of tears. Anne loved her grandmother very much. Her grandmother fell ill in the summer of 1941. She had









Summary

Scan to know an operation but she died in more about January, 1942. Her death was this topic all the more troublesome. At Anne’s birthday celebrations, a separate candle was lit for the grandmother. In her diary, Anne wrote that all the four members were doing well. She From the diary was much dedicated to her of Anne Frank diary. This event was written by Anne on 20th June, 1942 on Saturday. In her diary, Anne made a mention of her schoolexperience. The complete class was nervous about their going to the next form. Some of the students had made bets and staked all their savings. Regarding her, they were declaring ‘Pass’ but Anne was not sure of maths. All had been telling one another not to lose heart. There were nine teachers. Mr. Keesing taught Maths. He remained annoyed with Anne because of her talkative nature. So, he had given her some extra work, to write an essay on the title ‘A Chatter Box‘. After the homework, Anne started thinking on the essay. An idea flashed in her mind. She wrote “Talking is a student’s trait and I would do my best to control it. But I won’t be able to cure this habit since my mother is also talkative. So, moving from the inherited trait cannot be done.” On reading her arguments, Mr. Keesing had a good laugh. Then the teacher gave her another essay, ‘An Incorrigible Chatter Box‘. It was a sort of punishment for Anne for talking in class. Anne wrote it and handed it in. For two whole lessons, nothing went wrong. But in the third lesson, Mr. Keesing, again, was annoyed with her and gave her an essay to write on ‘Quack Quack Quack, said Mistress Chatterbox’. At this topic, the whole class roared. Anne too laughed. Though Anne tried for

@PROCBSE 140

Oswaal CBSE Question Bank Chapterwise & Topicwise, ENGLISH LANG. & LIT., Class-X

this essay but her friend Sanne became ready to help her. In a way the teacher was playing a joke on her but in other words it was a joke on him. So, Anne wrote this essay like a poem. Anne read the poem in the class. It stated, “There was a mother duck and a father Swan with three ducklings. The ducklings were beaten to death by their father since they quacked too much“. It was Anne’s good luck that the teacher took it in the right way. He read the poem, gave his own comments. After that Anne

was allowed to talk and no extra work was given. Since then, Mr. Keesing too started making jokes.

Key Words







1. Listless: With no energy or interest 2. Confide: To tell personal things privately to a person one trusts. 3. Quaking in its boots: Shaking with fear and nervousness.

Extract Based Questions

depict yawning.







(i) indolence

(iii) tolerance





Q. 4. If the diary were a 'real friend', what qualities would Anne expect it to have? Pick the option that lists these correctly. 1. Optimistic 2. Good listener 3. Confident 4. Energetic 5. Non-judgemental 6. Outgoing 7. Ambitious (A) 1 & 4 (B) 5, 6 & 7 (C) 2, 3 & 5 (D) 3 & 7 Ans. Option (C) is correct. Explanation: Anne wanted her friend to patiently listen to her problems, one who can help her in making decisions and would not judge her as good or bad. Option (C) is the correct option.





































Q. 5. Pick out the sentence that brings out the meaning of 'brooding' as used in the extract? (A) The mysterious house on the hill is still brooding above the village. (B) The people at the stock market always keep brooding about the gains. (C) He was brooding over the matter and took a long time to decide. (D) Suspense and drama were brooding at the site of the investigation. Ans. Option (C) is correct. Explanation: In the passage, brooding refers to thinking over some matter. In option (A), brooding









































(A) option (i) (B) option (ii) (C) option (iii) (D) option (iv) Ans. Option (A) is correct. Explanation: Indolence means laziness or idleness. Perseverance, Tolerance and Grit mean not getting tired and trying again and again – loosely meaning having patience.









Q. 2. Pick out the emoticon which clearly brings out the meaning of 'listless' as used in the extract?

(i) (ii) (iii) (iv) (A) Option (i) (B) Option (ii) (C) Option (iii) (D) Option (iv) Ans. Option (C) is correct. Explanation: Option (A) is to depict sleepiness, option (B) to depict sadness, option (C) is to depict cluelessness or listlessness and option (D) is to

(ii) perseverance

patience

(iv) grit





Q. 3. Pick the option that is ODD one out with reference to the meaning of 'patience'.



I. Read the following extract and answer the questions/ complete the sentences that follow: (5×1=5) "Paper has more patience than people. I thought of this saying on one of those days when I was feeling a little depressed and was sitting at home with my chin in my hands, bored and listless, wondering whether to stay in or go out. I finally stayed where I was, brooding: Yes, paper does have more patience and since I'm not planning to let anyone else read this stiff-backed notebook grandly referred to as a 'diary', unless I should ever find a real friend, it probably won't make a bit of difference." [CBSE-QB, 2021] Q. 1. 'Paper has more patience than people.' What does this imply? This implies that Anne: (A) believed in the power of writing more than speaking to people. (B) felt that she could pour her heart out on paper without any hindrance. (C) had more faith in sharing her thoughts and feelings with paper. (D) felt that she could share her feelings openly on paper. Ans. Option (B) is correct. Explanation: When one talks to people, they start reacting/suggesting/contradicting, etc., rather than show patience and listen to the feelings of the aggrieved. The paper, on the other hand, can not speak and interrupt or stop the writer to write down his/her inner feelings. 





(5 marks each)

@PROCBSE FROM THE DIARY OF ANNE FRANK

















Q. 2. What does 'had a good laugh' imply, in the context of Mr. Keesing? It means that he: (A) celebrated his ability to make Anne write the essay. (B) ridiculed Anne in front of the whole class. (C) pulled up Anne for her arguments in the essay. (D) realised the humour in it and was amused. Ans. Option (D) is correct. Explanation: Here, ‘Had a good laugh’ means laughing heartily in a positive sense. Mr. Keesing neither pulled up nor ridiculed Anne. He was not celebrating her ability to write essay as well. He realised the wittiness behind Anne’s remark.













Q. 3. Based on this extract, pick the option with the list of words that best describe Mr Keesing. (A) Jovial and creative (B) Strict and innovative (C) Tolerant and strict (D) Innovative and jovial Ans. Option (B) is correct. Explanation: Mr. Keesing was very particular about discipline in his class, so he was strict. Yet, he found new ways to make Anne realise that she is talkative by making her write essays. So, in that sense, he was innovative too. Jovial means cheerful which Mr. Keesing was not.





























Q. 5. How did Anne feel when she was punished the third time by Mr Keesing? She: (A) was happy as she had to write three essays on the same topic. (B) enjoyed making fun of Mr. Keesing in her own way.











Q. 4. Why do you think Mr Keesing chose the title - 'An Incorrigible Chatterbox' - for Anne, to write on? This was so because he expected: (A) Anne to express her inability to elaborate on such a topic. (B) that this would embarrass Anne and would check her indiscipline. (C) her to apologise and not repeat her talkative behaviour. (D) Anne to explore her creative writing skills. Ans. Option (B) is correct. Explanation: Mr. Keesing wanted Anne to realise that she was too talkative and not maintaining discipline in class. He wanted to embarrass her and maintain discipline in future classes.













II. Read the following extract and answer the questions/ complete the sentences that follow: (5×1=5) "Mr Keesing had a good laugh at my arguments, but when I proceeded to talk my way through the next lesson, he assigned me a second essay. This time it was supposed to be on 'An Incorrigible Chatterbox'. I handed it in and Mr Keesing had nothing to complain about for two whole lessons. However, during the third lesson, he'd finally had enough. "Anne Frank, as a punishment for talking in class, write an essay entitled-'Quack, Quack, Quack, said Mistress Chatterbox'." [CBSE-QB, 2021] 











(iii) (iv) (A) Image (i) (B) Image (ii) (C) Image (iii) (D) Image (iv) Ans. Option (B) is correct. Explanation: The word ‘chatterbox’ is used to refer to someone who can talk endlessly even when no one else is talking to him/her. This is denoted in option (B). In options (A) and (C), people are conversating with each other in pairs. In option (D) someone is addressing an audience.





(ii)



Q. 1. What convincing argument was made by Anne? (A) She was talkative just like any other student in the class. (B) She had the right to be talkative, as it was a classroom and not a prison. (C) She had inherited the trait from her mother, so couldn't stop being talkative. (D) She found it impossible to be quiet like the others as she couldn't change herself. Ans. Option (C) is correct. Explanation: Anne said that she had inherited talkativeness from her mother, who was equally talkative as Anne was.









Q. 7. Pick the option that lists the image that most appropriately corresponds to 'chatterbox'.

(i)

141



Q. 6. Anne doesn't plan to let anyone else read her diary. Which of the following is completely wrong with reference to the given statement? (A) She wanted to keep her secrets only for herself. (B) Her diary contained all her intimate feelings. (C) She was unwilling to share it with anyone. (D) She wants to cherish these moments by herself. Ans. Option (D) is correct. Explanation: She was an introvert. She didn't have any true friend and felt that paper had more patience than people. So she didn't want to reveal her intimate feelings to anyone. Her diary was a way to give vent to her feelings. The diary didn't have anything to do with cherishing those moments. So option (D) is most appropriate.



refers to rising high. In (B), it refers to worrying. In (C), it refers to thinking over the matter. In (D), it refers to spread.





@PROCBSE Oswaal CBSE Question Bank Chapterwise & Topicwise, ENGLISH LANG. & LIT., Class-X



Ans. Option (C) is correct. Explanation: Anne had innovatively written two essays with her original ideas, but after that she ran out of ideas. So, she was worried a bit about the third essay.



(C) was worried as she had run out of original ideas for her essay. (D) was thrilled at another opportunity to showcase her writing abilities.



142

SUBJECTIVE TYPE QUESTIONS Short Answer Type Questions



(40-50 words & 3 marks each)

Ans. DearestKittyblogger.com Anne Frank, 25 June, 1942 The Value of a True Friend Friendship is a virtue. It is a relationship of respect, peace and dignity. A true, sincere and sweet friend is worth than a thousand insincere fellows. He understands you and your feelings. A true friend is bonded by heart to you. You can pour yourself























Ans. Value Points: • Courage • Acceptance • Sense of freedom [CBSE Marking Scheme, 2016] 



Detailed Answer: Young Anne was a very intelligent and courageous girl. She had a flair for writing essays. She had the courage to accept that the trait of talkativeness was





C [CBSE-QB, 2021]



...................................................................................



...................................................................................



The Value of a True Friend ...................................................................................

Detailed Answer: Anne feels that it was fortunate that Mr. Keesing took the joke the right way because she had written a poem which was an allusion to Mr. Keesing. It had actually been written to turn the joke around on Mr. Keesing who could have taken it the other way also. She had written a poem referring to a father swan who bit his three ducks because they quacked too much. The message was conveyed metaphorically. Had Mr. Keesing taken it the other way round, the disciplinary action could have been severe. Q. 6. Anne justified her being a chatterbox in her essay. Do you agree that she had the courage to fight injustice? What values do you learn from Anne’s A [CBSE Term I, 2016] character through this?

DearestKittyblogger.com Anne Frank, 25 June, 1942



Ans. Content = 2 mark Expression = 1 mark (coherence and cohesion+ accuracy) – poem referred to the father swan who bit his three ducks because they quacked too much; allusion to Mr. Keesing – Poem written to turn the joke around on Mr. Keesing; metaphorically convey the value of talking – fortunate that Mr. Keesing got the joke, else, the disciplinary action could have been severe. [CBSE Marking Scheme, 2020]













Yours Sincerely Anne. Q. 3. Anne said that the only thing missing was the presence of a true friend. Imagine that Anne had access to the internet and had chosen to blog instead of writing a diary. As Anne, write a blog post on the value of a true friend.









Ans. Dear Mr. Keesing, It has been very fun loving time, understanding each other. Though you and I were not much understandable of each other yet I am going to miss those days. And about my talking traits, its going to remain the same. I’m surely going to send some more funny poems and essays to you to keep your company.



C [CBSE-QB, 2021]





Dear Mr. Keesing It has been..............................................................

out and share your feelings in front of him. Q. 4. Anne say that there were so many dummies in the class that about a quarter of them shouldn't be promoted. Is she being rude or brutally honest? C Evaluate. Q. 5. Annie writes that it was fortunate that Mr. Keesing took the joke the right way. Why does she feel so? [CBSE SQP, 2021] 













Q. 1. Do you agree with Anne when she says that teachers are the most unpredictable creatures on U [CBSE-QB, 2021] earth. Rationalise. Q. 2. Anne bid a 'heartbreaking farewell' to Mrs. Kuperus. As Anne, write a brief farewell note to Mr. Keesing after being promoted to the next class.

These questions are for practice and their solutions are available at the end of the chapter

@PROCBSE

FROM THE DIARY OF ANNE FRANK

that he decided not to punish her. It also reflects her fearlessness, critical thinking, humility and unbiased approach as well as her creativity and humorous approach to deal with her strict teacher. We should also learn such traits from her character and should have the courage to raise voice against injustice.

her birthright and she should have the freedom to enjoy it. When Mr. Keesing punished Anne for her talkativeness again and again, asked her to write essay, one after the other, she composed a poem on “Quack, Quack, Quack, said Mistress Chatterbox” and gave a message through it to the teacher. The teacher got so impressed by her little poem

Long Answer Type Questions

(100-120 words & 6 marks each)





Analyse the common character traits that would help in creating this special bond of friendship. A [CBSE-QB, 2021] Q. 4. Why did Mr. Keesing call Anne an ‘incorrigible R [CBSE SQP, 2020] chatterbox’? 



Some students failed to give a proper account of the required aspects from the entire story. A few students got mixed up with the different punishments given by Mr. Keesing.







Answering Tip











Commonly Made Error









Q. 2. You are stressed and anxious since your Annual results are going to be announced soon. You decide to meet the school counsellor to share your anxiety and apprehensions. Write the dialogue between you and your school counsellor. You may begin like this: School Counsellor: Hello Anne, please sit down. You look very disturbed and irritated. What's the matter? You: Well, to be honest, we all are disturbed. But the irritation is because of the two boys sitting behind .......................... School Counsellor: First of all, you need to relax. Take a deep breath. How do you feel we can deal with this situation rationally? You: ....................................................................... C [CBSE-QB, 2021] (continue) Ans. School counselor: Hello Anne, please sit down you look very disturbed and irritated. What’s the matter? 

















Positive reinforcement have indeed a great impact on student behaviour. In fact, it is considered one of the most effective strategies at producing measurable and sustainable behaviour change. One reason positive reinforcement is so efficient and effective is that it can be incorporated into all classroom activities including instruction, other classroom management systems, transitions and more. It gives teacher and other school personnel, the opportunity to make students feel safe, supported and successful at school. Positive reinforcement can be used as a strategy by the practitioners to ensure appropriate, student behaviour is being acknowledged and rewarded. Moreover, making reinforcement (positive) is a more powerful tool than punishment in most cases.

You: Well, to be honest, we all are disturbed. But the irritation is because of the two boys sitting behind me. They don’t allow me to concentrate on studies in class. Moreover they themselves did not study the whole year and now just when our Annual results are going to be announced soon they are worrying me by their negative thoughts. They are making bets. I’m really nervous. School counsellor: First of all, you need to relax. Take a deep breath. How do you feel we can deal with this situation rationally? You: Well, I had been working really very hard at home. In fact, I was well prepared for my Annual exams. I wanted to see my parents feeling proud on me. I was, even, very confident after my exams, that I’m going to pass with flying colours. But right now. I can’t bear the peer pressure. You know, the air around me is so uncertain and the teachers are quite unpredictable too. School counsellor: I think you are overthinking it. I thought you were a confident girl. Have faith in yourself, dear. Q. 3. Anne was a sensitive and mature girl. From the chapters and poems in your text book, First Flight, think of any two characters who could be her friends or confidantes.





In proposition



Ans.









Q. 1. Your teacher has organised a debate and you have been asked to speak on: 'Consequences and Positive Reinforcements Have a Great Impact on Student Behaviour'. Write the debate script with three-four points to supplement your stand, either as a proposition speaker or as an opposition one. C [CBSE-QB, 2021]

143

These questions are for practice and their solutions are available at the end of the chapter

Thorough reading of the chapter is a must. Writing practice can help the students to remember the facts.

@PROCBSE 144

Oswaal CBSE Question Bank Chapterwise & Topicwise, ENGLISH LANG. & LIT., Class-X

Solutions for Practice Questions Actually Anne was being brutally honest by saying so. The time of the declaration of the annual results was coming closer. The teachers were going to decide which students would be promoted to the next class. Anne and her friends laughed heartily at their classmate as they felt that many were dummies and should not be promoted to the next class.



Short Answer Type Questions

Ans.1. Mr. Keesing gave Anne a lot of homework for talking a lot in the class. When she presented an interesting poem, Mr. Keesing couldn’t help laughing. He appreciated her and stopped punishing her. Moreover no one in the class including Anne would know what there was in the minds of the teachers and what their next step would be.









easily be successful in creating a special boned of friendship with Anne. At the same time an argumentative mind of Anne could easily match the curious, disciplined and mature Valli. Ans.4. Anne was very talkative, didn’t stop despite being punished, wrote an essay as a punishment, justified her over-talkative nature. (Board Marking Scheme, 2020) Detailed Answer: Anne was a very talkative girl. Mr. Keesing, her maths teacher, didn’t like this habit of hers. He punished her by giving her an essay to write on the topic ‘ A Chatter Box.’ She wrote in her essay, “Talking is a student’s trait and I would do my best to control it, but I won’t be able to cure this habit since my mother is also talkative. So moving away from the inherited trait cannot be done”. On reading her arguments, Mr. Keesing had a good laugh. In other words, she had justified her over-talkative nature.





Ans.3. According to me, the two characters who could be Anne’s friends or confidantes are Wanda Petronski and Valli. Both Wanda and Valli are curious, intelligent and meticulous planners, just like Anne. Though Valli is younger than the other two but still she proves to very impressive and bold by finally deciding to ride in the bus. Alike Anne Valli’s childish innocence veiled by her smart and bold outlook amuse the people around her. Valli thoroughly enjoys, her surroundings. But the sight of the dead cow haunted her and dampened her spirits and saddened her. This is a typical tendency of a matured person. On the other hand, Wanda to be a generous, loving and mature girl who is very large hearted. She is of obliging and forgiving nature. She is not ‘dumb’ but a highly sensitive girl. She is not vindictive at all. Hence, the forgiving and mature Wanda would



Long Answer Type Questions





REFLECTIONS Were you able to relate with Anne and her feelings after reading this excerpt from her diary?

@PROCBSE

CHAPTER

5

GLIMPSES OF INDIA – Lucio Rodrigues

Topic-1 A Baker from Goa Revision Notes Introduction

Scan to know thing. The lady of the house more about prepared sandwiches on the this topic occasion of her daughter’s engagement. In those days, the bread sellers wore a particular dress known as ‘Kabai’. It was a single piece A Baker long frock up to the knees. from Goa Even today, they can be seen wearing a half pant that reaches just below the knees. People usually comment that he is dressed like a ‘pader’. Baking was a profitable profession in the olden days. The baker and his family never starved and they looked happy and prosperous.



‘A Baker from Goa’ is a pen portrait of a traditional Goan village baker who still has an important place in his society. The narrator is travelling down the memory lane thinking about the loaves of bread, a baker delivered at his home, in his childhood, every morning.

Summary

Key Words

1. Bol: Sweet bread 2. Starved: extremely hungry





Goa is very much influenced by the Portuguese. Their traditional work can be still seen there. The Portuguese are famous for preparing loaves of bread. The writer talks about his childhood days in Goa when the baker used to visit their home as their friend. He used to visit the house twice a day. In the morning, his jingling sound of the bamboo woke them from sleep. They all ran to meet him. The loaves were purchased by the man-servant of the house. The villagers were much fond of the sweet bread known as ‘bol’. The marriage gifts were meaningless without it. So, the bakers’ furnace in the village was the most essential

Extract Based Questions



are still there. We still have amongst us the mixers, the moulders and those who bake the loaves. Those age-old, time-tested furnaces still exist. The fire in these furnaces has not yet been extinguished. The thud and jingle of traditional baker’s bamboo, heralding his arrival in the morning, can still be heard in some places.“ [CBSE-QB, 2021] 



I. Read the following extract and answer the questions/complete the sentences that follow: (5×1=5) “Our elders are often heard reminiscing nostalgically about those good old Portuguese days, the Portuguese and their famous loaves of bread. Those eaters might have vanished but, the makers 





(5 marks each)

@PROCBSE Oswaal CBSE Question Bank Chapterwise & Topicwise, ENGLISH LANG. & LIT., Class-X











































Q. 3. Pick the idiom that brings out the same meaning of ‘reminiscing’ as used in the passage. (A) Train of thought. (B) Commit something to memory. (C) A trip down memory lane. (D) Jog somebody’s memory. Ans. Option (C) is correct. Explanation: Reminiscing, means recalling or recollecting old thoughts or memories.

Q. 1. Choose the answer that lists the correct option about the recording of the baker’s monthly accounts.









II. Read the following extract and answer the questions/ complete the sentences that follow: (5×1=5) The baker or bread-seller of those days had a peculiar dress known as the kabai. It was a single piece long frock reaching down to the knees. In our childhood, we saw bakers wearing shirt and trousers which were shorter than full-length ones and longer than half pants. Even today, anyone who wears a half pant which reaches just below the knees invites the comment that he is dressed like a pader! [CBSE-QB, 2021] 



















Q. 2. ‘Those eaters might have vanished but the makers are still there’. Pick the option that expresses the tone of the narrator. (1) Elated (2) Morose (3) Nostalgic (4) Hopeful (5) Sarcastic (6) Critical (7) Celebratory (A) (1) and (7) (B) (2) and (6) (C) (3) and (4) (D) (4) and (5) Ans. Option (C) is correct. Explanation: The narrator is feeling sentimental and yearning (nostalgic) and optimistic and positive (hopeful). The narrator is feeling nostalgic and sentimental remembering good old days and is feeling hopeful to know that some bread makers are still there.

(C) The furnaces are very strong and cannot be shifted for use in other areas. (D) The fire in the furnaces takes a long time to cease burning, once lighted. Ans. Option (A) is correct. Explanation: The fire in the furnaces has not yet been extinguished here refers to that they are still in use at present. It does not mean replacement of fire or strength of furnaces or duration for which furnaces keep burning.



Q. 1. The narrator says that the furnaces were ‘timetested’ because: (A) they had been thoroughly tested each time, before being used. (B) they had proved the test of time and were working well. (C) they had been tested by modern day experts. (D) they had the power to withstand inexperienced usage. Ans. Option (B) is correct. Explanation: Time-tested means that the furnaces have been used for a long period and have proved their well-working with passage of time.



146

















Q. 3. The statement that is TRUE about payment collection, according to the passage is: (A) The baker received payment on a daily basis. (B) The baker was paid for his services at the end of the month. (C) The baker insisted that customers pay before the month-end.























Q. 5. The ‘fire in the furnaces has not yet been extinguished’ implies that: (A) The furnaces are still being used to bake the loaves of bread. (B) The fire is in the process of being reviewed as a replaceable method for heating furnaces.

Q. 2. When the writer says, ‘Baking was indeed a profitable profession in the old days’, he means that: (A) baking isn’t as popular in Goa currently. (B) bakers have chosen to adopt other professions. (C) baking, as a job, isn’t as gainful as it used to be. (D) bakers’ goods were of a better quality in earlier times. Ans. Option (C) is correct. Explanation: This means that in the past, the bakers could earn more profits and now-a-days, baking is not as profitable or gainful.

























Q. 4. Why do you think the baker came in with ‘a thud and a jingle’? (A) He wanted to make everyone alert and active with his presence. (B) He wanted to wake up everyone from their slumber and ask them to visit the bakery. (C) He was used to make a loud noise as most people responded to just that. (D) He wanted to make people aware that he had come around to sell his goodies. Ans. Option (D) is correct. Explanation: He wanted to attract attention of the people with ‘a thud and a jingle’ towards the goods which he had brought with himself for selling.

(A) Option (1) (B) Option (2) (C) Option (3) (D) Option (4) Ans. Option (C) is correct. Explanation: Baker used to write the accounts on a wall. He did not merely keep it in mind or write it on paper or hand.

@PROCBSE

GLIMPSES OF INDIA



















Q. 2. Why did the baker and his family never starve? (A) They have enough to eat. (B) They were very rich. (C) They had servants. (D) Baking was a profitable business. Ans. Option (D) is correct. Explanation: This was because in the past, the baking profession was counted as a profitable business which gives huge income to the baker.











Q. 4. The records were maintained ______________. (A) daily (B) weekly (C) fortnightly (D) monthly Ans. Option (D) is correct. Explanation: As the baker received payment at the end of the month, so the records were maintained monthly.



























Q. 5. Which word in the extract is a synonym of ‘rich’? (A) Wealthy (B) Testimony (C) Profitable (D) Prosperous Ans. Option (D) is correct. Explanation: Wealthy means rich but has not been used in the passage. Testimony means witness and profitable means something that gives good returns. Prosperous means having lot of wealth, thereby meaning rich.











III. Read the following extract and answer the questions/ complete the sentences that follow: (5×1=5) The baker usually collected his bills at the end of the month. Monthly accounts used to be recorded on some wall in pencil. Baking was indeed a profitable profession in the old days. The baker and his family never starved. He, his family and his servants always looked happy and prosperous. Their plump physique was an open testimony to this. Even today,



Explanation: The guests, the troubles and the applications can be invited. Prayers are not invited, but they are offered.









Ans. Option (B) is correct.





(D) Option (4)



(B) Option (2)

(C) Option (3)



(A) Option (1)















Q. 3. A baker in Goa can be identified _______________ . (A) with his attire (B) with a jack fruit like physical appearance (C) with this plump physique. (D) Both (B) and (C) Ans. Option (D) is correct. Explanation: Plump physique means healthily built. Jackfruit like appearance was associated with bakers. So, both (B) and (C) are correct options.











Q. 5. The extract uses the phrase, ‘invites (the) comments’. Which of the following expressions is INCORRECT with respect to the word ‘invites’?



any person with a jackfruit like physical appearance is easily compared to a baker. [CBSE-QB, 2021] Q. 1. Where did the baker record his accounts? (A) In a diary (B) On a computer (C) On the wall in pencil (D) In his memory Ans. Option (C) is correct. Explanation: The line in the passage ‘recorded on some wall in pencil’ tells the answer.





Q. 4. The kabai was a ‘peculiar’ outfit as it was: (A) a tight-fitting apparel. (B) too colourful. (C) made of unsuitable materials. (D) a dress-like attire. Ans. Option (D) is correct. Explanation: The kabai was not a dress in the real sense, but it resembled a special or peculiar type of attire which can be used as a dress.







(D) The baker chose to receive payment any day of the month. Ans. Option (B) is correct. Explanation: The baker had to keep an account of purchases made by everyone and he was paid for his services at the end of the month.

147

SUBJECTIVE TYPE QUESTIONS Short Answer Type Questions











and bolinhas were a must for Christmas as well as other festivals. These were made with the bread. Q. 2. Inspired by the diversity in the chapter, ‘Glimpses of India’, you wrote an article for your school magazine on the topic, ‘Diversity-the Uniqueness of India’.

Q. 1. How did the baker become synonymous with celebrations and occasional in Goa? [CBSE-QB, 2021] Ans. The village baker was very important for festive occasions. Marriage gifts were meaningless without the sweet bread known as ‘bol’. Sandwiches, cakes





(40-50 words & 3 marks each)

@PROCBSE 148

Oswaal CBSE Question Bank Chapterwise & Topicwise, ENGLISH LANG. & LIT., Class-X















Ans. Needed for all sorts of functions, like marriages, birthdays, part of traditional cuisine, mothers prepare delicacies. (CBSE SQP Marking Scheme, 2020)

Detailed Answer: Bread is an important part of the Goa’s life because it is needed for all sorts of functions whether they are birthdays, marriages or festivals like Christmas. These occasions are incomplete without a special preparation which is made from bread for each event. Q .4. What do you learn about the financial condition of the bakers of Goa? [CBSE OD Set - I, 2017] Ans. The Bakers of Goa led a prosperous life. Baking was a profitable profession. They never starved. His family and servants always looked happy and prosperous. Their plump physique was an open testimony to this. Q 5. What do the elders in Goa still love to remember? [Term-II Delhi Set-I, 2017]























Write a paragraph, sharing two key opinions from the article. [CBSE-QB, 2021] Ans. Diversity makes India unique. India is unique country in every possible way and is colourful too. Festivals and dressing are two of them. Every culture, religion and custom has its own unique colour, which shows the diversity in one. In India, different people have different culture, speak different languages have different customs, castes, history, religions, which make India unique. Q. 3. How is bread an important part of life in Goa? [CBSE SQP, 2020]

Long Answer Type Questions



(100-120 words & 6 marks each)



























Commonly Made Error Most of the students missed the local words used in the chapter such as Kabai, bol, pader etc.

Answering Tip













and ride on the slow moving bulls. He says that his was a ‘transitory freedom’ which he enjoyed the most. On the contrary, during the narrator’s childhood in Goa, the baker used to be his friend, companion and guide. He used to come at least twice a day. The author and his friends used to climb a bench or the perpet and peep into the basket somehow. The author can still recall the typical fragrance of the leaves. Then he and his friends didn’t even care to brush their teeth. Thus, the narrator had carefree and enjoyable childhood which was full of childish activities whereas Nelson Mandela’s childhood was actually his training to become a good leader. Q. 3. Why were the children fascinated by the baker? How did they show their eagerness to see him? [CBSE OD Set-III, 2020]













Q. 1. The narrator shares, “Baking was indeed a profitable profession in the old days.” (a) What do you feel has changed now? Why? (b) State any one way, you feel, the paders can regain their lost glory. [CBSE-QB, 2021] Ans. (a) The baker or the pader used to be an essential part of the Goans’ life. With the passage of time, the bakers continued their profession but with their reduced fortune and importance. The profession of baking loaves of bread has not died with the end of the Portuguese rule. Goa still has the mixer, the moulders and the baker of those loaves of bread. The furnaces still bake those unique loaves. Marriages, feasts, Christmas and other festivals are still meaningless without the sweet bread called ‘bol’. However, the tradition hasn’t died completely yet. (b) The paders can regain their lost glory by consistent hard work. They need to be as traditional as they were earlier. As the family tradition is still carried on even today by the new generation of bakers. Soon, they will regain their lost glory. Q. 2. In the chapter, ‘A Baker from Goa’ the narrator talks about his childhood in Goa and his fond memories. Compare the childhood of Nelson Mandela with that of the narrator. [CBSE-QB, 2021] Ans. Both the narrator and Nelson Mandela have pleasant memories of their childhood. In his family, Mandela was the first one to gain a formal education. The concept of freedom was limited only to run in the field, swim in the local stream

These questions are for practice and their solutions are available at the end of the chapter

A vigorous reading of the text followed by writing practice is suggested.

@PROCBSE

GLIMPSES OF INDIA

149

Solutions for Practice Questions (Topic-1) for peeping into his basket and giving respect to the elders). He was very informal with the children. The children of Goa considered the baker as a friend and companion. He was not simply a vendor. The children ran to meet him not because of their love for the loaf, which was bought by the servant of the house. They actually longed for the bread bangles. The noise from his bamboo stick would wake the children up and they showed their eagerness to meet him by running out to greet him.

Short Answer Type Questions

Ans.5. The elders in Goa are nostalgic about the good old Portuguese days and the Portuguese loaves of bread. The Portuguese were very famous for their bread.

Long Answer Type Questions

Ans.3. The baker was a very respected person in the Goan society because he would guide the children about good behaviour (when he mildly rebuked them







REFLECTIONS Were you able to understand the rich culture and diversity of India? Were you able to know about the importance of Bakers in Goa?

Topic-2 Coorg

– Lokesh Abrol

Revision Notes

‘Coorg’ is a coffee producing area in Karnataka State of India. It is situated midway between Mysore and the coastal town of Mangalore. This land is famous for its rainforests and spices. The writer seems to be fascinated by the beauty of the place and says that it must have come from the Kingdom of God. It is the smallest district of Karnataka.





Introduction



Coorg is a heavenly place which lies midway between Mysore and Mangalore. It is the smallest district in Karnataka and has evergreen forests, spices and coffee plantations. The best season is between September and March when the weather is perfect for a visit to Coorg. The people are of Greek or Arabic descent. It is rumoured that a part of Alexander’s army drifted here and found it impossible to return. They married among the locals, so their traditions and rites may be different from other Indians. Some people say that Coorgis are of Arabic descent as many people wear a long black coat with







Summary

Scan to know embroidered waist belt which more about is similar to the kuffia worn by the Arabs. this topic The people of Coorg are known for their hospitality and recount many tales of bravery. General Cariappa, the first Army Chief was a Coorgi. The Kodavus are the Coorg only people in India to carry firearms without a license. A variety of wildlife like the Mahseer– a large fresh water fish, kingfishers, squirrels, langurs and elephants can be seen here. Coorg is also well-known for high energy adventures like river rafting, canoeing, rappelling, rock-climbing, etc. The Brahmagiri hills give the climbers an aweinspiring view of Coorg. A walk across the rope bridge leads to the sixty-four acre island of Nisargadhama. Bylakuppe in Coorg, is India’s largest settlement of Buddhist monks. These Buddhist monks can be seen here dressed in red, ochre and yellow robes.

@PROCBSE 150

Oswaal CBSE Question Bank Chapterwise & Topicwise, ENGLISH LANG. & LIT., Class-X

Extract Based Questions































Ans. Option (A) is correct. Explanation: The Coorgis are famous for their bravery, valour and courage, but their homes are famous for the friendliness, kindness and generosity they offer to the visitors. Q. 5. Which word in the passage means the same as ‘friendliness’? (A) Tradition (B) Valour (C) Hospitality (D) Regiment Ans. Option (C) is correct. Explanation: Tradition means something which has been followed for long. Valour means bravery. Regiment means an army battalion. Hospitality means friendliness.





















Q. 4. What is the tradition of coorgi homes? (A) hospitality (B) bravery (C) valour (D) courage







































Q. 2. Who was the first chief of the Indian Army? (A) A coorgi (B) General Cariappa (C) Kodavus (D) Both (A) and (B) are correct. Ans. Option (D) is correct. Q. 3. What is the special favour granted to the kodavus even now? (A) They can carry firearms with licence. (B) They can carry firearms without licence. (C) They can travel around the nation without licence. (D) None of the above is correct. Ans. Option (B) is correct. Explanation: Owing to their bravery and valour, the Kodavus have been given special permission to carry firearms without licence.

































































Q. 4. Coorg is famous for ____________. (A) coffee (B) tea (C) cotton (D) rubber Ans. Option (A) is correct. Q. 5. How much of Coorg is covered with rainforest? (A) Thirty percent (B) Forty percent (C) Fifty percent (D) Seventy percent Ans. Option (A) is correct. II. Read the following extract and answer the questions/ complete the sentences that follow: (5×1=5) Coorgi homes have a tradition of hospitality and they are more than willing to recount numerous tales of valor related to their sons and fathers. The Coorg Regiment is one of the most decorated in











Q. 3. Which is the best season in Coorg? (A) January to March (B) April to September (C) September to March (D) November to January Ans. Option (C) is correct. Explanation: The season of September to March is referred to as the season of joy so it is considered as the best season to visit Coorg.

the Indian Army and the first Chief of the Indian Army, General Cariappa, was a Coorgi. Even now, Kodavus are the only people in India permitted to carry firearms without a licence. Q. 1. What kind of stories are the Coorg people always ready to tell? (A) Tales of valour of their forefather. (B) Tales of bravery and courage of their sons. (C) Tales of their own bravery. (D) Both (A) and (B) correct. Ans. Option (D) is correct. Explanation: According to the given passage, Coorgi people told stories about their father’s, forefathers’ and sons’ bravery and courage.



I. Read the following extract and answer the questions/ complete the sentences that follow: (5×1=5) Coorg or Kodagu, the smallest district of Karnataka, is home to evergreen rainforests, spices and coffee plantations. Evergreen rainforests cover thirty per cent of this district. During the monsoons, it pours enough to keep many visitors away. The season of joy commences from September and continues till March. The weather is perfect, with some showers thrown in for good measure. The air breathes of invigorating coffee. Coffee estates and colonial bungalows stand tucked under tree canopies in prime corners. Q. 1. Coorg is a district in ___________. (A) Kerala (B) Andhra Pradesh (C) Karnataka (D) Tamil Nadu Ans. Option (C) is correct. Q. 2. Which word in the extract means ‘begins’? (A) Commences (B) Measure (C) Tucked (D) Canopies Ans. Option (A) is correct. Explanation: Measure means to ascertain size, degree, quantity, etc., of something. Tucked means pushed, folded or turned something to hide it. Canopies are ornamental cloth held up over something. Commences means to start something. 





(5 marks each)

@PROCBSE

151

GLIMPSES OF INDIA

SUBJECTIVE TYPE QUESTIONS Short Answer Type Questions











































there at some point of time. Cultural assimilation adds flavour to the existing structure of a society. Summarise your opinion on the given idea. [CBSE-QB, 2021] Ans. The brave and independent people of Coorg are of Greek or Arabic descent. According to a story, a part of Alexander’s army did not return and was settled here. They married among the locals. This culture assimilation can be seen in the martial tradition, marriage and religious customs. Q. 5. Why is Coorg called the land of rolling hills? (CBSE, Delhi Set I 2020) Ans. Coorg is called the land of rolling hills because the city is situated on the gentle sloping hills. The entire area is covered with these hills. Brahmagiri hills has a panoramic view. Q. 6. The people of Coorg have a tradition of courage and bravery. How has it been recognised in modern India? [CBSE 2015, Set 2/1/3] OR How is the Coorgi tradition of courage and bravery recognised in modern India?

Q. 1. Why does the author in ‘Coorg’ say that the visitors’ search for the heart and soul of India would be found in Coorg? [CBSE-QB, 2021] Ans. Coorgis are famous for hospitality, valor which is the heart and soul of India. Apart from it, Coorg is a beautiful region of rolling hills and lush green rainforests. It is also famous for coffee and spices. Q. 2. Do adventure sports like river rafting and rock climbing require a person to possess just physical strength? Why/Why not? [CBSE-QB, 2021] Q. 3. Coorgis belong to a valorous and hospitable race. Comment on this statement with reference to the text. [CBSE-QB, 2021] Ans. Coorgis are a proud race of martial men and beautiful women. They are very hospitable and entertain their guests by relating stories of bravery of their sons and fathers. Coorgis soldiers are brave. Coorgis are the only people who are permitted to carry firearms without a license. Q. 4. The culture, lifestyle and traditions of a place are influenced by the people who lived or settled





(40-50 words & 3 marks each)



Q. 7. Which season is the best to visit Coorg?





Ans.



Topper Answer, 2019

Long Answer Type Questions



(100-120 words & 6 marks each)

These questions are for practice and their solutions are available at the end of the chapter



Ans. As I stepped into the land of evergreen forest, I was blown away by the pristine beauty and lush green flora of the rain forests of Coorg. This place is really a piece of heaven that must have drifted from the kingdom of God. This land of rolling hills is inhabited by a proud race of martial men, beautiful women, coffee and wild elephants. The rain forests cover more than thirty percent of this district. Often it rains heavily. September is the most ideal time to enjoy the natural beauty of this place. The river Kaveri proves to the cherry on the cake. It obtains its water from the hills and rainforests of Coorg. The region abounds in langurs, bees, butterflies









Q. 1. Pen down a brief travelogue* entry, narrating any personal impression/s about a visit to Coorg. Include your reflections about the differences between the place where you live and the place you’ve visited. A travelogue is a person’s account of a journey to another country or place. It can either be a written report with many factual details or a narrative story about personal impressions and experiences.) You may begin like this... As I stepped into the land of evergreen forest, I was blown away by the pristine beauty and ................... (continue) [CBSE-QB, 2021]

(CBSE 2015, Set 2/1/3)

@PROCBSE 152

Oswaal CBSE Question Bank Chapterwise & Topicwise, ENGLISH LANG. & LIT., Class-X

Q. 3. ‘Coorg’ seems to share a lot of historical perspectives. Do you think knowledge of such historical facts is important for us? [CBSE-QB, 2021] Ans. In the lesson ‘Coorg’, the author tells us that the Coorgi people are descendants of either the Greeks or the Arabs. He says that they could have Greek origin because there was a legend that a part of Alexander’s army had moved to south and settled here. According to another legend, they had Arab origin because of the dresses they wore. The author also tells us that the first General of the Indian Army, General Cariappa was a Coorgi. It is important for us to know about the deep rooted cultural background of our country and heritage so that we become aware of the fact that our culture has always welcomed people from other cultures and absorbed them into its own. This spreads the message of being tolerant which is very important in today’s world when the world is torn with war because of these ethnic reasons.























and a lot of birds. The Brahmagiri hills provide a panoramic view of the entire misty landscape of Coorg. Moreover, the sixty-four acre island of Nisargadhama is a little paradise for the tourists. Q. 2. What do the people of Coorg teach us? [CBSE-QB, 2021] Ans. Coorg people are famous for their hospitality, love, respect and fellow feeling which is the need of the present hour. The adventurous sports like river-rafting, canoeing, rappelling, rock climbing and mountain biking are great motivators. The Coorg Regiment is famous for receiving maximum number of awards for its bravery. The first Chief of Indian Army, General Cariappa was a Coorgi. The Coorgis are more than willing to recount numerous tales of valour related to their sons and fathers. Even now, Kodavus are the only people in India permitted to carry firearms without a license.

Solutions for Practice Questions (Topic-2) 



Detailed Answer: The season of joy that commences from September and continues till March is the best to visit Coorg as some showers make the weather perfect and the air becomes fragrant with the aroma of coffee.





Ans.2. Apart from physical strength, a person does require many other skills. The person must be zealous, confident courageous and flexible. He should have excellent public speaking skills. He should not only have a good memory but also good destination knowledge.



Ans.7. • September to March, the weather is perfect/ pleasant. (Board Marking Scheme, 2015)

Short Answer Type Questions







REFLECTIONS Did you enjoy the description of the evergreen forests, spices and coffee and plantations of Coorg? Did you come to know in which month should you visit Coorg?

Topic-3 Tea from Assam

– Arup Kumar Datta

Revision Notes

This is a very short description of Assam, a NorthEastern State in India. This state is famous for its tea plantations. In this extract Pranjol, a youngster from Assam is Rajvir’s classmate at a school in Delhi. Pranjol’s father is a manager of a tea-garden in upper Assam and Pranjol has invited Rajvir to visit his home during the summer vacation.

Summary



Introduction

‘Tea from Assam’ is an interesting story about tea, its history and significance. Two boys Rajvir and Pranjol are travelling to Assam. Rajvir tells Pranjol that over 8,00,000,000 cups of tea are drunk every day throughout the world.

Scan to know more about this topic

Tea from Assam

@PROCBSE

GLIMPSES OF INDIA

These words ‘Chai’ and ‘Chini’ are Chinese words. It was only in the sixteenth century that tea came to Europe. By now, they had reached Marian junction where they got down and set off for Dhekiabari Tea Estate. On both sides of the road, there were tea bushes with women plucking tea leaves. Pranjol’s father told Rajvir that he would tell them many more things about tea plantations.





The train passes through green hills with a sea of tea bushes as far as can be seen. Rajvir is very excited but Pranjol, who has been brought up on a plantation, does not share his excitement. Rajvir then tells him about the various legends—Indian and Chinese—behind tea. He tells him how a Chinese emperor by chance discovered tea, back in 2700 BC. Another story was about how ten tea plants grew out of eyelids of Bodhidharma, a Buddhist ascetic.

Extract Based Questions



(5 marks each)





































    

   







   







Q. 5. Based on this extract, how do you think Rajvir felt while narrating? (A) (i) excited (ii) agitated (B) (i) hysterical (ii) nervous (C) (i) nervous (ii) agitated (D) (i) enthusiastic (ii) passionate











Q. 4. Based on the inference from the extract, which of these is NOT TRUE about tea drinking in the sixteenth century Europe? Dr. Smith is a doctor of sixteenth century Europe. (A) Dr. Smith encouraged drinking of green tea whenever available, to reduce chances of tooth loss. (B) Dr. Smith prescribed regular tea drinking to all his patients with a weak heart. (C) Dr. Smith always served tea as refreshment when he had guests, as they all enjoyed this beverage. (D) Dr. Smith usually recommended black tea to reduce inflammation in the body. Ans. Option (C) is correct. Explanation: In 16th century Europe, tea was consumed as a medicine and not as a beverage.

































(2) It increases levels of a neurotransmitter called gamma-aminobutyric acid (GABA) and improves overall sleep quality by shortening the time it takes to fall asleep and decreasing night-time awakenings. (3) It interferes with REM sleep, has some unwanted side effects, keeps sleep away and allows the possibility of inducing hours of sleeplessness and increased night-time awakenings. (4) It alleviates anxious thoughts and soothes the spirit before bedtime. It improves energy levels and helps banish stress and results in a better night’s sleep, naturally. (A) Option 1 (B) Option 2 (C) Option 3 (D) Option 4 Ans. Option (C) is correct. Explanation: Options 1,2,4 state the fact that tea initiates/ decreases/ results in better night sleep, whereas option 3 states that tea keeps sleep away.





I. Read the following extract and answer the questions/ complete the sentences that follow: (5×1=5) “Tell me another!” scoffed Pranjol. “We have an Indian legend too. Bodhidharma, an ancient Buddhist ascetic, cut off his eyelids because he felt sleepy during meditations. Ten tea plants grew out of the eyelids. The leaves of these plants when put in hot water and drunk, banished sleep.” “Tea was first drunk in China,” Rajvir added, “as far back as 2700 B.C.! In fact, words such as tea, chai and chini are from the Chinese. Tea came to Europe only in the sixteenth century and was drunk more as medicine than as beverage.” [CBSE-QB, 2021] (5 × 1 = 5) Q. 1. The main idea of this extract is: (A) Tea as a popular beverage in Europe and how it spread. (B) Origin of tea in India and why it became popular in Europe. (C) Importance of India in popularising tea and influencing Europe. (D) Indian legend on tea and how it travelled from China to Europe. Ans. Option (D) is correct. Q. 2. Why do you think Pranjol ‘scoffed’? (A) He was upset with the legend Rajvir shared. (B) He was mocking Rajvir for his lack of knowledge. (C) He was amused and tickled at what Rajvir shared. (D) He was impressed with what Rajvir had shared. Ans. Option (C) is correct. Explanation: Scoffed means ridiculed or mocked. So, Pranjol was amused and tickled at what Rajvir shared. Q. 3. Pick the option that includes the tea label information that corresponds to the given sentence. “The leaves of these plants when put in hot water and drunk, banished sleep.” (1) Its calming effects may be attributed to an antioxidant called apigenin, which is found in abundance in chamomile tea. Apigenin binds to specific receptors in your brain that may decrease anxiety and initiate sleep.

153

@PROCBSE







‘Chai garam .. garam chai’ a vendor called out in a high pitched voice. He came up to their window and asked, ‘Chai, sa’ab’? ‘Give us two cups,’ Pranjol said. They sipped the steaming hot liquid. Almost everyone in their compartment was drinking too.





(C) Fifth century

(D) Eighteenth century Ans. Option (B) is correct. Explanation: Prior to 16th century, Europeans were not aware of tea. Even in 16th century, Europeans consumed tea as a medicine and not as a beverage.

























































(B) Sixteenth century





Q. 5. When did tea come to Europe?















(C) Legend (D) Beverage Ans. Option (D) is correct. Explanation: Clattered means smashed, Junction means joint or intersection, Legend means famous story or person and Beverage means something that can be drunk.







Junction





(B)

(A) Seventeenth century













(A) Clattered















Ans. Option (C) is correct. Q. 4. Which word in the extract means ‘a drink’?



(D) Japan









(B) Europe

(C) China







Ans. Option (B) is correct. Q. 3. Where was tea first drunk? (A) Russia





(D) Limbs





(C) Ear

Eyelids







(B)













Ans. Option (A) is correct. Q. 2. What did he cut to prevent feeling sleepy? (A) Hair



Jain

(D) none of the above









(C) Chinese



(B)





(A) Buddhist

Do you know that over eighty crore cups of tea are drunk every day all over the world’? Rajvir said ‘When!’ exclaimed Pranjol. “Tea really is very popular. The train pulled out of the station. Pranjol buried his nose in his detective book again. [CBSE-QB, 2021] Q. 1. Who was speaking in a high pitched voice? (A) Rajvir (B) Tea vendor (C) Pranjol (D) Mr. Barua Ans. Option (B) is correct. Q. 2. Who ordered two cups of tea? (A) Mr Barua (B) Rajvir (C) Stranger (D) Pranjol Ans. Option (D) is correct. Q. 3. Which of the statements given below is NOT TRUE about the two friends –Rajvir and Pranjol? (A) Pranjol was a youngster from Assam. (B) Rajvir did not like reading detective stories (C) Rajvir was going to Assam for the first time. (D) Pranjol was not interested in the greenery outside. Ans. Option (B) is correct. Explanation: In the story, both the friends Rajvir and Pranjol have been shown to have immense interest in detective stories. Q. 4. The author has used a colloquial word in the given extract. Which of the following is it? (A) high-pitched (B) steaming (C) when (D) compartment Ans. Option (C) is correct. Q. 5. Choose the word that correctly states the meaning of ‘drank slowly’. (A) pitched (B) sipped (C) steaming (D) compartment Ans. Option (B) is correct. Explanation: Pitched (here) means shrill, Sipped means drinking slowly by taking small sips, Steaming means producing lot of steam or something which is hot and Compartment means a bogie of the train. 







Q. 1. Bodhidharma was a ______________ ascetic.







“We have an Indian legend too. Bodhidharma, an ancient Buddhist ascetic, cut off his eyelids because he felt sleepy during meditations. Ten tea plants grew out of the eyelids. The leaves of these plants when put in hot water and drunk banished sleep. “Tea was first drunk in China,” Rajvir added, “as far back as 2700 B.C.! In fact words such as tea, ‘chai’ and ‘chini’ are from Chinese. Tea came to Europe only in the sixteenth century and was drunk more as medicine than as beverage.” The train clattered into Mariani junction. The boys collected their luggage and pushed their way to the crowded [CBSE-QB, 2021] platform.



II. Read the following extract and answer the questions/ complete the sentences that follow: (5×1=5)

III. Read the following extract and answer the questions/ complete the sentences that follow: (5×1=5)





Ans. Option (D) is correct. Explanation: Enthusiastic is when one wants to tell maximum about something in the minimum possible time. Passionate is when someone loves to talk about something.



Oswaal CBSE Question Bank Chapterwise & Topicwise, ENGLISH LANG. & LIT., Class-X







154

@PROCBSE

GLIMPSES OF INDIA

155

SUBJECTIVE TYPE QUESTIONS Short Answer Type Questions

Q. 1. Pranjol and Rajvir discuss their next vacation destination. They shortlist Coorg and Goa. Rajvir is keen on Coorg and tries to convince Pranjol. Develop a conversation between the two, based on your understanding of Glimpses of India. You may begin like this:

Pranjol:



 



: :









Pranjo Rajvir



:





: :

Pranjol



: :

Rajvir











Pranjol







Pranjol Rajvir





:







Ans. Rajvir

[CBSE-QB, 2021]

Hey Pranjol! I think we should be visiting Coorg. It is a beautiful place with coffee plantations. I can smell the aroma already! We visited a tea plantation last year, in Assam; I want to… There is a lot more to do in Coorg than smelling the coffee! The place has rainforests, so the mega fauna will be worth watching. Not just this, Coorg provides opportunities to indulge in adventure sports like river rafting, rappelling, mountain biking, to name a few. That sounds interesting, but I would prefer some serene moments too, away from this postpandemic hustle-bustle. Oh! The answer is Coorg again! Oh, come on! You can’t be serious… Believe me, I am. Coorg is the place. It has beautiful natural walking trails and the Brahmagiri hills offer a panoramic view. I read that the place has the largest Tibetan settlement, so the environment will reflect peace and spirituality, I’m sure. Have to say, you’ve presented a fine case in favour of Coorg and convinced me. Let’s plan to leave for Coorg next Wednesday!

• The dialogues should be based on understanding of the chapter. There should be arguments from Pranjol showing he requires more to get convinced. Rajvir, being an explorer must be able to give an interesting account about Coorg to convince Pranjol. The persuasive element using examples from text descriptions need to be showcased. Apart from the above the following could be added:



• Coorg is culturally rich, the people there are brave and their hospitality is great.



• Some people in Coorg are the only ones allowed to keep firearms without license. It will be fun meeting such brave hearts.



• Rainforests can be visited for flora and fauna.



















These questions are for practice and their solutions are available at the end of the chapter

























• There flows the river Kaveri and sitting by the river could be peaceful. (CBSE Marking Scheme, 2021) Q. 2. Pranjol buried his head in his detective book Q. 4. Why did Mr. Barua feel surprised? while Rajvir was eager to look at the beautiful Q. 5. Rajvir did his study before his visit to the tea scenery during the train journey. Why was there a plantation. Is it good to do one’s research before difference in their attitude? [CBSE-QB, 2021] the start of a new venture or does it take away from Ans. Pranjol belonged to Assam. He was use to the the thrill of discovery? Elucidate your stance. scenic beauty of Assam since his childhood where as Rajvir was amazed to see huge fields full of [CBSE-QB, 2021] green plantations. He was enjoying such natural Ans. Indeed, Rajvir possessed qualities like time beauty for the first time as he lived in Delhi. management positivity and humility. His Q. 3. Which language are the words ‘Chai’ and ‘Chini’ research before the start to Assam helped him to derived from? When did tea come to Europe? explore more about the place. Knowing about the Ans. The words tea, chai and chini were derived destination and being families with the real place from Chinese language. Tea came to Europe in is of utmost importance. Rajvir also surprised the sixteenth century and was drunk more as a Pranjol’s father with his in-depth knowledge of the medicine than as a beverage. subject.



We visited a tea plantation last year, in Assam; I want to…





Hey Pranjol! I think we should be visiting Coorg. It is a beautiful place with coffee plantations. I can smell the aroma already! 

Rajvir:









(40-50 words & 3 marks each)

@PROCBSE





Oswaal CBSE Question Bank Chapterwise & Topicwise, ENGLISH LANG. & LIT., Class-X 



Q. 6. What excited Rajvir? Why did Pranjol not share his excitement? [CBSE Delhi Set-1, 2, 3, 2018] Ans. Objective: To test local and global comprehension, themes and ideas in the text. Mark Content - 1 Marks Expression - 2 Marks (any four) OR





156

(CBSE Delhi Set 1, 2016)



Q. 7. What legends are associated with the origin of tea?





Topper Answer, 2018





Ans. Chinese legend is that once a few leaves of the twigs burning under the pot fell into the water and gave a delicious flavour. It is said that they were tea leaves. According to the Indian legend, Bodhidharma cut off his eyelids because he felt sleepy during meditation. Ten tea plants which grew out of the eyelids, when boiled in (Board Marking Scheme, 2016) water and drunk banished his sleep.

Long Answer Type Questions







One Chinese legend says that it was accidentally discovered by a Chinese Emperor. Another legend, who is Indian, says that it was discovered by Bodhidharma. The legend says that he cut off his eyelids as he felt himself sleepy during meditation and tea plants grew out of them to banish sleep when they were drunk. He also knew that tea was first drunk in China in 2700 B.C. and it reached Europe in the sixteenth century as a medicine. He also informed Pranjol that the words ‘tea’, ‘chai’ and ‘chini’ are Chinese. He even had knowledge about the sprouting season on the second-flush which is the best in the world. Q. 3. What scenery did Rajvir notice while sitting in the train and in the tea estate? Q. 4. What details do you gather about tea from the lesson, ‘Tea from Assam’? Ans. Tea was first drunk in China. It dates back as far as 2700 BC. In the 16th century, tea came to Europe and was drunk as medicine. There are unlimited tea plantations in Assam. Tea is very popular in the entire world, over eighty crore cups of tea are drank every day throughout the world. The yielding period of tea is from May to July. There were many legends about it. One Chinese legend says that it was accidentally discovered by a Chinese emperor, while boiling water. Tea leaves fell in the teapot and water tasted better. Another legend is of Indian Buddhist ascetic, who cut off his eyelids because he felt sleepy while meditating. Ten tea plants grew out of eyelids. These leaves banished sleep.

These questions are for practice and their solutions are available at the end of the chapter





















Q. 1. “You seem to have done your homework before coming.” Answer the given question in the light of this statement. Rajvir seemed to have a lot of information about tea. What all did he tell? What character trait of his is revealed? Is it essential for children? Ans. Rajvir had been invited to spend his vacations with his classmate Pranjol whose father was the manager of a tea estate in Assam. He was quite excited about the trip and had gathered considerable amount of relevant information i.e., where tea originated and how it became such a popular beverage. He informed that about 80 crore cups of tea are consumed daily and that the name ‘tea’ itself has been originated from Chinese language. He narrated two popular legends about the origin of tea. Rajvir exhibits an important value trait, that is the pre-requisite to gain knowledge–curiosity and eagerness to know. Moreover, he is not a passive learner but is proactive in collecting prior information to any real experience. He likes to do his homework thoroughly before going to a place. This is something very appreciable and children should develop this habit of finding out about the place they are going to visit. These are extremely desirable traits for all learners. Q. 2. What information did Rajvir share with Pranjol and his father? Ans. Rajvir had a lot of information about tea. He told Pranjol that though there are many legends about it, no one actually knows about the origin of it.













(100-120 words & 6 marks each)

@PROCBSE

GLIMPSES OF INDIA

Answering Tip

A few students got mixed up with the two legends associated with the origin of tea.







Commonly Made Error

157

It is mandatory to prepare the chapter in writing during revisions.

Solutions for Practice Questions (Topic-1) Short Answer Type Questions

Ans.4. Mr. Barua was surprised that Rajvir knew so much about tea plantations, despite the fact that it was his first visit to Assam. He appreciated the fact that Rajvir had collected a considerable amount of information before coming.

Long Answer Type Questions

Ans.3. Rajvir could see green and magnificent view while sitting in the train. He could also see densely

wooded hills accompanied by a sea of tea bushes. He could also view doll like figures with bamboo baskets moving everywhere. He even saw an ugly building with tall chimneys, billowing out smoke from a distance. When they entered Dhekiabari tea estate, Rajvir saw, on both the sides of the gravelroad, acres upon acres of tea bushes, all neatly pruned to the same height. Group of tea-pluckers were plucking the newly sprouted leaves.







REFLECTIONS What is the story/ stories behind the origin of tea? Were you able to peep into the graphic description of the sea of tea bushes stretching as far as eyes can go in Assam?

@PROCBSE

CHAPTER

6

MIJBIL, THE OTTER – By Gavin Maxwell

Revision Notes Scan to know more about this topic



Introduction

left for a quick meal. When he retuned, he found that the box was still but Mij had created a mess by destroying the inner lining. As a result, blood was dripping out of the holes. Scared as he was, he hurried. Mijbil, the otter They were far away from the airport and there were only ten minutes left for the flight to take off. He cleaned it all, hurried in a cab and managed to reach just in time. He explained the series of events to a very kind and generous air hostess who advised him to keep the box in his lap. Gaven developed deep admiration for the air hostess as she was very kind to him. As soon as he opened the box, the otter leapt out and disappeared there by creating a chaos. Passengers were frightened. A lady climbed up a chair and in an attempt, to get hold of Mij, the author got himself covered in curry. The air hostess offered help and brought him back to Gaven and finally, they reached London. Mij was fond of playing with ping-pong balls and marbles. He even developed a game with the author’s damaged suitcase. It could keep him engrossed for a long period of time. Narrator took him for walks while taking the lead and played with him. People of London, being unfamiliar with otters, had wild guesses about what Mij was. Some thought it to be a baby seal, squirrel or even a hippo. The most shocking reaction came when a labourer digging the hole asked the author, “What is that supposed to be?”

Gavin Maxwell lives in a cottage in Camusfearna, in the west Highlands in Scotland. He is fond of keeping pets. After the death of his pet dog, Jennie, he was too sad to think of a dog again. The author is in Basra city of Iraq. On a visit there, he expresses his wish to domesticate an otter. He takes this otter to England where he has flat of his own. He had some interesting experiences during his air journey from Basra to Paris and London.

Key Words

1. Squished: to squeeze 2. Domesticate: to adapt to live with humans

The story begins with the author travelling to Basra along with his friend. During their journey, the author expresses his desire to domesticate an otter because after he had lost his pet dog, life had become lonely for him. His friend suggested that he should get one from the Marshes along the river Tigris in Iraq. When they reached the destination, they found that only the friend's mail had arrived. After a few days, the friend left while the narrator was still waiting to receive his mail. Upon receiving it, he went to his room only to find an otter (brought to him in a sack) accompanied by two Arabs with a note. It was a gift from his friend. The narrator named the otter, Mijbil or shortly, Mij. It took some time for Mij to open up and get acquainted with his surroundings. He was covered in mud to an extent that it took almost a month of cleaning and washing to reveal his actual colour. Mij loved playing with water so much so that he even learnt to open the tap on his own. He believed that each drop of water should be squished and splashed till the bowl had been emptied. Everything was going on smoothly in Basra, but now it was time to fly back to London. British Airlines did not allow animals, so he had to book another flight that allowed Mij with a condition that he had to be carried in a box. The narrator put him in a box an hour before the flight so that Mij could get accustomed to it and then





Summary

@PROCBSE

MIJBIL, THE OTTER

Extract Based Questions



(5 marks each)

(B) Body-belts basically resemble the waist belts and keep our body upright, in a standing position. (C) Body-belts are used for restraining while doing certain activities, in order to prevent injury. (D) A body-belt is an accessory used by body builders for weight training as a measure against muscle and bone injuries. Ans. Option (C) is correct. Q. 5. Select the option that best describes the final action of the otter, based on the given passage.



      





     



































































































(1) (2) (3) (4) (A) Option (1) (C) Option (2) (C) Option (3) (D) Option (4) Ans. Option (B) is correct. II. Read the following extract and answer the questions/ complete the sentences that follow: (5×1=5) When I returned, there was an appalling spectacle. There was complete silence from the box, but from its air holes and chinks around the lid, blood had trickled and dried. I whipped off the lock and tore open the lid and Mij, exhausted and bloodspattered, whimpered and caught at my leg. He had torn the lining of the box to shreds; when I removed the last of it so that there were no cutting edges left, it was just ten minutes until the time of the flight and the airport was five miles distant. I put the miserable Mij back into the box, holding down the lid with my hand. I sat in the back of the car with the box beside me as the driver tore through the streets of Basra like a ricocheting bullet. [CBSE-QB, 2021] (5 × 1 = 5) Q. 1. Choose the speaker that uses the phrase 'appalling spectacle' correctly. Speaker 1: We got back home to see that our pet dog had eaten kheer my mother had kept out of the fridge. Indeed, an appalling spectacle. Speaking 2: We got back home and witnessed that our cousin had dressed our pet dog in his t-shirt. Have to admit that it was an appalling spectacle. Speaker 3: We got back home to see that our pet dog is sitting with a mangled rat. It was an appalling spectacle. Speaking 4: We got back home from the airport and observed our pet dog sound asleep on the rug. What an appalling spectacle! (A) Speaker 1 (B) Speaker 2 (C) Speaker 3 (D) Speaker 4 Ans. Option (C) is correct.     

























I. Read the following extract and answer the questions/ complete the sentences that follow: (5×1=5) For the first twenty-four hours Mijbil was neither hostile nor friendly; he was simply aloof and indifferent, choosing to sleep on the floor as far from my bed as possible. The second night Mijbil came on to my bed in the small hours and remained asleep in the crook of my knees until the servant brought tea in the morning and during the day he began to lose his apathy and take a keen, much too keen, interest in his surroundings. I made a body-belt for him and took him on a lead to the bathroom, where for half an hour he went wild with joy in the water, plunging and rolling in it, shooting up and down the length of the bathtub underwater and making enough slosh and splash for a hippo. [CBSE-QB, 2021] (5 × 1 = 5) Q. 1. "Mij was neither hostile nor friendly; he was simply aloof and indifferent... " From the actions of the given set of friends, choose the person whose behaviour is likely to resemble Mijbil's. (A) Akanksha rudely tells her best friend to stop calling. (B) Piyush was amicable since the day he walked into his new classroom. (C) Rishabh became nasty as soon as he started earning money. (D) Urvashi was distant for a while after losing her grandmother. Ans. Option (D) is correct. Q. 2. Select the option that correctly puts the following events in correct order. (i) Maxwell planned to take Mijbil to the bathroom. (ii) Mijbil slept at a significant distance from Maxwell's bed. (iii) Mijbil immensely enjoyed playing in water. (iv) Mijbil slept in much closer vicinity of the author as opposed to the previous night. (A) i, iii, iv, ii (B) iii, i, ii, ii (C) ii, iv, i, iii (D) iv, iii, i, ii Ans. Option (C) is correct. Q. 3. The words "slosh and splash" are examples of the onomatopoeic words that go together phonetically-resembling the sound that they describe. Choose the option that DOES NOT fit the given description. (A) hip -hop (B) bow -wow (C) tick- tock (D) pitter -patter Ans. Option (B) is correct. Q. 4. Choose the option that correctly describes a bodybelt in the given context as explained by the four people given below. (A) A body-belt is a device especially made for swimmers and scuba divers, which helps them breathe underwater.

159

@PROCBSE Q. 2. Where could the author get 'one'? (A) Tigris Marshes (B) Basra (C) Europe (D) Arab Country Ans. Option (A) is correct. Q. 3. What did the author find at the Constable General? (A) His mail hadn't arrived. (B) His pet had arrived. (C) His friend's mail had arrived. (D) Both (A) and (C) are correct. Ans. Option (D) is correct. Q. 4. Who is the author of the above passage? (A) Lokesh Abrol (B) Lucio Rodrigues (C) Gavin Maxwell (D) Mijbil Maxwell Ans. Option (C) is correct. Q. 5. Find the extra word from the extract which means 'domesticated'. (A) Casually (B) Tamed (C) Arrived (D) Mentioned Ans. Option (B) is correct. IV. Read the following extract and answer the questions/ complete the sentences that follow: (5×1=5) Early in the New Year of 1956 I travelled to Southern Iraq. By then it had crossed my mind that I should like to keep an otter instead of a dog and that Camusfearna, ringed by water a stone's throw from its door, would be an eminently suitable spot for this experiment. [CBSE-QB, 2021] (5 × 1 = 5) Q. 1. What 'experiment' did Maxwell think Camusfearna would be suitable for? (A) To keep an otter as a pet instead of a dog. (B) To keep a dog as a pet instead of an otter. (C) To keep some marine animal as a pet. (D) To stay without a pet. Ans. Option (A) is correct. Q. 2. Why was it a suitable spot for his experiment? (A) Because it was marshy. (B) Because it was ringed by water. (C) Because he was a stranger there. (D) Because he loved that spot. Ans. Option (B) is correct. Q. 3. Where did the author travel to? (A) New York (B) Southern Iraq (C) Camusfearna (D) Tigris Marshes Ans. Option (B) is correct. Q. 4. When did the author travel? (A) In 1956 (B) In New York of 1956 (C) When he was young (D) None of the above Ans. Option (B) is correct. Q. 5. What is the meaning of the phrase 'crossed my mind'? (A) The narrator got angry.























Q. 2. "whipped off', is an example of a phrasal verb, i.e., a combination of a verb and a preposition. Choose the option that DOES NOT fit the given definition.



Oswaal CBSE Question Bank Chapterwise & Topicwise, ENGLISH LANG. & LIT., Class-X



160



      







































































































































































      



     















    

(1) (2) (3) (4) (A) Option (1) (B) Option (2) (C) Option (3) (D) Option (4) Ans. Option (D) is correct. Q. 4. Select the option that DOES NOT state the possible reason for Maxwell's cutting the box's edges off. (A) Mij could have injured himself further. (B) It could have made the place untidy. (C) Mij could have accidentally swallowed the torn pieces. (D) It could have obstructed the shutting of the lid. Ans. Option (B) is correct. Q. 5. Choose the option that correctly states Maxwell's likely feelings, in the situation below. "... put the miserable Mij back into the box, holding down the lid with my hand." (1) anxious (2) disappointed (3) terrified (4) guilty (5) insulted (A) 1 and 5 (B) 2 and 3 (C) 1 and 4 (D) 3 and 5 Ans. Option (C) is correct. III. Read the following extract and answer the questions/ complete the sentences that follow: (5×1=5) When I casually mentioned this to a friend, he casually replied that I had better get one in the Tigris marshes, for there they were as common as mosquitoes and were often, tamed by the Arabs. We were going to Basra to the Consulate-General to collect and answer our mail from Europe. At the Consulate-General we found that my friend's mail had arrived but not that of mine. [CBSE-QB, 2021 (Modified)] (5 × 1 = 5) Q. 1. What was 'they'? (A) Mosquitos (B) Moles (C) Otters (D) Dogs Ans. Option (C) is correct.

























     















    

(1) (2) (3) (4) (A) Option 1 (B) Option 2 (C) Option 3 (D) Option 4 Ans. Option (B) is correct. Q. 3. Choose the option that is closest in resemblance to the box's condition in which Mij was kept.

@PROCBSE





























































































(B) The Otter (C) Gavin Maxwell (D) Both (A) and (B) are correct. Ans. Option (C) is correct. Q. 3. What does Mijbil resemble? (A) A dog (B) A mole (C) A small dragon (D) A lizard Ans. Option (C) is correct. Q. 4. How would Mij follow the author? (A) Like a pet (B) With a lead (C) Without a lead (D) None of the above Ans. Option (C) is correct. Q. 5. Find the word from the passage that means the same as 'trickle'. (A) Shuffling (B) Dribble (C) Flick (D) Lead Ans. Option (B) is correct.



(B) The narrator crossed the road. (C) The narrator cross the country. (D) A thought came into the narrator's (my) mind. Ans. Option (D) is correct. V. Read the following extract and answer the questions/ complete the sentences that follow: (5×1=5) Very soon Mij would follow me without a lead and come to me when I called his name. He spent most of his time in play. He spent hours shuffling a rubber ball round the room like a four-footed soccer player using all four feet to dribble the ball and he could also throw it, with a powerful flick of the neck, to a surprising height and distance. [CBSE-QB, 2021] (5 × 1 = 5) Q. 1. Who is Mij? (A) The author's pet (B) Otter (C) Mijbil (D) All of the above Ans. Option (D) is correct. Q. 2. Who does 'I' stand for? (A) Mijbil 



161

MIJBIL, THE OTTER

SUBJECTIVE TYPE QUESTIONS Short Answer Type Questions































These questions are for practice and their solutions are available at the end of the chapter





















Detailed Answer: Mijbil enjoyed playing games. He would keep the broken suitcase in a sloping position and place the ball on the high end. Then he would run to the other end to catch it.











Ans. Would keep the broken suitcase in a sloping position, place the ball on the high end - would run to the other end to catch it. [CBSE Marking Scheme, 2016]

Q. 5. What would you say was the greatest obstacle faced by the narrator in transporting Mijbil to Camusfearna? [CBSE-QB, 2021] Ans. It was a difficult job for Maxwell to be transported from Basra to Camusfearna with Mijbil. The British airlines wouldn’t fly animals. He booked a flight to Paris on another airline which allowed animals to fly on conditions. Mijbil was put into a small box on hour before so that he would become accustomed to it. Q. 6. Peculiar behaviour's are a characteristic of Otter's. Discuss with respect to "Mijbil, the Otter". [CBSE-QB, 2021] Q. 7. Mijbil caused a sensation in London. Expound with reference to the chapter "Mijbil the Otter". [CBSE-QB, 2021] Ans. The average Londoners were surprised to see such a unique animal. They made wild guesses about the otter. Some called him 'a baby seal', 'a squirrel', 'a walrus'. Others called him 'a beaver', 'a bear cub' and 'a leopard'. The best remark came from a labourer. On seeing Mijbil, he asked, "What is that supposed to be?" Q. 8. What special characteristic of Mijbil did Maxwell learn after he took him to the bathroom? [Board Term-II, 2015, Set-2] Ans. After taking Mijbil to the bathroom, Maxwell witnessed Mijbil being almost mad with excitement, plunging and rolling in water, splashing water all around. It shows his love for water as marshes and wetlands are otter's natural habits at.

Q. 1. "...every drop of water must be, so to speak, extended and spread about the place...". Describe Maxwell's observation regarding Mij with reference to the given quote. [CBSE-QB, 2021] Ans. Mij loved to play in the water. Only once did Maxwell lead him to the bathroom. Next time he went to the bathroom on his own. He went wild in water, plunged and rolled in the tub. He shot up and down the bathtub and made enough slosh and splash in it. Q. 2. Maxwell's fear of sending Mij to England was [CBSE-QB, 2021] legitimate. Give a reason. Q. 3. Maxwell was justified in thinking very highly of the air hostess. Justify. [CBSE-QB, 2021] Ans. The air hostess was very friendly and cooperative. Maxwell took her into his confidence and gave her a parcel of fish for Mijbil. He admired her and called her "the very queen of her kind". The air hostess suggested that he should keep his pet on his knees during the flight. Q. 4. What game did Mij invent? [CBSE OD Set-I, 2016]





(40-50 words & 3 marks each)

@PROCBSE 162

Oswaal CBSE Question Bank Chapterwise & Topicwise, ENGLISH LANG. & LIT., Class-X

intelligence in the otters in general. They have basic congeniality. It is portrayed in his behaviour that it took him mere a day to shed his inhibitions and familiarise himself with the surroundings. Mijbil's intelligence is reflected in his turning on the tap to get a flow of water.













Q. 9. Give an example from the text to show that Mijbil is an intelligent animal. [CBSE OD Set- 3, 2014] Ans. Mijbil's behaviour and reactions to the circumstances throw light on the trait of

Long Answer Type Questions



(100-120 words & 6 marks each)



Q. 2. The People's Collective for Endangered Species has invited the narrator of "Mijbil, the Otter" as a guest speaker to address the audience about the importance of protecting Otters and their natural habitats like lakes and ponds. As the narrator, pen down a speech detailing your own experience with Mijbil and why it is important to conserve water bodies for animals like Otters. You may use the prompt given below:









Q. 1. Imagine yourself to be the narrator of "Mijbil, the Otter". A friend of yours is confused about whether to adopt an otter as a pet or not and has asked for your advice. Based on the experiences described in the chapter, write a letter to your friend giving your opinion on the matter. You may use the prompt given below: XYZ, Camusfearna







Sandaig Bay, UK

My Experiences with Mijbil

Theodore Roosevelt said, "Wildlife cannot speak, so we must and we will." Dear friends, I have had a pet otter named Mijbil as my roommate for several years now. What I want to share with you today, is how all forms of wildlife are integral for ............ [CBSE-QB, 2021] Ans. …………………… What I want to share with you today, is how all forms of wildlife are integral for maintaining balance in the eco system. Wild life includes animals, birds, insects as well as the aquatic life forms. Otters also have a pivotal role in the river eco system. They are predators, meaning they help control the populations of food species prey upon. This affects the eco system as a whole and as a result, their presence is a signal that the ecosystem is healthy. The wetlands are amongst the most biodiverse and economically valuable habitats and yet they continue to decline. This decline has been as much as 87% world wide. Otters are top predators using both terrestrial and aquatic environments and their loss has a serious impact on local food webs, biodiversity and habitat relationship. Otters need good water quality and also unpolluted, natural land habitat in which to live. This is essential for all species, including our own and so they are excellent environmental indicators. The link between the otter and wetlands is clear and it is our duty to work to protect these important ecosystem to the benefit of otters, other wetland species and ultimately ourselves. Q. 3. If Mijbil was living in the world of the poem, The Tale of Custard, the Dragon, how do you think he would have (a) behaved with Custard? (b) reacted to the appearance of the pirate? Rationalise your answer. [CBSE-QB, 2021] Ans. (a) Mijbil resembled like a very small imaginary dragon of the Middle Ages. Its body was coated





Waiting for a positive response from your end. Yours affectionately, Gavin









If you want you can accompany me to Basra. I am going there the next week. You too, can get an otter from there. Believe me it will be great fun.



And adopting an otter is one of the best decisions of your life. I advise you to bring your thought into action at the earliest. No doubt, otters prove to be the best companions. They demand nothing in turn of their unconditional love and company they provide you with. I have been living with Mij for several years now. What I want to share with you today, is that otters are very intelligent and fun-loving animals. The main characteristic of an otter is to extent and spread every drop of water. Mij loves to juggle with small objects with his paws lying on his back. His favourite toy is marbles.











Trust you too are doing well. I am glad to know that you are interested in adopting a pet. Believe me, your life is going to change absolutely after a pet enters your life.





Dear Kevin,





4 December, 1961





Sandaig Bay, UK





Ans. XYZ, Camusfearna



[CBSE-QB, 2021]





Kevin





Yours affectionately







Trust you and Mij are doing well. I am writing for a special reason that will surely interest. This is because it concerns the love of your life- otters! I have been....





Dear Gavin,









4 December '61

@PROCBSE

MIJBIL, THE OTTER







the second day and remained asleep on it. (b) Otters are friendly looking creatures. However, they can bite and they can fight with their tails if provoked. Territorial otters stay away from strangers. Thus, on seeing the pirate, Mijbil too would have joined Custard in gobbling every bit of him. Q. 4. What things does Mij do, which tell you that he is an intelligent, friendly and fun loving animal who needs love? [Board Term-II, 2015, Set-2]

with pointed scales. Between them a soft velvet fur was visible. It was like a chocolate brown mole. On the contrary, Custard was a huge ferocious looking dragon. Mijbil would have behaved as friendly as he was with the narrator. Initially, Mijbil was neither hostile nor friendly with the narrator. He was aloof and indifferent. He slept on the floor far away from Maxwell’s bed. However, a change came in Mijbil afterwards. He started taking keen interest in his surroundings. He went to Maxwell’s bed on

163





Commonly Made Error Instead of sharing their own experiences regarding their pets, the candidates were supposed to discuss Maxwell and his experiences with Mijbil.





Answering Tip Long Answer Type Questions need to be answered in correlation to the concerned chapter.

Solutions for Practice Questions Long Answer Type Questions





Ans. 2. When Maxwell opened the box, in which he carried Mijbil in the aircraft, it ran out and disappeared at high speed. He caused a great hue and cry in the aircraft. There were sounds of shrieks across the flight. Ans. 6. Mijbil, the otter, had some 'compulsive habits'. On this way home everyday, Mij used to tug Maxwell to the low wall of a primary school opposite to his flat. Mij would jump to it, gallop the full length of its thirty yards and cause a hopeless distraction to both pupils and staff within the primary school.

Ans. 4. In the bathroom, Mijbil fumbled with the taps and went mad with joy on seeing full flow of water. He played, splashed and thoroughly enjoyed like a hippo in water. Aloof at first, he started answering to his name, played for hours with rubber ball and juggled with small objects, after two days. Mijbil's behaviour and reactions to the circumstances throw light on the trait of intelligence in the otters in general. They have basic congeniality. And it is portrayed in his behaviour that it took him merely a day to shed his inhibitions and familiarize himself with the new surroundings. Mijbil's intelligence and sense of logic is reflected in his turning on the tap to get a flow of water.

Short Answer Type Questions





REFLECTION Have you tamed a pet ever? Mention some of your memorables experiences with the pet.

@PROCBSE

CHAPTER

7

MADAM RIDES THE BUS

– By Vallikkannen

Revision Notes Scan to know more about this topic

Introduction

When they reached the town, Valli gaped at everything in amazement—the big shops and so on. The conductor asked her to get down and have a look at the sights but Valli refused. She said that she Madam rides had come only for the bus ride. the bus The conductor offered to buy a cold drink for her but she refused. On the journey back, Valli saw a young cow dead by the roadside. The conductor told Valli that it was the same cow that they had seen earlier. Valli felt sad. The memory of the cow haunted her. When they reached the village, the conductor told her to come again, but to remember to bring the fare. When Valli reached home, she found an aunty with her mother. Her mother asked her where she had been. Valli did not replied but only smiled secretively.



This is a sensitive story of an eight-year old girl, Valli’s first bus journey into the world outside her village. At a very tender age, she tries to understand the mystery of life and death in her own terms.

Key Words



1. Longed: Have a strong desire or wish 2. Haunted: Tormented 3. Secretively: Hiding feelings, thoughts or action from others









‘Madam Rides The Bus’ is the story of a young girl who travels for the first time outside her village by a bus. Valliammai, Valli for short, was an eight-year-old girl who wanted to know a number of things. She had no one to play with. A bus ran between her village and the nearest town and Valli longed to travel by that bus. Listening to people who had travelled by the bus, she gathered a little details about the trip. She decided to gather enough money to travel by it. When she had the required money, she went on the bus. The conductor was surprised to see a young girl, travelling by herself but jokingly called her ‘Madam’ and offered her the front seat. Valli observed everything very carefully e.g., the trees that seemed to be running towards them, a cow that suddenly moved in front of the bus and so on.



Summary

Extract Based Questions

I. Read the following extract and answer the questions/ complete the sentences that follow: (5×1=5) Suddenly she was startled by a voice. “Listen, child,” said the voice, “You shouldn’t stand like that. Sit down.” Sitting down, she looked to see who had spoken. It was an elderly man who had honestly been concerned















(5 marks each) for her, but she was annoyed by his attention. “There’s nobody here who’s a child,” she said haughtily. “I’ve paid my thirty paisa like everyone else.” The conductor chimed in. “Oh, sir, but this is a very grown-up madam. Do you think a mere girl could pay her own fare and travel to the city all alone?”

@PROCBSE MADAM RIDES THE BUS





























Ans. Option (B) is correct. Explanation: Indulgence here has been used in a sarcastic sense as to mean that everyone was tolerating Valli for her rude attitude who was considering everyone below her level. II. Read the following extract and answer the questions/ complete the sentences that follow: (5×1=5) “Day after day she watched the bus and gradually a tiny wish crept into her head and grew there: she wanted to ride on that bus, even if it was just once. This wish became stronger, until it was an overwhelming desire. Valli would stare wistfully at the people who got on and off the bus when it stood at the street corner. Their faces would kindle in her longings, dreams and hopes. If one of her friends happened to ride the bus and tried to describe the sights of the town to her, Valli would be jealous to listen and would shout, in English: “Proud! Proud!” [CBSE-QB, 2021] Q. 1. Valli would stare wistfully at the people as she: (A) had been watching the bus for a long time. (B) was inspired by the people travelling on the bus. (C) had a strong desire to take a ride on the bus. (D) envied the people who could travel on the bus. 



















































(A) F - 1,2 and O - 3,4 (B) F - 2,3,4 and O - 1 (C) F - 2,4 and O - 1,3 (D) F - 3 and O - 1,2,4 Ans. Option (D) is correct. Explanation: Fact is something that is true and actually happened – Valli paid for her ticket. Opinion is something which is a discussion and views of one person may differ from one person to another – Some will think that conductor should have allowed Valli to board, while some will think he shouldn’t have. Some will think that Valli was responsible, while others will think that she was irresponsible. Some will think that Valli had gone











Ans. Option (C) is correct. Q. 2. Pick the option that shows the list of words that collocate with ‘overwhelming’. (e.g., overwhelming desire) (i) response (ii) lies (iii) support (iv) majority (v) pets (vi) places (A) i, v & vi (B) i, iii & iv (C) ii & v (D) iii, v & vi Ans. Option (B) is correct. Explanation: Overwhelming response is said to be when an unexpected response in favour is received. Overwhelming support is said to be when unexpectedly high level of support is extended. Overwhelming majority is said to be when one person or party gets votes in favour and the difference between the winner and the others is too high.

























Q. 5. Choose the characteristic displayed by the conductor when he addressed Valli and declared that she couldn’t be ‘a mere girl’. (A) Encouragement (B) Indulgence (C) Embarrassment (D) Authority



























Q. 3. Pick the option that correctly classifies fact/s (F) and opinion/s (O) of the students below. [CBSE-SQP, 2021]



Q. 4. Which word does ‘chimed in’ correspond to? (A) Intervened (B) Interrupted (C) Intersected (D) Interjected Ans. Option (C) is correct. Explanation: ‘Chimed in’ refers to a situation when two different events meet at one point which is same as ‘Intersect’.





















Q. 2. Choose the option that lists the set of statements that are NOT TRUE according to the given extract. (i) Valli was travelling without a guardian. (ii) The conductor was bullying Valli. (iii) Valli was nervous in her interactions. (iv) Valli was a child passenger. (v) Valli followed the elderly man’s advice. (vi) Valli’s response to the elderly man was respectful. (vi) It took Valli some time to participate in the jocular conversations. (a) ii, iii, iv (b) v, vi, vii (c) iii, v, vii (d) ii, iii, vi Ans. Option (D) is correct. Explanation: The conductor was not bullying Valli. Valli was outspoken and bold in her interactions with the fellow passengers. Valli was annoyed by the concern shown by the elderly man and shouted at him showing a disrespectful attitude towards him.

165

out just once for adventure, while others may not think so about her.

Valli shot an angry glance at the conductor and said, “I am not a madam. Please remember that. And you’ve not yet given me my ticket.” “I’ll remember,” the conductor said, mimicking her tone. Everyone laughed and gradually Valli too joined in the laughter. [CBSE SQP, 2020-21] Q. 1. How did Valli feel when the elderly man expressed concern that she was standing? (A) Rather shocked (B) Somewhat happy (C) Fairly displeased (D) Quite embarrassed Ans. Option (C) is correct. Explanation: Valli was annoyed or displeased by the concern expressed by the elderly man because he addressed her as ‘a child’. 







@PROCBSE Q. 1. Pick the option that matches the expressions with ‘mind’ correctly.

Q. 3. ‘Valli would be jealous to listen...’ This indicates Valli’s: (A) longing to ride the bus. (B) nature as an envious person. (C) denial to accept her situation. (D) inclination to trouble travellers. Ans. Option (A) is correct. Explanation: Valli was jealous that her friends took the bus ride but she could not. She also wanted to take the bus ride like her friends. In short, she longed to have the bus ride.



Oswaal CBSE Question Bank Chapterwise & Topicwise, ENGLISH LANG. & LIT., Class-X

Expression

Meaning

i

bring to mind

a.

be receptive to someone's ideas

ii.

be in two minds

b.

think about only one subject

iii.

have an open mind

c.

recall something

iv.

put your mind at ease

d.

undecided

e.

reassure you

f.

reach a decision





(D) (i) vigorous (ii) overpowering (iii) longingly Ans. Option (D) is correct.















Q. 4. The old woman asked her questions: (A) due to anxiety about Valli travelling alone. (B) out of curiosity and concern for Valli. (C) due to interest in Valli’s life and need for travel. (D) due to the habit of interfering in Valli’s life. Ans. Option (B) is correct.









‘You needn’t bother about me. I can take care of myself,’ Valli said, turning her face towards the window and staring out. [CBSE-QB, 2021]

Q. 3. The old woman went on with her drivel. Why is the word ‘drivel’ used here? It is used to show that the old woman: (A) was talking incessantly. (B) kept muttering a lot of nonsense. (C) was volunteering advice on request. (D) spoke non-stop about herself. Ans. Option (B) is correct. Explanation: Drivel means nonsense. Here, the old lady was continuously muttering something to herself which no one else could make any meaning of.





‘Oh, why don’t you mind your own business', said Valli. But she laughed all the same and the conductor laughed too. But the old woman went on with her drivel. ‘Is it proper for such a young person to travel alone? Do you know exactly where you’re going in town? What’s the street? What’s the house number?'













III. Read the following extract and answer the questions/ complete the sentences that follow: (5×1=5) ‘”Yes, I’m travelling alone,” she answered curtly. And I’ve got a ticket too.’ ‘Yes she’s on her way to town,’ said the conductor. ‘With a thirty paisa ticket.’





Ans. Option (A) is correct.



















(A) The wish developed in her head over a period of time. (B) The wish was a feeling of wanting to prove her strength. (C) The wish was planted in her head by someone’s suggestion. (D) The wish was small and was overpowered by a sense of doubt.

Q. 2. Valli told the conductor to mind his own business and laughed as well. This shows that Valli ... (A) was not really bothered about what the conductor said. (B) didn’t like the conductor speaking about her. (C) disliked what the conductor said but was amused. (D) was impressed with what the conductor said. Ans. Option (C) is correct. Explanation: Commenting to mind own business indicates that she did not like what the conductor said. She laughed indicates that she was amused by what the conductor said.



Q. 5. What does the line ‘gradually a tiny wish crept into her head and grew there’, mean?



Explanation: Vigorous means strong and forceful. Overpowering means intense or uncontrollable. Longingly means yearningly or desirously.





(iii) joyfully



(ii) overt





(C) (i) staunch





(B) (i) vigorous (ii) overpowering (iii) cheerfully



(iii) longingly



(ii) overt



(A) (i) active

(A) i-f, ii-c, iii-a, iv-d (B) i-c, ii-d, iii-a, iv-e (C) i-b, ii-f, iii-e, iv-a (D) i-f, ii-d, iii-c, iv-b Ans. Option (B) is correct. Explanation: Bring to mind refers to recollecting an old thought or memory. Be in two minds refers to confused between two or more ideas. Have an open mind refers to consider someone else’s ideas also. Put your mind at ease refers to giving surety to someone that the problem will be resolved.

Q. 4. A part of the extract has been paraphrased. Choose the option that includes the most appropriate solution to the blanks. The desire became so (i) _______ that it transformed into an (ii) _______ one. Valli would look (iii) ________ at people who boarded the bus and got off.





















166

@PROCBSE

(A)

(i)

irritated

(ii)

agitated

(B)

(i)

concerned

(ii)

upset

(C)

(i)

upset

(ii)

agitated

(D)

(i)

upset

(ii)

irritated

167

Ans. Option (D) is correct. Explanation: Valli was not happy (upset) and also wanted to get rid of (feeling irritated) the old lady.

Q. 5. How do you think Valli felt when the old woman asked her questions? Pick the option that best describes her feelings.



MADAM RIDES THE BUS





SUBJECTIVE TYPE QUESTIONS Short Answer Type Questions



(40-50 words & 3 marks each) medication, her sunglasses, Id proofs, some woollen clothes if she is going to a cooler place. Sunscreen lotion and moisturising lotion are also advisable. She should also carry some eatables and water to drink.

Q. 6. What was the most fascinating thing that Valli saw on the street? [Delhi Set-I, 2019] Ans. Watching the bus travel between her village and the nearest town; sight of the bus filled each time with new set of passengers was a source of unending joy for Valli. [CBSE Marking Scheme, 2019]















Detailed Answer:





Q. 7. How did Valli save money for her first bus journey? [Board Term-II, 2016, Set-2/2/1][NCERT] 











The most fascinating thing that Valli saw was the bus that travelled between her village and the nearest town. It passed through her street each hour, once going to the town and once coming back. Each time it was filled with a new set of passengers.



[CBSE Marking Scheme, 2016]









Ans. Value Points: Resisted the temptation of buying balloons, toys, etc., didn’t enjoy merry-go-round in the village fair –collected the amount required. Detailed Answer:







To save money for her first bus journey, she resisted the temptations of buying balloons, toys, etc., in the village fair. She also didn’t enjoy merry-go-round there. Thus, she collected the amount, required for the bus-journey.

These questions are for practice and their solutions are available at the end of the chapter



Q. 8. How did Valli react when she saw the dead cow by the roadside? [Board Term-II, 2016]



Ans. If Valli had to plan a day-long budget trip, I would suggest her to keep a few medicines for personal































Q. 1. During her journey, Valli absorbed the natural beauty and clapped her hands in happiness on seeing a young cow running very fast. What does this reveal about Valli? [CBSE-QB, 2021] Ans. Valli thoroughly enjoyed her ride to the town and laughed and clapped when the young cow ran on the middle of the road. Valli, on the whole, can be described as a curious, joyful, smart, bold and at the same time a mature girl. Q. 2. Valli didn’t like the way adults treated her during her bus journey. Describe how you would feel and react if you were to find yourself in a similar situation. [CBSE-QB, 2021] Ans. Valli showed that she was a self-respecting girl. She politely refused to accept a free drink from the conductor. Even I would have reacted in the same way. I would have not relished the unpleasant smell that comes from the betel nut the women was chewing. Q. 3. Imagine you come across a little girl travelling alone in a bus. Write down any two questions you would ask her and your reasons for asking them. [CBSE-QB, 2021] Ans. The two questions I would ask the little girl are: "Are you travelling alone, child? What was such an urgency that your parents allowed you to travel alone?" The reason for asking such questions is honest concern for the child and to help her. Q. 4. Do you think Valli’s mother would have taken her for the bus ride if she had requested her to do so? Why/ Why not? [CBSE-QB, 2021] Q. 5. Imagine Valli had to plan a day-long budget trip to a domestic destination for her family all by herself. What things do you think she would keep in mind while planning?

@PROCBSE 168

Oswaal CBSE Question Bank Chapterwise & Topicwise, ENGLISH LANG. & LIT., Class-X

Long Answer Type Questions



(100-120 words & 6 marks each)















Ans.



















Q. 4.













These questions are for practice and their solutions are available at the end of the chapter













Ans. One must be ambitious in life – Valli was so – a simple desire, to ride a bus- watched the bus, its schedule, listened to people’s conversation about their journey - fare - collected 30 paisa (any other relevant points) [CBSE Marking Scheme, 2017] 



9:00 pm

I feel I have learnt a big lesson in life after reading about Valli. I feel ashamed for the way I have been............ [CBSE-QB, 2021] Ans. 15 August, 2020, Saturday 9:00 PM I feel I have learnt a big lesson in life after reading about Valli. I feel ashamed for the way I have been unreasonable about the demands I have forced on my parents. I have thoughts of the hurt I was unknowingly causing them due to my rigid and





Q. 5.



15 August, 2020, Saturday

irrational behaviour. Many a times, I have seen my mother buy me those unnecessary things which made me happy. I wanted to possess them just to show off to my friends. But now I have well understood that this is always going to be a better version of everything you own. So let go of your need to have the latest and greatest things and be happy with what you have. I have understood that no matter how much you have, it never feels like enough. This is because we are trying to fill an emotional void with a tangible thing. The more you have, the more you crave. Being able to live an extravagant life seems to make it more difficult to enjoy the small things. Slow down, live in the present and savour in every positive experience for a longer-lasting smile. So now, I believe in less stuff, less clutter and less stress. More isn’t always better. Valli was sensitive, but at the same time, she was fearless and quite determined. Think of another character from your textbook, First Flight, who displayed similar character traits. Compare and contrast the lives of Valli and the other character. How did Valli fulfil her desire to ride a bus to the town and back? (CBSE, 2020) Valli made detailed plans about her bus ride. She gathered information about the distance the bus travelled, the time it took and the fare by listening to the conversations of others and asking discreet questions. She planned the timings, i.e., when she would board the bus and when she would return. She saved money for the trip by not buying peppermints and sweets and also by not going to the fair. Two characteristics which made Valli to fulfil her dream of a bus ride were her courage and her ability to plan. These are the traits which need to be consciously developed so that a person can undertake what he wants to do. Courage to do something by taking risk and ability to plan will make a person go ahead in life. Whenever we want to achieve something, difficulties always come in our way. What did Valli have to do to go and ride in a bus? [CBSE Delhi Set-I, 2017] OR Once we decide to achieve something, so many difficulties come in our way. With focused attention we can make that achievement. How did Valli succeed in fulfilling her desire of riding a bus? [CBSE OD Set-I, 2017] 

























Q. 3.































Q. 1. Travelling teaches us many life lessons and enriches us with experiences. You and your classmate have a discussion on this statement and your love for travel. Write the dialogue. You may begin like this... Friend: I think travelling is one of the best ways to explore not just the world; around you but also helps you introspect about life itself. You: Friend: Travelling enriches us with so many varied experiences. You: [CBSE-QB, 2021] Ans. Friend : I think travelling is one of the best ways to explore not just the world around you, but also helps you introspect about lifestyle. You : Yes, you are right. It also improves social and communication skills. Friend : Travelling enriches us with so many varied experiences. You : Absolutely, one gets original and creative thoughts. One gets out of his comfort zone. You learn the most in uncomfortable and unfamiliar situations, you know how to act and respond to people and your surroundings. You : Travelling also broadens your horizons and enhances your tolerance for uncertainty. Friend : It was really a great experience talking to you. You : These advantages of travelling are incredible. Thank you. Q. 2. After reading the story about Valli; you realise that she had such a small desire and demanded nothing from her mother. After reflecting on this, you decide to write a diary entry on how you felt you have been unreasonable about the demands you have posed to your parents. You also realise that life is not all about materialistic possessions. You may begin like this:

@PROCBSE

MADAM RIDES THE BUS









Detailed Answer: It was Valli’s earnest wish to ride on the bus that travelled from her village to the nearest town. She knew she might have to face many problems, but her desire didn’t let anything come in the way. She planned meticulously and started saving money for the ride. She resisted every temptation that came her way. Meanwhile, she collected all the information regarding the bus journey and waited patiently for the day to arrive. Finally, the day arrived and when her mother was asleep in

the afternoon, she went for the bus ride. She paid the fare to the conductor, took a seat and enjoyed the outside scenery from the window. Her confidence and dignity made the conductor call her ‘Madam’. With extraordinary courage and firm determination, she completed her journey. She emerged out as a victorious person who had done something worthwhile. Q. 7. “Never mind”, she said, “I can get on by myself.” “You don’t have to help me,” said Valli to the conductor. She shows extraordinary courage in taking the bus journey all alone. Taking inspiration from Valli’s character, write how ability and courage to take risk are essential to fulfil one’s dream. [CBSE, 2015]



Valli had set the target - to have a joy-ride to an unknown destination and realised her dream – One example from the life experience of some great man who had set the goal and taken risk to achieve the goal. [CBSE Marking Scheme, 2015] Detailed Answer: It is evident that ability and courage to take risk are essential to fulfill one’s dream. Valli, an eight year old girl, never stepped out of her village. Her parents never allowed her to go outside her house. But she mustered up the courage to take up a bus journey all alone. She enjoyed the journey and understood the reality about the outside world. She was a young girl but she made the journey without any difficulty because she had the courage to take risk. Valli had set the target to have a joy-ride to an unknown destination and she was successful in hitting the bull's eye as she had realised her dream. She couldn't resist her overwhelming desire to ride the bus atleast once. Nevertheless, we can say that the task that seems to be difficult is actually not so tough. We can easily accomplish it if we have the ability and courage to take risk. it, we do not dare to take risk, we lose opportunities to get better.



Ans. –











Ans. • Valli’s earnest wish to ride on the bus and get to the nearest town, meticulous planning, strong will power. • Saved money, gathered required information, waited patiently. • Her confidence and dignity made the conductor call her ‘Madam’. • Displays extraordinary courage, completes the journey. [Board SQP Marking Scheme, 2020]

















Detailed Answer: Yes, it is true that whenever we want something, difficulties do come in our way. Achieving goals require perseverance. Valli wanted to ride the bus; however, she did not have the money to do so. The challenge in front of her was to arrange for the required amount of money. Valli had carefully saved whatever stray coins came her way, resisting every temptation to buy peppermints, toys, balloons and the like. Finally, she had saved sixty paisa. At the village too, she made the best efforts to save as much money as possible. She was tempted to ride the merry-go-round as she had the money. However, she suppressed her strong desire and saved the money for the bus ride. Her data collection, planning, conduct and self control were executed with great confidence and courage, making her ‘excursion’ on the bus a great success. Q. 6. Valli shows extraordinary courage in taking a bus journey all alone. Explain how ability and courage are essential to fulfil one’s dream. [CBSE SQP, 2020]





Commonly Made Error Many candidates expressed their own desires and wants instead of describing the desires of Valli.





Answering Tips





169

The answer has to be written specifically about Valli. A list can be prepared to understand the characters in the chapter.

@PROCBSE 170

Oswaal CBSE Question Bank Chapterwise & Topicwise, ENGLISH LANG. & LIT., Class-X

Solutions for Practice Questions (Topic-1) Long Answer Type Questions















Ans. 3. A character who has similar characteristics, of fearlessness and determination like that of Valli, would be Anne Frank. In the lesson, she was brave enough to speak and prove herself to her teacher so candidly. Even though she was extremely talkative, it never affected her academics. She was a committed student, daughter, sister and more importantly, a true being. As a young girl of 13 years, she had to go through several difficulties during the World War era. Her family had to live in secrecy, their lives were under great threat when the Nazis caught hold of them and they were sent to the concentration camps. Anne’s life was secluded like that of Valli’s, both of them had limitless ideologies of being adventurous. It is also astonishing to notice such great heights of maturity and responsibility in children of such tender ages. Lastly, both Anne and Valli had clear goals which inspired them to dream and strive for letting them come true.

Ans. 4. Definitely, Valli’s mother would have taken her for the bus ride if she had requested her to do so. It was so because no parent, especially the mother, would break the heart of her child. The parents try to fulfill each and every desire of their children. Ans. 8. Value Points: She became sad, no more enthusiasm on seeing the dead cow, now in pensive mood, negative views towards life. CBSE Marking Scheme, 2016] Detailed Answer: When Valli saw the dead cow by the roadside, she became very sad. She was now in pensive mood as it was shocking to her that the animal, she’d been laughing at just a while ago, was lying in the stillness of death.



Short Answer Type Questions





REFLECTION One must be ambitious in life. Do you have any ambition? How will your achieve it?

@PROCBSE

CHAPTER

8

THE SERMON AT BENARES – By Betty Renshaw

Revision Notes Scan to know more about this topic

Introduction

asking medicine for him. At last, a man told her to go to Buddha, the Sakyamuni. Buddha told her that he would cure her son if she brought some mustard seeds from a house where no death The Sermon has taken place. Kisa went at Benares from house to house, but was unable to find one where no death had been seen. She finally realised that death was common to all and no one could avoid dying. No one can save their relatives. People weep over their dead ones. It is only the wise who do not grieve as they have accepted the truth. If a person weeps, his sufferings only become greater. Those who do not grieve have peace of mind and will overcome sorrow.





‘The Sermon at Benares’ is a chapter written by Betty Renshaw. The chapter covers the journey of Gautam Buddha from princehood to his saintly life. After seeing the suffering of the world, he decided to give up all worldly pleasures and sought enlightenment. He finally attained salvation under a tree at Bodhgaya.



Key Words

Extract Based Questions



(5 marks each)













Q. 1. The community’s response to Kisa in the above extract was somewhat different from before. Why do you think that was the case? (A) They had learnt from Buddha’s sermons. (B) They were able to help Kisa in some way this time. (C) They understood parental grief. (D) They liked Kisa and enjoyed talking to her. Ans. Option (B) is correct. Explanation: Earlier, they could not help Kisa, but this time they wanted and were able to help Kisa in their own ways.













I. Read the following extract and answer the questions/ complete the sentences that follow: (5×1=5) Poor Kisa Gautami now went from house to house and the people pitied her and said, “Here is mustard-seed; take it!” But when she asked, “Did a son or daughter, a father or mother, die in your family?” they answered her, “Alas! the living are few, but the dead are many. Do not remind us of our deepest grief.” And there was no house but some beloved one had died in it. [CBSE-QB, 2021]

or





1. Enlightenment: To give knowledge understanding 2. Salvation: Process of being saved. 3. Grieve: To cause sorrow or distress to







Gautam Buddha (563 B.C. to 483 B.C.) was a prince. His parents had named him Siddhartha Gautama. He had been shielded from the sufferings of the world. At the age of twenty five, he saw a sick man, an aged man and a dead man. He also saw an ascetic begging for alms. Unable to understand that, he went in search of spiritual knowledge. After wandering for seven years, he sat under a Peepal tree and vowed that he would stay there until he got enlightenment. He got that after a week and named the tree as ‘Tree of Wisdom’. He himself came to be known as Buddha. He gave his first sermon at Benares. This sermon contained ten important points. These points were conveyed through the story about Kisa Gautami whose only son had died. She went to people



Summary

@PROCBSE Oswaal CBSE Question Bank Chapterwise & Topicwise, ENGLISH LANG. & LIT., Class-X

















Q. 3. “Do not remind us of our deepest grief.” The tone of the speaker(s) is ________________. (A) disillusioned (B) sceptical (C) ironic (D) solemn



















Ans. Option (D) is correct. Explanation: Disillusioned means disappointed or disheartened. Sceptical means not convinced or having doubts. Ironic means satirical or sarcastic. Solemn means sincere or grave. The speaker here is feeling solemn for Kisa. Q. 4. Pick the option that explains — ‘...the living are few, but the dead many.’ (A) It shows the high death rate and low birth rate in the city of Benares. (B) It highlights the holy status of Benares where many Hindus go to die. (C) It throws light on the numerous loved ones the villagers had lost over time. (D) It reflects that many children had died in the village for various reasons.

To live in heart we

In another time in

leave behind is not

a happier place

to Die

we will meet again











Ans. Option (C) is correct. Explanation: Kisa Gautami was considered and referred to as unfortunate because of her emotional losses.

















(A) 1 and 2 (B) 2 and 3 (C) 3 and 4 (D) 1 and 4 Ans. Option (B) is correct. II. Read the following extract and answer the questions/ complete the sentences that follow: (5×1=5) Not from weeping nor from grieving will anyone obtain peace of mind; on the contrary, his pain will be the greater and his body will suffer... He who seeks peace should draw out the arrow of lamentation and complaint and grief. He who has drawn out the arrow and has become composed will obtain peace of mind. [CBSE-QB, 2021] Q. 1. If the Buddha were a counsellor, which of the following options would you find in a brochure of his service? 

Q. 2. Which of the following options represent the correct understanding of the word “poor” in the phrase “Poor Kisa Gautami”? (A) In need of money (B) Weak (C) Unfortunate (D) Inferior



172

Thomas Campbell



Unknown (ii)



(i)





Grief 1. Denial 2. Anger 3. Bargaining 4. Depression



5. Acceptance

(iii)













(A) Image (i) (B) Image (ii) (C) Image (iii) (D) Image (iv) Ans. Option (C) is correct. Explanation: Buddha advocated that death is the ultimate truth of life which cannot be escaped. He preached that we should work according to our dharma and try to get rid of sufferings. So, image (iii) best suits the teachings of Buddha. Q. 2. Which of the following statements cannot be attributed to the Buddha based on the given extract? (i) Pain and grief are unavoidable and necessary. (ii) The inevitability of death makes grieving futile. (iii) Universality of grief and pain makes us sad. (iv) Understanding that life is finite leads to wisdom. (A) (i) and (ii) (B) (i) and (iii) (C) (iii) and (iv) (D) (i) and (iv) Ans. Option (B) is correct. Explanation: Pain and grief cannot be avoided as they are part and parcel of life, but they are not necessary always. Sometimes we learn from

























(iv)











Ans. Option (C) is correct. Explanation: It refers to the few loved ones who are alive, but dead loved ones who are many more in number and lost by all the villagers. Q. 5. Imagine you are a photo journalist visiting the city at the time Kisa Gautami went from house to house. You documented her experience given in the above extract in a photo series. Your publisher wants to publish the photo series in three parts wherein Part 1 shows Kisa’s visits to the houses; Part 2 depicts her conversations with people and Part 3 captures Kisa’s reflections at the end of the day sitting by the wayside. The publisher would also like you to choose titles for the series and its three parts. Look at the titles given below and choose the options that provide the most appropriate set of titles. (1) Series Title - From Darkness to Light. Part I - Living in Loss; Part II - A Mother’s Journey; Part III - Mustard Seed (2) Series Title - Mustard Seed. Part I - A Mother’s Journey; Part II - From Darkness to Light; Part III -Living in Loss (3) Series Title - A Mother’s Journey. Part I - Mustard Seed; Part II - Living in Loss; Part III - From Darkness to Light (4) Series Title - Living in Loss. Part I - From Darkness to Light; Part II Mustard Seed; Part III - A Mother’s Journey

The 5 Stages of

Death, after all, is the common exception from birth. Neither heroes nor cowards can escape it. Ellis Peters

@PROCBSE THE SERMON AT BENARES 













































Q. 5. Gautam Buddha got enlightenment _________ tree . (A) Banyan (B) Mango (C) Ashoka (D) Bodhi Ans. Option (D) is correct.







under























Q. 3. The Prince left the palace in search of _________ . (A) freedom (B) treasure (C) knowledge (D) enlightenment Ans. Option (D) is correct. Q. 4. Find a word from the passage that means the same as ‘looked at’. (A) Shielded (B) Glanced (C) Procession (D) Enlightenment Ans. Option (B) is correct. Explanation: Shielded refers to giving protection. Glanced refers to giving a quick look. Procession refers to a great crowd of people following something or someone. Enlightenment means attaining the divine light.















III. Read the following extract and answer the questions/ complete the sentences that follow: (5×1=5) At about the age of twenty five, the Prince, thereof shielded from the sufferings of the world, while hunting out glanced upon a sick man, then an aged man, then a funeral procession and finally, a monk begging for alms. These sights so moved him that he at once became a beggar and went out into the world to seek enlightenment concerning the sorrows he had witnessed. (CBSE 2016, [Modified]) Q. 1. What was the age of Gautam Buddha when he shielded himself from the sufferings of the world? (A) Twenty two (B) Twenty four (C) Twenty eight (D) Twenty five Ans. Option (D) is correct. Q. 2. The Prince was _________ by the effect of sights. (A) moved (B) carried away (C) not moved (D) not disturbed Ans. Option (A) is correct. Explanation: ‘Moved’ here means stirred and stimulated.

















Q. 5. The given extract is paraphrased below. Choose the option that includes the most appropriate solutions for the blanks. (i) The act of ______________ is not only pointless, but (ii) rather _________, causing pain and suffering. It only serves to (iii) __________ peace of mind. (iv) __________ without complaint is crucial to well-being and peace. (A) grief, harmful, destroy, accepting (B) grieving, counterproductive, take away, acceptance (C) weeping, productive, take away, lamentation (D) grieving, harmful, destroy, lamenting. Ans. Option (B) is correct.

173













Q. 4. According to the Buddha, peace of mind is attainable by those who: (A) renounce worldly life like the Buddha. (B) take out the arrow and become strong. (C) grieve, suffer and then move on. (D) recognize terms of life and let go of complaint. Ans. Option (D) is correct. Explanation: Buddha preached that peace of mind can be achieved only when we accept the reality and stop complaining about what we have not got.











Q. 3. Choose the option that appropriately completes the following: lamentation: grief:: _______: _______ (A) laughter: joke (B) discomfort: fear (C) celebration: joy (D) resignation: loss Ans. Option (C) is correct. Explanation: Expression of grief is lamentation; expression of joy is celebration.





others’ experiences also. Similarly, our own grief and pain makes us sad but universality of grief and pain pacifies us and we become wise after developing this understanding that no one is free from suffering or pain of loss.





SUBJECTIVE TYPE QUESTIONS Short Answer Type Questions















Q. 2. Elucidate any one quality that Siddharth demonstrated when he gave up his status and family. Explain your choice. [CBSE-QB, 2021] Ans. The sudden decision to renounce the world, his family and princehood shows Siddharth was a strong-willed and determined man. He had the courage to take such a difficult decision.

Q. 1. What is the significance of the Buddha’s request for a handful of mustard seeds and the addition of a condition to it? [CBSE-SQP, 2021-22] Ans. Mustard seeds were easy to procure as it was available in every household she knocked at. The Buddha’s request for a handful of mustard seeds was to make the woman understand that death and sufferings were inevitable.





(40-50 words & 3 marks each)

@PROCBSE Oswaal CBSE Question Bank Chapterwise & Topicwise, ENGLISH LANG. & LIT., Class-X





























Ans. Found no house where some beloved had not died; understood death is common to all; she was being selfish in her grief. [Board Marking Scheme, 2019] Detailed Answer: Kisa Gautami’s only son had died. She went to Buddha to seek some medicine to revive her son. The Buddha asked her to bring some mustard seeds, from the house where there had been no death ever, but Gautami could not find any such house. Thus, she realised that death is inevitable for every mortal being. Cycle of life and death is a normal process. Q. 8. Why did Buddha choose Benares to preach his first sermon? [CBSE, 2017] Ans. The Buddha went out to seek enlightenment which came after sitting under a peepal tree. He renamed it as ‘Bodhi Tree’ and went to Benares to preach his first sermon. He went there because it is the holiest of the dipping places along the river Ganges. Q. 9. What did the Buddha want Kisa Gautami to understand? [Board Term-II, 2015, Set 2/1/3] Q. 10. Why was Kisa Gautami sad? What did she do in her hour of grief? [Board Term-II, 2015] Ans. Kisa Gautami was sad because her only son had died. In her hour of grief, she went from house to house in search of a medicine to cure him. She had become selfish in wanting her son back.





























death comes to any beloved one. Also, everyone born on this earth has to die one day regardless of one’s power and position. Q. 7. How did Kisa Gautami realise that life and death is a normal process? [Delhi Set I, 2019]



Q. 3. Do you think being enlightened placed a far greater responsibility on the Buddha than being king would have? Justify your stance. [CBSE-QB, 2021] Ans. Yes, I do think that being enlightened placed a far greater responsibility on the Buddha. If he would have been just a king, then he would have to just serve his kingdom and people. By being enlightened, he had to serve for the whole humanity which is much greater work then being a king. Q. 4. Kisa understood the temporality of life as she sat at the wayside watching the city lights. Can you think of any other object or phenomenon, natural or man made, that might similarly reflect the fragile nature of human life? Justify your choice. [CBSE-QB, 2021] Ans. The rising and setting of the sun can be compared to the fragile nature of human life. Just like the sun rises every day and later the darkness of the night spreads everywhere, similarly, the human beings take birth, then the evening of life, i.e., the old age draws in and the sun of life sets in. Q. 5. Grief is often seen as a measure of love. Do you think the Buddha’s sermon undermines a mother’s love? Justify your response. [CBSE-QB, 2021] Q. 6. What message might the Buddha’s story hold for those who are in positions of power and privilege? [CBSE-QB, 2021] Ans. Buddha’s story holds the message of the grief and suffering that everyone has to face after the death of beloved, be it the man with position and power. Everyone is treated equally by death. Everyone has to face the same sadness, grief and pain when



174

Long Answer Type Questions











tree after wandering for seven years in search if enlightenment. He gave his first sermon at Benares on the River Ganges. He preached that all men, women and children are mortals. And all mortals, be it a rich or poor, are destined to die. Actually, death and decay is the fate of all mortals in this world. Death spares none. Those who are born can’t avoid dying. Therefore, the wise do not grieve. Q. 2. The Sermon at Benares could just as well be considered another glimpse of India. If it were part of the story ‘Glimpses of India’, what ideas, values and/ or experiences would it highlight? How would you present this historical part of India? [CBSE-QB, 2021] Ans. India is a land of unity in diversity. The high mountain ranges, vast seas and countless rivers depict the diversity in unity. Each and every region of the country, from Jammu and Kashmir to Kanyakumari portrays different customs and traditions. The story of Buddha, too, portrays a beautiful picture of Indian history.

These questions are for practice and their solutions are available at the end of the chapter



Can you relate this to the Buddha’s life before and after he attained enlightenment? Provide examples from the text to support your answer. [CBSE-QB, 2021] Ans. Gautam Buddha began his life as a royal prince. He was named Siddharth Gautam. At the age of sixteen, he got married to a princess. The prince was deliberately shielded from all sufferings of the world. But this attempt failed when the prince while out hunting chanced upon a sick man. Then, he saw an aged man. He also chanced to see a funeral procession. Finally, he saw a monk begging for alms. These sights of sufferings, sickness and decay shocked and moved the prince. He wanted to seek the final solutions of all these sorrows and sufferings. Finally, he sat down under a big





Q. 1. Read the given quote by actor Jim Carrey.









(100-120 words & 6 marks each)

PROCBSE THANK YOU FOR DOWNLOADING BOOK. WE HERE AT PROCBSE MAKE SURE THAT EVERY PIECE OF MATERIAL REACHES YOU WITHOUT DIVING DEEP INTO THE TABS OF YOUR BROWSER. THIS E-BOOK HAS BEEN DOWNLOADED TOTALLY FREE FROM PROCBSE ON TELEGRAM. UPI ID - PROCBSE.FAMC@IDFCBANK

CLICK TO VIEW/OPEN

@ PROCBSE JOIN US FOR QUESTIONS BANK , SAMPLE PAPER , PYQ , NOTES , MIND MAPS AND MANY MOTE THINGS ⚡

@PROCBSE

You can contribute a little to our efforts. Even a small amount can do wonders.

DONATE US PLEASE HELP US

OR

CONTRIBUTE TO OUR UPI ID PROCBSE.FAMC@IDFCBANK

@PROCBSE THE SERMON AT BENARES

175

one to death. Kisa was unsuccessful in getting any seeds. She was distraught on the loss of her only son. The futile search made her realize the truth that humans are mortal. They have to face pain and suffering in their lives. It is a part and parcel of their life and it is of no use grieving over them. She was ignorant of the fact that death is inevitable and must come to all. In other words, she understood that human life is full of trials and tribulations which can be answered by a human being through his own efforts. Man has to face all. Weeping and grieving over the loss won’t fetch peace of mind. Q. 6. What lesson on death and suffering did Buddha teach Gautami in the chapter ‘The Sermon at Benares’? (CBSE, 2018) OR Why did Gautami go to the Buddha? What lesson did he teach her? (CBSE O.D. Set-3, 2019)



























• •





















Her only son died. In grief, she carried the dead son to all neighbours for medicine. A man asked her to go to the Buddha to get medicine to cure him. The Buddha asked her to bring a handful of mustard seeds from a house where no one had died. Kisa Gautami could not find the house where some beloved had not died. Came to know that death is common to all. The Buddha made her realise that humans are mortals. (CBSE Marking Scheme, 2019)





These questions are for practice and their solutions are available at the end of the chapter



Detailed Answer: Kisa Gautami was devastated by the death of her only son. As suggested by Shakyamuni, she wandered from home to home, asking for mustard seeds. The only condition was that the seeds should be from a house where people had not lost a loved











Kisa went from home to home, asked for mustard seeds - where no one had died, unsuccessful in getting any, distraught on the loss of her only son, loses her senses, doesn’t accept reality, ignorant of the fact that death is inevitable. • Lesson- humans are mortal, face pain and suffering, no use grieving. (Board Marking Scheme, 2020)

Detailed Answer: Kisa Gautami had only one son. The son died. In her grief, she carried him to her neighbours, asking them for some medicine. The people said that Gautami had lost her senses. Someone sent her to the Buddha. Kisa Gautami went there and asked him to give her some medicine for her child. The Buddha asked her to bring a handful of mustard seeds. The Buddha further told her that the mustard seeds had to be brought from that house that had not lost a child, husband, parent or friend. Kisa Gautami went from home to home, but she could not find any house where nobody had ever died. Kisa Gautami became weary and hopeless. The Buddha told her that the life of the mortals in this world is troubled and brief and combined with pain. The Buddha further told Kisa Gautami that every person who is born, has to die one day. The Buddha further told her that some die at old age, while some others die quite young. Thus, Buddha was able to teach Gautami the lesson about the inevitability of death. Q. 7. “The life of mortals in this world is troubled and brief and combined with pain.... .” With this statement of the Buddha, find out the moral value that Kisa Gautami learnt after the death of her child. [CBSE Set-I, 2016]









Ans. •



Ans. • •

Q. 3. Describe the journey of Siddhartha Gautama becoming the Buddha. [CBSE Delhi Set-2, 2020] Q. 4. What lesson did Kisa Gautami learn the second time that she had failed to learn the first time? [CBSE Outside Delhi Set-1, 2020] Ans. Kisa Gautami was devastated by the death of her only son and wandered door to door, seeking help. Someone directed her to Shakyamuni, the Buddha, who asked her to bring a handful of mustard seeds. This raised a hope in Gautami’s heart that her son could be revived. But the condition imposed by Shakyamuni was that the seeds should be from a house where people had not lost a loved one to death. Kisa Gautami’s futile search made her realize the bitter truth that sorrows are a part and parcel of life and one can attain peace only by acceptance. Buddha says that everything in this world is subject to death. He further says that the world is deeply affected by suffering, disease or pain. Inevitably, there is death and decay, therefore the wise do not grieve, knowing the terms of the world. Neither from weeping nor from grieving can anyone obtain peace of mind. Q. 5. Life is full of trials and tribulations which can be overcome by a human being through his own efforts. Explain with reference to Kisa Gautami’s life. [CBSE SQP, 2020]









When Kisa Gautami went to Buddha for the medicine, Buddha told her to get a handful of mustard seeds from a house where no one had died. Kisa became weary and hopeless after going to several houses and getting the same response. She finally grasped the Buddha’s underlying meaning, thereafter, the Buddha sermonized her that the life of mortal in this world is troubled and painful. That the world is afflicted with death and decays and so there is no point in grieving over something which is inescapable. He gave her examples of ripe fruits and earthen vessels whose ‘lives’ are short. This way he made her realize that death is unavoidable and not even the near and dear ones can save anyone from death





@PROCBSE Oswaal CBSE Question Bank Chapterwise & Topicwise, ENGLISH LANG. & LIT., Class-X







Ans. Value Points: • Death is common to all, accept the truth, things not everlasting, detachment from mundane life, farsightedness • Weeping and grieving cannot bring peace of mind. • He who overcomes sorrow will be blessed. [CBSE Marking Scheme, 2016]









176





Topper's Answer, 2018

Detailed Answer:



When Kisa’s only son had died, she went to Buddha. Being plunged into deep grief, first she went to every neighbour, asking for medicine for her dead son. She had lost all her senses and forgot that no medicine could bring back the dead. Then she went to Buddha for making his son alive. Buddha asked her to bring some mustard seeds

from a house where no death had occurred. But she couldn’t find such a house. Buddha made her realise that death is common to all. One must accept the truth and should have far-sightedness. Weeping and grieving over the death of near and dear ones cannot bring peace of mind. Blessed is the person who overcomes sorrow. She learnt that a person who remains composed after the death of his kinsmen obtains peace of mind.





Commonly Made Error Students do not focus on moral values taught through the lesson. They write the answer related to Kisa Gautami’s incident.





Answering Tip Students must read the question, understand what is being asked and then write the answer focusing on what is being asked in the question.

@PROCBSE

THE SERMON AT BENARES

177

Solutions for Practice Questions (Topic-1) Short Answer Type Questions







Ans. 5. Grief is a measure of love for those who are not enlightened and are still trapped in this materialistic world. A spiritually enlightened person can understand and accept that the physical life is only temporary, just a stop where the soul stays temporarily, before the transition. Ans. 9. Buddha wanted Kisa Gautami to understand that death is common to all and no one can avoid dying, no one can save their relatives. So the wise do not grieve after accepting this truth of death.

Long Answer Type Questions

Ans. 3. Gautam Buddha was a prince. His parents had named him Siddhartha. He had been shielded

from the sufferings of the world. At the age of twenty five, he saw a sick man, an aged man and a dead man. He also saw an ascetic begging for alms. These sights moved him so much that he at once went out into the world to seek enlightenment concerning the sorrows he had witnessed. He wandered for seven years and finally sat down under a peepal tree, where he received enlightenment after seven days. He renamed the tree as the Bodhi Tree (Tree of Wisdom). He himself came to be known as the Buddha (The awakened or the Enlightened). The Buddha preached his first sermon at the city of Benares, the most holy dipping places on the River Ganges. The Sermon was preserved. It reflects the Buddha’s wisdom about one inscrutable kind of suffering..





REFLECTION Were you able to identity the underlying message "Death is inevitable", based on your reading of the chapter?

@PROCBSE

CHAPTER

9

THE PROPOSAL – By Anton Chekhov

Revision Notes Scan to know more about this topic

Introduction

‘The Proposal’ (originally titled ‘A Marriage Proposal’) is a one-act play, a farce by Anton Chekhov. The play highlights the tendency of wealthy families seeking ties with other wealthy families to increase their estates by encouraging marriages that make good economic sense.

educated. He feels that it is the time he should settle down. Natalya, who doesn’t know he has come to propose her, feels that he has come to claim The proposal Oxen Meadows as his own. This results in a quarrel with her father Chubukov joining in. After much mudslinging, when Natalya comes to know that he has come to propose her, she becomes willing to marry him. The proposal is made and accepted, with the knowledge that quarrels will take place in life and important things should not be side-lined because of this.

Summary

Key Words

1. Quarrelsome: Argumentative 2. Trivial: Of little importance, ignorable



This play is about how wealthy families seek to tie up with one another through marriages in order to increase their property and wealth. Lomov is a rich bachelor who wishes to marry Natalya, the daughter of Chubukov who himself is a rich man. The play deals with how all the three characters are very quarrelsome. They quarrel over trivial issues and call each other names. Lomov is not really in love with Natalya but wants to marry her as he feels she is a good house-keeper, beautiful and well

Extract Based Questions

CHUBUKOV: Your grandfather was a drunkard and your younger aunt, Nastasya Mihailovna, ran away with an architect and so on...





I. Read the following extract and answer the questions/complete the sentences that follow: (5×1=5)



NATALYA: All, all, all!



CHUBUKOV: Your father was a guzzling gambler! [CBSE-QB, 2021] 

Q. 1. Choose the option that correctly identifies the tone of the characters in the given extract.

CHUBUKOV: You Lomovs have had lunacy in your family, all of you!

LOMOV: And your mother was hump-backed. [Clutches at his heart] Something pulling in my side... My head.... Help! Water!



Never mind about my people! The Lomovs have all been honourable people and not one has ever been tried for embezzlement, like your grandfather!













(5 marks each)

@PROCBSE

THE PROPOSAL

(B) (ii) and (iv)

(C) (i)and (iii)

(D) (iii) and (iv)

Ans. Option (B) is correct.









Explanation: Vicious means spiteful or malicious.















(A) (i)and (ii)

Ownership means being owner of something.



Ans. Option (C) is correct. Explanation: The tone of the characters is that of dislike and resentment (antagonism) and disapproval and disrespect (contempt).

Circus (here) means show or extravaganza. Civility Q. 5. Which of the following options comes closest to



means politeness and graciousness. the meaning of ‘tried’ as used in the extract?

Q. 2. The playwright’s intention in the given extract is to:

(A) She mastered the game through a process of

(A) throw light upon the weaknesses of the rich in any society.

(B) He followed the trial closely and was seen in







trial and error.





(D) (i) ugly ; (ii) neighbourhood ; (iii) barrage; (iv) goodness



(iv) Irony



(ii) Humour

(iii) Contempt



(i) Antagonism

179

(B) emphasize that family history is important in a marriage proposal.

court every day.

(C) This had been a tried and tested formula for

(C) satirise the superficiality of the upper class in Russian society.

the organisation.

(D) They tried with all their might to repeat their

(D) send a message that ego is not healthy in any relationship.



Ans. Option (B) is correct. Explanation: In the given extract, tried refers to

Q. 3. If according to Chubukov and Natalya, Lomovs are not “honourable people”, why do they still consider Lomov’s proposal?

proceedings of a court case which is closest to that



Ans. Option (C) is correct.

used in option (B).

(A) Natalya can take care of her father if she marries close by.

III. Read the following extract and answer the 











earlier successes.

(5×1=5)





questions/complete the sentences that follow: CHUBUKOV: What a weight off my shoulders, ouf! NATALYA: But, still you will admit now that Guess is worse than Squeezer.

supersedes

NATALYA: Worse! CHUBUKOV: Well, that’s a way to start your family bliss! Have some champagne!



Q. 4. Imagine you found the playwright’s notes for each scene in the play and noticed that some of the words were missing.





Ans. Option (D) is correct.

LOMOV: Better!



society



in



NATALYA: Worse! Worse! Worse!

CHUBUKOV:

[trying

to

shout



her

down]

[CBSE-QB, 2021]

Q. 1. Which of the following titles of Shakespeare’s

plays might best apply to the relationship of Lomov and Natalya’s relationship?

(A) The Taming of the Shrew (B) Romeo and Juliet



(C) As You Like It (D) Two Gentlemen of Verona

A conversation that starts pleasantly quickly turns into a /an (i) _______ argument. With (ii) ________ of Oxen Meadows at the heart of the matter, Lomov and Natalya quarrel and are later joined by Chubukov. Thus, begins a/an (iii) __________ of insults, accusations and name-calling. All (iv) __________ disappears. Eventually, Lomov leaves clutching at his heart, his foot numb.

LOMOV: He’s better!

Champagne! Champagne!

Choose the option that fills the missing words most appropriately.



civility

(C) (i) curious ; (ii) land ; (iii) outpouring ; (iv) laughter



Ans. Option (C) is correct. Q. 2. How would you characterise Chubukov’s mood based on the given extract? (A) Cautious and celebratory

(B) (i) vicious ; (ii) ownership ; (iii) circus ; (iv)







(A) (i) petty ; (ii) history ; (iii) series; (iv) politeness











(D) Lomov’s status everything.





(C) They understand that honour is superficial and overrated.





(B) They were exaggerating in the argument and didn’t mean it.

@PROCBSE 180





III. Read the following extract and answer the questions/complete the sentences that follow: (5×1=5) 



Q. 3. Based on the extract, Lomov and Natalya’s closing lines can best be seen as:

But, please, Stephen Stepanovitch, how can they be yours? Do be a reasonable man. My aunt’s grandmother gave the meadows for the temporary and free use of your grandfather’s peasants. The peasants used the land for forty years and got accustomed to it as if it was their own, when it happened that .............. . [CBSE, 2018 (Modified)]











(D) Grateful, yet hurt Ans. Option (B) is correct.

Ans. Option (D) is correct. 

(C) Merry, yet thoughtful





(B) Relieved and jubilant



Oswaal CBSE Question Bank Chapterwise & Topicwise, ENGLISH LANG. & LIT., Class-X



(A) a humorous and ironic take on typical marriage vows.

Q. 1. Who is the speaker of above lines?





(C) a sign of the prospective instability in their marital bliss.





(B) a reflection of their affections for Guess and Squeezer.

(A) Stephan Stepanovitch

(B) Grandmother (C) Lomov





Ans. Option (A) is correct.

(D) Natalya

Ans. Option (C) is correct. Q 2. Why did his aunt’s grandmother give the



meadows?

(A) For temporary and free use (B) In return for making bricks for her

(C) In charity

In a team, it is important to (i) __________ . Otherwise, you would become (ii) __________ and incur the displeasure of the others. When that happens, you might feel (iii) __________ . However, for the others, not having to carry you along would be a (iv) __________ .





Q. 4. Choose the option that correctly uses the idioms to the fill in the blanks of the paragraph below.

(D) Both (A) and (B) are correct











(D) the writer’s statement that disagreement is part of marriage.

Ans. Option (D) is correct.





(A) (i) pull your weight; (ii) dead weight; (iii) the weight of the world on your shoulders; (iv) weight off the shoulders.

Explanation: The aunt’s grandmother had given the meadows for small time period and without

(B) (i) weight yourself; (ii) the weight of the world on your shoulder; (iii) like dead weight; (iv) weight off their shoulders

any rent or on charity and for they were making





Q. 3. Why did the peasants treat the land as their own? (A) It was their own land.

(B) They had used it for forty years.

(C) They had got accustomed to it as their own.







Q. 4. What light do these lines throw on the speaker’s



logical (reasonable having common sense), rational (normal) and gracious (polite).









(C) Accustomed



Ans. Option (A) is correct.

(D) Free





(C) reconciliation on matters leading to argument.

(B) Temporary



(A) Reasonable



(B) good counsel and advice in a serious matter.





Q. 5. Find the synonym of ‘Sensible’ from the above passage.









Explanation: These lines tell us that Lomov is

(A) apologies in a solemn and heartfelt tone.

(D) strong feelings with a raised voice.

(D) All the above







(C) polite

(B) rational

Ans. Option (D) is correct.



Q. 5. The liberal use of exclamatory marks in the given extract indicate that the characters are expressing:

character? He is_________. (A) logical



Explanation: Pull your weight refers to join hands and get together. Dead weight refers to becoming a burden on team, society, etc. The weight of the world on your shoulders refers to feeling overburdened. Weight off the shoulder refers to feeling relived after removing of some burden.



Ans. Option (A) is correct.

(D) Both (B) and (C) Ans. Option (D) are correct.







(D) (i) lend weight to someone; (ii) heavy weight; (iii) like pulling your weight; (iv) weight off their shoulders





(C) (i) throw weight around; (ii) worth your weight in gold; (iii) weight off your shoulders; (iv) dead weight

bricks for her.

@PROCBSE Q. 3. The speaker is very ________.











Ans. Option (B) is correct. Q. 5. What does the word ‘ball’ mean here? (A) A toy (B) Cricket ball (C) A formal dance party (D) None of the above





(D) Stephan





Q. 3. What is the dispute over?

(A) Over the ownership of oxen meadows

(B) Over their relationship



(C) Over their marriage

VI. Read the following extract and answer the questions/complete the sentences that follow: (5×1=5)

Ans. Option (A) is correct.

These meadows aren’t worth much to me. They only come to five dessiatins and are worth perhaps 300 roubles, but I can’t stand unfairness. Say what you will, I can’t stand unfairness. [CBSE Delhi Set-I, 2016 (Modified)]

Q. 4. Which word describes the speaker the best?

(D) Both (A) and (B)













Q. 1. What was the measurement of the meadows?



Q. 5. What does the word ‘restrain’ mean?



(D) 5 Km





Ans. Option (C) is correct.





(B) three hundred





(A) two hundred











Ans. Option (D) is correct. Q. 4. Identify the speaker and the listener of the above lines.







(D) Natalya to Chubukov

(D) forty



(C) four hundred









(D) Fairness













Ans. Option (A) is correct.

(B) Fairly

(C) Fairway



(A) Unfairness

(D) To wish Natalya her birthday



(C) To propose Natalya Ans. Option (C) is correct.













(B) To attend his engagement

Ans. Option (A) is correct. Q. 5. Find a word in the extract that means ‘not based on what is just or fair'?

Q. 2. Why is ‘you’ in an evening dress?







(C) Natalya/Chubukov (D) Chubukov/Lomov

(A) To attend a party



(B) Lomov/ Natalya





(A) Natalya/ Lomov

Ans. Option (C) is correct.



(C) Chubukov to Lomov





Ans. Option (C) is correct. Q. 3. Natalya’s family had the land for nearly _____ years.

Q. 1. Who is speaking and to whom? (B) Lomov to Natalya

(D) 400 roubles







say you look better ....’’ [CBSE OD Set-I, 2017 (Modified)]

worth

(B) 200 roubles





(C) 300 roubles

meadows





Are you going to a ball or what? Though I must

the



(A) 500 roubles

(5×1=5) ‘‘Why, you’re in evening dress ! Well, I never !



Q. 2. How much are approximately?





V. Read the following extract and answer the questions/complete the sentences that follow:

(A) Natalya to Lomov













Ans. Option (C) is correct.



(C) 5 dessiatins



(D) Limited

(B) 5 meters



(C) Control

(A) 5 acres



(B) Unchecked



(A) Uncontrolled





Ans. Option (D) is correct.







(B) Short tempered

(C) Polite



(A) Possessive













(D) Over their love affair

Ans. Option (C) is correct. Explanation: 'Ball' is generally understood as a toy or as a sport equipment. In this extract, however, ball refers to a formal dance party.



Ans. Option (A) is correct.







(C) Chubukov



(B) Natalya



(D) Not at all









(A) Lomov

(B) Yes

(C) Neutral











Ans. Option (B) is correct. Q. 2. Who is the speaker speaking to?





(A) No











(D) None



(C) Chubukov











Ans. Option (C) is correct. Q. 4. Did the speaker seem happy to the listener?



(B) Natalya

(D) old



(C) possessive

Q. 1. Who is the speaker? (A) Lomov

(B) relaxed





(A) happy





“Please don’t shout! You can shout yourself hoarse in your own house, but here I must ask you to restrain yourself!” [CBSE, 2017 (Modified)]

181





IV. Read the following extract and answer the questions/complete the sentences that follow: (5×1=5)



THE PROPOSAL







@PROCBSE 182

Oswaal CBSE Question Bank Chapterwise & Topicwise, ENGLISH LANG. & LIT., Class-X

SUBJECTIVE TYPE QUESTIONS Short Answer Type Questions Q. 1. Which two issues about himself convinced Lomov of his decision to get married? [CBSE-SQP, 2021-22]

had the most friendly and I might almost say the



most affectionate, regard for each other.” How







(40-50 words & 3 marks each)



would you evaluate Lomov and Chubukov’s [CBSE-QB, 2021]



relationship as neighbours?

Ans. Lomov and Chubukov are neighbors. They have







Ans. Value Points: • He is expected to marry and cannot stay unmarried • He ought to lead a quiet, settled and regular life at his age(‘critical age’/ thirty-five). • He needs a partner as he suffers from palpitations and is always getting upset. Guidance The question needs an answer to the points he thought about himself. Those favouring Natalaya are not relevant here. Content— Award 1 mark for any 2 relevant points Award ½ mark for any one point Expression— 1 mark when both aspects included  Answer organised effectively  usage of words for listing/ order (primarily/ To begin with/ apart from this/ In addition to this/ Also/ secondly etc.) ½ mark when either aspect is missing Deduct ½ mark from the overall score if the error density is high (more than a total of 2 spellings and grammatical errors). [CBSE Marking Scheme, 2021]

a controversy on the issue of ownership of the Oxen meadows. They also have a difference of



opinion on the superiority of the breed of their



dogs, Lomov wants to resolve the issue and



visits Chubukov with a proposal is accepted by Chubukov and both Lomov and Natalya got



Q. 5. Do you think Chubukov is a good father? Justify [CBSE-QB, 2021]



Q. 6. Read the statements given below: Statement 1 - ‘Lomov is looking to marry.’













your opinion based on your reading of the text.











married.



Statement 2 - ‘Lomov is looking to marry Natalya.’





Bring out the difference between the two statements and explain what it tells you about [CBSE-QB, 2021]





Lomov.



Ans. Indeed, Lomov is looking to marry but only Natalya because he considered her as a good home

Ans. Lomov was quite convinced that he ought to lead a quiet, settled and regular life at the critical age of thirty-five. Moreover, he needed a partner as he suffered from palpitations and was always getting upset. As a result, he decided to get married.

maker. Though he was not in love with Natalya, to weak heart and sleep-sickness. He wanted a companion who could look after him. Q. 7. How did Chubukov react when Lomov asked for



Q. 2. To what end does the playwright employ Lomov’s palpitations in the play? [CBSE-QB, 2021]

he wanted to marry her as he was suffering due





the hand of his daughter in marriage?











Ans. was excited; embraced and kissed Lomov and blessed him. [Board Marking Scheme, 2019]



Detailed Answer:

Q. 3. Why do you think Natalya Stepanova asked her father to call Lomov back when she heard that he had come with a proposal?

[CBSE-QB, 2021]

Q. 4. “The Lomovs and the Chubukovs have always



[Outside Delhi, 2019]



Ans. Ivan Lomov is a pretentious, proud, self-serving, argumentative and a hysterical hypochondriac. A wealthy landowner Lomov comes to his neighbour with the intentions of marriage, but he really wishes to expand his own land boundaries and he himself is suffering due to a weak health and sleep-sickness.











Detailed Answer:

These questions are for practice and their solutions are available at the end of the chapter

Chubukov was eager to marry off his daughter, Natalya. So, he was quite happy when Lomov asked for the hand of his daughter in marriage. He kissed Lomov and blessed him.

@PROCBSE

THE PROPOSAL

Long Answer Type Questions



(100-120 words & 6 marks each) invited to speak about nineteenth century upper class Russian society.





Based on your reading of the play, how might you describe it? Support your evaluation with instances from the text. [CBSE-QB, 2021]





As the story ‘The Proposal’, belongs to the Russian characters so through the dialogues of the characters, we come to know a lot about Russian man. According to the story, the Russians are very particular about their dress and have particular dresses for a special occasion. Secondly, they are hypocrites. They show off to be sweet but the very next moment, they start fighting. As in the story, Chubukov call Lomov ‘my angel’ but later fought with him. They pretend to be good neighbours but within their heart, they have so many grades. They love to have dogs as a pet and consider their pets better than the other one. All of them are stubborn, short-tempered, quarrelsome and rigid. Such behaviour and thought is clearly visible through the interactions of its characters in the story.



















Ans. Noble titles and land ownership were the main determinants of precise league in the nineteenth century upper class Russian society. Protective of their wealth and precise league, Russian’s landed aristocracy was arguably the most conservative force.

Students may draw upon the following: • Introductory sentence:  farcical characters, their ridiculous and odd behaviour, unlikely and exaggerated situations (profuse in the play) • Evaluation:  humor in the play, improbable situations, childish behavior of the characters (making a mountain out of a molehill), arguments and quarrels  hurling of accusations and insults without a second thought  resolving the differences and fighting again over another topic  Lomov’s nerve problems and other ailments, Chubukov’s theatrical statements, Natalaya’s impulsive and belligerent remarks and other relevant evidence. • Concluding Statement:  the manner in which the final proposal is made amidst all the chaos, makes the play a farce. [CBSE Marking Scheme, 2021]







Ans. Value Points:









Q. 1. Farce is a kind of comedy which includes situations and dialogues that are ridiculous, exaggerated and even absurd. Evaluate the play, The Proposal, as a farce. [CBSE-SQP, 2021]

Q. 2. Though the play revolves around three people, Russian society emerges as an important



in which the final proposal is made amidst all the chaos, makes the play a farce.

character through the ongoings of the play and the thoughts and interactions of its characters. Imagine yourself to be a historian who has been

Q. 3. Look at the given matrimonial advertisement published in the Morning Post (newspaper) in December 1822.



‘The proposal’ is a humorous short story on the strain between two wealthy neighbours. Ivan Lomov is thirty five year old bachelor who suffers from heart palpitations. He approached for Natalya’s hand in marriage. The situation is hilarious as quarrels for land emerge and the situation worsens. Lomov thinks he is seeking her so she would take care of him. Natalya thinks he is a fool for choosing her and over the course of story, she passes implusive remarks for her hand in marriage. The next moment, they solve their differences and fighting without a second thought. Chubukov’s theatrical statements in between the quarrels give things a satirical twist. The manner







Detailed Answer:



183

It was fairly common practice in the nineteenth century for gentlemen and ladies to seek courtship and matrimony by placing a suitable advertisement. MATRIMONY A Middle-aged Gentleman, of a respectable rank in life, who enjoys an income of Two Hundred Pounds a Year, with fair expectations and who might otherwise. Offer a favourable account of himself, wishes to MARRY. The intimations of any Lady, of unbelievable character, of an age not less than thirty, nor more than thirty-seven and of an income at least equal to his own (as his rank requires such), having; a similar wish and condescending to favour him, will be received with due respect and attended to with strict honour and inevitable secrecy. Letters to be addressed, poet paid, S. H., No. 8, Park-place, Kenningtoit.

@PROCBSE 184

Oswaal CBSE Question Bank Chapterwise & Topicwise, ENGLISH LANG. & LIT., Class-X

Imagine yourself as the playwright of the play. Based on your understanding of the personality traits of Natalya and Lomov, write a dialogue based on an imaginary event, showcasing another argument between them. (CBSE SQP, 2021) 

Lomov backed.











: Yes, Of course! She was hump-

Lomov



: Don’t start another fight. Never mind about me, I’ll be all right in some time but your mother wasn’t able to do any thing about it. 



Natalya : And what do you have to say about your family? It’s so mean of you talking like that. Your head is swirling, you’re seeing stars in front of your eyes and even then you’re talking ill of my family!



Ans. Natalya : Oh my God! Did I hear you right when you called my mother ‘hump backed’?













Q. 5. Among neighbours, we should have cordial relations and not lose our temper. How do Natalya and Lomov lose their temper on trivial issues? [Compt. QP Set-1, 2, 3, 2018] 





Neighbours must have a cordial relationship which Lomov and Natalya do not have. Describe the first fight between them.









Q. 4. Natalya and Lomov argued about the ownership of Oxen Meadows and the superiority of their respective dogs in the play, The Proposal.







(b) Chubukov is a practical man who, like every other father, wants a good match for his daughter. He would shortlist the words ‘handsome’ and ‘wealthy landowner’ for his daughter. ‘Handsome’ because his daughter is ‘not bad looking’ and ‘wealthy landowner’ because he wishes to make his daughters’ future safe and wants her to lead a comfortable and contented life.

: It is the only thing I can expect from such an ill-mannered lady like you. Hugh!

Ans. Value Points: • Lomov and Natalya are next-door neighbours. - Moreover the former came to Natalya to propose her. • The very first meeting had in itself seeds of things to be expected in their married life. - In spite of being very rich landlords, they quarrel over a very tiny piece of land called, “Oxen Meadows” • Lomov said that they were given to the peasants of Natalya’s great grandfather in return for making bricks. So it happened to be considered to be Natalya’s property. • Natalya does not need the explanation. Instead, she offers to make a present of it to Lomov who in turn rejects the suggestion. The quarrel continues endlessly. • The would-be couple were very materialistic. Their mask of hypocrisy fell off sooner than expected. - They did not have qualities which go to make a successful marriage: tolerance, compassion and consideration for the other person’s feelings. Detailed Answer:

he would never mention in his matrimonial advertisement and wouldn’t discuss it with others either.

Lomov











Ans. (a) Lomov was a rich landowner of thirty-five. In his matrimonial advertisement, he would surely mention the fortune he owned and the fact that he is eligible, assertive and a rich bachelor. On the other hand, he suffered from sleep-sickness and a weak heart, which

Natalya : You are getting on my nerves! Lomov. Another word and I will throw you out of the house.



Think of what might make Chubukov shortlist Lomov’s advertisement if he were scanning the matrimonial section of the newspaper. [CBSE-QB, 2021] 

(b)





Based on your reading of the play and an understanding of the characters, what details would Lomov provide (and specifically leave out) in a matrimonial advertisement to find a bride like Natalya?



(a)

Lomov and Natalya were next-door neighbours. One day, Lomov came to Natalya's place to propose her. Natalya, who didn’t know that he had come to propose her, thought that he had come to claim Oxen Meadows as his own. When Chubukov, Natalya’s father, interfered with them and also pleaded that the Oxen Meadows were the properties of Chubukov, the fight aggravated. For a few moments, Lomov’s real purpose was overshadowed by that issue and he continued the bias that meadows belonged to his aunt’s grandmother who gave them to the peasants of Chubukov’s grandfather on the condition that

@PROCBSE

THE PROPOSAL

they would prepare bricks for her. So how it could be considered as Natalya’s property.

agree with Lomov’s explanation. Instead, she offered to make a present of it to Lomov who, in turn, rejected the suggestion. And the quarrel continued endlessly. Such first meeting of Lomov and Natalya had in itself seeds of things (quarrels over petty issues) to be expected in their married life.

Though both Lomov and Chubukov, Natalya’s father, were very rich landlords, they quarrelled over a tiny piece of land, called, ‘Oxen Meadows’. But that was not the end of quarrel. Natalya didn’t



185





Commonly Made Error Most of the students simply mentioned the events and incidents of disputes instead of fulfilling the demand of the question.





Answering Tip Students must practice to analyse an event or character related to the question.

Solutions for Practice Questions (Topic-1) Short Answer Type Questions

Ans.3. Natalya is not so pretty, middle-aged woman. She So the moment she comes to know that Lomov had come to propose for her hand, she cries and shouts and forces her father to bring Lomov back.



Ans.5. Stepan Stepanovitch Chubukov loves his

daughter and plays the role of a good father. He is in search of a good match for his daughter. He is in search selector of words. He calls Lomov my angel, treasure etc. When he comes to know about his proposal. When Chubukov finds Lomov fighting with Natalya, he immediately changes and starts to support his daughter.





REFLECTION Have you ever applied the principle of ‘forgive and forget’ to maintain cordial relationship with your neighbours?

@PROCBSE 186

Oswaal CBSE Question Bank Chapterwise & Topicwise, ENGLISH LANG. & LIT., Class-X

@PROCBSE

CHAPTER

1

DUST OF SNOW – By Robert Frost

Revision Notes



The way a crow Shook down on me The dust of snow From a hemlock tree Has given my heart Dust of Snow A change of mood And saved some part Of a day I had rued. Explanation: The poet is upset and sitting under the hemlock tree. Suddenly a crow, sitting on the tree, shakes the particles of snow from the tree which fall on the poet. The soft and cold touch of snow changes the poet’s mood from sad to happy. He starts feeling soothed and refreshed. In this way, a simple moment proves to be very significant and saves the rest of the day of poet from being wasted and held in regret.











1.



‘Dust of Snow’ is a beautiful short poem, written by Robert Frost. This poem tells that even a simple moment has a large impact and significance. The poet has mentioned crow and hemlock tree in this poem. Crow signifies his depressive and sorrowful mood and hemlock tree is a poisonous tree. Both of these signify that the poet was not in a good mood and so he describes the dark, depressive and bitter side of nature to present his similar mood. In such a sad, depressive mood, the poet was sitting under a hemlock tree. A crow, sitting on the same tree, shook off the dust of snow i.e., small particles of snow that remain on the surface after the snowfall, on the poet. This simple action changed the poet’s mood. He realised that he





Summary



‘Dust of Snow’ by Robert Frost is a brief poem but it conveys the poet’s message coherently. The poem shows how human judgement on certain issues can be misleading. The poet depicts the crow and the hemlock tree as reasons for his changed mood and increased optimism.

had just wasted a part of his day repenting and being lost in sorrow. But the change in his mood made him realise that he should utilize the rest of the day in some useful activity. His sorrow was washed away by the light shower of snow dust. His spirit was revived and he got ready to utilise the rest of the day Scan to know constructively. more about this topic Explanation of the Poem





Introduction

Key Words







1. 2. 3. 4.





Rhyme Scheme : abab Alliteration : Has given my heart. And saved some part.p







Poetic Devices

Significance: Importance Depressive: Causing unhappiness Particles: The smallest fragment Repenting: feeling of regret

@PROCBSE 188

Oswaal CBSE Question Bank Chapterwise & Topicwise, ENGLISH LANG. & LIT., Class-X

OBJECTIVE TYPE QUESTIONS Extract Based Questions















(D) And saved some part / Of a day I had rued. Q. 5. Choose the option showing the reason NOT corresponding with “... a crow/ Shook down on me / The dust of snow”.

(A) The crow’s landing on the branch of the tree. (B) The shivering of the crow, due to the cold. (C) The readjustment of position of the crow on the branch. (D) The cawing of the crow hidden in the foliage. Ans. Option (D) is correct. Explanation: The dust of snow fell upon the poet when the crow makes some movement like lands on the branch of tree, shivers due to cold or readjusts its position on the branch. The mere cawing of crow does not make any movement and will not correspond with “...a crow / Shook down on me / The dust of snow”.



















































These questions are for practice and their solutions are available at the end of the chapter









Q. 4. Synecdoche is a poetic device that uses a part to represent the whole. E.g. That’s a great set of wheels! (Set of wheels has been used for car.)











Q. 3. What fell on the poet? (A) Dust of tree (B) Dust of snow (C) Dust of rain (D) Dust of crow Q. 4. Earlier the poet was in a …….. mood. (A) happy (B) pleasant (C) gloomy (D) good









(D) Others finally rue the one who is dishonest and heartless. Ans. Option (D) is correct. Explanation: The meaning of 'rue' is sorrow or regret. The ones who are dishonest or heartless are not sorrowed upon or regretted. They are rather hated.





Q. 2. Which tree did the crow shake? (A) Banyan (B) Mango (C) Oak (D) Hemlock Ans. Option (D) is correct. Explanation: The closing line of the extract ‘From a Hemlock Tree’ indicates the name of the tree which was shook by the crow.



(C) It wasn’t long before I rued my disobedience and my deceit.







(B) I am sure she rued the day she listened to a fortune-teller.







(A) The film was a disaster and he rued his decision to act in it.

























Q. 1. Who shook down the Hemlock tree? (A) Cuckoo (B) Crow (C) Mynah (D) Parrot Ans. Option (B) is correct. Explanation: The crow shook down the hemlock tree when the poet was passing underneath it.







The way a crow Shook down on me The dust of snow From a hemlock tree













II. Read the following extract and answer the questions / complete the sentences that follow : (5×1=5) 

















Q. 2. Choose the option that lists the possible feelings of the poet prior to the experience shared in the poem. (1) reassured (2) disappointed (3) curious (4) demotivated (5) thankful (6) disheartened (7) impulsive (A) 1,3 & 7 (B) 2, 4 & 6 (C) 5 & 7 (D) 1 & 3 Q. 3. Identify the option that DOES NOT use the word ‘rue’ correctly.































(A) Has given my heart / A change of mood. (B) The way a crow/ Shook down on me. (C) The dust of snow / From a hemlock tree.

Q. 1. Of the many symbols the hemlock tree represents, choose the one that Frost drew upon in all likelihood, for this poem. symbol of (A) longevity. (B) togetherness. (C) healing. (D) protection. Ans. Option (C) is correct. Explanation: Frost had tried to bring his aching heart to peace. In other words, he has used it as a symbol of healing for his aching heart.

Pick an example of synecdoche from the poem.



I. Read the following extract and answer the questions/ complete the sentences that follow : (5×1=5) The way a crow Shook down on me The dust of snow From a hemlock tree Has given my heart A change of mood And saved some part Of a day I had rued. [CBSE-QB, 2021] 





(5 marks each)

@PROCBSE

















Q. 2. Give a rhyming word of 'crow' from the extract. (A) heart (B) rued (C) Hemlock (D) snow Ans. Option (D) is correct. Q. 3. What was saved for the poet? (A) His whole day (B) A few days (C) Some part of the day (D) The whole week Ans. Option (C) is correct. Explanation: The poet had already wasted a part of his day grieving over a moment of sorrow, but after the incident in the poem, he got motivated not to waste the rest of the day. So, some part of the day was saved for the poet.



























189









Q. 5. The poet of the poem is? (A) Walt Whitman (B) John Berryman (C) Robin Klein (D) Robert Frost Ans. Option (D) is correct. Explanation: The poem 'Dust of snow' is written by Robert Frost.

























Q. 4. Which word is the synonym of ‘feel remorse’ or ‘regret’ as used in the stanza? (A) Change (B) Rued (C) Mood (D) Saved Ans. Option (B) is correct. Explanation: ‘Change’ means modify, ‘Rued’ means regret or sorrow, ‘Mood’ is an emotion and ‘Saved’ means preserved.































Has given my heart A change of mood And saved some part Of a day I had rued. Q. 1. What was the effect of the fall of the dust of snow on the poet? (A) It changed his mood (B) Changed his luck (C) Changed his car (D) Changed his job Ans. Option (A) is correct. Explanation: When the dust of snow fell on the shoulder of the poet, it suddenly changed the mood of the poet to full of hope and motivated, not to waste the rest of the day over a petty moment of sorrow.













Q. 5. What is the rhyme scheme of the poem? (A) abab (B) aabb (C) abcabc (D) no rhyme scheme III. Read the following extract and answer the questions/complete the sentences that follow : (5×1=5)





Ans. Option (C) is correct. Explanation: Happy, pleasant and good represents the moods when the person is in a state of joy or enjoying something. Gloomy represents the mood when one is disappointed, feeling lonely and sad which are the feelings of the poet in the beginning of the poem.



DUST OF SNOW









SUBJECTIVE TYPE QUESTIONS Short Answer Type Questions



(40-50 words & 3 marks each) incident proved a blessing in disguise for the poet. [CBSE Marking Scheme, 2018]









Ans. While sitting under a tree, dust of snow falls from a poisonous hemlock tree. A seemingly upsetting

U [Board Term-I, 2016-17, Set F1X1EH5]

Ans. With reference to the poem ‘Dust of Snow’, the underlying message for us in our hectic life is that we should enjoy nature to the utmost and we should have a positive outlook even towards simple acts because they can lead to the learning of greater lessons of enjoying life.

These questions are for practice and their solutions are available at the end of the chapter







A [CBSE QB, 2020-21] different in this poem? Q. 4. In the poem, ‘Dust of Snow’, how does the poet’s mood get changed? R [CBSE Comptt. Set 1, 2, 3, 2018]







Q. 3. The crow and hemlock are usually used as negative references in literature. How is this

Detailed Answer : While sitting under a tree, dust of snow falls on the poet. This seemingly upsetting incident changed the mood of the poet. He realises that he has just wasted a part of his day repenting, rather he could have utilised the same in doing some productive activity. Q. 5. What is the underlying message for us in our hectic life with reference to the poem, ‘Dust of Snow’ ?







Q. 2. The poem evokes a sense of black and white. U [CBSE QB, 2020-21] Justify.





Ans. ‘Dust of snow’ is a beautiful poem written by Robert Frost. It depicts a simple moment, but it is of great significance. The poet is depressed and has lost all hope. He is sitting under a hemlock tree. A small incident changes his sad mood. The poem teaches us that one should never be desperate and hopeless.













Q. 1. ‘Dust of Snow’ is one of Frost’s most loved poems. Elaborate why you think this is so. E [CBSE QB, 2020-21]

@PROCBSE 190

Oswaal CBSE Question Bank Chapterwise & Topicwise, ENGLISH LANG. & LIT., Class-X

Long Answer Type Questions



(100-120 words & 6 marks each)









Ans. Q. 2. A simple moment proves to be very significant and saves the rest of the day of the poet from being wasted. Explain on the basis of the poem Ap ‘Dust of Snow’.





Ans. (i)









(ii) (iii)





(iv)

Nothing is good or bad only our thinking makes it so Best things in life are free Life is full of problems we should solve them rather than sitting and regretting Sometimes unimportant things change the meaning of life









Like a lesson taught by the crow and the hemlock on how to take adverse season in stride and work for the better life. (CBSE Marking Scheme, 2018) Detailed Answer : The poet, Robert Frost tries to convey that even simple and ordinary incidents have great significance in our life. Nothing is good or bad. It is only our thinking that makes it so. Best things in life are free of cost. Life is full of problems and obstacles. We should try to solve them rather than sitting and regretting. Sometimes even the unimportant things change the meaning of life. A simple action of a crow, who shook off the dust of snow on the poet, taught him a great lesson. The poet realised that he had wasted his day in repenting rather than doing some useful activity. He understood how to take adverse season in stride and work for a better life.





Imagine yourself to be a crow and write your opinion about this prejudice. Make a case for A [CBSE QB, 2020-21] breaking stereotypes.

(v)





Q. 1. As a crow you feel highly injured by the incorrect perception humans have about your species.





Commonly Made Error Some students failed to bring in the underlying message of the poem.



Try to read in between the lines and understand the literary meaning of the given poem.

Solutions for Practice Questions Explanation: The first and third lines rhyme (crow and snow) and the second and four lines rhyme (me and tree). So, the rhyme scheme is abab.

Extract Based Questions

I. Ans.2. Option (B) is correct. Explanation: The poet was upset or saddened (disappointed), discouraged (demotivated) and dismayed and dejected (disheartened).



II. Ans.3. Option (B) is correct. Explanation: When the poet was passing under the hemlock tree, a crow made some movement and the powder of the snow indicated as ‘Dust of Snow’, accumulated on the tree and its branches, fell upon the poet. Ans.5. Option (A) is correct.

These questions are for practice and their solutions are available at the end of the chapter





Ans.4. Option (A) is correct. Explanation: The phrase ‘has given my heart’ indicate the change of mood from sorrow or disappointment to love or care.

Short Answer Type Questions Ans.2. The setting of the poem is in contrast of black and white. While the surrounding of the poet is set in white with everything covered in show, a black crow appears in contrast of it. Also the poet's heart is filled with darkness, i.e., thoughts of sadness and disappointment which later changes into that of hope and optimism. Thus, evoking the sense of black and white. Ans.3. The crow and hemlock tree are used as a negative means to demonstrate the mood of the poet. Often the crow is symbol of ill omen and the hemlock tree is a poisonous one. But the poet used them beautifully to portray the fact that inauspicious things too can bring joy and happiness.





Answering Tip

@PROCBSE

DUST OF SNOW



Ans.1. It is really bad to have a wrong perception for anyone be it any bird or any animal. But whenever we differentiable between such species like me. They are differentiated by the qualities like colour, voice or the work allotted to them. But as a crow, I really feel very sad by this partiality. Black is just a colour and even it has its own significance. Only on the basis of colour, one cannot call me symbol of misfortune or bad luck. My black colour is by birth. It is really rare and unique. Even, we

crows are meant to be the most obedient and kind birds. So, I think colour is not the only point to be differentiated. Think Tank ordered the crew to consume the vitamins to enable them to decode the information. Much to the amusement of the readers, he misinterpreted the nursery rhymes, and thought that the earthlings were powerful creatures and were planning an attack on them. He assumed Humpty-Dumpty in the rhyme as himself and fled to Alpha Centauri to save his life.

Long Answer Type Questions

191

Were you able to understand that every cloud has a silver lining based on the poet's experience that sometimes unimportant things change the meaning of life?





REFLECTION

qqq

@PROCBSE

CHAPTER

2

FIRE AND ICE – By Robert Frost

Revision Notes Introduction



Summary ‘Fire and Ice’ is a short poem by Robert Frost. In this poem, the poet refers to two predictions of how the world will end. Some say it will end in fire while others say it will end in ice. According to the poet, ‘fire’ stands for desire, greed, avarice or lust. The more you try to satisfy them, the more they grow. There is no end to it. They spread rapidly like fire and engulf your whole life. One becomes selfish and sometimes cruel also. On the other hand, ‘ice’ according to the poet, stands for hatred, coldness and rigidity. One becomes insensitive and indifferent towards the feelings of others. The poet says that both fire and ice are growing with such a rapid speed that the world will soon perish either way, in fire or in ice.

Poetic Devices Symbolism : Fire symbolises human desires, whereas ice stands for hatred and indifference. Alliteration : Some say the world will end in fire. I hold with those who favour fire. Rhyme Scheme : abaa abab













This short poem outlines the familiar question about the fate of the world, wondering if it is more likely to be destroyed by fire or ice. People are on both sides of the debate.



But if it had to perish twice, I think I know enough of hate To say that for destruction ice Is also great And would suffice. Explanation: In these lines, the poet stops discussing the first theory Scan to know and goes on to talk about the more about second theory – the theory this topic about the destruction of earth by ice. He does not disagree with the first theory about fire. He simply considers what might happen if the earth Fire and Ice were to be destroyed a second time. He is sure that the second time ice will be enough to destroy the earth as nearly and effectively as the fire. He compares ice with hatred and indifference.



2.









Some say the world will end in fire, Some say in ice. From what I’ve tasted of desire I hold with those who favour fire. Explanation: The poet says that there are two notions in the society about the destruction of the world. The first of these theories states that fire will cause the disaster to happen. He equates fire with human passion and desire. The poet frankly confesses that he agrees with those people who believe that the world will be burnt by fire.

Key Words





1. Predictions: A statement of what will happen in future. 2. Avarice: Greed for wealth 3. Perish: To decay and disappear









1.



Explanation of the Poem

@PROCBSE

FIRE AND ICE

193

OBJECTIVE TYPE QUESTIONS Extract Based Questions 

I. Read the following extract and answer the questions/complete the sentences that follow : (5×1=5) Some say the world will end in fire, Some say in ice. From what I’ve tasted of desire I hold with those who favor fire. But if it had to perish twice, I think I know enough of hate To say that for destruction ice Is also great And would suffice. [CBSE-QB, 2021] Q. 1. Choose the CORRECT statement about the given poem.



















II. Read the following extract and answer the questions/ complete the sentences that follow : (5×1=5) Some say the world will end in fire Some say in ice. From what I’ve tasted of desire I hold with those who favour fire. But if it had to perish twice I think I know enough of hate to say that for destruction ice Is also great And would suffice. Q. 1. The people of the world think that the world will end in _______ . (A) fire and tsunami (B) earthquake and cyclone (C) melting of glaciers (D) fire and ice 





















These questions are for practice and their solutions are available at the end of the chapter

















The poet (A) is inclined to believe that the world would most likely end with fire. (B) has heard divided opinions about the way the world would end in all likelihood. (C) preaches love and kindness to combat the spread of hate among all. (D) declares the power of ice to be as destructive as that of fire. Ans. Option (C) is correct. Explanation: The poem is a warning that human negativity will bring an end of this world. The poet has never in the poem advocated or preached that we should overcome this negativity and spread love and kindness to prevent the earth from ending in either ice or fire. 

























Q. 4. The poet uses the phrasal verb -hold with. Choose the option that DOES NOT indicate a valid phrasal verb. [CBSE SQP 2020]















(A) powerful warning (B) heart-felt apology (C) earnest appeal (D) vengeful threat Ans. Option (A) is correct. Explanation: The poet has warned in the poem that the world will end due to escalation of negative human emotions like greed, hate, violence, rage and indifference.

Q. 5. Pick the option that is NOT TRUE about the poet according to the extract.















(1) rage (2) violence (3) indifference (4) hate (5) greed (A) Fire- 3, 4; Ice- 1, 2, 5 (B) Fire - 2, 5; lce -1, 3, 4 (C) Fire - 1, 3, 5; Ice - 2, 4 (D) Fire - 1, 2, 4; Ice - 3, 5 Q. 3. The poem is a________, put across by the poet.









(A) Fire and ice are images — they help the readers visualise the power of nature over man. (B) Fire and ice are symbols — not of natural disasters, but of humanity’s ability to create disasters of its own. (C) Fire and ice are elements — not of Nature but man-made and possess the ability to create havoc for mankind. (D) Fire and ice are agents—they change the thinking of mankind from negative to positive and bring harmony. Ans. Option (B) is correct. Q. 2. Select the option that correctly classifies the connotations for fire and ice, as suggested in the poem.

(A) Option 1 (B) Option 2 (C) Option 3 (D) Option 4 Ans. Option (D) is correct. Explanation: The phrasal verb ‘hold off ’ means fail to occur. The phrasal verb ‘hold back’ means to stop. The phrasal verb ‘hold on’ means to wait or to stop. There is no phrasal verb as ‘hold into’.



































(5 marks each)

@PROCBSE Oswaal CBSE Question Bank Chapterwise & Topicwise, ENGLISH LANG. & LIT., Class-X















































These questions are for practice and their solutions are available at the end of the chapter







Q. 4. What does the poet want to convey through this poem? (A) We should not be greedy. (B) All humans should check their desires and hatred. (C) We should restrain our desires and love fellow beings. (D) Both (A) and (C)

















(A) abcab (B) ababc (C) ababa (D) aabbC Ans. Option (C) is correct. Explanation: The first, third and fifth lines rhyme (twice/ice/suffice) while the second and fourth line rhyme (hate/great).









Q. 3. Here the rhyming scheme used by the poet is __________.





































































Q. 3. What does 'ice' symbolise? (A) Indifference (B) Love (C) Sympathy (D) Passion Ans. Option (A) is correct. Explanation: According to the poet, the 'ice' symbolises indifference which is a cold emotion.

But if it had to perish twice I think I know enough of hate To say that for destruction ice Is also great And would suffice Q. 1. What does ‘It’ here refer to? (A) Fire (B) Ice (C) Earth (D) World Q. 2. The word ‘perish’ means ____________. (A) decay (B) loose one’s life (C) appear (D) improve Ans. Option (A) is correct. Explanation: ‘Loose one’s life’ means death. ‘Appear’ means come into sight. ‘Improve’ means becoming better. ‘Decay’ means slow and gradual decomposition resulting in an end of the thing (perish).













III. Read the following extract and answer the questions/ complete the sentences that follow : (5×1=5) Some say the world will end in fire Some say in ice. From what I’ve tasted of desire I hold with those who favour fire. Q. 1. What can be the cause of world’s destruction as per these lines? (A) Fire (B) Ice (C) Both (A) and (B) (D) None of these Q. 2. According to the poet, fire refers to ____________. (A) violent desires (B) passions (C) jealousy (D) all of these Ans. Option (D) is correct. Explanation: According to the poet, fire represent human emotions that develop burning sensation inside us. Such emotions are violence, passion, jealousy, hatred, etc.















Q. 4. Who is the poet of this poem? (A) R. L. Stevenson (B) Sarojini Naidu (C) William Wordsworth (D) Robert Frost Q. 5. The rhyme scheme of the given line is __________. (A) abab; abab (B) abba; abbc (C) abaa ; abab (D) aabb ; aabb Ans. Option (C) is correct. Explanation: In the first four lines: the first, third and fourth lines rhyme (fire/desire/fire). In the next four lines: the first and third lines rhyme (twice/ ice) and the second and fourth lines rhyme (hate/ great).









Q. 5. What is the rhyme scheme of the extract? (A) abaa (B) aabb (C) abca (D) abba Ans. Option (A) is correct. IV. Read the following extract and answer the questions/complete the sentences that follow : (5×1=5)





















Q. 3. Find the word from the passage which means the same as ‘endless greed’. (A) favour (B) avarice (C) perish (D) destruction Ans. Option (B) is correct. Explanation: ‘Favour’ means support/kindness/ preference. ‘avarice’ means an endless want for material wealth or the other. ‘Perish’ means end. ‘Destruction’ means causing irreparable damage.





















Rest all options represent burning emotions or fire. Q. 4. What message does the poet want to convey through this poem? (A) Everything is transitory (B) Life is unpredictable (C) Death is inevitable (D) Both (A) and (C) Ans. Option (D) is correct. Explanation: Through the poem, the poet has given the message that end or destruction is inevitable. Only the mode of end may be undecided - fire or ice. Also, whether fire or ice - every human emotions is a temporary or a transition phase and will change sooner or later.



Q. 2. The poet’s opinion is that the world will end in _______ . (A) fire (B) ice (C) earthquake (D) tsunami Ans. Option (A) is correct. Explanation: The poet is more inclined towards the view that the earth will end in fire, that is greed and violence.



194

@PROCBSE

















195

Q. 5. The word ‘suffice’ means ____________. (A) sufficient (B) insufficient (C) unhappy (D) courageous Ans. Option (A) is correct.



FIRE AND ICE





Ans. Option (C) is correct. Explanation: The poet warns the humankind to control their desires otherwise the world will be destroyed due to greed, hatred, violence, etc.

SUBJECTIVE TYPE QUESTIONS Short Answer Type Questions

Ans. – –







U [Board Term-I, 2016-17]







Q. 7. What is the message of the poem ‘Fire and Ice’ ? [CBSE QB, 2020-21] Ans. The metaphor of ‘Fire and Ice’ are used very effectively to convey a definite message to the readers. ‘Fire’ stands for our uncontrolled passions which are disastrous for human beings. Similarly cold reasoning devoid of human love will bring insensitivity. This hatred will lead to the end of this world.































Ans. In the poem ‘Fire and Ice’ the poet considers the age-old question of whether the world will end in fire or in ice. This is similar to another age-old whether it will be preferable to freeze to death or burn to death. The either option would achieve its purpose sufficiently may be in storm of volcanic eruptions, forest fires or melting ice caps. Q. 4. Fire and Ice projects a pessimistic outlook. U [CBSE QB, 2020-21] Comment. Ans. According to me, ‘Fire and Ice’ projects a realistic outlook rather that a pessimistic one. The poet brings forth the view that the world will soon end. It can be either by fire or ice. He mentions that he has tasted everything in life after the World War I. So, it would soon end either with greed or indifference. Q. 5. The use of symbolism in the poem ‘Fire and Ice’ conveys the poet’s message effectively. Expand with reference to the poem. [CBSE QB, 2020-21]













Ans. ‘Fire and Ice’ is a symbolic poem by Robert Frost. The poet wants to aware the humanity that everything will end one day as a result a human misdeeds. His aim is aphorism-brevity for Frost truth remains ambiguous. That’s why the tone contrasts with the seriousness of the subject matter. Q. 3. Evaluate the line- Some say the world will end in fire/Some say in ice- in the context of volcanic eruptions, forest fires, meteor collisions, melting U [CBSE QB, 2020-21] ice caps etc.

Poem discusses the way the world can end. Fire stands for greed/lust/human desire; Ice stands for hatred/ coldness/human indifference; – World can end with either greed or indifference [CBSE Marking Scheme, 2020] Detailed Answer : The use of symbolism in the poem ‘Fire and Ice’ conveys the poems, the poet has compared fire to the dark side of humanity i.e., hatredness and lust. According to the poet lust is a greater evil. Desire or lust is the cause of war. Hatredness is also harmful. So, the poem ‘Fire and Ice’ represents the two evils hatredness and lust which lead to the destruction of the world. According to the poet fire stands for desire, greed and avarice. The more you try to satisfy them, the more they grow. There is no end to it. They spread rapidly like fire and ruin your whole life. On the other hand, ‘ice’ according to the poet, stands for hatred, coldness and rigidity. One becomes insensitive and indifferent towards the feelings of others. One becomes selfish and sometimes cruel also. The poet says that both fire and ice are growing with such a rapid speed that the world would soon perish either way, in fire or in ice. Q. 6. What does ‘fire’ and ‘ice’ stand for and what is the general opinion regarding the world ?













Q. 1. “Fire and Ice” was first published in 1920. Briefly explain how the WWI might have influenced the U [CBSE QB, 2020-21] theme of the poem. Q. 2. The tone of the speaker contrasts with the seriousness of the subject matter. Justify. E [CBSE QB, 2020-21]





(40-50 words & 3 marks each)

Long Answer Type Questions













Q. 1. Imagine that Nelson Mandela read ‘Fire and Ice’. As Mandela, write a diary entry associating the impact of the troubled history of South Africa with the idea expressed in the poem. You may begin like this:









(100-120 words & 6 marks each)

These questions are for practice and their solutions are available at the end of the chapter

10 May 1994, Tuesday 10 p.m. Today, when I am the President of South Africa, I can’t help recalling Robert Frost’s ‘Fire and Ice’...... ......................... (continue)........................... [CBSE QB, 2020-21]

@PROCBSE Oswaal CBSE Question Bank Chapterwise & Topicwise, ENGLISH LANG. & LIT., Class-X



















Do not merely read a poem but also try to understand the complete meaning of a poem. Try to understand the poet’s purpose in the poem. Students should learn how the poet’s use of vocabulary or imagery adds to the effect of the poem. Develop a personal response to a poem.

Solutions for Practice Questions and indifference) will be the cause of world’s destruction.

Extract Based Questions



Ans.4. Option (D) is correct. III. Ans.1. Option (C) is correct. Explanation: According to the poet, either fire (violence and desire) or ice (coldness



Short Answer Type Questions

Ans.1. The poem was published shortly after WWI and weighs up the probability of two different scenarios represented by the elements of the poem’s title. The speaker believes fire to the world ender. Ironically, he also adds that icerepresenting hate-would ‘also’ be ‘great’ to bring the end of the world. Ans.6. Fire stands for fury, hatred, anger, cruelty; ice is symbolic of insensitivity, coldness and intolerance-General opinion-some say ice, while others say fire will destroy the world. [CBSE Marking Scheme, 2016] 





II Ans.1. Option (D) is correct. Explanation: According to the poet, the people of the world think that earth will end in either fire (representing violence and hate) or ice (representing rigidity and indifference

IV. Ans.1. Option (D) is correct. Explanation: The poet is talking about the destruction of the world which is inevitable but its cause is presently uncertain - fire or ice.





I. Ans.2. Option (D) is correct. Explanation: In the poem, the ‘fire’ has been used as a symbol to denote aggressive emotions like rage (anger), violence and hate. The ‘ice’ has been used as a symbol to denote pacifying emotions like indifference and greed.









Sometimes students fail to understand the symbolic meaning of Fire and Ice and write the summary of the poem.

Answering Tip



their tyranny. Hope the efforts of my people will change the entire dynamic of Africa. Q. 2. What deep meaning does the poem ‘Fire and Ice’ U carry in it ? Q. 3. What is the rhyme scheme of the poem? How does it help in bringing out the contrasting ideas in the U poem? Ans. The rhyme scheme of the poem in the first stanza is abaa and in the second stanza it is ababa. The lines ending with same rhythm have the same idea but the line that ends with a different note has the contrasting idea. In the first stanza, the first, third and fourth lines end with the words fire, desire, fire - same rhythm but the second line ends with word ‘ice’ a different note, also it contrasts with ‘fire’. Thus, the poet has brought out the contrasting ideas in the poem by using different rhythm.

Commonly Made Error







Ans. 10 May 1994, Tuesday 10 p. m. Today, when I am the president of South Africa, I can’t help recalling Robert Frost’s ‘Fire and Ice’ The poem talks about Fire, symbol of the dark side of humanity i.e., hatredness and lust, and ice stands for coldness and rigidity. Humans have become insensitive and indifferent towards the feelings of others. They have become selfish and sometimes cruel also. The day, I read the poem ‘Fire and Ice’ in one of my events. The reaction of the audience to it was spectacular. Given to the history of South Africa and its recent troubles, the poem actually reflects the sufferings of the black people. The Africans have been treated terribly by the ruling governments. It would take many years to recover from that profound hurt. The racial discrimination unleashed a reign of terror, oppression and brutality on us. We needed freedom from the white people and





196

These questions are for practice and their solutions are available at the end of the chapter

@PROCBSE







FIRE AND ICE

Detailed Answer : Fire stands for fury, desire, lust, anger, avarice, cruelty and greed. Ice is symbolic of hatred, coldness, rigidity, insensitivity and intolerance. The general opinion regarding the world is that the world will end in fire and some say ice. Both the two reasons contrast each other and one equally opposite to each other. People who favour fire believe that it will be the heat and passion which will end the world. On the other hand, some people think that it will be the ice which will freeze the world.

Ans.2. The poet has compared fire to the dark side of humanity i.e., hatredness and lust. According to the poet, lust is a greater evil. Desire or lust is the cause of war. Hatred is also harmful. So, the poem ‘Fire and Ice’ represents the two evils hatred and lust which lead to the destruction of the world. According to the poet, fire stands for desire, greed and avarice. The more you try to satisfy them, the more they grow. There is no end to it. They spread

rapidly like fire and ruin your whole life. On the other hand, ‘ice’ according to the poet, stands for hatred, coldness and rigidity. One becomes insensitive and indifferent towards the feelings of others. One becomes selfish and sometimes cruel also. The poet has used fire and ice as symbols - not of natural disasters, but of humans ability to create disasters, of its own due to their inner evil/feelings. The poet says that both fire and ice are growing with such a rapid speed that the world would soon perish either way, in fire or in ice. Think Tank ordered the crew to consume the vitamins to enable them to decode the information. Much to the amusement of the readers, he misinterpreted the nursery rhymes, and thought that the earthlings were powerful creatures and were planning an attack on them. He assumed Humpty-Dumpty in the rhyme as himself and fled to Alpha Centauri to save his life.

Long Answer Type Questions

197

Were you able to identify the feeling of patriotism solidarity and sensitivity hidden deep within the meaning of the poem 'Fire and Ice'?





REFLECTION

qqq

@PROCBSE

CHAPTER

3

A TIGER IN THE ZOO – By Leslie Norris

Revision Notes body which are quite prominent. Its paws are soft like velvet. No noise is produced when it walks on its ‘pads of velvet’. The tiger is in rage but silent because he finds himself helpless in the locked cell.

Introduction

He should be lurking in shadow, Sliding through long grass Near the water hole Scan to know Where plump deer pass. more about Explanation: The poet this topic suggests that the cage is not the proper place for the tiger. It is not its natural habitat. He should have been sitting under long grass near a stream. As soon as he would A tiger see his prey, like deer coming, in the zoo he would slide silently through the grass and kill them. The poet wants to convey that the tiger should be there in the forest, in its natural habitat and not in a cage.



Key Words







1. Frustrated: disappointed 2. Terrorised: to fill with terror or fear 3. Enclosures: Something enclosed.



He stalks in his vivid stripes The few steps of his cage, On pads of velvet quiet, In his quiet rage. Explanation: The tiger in the zoo walks in the limited space provided to it in a concrete cell in a zoo. It takes a few steps. There are stripes on its





4.











Explanation of the Poem 1.

He should be snarling around houses At the jungle’s edge, Baring his white fangs, his claws, Terrorising the village ! Explanation: The poet then gives another suggestion that the tiger should be sitting at the jungle’s edge in close vicinity of a village. He should be terrorising the people passing that way by its sharp teeth, baring its claws and producing low sound of anger. But he’s locked in a concrete cell, His strength behind bars,





3.



‘This poem contrasts a tiger in the zoo with the tiger in its natural habitat. The poem moves from the zoo to the jungle, and again back to the zoo. In the zoo, he has no freedom. He is kept in a cemented cell behind the bars. He feels angry, frustrated and helpless. This reminds him of his natural habitat, his hiding and sliding in the long grass near the water hole and pouncing upon the fat deer, the way he terrorised the villagers, displaying his sharp teeth and claws. At night in the zoo, he hears the sounds of patrolling cars. The tiger in the zoo appears helpless as a mere showpiece and a source of entertainment to people. The poet wants to convey that it is cruel to keep wild animals in small enclosures of the zoo, away from their natural habitat. They feel angry, helpless and unhappy in the cage. He pays no attention to the visitors who come to watch him. In the silence of the night, he stares at the brilliant stars with his bright eyes.







Summary

2.





The poem gives a sharp contrast of a tiger. The poem provides a contrast in the mood and environment of a tiger, when he is in the zoo and when he is in the forest, when it is in its natural habitat and when it is imprisoned.

@PROCBSE

A TIGER IN THE ZOO



and feeling unpleasant and restless. He hears the sound of the patrolling cars of the zoo authorities. He shows no interest in them. He stares at the shining stars with his brilliant eyes and hopes for his freedom. The poet raises a moral issue here. He intends to present a strong case against such animal cruelty.

Poetic Devices

Imagery : The phrase ‘lurking in shadow’ creates a word picture of some danger hovering for the animal who has come to drink water. Other such word pictures are “sliding through” and “plump deer”. Alliteration: (a) He stalks in his vivid stripes. (b) His strength behind bars.















5.









Stalking the length of his cage, Ignoring visitors. Explanation: The poet now shows deep sympathy at the plight (unpleasant situation) of the tiger that is imprisoned in a cell made of concrete. He cannot come out of the cell because strong bars are fixed. Thus, the poet says that the tiger’s strength is locked behind the bars. He shows no interest in looking at the visitors. Rather he keeps on stalking (in anger) in the limited space of the cage. He hears the last voice at night, The patrolling cars, And stares with his brilliant eyes At the brilliant stars. Explanation: The poet again takes the readers to the cage where the tiger is sitting in the cage

199

OBJECTIVE TYPE QUESTIONS Extract Based Questions 

















































These questions are for practice and their solutions are available at the end of the chapter





















































Q. 3. The contrast indicated in the given extract is between (A) the-tiger's reality and the tiger's ideal situation. (B) the luxury of a zoo and the simplicity of a jungle (C) the tiger's actual feeling and how the tiger should feel (D) the silence of the tiger in a zoo and his dominance in the jungle

Ans. Option (A) is correct. Explanation: The tiger's ideal situation is in jungles where he lives a free life. But the poor creature is kept in a cage in a zoo (captivity). Q. 4. Select the option that describes the tiger in the lines below. Lurking in shadow, Sliding through long grass (1) sly (2) scared (3) dominant (4) light-footed (5) eye-catching (A) (1) and (4) (B) (3) and (5) (C) (1), (2) and (4) (D) (2), (4) and (5) Q. 5. Which of these has the same rhyme scheme as the stanzas in the extract? (A) Tell me not, in mournful numbers, Life is but an empty dream!— For the soul is dead that slumbers, And things are not what they seem. (A Psalm of Life by Henry Wadsworth Longfellow) (B) Hold fast to dreams For if dreams die Life is a broken-winged bird That cannot fly. (Dreams by Langston Hughes) (C) "I cannot go to school today," Said little Peggy Ann McKay. "I have the measles and the mumps, A gash, a rash and purple bumps." (Sick by Shel Silverstein) (D) We listened and looked sideways up! Fear at my heart, as at a cup, My life-blood seemed to sip! The stars were dim, and thick the night, (The Rime of the Ancient Mariner by Samuel Taylor Coleridge)



I. Read the following extract and answer the questions/ complete the sentences that follow : (5×1=5) He stalks in his vivid stripes The few steps of his cage, On pads of velvet quiet, In his quiet rage. He should be lurking in shadow, Sliding through long grass Near the water hole Where plump deer pass. [CBSE-QB, 2021] Q. 1. Which of these activities is mentioned in the extract as something the tiger should do in the jungle? (A) roar at visitors (B) stare at people (C) wait for his prey (D) drink clean water Q. 2. Why does the poet describe the cage as having 'few steps'? (A) to draw attention to the stairs in the cage (B) to show admiration for the size of the cage (C) to convey the importance of protecting tigers (D) to highlight the restriction on the tiger's freedom Ans. Option (D) is correct. Explanation: The tiger has been kept in the cage and his movements are restricted within the cage. 





(5 marks each)

@PROCBSE Oswaal CBSE Question Bank Chapterwise & Topicwise, ENGLISH LANG. & LIT., Class-X





















































































































































(B) Option (2) (D) Option (4)









These questions are for practice and their solutions are available at the end of the chapter

(A) Option (1) (C) Option (3)













Q. 5. The tiger is reacting to his imprisonment in the zoo by: (A) quietly walking in the cage (B) showing his anger openly (C) stalking in the cage (D) ignoring visitors Ans. Option (D) is correct. IV. Read the following extract and answer the questions/ complete the sentences that follow : (5×1=5) But he’s locked in a concrete cell, His strength behind bars, Stalking the length of his cage, Ignoring visitors. He hears the last voice at night, The patrolling cars, And stares with his brilliant eyes At the brilliant stars. [CBSE QB, 2020-21] Q. 1. Choose the image that best describes the condition of the tiger based on the given extract.



















(A) Many tigers chase prey into the water and holds the victim’s head under water until it drowns. (B) Prey feed in the water on water-lilies, and often wander into the middle of the water hole, where they are vulnerable and easy for the tiger to kill. (C) Prey that has quenched its thirst ensures consumption of hydrated meat for the tiger. (D) Chasing the panicked prey from shallow to deep water where the tiger grabs it. Q. 3. Pick the option that DOES NOT use ‘lurking’ correctly to fill in the blank. (A) The thug was ______ in the alley late evening, for unsuspecting passers-by. (B) The hyena was ______ in its den after a good meal. (C) The detective cautioned her team about the ______ dangers likely to impact the case. (D) The prejudices ______ beneath the surface create misunderstandings. Q. 4. ‘shadow’ here, refers to the shadow of (A) the tiger. (B) long grass. (C) water hole. (D) deer Q. 5. Pick the phrase that DOES NOT suggest that the forest in the extract is lush. (A) long grass (B) the water hole (C) plump deer (D) lurking in shadow Ans. Option (D) is correct. III. Read the following extract and answer the questions/ complete the sentences that follow : (5×1=5) He should be snarling around houses At the jungle’s edge, Baring his white fangs, his claws,





















He should be lurking in shadow, Sliding through long grass Near the water hole Where plump deer pass. [CBSE-QB, 2021] Q. 1. According to the extract, the poet wishes for the tiger to be ‘sliding’ through the foliage as this would (A) assist in keeping the prey unsuspecting of the predator’s sound. (B) aid in camouflaging the presence of the predator before it rushes in. (C) help the predator pounce on the prey comfortably without getting tired. (D) Support the predator’s vision as it eyes its prey. Q. 2. Which fact DOES NOT connect with the significance of the water hole for the tiger?











II. Read the following extract and answer the questions/ complete the sentences that follow : (5×1=5)

Terrorizing the village ! But he’s locked in a concrete cell, His strength behind bars, Stalking the length of his cage, Ignoring visitors. [CBSE-QB, 2021] Q. 1. The poet of the poem is (A) Walt Whitman (B) Carolyn Wells (C) Leslie Norris (D) Robin Klein Q. 2. We should protect the tigers as : (A) they are majestic to look at. (B) they attract visitors to the zoo. (C) they are ferocious. (D) they are part of our environment. Q. 3. The tiger terrorizes the villagers as: (A) by killing their cattle. (B) as he does not like to be hunted. (C) as they have cleared his habitat. (D) as he has got tired of being in the forest. Ans. Option (A) is correct. Q. 4. But he's locked in a concrete cell, His strength behind bars, Stalking the length of his cage, Ignoring visitors. is an example of (A) Metaphor (B) Simile (C) Irony (D) Personification Ans. Option (D) is correct. Explanation: The tiger is personified because the poet refers him as 'he'.



Ans. Option (B) is correct. Explanation: The rhyme scheme of the stanza in the extract is abcb. Option B, too, consists the same rhyme scheme.





200

@PROCBSE A TIGER IN THE ZOO







But he’s locked in a concrete cell, His strength behind bars, Stalking the length of his cage, Ignoring visitors. He hears the last voice at night, The patrolling cars, And stares with his brilliant eyes At the brilliant stars [CBSE QB, 2020-21] Q. 1. The fact that the tiger is ‘stalking the length of his cage’ tells us that he is: (A) restless. (B) reckless. (C) resilient. (D) reverent. Ans. Option (A) is correct. Q. 2. What is the rhyme scheme of the given stanzas? (A) abcb; abcb (B) abcb; abcd (C) abcd; abcd (D) abcd; abcb Ans. Option (D) is correct. Explanation: In the first four lines, none of them rhymes with the other, so, they follow the rhyme style abcd. In the next four lines, second and fourth lines rhyme while first and third don’t rhyme, So, they follow the rhyme style abcb. 





























































These questions are for practice and their solutions are available at the end of the chapter









(A) hopes to be free and be in the wild, someday. (B) is looked after well and is nourished and healthy. (C) enjoys staring at the bright stars each night. (D) is well-rested and hence, wide-awake. Ans. Option (A) is correct. Explanation: The tigers stares at the stars that one day he would see them enjoying its freedom in its natural habitat.









Q. 5. The tiger’s ‘brilliant eyes’ reveal that he:





Q. 5. Choose the option listing the most likely reason for the tiger to ignore visitors, according to the extract. (A) He is scared of their constant stares. (B) The visitors don’t provide him with any food. (C) He knows that none would help him out of captivity. (D) The visitors don’t speak to him kindly. Ans. Option (C) is correct. Explanation: The tigers ignore the visitors as they see many visitors coming daily but none of them



















(A) Option 1 (B) Option 2 (C) Option 3 (D) Option 4 Q. 4. The main contrasting idea suggested by the extract is that of (A) strength and weakness. (B) nature and culture. (C) beasts and mortals. (D) confinement and freedom. Ans. Option (D) is correct. Explanation: The tiger has been kept in captivity and his movements are restricted within the cage. So, the contrasting ideas suggested are that of confinement and freedom.









Q. 3. The last voice that tiger hears is of (A) stars (B) visitors (C) patrolling cars (D) None of the above Ans. Option (C) is correct. Q. 4. What is the caged tiger NOT likely to say to the visitors? (A) “Stop staring”. (B) “Set me free”. (C) “Join me”. (D) “Go away”. Ans. Option (C) is correct. Explanation: The caged tiger may threaten its visitors by saying not to stare or asking them to go away. It may also request its visitors to set it free. But, it is most unlikely to invite them to join it inside the cage.



































II. Read the following extract and answer the questions/ complete the sentences that follow : (5×1=5)



Q. 2. Which option correctly lists the reason for the tiger ‘stalking the length of his cage’? (A) Animals tend to cover large distances and burn a lot of their energy by hunting for prey, in their natural habitat. Zoos deprive them of such stimulation and they are restless and bored. (B) Animals are scared of visitors gazing at them in their unnatural surroundings. Zoos are places where animals are far removed from the privacy of their natural habitat. (C) Animals dislike human noises in the city and react to them aggressively. Zoos are often located in cities or outskirts. (D) Animals require human love and care and miss this when in captivity. Zoos are places where they walk around mechanically to attract human attention. Q. 3. Which option identifies a patrolling car correctly?

201

helps them to get rid of the cage and live a life of freedom.



Ans. Option (B) is correct. Explanation: (A) represents that someone has been awarded or given a special honour. (B) represents someone has been put in jail and kept in arrest. (C) represents someone is appearing as witness in a court of law. (D) represents a baby in its crib and toys scattered outside.







@PROCBSE 202

Oswaal CBSE Question Bank Chapterwise & Topicwise, ENGLISH LANG. & LIT., Class-X

SUBJECTIVE TYPE QUESTIONS Short Answer Type Questions Q. 1. How does the poem point to the cruelty of animals U [CBSE QB, 2020-21] in captivity? Q. 2. Do you think the tiger in the poem had lost its natural instinct due to captivity? Support your response with evidence from the poem. 













(40-50 words & 3 marks each)







C [CBSE 2015] village? Ans. The tiger makes his presence felt in the village by snarling around the houses which are at the edge of the forest. He tries to terrorise people by showing his white fangs and claws.









Ans. That’s just one sided. Zoos are beneficial too. Firstly, they save species from extinction and other dangers by providing natural environment to live in. Secondly, it includes the conservation education and research programs that are designed to preserve and protect populations of wild animals as well as educate the public about the threats that they face. Q. 5. How does the tiger make his presence felt in the















[CBSE QB, 2020-21] Ans. The caged tiger walks in a proud manner. He walks quietly. But his eyes show that he is very angry at having been imprisoned in a cage. All his activities prove that the tiger in the poem had not lost its natural instinct due to captivity. He is still chivalric, confident and graceful. Q. 3. What do you think the tiger would say to you, as a visitor? You may begin like this: Please stop staring. You have no idea........................... C [CBSE QB, 2020-21] (continue)....................... Q. 4. Write the appropriate response comprising of at least two reasons to present a counter to the C following :

Long Answer Type Questions













9:00 P.M.

I feel so vulnerable and annoyed at my state. I don’t know if I can ever be a free denizens of the forest. As the most fearless and ferocious animal of the jungle, I really feel horrible and frustrated to be inside the cage of the metal bars. I feel more protective and secured to live in my domain where I can run, chase and prey on other animals and collect food for my family. Living inside this cage seems to be like hell, where I am void of freedom and happiness. I feel like moving back to my heaven. It is my real green world where I can perform my regular’s activities without any fear of being trapped.

These questions are for practice and their solutions are available at the end of the chapter



Ap [CBSE QB, 2020-21]

Ans. 15 October, 2020, Thursday





15 October 2020, Thursday 9:00 pm I feel so vulnerable and annoyed at my state. I don’t know if I can ever be a free denizen of the forest.





You may begin like this:

Q. 2. Imagine that Man has been accused by the tiger of cruelty in an animal court. What would be the tiger’s two major accusations and how would Man defend them? Present your answer in two [CBSE QB, 2020-21] paragraphs. Q. 3. The tiger in the poem ‘A Tiger in the Zoo’ presents a contrasting image with the tiger in the poem ‘How to Tell Wild Animals’. Compare and contrast the two tigers with reference to the manner in which they have been presented in both poems. [CBSE QB, 2020-21] Ans. The tiger in ‘A Tiger in the zoo’ is a pathetic animal. He is locked in a concrete cell in the zoo. The mighty and ferocious animal is put behind the bars. There he stalks in ‘quite rage’ the length of his cage. He becomes just a piece of entertainment for visitor. But he ignores them. At night from behind the bars, he keeps staring at the brilliant stars which only intensifies his loss of freedom. On the contrary, the tiger on the poem, ‘How to tell wild animals’ is a very grand and impressive animal. His hide is yellowish. There are black strips all over his body. He is very agile. The moment he noticed someone, he will simply eat away that creature at once. He is free, confident, graceful and ferocious. He is ready





Q. 1. Imagine the tiger writes a diary entry conveying how he feels helpless, angry and frustrated in the concrete cell. He writes about his desire to be free. Write that diary entry.







(100-120 words & 5 marks each)

@PROCBSE

A TIGER IN THE ZOO

in a zoo. Animals, particularly the animals of the wilds, feel free only in their natural habitats. Locking them in cages will be against natural justice. The tiger roams around in the jungle hunting its prey at will. He rarely kills them for sport. He knows how to ambush his prey. He also knows where he can find his favourite plump deer. The same tiger feels depressed and low in spirits when he is put behind the bars. He stalks constantly the length of his cage in rage. He sadly keeps staring at the stars in the open sky. This reminds him of his loss of freedom and intensifies his grief.













to pounce upon its prey. He shows no mercy and jumps at its prey at once. Thus, the former tiger is the example of confinement whereas the latter suggests the contrasting idea of freedom. Q. 4. Freedom is such an essential virtue that is valued not only by human beings but also by animals alike. Justify the statement with reference to the poem ‘A Tiger in the Zoo’. Ans. Freedom is an essential virtue valued by all. Not only humans but even the denizens of the forest value it. No one knows it better than a caged tiger

203









Commonly Made Errors Students generally don’t pay much attention to the poem and the deep message that it wants to convey. Many students failed to draw the contrast between the caged tiger and the free tiger in the forest.



Understanding the poem and its deep message is a must. Students must focus on the thorough reading and understanding of the poem.

Solutions for Practice Questions Explanation: The tigers are an important chain of food cycle.

Extract Based Questions



Ans.3. Option (B) is correct. Explanation: ‘Lurking’ means Hiding and waiting for the prey to come. After having the meal, the hyena will relax and not lurk in the den. The thug, the dangers and the prejudices may lurk for the prey.

Ans.4. Option (B) is correct. Explanation: The tiger waits for its prey hiding in the shadow of the long grass around it.



III. Ans.1. Option (C) is correct. Ans.2. Option (D) is correct.



Ans.3. Option (B) is correct. Explanation: (1) represents a tow vehicle, (2) represents police or patrolling car, (3) represents race car and (4) represents an ordinary car.

Ans.2. Option (C) is correct. Explanation: Water hole for the tiger attracts the unsuspecting preys towards it and makes it easy for the tiger to drown its prey and kill it. Drinking water by the prey does not turn its meat hydrated for the tiger.

Short Answer Type Questions Ans.1. Through the poem, the poet wants to convey that it is cruel to keep wild animals in small enclosures of the zoo, away from their natural habitat. They feel anger, helpless and unhappy away from their life and environment in the forest. Ans.3. Please stop staring. You have no idea how uncomfortable and restless your stare makes me. I’m trying to ignore each one of you as I consider you humans to be devoid of feelings. None had ever tried to help me out to make me free from this cage. My freedom has been curtailed by you humans. I’m forced to live in a cage which is so small. I’m just fed up of this situation and visitors like you.

These questions are for practice and their solutions are available at the end of the chapter







Ans.4. Option (A) is correct. II. Ans.1. Option (A) is correct. Explanation: The sliding action will produce minimum sound and distraction and will thus keep the prey unprepared for attack.

IV. Ans.2. Option (A) is correct. Explanation: Wild animals-predators - are in habit of roaming all day around in the jungle, hiding, running behind the prey and then savour their meal. In the zoo, this is not possible and the predators feel bored and restless. They burn their energy by taking long strides within their cages.





I. Ans.1. Option (C) is correct. Explanation: It is a well known fact that a tiger is a beast of prey.





Answering Tip

@PROCBSE 204

Oswaal CBSE Question Bank Chapterwise & Topicwise, ENGLISH LANG. & LIT., Class-X

Long Answer Type Questions









Ans.2. The tiger ’s two major accusations would be as follows: The tiger is a denizen of the forest. He loves to roam around freely in his natural habitat-the jungle. There, he is totally free with no restriction on his movements and activities but the humans make them captives and put them behind the cages. Not only this, the humans also destroy

their natural habitat to satisfy their greed. Apart from this, the humans are also indulged in illegal killing of the tigers. Man’s Defense: Keeping the tigers in the confined wall, save them from illegal poaching. At least this way, they all will not be killed and become extinct. Tigers hunt humans for food and we humans kill them to make money. It is part of our life cycle.

Were you able to understand the importance of freedom of living beings based on the poem 'A tiger in the Zoo' ?





REFLECTION



qqq

@PROCBSE

CHAPTER

4

HOW TO TELL WILD ANIMALS – By Carolyn Wells

Revision Notes forests you see a large and terrible animal with dark yellowish skin, you should be sure that he is the Asiatic lion. And if he roars loudly and the roar terrifies you to death, then you are hearing the roar of an Asiatic lion.

Introduction

Or if some time when roaming round A noble wild beast greets you, With black stripes on a yellow ground Just notice if he eats you. This simple rule may help you learn Scan to know The Bengal Tiger to discern. more about Explanation: If you are this topic roaming around in the forest







2.



and you see a noble beast, you must understand that you are face to face with the Bengal tiger. This noble beast has black stripes over his How To Tell Wild Animals yellowish hide. He is so quick that the moment you notice him, he will eat you at once. This is the simple rule of distinguishing the Bengal Tiger.

Key Words 1. Tawny: of an orange brown or yellowish-brown colour. 2. Victim: One that is harmed or injured.



‘How To Tell Animals’, is a beautiful poem by Carolyn Wells about the strange habits and behaviour of some wild animals. The poetess says that the Asian Lion is a large tawny beast with a fearful roar. A tiger has black stripes on his yellow skin and is always ready to eat his victim. A Leopard has black spots on his skin. He leaps on his prey and eats it up. A bear can come to human colonies. He hugs up against a person and can put to death. A crocodile always sheds tears while eating its victim. A hyena will always look smiling. A chameleon always changes its colours according to its surrounding. This is the strange world of some of the wild animals.



Summary







This humorous poem suggests some dangerous ways to identify wild animals. The poetess gives a beautiful description of the Asian Lion and the Bengal Tiger. Then she points out about the Leopard and the Bear. She describes the Hyena for its smiling face and the crocodiles for its tears. Then the poetess describes how the chameleon changes its colour.

Explanation: If you go, by chance, to the jungle of the countries lying in the east, you will see a grand and majestic wild beast. If in those thick













3.



If ever you should go by chance To jungles in the east; And if there should to you advance A large and tawny beast. If he roars at you as you’re dyin’, You’ll know it is the Asian Lion.















1.



Explanation of the Poem

If strolling forth, a beast you view, Whose hide with spots is peppered, As soon as he has leapt on you, You’ll know it is the Leopard. Twill do no good to roar with pain, He’ll only lep and lep again Explanation: If you are walking leisurely in the forest and view a beast, it might be a leopard. His

@PROCBSE Oswaal CBSE Question Bank Chapterwise & Topicwise, ENGLISH LANG. & LIT., Class-X







Explanation: If you are walking around your yard, you may meet a creature there. The moment he sees you, he hugs you very hard. If he does it so, you must be sure that you are hugged by a bear. If you have any doubt in your mind, then he will just give you another tight hug. This hug may seem friendly but it can be fatal.

Alliteration :

roaming round lep and lep again





Rhyme Scheme : ababcc

who hugs you very very hard





Poetic Devices

A novice might nonplus.





Though to distinguish beasts of prey A novice might nonplus, The Crocodile you always may Tell from the Hyenas thus: Hyenas come with merry smiles; But if they weep they’re Crocodiles.



5.

The true Chameleon is small, A lizard sort of thing; He has not any ears at all, And not a single wing. If there is nothing on the tree, Tis the Chameleon you see. Explanation: A true chameleon is a small animal. It looks like a lizard. It has no ears at all. It doesn’t have a single wing either. If you see only this creature on the tree and not any other thing, you must be sure that you are seeing a chameleon.





6.





If when you’re walking round your yard You meet a creature there, Who hugs you very, very Hard Be sure it is a Bear. If you have any doubt, I guess He'll give you just one more caress.



4.

Explanation: It is not an easy job for an inexperienced person to distinguish beasts of prey. He will be confused in distinguishing them. He may not be able to differentiate between a hyena and a crocodile. Hyenas are very deceptive. They will attack their victims with weird smiles and kill them. Crocodiles pretend to be weeping before attacking or killing their prey.



hide is covered with dark spots. The moment he sees his prey, he pounces upon it. His attack is sudden. There is no escape from him even if you cry with pain. He will continue jumping at you and start eating you before you understand that he is a leopard.



206

OBJECTIVE TYPE QUESTIONS Extract Based Questions

I. Read the following extract and answer the questions/ complete the sentences that follow : (5×1=5) If strolling forth, a beast you view, Whose hide with spots is peppered, As soon as he has lept on you, You’ll know it is the Leopard. ’Twill do no good to roar with pain, He’ll only lep and lep again. [CBSE-QB, 2021] Q. 1. Choose the option listing the stanza that would follow the given extract.

(3)

His claws and paws will make you forget, If you're in a dream or reality. And you'll feel clawed and pawed, I bet, The peppered pounce a certainty. Tis a beast that runs and strikes fast, If you're caught you sure won't last.

(4)

The leopard and the lion chose to become friends, For they were all proud of claws on their paws They each glorified one another for their mighty. Ability to live on meat of other fauna throughout a year, They each admired one another for running speed, They each remained firm and loyal to one rule, Lions don't eat leopards neither leopards eat lions.



























(5 marks each)

(1)

(2)

The leopad stalks on silent paws. With deadly armor of fangs and claws. He lies in his tree perch through out the day. Sleeping and resting the hours away. A spotted leopard in a banyan tree, Kept his yellow carnivore-ic eyes on me. I knew better than to move fast or try to flee. His eyes narrowed, but I refused to see. How fast that cat was chasing after me. If I can just get to that tall tall....

@PROCBSE

HOW TO TELL WILD ANIMALS

(B) Samiksha has been teaching for last ten years. (C) Srishti went for her first French class yesterday. (D) Gautam baked a second cake to improve his skills. Ans. Option (B) is correct. Q. 2. Which option lists the image that DOES NOT indicate what the poet means by ‘beasts of prey’?





















































































These questions are for practice and their solutions are available at the end of the chapter





















(A) Lakshman has played cricket for the first time today.

















(1) (2) (3) (4) proud wild wild loud loud jungle proud jungle child child child wild wild mingle loud child jungle loud jungle mingle mingle proud mingle proud (A) Option 1 (B) Option 2 (C) Option 3 (D) Option 4 II. Read the following extract and answer the questions/ complete the sentences that follow : (5×1=5) Though to distinguish beasts of prey A novice might nonplus, The Crocodile you always may Tell from the Hyena thus: Hyenas come with merry smiles; But if they weep they’re Crocodiles. [CBSE-QB, 2021] Q. 1. Choose the option that DOES NOT describe a ‘novice’.



















































(A) Option 1 (B) Option 2 (C) Option 3 (D) Option 4 Ans. Option (D) is correct. Q. 3. What, according to the extract, would cause bewilderment? (A) Discovering the similarity between different preys of beasts. (B) Analysing habits of beasts that prey on hyenas. (C) Knowing the difference between several beasts of prey. (D) Drawing the similarities between crocodiles and hyenas. Q. 4. Choose the line from the given stanza that the poet takes liberty with, to fit to the rhyme scheme. (A) Though to distinguish beasts of prey (B) A novice might nonplus (C) The Crocodile you always may (D) Hyenas come with merry smiles Ans. Option (B) is correct. Q. 5. Choose the crocodile fact that is related to the given extract. (A) They have webbed feet which, though not used to propel them through the water, allow them to make fast turns and sudden moves in the water or initiate swimming. (B) Absence of sweat glands and so, release heat through their mouths making them often sleep with their mouths open. (C) 99% of crocodiles are eaten in the first year of their life by large fish, hyenas, monitor lizards and larger crocodiles. (D) While eating, they swallow too much air, which gets in touch with lachrymal glands and causes them to weep. III. Read the following extract and answer the questions/ complete the sentences that follow : (5×1=5) Or if some time when roaming round A noble wild beast greets you With black stripes on a yellow ground Just notice if he eats you. This simple rule may help you learn The Bengal Tiger to discern. [CBSE SQP, 2020-21] Q. 1. Who is the noble beast? (A) Lion (B) Tiger (C) Leopard (D) Cheetah































(A) Option 1 (B) Option 2 (C) Option 3 (D) Option 4 Ans. Option (C) is correct. Q. 2. Given below are four examples of activities that Jasmeet does. Choose the option that correctly demonstrates ‘strolling’. (A) Jasmeet runs with a great speed after being chased by a dog. (B) Jasmeet walks in the garden, relaxing while listening to his favourite song. (C) Jasmeet skids sharply on the icy skate rink. (D) Jasmeet rushes to switch off the water pump in the backyard. Ans. Option (B) is correct. Explanation: The word 'strolling' means 'an idle and leisurely wall. And this is what Jasmeet is dring in Option (B). Q. 3. Which option lists the statement that is NOT TRUE according to the extract? (A) The poetess asks the reader to hide on seeing the leopard. (B) The poetess cautions the reader about a leopard when walking through its territory. (C) The poetess informs the reader that a leopard can launch repeated attacks. (D) The poetess tells the reader that a leopard attack can result in pain. Ans. Option (A) is correct. Q. 4. The repetition used in “he’ll only lep and lep again” is an example of (A) poetic justice. (B) satire (C) allusion. (D) poetic licence. Q. 5. Choose the option that matches with the rhyme scheme of the extract.

207

@PROCBSE































































Q. 5. Pick out the antonym of 'expert' from the above lines. (A) nomplus (B) novice (C) beast (D) prey



























Q. 4. What is the rhyme scheme of this stanza? (A) ababcc (B) aaabcc (C) ababbc (B) aabbcc



















Q. 3. Who weeps according to the poetess? (A) crocodile (B) alligator (C) tiger (D) lion Ans. Option (A) is correct.































Q. 2. Hyenas and Crocodiles are ________ in nature. (A) somewhat same (B) somewhat different (C) some (D) different





















Q. 1. How can a hyena be recognized by? (A) its colour (B) its appearance (C) its size (D) its smile Ans. Option (D) is correct.



















































V. Read the following extract and answer the questions/ complete the sentences that follow : (5×1=5) Though to distinguish beasts of prey A novice might nonplus, The Crocodiles you always may Tell from the Hyenas thus: Hyenas come with merry smiles: But if they weep they're Crocodiles. [CBSE QB, 2020-21]



















Q. 5. The synonym of 'authentic in the above extract is_________ . (A) single (B) sort (C) true (D) nothing Ans. Option (C) is correct.





















Q. 4. Which other reptile is spoken about? (A) Lizard (B) Snake (C) Frog (D) Crocodile Ans. Option (A) is correct.





Ans. Option (B) is correct. Q. 2. The tiger has black stripes on which ground? (A) Yellow (B) Golden (C) Brown (D) White Ans. Option (A) is correct. Q. 3. How will the simple rule help you? (A) To understand you are in a forest. (B) To understand a wild animal. (C) To understand you are seeing the Bengal Tiger. (D) To understand where you are. Ans. Option (C) is correct. Q. 4. Which word means the same as 'assist'? (A) noble (B) grass (C) help (D) discern Ans. Option (C) is correct. Q. 5. What is the rhyming scheme of the above stanza? (A) abbacc (B) ababcc (C) aabbcc (B) abcabc IV. Read the following extract and answer the questions/ complete the sentences that follow : (5×1=5) The true Chameleon is small, A lizard sort of thing He has not any ears at all. And not a single wing. If there is nothing on the tree. This the Chameleon you see. [CBSE QB, 2020-21] Q. 1. How does a chameleon look like? (A) Frog (B) Lizard (C) Mongoose (D) Snake Q. 2. Where is a Chameleon generally found? (A) Water (B) Marshes (C) Tree (D) Desert. Ans. Option (C) is correct. Q. 3. He does not have any____at all. (A) Wings (B) nose (C) ears (D) Both (A) and (C)



Oswaal CBSE Question Bank Chapterwise & Topicwise, ENGLISH LANG. & LIT., Class-X







208

SUBJECTIVE TYPE QUESTIONS Short Answer Type Questions











A [CBSE QB, 2020-21]

Ans. According to me, the poem belongs to ‘Light Verse’ genre which is a poetry that is mostly for fun. The poem ‘How to tell Wild Animals’ is a humorous poem. The poet suggests some dangerous ways to identify wild animals that create humour.

These questions are for practice and their solutions are available at the end of the chapter























Q. 2. Why does the poetess use the term ‘noble’ and ‘wild’ for the tiger? (How to tell wild Animals.). A [CBSE OD Set-I, 2020] Q. 3. Which genre is most appropriate for the poem? Substantiate your choice with reference to the poem “How to Tell Wild Animals”.





Q. 1. Hyperbole is a literary device used when the poet exaggerates an image to make it comical. State two instances from the poem “How to Tell Wild Animals” where the literary device is used. U [CBSE QB, 2020-21] Ans. The two instances of hyperbole are as follows: (i) The Crocodiles you always may tell from the Hyenas thus; Hyenas come with merry smiles; But if they weep they are crocodile. Practically, we have never seen a hyena smile nor a crocodile weep. (ii) ‘If there is nothing on the tree, this the chameleon you see’. A chameleon may be camouflaged but not invisible.







(40-50 words & 3 marks each)

@PROCBSE HOW TO TELL WILD ANIMALS

Q. 4. “All knowledge is useful. But not all knowledge is











Ans. It is the tendency of a bear to kill his victim by embracing him. He hugs the victim so tight that he usually dies. But a novice who does not know the way of bears may misunderstand his hug. He may think that it is a loving embrace. Q. 8. What is the theme of the poem-‘How to tell wild U Animals’? Ans. The theme of the poem is to create humor. The poet creates humour by suggesting dangerous ways of identifying wild animals. You can identify most of the beast while attacking you. Instead of shouting for help or trying to protect yourself. You are busy identifying the attacker- the idea creates humour.







Q. 5. Would you agree that the poetess has an in-depth knowledge of the wild? Support the statement in context of the poem “How to Tell Wild Animals”. 

U [CBSE QB, 2020-21] Ans. Yes, I do agree that the poetess has an in-depth knowledge of the wild. She has described the various wild animals. These animals are very dangerous and she has introduced them one by







Q. 7. Why does the poetess say that a bear’s ‘hug’ may A confuse a novice?



Ans. The above quote is very true in the case of the poem ‘How to Tell wild Animals’. Actually the knowledge provided through this poem is not worth the cost. But it definitely creates humour. And humour is the fuel of life. Thus the poetess successful use of humour helps liven our minds and brings a smile on our faces.







Elaborate on the quote in the context of the poem “How to Tell Wild Animals”.



A [CBSE QB, 2020-21]



worth the cost.”

Long Answer Type Questions

(100-120 words & 6 marks each)







Dear Ms. Wells

I just read your poem “How to Tell Wild Animals” and enjoyed it thoroughly! (continue)



Yours sincerely







Q. 4. Every animals is unique and has some special characteristic. What tricks does the poet adopt in U distinguishing various wild animals? 





























18 JulyXXXX

Ans. Dear Ms. Wells I just read your poem “How to Tell Wild Animals” and enjoyed it thoroughly! The humorous poem wonderfully tells us how a person can recognize animals. The methods suggested of recognizing wild animals i.e. watching them how they eat us is quite funny. I really; didn’t like the mention of hyenas and crocodiles as it was quite rare and unfamiliar. The description of these animals gives a chill in ones spinal code. But the description of rest of the animals was quite hilarious. I, hereby, want you to compose one such poem on elephants. I’m sure children will definitely like it as they are very fond of elephants. They also fascinate other animal groups due to their humongous size. Also, I request you describe its virtue, how it is helpful to us. Your Sincerely XYZ Q. 3. The poet has successfully used humour to be able to ‘Tell’ or identify ‘Wild Animals’. How do you A think we need lots of it in our daily life?











XYZ USA





















Q. 1. Imagine the poet meets Mijbil, the otter. Write a detailed account of the characteristics the poet could use for her poem, if she were to write about A telling an otter. Q. 2. Write a letter to the poet detailing your favourite aspects of the poem and the ones that you did not like. Include a request about which animal you’d like her to exclusively compose a poem on. Give C reason/s for your choice.

209

one and in great details. She treats even ferocious animals and beasts with a coat of gentle but weird humour. U Q. 6. What is the brief summary of the poem?











Commonly Made Error Most of the students got confused in identifying the characteristics of the Bengal Tiger and Leopard and Crocodile and Chameleon.

These questions are for practice and their solutions are available at the end of the chapter

@PROCBSE 210

Oswaal CBSE Question Bank Chapterwise & Topicwise, ENGLISH LANG. & LIT., Class-X





Answering Tip Students are advised to read the chapter thoroughly for better performance.

Solutions for Practice Questions Extract Based Questions





Ans.2. The poetess refers to the Bengal Tiger as ‘noble’ and ‘wild’ both. The tigers look impressive when they walk majestically. But the same tigers become wild and would brutally kill anyone for food. Ans.6. The poetess tries to distinguish different animals in a humorous manner. She is also educating the readers by describing the various features of wild animals. She is introducing all the animals of East one by one in a very practical way.

Long Answer Type Questions Ans.1. If you ever happen to visit the West Highlands of Scotland for holidaying and you are restless to keep a pet. Then, instead of a dog another is advisable. Because it won’t let you rest at all. An otter is a wonderful creature which resemble a very small dragon. It is coated with pointed scales. Between the scales is visible a soft velvet fur. Only to see its beautiful scaled coat and velvety fur you have to remove the mud completely by bathing it nearly for a month. Only then the otter could be seen in its true colours. And while bathing it you have to keep your nose covered because





Short Answer Type Questions



it may stink badly. This chocolate brown mole like cremate will never even remind you of a chocolate. But it will definitely help you to extend and spread every drop of water and last but not the least let the people around you not recognize what it actually is and give wild guesses about him. Ans.3. Humour is the fuel of life. Without it, it would become difficult to continue living. Today, a life of the common man is typically mired in stress, tension problems and sadness. It is amazing how the smallest problems may take on epic proportions for a common person. In such a situation, it becomes necessary to have a little humour in life. It gives us the much-needed respite from our monotonous routine and helps us face life more positively. Thus, the poetess successfully use humour to liven our minds and bring a smile on our faces. Ans.4. No, doubt every animals is unique. Every wild animal has his own special trait, colour, size and characteristic. The Asian lion is found in the jungles of the eastern regions. It is a huge and mighty creature with brownish hide. His roar is enough to terrorise a person to death. The Bengal tiger is a noble and impressive wild animal. He has yellowish hide and black stripes all over it. The leopard has dark spots all over his body. He is very agile. The moment he sees its prey, he pounces upon it without showing any mercy. The bear is known by his strong and tight hug. It becomes rather difficult for a novice to distinguish among wild animals. However, hyenas and crocodile can be easily recognized. Hyenas come smiling merrily while crocodiles appears to be weeping. The Chameleon is a small creature like a lizard. He has no ears and doesn’t have even a single wing. You can find him sitting on a tree.









I. Ans.4. Option (D) is correct. Ans.5. Option (C) is correct. II. Ans.3. Option (C) is correct. Ans.5. Option (D) is correct. III. Ans.5. Option (B) is correct. IV. Ans.1. Option (B) is correct. Ans.3. Option (D) is correct. V. Ans.2. Option (D) is correct. Ans.5. Option (B) is correct.

Were you able to identify the wild animals on the basis of the distinguished features they have ?





REFLECTION

qqq

@PROCBSE

CHAPTER

5

THE BALL POEM – By John Berryman

Revision Notes









An ultimate shaking grief fixes the boy As he stands rigid, trembling, staring down All his young days into the harbour where His ball went. I would not intrude on him; A dime, another ball, is worthless. Explanation: The boy is very Scan to know much troubled at the loss of more about his ball and plunges into grief. this topic He stands stiff and trembling while staring at his ball. He is upset as he looks into the gloomy water because it has been with him for a long time. When the ball bounces into the water, all his memories of The Ball Poem the childhood days flash in front of him. Moreover the poet doesn’t offer him money to buy another ball because that would be worthless. Now he senses first responsibility In a world of possessions. People will take balls will be lost always, little boy. And no one buys a ball back. Money is external. Explanation: The boy cannot find his ball in the gloomy water. This is when he gets his first sense of responsibility. The poet suggests that from the loss of the ball, the boy is learning what it means to lose something in the world of possessions, where he will lose things, he will buy some more to replace the ones lost, but would never be able to buy back the thing that he had lost. The poet, thus, makes the boy understand about his responsibility as the loss is immaterial. Money is external as it cannot buy memories, nor can it replace the things that we love, the things that really matter.





This poem is written in blank verse. This poem is about losing something that you love, and learning to grow up. It is about a little boy, who, for the first time in his young life, is learning what it is like to experience grief at the loss of a much beloved possession – his ball.

2.



Introduction

Summary









3.







Once a boy was playing with his ball. It was bouncing in the street up and down. The boy was happily enjoying the game. While bouncing, suddenly it fell into the water and was lost. Without the ball, the boy became full of grief. The poet said and that there were other balls. He could purchase another since it was not a great loss. There was no need to worry. But, the boy had that ball for a long time so he was deeply grieved due to his attachment with it. It was linked to his memories. The poet thinks that it is of no use to purchase another ball. He must feel his responsibility of the loss.

Key Words 1. Grief: Emotional pain or suffering linked : connected. 2. Linked: connected

What is the boy now, who has lost his ball, What, what is he to do ? I saw it go Merrily bouncing, down the street, and then Merrily over — there it is in the water! No use to say ‘O there are other balls’. Explanation: The poem is about a little boy. He loses his ball and watches it bouncing down the street into the water. To us, the loss of ball is of minor consequence but to the little boy, it was a valued possession. The ball had been with him for a long time and it was linked to the memories of the days when he played with it.













1.



Explanation of the Poem

@PROCBSE 212











Symbolism : The ball is symbol of the boy’s young and innocent days. Repetition : What, what Balls, balls Alliteration : buys a ball back Balls, balls What, What





Poetic Devices





He is learning, well behind his desperate eyes, The epistemology of loss, how to stand up Knowing what every man must one day know And most know many days, how to stand up. Explanation: The poet suggests that from the loss of the ball, the boy is learning how to stand up in a world of possessions. The boy is learning what it means to lose something. The poet says that knowing that every man has to stand up after such losses, the boy too will learn how to stand up and leave the losses behind as he would have understood the true meaning and nature of loss.



4.

Oswaal CBSE Question Bank Chapterwise & Topicwise, ENGLISH LANG. & LIT., Class-X

OBJECTIVE TYPE QUESTIONS Extract Based Questions



(5 marks each)

































These questions are for practice and their solutions are available at the end of the chapter





























(3) I dont know where I’ve placed my ID-card. Let me check the bag once more. Ah, finally! (4) I’ve been trying to call mom for the past 20 minutes and can’t get through. I don’t know how...Aarrgh! Again!













(2) I knew it! I knew he’ll fare well in his auditions for ‘Young Chef. Now, we prepare for the semi-finals.























(1) Hey ! Hey! That’s no way to dispose off the garbage. Have you no community sense? Please but it in the bin.



























What is the boy now, who has lost his ball, What, what is he to do? I saw it go Merrily bouncing, down the street, and then Merrily over — there it is in the water! [CBSE QB, 2021] Q. 1. The extract suggests that the poet is: (A) An onlooker observing (B) A parent recounting the incident (C) The boy talking about himself (D) Imagining the incident Ans. Option (A) is correct. Q. 2. The poet seems to have indicated the merry bouncing of the ball to (A) create a sense of rhythm in these lines. (B) support the happiness of the experience of playing. (C) contrast with the dejected feeling of the boy. (D) indicate the cheerful mood of the boy. Ans. Option (C) is correct. Explanation: The happiness of the bouncing ball has been compared and contrasted with the disappointed feelings of the boy. Q. 3. Choose the situation that corresponds to the emotion behind the exclamation mark in the poem.

(A) Option 1 (B) Option 2 (C) Option 3 (D) Option 4 Ans. Option (C) is correct. Explanation: The exclamation mark used in the poem denotes confusion which is represented in option 3. Q. 4. The poem begins with a question. Based on your reading of the poem, the speaker: (A) wants the boy to answer the question. (B) expects the passers-by to respond. (C) is looking for answers in a self-help book. (D) is thinking to himself. Q. 5. Alliteration is a literary device that occurs with the same letter or sound at the beginning of adjacent or closely connected words. Pick the option that showcases an example of alliteration from the extract. (A) What is the boy now (B) who has lost his ball (C) I saw it go (D) and then/ Merrily over Ans. Option (B) is correct. II. Read the following extract and answer the questions/ complete the sentences that follow : (5×1=5) An ultimate shaking grief fixes the boy As he stands rigid, trembling, staring down All his young days into the harbour where His ball went. I would not intrude on him; [CBSE QB, 2021] Q. 1. The poet uses the word ‘ultimate’ to describe the boy’s reaction. Pick the meaning that DOES NOT display what, ‘ultimate’ means in the context given. (A) Consequent (B) Final (C) Conclusive (D) Fateful Ans. Option (C) is correct. Explanation: The boy's reaction is not decisive or reaching a final solution, In short, the boy’s reaction is not conclusive.





I. Read the following extract and answer the questions/ complete the sentences that follow : (5×1=5)

@PROCBSE THE BALL POEM

213

(4) a reaction to the failure of retrieving the toy. (5) justified and similar to what it would be currently. (A) 5 & 2 (B) 1 & 3 (C) 2 & 4 (D) 3 & 5







Q. 3. Pick the option that lists the boy’s thoughts, matching with the line-As he stands rigid, trembling, staring down.































These questions are for practice and their solutions are available at the end of the chapter





























































III. Read the following extract and answer the questions/ complete the sentences that follow : (5×1=5) I saw it go Merrily bouncing, down the street, and then Merrily over — there it is in the water! No use to say ‘O there are other balls’: An ultimate shaking grief fixes the boy As he stands rigid, trembling, staring down All his young days into the harbour where His ball went [CBSE SQP, 2021] Q. 1. The poet uses the ball as a symbol of the boy’s: (A) sense of adventure. (B) carefree childhood days. (C) ability to bounce back. (D) extended family. Q. 2. The poet feels that there is no point consoling the boy as: (A) it would give him false hope. (B) he might demand for a new ball. (C) it might distress him further. (D) whatever he has lost, is irretrievable. Ans. Option (D) is correct. Explanation: The ball has gone down the alley and it is impossible to find the ball back in the vast water body where it has fallen. Q. 3. The word ‘harbour’ DOES NOT have a meaning similar to: (A) port. (B) pier. (C) dock. (D) cargo. Ans. Option (D) is correct. Explanation: Port, pier and dock are the places where ships are moored. Cargo is the luggage that is loaded on the ship for transportation. Q. 4. ‘Merrily over — there it is in the water!’ The dash here is meant to convey: (A) some familiar experience. (B) a feeling of excitement. (C) a sense of unexpected interruption. (D) some thoughtful moments. 





















(A) Option 1 (B) Option 2 (C) Option 3 (D) Option 4 Ans. Option (D) is correct. Explanation: The boy started crying over his loss. This means that he was not thinking of retrieving his ball or running and catching it at the end of the alley. He was not feeling happy to get rid of the ball. Option (4) represents the true thoughts of the boy. Q. 4. Why does the speaker choose not to intrude? This is so because the poet: (A) knows that it would embarrass the boy in his moment of grief. (B) feels that it’s important that the boy learn an important life lesson, undisturbed. (C) realises that he doesn’t have sufficient funds to purchase a new ball for the boy. (D) experiences a sense of distress himself, by looking at the boy’s condition. Ans. Option (B) is correct. Explanation: The poet wanted the boy to learn and understand that there are many occasions in life when a loved thing or person is lost but the life continues. Q. 5. Choose the option that lists the meaning of ‘harbour’ as used in the extract. Noun: (1) a place on the coast where ships may moor in shelter. (2) a place of refuge. Verb: (3) keep (a thought or feeling, typically a negative one) in one’s mind, especially secretly. (4) shelter or hide (a criminal or wanted person). (A) Option 1 (B) Option 2 (C) Option 3 (D) Option 4 Ans. Option (A) is correct. Explanation: In the poem, harbour has been used as a noun to indicate the place where the ships are moored or held in waiting for the time being.



















Q. 2. The boy is very young in this poem. As a mature, balanced grown-up, he might look back and think that his reaction of ‘ultimate shaking grief was: (1) disproportionate to the loss. (2) pretension to procure a new toy. (3) according to his exposure and experience then.





@PROCBSE



















Q. 5. The word that DOES NOT indicate a physical manifestation of sorrow in the boy, is: [CBSE SQP 2021] (A) worthless. (B) shaking. (C) trembling. (D) rigid. Ans. Option (A) is correct. Explanation: Shaking, trembling and rigid are not expressing only sorrow. The feeling of worthlessness develops sorrow.



Ans. Option (C) is correct. Explanation: This dash brings an interruption in the flow of the poem. This interruption is what the poet wants to convey in the mind of the reader about the sudden interruption in life due to loss of something or someone.



Oswaal CBSE Question Bank Chapterwise & Topicwise, ENGLISH LANG. & LIT., Class-X



214

SUBJECTIVE TYPE QUESTIONS Short Answer Type Questions























Q. 3. When we think of losses, we generally think of people or possessions. Time is considered a very precious commodity. Explain why time can probably be one of the things people bitterly regret losing/wasting. A [CBSE OD Set-I, 2020] Ans. ‘A stitch on time saves nine’. ‘The time that flies, comes back never’. So we should not waste our precious time because time doesn’t work according to us best we need to work according to time we should utilize the present fully because we can’t do anything about the time that has passed and the future is uncertain. Q. 4. ‘He senses first responsibility? What responsibility is referred to here? (The Ball poem) U (CBSE-Delhi 10D-2018)



Q. 1. Suggest a suitable by-line for “The Ball Poem” with a reason for your choice. [CBSE OD Set-I, 2020] Q. 2. What feelings do you think, might be experienced, at the loss of a mobile phone, for a youngster today? Explain how these would be different from those felt by the boy in the poem. U [CBSE OD Set-I, 2020] Ans. The youngsters today suffer from ‘No Mobile Phone Phobia’. Such people may become depressed due to the loss of their mobile phone. It is quite different from the grief experienced by the boy in the poem because the boy in the poem is not addicted to the ball. But today’s generation is so much addicted to phone.





(40-50 words & 3 marks each)

Topper Answer, 2018

These questions are for practice and their solutions are available at the end of the chapter

(100-120 words & 5 marks each) how games and sports are a good way to train U children to take losses in their stride. 

Q. 1. The poem deals with a child understanding loss for the first time. Matches and championships too, deal with a different sense of loss. Explain





Long Answer Type Questions

@PROCBSE THE BALL POEM

Q. 4. What is the epistemology of loss in this world of possessions ? How has the child learned to stand A up in life? 









Ans. Gains and losses are the two sides of the same coin. Getting, spending and losing things form a natural cycle of life. The boy is inconsolable at the loss of his ball. Actually, it is not the ordinary ball but his long association and attachment with it that makes the loss so unbearable. It is like the good sweet days of childhood that the boy cherishes so much but are lost and gone forever. They will never come back again. He can bear this loss by understanding the epistemology or nature of the loss. In this world of material wealth and possessions, it seems that money can by anything. However, it is a false conception. Money has its own limitations. Its nature is external. It can not compensate for the losses that a person suffers emotionally and internally. No wealth can buy back the ball that has been lost forever. Similarly, no wealth can buy back the lost childhood.





Think and create this address for people of your C age.

Q. 3. Why does the poet not offer money to the boy to buy another ball? U [CBSE Delhi Set-I OD Set-III 2020]





Commonly Made Error Students fail to understand the message that the poet wants to convey through the poem.



Answering Tip Students must focus on the deep meaning and realise what does the word ‘responsibility’ means here.

Solutions for Practice Questions Ans.1. The by-line can be ‘Loss of growing up and Transformation’. This is something of a coming of age that is seen through the lens of loss and suffering. The child comes to understand the world better when he understands that he is never going to get his lost ball back. It is gone for good and there is nothing anyone can do about it. The child undergoes a transformation. Ans.4. The responsibility of taking care of his things in this world of possessions and cope up with the loss. [CBSE Marking Scheme, 2018]

III. Ans.1. Option (B) is correct. Explanation: The childhood days are full of fun and carelessness. These days, once lost, can never be regained.











II. Ans.2. Option (B) is correct. Explanation: The boy was shrieking with grief over loss of a petty ball. So, his grief was disproportionate to his loss. He was crying because according to his little world and limited exposure to the outside world, it was a big loss.



Short Answer Type Questions

I. Ans.4. Option (D) is correct. Explanation: The speaker is not asking the questions from the reader in particular. He is trying to find the answers on his own.

Extract Based Questions







215







Ans. Research has shown that losing games is helpful for children because it teaches them to show empathy and cope with experience of losing. They need to practice losing a competition in front of their peers. Children who do not experience losing can grow up to be anxious, because they start seeing the possibility of not wining and they cannot deal with situations that do not go their way. Losing a game is the only way for children to learn from their mistakes and think about strategies to improve when children improve their skills and win the next time, they do not only get better at the sport or game, but they also learn something new. Learning new things increase children’s confidence and their self-belief and they start to be proud of their ability. Q. 2. If the Buddha were to summarise the life lesson of “The Ball Poem’, what would that sermon be?









These questions are for practice and their solutions are available at the end of the chapter

Detailed Answer : The poet thinks that it is of no use to purchase another ball. The child must feel his responsibility of taking care of his things and learn to cope up with the loss.

@PROCBSE 216

Oswaal CBSE Question Bank Chapterwise & Topicwise, ENGLISH LANG. & LIT., Class-X





Ans.2. Losses are integral part of life. We must look at them in totality. First of all, we must always keep in mind that life’s each and everything is God’s gift for which we must be thankful. The things which God has given can be taken back anytime. We must not get attached with these gifts. Once these attached gifts are taken away, we become sad we feel the lost thing is irreplaceable with any other things. The lost ball stands for the general losses a human being suffers as he grows old. The losses may be the loss of a personal possession or the death of a dear one or separation from a beloved one. As long as there is life, there will be many types of losses; what each one has to learn is bearing those losses.

Ans.3. The boy is very much troubled at the loss of his ball and plunges into grief. He stands stiff and trembling while staring at his ball. He is upset as he looks gloomily into the water because it has been with him for a long time. When the ball bounces into the water, all his memories of the childhood days flashes in front of him, the poet doesn’t offer him money to buy another ball. The poet suggests that from the loss of the ball, the boy is learning how to stand up in a world of possessions. The boy is learning what it means to lose something. The poet says that knowing that every man has to stand up after such losses, the boy too will learn how to stand up and leave the losses behind. The loss of the ball will make him learn that things are often lost and losses have to be accepted.

Long Answer Type Questions





REFLECTION Were you able to explore the theme of loss marked by the loss of the ball after reading the poem ‘The Ball’?

qqq

@PROCBSE

CHAPTER

6

AMANDA! – By Robin Klein

Revision Notes world where there are no restraints. She imagines herself to be a joyful mermaid sailing without any confines on the soft waves of the green sea. Did you finish your homework, Amanda ? Did you tidy your room, Amanda ? I thought I told you to clean your shoes, Amanda ! Scan to know Explanation: Amanda is more about now being asked about whether she had finished her this topic homework or had made her room tidy and also whether she had cleaned her shoes. These set of instructions mark a shift from the instructions given in stanza. Her name Amanda! is being called again with an exclamation mark which shows that her mother seems to be losing their cool and are troubled. I am an orphan, roaming the street. I pattern soft dust with my hushed, bare feet. The silence is golden, the freedom is sweet. Explanation: Amanda is again lost into her own world. After getting a fresh list of instructions from her mother, she wishes to be away from this daily routine life. Amanda perceives herself as an orphan roaming on the streets, moving aimlessly without any purpose. She seems happy as she draws patterns using her bare feet.

Introduction





3.







4.

Key Words







1. Nagging: persistent annoying someone 2. Restrictions: Limitations 3. Languid: Spiritless, lacking enthusiasm

5.

Don’t bite your nails, Amanda ! Don’t hunch your shoulders, Amanda ! Stop that slouching and sit up straight, Amanda ! Explanation: In the above stanza, Amanda is getting instructed, most probably by her mother, for not biting her nails and for sitting lazily with her shoulders bent. The tone of instruction here is not a friendly one and thus fails to make any sense to Amanda. There is a languid, emerald sea, where the sole inhabitant is me— a mermaid, drifting blissfuly. Explanation: Amanda uses her imagination as an escape point from day to day commands of her parents. She finds peace in her own created









2.











1.



Explanation of the Poem







The poem ‘Amanda!’ is about a small child whose parents are forever nagging her about what to do and what not to do. This frequent interference makes her very unhappy. She feels that she is not free to do anything that she wants to do. She wants freedom from all restrictions. She dreams to lead a life of a mermaid in a languid, emerald sea with her being the only inhabitant there. She feels that an orphan has more freedom than her. She also wishes to lead a calm and quiet life in a tower like that of Rapunzel with nobody to disturb her.



Summary







In the above poem, the poet Robin Klein says that a child should never be denied freedom. It deals with the upbringing of a small child, Amanda. It highlights the struggles faced by the child.

Don’t eat the chocolate, Amanda ! Remember your acne, Amanda ! Will you please look at me when I’m speaking to you Amanda !

@PROCBSE 218

Oswaal CBSE Question Bank Chapterwise & Topicwise, ENGLISH LANG. & LIT., Class-X









Anyone would think that I nagged at you, Amanda ! Explanation: Her mother keeps instructing Amanda on the do’s and don’ts, but Amanda remains lost in her own dreams. Her mother believe that Amanda is not reacting because she is annoyed. Her behaviour has made her mother feel bad and they get worried about her image. She is concerned about how society will perceive her if her child always remains in a foul mood.

Poetic Devices 

Stop that slouching and sit up straight Stop that sulking at once, Amanda! Mermaid (Taken from fairy tales) Rapunzel (Taken from German fairy tales) Languid, emerald sea Silence is golden Freedom is sweet





Allusion :







Alliteration :







Metaphor :



7.









6.





Explanation: Amanda is stopped by her mother from eating a chocolate as previously it caused her acne. Amanda is still lost in her own thoughts and doesn’t care enough to look up to her scolding mother. This carelessness further angers the mother and ask for her attention when she is being scolded. I am Rapunzel, I have not a care; life in a tower is tranquil and rare; I’ll certainly never let down my bright hair! Explanation: Amanda is still lost in her own dreams. She pictures herself to be the long goldenhaired Rapunzel who lived in a castle and had no care about anything. Amanda imagines that the life of Rapunzel must have been very peaceful and fantastic in the tower. But she is unwilling to let her hair down. She only wants to live by herself as the presence of another being in her life would require her to act in a certain way. Amanda wants to live free and happy. Stop that sulking at once, Amanda ! You’re always so moody, Amanda !

OBJECTIVE TYPE QUESTIONS Extract Based Questions

Explanation: The figure 'I' has a less than realistic view of being an orphan and according to the lines 'B' statement cannot be clearly inferred, so option B is correct. Q. 3. The golden silence is contrasted with the ________. (A) chaos of the street (B) constant instructions received (C) sweetness of freedom (D) hushed, bare feet Ans. Option (B) is correct. Explanation: According to the lives, golden silence is contrasted with the regularly received construction. Q. 4. The rhyme scheme ‘aaa’ in the above extract is followed in all other stanzas of the poem that are written in parenthesis, i.e., (). Why? Read the reasons given below and choose the option that lists the most accurate reasoning: (i) It shows the simplicity of the child’s thoughts. (ii) It reflects the harmony and rhythm of the child’s inner world. (iii) It mirrors a child’s expression. (iv) It highlights the poet’s aesthetic sensibility. (A) (i) and (iv) (B) (i) and (ii) (C) (ii) and (iii) (D) (iii) and (iv)



















These questions are for practice and their solutions are available at the end of the chapter



















































Q. 2. Read the statements A and B given below, and choose the option that correctly evaluates these statements. Statement A : The figure ‘I’ imagines a less than realistic view of being an orphan. Statement B : The figure ‘I’ does not like the speaker. (A) A is true, B is false, according to the extract (B) A is true, B cannot be clearly inferred from the extract. (C) A cannot be clearly inferred from the extract, B is false. (D) A is true and can be inferred from the poem, B is true too. Ans. Option (B) is correct.



























I. Read the following extract and answer the questions/ complete the sentences that follow : (5×1=5) I am an orphan, roaming the street. I pattern soft dust with my hushed, bare feet. The silence is golden, the freedom is sweet. [CBSE QB, 2021] Q. 1. The tone of the given lines is : (A) analytical. (B) despairing. (C) peaceful. (D) nervous. Ans. Option (C) is correct. Explanation: The child is imagining being. orphan and roaming the streets peacefully.





(5 marks each)

@PROCBSE 219

Q. 2. Pick the option that lists the image which correctly corresponds to the speaker’s mood in the extract.



Q. 5. Pick the option that lists the usage of the word ‘pattern’, as in the extract above. (A) That is a lovely pattern for a wallpaper. (B) He decided to wear a patterned shirt to the party. (C) Poetry is a form of pattern making. (D) She patterned her hair after her favourite celebrity. Ans. Option (D) is correct. Q. 6. Which option completes the popular adage given below? ___________________ silence is gold. (A) Precious are words for (B) Speech is silver (C) Silver is the tongue (D) Ideas may be precious but Ans. Option (B) is correct. Explanation: It's a popular proverb. Q. 7. A transferred epithet is a literary device where the modifier or epithet is transferred from the noun it is meant to describe to another noun in the sentence. Based on the given definition of ‘transferred epithet’, choose the option that lists an example of transferred epithet. (A) soft dust (B) hushed (bare) feet (C) freedom (is) sweet (D) silence (is) golden Ans. Option (B) is correct. Q. 8. Which image corresponds to what Amanda, is doing in this extract?



AMANDA!

    





      











Q. 3. Alliteration is a literary device used in the extract. Which of the following options DOES NOT include example of this literary device? (A) The moon and the shimmering stars watched over us. (B) With that charming chat, Catherine chose comfort. (C) Away ran the pathetic pooch pouting like a princess. (D) Dee dee was driving down day after day. Q. 4. What does the repetition of “Amanda!” at the end of each line reflect? (A) It describes who the speaker is talking to. (B) It represents the absent-mindedness of the listener. (C) It shows the frustration of the speaker. (D) It helps create a rhyme scheme. Q. 5. Select the option that fits with the following: 



























(1) (2) (3) (4) (A) Option 1 (B) Option 2 (C) Option 3 (D) Option 4 Ans. Option (C) is correct. Explanation: The speaker is angry. Option 1 shows toil, Option 2 shows confusion, Option 3 shows anger and Option 4 shows surprise.         

















slouching : straight:: ___________ :___________ (A) transparent: translucent (B) lazy: agile (C) forgetful: lively (D) generous: liberal Ans. Option (B) is correct. Explanation: Slouching and Straight are closely related antonyms and so are Lazy and Agile.

    



















      







III. Read the following extract and answer the questions/ complete the sentences that follow : (5×1=5)













        

(1) (2) (3) (4) (A) Option 1 (B) Option 2 (C) Option 3 (D) Option 4 Ans. Option (B) is correct. Explanation: Amanda loves to stay in the world of her imagination. II. Read the following extract and answer the questions/ complete the sentences that follow : (5×1=5)



























































































There is a languid, emerald sea, where the sole inhabitant is me— a mermaid, drifting blissfully. [CBSE Delhi Set-I, 2019] Q. 1. Who do these lines refer to? (A) Amanda (B) Mermaid (C) Amanda’s sister (D) None of these





These questions are for practice and their solutions are available at the end of the chapter













Q. 2. Why is this stanza given within brackets? (A) To show inner thoughts of mother (B) Day dreaming by Amanda (C) To show the contrast (D) Both (B) and (C) Ans. Option (D) is correct.



















Don’t bite your nails, Amanda! Don’t hunch your shoulders, Amanda! Stop that slouching and sit up straight, Amanda! [CBSE QB, 2021] Q. 1. The purpose of the speaker’s words in the given extract is to: (A) show the speaker’s power over the listener. (B) make the listener a better human being. (C) advise the listener as an elder. (D) improve the listener’s posture and habits. Ans. Option (D) is correct. Explanation: because the speaker is adversing Amanda not to bite her nails and sit straight.

@PROCBSE Oswaal CBSE Question Bank Chapterwise & Topicwise, ENGLISH LANG. & LIT., Class-X







(B) Metaphor (D) Personification













(A) Simile (C) Alliteration Ans. Option (B) is correct.



Q. 5. Which poetic device is used in line-1?





















(D) Away from everyone Ans. Option (C) is correct. Explanation: Amanda want to move away from the constant nagging of her elders.



Q. 3. Give the word from the stanza which means “free flowing act of going with motion and force”. (A) Languid (B) Blissfully (C) Drifting (D) Emerald Q. 4. Why does Amanda want to be sole inhabitant of sea? (A) Away from noise (B) Away from mother (C) Away from this noisy and disturbing atmosphere



220

SUBJECTIVE TYPE QUESTIONS Short Answer Type Questions

Q. 1. Comment on the tone of the speaker when she says ‘Will you please look at me when I’m speaking to you, Amanda!’. [CBSE SQP, 2021-22] 







(40-50 words & 3 marks each)

Ans. Value Points

Guidance











The question indicates that the examiner knows that Tone Amanda is being questioned and she chooses not to • exasperated respond and hence that requires no elaboration. • irritated The question needs an answer to the tone of the • frustrated speaker for the line mentioned in the question. • annoyed Content: Or words with similar meanings Illustration- Exclamation mark, suggestive of the Award 1 mark for full explanation of the identification of the tone, supported by reference to the supporting inherent emotion evidence. Award ½ mark for partial explanation. Expression:











These questions are for practice and their solutions are available at the end of the chapter











Ans. The speaker i.e. Amanda’s mother is very caring and concerned towards Amanda. So much so that she has rather become over protective. Amanda’s careless attitude has made her worry for her future. She wants to teach Amanda all the rules of proper social conduct lest people should blame her for nagging Amanda. Q. 4. ‘Amanda is alone but not lonely in the world she envisions.’ Justify the statement with reference to any one image she pictures in her mind. [CBSE QB, 2020-21] Q. 5. This is Calvin, a character from a comic strip created by Bill Watterson. Do you think this might be something Amanda would say? Why/ why not? Give reasons.











Detailed Answer: Amanda's mother is the speaker of the above lines. Her tone is laced with frustration and disappointment. She is irritated that Amanda isn't looking at her when being spoken to. Amanda was distracted in her thoughts. As a result her mother is annoyed by her behaviour. Q. 2. What does the line “never let down my bright A hair” tell us about Amanda? [CBSE QB, 2020-21] Q. 3. How would you characterise the speaker in the poem Amanda!’? List any two qualities, supporting the reason of your choice, with evidence from the U [CBSE QB, 2020-21] poem.







1 mark when both aspects included  Answer organised effectively  usage of words supporting illustration (as supported by, as illustrated by, as can be seen from etc.) ½ mark when either aspect is missing Deduct ½ mark from the overall score if the error density is high (more than a total of 2 spellings and grammatical errors).

@PROCBSE

AMANDA!

221



























Q. 6. Justify the poet’s allusion to Rapunzel in ‘Amanda’. E [NCERT] Ans.– It is justified as Rapunzel lived with a witch who would constantly keep a check on her and had several rules to be followed. – Amanda too, feels caged by the impositions – Amanda desires peace in isolation like Rapunzel had in her tower; clarifying she will not let her hair down for anyone (2 × 3= 6 marks) [CBSE Marking Scheme, 2020]





Detailed Answer: Rapunzel was a beautiful girl with long blond hairs, who lived with a witch who would constantly keep a check on her and had several rules to be followed. Amanda, like Rapunzel, feels caged by the restrictions imposed on her. She desires to live a peaceful life in isolation like Rapunzel lived in a tower. But on the other hand, she clarified also that she would not let her hair down for anyone in order not to interrupt her peaceful life.

Long Answer Type Questions











Anne: Gosh! You do run off into some strange worlds, don’t you? Amanda: (nods) Well... I do [CBSE QB, 2020-21]





Commonly Made Error Most of the students failed to identify the poetic devices used in the poem.



Answering Tip





[CBSE QB, 2020-21] Ans. The given quote is fro the book `The Little Prince’ by Antoine de saint-Exupery. Bringing up teenagers is truly challenging and fun. Teens can be rebellious and apparently not headed with a highly

independent perspective. In such circumstances, yelling at them, and nagging them, as done in the poem with Amanda, will only make matters worse. Forcing decisions on them only make them detached, as we see in Amanda’s case, where she wishes to be the alone mermaid, an orphan, or the lonely Rapunzel. Amanda’s condition may be witnessed in every child’s life. When children are told a long list of do’s and don’ts, they feel targeted. Then they start comparing their own life with that of the grown-ups around then be it their own parents, teachers or kinds. They feel like shutting off their mind from the outside world. But they should always remember that whatever the parents or teachers are telling is for their good only. Grown-ups can befriend the children but to a certain limits. The habits inculcated in children by their parents will help them in their adult life. Q. 2. What advice might ”chatterbox” Anne (Frank) have for the quietly imaginative Amanda? Present this as a conversation between the two.



Q. 1. Read the quote given below carefully. Based on your understanding of the poem, explain how and why this quote can be applied to the poem.





(100-120 words & 6 marks each)

A thorough reading of the poem can help the students to write proper answers.

These questions are for practice and their solutions are available at the end of the chapter



Ans. According to me, Amanda would definitely say this. She is a teenager and like other teens she loves freedom. She wants to have her own kind of life in her imaginary world. Her world is not the world of harsh realities, but it is an imaginary world of fairies and mermaids. She finds solace in roaming around. And is not at all ready to accept the reality. She craves to live in a lonely tower peacefully like Rapunzel.



A [CBSE QB, 2020-21]

@PROCBSE 222

Oswaal CBSE Question Bank Chapterwise & Topicwise, ENGLISH LANG. & LIT., Class-X

Solutions for Practice Questions bare feet. She finds silence ‘golden’ and freedom sweet. Hence, she is not lonely in her own company.

Extract Based Questions

Ans.3. Option (C) is correct. Explanation: Languid means weak or faint. Blissfully means very happily. Drifting means freely flowing. Emerald is a precious stone.

Short Answer Type Questions







Ans.2. The above line tells us about Amanda that she doesn’t want to be with anyone. Her desire is to be alone like an orphan in a tall tower where nobody can approach her. She doesn’t even want to be always nagged by her parents and listen to her mother scolding. Ans.4. Amanda feels that she is being constantly pressurised in being a well-behaved girl as per her mother’s demand. She wants to be alone and enjoy being herself. She wants to roam around in the sheets alone and pattern the dust with her







: Gosh! You do run off into some strange worlds, don’t you? Amanda : (nods) Well…………. I do. Anne : Is there anything wrong with you? Amanda : No, I just want to be left alone. I want to be on my own. Anne : Are you crazy? Children pine to be with their parents, friends and neighbours? Think of those orphans who have never enjoyed the love of caring mother and loving father. Amanda : I don’t know. But I don’t like my mom consistently nagging me for my mistakes. I’m fed up of being watched by my parents continuously. I want to enjoy my freedom. I’m frequently scolded for eating too many chocolates as this may cause pimples. Anne : I do understand that this constant nagging has made you so sad that now you have even stopped to imagine yourself as someone else. But my dear, have faith in your parents. They want you to grow up as a responsible adult. They bear a great responsibility. It is their duty to teach you the rules of social conduct. I think, you need to talk to them regarding this. 









Ans.4. Option (C) is correct. III. Ans.1. Option (A) is correct. Explanation: In her imagination Amanda wanted to be a mermaid





II. Ans.3. Option (A) is correct. Explanation: ‘charming char Catherine’, ‘pathetic pooch pouting’ and ‘driving down day’ represent alliteration.

Long Answer Type Questions Ans.2. Anne



I. Ans.4. Option (C) is correct. Explanation: The words in parenthesis actually represent the inner voice of Amanda.





REFLECTION Were you able to understand the importance of freedom even for children as highlighted by the poet?

qqq

@PROCBSE

CHAPTER

7

THE TREES – By Adrienne Rich

Revision Notes but it has been used as an Scan to know imagery by the poet to show more about the destroyed forests and the this topic false nature that humans have tried to keep in their houses. Since the forest outside was empty, no birds could sit on the branches of trees, no insects could hide in the The Trees trees and sunlight could never disappear under the shadows of the trees. The speaker feels that the empty forest will be full of trees by the next morning. All night the roots work to disengage themselves from the cracks in the veranda floor. The leaves strain toward the glass small twigs stiff with exertion long-cramped boughs shuffling under the roof like newly discharged patients half-dazed, moving to the clinic doors. Explanation: It seems like the trees work silently in the night in order to complete their mission of getting free from the boundaries of the house. Therefore, they work all night to free themselves through the cracks on the veranda floor. The leaves of the trees try very hard to put a lot of pressure on the glass so that they could break it. The small twigs have become very hard due to applying so much pressure to free themselves.

Introduction

‘The Trees’ shows the conflict between man and nature. With the growth and development of society, human beings have used nature for their own benefit and caused a lot of harm to it. In order to use natural resources, men have forgotten the importance of nature.

Summary



















2.







Adrienne Rich’s poem, ‘The Trees’, is a voice with a body engaged in activities and sensing intrusions that are not organic to the conventions of a nature poem. This poem narrates the struggle of a population of trees to escape the confines of a green house or container of nature. The poet tells us that how trees want to break out of the bondage of man-made things and reunite with their natural surroundings. The message is that the forests have disappeared. So people have planted trees in their homes. Trees are revolting as they have lost their natural usefulness. No bird nests in them, nor do they spread their shadows to the tired people.

Key Words





1. Personified: to have all the attributes of (herehumans) 2. Confined: not free to move 3. Grandeur: greatness, vastness 4. Dimensions: A single aspect of a given thing.

The trees inside are moving out into the forest, the forest that was empty all these days where no bird could sit no insect hide no sun bury its feet in shadow the forest that was empty all these nights will be full of trees by morning Explanation: The poet observes that the trees in his house are moving outside into the forest which has been empty for a long time. It is important to understand that the trees are not actually moving,

















1.



Explanation of the Poem

@PROCBSE 224

Oswaal CBSE Question Bank Chapterwise & Topicwise, ENGLISH LANG. & LIT., Class-X







Simile : Like newly discharged patients Still reaches like a voice into the rooms The moon is broken like a mirror Imagery : Its pieces flash now in the crown of the tallest oak. Alliteration : Writing long letters in a sky still open the smell of leaves and lichen.















4.

Poetic Devices





I sit inside, doors open to the verandah Writing long letters In which I scarcely mention the departure of the forest from the house. The night is fresh, the whole moon shines in a sky still open the smell of leaves and lichen still reaches like a voice into the rooms. Explanation: The poet sits in her house, writing long letters, with the doors of house opening to the Veranda. She mentions in her letters, the trees that are moving out to the empty forest. It is a full moon night where the moon is shining in the open sky and the night is very fresh. The poet smells the leaves and lichens coming from the trees that reach her like a voice reaching into the rooms. My head is full of whispers which tomorrow will be silent. Listen. The glass is breaking. The trees are stumbling forward



3.

into the night. Winds rush to meet them. The moon is broken like a mirror, its pieces flash now in the crown of the tallest oak. Explanation: The poet listens to the sounds coming from the leaves and lichens of the trees. These sounds will not be there in the morning as the trees will move out to the forest in the night and will not be in the house by morning. Now, the poet can hear the glass breaking due to the efforts of the twigs. The trees hurry outside stumbling on each other. As the trees go in the open, it seems like the wind is moving fast towards them to meet them. The trees are so tall that they have the moon into pieces like a broken mirror. The moon sits like a crown on the head of the tallest oak tree.



The large branches of the trees try to move slowly from there and look like newly discharged patients from a hospital, who become half-shocked on coming to the outside world.

OBJECTIVE TYPE QUESTIONS Extract Based Questions

Q. 2. Choose the image that represents the main object of the extract literally.



























I. Read the following extract and answer the questions/ complete the sentences that follow : (5×1=5) All night the roots work to disengage themselves from the cracks in the veranda floor. The leaves strain toward the glass small twigs stiff with exertion long-cramped boughs shuffling under the roof like newly discharged patients half-dazed, moving to the clinic doors. [CBSE QB, 2021] Q. 1. Choose the option that lists the examples of exertion. 1. Neha had been up all-night keeping accounts and now she’s resting. 2. Swati was running for five miles non-stop last evening. 3. Hemant is watching his favourite show and having snacks. 4. Sachin helped a senior citizen board a flight in the morning. (A) 1, 2 (B) 2, 3 (C) 3, 4 (D) 1, 4 Ans. Option (A) is correct. Explanation: Both Neha and Swati are tired due to extra physical labour.





(5 marks each)







(ii)



































(iii) (iv) (A) Option (i) (B) Option (ii) (C) Option (iii) (D) Option (iv) Q. 3. Choose the correct option that mentions the correct poetic device against the correct example.













(i)

(1)

simile

long cramped boughs

metaphor

like newly discharged patients

@PROCBSE THE TREES

small twigs stiff with exertion

(B) some part of the sky still left open for women to explore, away from limitations set by patriarchal norms. (C) the opportunities symbolized by an open sky outside can be utilized by a single woman. (D) The portion of the sky bathed in moonlight versus the part that is still dark. Ans. Option (B) is correct. Q. 3. The extract uses ‘open’ twice, suggesting different meanings. Pick the option that lists the correct meanings, respectively. (A) entry point—vastness (B) sharing a common space—great view (C) allowing access and vision—expanse of space (D) advantage—obstacles ahead. Q. 4. Choose the option that DOES NOT state the meaning of the underlined word. ‘I scarcely mention…’ (A) tactfully (B) hardly (C) barely (D) only just Ans. Option (A) is correct. Q. 5. Choose the option that correctly depicts the imagery in the last two lines of the extract.

personifi(2) cation simile

225



like newly discharged patients under the roof

simile

leaves strain toward the glass























(ii)

























These questions are for practice and their solutions are available at the end of the chapter





















Q. 2. The poet uses the indefinite article for ‘sky’ instead of the definite one. This is so because the poet is referring to: (A) the ceiling of the verandah as ‘a sky’ versus the door kept open, being ‘the sky’.









































Q. 3. The winds are rushing to meet the_________. (A) forest (B) moon (C) trees (D) windows Ans. Option (C) is correct. Q. 4. Where are the pieces of the broken moon shining? (A) In the sky (B) In the forest (C) On the roof (D) On the top branches and leaves of the tallest oak tree. Q. 5. Find the word from the extract which means the same as 'Shine'. (A) crown (B) mirror (C) flash (D) None of the above













Q. 2. The crown of the oak lies in its (A) top most branch (B) roots (C) rich and tasty fruit (D) broad leaves Ans. Option (A) is correct.

























.....Winds rush to meet them. The moon is broken like a mirror, its pieces flash now in the crown of the tallest oak. [CBSE, 2020 (Modified)] Q. 1. The above lines were written by (A) Robert Frost (B) John Berryman (C) Robin Klein (D) Adrienne Rich























(iii) (iv) (A) Option (i) (B) Option (ii) (C) Option (iii) (D) Option (iv) Ans. Option (D) is correct. II. Read the following extract and answer the questions/ complete the sentences that follow : (5×1=5) I sit inside, doors open to the veranda writing long letters in which I scarcely mention the departure of the forest from the house. The night is fresh, the whole moon shines in a sky still open… [CBSE QB, 2021] Q. 1. The poet talks about the house as: (A) a safe sanctuary from predators. (B) a place of confinement. (C) a representation of family. (D) a concrete alternative to a natural on Ans. Option (B) is correct. Explanation: It is so because the houses today are congested, small and lack ventilation.













(i) (ii) (iii) (iv) (A) Option (i) (B) Option (ii) (C) Option (iii) (D) Option (iv) Ans. Option (C) is correct. III. Read the following extract and answer the questions/ complete the sentences that follow : (5×1=5)

(i)



























































personifi- like newly discharged patients (4) cation refrain half-dazed, moving (A) Option (1) (B) Option (2) (C) Option (3) (D) Option (4) Ans. Option (B) is correct. Q. 4. The poet of the poem is (A) Robin Klein (B) Walt Whitnan (C) Anrienne Rich (D) Robert Frost Ans. Option (C) is correct. Q. 5. Choose the option listing the image that the poem metaphorically represents.





refrain



(3)

@PROCBSE 226

Oswaal CBSE Question Bank Chapterwise & Topicwise, ENGLISH LANG. & LIT., Class-X

SUBJECTIVE TYPE QUESTIONS Short Answer Type Questions



Q. 4. Change never comes easy. Justify the quote in the A context of the poem “The Trees” [CBSE QB, 2020-21] Q. 5. The forest undergoes tremendous transformation overnight. Discuss with reference to the poem A [CBSE QB, 2020-21] “The Trees” Q. 6. The poet uses trees as a symbol for women breaking patriarchal notions. Do you agree? Why/ A Why not? [CBSE QB, 2020-21] Ans. Indeed, the poet uses trees as a symbol for women breaking patriarchal notions. In a male dominated society, women are in bondage. They are struggling to free themselves. Similarly, nature representing tree and forests, is struggling to escape itself form the onslaughts of men to destroy it. Q. 7. In the poem ‘The Trees’, where are the trees ? What are their roots, twigs, etc trying to do ? R [CBSE OD Set-I, 2017] Ans. The trees are inside the house. The roots work to disengage themselves from the cracks in the veranda floor, leaves strain towards the glass and the small twigs and boughs are trying to move to the doors. Q. 8. What are the three things that can’t happen in a R [CBSE Delhi Set-I, 2017] treeless forest ?



Q. 1. In the poem “The Trees”, the poet compares the trees to “newly discharged patients”. Highlight U any two reasons for the same. [CBSE QB, 2020-21] Q. 2. The process of “moving out” was tedious for the trees. Support the statement with reference to the U [CBSE QB, 2020-21] poem.



Q. 3. The trees have been shown to have human qualities in the poem. Justify the statement with A any two instances from the text. [CBSE QB, 2020-21]













Ans. Every part of the confined trees rise in revolt and struggle hard to free itself. The roots and leaves struggle to come out of the glass. Small twigs become hard due to over work and the long cramped boughs are trying to break open the roof. So ‘moving out’ is tedious for trees.









Ans. Nature represented by trees and forest, is struggling like women to escape itself from the onslaughts of man to destroy it. The victorious march of the trees gives a message of hope that nature can’t be tamed or subdued by the arrogance of man.





















































(40-50 words & 3 marks each)

Long Answer Type Questions





















-Sr. correspondent, The U.S. Chronicle It was a movement to behold! The trees… U [CBSE QB, 2020-21] (continue) Ans. Change Sr. Correspondent, The U.S. Chronicle It was a movement to behold! The trees along with their leaves, twigs, bough and roots were struggling hard to free themselves. It was a historical movement. Everyone around was perhaps shocked and embarrassed by the sudden and unexpected happenings. It appeared as if every part of the confined trees rose in revolt and struggled hard to



CHANGE THIS WEEK

free itself. The roots and leaves strived to break free. Small twigs become hard due to overwork. Long cramped boughs were trying to break open the roof and come out. The confined trees put in artificial glass houses wanted to be free. They are shuffling hard to come out in the open. They are suffocated with the limited space available. Humans must understand the negative impact of their actions on nature and mend their ways before it is too late. Man is doing more and more harm to nature. Nature is getting angry. Man’s existence on this planet is in danger. Thus, it become foremost duty of every citizen to work for the protection of the environment. Q. 2. Imagine yourself to be the tallest oak among the trees. Write a speech motivating the other trees to work hard in order to move out of the veranda into the forest. You may use the clue given belowThe Importance of Working Together Dear fellow trees, I see how charged up you all are to move away from the confines of…(continue) [CBSE QB, 2020-21]



Q. 1. Imagine you are a senior journalist covering this event of the trees moving. Pen down a 120 words’ submission for the weekly newspaper column titled-‘Change’ Include a brief account of your observation as well as your opinion. You may begin like this:











(100-120 words & 6 marks each)

These questions are for practice and their solutions are available at the end of the chapter

@PROCBSE THE TREES





and subdue nature created by man. We have been deprived of our homes so strive to free yourselves from the clutches of man. Your toil won’t go in vain. By morning, we will be marching victoriously to the forest. And the forest will be full of trees again. Q. 3. The poem ‘Trees’ appears to be about trees, environment, deforestation and afforestation. On a deeper level we discover that Adrienne Rich is expressing her concerned about women and their emancipation. Elaborate with reference to the U [CBSE QB, 2020-21] poem. Q. 4. What message does Adrienne Rich want to convey A through the poem, ‘The Trees’? 









The Importance of Working Together Dear fellow trees, I see how charged up you all are to move away from the confines of these artificial glass walls to the forest, our natural habitat. See outside the night is fresh. The full moon is shining in the open sky. You can feel the mild wind blowing as if getting ready to welcome our victorious arrival. Come on friends, do not forget that freedom is essential for all. We wanted this freedom since a long time. Now that time has come so put in your maximum efforts and come out in the open. We together will break open all the barriers to contain



Ans.

227





Commonly Made Error Most of the students fail to co-relate the underlying themes of deforestation, afforestation and women emancipation.





Answering Tip A thorough study of the poem in written is suggested for better performance.

Solutions for Practice Questions

Long Answer Type Questions

Short Answer Type Questions







Ans.1. The comparison converges two different aspects: Physical aspect- the trees like woman feel like patients who are on the way to recovery and emotional aspect- the optimism of full recovery and excitement of leaving the confines. Ans.4. Change never come easy. But when people come together for a good cause, we can accomplish great things. In the same way, each and every part of the tree rises against the slaughtering of men and wage a long and hard struggle to liberate themselves from the bondage of man. Ultimately they are victorious. They return to their original home where they should be. Ans.5. The roots of these trees are engaged into the cracks of the veranda floor. At night, these roots try to free themselves from the cracks. The leaves try to



Ans.3. In her poem ‘The Tree’ poetess Adrienne Rich subtly drives home the message about the importance of tree. Without trees, the birds would have no place to sit, insects will have no place to hide and the sun would not bury its feet in shadow. The trees do not look attractive indoors. The poetess hereby emphasizes that trees need to be kept alive, but should not be ‘imprisoned’ inside the house as they look more beautiful and tend to thrive outdoors and that is where trees belong. On the deeper level, we discover that Adrienne Rich voices concern about women and their emancipation. The liberation of the crushed women is based on the same theme. In a maledominated society, women are struggling to come out of the clutches of men to gain freedom. The victorious march of the trees gives a message of hope that nature can’t be tamed or subdued by the onslaughts and arrogance of man.











I. Ans.2. Option (D) is correct. II. Ans.3. Option (C) is correct. III. Ans.1. Option (D) is correct. Ans.4. Option (D) is correct. Ans.5. Option (C) is correct.

move towards the glass. Twigs become harden and the boughs try to expand under the roof. Ans.8. The three things that cannot happen in a treeless forest are—the sitting of a bird on trees, the hiding of insects and the sun burying its feet in the shadow of the forest.

Extract Based Questions

These questions are for practice and their solutions are available at the end of the chapter

@PROCBSE 228

Oswaal CBSE Question Bank Chapterwise & Topicwise, ENGLISH LANG. & LIT., Class-X



Ans.4. The poem ‘The Trees’ states that in the conflict between man and nature, man has caused much harm to nature. Man has learned to acquire a lot of material goods, but forgotten the importance of preserving nature. As a result, large forests have been cut down, animals have been killed and water bodies have been destroyed. Man has judged nature wrongly to be weak, whereas nature is the most powerful. Any natural disaster can not be prevented, though we are moving in a technologically advanced world. The destructive forms of nature e.g. flood, earthquake, volcanoes,

tornadoes etc. teach man that the real power lies with nature and it can do anything. The poet wants to give the message that the presence of a few trees inside our homes does not become equal to nature, but a very false idea of nature. Real nature is outside, in the forests that we have destroyed. The poem tells us how the trees want to break free from the walls that humans have put around them, and go out in the forest. We shall stop cutting trees, otherwise we will be responsible for the deleterious consequences.

Were you able to make out the relation between the longing for freedom by women as represented by the trees, in the poem 'The Trees'?





REFLECTION

qqq

@PROCBSE

CHAPTER

8

FOG – By Carl Sandburg

Revision Notes Explanation of the Poem The fog comes Scan to know on little cat feet more about It sits looking this topic over harbour and city on silent haunches and then moves on. Explanation: The fog enters silently like a small cat. The fog seems to change into a cat Fog and the cat again morphs into the fog. Then, it sits there for a while as a cat on its haunches and then moves ahead.







1.





In this short poem, the poet Carl Sandburg portrays the nature and its silent working. The poet says that the fog comes silently and slowly like a cat. It is a fine example of the use of metaphor in the poetry.



Introduction

Metaphor :

The fog comes on little cat feet. Personification : Fog is personified. Analogy : The fog is featured as that of a cat.





Key Words

Poetic Devices



In the poem ‘Fog’ the poet has portrayed the taciturn working of nature. To prove his point, the poet gives a comparison of the fog and a cat. The poet says that the fog comes silently and slowly like a cat. When it comes, it does not disturb us. It sits silently as a cat does. It goes on falling and looking silently over harbor and city. Then it moves on all at once with a gust of wind.





Summary











1. Portray: to describe 2. Taciturn: silent, temperamentally untalkative

OBJECTIVE TYPE QUESTIONS Extract Based Questions







(1) Cats are independent animals, they don’t follow rules, they slip and slide in and out of our lives as they please.



Q. 1. Choose the option with qualities of the cat that Carl Sandburg applies to the fog?

(2) Cats are distrustful of strangers and can be jealous and moody.















I. Read the following extract and answer the questions/ complete the sentences that follow : (5×1=5) The fog comes on little cat feet. It sits looking over harbour and city on silent haunches and then moves on. [CBSE QB, 2021]





(5 marks each)

@PROCBSE 230



Q. 4. Choose the option that DOES NOT list movement of the fog.

(2)

A fog drifts in, the heavy-laden Cold white ghost of the sea -Sara Teasdale

(3)

The fog it tricks the eye The wonder of the veil of white Under the clear blue sky. -K.C.Cox

(4)

...comes gliding, rolling in at ground level, pressing its white featureless face against... R.H.Grenville





It stole in on us like a foot-pad Somewhere out of the sea and air... -E.J.Pratt















Q. 5. The poet has used short lines to compose the given poem. Choose the option that lists the most appropriate explanation for the same. (A) By keeping the lines short, the poet keeps the liberty in stepping away from rule, traditional form, logic, or fact, in order to produce a desired effect. (B) By keeping the lines short, the poet wants the reader to omit some parts of a sentence, which gives the reader a chance to fill the gaps while reading it out. (C) By keeping the lines short, the poet wants to introduce ambiguity or contradiction into an otherwise straightforward sentence. (D) By keeping the lines short, the poet is controlling the pace to make the reader slow down thereby reflecting the slow rolling in of the fog. Ans. Option (D) is correct. Explanation: The pace of the fog and the poem resembles a lot.

(3)

Imagery is the language used by poets, and writers to create visual representation of ideas in the minds of the readers.

(A) Option (1) (B) Option (2) (C) Option (3) (D) Option (4) Ans. Option (C) is correct. Explanation: The words in parenthesis actually represent the inner voice of Amanda.



Transferred epithet is a literary device when an adjective usually used to describe one thing is transferred to another.



(2)



(1)

Personification is a literary device where you give an animal, object or natural phenomenon, qualities or abilities that only a human can have.







(1)

















(3) Cats are stealthy, moving in slow motion at times and they appear to be moving in a mysterious fashion. (4) Cats often communicate with a combination of a distinctive sound and body language. (5) Cats like to move on at their own pace and before you know it, they've disappeared. (A) Only 1 (B) 2, 3 and 4 (C) 1,3 and 5 (D) Only 4 Ans. Option (C) is correct. Q. 2. Which one/s of the following applies to the given lines?



Oswaal CBSE Question Bank Chapterwise & Topicwise, ENGLISH LANG. & LIT., Class-X

Oxymoron is a figure of speech in which contradictory terms appear side by side. (A) 1 and 4 (B) 2 and 3 (C) Only 1 (D) Only 3 Ans. Option (D) is correct. Q. 3. Pick the option that includes an image of the cat on its haunches.





















    

(3) (4) (B) Option (2) (D) Option (4)

      





        

(1) (2) (A) Option (1) (C) Option (3)





















(4)

SUBJECTIVE TYPE QUESTIONS Short Answer Type Questions



















Ans. The fog enters silently like a small cat. Then, it sits there for a while overlooking and engulfing the harbour. It continues sitting silently there for a while as a cat on its haunches and moves ahead. The

phrase silent haunches helps the readers to create mind pictures, hence is a good example of imagery. Q. 3. Comment, in detail, on the significance of the U closing phrase of the poem-“moves on”. [CBSE QB, 2020-21] Ans. The fog like a cat keeps on lying for hour. It keeps on overlooking the harbour and the city below. The atmosphere around is filled with silence and mystery and then like a stalking cat, it moves on. It disappears silently unnoticed.

These questions are for practice and their solutions are available at the end of the chapter



Q. 1. Would you describe the role of the fog as active or passive? Support your answer with reference to U [CBSE QB, 2020-21] the poem “Fog”. Q. 2. Why does the poet use the phrase “silent haunches”? Substantiate your answer with reference to the poem “Fog”. [CBSE QB, 2020-21]





(40-50 words & 3 marks each)

@PROCBSE

FOG

Long Answer Type Questions



(100-120 words & 6 marks each)



















Commonly Made Error Most of the students do not understand the underlying meaning of the poem which the poet wishes to convey.





Answering Tip Students are advised to read the poem thoroughly and understand the relation between the fog and the cat.

Solutions for Practice Questions Extract Based Questions

I. Ans.3. Option (D) is correct.

Short Answer Type Questions



Ans.1. I would describe the role of the fog as passive one. The fog is compared to a cat that likes to sit on window edges and look outside. The fog too sits silently for a while and then moves on.

Long Answer Type Questions

Ans.2. In the poem ‘Fog’ the poet Carl Sandburg compares the fog with the cat which enters our

home in an unpredictable manner. Next he says that the fog sits over the whole of the city as a cat sits silently by folding her logs behind itself and looks around the nearby places and thinks. Similarly, it seems that the fog silently covers the city but later leaves it unnoticed. ‘How Tell Wild Animals’ that do not have qualities that can be drawn out to be compared with a fog are the Asian Lion and the Bear. Both of these animals have a huge and terrible built. There presence can never be unnoticed. One can notice them from a far distance Moreover, the loud roar of the Asian Lion and the black color the Bear can easily attract everyone’s attention.

REFLECTION



a straight manner in the same way the snake can never walk straight. Its movement is graceful and beautiful. Many poets have described the wavy motion of the snake with that of the wavy water. Q. 2. Do you agree with the poet’s comparison of fog to a cat or do you feel some other image would have done justice to the poem? Justify your answer by substantiating your claims with examples. OR The poet beautifully compares the fog to a cat. Choose any two animals from ‘How to Tell Wild Animals’ that do not have qualities that can be drawn out to be compared with a fog. A [CBSE QB, 2020-21]

Were you able to figure out the similarities between the fog and a cat?













Q. 1. In the poem “Fog”, the poet features the fog as animated. If you had to write a similar poem for one other natural phenomenon, which animal would you use to convey the characteristics of that particular phenomena? Justify, in detail, your choice of animal corresponding to it by bringing U [CBSE QB, 2020-21] out similarities. Ans. The poem ‘Fog’ describes the advancement of fog toward the city and harbour. He says the fog comes like a cat comes on its little feet. This means the fog advances towards the city very slowly and calmly just like a cat. It enters our home in an unpredictable manner. Similarly, the zig zag movement of the flowing river can be compared to the zig zag movement of a snake. Just like the river water never flows in

231

qqq

These questions are for practice and their solutions are available at the end of the chapter

@PROCBSE

CHAPTER

9

THE TALE OF CUSTARD, THE DRAGON

– By Ogden Nash

Revision Notes Introduction

Key Words







1. Cowardly : lacking in courage. 2. Terrified: extremely frightened 3. Gobbled: ate hastily or greedily.







2.





‘The Tale of Custard, the Dragon’ is a funny poem that tells the story of a little girl Belinda and her pets. Belinda lives with her little pets – a kitten, a mouse, a dog and a dragon, in her little house. Belinda and all her pets except the dragon have been described as very brave creatures. But the dragon is so coward that it is always looking for the safety of a cage.











3.





This is a story of a girl Belinda, who had some strange pets. They had queer names. She called the black kitten ‘Ink’, the little grey mouse as Blink, the yellow dog as ‘Mustard’ and the cowardly dragon ‘Custard’. Belinda’s, kitten, mouse and dog were very brave but the dragon was a coward. One day, a pirate came armed with a pistol into their house. Except the dragon, Custard, all others were terrified of the pirate. Custard attacked the pirate and gobbled him up. Belinda then embraced him, while Mustard licked him. Even after this brave incident, Custard did not give up his cowardly ways. He still kept on crying for a safe cage.



Summary

Explanation of the Poem





4.



Belinda lived in a little white house, With a little black kitten and a little grey mouse, And a little yellow dog and a little red wagon, And a realio, trulio, little pet dragon. Explanation: In the first stanza of the poem, the characters of the poem are introduced. They are: Belinda, a girl, who lived in a little white house with her pets and a wagon. Her pets include a black kitten, a little grey mouse, a little yellow dog and a little dragon.













1.

Now the name of the little black kitten was Ink, And the little grey mouse, she called him Blink, And the little yellow dog was sharp as Mustard, But the dragon was a coward, Scan to know and she called him Custard. more about Explanation: Belinda called this topic her little black kitten as Ink and her little grey mouse was called Blink. The colour of the dog was as sharp and yellow as mustard. Her dragon was The Tale of called Custard. He was not Custard, brave like other pets. the Dragon Custard the dragon had big sharp teeth, And spikes on top of him and scales underneath, Mouth like a fireplace, chimney for a nose, And realio, trulio daggers on his toes. Explanation: This stanza describes the dragon, Custard’s physical appearance. He carried all the features of a dragon that could make him powerful, scary and brave. It had big sharp teeth and spikes on its back that could easily scare anyone away. Custard also had the ability to spit fire which made his mouth like a fireplace and thus his nose became a chimney. His toes were so pointed and sharp, that it could cut anything like a dagger. Belinda was as brave as a barrel full of bears, And Ink and Blink chased lions down the stairs, Mustard was as brave as a tiger in a rage, But Custard cried for a nice safe cage.

@PROCBSE













10.











11.





When Custard cried for a nice safe cage.

Suddenly, suddenly they heard a nasty sound, And Mustard growled, and they all looked around.



Meowh! cried Ink, and ooh! cried Belinda,





Pistol in his left hand, pistol in his right, And he held in his teeth a cutlass bright, His beard was black, one leg was wood; It was clear that the pirate meant no good.







13.



Explanation: The pirate looked very dangerous as he was carrying many weapons with him. He had pistol in both his hands. He also held a sharp blade between his teeth. He had black beard and a wooden leg. His looks and gesture made it quite clear that he meant to harm the house and the

The pirate gaped at Belinda’s dragon, And gulped some grog from his pocket flagon. He fired two bullets but they didn’t hit, And custard gobbled him every bit. Explanation: The pirate was shocked to see Belinda’s dragon. He gaped at him and drank some wine from a bottle that he carried in his pocket. The pirate then took out his pistol and fired two bullets at Custard. But he failed to hit him and Custard was unharmed. Then Custard swallowed the whole of the pirate, without leaving any trace of him. Belinda embraced him, Mustard licked him, No one mourned for his pirate victim. Ink and Blink in glee did gyrate Around the dragon that ate the pirate. Explanation: Everyone was surprised to see Custard eating the pirate. There was joy all over the house. Belinda embraced him to express her gratitude for saving everyone. Mustard licked the dragon to show his love. Everyone celebrated the death of the pirate. They were all happy. Ink and Blink started dancing around the dragon out of happiness.





12.





For there was a pirate, climbing in the winda. Explanation: When all the members were busy making fun of Custard, suddenly there came an unpleasant sound from the house. Mustard, the dog growled and looked around for the sound. Ink cried ‘Meowh’ and Belinda cried ‘Ooh’ when they all saw a pirate, climbing the window of the house to rob it.

Explanation: When Belinda looked at the pirate, she turned pale with fear and cried loudly for help. All her brave pets fled from the place. Mustard cried and ran away. Ink silently went to the bottom of the house and Blink very smartly disappeared in a mouse hole, leaving Belinda all alone. But up jumped Custard, snorting like an engine, Clashed his tail like irons in a dungeon, With a clatter and a clank and a jangling squirm, He went at the pirate like a robin at a worm. Explanation: When all the pets hid themselves, it was the coward dragon, Custard, who came to their rescue. He jumped in front of the pirate and made huge sounds like an engine, in anger. He moved his tail and made clattering sounds like an iron rod striking the walls of a prison in a dungeon. Making all these dangerous sounds, he followed the pirate like a robin who follows a worm to eat it.

Ink and Mustard rudely asked his age,









8.

Belinda paled, and she cried Help ! Help ! But Mustard fled with terrified yelp, Ink trickled down to the bottom of the household, And little mouse Blink strategically mousehold.

Belinda giggled till she shook the house,

Explanation: Belinda laughed a lot at the dragon. She laughed so loudly that it appeared that the house was shaking due to her laughter. Blink, the mouse said, ‘Week’ which is the giggling sound of the mouse. Ink and Mustard asked the dragon’s age to make fun of him as he always behaved like a kid. But Custard, the dragon, only wanted a nice safe cage for himself.

233

people living in it. 9.

And Blink said Weeck! which is giggling for a mouse,



7.

Explanation: This stanza describes all the members of the house. Belinda as brave as many bears combined together into one. Ink and Blink had the power to chase away someone as strong as a lion. The bravery of Belinda’s dog is compared to that of a tiger. But Custard was just opposite to his physical appearance. He was not at all brave, but he always looked for a nice safe cage. Belinda tickled him, she tickled him unmerciful, Ink, Blink and Mustard, they rudely called him Percival, They all sat laughing in the little red wagon At the realio, trulio, cowardly dragon. Explanation: Belinda tickled the dragon quite cruelly to make it laugh. The dragon was named Percival by Ink, Blink and Mustard. Seeing the dragon react to the tickling, made all of them laugh and they laughed sitting in the wagon at the cowardly dragon.



6.

THE TALE OF CUSTARD, THE DRAGON









5.







But presently up spoke little dog Mustard, I’d have been twice as brave if I hadn’t been flustered. And up spoke Ink and up spoke Blink, We’d have been three times as brave, we think, And Custard said, I quite agree

@PROCBSE 234

Oswaal CBSE Question Bank Chapterwise & Topicwise, ENGLISH LANG. & LIT., Class-X



Belinda is as brave as a barrel full of bears, And Ink and Blink chase lions down the stairs, Mustard is as brave as a tiger in a rage, But Custard keeps crying for a nice safe cage.









15.



Explanation: Same as stanza 4

Poetic Devices



















Simile : Mouth like a fireplace Belinda was as brave as a barrel Snorting like an engine Dashed his tail like irons in a dungeon Went at pirate like a robin at the worm. Repetition : Suddenly, suddenly they heard She cried Help! Help! Alliteration : And he held in his teeth.









14.







That everybody is braver than me. Explanation: After the pirate incident, it was quite clear who was brave and who was coward. But, Mustard gave an excuse for his cowardice and said that he would have been twice as brave as Custard if he had not got nervous; when the pirate came. Ink and Blink said that they would have been thrice as brave as the dragon. They started giving excuses for their running away. The humble dragon accepted that he was the most coward of all and everyone was braver than him. Belinda still lives in her little white house, With her little black kitten and her little grey mouse, And her little yellow dog and her little red wagon, And her realio, trulio little pet dragon. Explanation: Same as stanza 1

OBJECTIVE TYPE QUESTIONS Extract Based Questions













Q. 4. Select the option that fits with the reaction of the characters in the context of the extract. Ink : terrified : : Blink : (i) __________________ Pirate: (ii) ______________:: Custard: undaunted (A) (i) shocked (ii) displeased (B) (i) petrified (ii) wondered (C) (i) upset (ii) dazed (D) (i) petrified (ii) shocked Ans. Option (D) is correct. Explanation: Just as Ink was terrified, Blink appeared petrified or alarmed. Similarly, Pirate was dazed or amazed while Custard remained undaunted.







































Q. 3. Pick an option that best fits the usage of the word’ trickled’ as used in the extract. (A) The water trickled down the tap and filled the trough. (B) Students trickled into the classroom as the teacher entered. (C) Tears trickled down her cheeks as she heard the sad news. (D) His enthusiasm for the task slowly trickled away. Ans. Option (B) is correct. Explanation: In the poem, trickled has been used to refer to forming a line and entering some place.







Ans. Option (C) is correct. Q. 2. What is the poet’s purpose of using the onomatopoeic words given in the extract? (A) It is to emphasize on the might and boldness of Custard. (B) It is to introduce the character Custard to the readers. (C) It is to impress upon the readers that Custard was ready.

(D) It is to make Custard bold enough to face the situation. Ans. Option (A) is correct. Explanation: The words forming a sound with what it is named as or the onomatopoeic words have been used in the poem to highlight and attract attention towards the courage of Custard.



I. Read the following extract and answer the questions/ complete the sentences that follow: “Ink trickled down to the bottom of the household, And little mouse Blink strategically mouseholed. But up jumped Custard, snorting like an engine, Clashed his tail like irons in a dungeon, With a clatter and a clank and a jangling squirm, He went at the pirate like a robin at a worm” [CBSE QB, 2021] Q. 1. Which option lists the quotes that support the ideas in the extract? (1) Fear makes strangers of people who would be friends. (2) If you’re brave enough to start, you’re strong enough to finish. (3) Courage doesn’t mean you don’t get afraid. Courage means you don’t let fear stop you. (4) You get in life what you have the courage to ask for. (5) Fear has a large shadow, but he himself is strong. (A) 1 and 5 (B) 2, 3 and 4 (C) 2 and 3 (D) 1, 3 and 5 

















(5 marks each)

@PROCBSE THE TALE OF CUSTARD, THE DRAGON

B

Word

Meaning A.

to smile in a half-suppressed mocking way.

2. snigger

B.

to smile in an irritating, conceited manner.

3. smirk

C.

to let out a quiet and suppressed laugh.

D.

to let out a laugh, heartily and loudly.





1. chuckle













































III. Read the following extract and answer the questions/ complete the sentences that follow: Now the name of the little black kitten was Ink, And the little grey mouse, she called him Blink, And the little yellow dog was sharp as Mustard, But the dragon was a coward, and she called him Custard. [CBSE OD Set - III, 2019]



























(A) 1- D, 2-A, 3-C (B) 1-C, 2-A, 3-B (C) 1-B, 2-D, 3-C (D) 1-A, 2-C, 3-D Ans. Option (B) is correct. Q. 5. Which belief about dragons is in contrast to Custard’s behaviour in the extract? (A) Dragons are brave and feared. (B) Dragons can grant wishes. (C) Dragons can become invisible at will. (D) Dragons are soft-hearted and kind. Q. 6. A hyperbole is a literary device where the poet/writer/speaker purposely and obviously exaggerates to an extreme. Choose the option that includes an example of hyperbole, from the extract. (A) And Mustard growled, and they all looked around (B) Ink and Mustard rudely asked his age (C) Belinda giggled till she shook the house (D) When Custard cried for a nice safe cage. Ans. Option (C) is correct. Explanation: It is too much of an exaggeration that a small girl laughed so much that the house started shaking. So, this is an example of hyperbole.











Q. 2. A cage means captivity. Why is Custard inclined to remain in a cage despite what it symbolizes? This is so because he viewed it as a : (1) sanctuary (2) guardhouse (3) cubicle (4) refuge (5) booth

























A



















(A) 1, 2 & 5 (B) 1 & 4 (C) 3, 4 & 5 (D) 3 & 5 Ans. Option (B) is correct. Q. 3. Why has the poet used the word ‘weeck’ to signify the giggling of the mouse? The poet: (A) uses it to add suspense in the poem. (B) has imagined how the mouse would sound in this mood. (C) has mocked at the mouse for giggling at Custard. (D) uses it to create a scary effect for readers. Ans. Option (B) is correct. Q. 4. Pick the option with the correct matches for columns A and B.















Q. 7. The extract mentions ‘irons’ in dungeons. According to this extract, ‘irons’ is a reference to : (A) Iron racks for sacred books. (B) Iron cases housing treasures. (C) Iron chains holding the prisoners captive. (D) Iron coffins for burying the royal dead. II. Read the following extract and answer the questions/ complete the sentences that follow: Belinda giggled till she shook the house, And Blink said Weeck! which is giggling for a mouse, Ink and Mustard rudely asked his age, When Custard cried for a nice safe cage. Suddenly, suddenly they heard a nasty sound, And Mustard growled, and they all looked around. [CBSE QB, 2021] Q. 1. What can you infer from the repetition ‘suddenly, suddenly’ in the above extract? (A) There was an immediate change in the scene. (B) It focuses on the hasty attack and the loud noise. (C) It emphasises that an unexpected noise was heard. (D) It draws our attention to the loud cry that occurred. Ans. Option (C) is correct. Explanation: The repetition of the word ‘Suddenly’ indicates the fraction of second in which the event took place.

235











Q. 5. ‘He went at the pirate like a robin at a worm’. Why has this comparison been used here? Just like the robin catches the worm, (A) Custard attacked the pirate after careful observation. (B) Custard attacked the pirate without delay. (C) Custard attacked the pirate valorously. (D) Custard attacked the pirate stealthily. Q. 6. Pick the option that does not display a simile from the extract. (A) Clashed his tail like irons… (B) …at the pirate like a robin at a worm. (C) …Custard, snorting like an engine… (D) …trickled down to the bottom… Ans. Option (D) is correct. Explanation: Simile is when one thing is compared to another. ‘Tail like irons’, ‘pirate like a robin’ and ‘snorting like an engine’ are comparisons and thus, examples of similes. ‘Trickled down to bottom’ means flowing down to the bottom which is not a comparison and thus, not a simile.







These questions are for practice and their solutions are available at the end of the chapter

@PROCBSE



























































































































Q. 5. What do the words ‘realio, trulio’ mean? (A) Really and Truly (B) Real and True (C) Neither (A) nor (B) (D) Both (A) & (B) are correct Ans. Option (A) is correct.



































Q. 1. Who was tickled by Belinda? (A) Ink (C) Blink (C) Custard, the dragon (D) Mustard Ans. Option (C) is correct. Q. 2. Why did she tickle ‘him’? (A) To tease (B) Make fun of him (C) To help (D) Both (A) & (B) Ans. Option (D) is correct. Q. 3. Who are Ink, Blink and Mustard? (A) Ink-black kitten (B) Blink- grey mouse (C) Mustard- Yellow dog (D) All the above are correct Ans. Option (D) is correct. Q. 4. Why did they all laugh at ‘him’? (A) Because he was a joker (B) Because he was very huge (C) Because he was spiky (D) Because he was a coward Ans. Option (D) is correct. Explanation: They all had always heard that dragons are something to be afraid of. Custard was contrast to what they had heard about dragons. So, they all thought him to be a coward.



Q. 1. Name the poem and its poet. (A) The Tale of Custard, the Dragon-Ogden Nash (B) Fire and Ice-Robert Frost (C) How to Tell wild Animals-Carolyn Wells (D) The Ball Poem-John Berrymore Ans. Option (A) is correct. Q. 2. What is the colour of Belinda’s dog? (A) Yellow (B) Black (C) White (D) Brown Ans. Option (A) is correct. Q. 3. What were the kitten and the mouse called? (A) Blink and Ink (B) Custard and Pirate (C) Ink and Blink (D) Mustard and Blink Ans. Option (C) is correct. Q. 4. Which word in the stanza is the antonym of ‘dull’? (A) Black (B) Sharp (C) Coward (D) Called Ans. Option (B) is correct. Q. 5. Which of these are character traits of Custard? (A) Fierce looking (B) Coward (C) Both (A) and (B) (D) None of these Ans. Option (C) is correct. IV. Read the following extract and answer the questions/ complete the sentences that follow: (5×1=5) Belinda tickled him, she tickled him unmerciful, Ink, Blink and Mustard, they rudely called him Percival, They all sat laughing in the little red wagon At the realio, trulio, cowardly dragon. [CBSE All Sets, 2018 (Modified)]



Oswaal CBSE Question Bank Chapterwise & Topicwise, ENGLISH LANG. & LIT., Class-X





236

SUBJECTIVE TYPE QUESTIONS Short Answer Type Questions





These questions are for practice and their solutions are available at the end of the chapter

Expression— 1 mark when both aspects included  Answer organised effectively  usage of words linking ballad to evidence from the poem (similarly, just as etc.) ½ mark when either aspect is missing Deduct ½ mark from the overall score if the error density is high (more than a total of 2 spellings and grammatical errors). [CBSE Marking Scheme 2021] 







The question needs an answer identifying the features that match to the listed aspects of a ballad Content— Award 1 mark for full explanation of the two strands (Tale and Surprise ending). Award ½ mark for partial explanation.











Ans. Value Points • Tale: The poem tells the story of Custard, the dragon (setting, characters, rising action, climax, resolution). • portrays his life with Belinda and the other pets where he is considered a coward • Surprise ending: The end of the poem shows how Custard rose to the occasion; gobbled the pirate and proved his bravery. Guidance The question indicates that the examiner knows the elements of a ballad and that the poem has a few and hence that requires no elaboration.



















Q. 1. A ballad includes the telling of a tale as well as a surprise ending. Using evidence from the poem, explain how these features are included in ‘The Tale of Custard the Dragon’. [CBSE SQP, 2020-21]







(40-50 words & 3 marks each)

@PROCBSE

THE TALE OF CUSTARD, THE DRAGON































message. If the poem was written in any other style, do you think it would have created the same effect on readers? Justify your response with reason(s). [CBSE QB, 2020-21] Ans. Generally, ballads are tales of adventures and heroism. In the poem, the poet does present the encounter of custard and the pirate in a typical spirit of a ballad . Hence, no other style would have created the same effect on readers other than a ballad. Q. 5. Imagine that Custard writes a blog on ‘Overcoming fears’. A part of the blog focuses on ways to overcome the fears within. As Custard, write this part of the blog. You may proceed like this : While we agree that fear is like a big monster that resides within us. This monster shall continue to overpower us if we do not overpower it. To begin with …… [CBSE QB, 2020-21] Ans. The best thing to do is to examine the reasons behind the fears, ask yourself it is a caution or irrational fear and focus on the positive. All fears are worse when we face them alone. So find someone to share your fears with. Finally, face your fears. Accept failure as a part of life and get through it and over it.











Detailed Answer: The poem tells the story of Custard, the dragon. It also portrays the his life with Belinda and the other pets. Custard is made fun of because he is a coward However, there is surprising twist where custard rose to the occasion, gobbled up the pirate and proved his bravery. Q. 2. Custard humbly accepts that other animals are braver than him. Give a reason to support your stance that humility is a virtue worth possessing. [CBSE QB, 2020-21] Q. 3. Belinda shares a post on Instagram with the following hashtags. #Ink#Blink#Mustard#Bravehearts#My family#My world# She doesn’t mention Custard in her post. As a friend of Belinda, what would be your advice to her and why? [CBSE QB, 2020-21] Ans. As a friend, I would advice Belinda to be nice to custard. She should have made him feel comfortable and safe in the house and also give him equal importance instead of making fun of him. Because those who are inferior in strength and power should have no right to haunt and ridicule others. Q. 4. The poem is a light-hearted ballad with a powerful

Long Answer Type Questions



(100-120 words & 6 marks each) confident and continue bullying people. Such cases should be reported immediately. Nelson Mandela: I strongly believe that the people who are victim of bullying learn to handle these situations independently if they receive proper assistance. Therefore, a person should be taught to retaliate against the bully, and at the same time, they should be taught not to bully on the weaker people to show them the power. The oppressor should not suppress, and the victim should protest against this.













Q .2. Custard though mocked for his cowardice, displayed courage and was the one who saved the day in the end. Pick a character from any of the units (lessons/ poems) in First Flight, who displayed similar characteristics, like Custard and overcame a troubling issue? What similarities and differences do you find between the character and Custard? [CBSE QB, 2020-21]



Commonly Made Error













Q. 1. Custard was discriminated against, bullied and made fun of. Such behaviour often has a disastrous impact on the victims and etches a permanent scar on their minds. Write opinions of Wanda and Nelson Mandela on this issue. Wanda: In my personal opinion, ……………………………….(continue) Nelson Mandela: I strongly believe that…………………….(continue) [CBSE QB, 2020-21] Ans. Wanda: In my personal opinion, a person who is bullied regularly or has been bullied before, suffers huge mental trauma and physical trauma. Some people lose their mental steadiness due to these bullies. They also lose their confidence after getting bullied. They also lose their confidence after getting bullied either temporarily or permanently. Moreover, if the person who is bullying other people succeeds in doing so, then he will get

237

Many students got confused while mentioning the names of Belinda’s pets. As a result, they did not mention their names in their answers.

These questions are for practice and their solutions are available at the end of the chapter

@PROCBSE 238

Oswaal CBSE Question Bank Chapterwise & Topicwise, ENGLISH LANG. & LIT., Class-X





Answering Tip Attentive reading can help the students to write creative and authentic answers.

Solutions for Practice Questions Extract Based Questions





Short Answer Type Questions

Ans.2. Custard was actually very humble. He had a very low opinion of himself. Belinda and other animals pretended to be brave and inject mocked at Custard. But on seeing the pirate they behaved in a cowardly way.

Long Answer Type Questions

Ans.2. Custard, the dragon is portrayed as a timid dragon. He is also a humble dragon, pure in character. He never boasted about his bravery. Courage can be measured at the time of serious





I. Ans.5. Option (B) is correct. Ans.7. Option (C) is correct. II. Ans.5. Option (C) is correct. Explanation: Dragons are believed to be brave and need to be feared of. In reality, Custard was fearful of other pets and never seemed brave for a moment in the initial part of the poem.

situation. And Custard proved that he was the most courageous among all by not running away from the danger. According to me, the similar characteristics, like Custard can be seen in Wanda Petronski from the chapter ‘The Hundred Dresses’ written by Eleanor Ester Wanda was very shy and quite. She did not talk to anyone and hardly spoke anything in class or laughed. She quietly sat in the last row of the class with naughty boys so that nobody noticed her. Everybody teased her in class for her dress and shoes everyday. In anger, she claimed of having a hundred dresses and six pairs of shoes at home. She was very determined and humble. She showed her determination in drawing competitions by displaying a hundred sketches of dresses. She claimed to own. She possessed a great capacity for forgiveness and extending friendship. She, too, overcame a troubling issue like Custard, the dragon with humility.

Were you able to identify the underlying message of the poem, 'Physical appearance can often be deceptive'?





REFLECTION

qqq

@PROCBSE

CHAPTER

10

FOR ANNE GREGORY – By William Butler Yeats

Revision Notes



The poem is in a form of dialogue between two people, Anne Gregory and another is identified as speaker. It could be anyone i.e. Anne’s lover or a friend or the poet himself. The poem is about perception of love by different people.



Scan to know And not your yellow hair" more about Explanation: The speaker, this topic addressing Anne Gregory, says that her beautiful honeycoloured hair can make any man fall in love with her. This love is not for Anne but for her beautiful external For Anne Gregory features. Her beautiful hair is compared to wall, symbolising outer beauty. This beauty can capture any man’s attention. But he may not be able to look beyond that into Anne’s character. So the speaker says that no one can love Anne, for what she is. One can love her only for her beautiful yellow hair and her physical beauty. "But I can get a hair-dye And set such colour there, Brown or black, or carrot, That young men in despair May love me for myself alone And not my yellow hair" Explanation: In this stanza, Anne replies to the speaker that she can change the colour of her beautiful hair and dye them in black, brown or carrot. She wants to tell the speaker that anyone falling in love with her must see the actual person behind the beauty. She thinks that young men, who fall in love with her, must love her for what she is and not for her yellow hair.



Introduction

Summary













2.







In this poem, a young man discusses why a person falls in love, i.e. what determines it. According to them, a person is not loved for his basic nature but because of his/her physical feature, i.e., outward physical appearance. It is not possible to love one for oneself. Only God can do so. The poem ‘For Anne Gregory’ by W.B. Yeats is a poem of eighteen lines in all. In this poem, the poet addresses young Gregory and tells her that her hair is of the same colour as honey and when it falls, the poet begins to think of her beauty being spell bound. Her hair is so beautiful that every man falls in love with her. At this, Gregory gives response to the poet that man loves her only for her outward beauty while this outward appearance may change at any time. At this, the poet proclaims it a truth since time immemorial that man can not easily judge a woman other than her looks. He tells Anne that she can never be ugly inward or outward even if she wishes to be so.

Key Words



1. Proclaims: announce officially or publicly 2. Immemorial: ancient; very old 3.



"Never shall a young man, Thrown into despair By those great honey-coloured Ramparts at your ear, Love you for yourself alone











1.



Explanation of the Poem

‘‘I heard an old religious man But yesternight declare

@PROCBSE 240

Oswaal CBSE Question Bank Chapterwise & Topicwise, ENGLISH LANG. & LIT., Class-X

beauty. He means that humans do not have the insight and understanding to look into the soul of a person. They are swayed away by the glitter of outer beauty. Therefore, only God can love Anne only for herself and not for her beauty.



Poetic Devices



Metaphor : Honey coloured ramparts. Alliteration : Your yellow hair Rhyme Scheme : abcbdb













That he had found a text to prove That only God, my dear, Could love you for yourself alone And not your yellow hair.’’ Explanation: In the last stanza, the speaker replies to Anne about the importance of love for internal beauty and not the external one. The speaker talks about an old religious man, who announced that he had found a text in which it is written that only God is capable of looking beyond external

OBJECTIVE TYPE QUESTIONS Extract Based Questions



      





     





     































GET SET My home. My gym.

LOOK BOOK No game over, get makeover.

PLANET FOOD Eat! Eat! Eat! Repeat!



(B) Option (2)





LITE LIFE Food that makes you light.

(A) Option (1)

(D) Option (4)





(C) Option (3) Ans. Option (C) is correct.















(1) (2) (3) (4) (A) option 1 (B) option 2 (C) option 3 (D) option 4 Ans. Option (C) is correct. Q. 5. Choose the phrase that best represents, ‘love you for yourself alone”. (A) Unconditional love (B) Unseemly love (C) Unrequited love (D) Understated love Ans. Option (A) is correct. II. Read the following extract and answer the questions/ complete the sentences that follow: (5×1=5) But I can get a hair-dye And set such colour there, Brown, or black, or carrot, That young men in despair May love me for myself alone And not my yellow hair. [CBSE SQP, 2020-21] Q. 1. Given below are the tag lines of four hypothetical brands. Choose the correct option that fits the best with the first three lines of the given stanza.

















Q. 3. The above stanza strongly defies the idea of ___________ . (A) internal qualities (B) external beauty (C) spiritual self (D) divinity Ans. Option (B) is correct.



Nawaz



Joel

God loves you uncondi- Your time is limited so tionally, as you are and don’t waste it living not as you should be. someone else’s life. (A) Harpreet (B) Manav (C) Joel (D) Nawaz Ans. Option (C) is correct. Explanation: When everyone leaves your side, God is always there with you.



You can never meet Never apologize for your potential until you being yourself. truly learn to love yourself.



Manav



Harpreet



Q. 4. Choose the option that displays the likely image of the man mentioned in the extract.















































I. Read the following extract and answer the questions/ complete the sentences that follow: (5×1=5) ‘‘I heard an old religious man But yesternight declare That he had found a text to prove That only God, my dear, Could love you for yourself alone And not your yellow hair.’’ [CBSE SQP, 2020-21] Q. 1. Pick the word from the given extract which is a compound word. (A) Religious (B) Yesternight (C) Yellow (D) declare Ans. Option (B) is correct. Q. 2. Choose the option that lists the person who correctly wrote a quote best representing the central idea of the given stanza. Choose the correct option out of the ones given below.





(5 marks each)

@PROCBSE



And not my yellow hair [CBSE Delhi Set - 1, 2017] Q. 1. Who is speaking these line (A) Anne (B) The poet (C) The lover (D) The poet’s friend Ans. Option (A) is correct. Q. 2. Why are young men in despair ? (A) They are in love with the speaker (B) They want to win the love of the speaker (C) The speaker does not respond to their love (D) All the above are correct Ans. Option (D) is correct. Q. 3. What is the colour of the hair of the speaker ? (A) Brown (B) Black (C) Carrot (D) Yellow Ans. Option (D) is correct. Q. 4. How does the speaker want to be loved ? (A) for her external beauty (B) for herself alone (C) for her good looks (D) for her nature Ans. Option (B) is correct. Q. 5. What is the antonym of word ‘despair’? (A) happiness (B) sadness (C) grief (D) cheerfulness Ans. Option (D) is correct. IV. Read the following extract and answer the questions/ complete the sentences that follow: (5×1=5) “Never shall a young man thrown into despair By those great honey coloured Ramparts at your ear Love you for yourself alone And not your yellow hair” [CBSE OD Set - 1, 2017] Q. 1. Whom are these lines addressed to ? (A) You(reader) (B) Anne Gregory (C) Poet (D) Someone unknown Ans. Option (B) is correct. Q. 2. What would throw a young man into despair ? (A) beautiful dress (B) honey coloured hair (C) serious problem (D) carelessness Ans. Option (B) is correct. Q. 3. What is the rhyming scheme of the above stanza ? (A) abcbdb (B) abccdb (C) abcabc (D) aabbcc Ans. Option (A) is correct. Q. 4. Identify the poetic device used in line 3 of the above passage. (A) Alliteration (B) Metaphor (C) Personification (D) Simile Ans. Option (B) is correct. Explanation: The colour of Anne's hair is compared to that of honey.



















241

























Q. 2. The form of the given stanza is a part of (A) an agreement. (B) a disapproval. (C) an engagement. (D) an argument Ans. Option (D) is correct. Q. 3. Choose the option that describes a situation of ‘despair’ as mentioned in the extract.



FOR ANNE GREGORY







(2) Worked very hard and was once again, for the 3rd time, unable to get selected in the audition for the dance competition.

(3)

(4)

















































































Q. 5. Find the word in the passage which is ‘compared to wall’? (A) protected (B) covered (C) guarded (D) ramparts Ans. Option (D) is correct.





























      



     















     

(1) (2) (3) (4) (A) Option (1) (B) Option (2) (C) Option (3) (D) Option (4) Ans. Option (D) is correct. III. Read the following extract and answer the questions/ complete the sentences that follow: (5×1=5) But I can get a hair-dye And set such colour there, Brown or black, or carrot, That young men in despair May love me for myself alone















































































Worked very hard for Worked very hard to get the dance competition selected in the audition for audition. My best the dance competition, but friend got selected but the auditions got postponed. I didn’t. (A) Option (1) (B) Option (2) (C) Option (3) (D) Option (4) Ans. Option (B) is correct. Q. 4. Read the statements 1 and 2 given below and choose the option that correctly assesses these statements. 1– The speaker wants to change her hair colour so that it may appeal to young men. 2– The speaker’s appearance due to her gorgeous yellow hair is largely the reason young men hope to get her attention. (A) 1 is false, 2 is true and clearly inferred from the poem. (B) 1 is true, 2 cannot be clearly inferred from the poem. (C) 1 cannot be clearly inferred from the poem, 2 is false. (D) 1 is true and can be inferred from the poem, 2 is false. Ans. Option (A) is correct. Q. 5. Choose the product that best shows what Anne would invest in, to ensure that young men love her for herself and not her hair.















(1) Worked very hard and got selected in the audition for the dance. competition.

@PROCBSE 242

Oswaal CBSE Question Bank Chapterwise & Topicwise, ENGLISH LANG. & LIT., Class-X

SUBJECTIVE TYPE QUESTIONS Short Answer Type Questions



















“If only our eyes saw souls instead of bodies, how very different our ideals of beauty would be.” You may begin like this: The poet shares that God loves us unconditionally, for who we are and not our appearance. This quote…………………(continue)……………… [CBSE QB, 2020-21] Ans. The poet shares that God loves us unconditionally, for who we are and not our appearance. This quote is absolutely true. People are not objects. Appearances may be deceptive. A person should be liked and loved for being himself or herself. Outward appearances do tempt and dazzle us but it is temporary and often brings us dejection and gloom.

Q. 1. The “yellow hair” is a symbol in the poem. Briefly support the statement with reference to the poem “For Anne Gregory”. [CBSE QB, 2020-21] Ans. The ‘yellow hair’ is a symbol of outward physical beauty which is temporary. Shallow minded people adore only physical beauty we should look for spiritual beauty before falling in love with a lady. Physical beauty is just skin-deep and is momentary. Q. 2. Expound on any two ways in which Anne Gregory could practically put the poet persona’s advice to use. [CBSE QB, 2020-21] Q. 3. Evaluate the poem in the context of the given quote.





(40-50 words & 3 marks each)

Long Answer Type Questions



 



















These questions are for practice and their solutions are available at the end of the chapter

















Ans. • •





Q. 3. Imagine the poem were written for Anne Gregory’s brother, Miller Gregory. (a) Would that poem be different from the original poem? If so, highlight the differences it would have. (b) Pen down the advice, you think the poet persona would have given to Miller Gregory. [CBSE QB, 2020-21] Ans. (a) Females are considered more beautiful, delicate and praise worthy than males. So if the poem were written for Anne Gregory’s brother, Miller Gregory then it would be absolutely different from the original poem. Then instead of describing the physical beauty of Miller Gregory, perhaps the poet might have talked about his chivalry or courage. (b) Perhaps, the poet would have advised Miller Gregory to praise the inner qualities of his lady love instead of seeking for physical beauty. The poet would have suggested him to not get attracted by the outward appearance. It do tempt and dazzle us but he should know that it is temporary and deceptive. Only God can love a person for ‘himself of herself alone’. Q. 4. The poet in the poem, ‘For Anne Gregory’ conveys that we should give importance to the inner beauty and not to the physical appearance. Elaborate with reference to the poem. (CBSE Delhi Set-1 2019)





Q. 1. Imagine Anne Gregory grows up to be a motivational speaker. As Anne Gregory, write a short speech on the importance of loving oneself. You may begin this way: Dear friends The importance of believing in oneself can never be overstated. It becomes most important, therefore, to love oneself. I learnt this, as a young teen, the day a poet pointed out that…… [CBSE QB, 2020-21] (continue) ……… Ans. Dear friends The importance of believing in oneself can never be overstated. It becomes most important, therefore, to love oneself. I learnt this, as a young teen, the day a poet pointed out that this is a world of pomp and show. Things and people are often like and loved not because of their merit but because of their external appearances. There are many who may love me only for my yellow hair. However, I don’t like to be loved this way. If my lover likes me only for my beautiful hair, I won’t accept him. I should be loved for ‘myself alone. When you love all that you are, unconditionally life reflects that back to you. When you create a happy, loving environment to flourish in. Q. 2. Just as a love for outer appearances is not true love, love for the sake of money is not true love either. Pen down a dialogue exchange between Anne Gregory and any character from “The Proposal” where Anne tries to explain how loving others simply for who they are is the most important virtue that everyone must possess. [CBSE QB, 2020-21] 











(100-120 words & 6 marks each)

poet tries to show inner beauty is real beauty physical appearance changeable - hence, unimportant

@PROCBSE

FOR ANNE GREGORY



very rich. But now he has fallen from the position of riches. He loved his beloved, Anne Gregory very much. But it appeared that his love was superficial and he loved her for her blond hair. The girl was not happy at this. She wanted that her lover should love her for her inner beauty. She told him that she could dye her hair brown, black or carrot. Black hair is of Negroes in the English society, and brown and carrot hair is of the witches. In this way black, carrot or brown hair represent ugliness. The girl wanted that her lover should love her irrespective of the colour of her hair. Physical appearance is changeable and hence, unimportant.















first speaker tells Anne - young men love her for her beautiful yellow hair and may never love her for what she really is • Anne says her hair-colour can be changed into black, brown or carrot • implies external beauty is all superficial and men should not love her for that. • through Anne’s reply, the poet makes clear his preference for internal beauty to physical appearance [CBSE Marking Scheme 2019] Detailed Answer:

243

This poem is a sort of dialogue between a lover and his beloved, Anne Gregory. The lover had been once





Commonly Made Error Students failed to give precise answer to the questions related to the poem.





Answering Tip Students must try to understand what is asked in the question then answer it precisely.

Solutions for Practice Questions

Short Answer Type Questions





Ans.2. Firstly, Anne can dye her beautiful blonde hair the way she likes - brown, black, carrot or yellow. Secondly, instead of looking after her physical appearance and taking care of her outward appearance Anne can nurture her soul with spiritual care by devoting her self in God’s will.

Long Answer Type Questions





Ans.2. Anne : Dear, loving others simply for who they are is the most important virtue that everyone must possess. When we lose sight of what’s most important - loving one unconditionally - We lose sight of our goals and dreams and being happy and healthy.

Lomov : But, I can’t agree with you. Money is motivation. You can pun a smile on the people around you. Without money you can’t do that. If one is not rich then he is at a risk of getting depressed. His social reputation is at risk too. Money is a barrier from being happy to being sad. Money is more important than love. Anne : I can’t believe it. I strongly feel that money can never bring happiness. One can buy an expensive bed with money but sound sleep can’t be bought. Love can do it. Ultimately, to live a fulfilling and happy life, first and foremost, requires that you love all that you are and trust that life loves you in return.

Were you able to figure out the underlying message of the poem 'for Anne Gregory', that one must be loved for who he/she is and not for their outer appearance?





REFLECTION

qqq

@PROCBSE 244

Oswaal CBSE Question Bank Chapterwise & Topicwise, ENGLISH LANG. & LIT., Class-X

@PROCBSE

SUPPLEMENTARY READER CHAPTER

1

A TRIUMPH OF SURGERY

– By James Herriot

Revision Notes



Introduction

Scan to know more about Tricki could not even wag this topic his tail or kick with his legs. His eyes too had become dull and he lay down motionless on the carpet. The other dogs at the surgery ignored him as an uninteresting object. The doctor gave him plenty A Triumph of water. He became a bit of Surgery energetic after two days. On the third day, he trotted out and enjoyed the company of other dogs. He also started eating properly. The doctor did not give Tricki any medicinal treatment. All day, he frolicked with other dogs cheerfully. He got out of danger and started convalescing rapidly. Mrs. Pumphrey enquired about Tricki on phone many times a day. She was suffering from the pangs of his separation. Ultimately, she was told that Tricki had recovered. She reached the 'surgery' by car to collect him. The doctor brought Tricki in his arms. Tricki jumped into Mrs. Pumphrey’s lap and started licking her face. Tears were shining in her eyes. She thanked the doctor with trembling lips.



The story recounts the episode where the narrator, a veterinary surgeon, saves the life of a dog, Tricki, by hospitalising him to prevent him from being overfed by his wealthy owner, Mrs. Pumphrey, who was simply worsening his condition.



Key Words

1. tolled: to hang from mouth. 2. releated: to soften in temper 3. trolled : to move along briskly

Dr. Herriot had pulled up his car when he saw Mrs. Pumphrey with her little dog on the street. The dog was very fat. He had blood-red and rheumy eyes. His tongue lolled from his jaws. Dr. Herriot was shocked to see his appearance. Mrs. Pumphrey called Tricki (the little dog) as listless. He had no energy. So, she gave him extras between meals. But his health could not be built up. Even the malt, cod liver oil and bowl of Horlicks failed to make him sleepy. Mrs. Pumphrey cut down on the sweet things but relented when Tricki started getting weaker. She gave him his favourite cream cakes and chocolates. She gave him food at odd and irregular hours. She gave him only little walks and no other exercise. The doctor advised Mrs. Pumphrey to cut Tricki’s food right down and to give him more exercise. Mrs. Pumphrey got determined to adhere to the doctor’s stern advice. Dr. Herriot watched Tricki’s progress but was not satisfied. Soon after he left eating and kept vomiting and panting. The doctor suggested that Tricki should be kept under observation in the hospital. Mrs. Pumphrey started wailing when Dr. Herriot took Tricki to his hospital. Everybody in her household, including her maids, was in tears.













Summary

@PROCBSE 246

Oswaal CBSE Question Bank Chapterwise & Topicwise, ENGLISH LANG. & LIT., Class-X

OBJECTIVE TYPE QUESTIONS Extract Based Questions 

I. Read the following extract and answer the questions/complete the sentences that follow : The entire staff was roused and maids rushed in and out bringing his day bed, his night bed, favourite cushions, toys and rubber rings, breakfast bowl, lunch bowl, supper bowl. Realising that my car would never hold all the stuff, I started to drive away. As I moved off, Mrs. Pumphrey, with a despairing cry, threw an armful of the little coats through the window. I looked in the mirror before I turned the corner of the drive; everybody was in tears. Out on the road, I glanced down at the pathetic little animal gasping on the seat by my side. I patted the head and Tricki made a brave effort to wag his tail. “Poor old lad,” I said. “You haven’t a kick in you but I think I know a cure for you.” [CBSE-QB, 2021] (5 × 1 = 5) Q. 1. What might be the atmosphere of the household in the above extract signify? (A) Mrs. Pumphrey’s status in society reflected in Tricki’s lifestyle. (B) The staff’s love for Tricki, which matched that of Mrs. Pumphrey (C) The grand life of comforts and luxuries that Tricki enjoyed. (D) Mrs. Pumphrey’s indulgence and anxiety acted upon by the staff. Ans. Option (D) is correct. Explanation: The staff was highly influenced by the behaviour and reactions of Mrs. Pumphrey.























Q. 5. The narrator describes Tricki as a “pathetic little animal”. The use of the word ‘pathetic’ indicates that the narrator: (A) Was very fond of Tricki. (B) Thought Tricki was contemptible. (C) Pitied Tricki’s condition. (D) Believed Tricki’s health was deteriorating. Ans. Option (C) is correct. Explanation: Pathetic is used to describe someone who is not in good condition and needs sympathy or pity.















































Q. 3. Given below are some well-known quotes shared by the staff to console Mrs. Pumphrey, after Tricki’s departure. Choose the option that correctly identifies the quote that IS NOT appropriate to the consolation offered.

















Q. 2. Given below are emoticons reflecting various expressions and reactions. Choose the option that correctly describes the narrator’s mindset in the given extract.

(A) Options (i) and (iii) (B) Options (ii) and (iv) (C) Options (iii) and (v) (D) Options (ii) and (v) Ans. Option (B) is correct. Explanation: The narrator is confused and undecided. These are represented by emoticons (ii) and (iv).

(i) (ii) (iii) (iv) (A) Option (i) (B) Option (ii) (D) Option (iv) (C) Option (iii) Ans. Option (D) is correct. Q. 4. As the extract indicates, Mrs. Pumphrey indulged Tricki and bought him many things. Choose the option that best describes the kinds of advertisement/s that seem likely to persuade Mrs. Pumphrey to buy something for Tricki. (i) Statistics Appeal – Such advertisements use facts and data to convince consumers to buy products. (ii) Scarcity Appeal – Such advertisements create a feeling of exclusivity and are often used to convince people to take advantage of a sale or limited period offer. (iii) Personal Appeal – Such advertisements focus on evoking emotions to convince consumers and often relate to family or other interpersonal interactions. (iv) Fear Appeal – Such advertisements focus on inspiring some kind of fear to convince consumers to take action in order to avoid certain negative or undesirable consequences. (A) Options (i), (ii) and (iv) (B) Options (iii) and (iv) (C) Options (i), (iii) and (iv) (D) Option (ii) only Ans. Option (B) is correct. Explanation: Mrs. Pumphrey is likely to purchase anything which she is afraid of or evoke emotions. She is not going to be influenced by scarcity appeal or statistical data.















(5 marks each)

@PROCBSE A TRIUMPH OF SURGERY

(B) To mention the simple pleasures of canine life.

(C) To compare it to Tricki’s earlier play-time at the house.

Ans. Option (B) is correct.





(D) To direct attention towards Tricki’s successful recovery.

Q. 4. “All the while, Mrs. Pumphrey hovered anxiously in the background”. Given below are different types of Pet Parenting styles described in Country Living, (an e-magazine).





Select the option that captures the essence of the extract correctly.











Explanation: Although these are treated as pathetic in human life, but in canine behaviour, these are normal routines or pleasures that need to be enjoyed.

Q. 1. Read the following statements, each of which describes the gist of the given extract.

Choose the option that best reflects the kind of Pet owner Mrs. Pumphrey was.





Statement (i) – It highlights the kind of comforts and luxuries that Tricki was used to at Home. Statement (ii) – It brings out a contrast between Tricki and Mrs. Pumphrey’s state of being.

(i) Traffic Light Pet owners have a healthy balance of rules and freedom and give clear and consistent signals for ‘yes’ and ‘no’.



247



II. He discovered the joys of being bowled over, tramped on and squashed every few minutes. He became an accepted member of the gang, an unlikely, silky little object among the shaggy crew, fighting like a tiger for his share at meal times and hunting rats in the old hen house at night. He had never had such a time in his life. All the while, Mrs. Pumphrey hovered anxiously in the background, ringing a dozen times a day for the latest bulletins. [CBSE-QB, 2021] (5 × 1 = 5)







Statement (iii) – It reflects that Tricki was happier at the surgery and loved being with other dogs.

(ii) Entranced Pet owners have the best intentions, but as soon as their pet locks eyes with them and gives their command, they are at their pet’s beck and call.



Statement (iv) – It shows Tricki’s journey with his peers at the surgery and documents of his recovery.

(A) Statements (i) and (ii)

(B) Statements (iii) and (iv)

(iii) The Goose pet owners go all-out in protecting their pet. They often limit their time away from their pet, especially puppies.





(C) Statements (i) and (iii)



Ans. Option (D) is correct. Explanation: Statement (i) is not correct as Tricki found having meal at Doctor’s place more satisfying than at Home. Statement (iii) is not correct as Tricki enjoyed his company more than others initially. So, only Statements (ii) and (iv) are correct.



(iv) The Baggage Handler pet owners love being close to their pets and going on adventures together. They are always mindful of the pet’s comfort and security.







(2) happy

(1) selfish

Ans. Option (C) is correct.









Q. 5. Pick the option that reveals Tricki’s characteristics in the context of ‘fighting like a tiger for his share at meal times and hunting rats in the old hen house at night.’



(6) cruel

Q. 3. Why does the narrator describe being “tramped on and squashed” as joys?

(C) 1 and 5











(B) Only 2 (D) 3, 4 and 6



Ans. Option (A) is correct.







(A) 2, 4 and 5



(A) To suggest the irony about the strange ways of dogs.



(4) confident

(5) sturdy



(3) greedy

Explanation: Silky here refers to his well grooming and little refers to his delicateness living a life of luxury.







Explanation: Mrs. Pumphrey wanted the best for her pet even though it appears to be strict, but as soon as she looks at her pet, she forgets all about it and becomes lenient.

(D) The Narrator’s mockery of Tricki’s life and treatment with Mrs. Pumphrey.







Ans. Option (B) is correct.

(B) Tricki was comfortably attired in fine silks and warm coats. (C) Unlike the other dogs, Tricki had lived, in the lap of luxury with care and grooming.

(D) Option (iv)





(A) Tricki was a very small and rather pampered dog.

(B) Option (ii)



(C) Option (iii)





(A) Option (i)

Q. 2. What does the reference to Tricki as a “silky little object” signify?







(D) Statements (ii) and (iv)

@PROCBSE 248

Oswaal CBSE Question Bank Chapterwise & Topicwise, ENGLISH LANG. & LIT., Class-X

SUBJECTIVE TYPE QUESTIONS Short Answer Type Questions



(40-50 words & 3 marks each)















































Detailed Answer: It was important for Mrs. Pumphrey to be a little cruel towards Tricki because he was being overfed which impacted his health. She needed to cut down the food intake of Tricki. It was the only way to help him regain his health. Q. 5. According to a popular quote— Where there is no struggle, there is no strength. In what way is this quote relevant to the events of the story? [CBSE-QB, 2021] Ans. Strength gets accumulated after facing struggles in one’s life. If the person does not face any challenges in his life, he will remain weak forever. In the same way, when Mrs. Pumphrey had to stay away for a long time, she understood true worth of Tricki and good health for him.



















– she was overfeeding Tricki; being overindulgent – she needed to cut down Tricki’s food intake to help him regain his health (be cruel to be kind) (CBSE Marking Scheme, 2020)



Ans.

Q. 6. ‘Privilege often confuses actual needs for perceived ones.’ Evaluate this statement with reference to Mrs. Pumphrey. [CBSE-QB, 2021] Ans. Mrs. Pumphrey loved her dog too much. She treated him like her own son. She could never refuse to give Tricki whatever he demanded for. In fact, she herself had made assumptions about Tricki’s likes and dislikes so she couldn’t understand that Tricki needed controlled diet rather that sweets and drinks. Q. 7. Imagine Mrs. Pumphrey came to know how Tricki was actually treated. Write a brief note to the narrator on her behalf. You may begin this way: Dear Mr. Herriot I recently met Mrs. Mallard when she brought her Pixie to visit dear Tricki. I spoke to her about the great triumph of surgery that had brought him back to me. Imagine my surprise when she told me how mistaken I was! I am writing to you to share……(continue)……... [CBSE-QB, 2021] Ans. ……………. I am writing to you to share my feelings regarding Tricki’s recovery. When dear Tricki came back from the hospital, he was transformed into a flexible, hard muscled animal. He was stretching out in great bounds. But, I was really left heart broken when Mrs. Mallard informed me that Tricki was given no food for two days. Any ways, I must thank you Dr. Herriot for doing what was needful for Tricki and curing him. Q. 8. Why is Mrs. Pumphrey responsible for Tricki’s condition? (CBSE Delhi Set 2, 2020) Q. 9. What was the major flaw in Tricki ? [CBSE OD Set-1, 2020] Ans. Mrs. Pumphrey had a dog named Tricki. Indirectly, she was responsible for the dog’s illness. She gave him little extra between meals i.e., malt, cod-liver oil, etc. He was not given any physical exercise. So, the dog fell ill and started vomiting also. It made Mrs. Pumphrey so much worried that she had to call the veterinary surgeon, Dr. Herriot for his treatment.



















Q. 1. What do you think would happen to Tricki after he went back to Mrs. Pumphrey? [CBSE-QB, 2021] Ans. After he went back to Mrs. Pumphrey, Tricki was very happy. As a result, Mrs. Pumphrey must have taken good care of the dog and must not have overfed it. So that Tricki would stay as healthy as at the end of the story. Q. 2. “I was really worried about Tricki this time.” Comment on the writer’s choice of beginning the story in this manner. What purpose does it serve? [CBSE-QB, 2021] Ans. The above line shows the narrator’s concern about Tricki when the narrator saw the dog after a long time he was really in bad health. And when he asked Mrs. Pumphrey about his diet and exercises, she made excuses and was not following it. Q. 3. Do you think the narrator ’s decision to not reveal the actual treatment to Mrs. Pumphrey was unprofessional? Justify your stance. [CBSE-QB, 2021] Q. 4. Why was it important for Mrs. Pumphrey to be a little cruel in order to be kind towards Tricki? [CBSE SQP, 2020]

Long Answer Type Questions



These questions are for practice and their solutions are available at the end of the chapter







Ans. (a) I am sad to leave you people and go back to Mrs. Pumphrey’s house. It’s not that I don’t love her. I really love her a lot and also care for her affection and care. But I miss the company which I got her. There I keep lying on my couch and keep eating the delicacies which







Q. 1. Imagine that one of Mr. Herriot’s partners can understand the language of dogs and listens to Tricki on his last night with them (a) What might Tricki share about his experience? (b) How would he evaluate it in comparison to his home experience? [CBSE-QB, 2021]









(100-120 words & 6 marks each)

@PROCBSE A TRIUMPH OF SURGERY



















Students keep on rambling and do not write the required points in the answer.

Answering Tip

Short Answer Type Questions





Ans.3. No, the narrator was not all unprofessional rather he was highly practical. He knew every well that it would be very difficult for Mrs. Pumphrey to hear about Tricki’s treatment at the hospital. At the same time, it was necessary to save Tricki. Ans.8. Mrs. Pumphrey had a dog named Tricki. Indirectly, she was responsible for the dog’s illness. She gave him little extra meals i.e. malt, cod-liver oil, etc. He was even fed with desserts like cream cakes and chocolates. He was not given any physical exercise. So the dog fell ill and started vomiting also. He had become hugely fat, like a bloated sausage. Dr. Harriot, the veterinary surgeon had to be called for his treatment.

Long Answer Type Questions Responsible pet ownership Owing a pet is a privilege, but the benefit of pet ownership come with responsibilities. Pets bring companionship, personality and humor in the



Ans.2.





Commonly Made Error

Students must focus on the points that are to be written in the answer.

Solutions for Practice Questions (Topic-1)





249

Pumphrey. All those materialistic playthings …. [CBSE-QB, 2021] Ans. 12 May, 1950, Friday Walt’s poem Animals has compelled me to think again about my experience with Tricki and Mrs. Pumphrey. All those materialistic playthings and uncontrolled food couldn’t help Tricki to retain his health. When Mrs. Pumphrey found that Tricki was no more active and energetic, she tried to give more nutrients in his food to make him more energetic. But she was not giving him exercises. In fact, Tricki was not suffering from any disease. Poor animal couldn’t resist to eat food and used to have meal at any hour of the day or night. As a result, he had become glutton. But he didn’t complain about his condition. Nor did he complain when he was kept starving for two days in my hospital. The same wonderful and insightful message has been given by Walt Whitman in his poem Animals. He says animals are a more content and wiser than the humans. They are calm, placid and self-contained.









are provided to me. I long for my friends. I too want to run, play and exercise like them. This is the reason I’m unwilling to go back to Mrs. Pumphrey’s house. (b) I received great care here in the hospital. The first two days I missed the food that I got in my house. But I actually, felt lighter the third day. I loved joining my friends and their friendly barking, fighting with them for share of food and hunting rats at night. You know, food, good bedding and best care can never replace the feeling of freedom that I experienced here in the hospital. Q. 2. Mrs. Pumphrey and Dr. Herriot have been invited to speak at a community pet adoption drive. There were some differences in Mrs. Pumphrey and Dr. Herriot’s notions of responsibility and experiences of keeping a pet. As a reporter for the local pet magazine, write an article recording and comparing their perspectives. Don’t forget to give it an interesting [CBSE-QB, 2021] title. Q. 3. Imagine that Walt Whitman shares his poem ‘Animals’ in response to Dr. Herriot’s narration of Tricki’s story. Dr. Herriot reflects on the poem and his experience with Tricki and Mrs. Pumphrey, draws a connection between them and records his thoughts in his diary. Write the diary entry as Dr. Herriot. You may begin this way: 12 May 1950, Friday 9.30 pm Walt’s poem has compelled me to think again about my experience with Tricki and Mrs. 











These questions are for practice and their solutions are available at the end of the chapter

household. Having a pet shows you responsible because they need full time and care. Animals, too, need food, shelter, attention and love just the ways humans do. Coming home every day to the wagging tail and happy pet after a long day is an awesome feeling. Your pet senses if you are sad, happy or just need to be comforted. In the same way, it is our responsibility as the pet owner to take care of the physical, mental and emotional health of our pets. Only feeding them with delicacies and providing them comfortable bed is not enough. Time to time medical consultation is very much important. Not only this, but the doctors advise also needs to be followed strictly. But the emotional and mental health of our pets is as important as their physical health. Becoming responsible, compassionate and having full time protection are the three positive outcomes of owning a pet. Many positive experiences are to be had in being a pet owner.

@PROCBSE 250

Oswaal CBSE Question Bank Chapterwise & Topicwise, ENGLISH LANG. & LIT., Class-X







REFLECTIONS Did you realize that the dog, Tricki was cured without the dose of medicines? Were you able to observe that sometimes the blind love which we possess for the one whom we love the most can be harmful?

@PROCBSE

CHAPTER

2

THE THIEF’S STORY

– By Ruskin Bond

Revision Notes Scan to know more about this topic



Introduction

sleep. He took the money and slipped out of the room. He had six hundred rupees in his possession. He reached the railway station just as the Lucknow Express was picking up speed. He could have The Thief’s story boarded it but something held him back. The train was soon out of his sight. Hari Singh was left alone on the platform. He was in a dilemma where to go. He had no acquaintance there. He did not like to go to a hotel lest he should be suspected. The thief thought that Anil would be sad not for the loss of money but for the loss of trust he had reposed in him. It was a cold night. It was drizzling when he went to the maidan and sat down on a bench. His clothes got drenched. He went back to the bazaar. There he took shelter under the clock tower. It was midnight. He thought and realized that he had let go a chance of being a respectable man. Anil was taking pains to teach him reading and writing. He decided to go back to Anil and reached his home. Anil was still asleep. He slipped the money at its usual place. He slept till late hours. Anil had made tea before Hari Singh woke up. Anil offered him a fifty rupee note. He also promised him to pay him regularly. The note was still wet. Anil understood everything but he did not expose the thief ’s doing. Instead, Anil said that they would start writing sentences that day. Hari Singh felt guilty and smiled shame-facedly.

‘The Thief ’s Story’ by Ruskin Bond is about a fifteen year old boy, Hari Singh, whose life changes when he meets Anil, a twenty five year old writer. Anil’s unspoken words and kind gestures leave a positive imprint on Hari Singh who is an artful thief.



Key Words

1. acquaintance: fellowship, intimacy 2. temptation: attractive, appealing 3. drenched: completely wet

Hari Singh is the narrator of this story. He was an artful thief. One day, he met a young man of twenty-five named Anil, at a wrestling match. He gave him an artificial smile. He desired to have an acquaintance with him in order to make him his next victim. Anil avoided him. Hari Singh followed Anil after the match was over. He asked Anil for work. He was ready to work for Anil if he could just feed him. Anil agreed to engage Hari Singh if he could cook food for both. The thief lied that he was a good cook as Anil had to throw the bad meal, prepared by Hari Singh to dogs, on the first day. Anil took upon himself to teach him how to cook and read and how to write. The thief made the morning tea and bought the daily supplies. Anil was aware of the fact that Hari Singh made a profit from the purchases. Anil was not a rich man. He borrowed money sometimes but repaid the loan whenever he earned money through his writings for magazines. The thief thought it a strange way of making money. One day, Anil sold a book to a publisher and brought home a bundle of notes. He put the money under his mattress. The thief had not stolen anything since a month of his working there. He had no desire to rob a careless person who trusted him blindly. However, he could not resist the temptation of getting the bundles of notes. The thief got up quietly when Anil had gone to











Summary

@PROCBSE 252

Oswaal CBSE Question Bank Chapterwise & Topicwise, ENGLISH LANG. & LIT., Class-X

Extract Based Questions













II. I think he knew I made a little money this way but he did not seem to mind. Anil made money by fits and starts. He would borrow one week, lend the next. He kept worrying about his next cheque, but as soon as it arrived, he would go out and celebrate. It seems he wrote for magazines—a queer way to make a living! [CBSE-QB, 2021] (5 × 1 = 5) Q. 1. Anil made money ‘by fits and starts’ means that he: (A) Deemed it fit to start investing money. (B) Started earning money in the recent past. (C) Received money intermittently. (D) Put his money to use frequently. Ans. Option (C) is correct. Explanation: Intermittently means irregularly which is the same as the proverb ‘by fits and starts’.











Q. 3. If borrow: :lend, then pick the ODD pair from the options below: (A) Give : : take (B) Lose : : find (C) Hop : : skip (D) Buy : : sell Ans. Option (C) is correct. Explanation: borrow or lend work as synonyms. Give and take, Lose and find, Buy and sell are antonyms. Hop and skip are the same ways of expressing a method of walking.













































Q. 2. The information in the extract suggests that Anil could be a: (A) Salaried professional (B) Freelancer (C) Business man (D) Volunteer Ans. Option (B) is correct. Explanation: Anil used to work for anyone who is a source of income to him. So, he is a Freelancer.







Q. 3. ‘I might be able to get into the young man’s confidence.’ Choose the option that DOES NOT display what the statement means. (A) He wanted to win his trust. (B) He wanted him to share his thoughts without caution. (C) He wanted him to feel comfortable revealing more details about himself. (D) He wanted to be able to spend quality time with him. Ans. Option (D) is correct. Q. 4. Anil looked easy-going, kind and simple to the Narrator. Which of the given characteristics would NOT fit in with this description? (A) Compassionate (B) Suave (C) Uncomplicated (D) Carefree Ans. Option (B) is correct. Explanation: Easy going, kind and simple reflect that Anil was a smooth character. The word Suave means smooth.



















































Q. 2. ‘I hadn’t had much luck of late’ means that the boy hadn’t: (A) Ever conned people successfully. (B) Been successful in duping people lately. (C) Understood the consequences of Thievery till date. (D) Considered the role of fate in deceiving others. Ans. Option (B) is correct. Explanation: The Boy was not talking about some winnings. He was just trying to say that he had not been able to dupe anyone that night in carefully wrapped words.

Q. 5. Based on the line, “A little flattery helps in making friends.”, choose the option that displays the quote closest in meaning. (A) Imitation is the best form of flattery; people generally understand that my comedy is not intended to hurt anybody. (B) I know imitation is the highest form of flattery, but stealing one’s identity is totally different. (C) Nothing is so great an example of bad manners as flattery. If you flatter all the company, you please none; if you flatter only one or two, you offend the rest. (D) One may define flattery as a base companionship which is most advantageous to the flatterer. Ans. Option (D) is correct. Explanation: Flattery can be in any form and differs from one person to another. One person may be flattered by one thing and another person may be flattered by some other thing. So, flattery forms only the base of companionship which benefits the flatterer.



I. Anil was watching a Wrestling Match when I approached him. He was about 25- a tall, lean fellow – and he looked easy-going, kind and simple enough for my purpose. I hadn’t had much luck of late and thought I might be able to get into the young man’s confidence. ‘You look a bit of a wrestler yourself,” I said. A little flattery helps in making friends. [CBSE-QB, 2021] (5 × 1 = 5) Q. 1. According to the extract, the young boy was watching the Wrestling Match because he: (A) Had been invited there by the Wrestlers. (B) Was supposed to meet someone there. (C) Was looking for simple people to dupe. (D) Loved wrestling and followed it very closely. Ans. Option (C) is correct. Explanation: The young boy used to make fool of simple people who trusted him too much. So, he was keenly looking at the match so that someone approaches him and he can dupe that simple person.





(5 marks each)

@PROCBSE THE THIEF’S STORY

(A) So what if I don’t have much money? Giving it to that person is important as they could do with a helping hand. (B) I better learn how to protect my money. I think I’m being looted. (C) I earn money with such tremendous effort. Where does it all go? (D) When I become rich, I can begin to help friends then. Right now, I will spend only on myself. Ans. Option (A) is correct.



























253



Q. 4. The reference to making a little money ‘this way’ refers to a way that is viewed by most people as: (A) Sensible (B) Inappropriate (C) Charitable (D) Aggressive Ans. Option (C) is correct. Explanation: The method of earning money by Anil is considered as charitable for those who help him. Q. 5. Based on your understanding of Anil in the extract, choose the option that synchronises with his thinking.





SUBJECTIVE TYPE QUESTIONS Short Answer Type Questions

Ans. Hari Singh came back to Anil because he remembered the good times he had spent with Anil. His inner conscience stopped him from breaking Anil’s trust. Hari Singh stole six hundred rupees while Anil was sleeping. He went to the railway station after stealing the money, but did not board the train. His conscience pricked him for robbing a man like Anil who had trusted him. He roamed around aimlessly and remembered the good times he had spent with Anil. He felt lonely and sad at the thought of leaving Anil. So, he immediately decided to go back to Anil. Hari Singh knew that one day he will be able to earn more money by becoming a really big, clever and respected man which was much more than stealing a few hundred rupees at that time. So he returned home, back to Anil. Q. 5. Why did not Anil hand Hari Singh over to the police ? (CBSE OD Set 3 2019)





Q. 1. It’s not easy to be mean to people who are very nice to you. Do you think that the young thief had to work on giving himself valid reasons to be able to commit the theft? Or did the thieving come as naturally to him this time, too? Write a brief analysis of his mood [CBSE-QB, 2021] before the theft. Ans. No doubt, Hari Singh was experienced and clever enough to justify his actions. He justified the robbing of careless and unsuspecting man like Anil. He told himself that he was out of practice. He must rob. If he didn’t take the money, Anil would waste it on his friends, Moreover Anil didn’t pay him. Q. 2. Why do you think the narrator used to feel that ‘friends were more trouble than help’? What could have led him to believe this? [CBSE-QB, 2021] Ans. Hari Singh was a robber. If he and his friend went to rob something and one of his friends would be caught then he could reveal their secrets in such case the whole gang would be handed over to police for robbery. Moreover he did not want anybody to know about his secrets and whereabouts. So he felt friends were more trouble than help. Q. 3. At the end of the story, we realise that Anil knew about the incident of the previous night, yet he chose to overlook such a breach of trust. Explain. [CBSE-QB, 2021]



















Q. 1. Based on the given context, write a diary entry, as the narrator, on what you experienced and the possible reason for it. I should have been able to jump into one of the carriages, but I hesitated — for some reason I can’t explain — and I lost the chance to get away. Ans. I should have been able to jump into one of the







Long Answer Type Questions















Ans. Anil forgave Hari Singh to give him an opportunity to reform himself. 2 × 4=8 [CBSE Marking Scheme 2019] Detailed Answer : Anil was a very kind person. He knew that Hari Singh was not bad at heart. If he had handed over him to the police, Hari Singh would have become a hardened criminal and his mind would be filled with bitterness. So, Anil did not hand over Hari Singh to the police. Instead, he forgave him and gave him an opportunity to reform himself. 2 Q. 6. In which queer way did Anil make a living? (CBSE Delhi 10D, 2018)





Q. 4. What made Hari Singh come back to Anil? [CBSE Delhi Set-I, 2020]

























(40-50 words & 3 marks each)

These questions are for practice and their solutions are available at the end of the chapter

(100-120 words & 6 marks each) carriages, but I hesitated— for some reason I can’t explain— and I lost the chance to get away perhaps God has some good plans for me this time. That’s why I can’t feel the pleasure of stealing six hundreds rupees. Instead, I’m feeling guilty of having violated the trust that Anil had reposted on me. That’s why I could not board the train.

@PROCBSE Oswaal CBSE Question Bank Chapterwise & Topicwise, ENGLISH LANG. & LIT., Class-X





















Commonly Made Error

Students get confused in the names of characters and thus write wrong answers.

Answering Tip A thorough reading of the chapter must be done to get a clear picture of all the characters. Sometimes simply, reading the solved question and answers can also be of great help.





Q. 4.

despite knowing that the money had been taken (dampness of notes) Anil chose to remain quiet about the matter – pretended that all was normal- realized that Hari Singh’s honesty had prevailed over his temptation – wanted to reward him/ encourage himdiscussing the theft would have been counterproductive. (5 marks) [CBSE Marking Scheme 2020] Detailed Answer : Anil was a large-hearted person he realized, from the dampness of notes, that Hari Singh had tried to steal the money but since he came back and kept the money back at its place, made Anil realize that Hari Singh was sorry for his act. So, he decided to remain quiet about the matter. He knew that the discussion of the theft would be counterproductive. He pretended that all was normal and nothing had happened. Anil decided to give him another chance as it dawned upon him that Hari Singh’s honesty had prevailed over his temptation. He didn’t hand him over to the police and ‘neither his lips nor his eyes showed anything’.





Ans.

Ans. –





Q. 3.



Solutions for Practice Questions Long Answer Type Questions

Ans.3. Anil was careless and unsuspecting but not a fool. He knew all about the theft of his six hundred rupees. He could have easily handed him over to police. However, he did not want to ruin Hari Singh’s life. He wants to educate the unfortunate boy. So, he gave a fifty rupee note and promised to pay him regularly. Also he chose to overlook such a breach of trust. Ans.6. Wrote for magazines, made money by fits and starts. (CBSE Marking Scheme, 2018) Detailed Answer : Anil made a living in a queer way by borrowing money but repaid the loan whenever he earned money through his writings for magazines. He lived life according to the flow of money. When he got money, he enjoyed out with his friends. When he was having less money, he used to reduce his expenses.

Ans.2. Hari Singh, ultimately, realized the significance of education in life. He understood the robbing would only provide him money temporarily. Also there was the risk of being caught. Education provides one with respectability. Education can heal a wrongly talented soul. Educated people may see the difference between right and wrong and enables them to choose the right path. It leads them on the path of truth and rational thinking. Education could bring one respect and money. After robbing Anil, the boy thief realized the value of education that could give him respect and more money than he could get by stealing. He realized where he should go. Large hearted Anil not only forgave him but also gave him a fifty rupee note, and promised to pay him regularly. Thus, education is an essential aspect of any child’s life and can help enable an honourable way of living.











Short Answer Type Questions

These questions are for practice and their solutions are available at the end of the chapter











Anil always treated me like his friend. I too, valued Anil’s friendship feelings for me. But after discovering the theft, I’m sure, Anil would not be sad for the loss of money, but for loss of trust. The only person I knew and who could help me was Anil whom I had robbed a few hours ago. I robbed that person who was so large-hearted and the one who wanted me to get educated so that I, too, could earn respect, name and money. Alas! God, forbid me and my sins. Education means different things to different people. Evaluate how it impacted the young thief in the story. Education, love and sympathy can transform even a thief. How is it true in the case of Hari Singh? For some people, deception is a way to lead life easily, i.e., by changing name, by making excuses, by making new way of making money etc. Anil was the most trusting person. Hari Singh had ever met. But he took undue advantage of his trust. He was a thief and couldn't be expected to be honest with Anil. He came to Anil with a false identity. Anil had a blind faith in human nature. He employed him just out of sympathy and compassion for him. He trusted him and ignored his misdeed. He handed over all the financial transactions without any verification. In a way, he gave him a chance to improve. Hari stole money and then came back with a change of mind. Anil knew even this fact also, but said nothing. It made Hari Singh to decide to leave the style of deception as a way of life. Living a simple and straight life over-powered Hari's deceptive life. Finally, he opted for the simple life by educating himself and becoming a clever, big and respectable man in future. It was very good decision of Hari Singh. Hari Singh says- “He knew it but neither his lips nor his eyes showed anything.” Anil successfully addressed the situation by not addressing it at all. Elaborate based on your reading of The Thief’s Story.



Q. 2.







254

@PROCBSE

THE THIEF’S STORY

255





REFLECTION Were you able to understand the fact that how love and kindness can transform even the most destructive minds of criminals and thieves in this cruel world?

PROCBSE THANK YOU FOR DOWNLOADING BOOK. WE HERE AT PROCBSE MAKE SURE THAT EVERY PIECE OF MATERIAL REACHES YOU WITHOUT DIVING DEEP INTO THE TABS OF YOUR BROWSER. THIS E-BOOK HAS BEEN DOWNLOADED TOTALLY FREE FROM PROCBSE ON TELEGRAM. UPI ID - PROCBSE.FAMC@IDFCBANK

CLICK TO VIEW/OPEN

@ PROCBSE JOIN US FOR QUESTIONS BANK , SAMPLE PAPER , PYQ , NOTES , MIND MAPS AND MANY MOTE THINGS ⚡

@PROCBSE

You can contribute a little to our efforts. Even a small amount can do wonders.

DONATE US PLEASE HELP US

OR

CONTRIBUTE TO OUR UPI ID PROCBSE.FAMC@IDFCBANK

@PROCBSE

CHAPTER

3

THE MIDNIGHT VISITOR – By Robert Arthur

Revision Notes Scan to know more about this topic

Ausable, a secret agent, is expecting to get a very important report. Another secret agent, Max, threatens him with a pistol, demanding the report. This story tells how Ausable outwits Max by making up convincing stories.

his window. The management had not still blocked it off and intruders could get in from the apartment. Max asked Fowler to sit down, rather than stand stiffly for half an hour till the report arrived. The Midnight Ausable was at a loss to find Visitor how Max had learnt about the incoming report. Just then, there was a knock at the door. Fowler jumped at it. Ausable concocted another story that the police might have come to provide him security for the important paper. Ausable told Max that the police would enter the room any way and shoot if he did not open the door. Max got unnerved. Max got infuriated and started backing swiftly towards the window. He opened the window with one hand behind. He wanted to escape through the balcony. He informed Ausable that he would wait on the balcony. He would shoot and take his chances if Ausable did not send the police away. The knocking became louder. Somebody called Ausable repeatedly. Max dropped to the balcony, screamed loudly and died. A waiter entered the room with a bottle and two glasses set on a tray with the drink ordered by Ausable. Fowler was perplexed. Ausable explained that he was waiting for the waiter and not for the police. Fowler was afraid lest Max should shoot them from the balcony. Ausable assured Fowler that Max would never return.





Introduction



Key Words







1. 2. 3. 4. 5.



Ausable was a secret agent in an organisation. He was a fat man with a strange accent. He was an American and could speak German and French fluently. He lived in a small room on the top floor of a French hotel in Paris. Fowler was a young and romantic writer. One day, he visited Ausable in the hope of finding some unusual happenings there. He was also a spy and dealt in espionage and danger. However, he felt disappointed at Ausable’s sight. Ausable and Fowler spent the evening in a French music hall. Fowler was feeling bored. Ausable informed him that he would soon receive an important paper. The paper might affect the course of history. It had also made many men and women risk their lives. He had got a telephone call making an appointment in his room. Ausable switched on the light after closing the door. There was a man with a pistol standing half way across the room. Fowler was thrilled. Ausable was surprised to see Max (another secret agent belonging to another agent’s organisation) in his room. On being asked, Max informed him that he had come there at an odd hour to take from him the prospective report about some new missiles. Ausable sat in an armchair in a serious and angry mood. He started saying that it was the second time in a month that somebody had got into his room through a balcony. Max had no knowledge about the balcony. He reported that he had a pass key. Ausable explained that the balcony belonged to the next apartment which used to be the living room. The balcony extended under













Summary

accent : modulation of voice in speaking espionage: spying Concocted: invented, made up infuriated: extremely angry perplexed: confused

@PROCBSE

Extract Based Questions



(5 marks each)

















































Q. 5. Choose the option that lists the INCORRECT statements about the given passage. 1. Fowler had no expectations at all from Ausable. 2. Fowler was elated when he met Ausable. 3. Ausable was living in France as a spy. 4. Ausable had acquired the French accent over twenty years ago.















Q. 5. Which word replaces ‘sat down heavily’ in the sentence—’Ausable moved to an armchair and sat down heavily’? (A) dumped (B) flumped



































(A) Option 1 (B) Option 2 (C) Option 3 (D) Option 4 Ans. Option (D) is correct. Explanation: Loud music, most often, late at night can disturb anyone's sleep. Hence it was enough to ‘raise the devil’ with Delnaaz.





















3. She works late at 4. She plays music night and stays home louder than is necin the afternoon only. essary late into the night.





















2. She feeds the stray dogs in the street.

















Q. 2. Choose the option that displays the correct analogy. (A) crafty : fox :: menacing : gun (B) report : missile :: management : devil (C) crafty : fox :: management : balcony (D) armchair : grim :: balcony : nuisance Ans. Option (A) is correct. Q. 3. Choose the part of the house through which Ausable thought Maxwell had entered his room. (A) Main Door (B) Balcony (C) Window (D) None of these Ans. Option (B) is correct. Q. 4. Delnaaz is a new tenant in her locality and Mrs. Mehta is the landlady. Choose the option that lists an action that would require Mrs. Mehta to ‘raise the devil’ with Delnaaz.

(C) pumped (D) jumped Ans. Option (B) is correct. II. Ausable did not fit any description of a secret agent Fowler had ever read. Following him down the musty corridor of the gloomy French hotel where Ausable had a room, Fowler felt let down. It was a small room, on the sixth and top floor, and scarcely a setting for a romantic adventure. Ausable was, for one thing, fat. Very fat. And then there was his accent. Though he spoke French and German passably, he had never altogether lost the American accent he had brought to Paris from Boston twenty years ago. “You are disappointed,” Ausable said wheezily over his shoulder. [CBSE-QB, 2021] (5 × 1 = 5) Q. 1. “Following him down the musty corridor of the gloomy French hotel where Ausable had a room…” Choose the correct option to replace the underlined words. (A) expansive, abnormal (B) lighted; formal (C) damp; dingy (D) uncanny; smart Ans. Option (C) is correct. Q. 2. Amy is an American woman who teaches French and German in a primary school. This means that Amy is (A) an empathetic teacher. (B) a rational human being. (C) an experienced guide. (D) a multilingual instructor. Ans. Option (D) is correct. Q. 3. It can be inferred from the extract that a secret agent is expected to be (1) fit. (2) compassionate. (3) apologetic. (4) suave. (5) violent. (A) Option 1, 3 & 5 (B) Option 1 & 4 (C) Option 2, 4 & 5 (D) Option 2 & 3 Ans. Option (B) is correct. Q. 4. The reason Fowler was disappointed by Ausable when he met him was because Ausable (A) lived in a shady French hotel. (B) was related to the cops. (C) didn’t have the personality of a spy. (D) had a very strange accent. Ans. Option (C) is correct. Explanation: Ausable was a fat and sloppy fellow.















I. Max was slender, a little less than tall, with features that suggested slightly the crafty, pointed countenance of a fox. There was about him — aside from the gun — nothing especially menacing. “The report,” he murmured. “The report that is being brought to you tonight concerning some new missiles. I thought I would take it from you. It will be safer in my hands than in yours.” Ausable moved to an armchair and sat down heavily. “I’m going to raise the devil with the management this time, and you can bet on it,” he said grimly. “This is the second time in a month that somebody has got into my room through that nuisance of a balcony!” [CBSE-QB, 2021] (5 × 1 = 5) Q. 1. Owning a gun gave a menacing impression to Max’s character because (A) a gun is only used by police officials. (B) he was carrying it without any official license. (C) a gun has a threatening implication. (D) the gun was particularly huge. Ans. Option (C) is correct. Explanation: Max's further actions were totally influenced by the presence of that gun.

1. She uses the elevator five times a day.



257

THE MIDNIGHT VISITOR

@PROCBSE 258































































































































































Q. 5. Which word in the extract means the same as ‘genuine/real’? (A) Thrill (B) Authentic (C) Automatic (D) None of the above Ans. Option (B) is correct. IV. Max bit his lip nervously. The knocking was repeated. “What will you do now, Max ?” Ausable asked. “If I do not answer the door, they will enter anyway. The door is unlocked. And they will not hesitate to shoot.” [CBSE Delhi Set-1, 2020] (5 × 1 = 5) Q. 1. Who is Max ? (A) a spy (B) a tourist (C) a businessman (D) a waiter Ans. Option (A) is correct. Q. 2. Max became nervous because: (A) he got scared of Ausable. (B) he thought the police had come. (C) the door was unlocked. (D) Ausable did not get up to open the door. Ans. Option (B) is correct. Q. 3. Actually the door was knocked at by __________. (A) Henry, the waiter (B) The Hotel Manager (C) The Police (D) Ausable’s assistant Ans. Option (A) is correct. Q. 4. Who was the third person present in the room? (A) Max (B) Ausable (C) Fowler (D) Henry Ans. Option (C) is correct. Q. 5. Which word in the extract is opposite in meaning to this word ‘confidently’? (A) Bit (B) Nervously (C) Hesitate (D) Anyway Ans. Option (B) is correct.



















(A) 1, 2 (B) 3, 4 (C) 1, 3 (D) 2, 4 Ans. Option (D) is correct. III. And as the light came on, Fowler had his first authentic thrill of the day. For halfway across the room, a small automatic pistol in his hand, stood a man. Ausable blinked a few times. [CBSE Delhi Set-1, 2020] (5 × 1 = 5) Q. 1. Who was standing in the room with a pistol in his hand? (A) Ausable (B) Fowler (C) Max (D) A waiter Ans. Option (C) is correct. Q. 2. Ausable blinked because he (A) was getting adjusted to the light (B) got afraid of the man with a pistol (C) was thrilled to have reached his room (D) started thinking of how to get rid of the man Ans. Option (A) is correct. Q. 3. What was the thrill of the day for Fowler? (A) Coming of the light (B) Opening of the main door (C) Seeing a man with an automatic pistol in his hand (D) Ausable taking out a gun from his pocked. Ans. Option (C) is correct. Q. 4. How did Fowler really feel before entering the room? (A) excited (B) entertained (C) relaxed (D) boring Ans. Option (D) is correct. Explanation: Fowler felt boning before entering the room as he was disappointed to meet Ausable. Ausable was absolutely different type of a secret agent than Fowler had imagined of.



Oswaal CBSE Question Bank Chapterwise & Topicwise, ENGLISH LANG. & LIT., Class-X

SUBJECTIVE TYPE QUESTIONS Short Answer Type Questions



























Q. 3. Henry unintentionally saved Ausable’s life. Briefly comment. [ [CBSE-QB, 2021] Ans. Henry was a waiter in the same hotel in which Ausable was living in. Henry, fortunately, knocked at the door, when Ausable was convincing max about an imaginary balcony. Assuming that it was police at the door, Max desperately jumped through the window. Hence, Ausable was saved. Q. 4. Fowler’s day took a one-eighty degree turn by the end of the evening. Examine the statement with reference to “The Midnight Visitor”. [CBSE-QB, 2021] Ans. Fowler was, initially, disappointed to meet Ausable as he found him fat and boring man. But Fowler’s day took a one eighty degree turn by the end of the evening, as he was much surprised by the tactful nature of Ausable. He cocked-up a story about 





Q. 1. Discuss, briefly, the reasons owing to which Fowler was interested in meeting Ausable. [CBSE-QB, 2021] Ans. Fowler had come to gather necessary material for his book on secret agents. Also, like all people, he too was interest in meeting a real secret agent. He had formed quite a romantic picture of secret agent as represented in books and films. Q. 2. Ausable was always one step ahead of everyone. Highlight any two instances from “The Midnight Visitor” when this holds true. [ [CBSE-QB, 2021] Ans. Although, Ausable was fat and sloppy, he knew all the tricks of his trade. He not only convinced crafty Max that there was a balcony below the window of his room but also was successful in killing Max without using any weapon.





(40-50 words & 3 marks each)

@PROCBSE

















Q. 6. Max was unprepared for his mission. Discuss with reference to the chapter “The Midnight [CBSE-QB, 2021] Visitor”. Q. 7. How is Ausable different from other secret agents? [CBSE QD Set-I, 2020] Ans. Fowler was disappointed on seeing Ausable who was unlike other secret agents. According to him, the secret agents are tall, smart, agile, dashing and alert people surrounded by darkness, pistols and drugs in the wine. On the contrary, Ausable was fat was fat and sloppy with heavy American accent. He was not at all flashy or romantic like other agent.

Long Answer Type Questions

(100-120 words & 6 marks each)



Ausable has my enthusiastic recommendation. He is a kind, intelligent, quick-witted and a strong person who has a clear sense of direction, purpose and decision-making. I am confident that he will bring the same warmth, support, insight and hard work to his new work place and position. Please do not hesitate to contact me for any further information. It’s a privilege to have such devoted employees like Ausable in our department wishing him a bright future ahead. Yours sincerely ABC Q. 2. A new secret agent has asked Ausable’s opinion on the traits they should be looking for in a person filling in his shoes. Create a conversation between Ausable and the new agent about indispensable traits that a secret agent must have. [CBSE-QB, 2021] Ans. New agent: Monsieur Ausable, you have been famous as a committed employee in our department. I have also heard a lot about your quick wiliness, intelligence and hard work. But when I saw you I was really surprised to see your simple way of living. Please guide me by throwing some light on the traits a secret agent must have. Ausable: Well! Being a secret agent may not match the glamorous and flashy images from movies and television. Spying is often a low-key, tireless and thankless endeavour. However, it is certainly not for those who lack commitment or the joint of heart. One needs to develop a number of practical and risk- a verse characteristics for spying. As a spy, you need to have keen senses and develop them further through training in how to collect and evaluate vital information. Also, you must have a natural adaptable and high functioning ability to interact with others. While doing so, it is essential that you keep your composure and be able to work independently. You see appearances are deceptive. You need to have a clever mind and be witty to become a successful secret agent.



These questions are for practice and their solutions are available at the end of the chapter



















Q. 1. Ausable’s employer has been asked to write a recommendation letter for Ausable so that he may be promoted to a higher rank in the secret services. As his manager, draft the letter detailing his personality traits with reference to the instance in the chapter as a prime example of Ausable’s genius. You may begin this way: 22, Rue Nationale Paris 26 March’ 78 The General Director Department for Internal Security 11-A, Rue de Dunkerque Paris Subject: Letter of Recommendation This is to state that Monsieur Ausable has been a committed employee with us for the past two years. In this time, he has … … … … … … … … … … … … (continue)………………………… Yours sincerely ABC [CBSE-QB, 2021] Ans. 22, Rue National Paris. 26 March 78 The General Director Department of Internal Security 11-A, Rue de Dunkerque Paris Subject : Letter of Recommendation This is to state that Monsieur Ausable has been a committed employee with us for the past two years. In this time, he has very well proved that presence of mind and intelligence are more powerful than gun. His sharp reaction, particularly to a dangerous situation, is very quick and thorough. He has outwitted and defeated many rival secret agents who had come to take away the important reports.

259



the balcony and the police. Ausable also got the intruder, Max, killed without firing a shot. Hence it proved to be a thrilling episode. Q. 5. “Stereotypes are often misleading”. Elaborate on the given quote in the context of “The Midnight Visitor”. [ [CBSE-QB, 2021] Ans. Ausable did not fit any description of a secret agent. He did not seem to be so smart, active, movable aggressive and romantic as we read in book or see in films. He was very fat and sloppy. The Crack of pistols, wine and dark eyed beauties- had nothing to do with Ausable. Hence, stereotypes are often misleading.



THE MIDNIGHT VISITOR







@PROCBSE 260

Oswaal CBSE Question Bank Chapterwise & Topicwise, ENGLISH LANG. & LIT., Class-X









Commonly Made Error Students usually get confused between Max and Fowler. Many students did not even know who was Henry in this chapter.





Answering Tip Students must read the text and not depend on the summary of the chapter.

Solutions for Practice Questions Short Answer Type Questions

Ans.6. Though Max was very cunning and he came prepared with a gun but he was indeed no match to Ausable. He was easily misled by Ausable into

believing that there was a balcony attached to the room. So we can say that Max was a little spy before Ausable and he was also unprepared for his mission.





REFLECTION Stereotypes are often misleading. Do you agree that with the fact that wisdom is more powerful than other weapons?

@PROCBSE

CHAPTER

4

A QUESTION OF TRUST

– By Victor Canning

Revision Notes



Introduction

Scan to know see a doctor if he was serious more about about his work. His sneezing this topic had brought her down from the top of the house. She thanked her stars that she had returned in time not expecting to see a burglar in the house. To avoid trouble, Horace A Question started conversing with her in of trust a pleasant manner. He replied that he too had not expected to find her there. She got furious when he asked her to let him go. She threatened to get him arrested. The thought of going to prison made Horace shudder. He pledged that he would never repeat such kind of thing if he was let off. The woman took a cigarette from a silver box. Horace took off his gloves and held his lighter towards her. She promised to let him go if he did something for her. Horace readily consented to the proposal. The woman said that she had come to take her jewels as she had to attend a party. She had forgotten the numbers to open the safe. She requested Horace to open it. She would get the safe repaired. Horace broke the safe open and gave her the jewels. Then, he was allowed to go. Horace’s fingerprints were found on the safe. He was arrested for the jewel robbery on the third day. The house lady was an aged woman of sixty. He realised that he was befooled by a lady who was also a thief.

Horace Danby collects rare books, and finances his hobby by breaking safes. This year, however, he is caught by a pretty young lady claiming to be the wife of the house owner. She tricks him into opening the safe and handing her the jewels. Horace is later arrested for the jewel robbery.



Key Words

1. Pacified : brought to peace 2. posed : presented 3. shudder : tremble with fear

Horace Danby was generally considered to be a good and respectable citizen. He was a fifty year old bachelor. He lived with a housekeeper. He was an expert lock maker. However, he was not completely honest. He had been to prison, fifteen years ago for stealing jewels. He had no desire to become honest. He was careful lest he should run into difficulties for his dishonesty. Horace Danby hated prison. He was heartily fond of rare and expensive books. He used to commit one theft a year on an average. The stolen money lasted for the year. He chose the house at Shotover Grange to commit his next burglary. He studied the design of the house carefully for a fortnight. He found that the family had gone to London and the servants to the movie. He broke into the house one afternoon. He hoped to get fifteen thousand pounds worth of jewels from the safe. The jewellery would enable him to buy the books he wanted. He put on his gloves. He took the key from its usual place. He pacified the pet dog by addressing it with love. He reached the safe, placed in the drawing room behind a painting. He was suffering from hay fever. The smell of the flowers at the table made him sneeze. Just then, a young beautiful woman in red clothes entered the house. She posed to be the mistress of the house. She suggested him that he should









Summary

@PROCBSE 262

Oswaal CBSE Question Bank Chapterwise & Topicwise, ENGLISH LANG. & LIT., Class-X

Extract Based Questions



































     









      







Q. 3. Choose the option that lists evidence that the police might have used against Horace, to arrest him.

     







      





Q. 4. Given below are four situations in Mr. Verma’s house. Choose the situation that depicts Mr. Verma being sharp-tongued with his family members. (A) Mr. Verma’s wife shares a life problem with him and he advices her honestly. (B) Mr. Verma’s daughter scores poor marks in his exams and Mr. Verma remains silent.





























(i) (ii) (iii) (iv) (A) Option (i) (B) Option (ii) (C) Option (iii) (D) Option (iv) Ans. Option (D) is correct. Explanation: Horace's fingerprints were found all over the room by the police.      















     

(i) (ii) (iii) (iv) (A) Option (i) (B) Option (ii) (C) Option (iii) (D) Option (iv) Ans. Option (C) is correct. Q. 4. In the line – Before he could think, Horace said, “Hay fever,” –the response was (A) rehearsed. (B) spontaneous and natural. (C) hesitant. (D) rapid and hostile. Ans. Option (B) is correct. Q. 5. The line from the extract that can be considered an example of sarcasm is (A) How foolish people are when they own valuable things. (B) Horace found that the flowers were hindering him in his work. (C) You can cure it with a special treatment. (D) I think you’d better see a doctor, if you’re serious about your work. Ans. Option (D) is correct. Explanation: The lady was sarcastic in her talk.









































II. But he never got the chance to begin his plan. By noon a policeman had arrested him for the jewel robbery at Shoot over Grange. His fingerprints, for he had opened the safe without gloves, were all over the room, and no one believed him when he said that the wife of the owner of the house had asked him to open the safe for her. The wife herself, a grey-haired, sharp-tongued woman of sixty, said that the story was nonsense. Horace is now the assistant librarian in the prison. He often thinks of the charming, clever young lady who was in the same profession as he was, and who tricked him. He gets very angry when anyone talks about ‘honour among thieves’. [CBSE-QB, 2021] (5 × 1 = 5) Q. 1. The plan devised by Horace was to (A) tell about the safe to owner of the house. (B) blackmail the wife of the owner later. (C) look for another safe for stealing jewellery. (D) tell police the truth before the lady could deceive him. Ans. Option (C) is correct. Q. 2. Based on the extract, choose what you think are the main feelings Horace probably has, for the young lady who tricked him, when he thinks about her? (A) anger and blame (B) admiration and respect (C) respect and gratitude (D) anger and vengeance Ans. Option (D) is correct.

I. How foolish people are when they own valuable things, Horace thought. A magazine article had described this house, giving a plan of all the rooms and a picture of this room. The writer had even mentioned that the painting hid a safe! But Horace found that the flowers were hindering him in his work. He buried his face in his handkerchief. Then he heard a voice say from the doorway, “What is it? A cold or hay fever?” Before he could think, Horace said, “Hay fever,” and found himself sneezing again. The voice went on, “You can cure it with a special treatment, you know, if you find out just what plant gives you the disease. I think you’d better see a doctor, if you’re serious about your work. I heard you from the top of the house just now.” [CBSE-QB, 2021] (5 × 1 = 5) Q. 1. According to the extract, Horace was sneezing due to a: (A) sudden allergy (B) prior infection (C) long-standing disease (D) nasal pain Ans. Option (B) is correct. Q. 2. Which magazine could have published the article mentioned in the given extract? (A) Paws & Claws (B) Outdoors & Landscaping (C) Interiors (D) The Investor Ans. Option (C) is correct. Q. 3. Choose the image that correctly describes the location of the safe, based on the given extract.





(5 marks each)

@PROCBSE A QUESTION OF TRUST















Q. 5. Honour among thieves is an example of an (A) idiom (B) slogan (C) proverb (D) lyric Ans. Option (A) is correct.



263









(C) Mr. Verma’s son cooks food for the first time and he is criticized by his father immediately. (D) Mr. Verma’s sister buys a new car to surprise him and he is overjoyed with the news. Ans. Option (C) is correct.





SUBJECTIVE TYPE QUESTIONS Short Answer Type Questions



Ans.













Q. 6.







Q. 7.



Q. 8.











Ans.





Q. 9.





[CBSE Delhi Set, 2019]



These questions are for practice and their solutions are available at the end of the chapter





Ans. Had flourishing business of locks; lived a happy life; had two helpers; earned well; was respectable but used to rob a safe every year for sustenance; proves he was not completely honest. [Board Marking Scheme] Detailed Answer : Horace Danby was a lock maker. He had a flourishing business of locks. He lived a happy life







































elderly couple he would immediately have caught the lady in red. Thus we can say that Horace acted foolishly at some points. “The most successful people work smart, not hard” Justify the quote with reference to “A Question of Trust”. [CBSE-QB, 2021] The quote ‘the most successful people work smart not hard’ is apt for the woman who duped Horace Danby not even once could Horace make out that she was not the wife of the house owner. The woman threatened to hand him over to the police and made him break the safe. Meanwhile she kept standing away smoking a cigarette and watched him do all work. There was no trace or fingerprint of that lady in the house. She framed a story of forgetting the numbers and Horace fell into her trap. Horace opened the safe gave her the jewels and went away. Thus without working hard, she got all the jewels, with her smartness. Based on the story, would it be fair to say that deception is a crucial aspect of robbery. Do you [CBSE-QB, 2021] agree? Why/why not? Thieves and swindlers have no sense of moral righteousness. Comment with references to instances from “A Question of Trust”. [CBSE-QB, 2021] Thieves and swindles work for their own benefit and are devoid of all feelings and emotions. They have no sense of moral righteousness. This is true in the case of the young lady in red who didn’t think even once that Horace would be caught by the police without getting any share of the robbery. She walked away with all the jewels, her conscience didn’t prick her that he was equal partner in the robbery. It was only Horace who was arrested and not the lady who actually got all the jewels. How can you say that Horace Danby was good and respectable but not completely honest ?



Q. 1. The woman posing as house owner’s wife had laid out a careful plan to dupe Horace Danby. Cite any two instances from the text that suggest the [CBSE-QB, 2021] same. Ans. The woman posing as house owner’s wife was a young and smart thief. She spoke to Horace in an authoritative tone and carried herself with confidence. She had laid out a careful plan to dupe. Horace is evident from the fact that she stayed calm even after seeing a thief in the house. She fabricated a story about forgetting the number to open safe also conveys this. Q. 2. In the chapter “A Question of Trust”, Horace Danby’s final arrest was a blessing in disguise. [CBSE-QB, 2021] Comment briefly. Ans. Horace Danby’s final arrest was blessing in disguise for the lady in red as she got a good fortune in form of jewellery without working hard for it. At the same time, she could escape with the jewellery unnoticed because everyone’s focus was on the other thief whose finger prints were found on the safe. Q. 3. “Society must be protected from men like you.” Comment on the irony of the given quote. [CBSE-QB, 2021] Ans. The words “Society must be protected from men like you” are spoken by the young lady in red and are ironical because she herself was a thief. She duped Horace Danby robbed the house and walked away with the jewellery kept in the safe. Q. 4. “A Question of Trust” is a story about the robbery of the robber. Expound. [CBSE-QB, 2021] Q. 5. “How foolish people are when they own valuable things” Does this statement hold true for Horace Danby himself? Support your answer with instances from “A Question of Trust”. [CBSE-QB, 2021] Ans. Horace Danby was a meticulous planner but he faltered when he was to rob the safe or Shot Ever Grange. These were about fifteen thousand pounds worth of jewels in the safe. But he acted foolishly and was tricked by a young lady who posed herself to be the owner of the house. Horace should have realized that the magazine article had described the house and many others would also have read it. He should have come prepared for it. Secondly, he had collected less information about the owner and his family e.g. the age of owners wife. If he had known that the owner and his wife were an





(40-50 words & 3 marks each)

@PROCBSE 264

Oswaal CBSE Question Bank Chapterwise & Topicwise, ENGLISH LANG. & LIT., Class-X























Long Answer Type Questions



















These questions are for practice and their solutions are available at the end of the chapter





Detailed Answer : Horace was not a typical thief. He used to rob every year enough money to last for twelve months to buy rare and expensive books which he loved to read. Also, being an introvert, he didn’t blurt about his theft to anybody. Even then we can’t call him a good human being. He is not completely honest. Though, he didn’t use his money in bad deeds but



Ans. Value Points : (a) Acceptance (b) Trust and courage (c) Absence of conflicts (d) Belief in self (CBSE Marking Scheme, 2016) 4

















Its D-day finally! Today I don't know whether to feel nervous or excited or both at the same time. I am nervous as I am afraid whether my robbery plan is fool-proof or not. I am well aware that Horace Danby too is a meticulous planner. He also must have planned it so well that it comes out as a successful robbery. Duping Danby won’t be an easy job. On the other hand, I am excited too because I might be the owner of jewels worth fifteen thousand pounds. Oh my God! This really would mean a dream come true God knows how fond am I of jewels and getting them all, without paying a single penny would be something that I can never imagine. Hope the plan turns out to successful. I am keeping my fingers crossed till then. XYZ Q. 3. Horace was not a typical thief. What values would you like to inculcate in him to transform him? Write in about 80-100 words. (CBSE 2016) 4





























Q. 1. Imagine that the young lady thief writes a letter to Horace Danby while he is in prison. Write the letter imagining youself to be that young lady with respect to the story. Ans. XYZ city 14 July 19xx Dear Horace You must be surprised to receive my letter as you might have never thought to hear from me after the robbery. I deeply regret the consequences of the robbery. I had not expected that you would be arrested. I really feel bad for you. I am angry with myself also for having cheated you but I wanted the jewels so I had to trap you. I kept thinking about the incident for the next two days and at one stage, decided to share the jewels with you. But you may say it my self-centredness took the better of me and I didn’t gather myself to make you my partner. I really am sorry but feel helpless too. Hope we will meet some day. Your sincerely ABC Q. 2. As the woman who dupes Danby, pen a diary entry for the night before the robbery. Pen down your thoughts, fears, and hopes from the robbery based on the events of the story "The Question of Trust". Wednesday, 12 July '78 9 PM Its D-day, finally! Today, I know how it feels to be nervous and excited at the same time……. (continue)………. Ans. Wednesday 12 July 1978 9pm Dear Diary





(100-120 words & 6 marks each)





Ans. She was a young and smart thief. She was very graceful, resourceful and impressive. She spoke to Horace in an authoritative voice as if she was the mistress of the house. She was very intelligent.





Q. 12.

Ans. Horace was arrested because his finger prints were all around the safe since he had opened the safe bare-handed. He told that the landlady had asked him to open the safe. No one believed his story. The real lady of the house was an old lady of sixty. The young lady was another thief. [CBSE Marking Scheme, 2016] Q. 14. How did Danby prepare for the robbery at Shotover Grange ? [Board Term I, NCT-2014] Ans. Danby studied the complete situation of the house, i.e., electric wiring, paths and garden. He collected information about the family. He knew all the movements of the servants. He always kept his tools ready.



Ans.







Q. 11.











Q. 10.

She was able to outsmart Danby easily. She not only robbed at Shotover Grange but also Horace Danby. [CBSE Marking Scheme, 2016] Q. 13. Why was Horace arrested when he had not stolen anything ? [Board Term I, 2016-17 Set F1X1EH5]



as he earned well. Everyone thought he was a good and respectable person. However, Horace was not completely honest. He used to rob a safe every year to buy expensive books. This proves that he was not completely honest. What was Horace Danby's hobby? How did he manage to fulfill it? [CBSE Delhi 10D, 2018] [Board Term I, DDE-2014] What went wrong when Horace Danby robbed at Shotover Grange? While Horace Danby was inside the house, the lady in red came and announced that she was the owner of the house. Her appearance was such that he was fooled. She asked him to open the safe for her. Horace left his finger prints all over there. Hence, he was arrested. Describe the lady in red. [Board Term I, 2016-17]

@PROCBSE A QUESTION OF TRUST

265

of the house. She was well dressed. She entered and talked to him in such a way that, he was unable to doubt her integrity. Even the dog did not bark at her. She had planned her theft in such a planned and organised way and drafted her trick so meticulously that even the brilliant thief like him couldn’t suspect her. He could have detected the true identity of the lady if he had used his courage by informing the police about that lady who persuaded him to pull off his gloves. He should have used his presence of mind that the lady, who didn’t know the number of the safe, couldn’t be the owner of the house. He should have noticed the fact that she hadn’t have any surprise on seeing a burglar in her house; rather she promised him that she would not hand him over to police in lieu of jewels of the safe.



an act of stealing can’t be justified by such excuses. To transform him into a really good person, I’d like to inculcate the values of trust and courage in him. He‘d be taught to have trust in himself that there are some other ways also to fulfil his desire and he has that ability to have access to those ways. He should have the courage to ask some faithful friends to help him in getting the books that he wanted to read. I’ll teach him the art of having belief in himself, i.e., by ounce of hard work, he can do whatever he wants to do, that also in a fair and respectable manner so that he should lead a respectable life. Q. 4. Horace opened the safe and gave her the jewels believing they belonged to her. Why could he not guess the true identity of the lady? What values could have helped him detect the true identity of the lady? Write in about 80-100 words. (CBSE Set RXT 021S 2016) 4 









Commonly Made Error



Students, sometimes, write disjointed sentences, which have no connection with one another.

Answering Tip































Ans. Value Points : (a) Courage (b) Acceptance (c) Reconciliation (CBSE Marking Scheme, 2016) 4 Detailed Answer : Horace could not guess the true identity of the lady because she was very smart and clever, hence succeeded in befooling him that she was the owner

Thorough reading of the chapter is a must.

Solutions for Practice Questions Short Answer Type Questions





Ans.4. No doubt, ‘A Question of Trust’ is a story about the robbery of the robber. Horace Danby was not an ordinary thief. He planned meticulously before attempting a robbery but still he was fooled by a young lady in red who posed herself as the owner of the house. She made him break open the safe without Horace realising that she was a thief. To keep herself safe, she ensured that the fingerprints of Horace were left everywhere on the site. Thus it is true that it is the story about the robbery of the robber. Ans.7. The young woman deceived Horace Danby by making him believe that she was the wife of the

house owner. She caught hold of the jewels worth fifteen thousand pound, by deceiving Horace. She would never have been able to get them without Horace’s help and she was aware of this fact. But I believe that deceiving other in any form is not correct. It is not good for others to harm others for one’s own benefit. Thus I strongly disagree that deception is a crucial aspect of robbery. Deception, in fact is not a crucial aspect of any tout. Ans.10. Horace Danby was not an ordinary thief. He loved rare and expensive books. So, he robbed a safe every year and secretly bought the books he loved through an agent. Each year, he planned carefully just what he would do.





REFLECTION Do you think there should be honour among thieves? Write any one instance you experienced with any such incident.

@PROCBSE

CHAPTER

5

FOOTPRINTS WITHOUT FEET – By H.G.Wells

Revision Notes



Griffin was a brilliant scientist. He carried on experiments for years to prove that human body could become invisible. He swallowed certain rare drugs and became an invisible person. He had a quarrel with his landlord. In revenge, he set fire to the house. He removed his clothes, became invisible and got away without being seen. He began to wander about the streets of London, without clothes, without money and invisible. He happened to be in some mud. As a result, his footprints were left behind as he walked. Two boys were fascinated by the footprints without feet. They followed the foot-marks. However, Griffin easily dodged them. It was mid-winter. The air was bitterly cold and he was without clothes. When the cold became unbearable, he slipped into a big London store. When the store was closed, he broke open boxes and found warm clothes for himself. On wearing them, he became a visible person. He found cold meat and coffee in the kitchen. Then he took sweets and wine. At last, he slept on a pile of quilts. When he woke up the next morning, the assistants had already arrived. He got nervous and began to run. They chased him. Then he quickly took off his clothes, became invisible and escaped from there. This time he entered a theatre company. He made his way upstairs. Nobody could see him because he was invisible. A little later when he came out,









Summary



This story is about a scientist who discovered how to make himself invisible. As he was afraid of being discovered in London, where he had committed some crimes, he ran away to a remote village. However, he ran out of money and resorted to robbery, using his advantage of invisibility.





Introduction

Scan to know he was wearing bandages more about round his forehead, dark this topic glasses, false nose, big bushy six whiskers and a large hat. He attacked the shopkeeper from behind and robbed him of all that he could find. Griffin was eager to get away Footprints from crowded London. He without feet went to the village of Iping and booked two rooms at a local inn. His uncommon appearance attracted everybody’s attention. Mrs. Hall, the landlord’s wife made every possible effort to be friendly with him. But he told her frankly that he had come there to work in solitude and did not want to be disturbed. She was satisfied. She thought he was an eccentric scientist. Moreover, he had paid the rent in advance. When his stolen money was spent, he admitted that he had no ready cash. He said that he was expecting a cheque and then he would make the payment. Thus, he satisfied the landlord. Griffin took off his clothes and became invisible. He entered the clergyman’s home with an intention to steal money from there. The clergyman and his wife woke up when they heard noises in the study-room. The clergyman entered the room with a poker in his hand. To his surprise, the room appeared to be empty. He and his wife searched everywhere, but there was no trace of the burglar. Yet the desk had been opened and the money had been stolen. That morning the landlord and his wife got up early. They were surprised to find the scientist’s door wide open, because it was usually shut and locked and he did not allow anybody to enter his room. When they entered the room, they found the scientist missing. The clothes and bandages,

@PROCBSE

FOOTPRINTS WITHOUT FEET



people in the bar were horrified to see a headless man. In the meantime, Mr. Jaffers, the constable, arrived there. He was surprised to find that he had to arrest a headless man. He tried to get hold of the man who was becoming invisible. The constable was struggling with someone whom he could not see at all. Some people who tried to assist him also received blows from the invisible scientist. The constable was knocked unconscious. Griffin had shaken himself free. In their nervousness, people tried to hold him, but they did not know from where to catch him.

Key Words



1. dodged: to avoid with sudden quick movement. 2. solitude: state of being alone. 3. eccentric: strange





that he always wore, were lying in the room. In the meantime, the invisible scientist entered the room. He attacked Mrs. Hall with the bedroom chair. She and her husband left the room in terror. Mrs. Hall was convinced that the room was haunted by spirits. She held the strange scientist responsible for that. The neighbours felt that the trouble was caused by witchcraft. When they came to know of the burglary at the clergyman’s home, they suspected the strange scientist. Their suspicion was strengthened when he suddenly produced some ready cash because only a day before, he had admitted that he had no money. They secretly sent for the village constable. When Mrs. Hall saw the scientist emerging from his empty room, she demanded explanation. The scientist became furious. He threw off his bandages, whiskers, spectacles and false nose. The space above his shoulders was now empty. The

Extract Based Questions



(5 marks each)























Q. 6. Pick the option that includes the correct matches of Column A with Column B.







































Q. 5. The Neighbours thought it was ‘Witchcraft’. This tells us that Neighbours were: (A) Suspicious. (B) Superstitious. (C) Nervous wrecks. (D) Gossip-mongers. Ans. Option (B) is correct. Explanation: Witchcraft or black magic reflect superstition.

Column A

Column B

(i)

The stranger was 1.

(ii)

He had escaped







Q. 3. Pick the sentence that brings out the meaning of ‘hysterics’ as used in the extract. (A) My friend and I were in splits when we saw the Clown’s antics.

Q. 4. Pick the option that displays a cause → effect relationship. (A) Pushed and locked out → hysterical (B) Rising of the Chair → moaning (C) Troubled Neighbours → Witchcraft (D) Stranger → Haunted Spirits Ans. Option (A) is correct. Explanation: Rising of the Chair does not produce moaning sound. Troubled Neighbours do not represent Witchcraft. Stranger may not be followed by Haunted Spirits. But, pushed and locked out is enough to make someone hysterical.











Q. 2. Pick the option that best describes how Mrs. Hall must be feeling at the moment described in the extract. (A) Stunned and furious (B) Shocked and outraged (C) Outraged and nervous (D) Stunned and agitated Ans. Option (D) is correct. Explanation: Mrs. Hall was astonished (stunned) and frantic or disturbed (agitated) at the strange behaviour of the Chair and the Door.

(B) I don’t know why I suddenly felt worried about Flying Home. (C) The sight of Blood put the Old Man in frenzy. (D) The People who had witnessed the accident were spellbound. Ans. Option (C) is correct. Explanation: Hysterics here refers to fits in a negative or terrified sense. The same is reflected in the option (C) where the Old Man was in frenzy upon seeing the blood.





II. As she and her husband turned away in terror, the extraordinary chair pushed them both out of the room and then appeared to slam and lock the door after them. Mrs. Hall almost fell down the stairs in hysterics. She was convinced that the room was haunted by spirits, and that the stranger had somehow caused these to enter into her Furniture. “My Poor Mother used to sit in that Chair,” she moaned! To think it should rise up against me now! The feeling among the Neighbours was that the trouble was caused by Witchcraft.” [CBSE-QB, 2021] (5 × 1 = 5) Q. 1. Mrs. Hall felt that the Room was haunted by Spirits because: (A) She could see Evil Spirits. (B) She heard strange noise. (C) Uncanny things happened there. (C) The Door slammed shut. Ans. Option (C) is correct. Explanation: Uncanny means mysterious. Mrs. Hall was pushed by a Chair, the Door was slammed and locked on its own, it was full of mystery for her.

267

2.

eccentric, lonely and callous eccentric, callous and short-tempered

@PROCBSE 268

Oswaal CBSE Question Bank Chapterwise & Topicwise, ENGLISH LANG. & LIT., Class-X



































(A) (i)-2; (ii)-4; (iii)-3 (B) (i)-1; (ii)-3; (iii)-4 (C) (i)-3; (ii)-2; (iii)-1 (D) (i)-2; (ii)-3; (iii)-4 Ans. Option (D) is correct. Q. 7. Look at the different meanings of ‘haunt’. Pick the option that DOES NOT correspond to its meaning: (A) To be conscious of a strange phenomenon. (B) Be persistently and disturbingly present in (the mind). (C) (of something unpleasant) continue to affect or cause problems for. (D) A place frequented by a specified person. Ans. Option (A) is correct. Explanation: When One is conscious of a strange phenomenon, he/she is said to be aware of that phenomenon.



as he wore bandages round his forehead



4.

(A) Option (i) (B) Option (ii) (C) Option (iii) (D) Option (iv) Ans. Option (D) is correct. Q. 3. Pick the option that best describes how the Boys are feeling based on the extract. (A) Enchanted, curious, puzzled (B) Captivated, curious, puzzled (C) Repulsed, curious, captivated (D) Enchanted, repulsed, curious Ans. Option (B) is correct. Explanation: The Boys are hypnotised (captivated), interested to know (curious) and bewildered or confused (puzzled) by the sight.



Q. 5. Pick the option that best matches synonyms of the word ‘gazed’. (A) (i) gaped (ii) gawked (B) (i) admired (ii) disbelieved (C) (i) overlooked (ii) stared (D) (i) surveyed (ii) overlooked Ans. Option (A) is correct. Explanation: Gazed means stared. This meaning is brought about by the synonyms gaped and gawked.







Q. 6. Pick the option that lists correct direction of the footprints on the Stairs, as noticed by the Boys.





   





  

















Q. 2. Pick out the option that is NOT related to ‘started’ as used in the extract:

(i) (ii) (iii) (iv) (A) Option (i) (B) Option (ii) (C) Option (iii) (D) Option (iv) Ans. Option (B) is correct. Explanation: The observed the footsteps descending the Steps. Descending means coming from top to bottom. So, option (B) indicates the correct direction using a downward pointing arrow.   















































III. The Two Boys started in surprise at the fresh muddy imprints of a pair of bare feet. What was a Barefooted Man doing on the Steps of a House in the middle of London? And where was the Man? As they gazed, a remarkable sight met their eyes. A fresh foot mark appeared from nowhere! Further footprints followed, one after another, descending the Steps and progressing down the Street. The Boys followed, fascinated, until the muddy impressions became fainter and fainter, and at last disappeared altogether. [CBSE-QB, 2021] (5 × 1 = 5) Q. 1. Why were the Boys surprised to see a Barefooted Man in London? The Boys were surprised as: (A) It was an unusual sight to see someone this way. (B) Everybody in London moved around in Shoes. (C) It was pretty cold to move around Bare feet. (D) Only a person who is homeless and wandering does so. Ans. Option (C) is correct. Explanation: That time of season in London was very cold. So, it was not practical to find a Man walking barefoot on such Cold Streets or Floor.

Q. 4. The Boys felt that the footprints were: (A) Seen due to some Magic trick. (B) A figment of imagination. (C) Of a man who was Invisible. (D) Those of a Mysterious Man. Ans. Option (D) is correct. Explanation: The Boys never knew that someone could be Invisible. So, they thought that the footprints belong to a Mystery Man.









from Iping to London





3.



(iii) He had an uncommon appearance

@PROCBSE

FOOTPRINTS WITHOUT FEET

269

SUBJECTIVE TYPE QUESTIONS Short Answer Type Questions













































place. Griffin is an antithesis to this statement. Justify. [CBSE-QB, 2021] Ans. There is no denying the fact that scientists contribute to make the world a better place but it was not so in the case of Griffin. His unbridled ambition made him a cruel and heartless person. Whatever they did, it made everything worse. He didn’t show kindness or humanity towards anyone whether it was his land lord the shopkeeper or the people living at Iping. He set the house of his landlord on fire, he pushed a shopkeeper down the stairs by taking advantage of his invisibility. None of his actions were for the betterment of the humans. Thus, we can say that griffin is an antithesis to this statement. Q. 6. Your teacher conducted a debate in class on the topic ‘Schools must develop scientific temperament in students’. Write any two arguments that you used to supplement your stand, either as a proposition speaker or as an opposition one. [CBSE-QB, 2021] Ans. School Must Develop Scientific Temperament in Students. Today’s age is the age of science with so many inventions and advancements in science and technology, man has taken a leap. I am in favour of developing scientific temperament in students in their school itself. The sole of the school is very important. It is the school only where so many activities can be organized to improve the critical thinking skills of students Lectures can be organized by eminent scientists, science exhibitions can be organized to inculcate scientific temperament. These kind of activities are not possible at home. It is the prime duty of the schools to develop the scientific temperament through various activities.

Q. 1. The neighbours suspected Griffin of burglary. State why this was OR was not justified. [CBSE-QB, 2021] Ans. The neighbors suspected Griffin of burglary. This was justified because Griffin had always told Mrs. Halls that he had no more ready cash and he was expecting a cheque to arrive at any moment but after the burglary, he suddenly produced some ready cash which made the neighbours suspicious. Q. 2. What do you think would have happened if Griffin was caught by Jaffers and the latter hadn’t become unconscious? [CBSE-QB, 2021] Ans. If Griffin was caught by Jaffers and the latter hadn’t became unconscious, the situation would have been totally different. The cruelty and lawlessness of Griffin, the scientist, would have come to an end and the people of Iping would have heaved a sigh of relief. Q. 3. If you were a friend of Griffin, what suggestion would you have given to him about how he could use his talent and scientific acumen? Write one suggestion and the reason for giving it. [CBSE-QB, 2021] Ans. If I was a friend of Griffin, I would have suggested Griffin to use his talent and scientific acumen in a positive and constructive manner by helping the poor and the needy. I would have told him to indulge in such archives which would be beneficial for the society. Q. 4. Do you find the story based on being invisible a mere figment of one’s imagination or a phenomenon that is plausible? Elaborate with [CBSE-QB, 2021] your opinion. Q. 5. Scientists contribute to make the world a better





(40-50 words & 3 marks each)

Long Answer Type Questions







something wrong and he regrets later when his conscience pricks him, he always make amends and rectify his mistake but if he accepts everything and continues walking down the lane of errors he can never become a good human being. Q. 2. Griffin set the house on fire when he was thrown out. Anger and revenge are in no way a solution to any problem. You are a columnist for a class X e-zine(magazine). Create a column on importance of anger management for teens and its detrimental effects on health.

These questions are for practice and their solutions are available at the end of the chapter









Q. 1. Readers believe that Griffin was a man without conscience. You have a class discussion on why our conscience plays a vital role in shaping our character. Present a summary of your views on this. [CBSE-QB, 2021] Ans. We are what we think, Griffin, the invisible scientist had a negative bent of mind. He acted cruelly so many times but never once did his conscience prick him. Everyone is well a more that conscience plays a vital role in shaping one's character. It is a common saying to err is human. If a person does 









(100-120 words & 6 marks each)

@PROCBSE

Ans.4. The story based on being invisible is a mere figment of one’s imagination. It is very difficult





Short Answer Type Questions



to imagine the life of an invisible man. However, with the advancement in science and technology, becoming invisible may be possible.

REFLECTION





Solutions for Practice Questions (Topic-1)





Q. 3. Imagine that Griffin has been caught by the police and is produced in court. He needs to defend himself and prove that he is not guilty of the charges. As Griffin prepare a statement that you would use in court to prove your innocence. You may begin like this: I plead not guilty. I have been accused of …, however… Ans. Griffin- I plead not guilty – I have been accused of violence and cruelty by the people of Iping. However I deny all the charges. I didn’t hurt anyone intentionally I had no ill will against anyone. The circumstances, somehow, were such that I had to be violent or cruel in order to save myself. I have been accused of robbery at the clergyman’s house. My problem was that Mrs. Hall was nudging me all the time to pay the rent. I told her so many times that I was waiting for my cheque and would pay when the cheque arrived. But she became impatient and I had to rob the clergyman in order to get money. Once I attacked the shopkeeper from behind and robbed him of all the money. But it because I wanted to escape from that place. Thus I plead that I am innocent and all these points must be taken into consideration before announcing any judgement. Thank you.



You may begin like this: ANGER DOESN’T RESOLVE, IT DESTROYS by ..............................., X ..... Griffin, in Footprints Without Feet sets the house on fire ... ........................................................................................ ( CONTINUE) [CBSE-QB, 2021] Ans. Anger Doesn’t Resolve, It Destroys By Aradhna X-C Griffin in footprints without feet, sets the house on fire when he was thrown out. He was so angry that he set fire to this house without giving a single thought to it. His landlord had threatened him to evacuate the house. So he wanted to take revenge on him. But one must realise that anger and revenge are in no way a solution to any problem. It is an old saying that kindness pays, rudeness never. Anger can never resolve any problem. It rather aggravates it. It is always better to keep cool when someone is angry. Despite anger being a healthy normal emotion it must be under one’s control. It should not be allowed to escalate, uncontrollable anger leads to high blood pressure and anxiety. Parents are the role models of their children so they should not yell or scream at them. First they should keep their cool so that the child follows them.



Oswaal CBSE Question Bank Chapterwise & Topicwise, ENGLISH LANG. & LIT., Class-X











270

Were you able to observe that one should not misuse Science and its great discoveries as this can turn a scientist into a threatening lawless person?

@PROCBSE

CHAPTER

6

THE MAKING OF A SCIENTIST

– By Robert W. Peterson

Revision Notes Scan to know more about this topic

Introduction

there was no feedback, he started losing interest. Right from the eighth grade, he started doing real experiments. He soon started winning awards after awards. He then realised the importance of The making of a cells and showed how a cell scientist would divide and develop into normal butterfly wing-scales. He won the first prize at the International science Fair. Then he got a new blue print of its DNA. Other than Science, Ebright was a good debater, canoeist as well as an expert photographer.



This lesson is about one of the greatest scientist Richard H. Ebright who was an inspiration for the young generation of his times. In his childhood, he had the habit of collecting butterflies, rock fossils and coins. He started his scientific study and research on butterflies during his school time. His extreme curiosity and will to win for the right cause made him an exceptional scientist.

Summary

Key Words

1. Fossils: mineralized remains of an animal or plant. 2. Canoeist: someone who is skilled at racing in a canoe.



This story is the unbelievable account of Richard Ebright, efforts as a scientist, who did wonders in the field of science and made a niche for himself in the field of scientific research. He was the only child of his parents, who had a great fascination for butterflies, right from when he was very young. His mother was a source of inspiration for him right from his childhood and always encouraged him to have a wonderful collection of fossils, rocks, coins and butterflies in his room. Guided by Dr. Urquhart, he started tagging monarch butterflies, but when

Extract Based Questions

[CBSE-QB, 2021] (5 × 1 = 5)





































Q. 1. Choose the option listing Ebright’s QUALITIES as depicted by the above extract. 1. Persevering 2. Visionary 3. Determined 4. Liberal 5. Conceited (A) Options 1 & 2 (B) Options 3 & 5 (C) Options 1 & 3 (D) Options 4 & 5 Ans. Option (C) is correct. Q. 2. According to the Dictionary, ‘fair’ as a noun, shows the following meanings.

I. “I didn’t get any real results,” he said. “But I went ahead and showed that I had tried the Experiment. This time I won.” The next year his Science Fair Project was testing the Theory that Viceroy Butterflies copy Monarchs. The Theory was that Viceroys look like Monarchs because Monarchs don’t taste good to Birds. Viceroys, on the other hand, do taste good to Birds. So, the more they look like Monarchs, the less likely they are to become a Bird’s Dinner. Ebright’s Project was to see whether, in fact, Birds would eat Monarchs. He found that a starling would not eat ordinary Bird food. It would eat all the Monarchs it could get.





(5 marks each)

@PROCBSE Oswaal CBSE Question Bank Chapterwise & Topicwise, ENGLISH LANG. & LIT., Class-X

Choose the option that lists the meaning similar to the usage to that in the extract. (A) A gathering of Stalls and Amusements for Public Entertainment. (B) A Competitive Exhibition showcasing Products or Ideas. (C) A Periodic Gathering for the Sale of Goods. (D) An Annual Exhibition of Livestock, Agricultural products, etc., held by a Town, County or State. Ans. Option (B) is correct. Q. 3. Choose the option that is true for the two statements given about the information in the extract: Statement 1- Starling feeds on Viceroys. Statement 2- Starling does not eat Seeds and Insects. (A) Both statements are clearly mentioned in the extract. (B) Statement 1 cannot be clearly inferred from the Text and statement 2 is true. (C) Statement 1 is false and statement 2 cannot be clearly inferred from the Extract. (D) Both statements need to be inferred from the given Extract. Ans. Option (C) is correct. Explanation: Starling likes to eat Monarchs and not Viceroys. What the author means by ordinary Bird food is not clarified in the passage, so Statement 2 cannot be inferred from the extract. Q. 4. Choose the statements that are TRUE for the given extract contextually: 1. Ebright didn’t get any results for the experiment he conducted on butterflies. 2. Monarchs tasted awfully to the birds. 3. Ebright wanted to explore the possibility of Monarchs getting eaten by birds. 4. He wanted to prove that Viceroys are look alikes of Monarchs. (A) 1, 2 (B) 2, 3 (C) 1, 3 (D) 2, 4

along with his findings about Insect hormones, the photos gave him the answer to one of biology’s puzzles: how the cell can ‘read’ the blueprint of its DNA. DNA is the substance in the Nucleus of a Cell that controls heredity. It determines the form and function of the cell. Thus, DNA is the Blueprint for life. Ebright and his College Room-mate, James R. Wong, worked all that night Drawing Pictures and Constructing Plastic Models of Molecules to show how it could happen. Together they later wrote the paper that explained the Theory. [CBSE-QB, 2021] (5 × 1 = 5) 











Q. 1. Choose the option that shows the picture of the type of task Ebright and Wong were engaged in, as per the extract.













































II. When he saw those photos, Ebright didn’t shout, ‘Eureka!’ or even, ‘I’ve got it!’ But he believed that,



















Ans. Option (D) is correct.













Ans. Option (B) is correct. Explanation: Ebright did get the wanted results from his Experiment. Everyone knew that Monarchs are look alikes of Viceroys, so he did not want to prove this. Q. 5. Four Friends bring their Pets to a pet show. Choose the option that mentions the Friend with a starling as a pet. Friend 1 has a Turtle named Missy. Friend 2 has a Dragonfly named Majesty. Friend 3 has a Rabbit named Molly. Friend 4 has a Bird named Mitch. (A) Friend 1 (B) Friend 2 (C) Friend 3 (D) Friend 4

(A) Option (1) (B) Option (2) (C) Option (3) (D) Option (4) Ans. Option (C) is correct. Explanation: Ebright and Wong were engaged in Study of the Internal Functioning of a Biological Cell and how it formed chains with other similar cells. Q. 2. Ebright was perhaps expected to shout ‘Eureka!’ because he had: (A) Realised that he needed a Partner to work with to finalise his findings. (B) Discovered something new and ‘Eureka!’ was a cry to announce it. (C) Worked hard and was relieved at nearing the end of his Project. (D) Given shape to the teachings of his Teachers by choosing this Field of Science. Ans. Option (B) is correct. Explanation: Eureka is a famous cry made at the time of a New Invention or Discovery. It owes its origin to the famous Greek Inventor and Mathematician, Archimedes. Q. 3. “Thus, DNA is the Blueprint for Life”, is another way of saying that the DNA contains a Genetic __________. (A) Experiment (B) Ultimatum (C) Takeaway (D) Plan Ans. Option (D) is correct. Explanation: ‘Blueprint’ for Life means a ‘Plan’ as to how life will continue from one cell to another and from one generation to the next. Q. 4. Four Newspapers published a headline about Ebright and Wong. Choose the option that published a factually correct headline, as per the extract.







































272

@PROCBSE

THE MAKING OF A SCIENTIST











Blueprint Heredity

(B) Options 2 & 4 (D) Options 2 & 5

Ans. Option (D) is correct.

Ans. Option (B) is correct. Explanation: The First, Third and Fourth Newspaper Clippings criticise Ebright and Wong’s

2. 4.













(B) Newspaper 2 (D) Newspaper 4

Determines Nucleus Room-Mate Options 1 & 3 Options 1 & 4







1. 3. 5. (A) (C)









(A) Newspaper 1 (C) Newspaper 3

Choose the option that lists the compound words from the above extract.



Newspaper 4 EBRIGHT AND WONG’S THEORY PROVED WRONG



Newspaper 3 WONG AND EBRIGHT EXAGGERATE THEIR THEORY DEFY LOGIC

Q. 5. Compound words are those words which are formed by joining two separate words to create a new word with an entirely different meaning.



EBRIGHT COLLABORATES WITH ROOM-MATE WONG



WONG DENIES CONTRIBUTING TO EBRIGHT’S THEORY

Theory and present it in a negative sense. Wong and Ebright shared Room, Drew Pictures Related to Experiment, Made Plastic Models all night long.

Newspaper 2



Newspaper 1

273

SUBJECTIVE TYPE QUESTIONS Short Answer Type Questions

Ans. Ebright’s mother encouraged his interest in learning; took him for trips, bought him telescopes, microscopes, cameras, mounting materials and other equipment. [CBSE Marking Scheme 2019] 



















Q. 5. The book, ‘The Travels of Monarch X’ ignited Richard’s curiosity in butterflies further. Do you think sometimes, people or things around us can act as catalysts to shape our lives? Explain how. [CBSE-QB, 2021] Q. 6. How did Ebright’s mother help him in becoming a scientist? [CBSE OD Set 3 2019, 2017]











Ans. Value Points : Learned winners do real experiment, not simply display slides. [CBSE Marking Scheme, 2016]

These questions are for practice and their solutions are available at the end of the chapter













Detailed Answer : Ebright’s mother encouraged his interest in learning. She took him on trips, bought him telescopes, microscopes, cameras, mounting materials and other equipment and helped him in many other ways. By this, we can say that it was Ebright’s mother who led him to become a great scientist. 2 Q. 7. What lesson did Ebright learn when he could not win a prize at the science fair ? [Board Term-II, 2016 Set 2/2/1/NCERT] OR What lesson did Ebright learn when in the seventh grade he entered a country science fair? [CBSE 2017]

































Q. 1. Do you think Richard’s mother was too indulgent when she bought him things like cameras, telescopes, microscopes? Give two reasons to support your answer. [CBSE-QB, 2021] Ans. Yes, certainly Richard’s mother was concerned about her son’s future. He was the centre of herlife. She did all that was there in her capacity to help Richard in his feats. He carried on with his experiments, because his mother arranged for telescope and microscope. This way he was able to learn a lot. Q. 2. Suggest two ways by which you feel scientific temperament can be developed in a child. [CBSE-QB, 2021] Q. 3. Do you think we always need sophisticated instruments for all science experiments? Why/ Why not? [CBSE-QB, 2021] Ans. No, not really. We have seen many a times that the great scientists have led to great discoveries and inventions using crude and raw instruments which have brought changes all over the world. So, it is not necessary that we need sophisticated instruments to create wonders. Q. 4. Richard’s mother was his companion and spent a lot of time with him. A parent should try to be a companion to a child. Justify. [CBSE-QB, 2021] Ans. Yes, Richard’s mother was his true friend who helped him in all steps of life to become the world’s greatest scientist. It is necessary to be a friend to a child than be parents. As a friend, we can understand his problems better that he is facing because we have also gone through all this in our teenage years.





(40-50 words & 3 marks each)

@PROCBSE 274

Oswaal CBSE Question Bank Chapterwise & Topicwise, ENGLISH LANG. & LIT., Class-X

OR

Topper Answer, 2017



















significance of Science in our lives. It indeed is our life, the be all and end all of our existence on this very special planet mother earth. Many scientific discoveries and inventions have changed the course of our life, from a food gatherer man became a food producer; it’s all thanks to greatness of Science. It has revolutionised the entire world around. No man is left free without his dependence on scientific inventions and discoveries. So, we can say truly ‘Science is life’. Q. 11. The will to win, for the right reasons, was a quality that Richard had. Do you think this quality alone can help us accomplish great feats? Why/ Why not? [CBSE-QB, 2021] Ans. Yes, the will to achieve the impossible is a unique feature in only those individuals who like to do something different for the society. Richard Ebright was a bright student in his school days, who always wanted to reach heights in his chosen field. Though many people through his journey of life helped him to achieve this extraordinary feat; but it was his own strong will and determination to sail through this hard journey all alone by himself. Not even once, did he lose his hope in himself and his spirits were always alive to achieve recognition and awards in his very own way. Q. 12. Richard’s mother had been invited to a talk show after Richard’s graduation from Harvard with the highest honours. You attended that the talk show. Write the most memorable thing she spoke and what you think it reveals about her. [CBSE-QB, 2021]







Ans.









































Detailed Answer : When Ebright could not win a prize at the science fair, he learnt that winners do real experiment, they don’t simply display slides. Then he started conducting experiments. It was definitely a stepping stone towards his success. His competitive nature, his extra efforts and the will to win for the right reasons made him a successful scientist. Q. 8. How did Richard’s mother help him to become a scientist? [Board, Term-II, 2014 Code-2/1/3] Ans. Richard’s mother was a vigilant parent who keenly observed her child, identified his interests and inclinations and channelized his energies in the right direction. She very deftly kept him occupied with learning activities which was a constructive use of the leisure time. She bought him books to kindle his curiosity and thus provided the impetus towards scientific interests. She, thereby, played a significant role in ‘The Making of a Scientist’. Q. 9. Why did Richard Ebright raise a flock of butterflies? [Board, Term-II, 2014 Code-2/1] Ans. Richard Ebright as a child had developed a keen interest for collecting things like rocks, fossils, coins and butterflies. By the time he reached his second grade, he had collected all the twenty-five species of butterflies found in Pennsylvania. He also started tagging the Monarch’s at the behest of the author Dr. Fredrick A. Urguhart. Q. 10. You have been requested to write a brief blog post for a Science blog for kids on the theme-‘Science is Real’. Based on your understanding of the chapter, write the blog highlighting how science pervades every aspect of our lives. You may begin like this: Scienceforkidsblogger.com Monday, October 19, 2021 SCIENCE IS LIFE ............................................................................................ ............................................................................................ [CBSE-QB, 2021] Ans. Scienceforkidsblogger.com Monday, October 19, 2021 SCIENCE IS LIFE It’s true that above line is telling us about the

A TALK SHOW Emcee / Host : Well! Miss Ebright, how does it feel now when your son, Richard was given award

@PROCBSE

THE MAKING OF A SCIENTIST







his world of science and surroundings. Emcee / Host : Do you love him ? Mother : Yes, I do. As a mother he is very dear to me. Being the only child he is all the more closer to me than anyone else. For him, his world revolves around me. Emcee / Host : He graduated from Harvard University with greatest honours. What is your reaction? Mother : Well! he very rightly deserves it. He has worked hard for it because of his tremendous efforts, he has won over all obstacles to reach where he is today.









for the young scientist as the highest honour at International Science Fair? Mother : It feels great! I mean it. My son was always a special child who loved to experiment with existing things. His extreme curious nature and love for all forms of life led to this discovery. Emcee / Host : Do you treat him in a special way now that he has become so famous ? Mother : For me, he is my same child with normal needs and wants. Richard has always been a very polite child right from his childhood. He has never been too demanding. All he ever wanted was to left alone to himself. So, that he is into

Long Answer Type Questions

(100-120 words & 6 marks each)

Ans. Surely, this statement is true in case of Richard because not even once in the course of chapter he allowed himself to detach from his work. He very diligently worked towards achieving his purpose. This hard work and undying commitment towards his purpose brought him lot of laurels and respect in the world. He kept working so that he could reach his aim without letting any obstacle come his way. His scientific bent of mind led him to carry on experiment after experiment and reach to a purposeful conclusion. In his efforts to reach his goal, he was helped by many people in his journey, especially his mother whose contribution towards his success cannot be denied. Though Richard was a genius himself but never even once did he stop midway to conclude his work. He very patiently took each step one by one to make success out of him. This shows if one sets his eyes on something he will achieve it no matter how difficult the path is. Q. 3. Imagine you are Richard. You have been invited by Mr. Weiherer to deliver a short address to the students at your erstwhile school. You have been asked to encourage them to take an interest in the study of Science and how research in Science can help the world. Write that short speech. [CBSE-QB, 2021] Q .4. ‘‘Richard Ebright was a successful scientist who gave a new theory of cells to the scientific world’’. Elucidate. Ans. Richard Ebright conducted an experiment on a flock of butterflies and raised the eggs of female monarchs. In addition to this, he also studied the cause of a viral disease that killed all monarch caterpillars. He also tried to find out similarities between viceroy butterflies and monarch. The study of insect hormones helped him in giving a new theory on the life of cells. Eventually, he was able to find the hormone behind the full development of a butterfly wings. All these experiments led him to conclude that cells grew from a monarch’s wings. He found that cells would divide and grow into a normal butterfly if they were fed a hormone that was found in the gold spots of the monarch. Hence, he proved a new theory which formed the blue print of DNA. The DNA is the substance in the nucleus of a cell that controls heredity. It determines the form and function of the cell. This DNA is the blueprint for life. The experiments conducted by Ebright amply showed him how to be a successful scientist.

These questions are for practice and their solutions are available at the end of the chapter























Dear Mom It’s been long since I’ve written to you. I owe all my success to you. Had you not motivated me and guided me during my childhood, I would not have been at this place. You are such a dedicated mother. You never discouraged me. I always found you around me whenever I needed you. Whether it was during school education, the competitions held at school or the hobbies undertaken by me in my childhood. I really am grateful to you for being a constant source of support throughout my growing year. Thank you so much Mom. Yours lovingly Richard. Q. 2. Albert Einstein said, ‘The important thing is to never stop questioning.’ Richard was a genius who proved this quote true. Justify. [CBSE-QB, 2021]







Mrs. Ebright Reading Pennsylvania







20 September 1981





































Q. 1. Imagine Richard wrote a letter to his mother when he was studying at Harvard University for motivating him and being a constant source of support throughout his growing up years. Write that letter of gratitude as Richard. You may begin like this: Harvard University Cambridge Massachusetts 20 September 1981 Mrs. Ebright Reading Pennsylvania Dear Mom It’s been long since I’ve written to you. ……………………………... [CBSE-QB, 2021] Ans. Harvard University Cambridge Massachusetts

275

@PROCBSE 276

Oswaal CBSE Question Bank Chapterwise & Topicwise, ENGLISH LANG. & LIT., Class-X





Commonly Made Error Students generally get confused in the various experiments done by Richard Ebright thus giving wrong answers to most of the questions.





Answering Tip Students must be guided to read the text and make notes of different experiments conducted by Richard Ebright in order to get a clear idea.

Solutions for Practice Questions (Topic-1) Short Answer Type Questions





Ans.2. Scientific temperament in a child is already there, the need is to cultivate scientific attitude in child right from his childhood. His curiosity to know about strange things and will to learn new things should always be encouraged. Ans.5. Richard was bored with butterflies so his mother bought him a book “The travels of Monarch X” to read and enjoy himself. After reading the book, he studied the migration of butterflies and it opened the world of science to him. Truly, some people can be a source of inspiration to us because they play the role of catalyst to ignite our inner fire to reach our goal in life.

Long Answer Type Questions













Ans.3. Hello, My dear young friends, It’s an honour to stand amongst you and say a few words on how Science has changed the life of a man. Don’t you think what I am saying is

true; I sincerely feel that study of science has brought a revolution in the world in terms of progress and growth. There has been research after research carried on by scientists which has brought the world closer as a one single unit. Scientific development in one corner of world is affecting every person’s life in some way or the other. All comforts that we enjoy in life be it Computers, T.V., Fridge, Mobiles or Airplanes are all the products of scientist’s efforts and labour. I very earnestly feel that every child in his school time should pick up the study of science which prepares him to face the world with much more confidence. Being a science student myself, I feel highly blessed to take up the study of science which has developed me into a sound individual. Thanks! Have a great day! Good day.





REFLECTION Did you realize that with perseverance, dedication and hard work, one can achieve any dream?

@PROCBSE

CHAPTER

7

THE NECKLACE – By Guy de Maupassant

Revision Notes On reaching home, she decided to take a final look at herself. She was terribly shocked when she found the necklace was missing. The couple looked everywhere for it, but it was nowhere to be found. They went to the police The Necklace and also put an advertisement in the papers. In the meantime, they wrote to Madame Forestier that the clasp was broken and also that they would get the necklace repaired and return it. When the necklace was not found, they decided to replace it with a similar necklace. They bought it for thirty six thousand Francs, and had to take a loan for it. In order to repay the loan, they took a rented room and turned away the maid. All the household work was done by Matilda. Mr. Loisel took up overtime work to save money. This life continued for almost ten long years, by the end of which Mrs. Loisel looked old, worn out and scruffy. One day she happened to meet Mrs. Forestier who still looked young. Matilda told her the whole story. Shocked at the story, Mrs. Forestier told her a shocking truth i.e. the diamond necklace that she had lent to Matilda had been made of artificial diamonds and was worth only five hundred francs.



‘The Necklace’ is a touching story of a pretty lady named Matilda by Guy de Maupassant. He was a popular writer of his times very aptly remembered for his short stories with real life situations. In this particular story the main character Mme Matilda is married to a clerk named M. Loisel. She was very dissatisfied in her life because of her ‘poverty’. Once she borrowed a necklace from her friend Madame Forestier and lost it in the party. To replace the necklace, she had to live a very hard and miserable life. The tale is about how her desire to have luxurious life, lead to her ruin.

Scan to know more about this topic





Introduction



Key Words

1. Exasperated: intensely irritated and frustrated 2. Scruffy: Shabby and untidy or dirty 3. Francs: monetary unit of France, Switzerland, etc.

Matilda was born in a poor family. She was married to a clerk. She suffered a lot as she felt that she had been born for better things. All day, she would be day dreaming of luxurious houses, dainty dinners, elegant dresses and beautiful jewellery. One fine day, her husband came home very happy. He had an invitation to a ball for both of them from the Minister’s house. He thought his wife would be happy, but she got angry and threw the invitation card on the table. On being asked why she reacted in such a manner, she replied that she did not have any pretty dress for the party. Her loving husband gave her all the money he had set aside to buy a gun. When the dress was ready, Matilda’s husband saw that still she was not happy. On being asked the reason, she said that she had no jewels. Exasperated, her husband asked her to wear fresh flowers. When she refused to do so, in desperation he told her to borrow jewellery from her dear friend — Mrs. Forestier. Matilda borrowed a diamond necklace from her. She was a great hit at the party. Everyone admired her. She danced till 4 a.m. and then went home tired, but happy.









Summary

@PROCBSE 278

Oswaal CBSE Question Bank Chapterwise & Topicwise, ENGLISH LANG. & LIT., Class-X

Extract Based Questions







































Q. 4. Choose the characteristic displayed by M Loisel in the extract. (A) Conceited (B) Contended (C) Appeased (D) Subdued Ans. Option (B) is correct. Q. 5. The extract uses the phrase ‘Elegant Dinners’. Which of the following expressions is incorrect with respect to the word ‘elegant’?







(A) Option (i) (B) Option (ii) (C) Option (iii) (D) Option (iv) Ans. Option (C) is correct. Explanation: Tureen is a serving dish. Option (i) is cooker used to Cook food, option (ii) is like an Earthen pot used to store and pour liquids, option (iv) is a bowl used to store food. Option (iii) has a spoon with it which can be used to serve dishes.



I. “She suffered incessantly, feeling herself born for all delicacies and luxuries. She suffered from the poverty of her apartment, the shabby walls and the worn chairs. All these things tortured and angered her. When she seated herself for dinner opposite her husband who uncovered the tureen with a delighted air, saying, “Oh! the good potpie! I know nothing better than that...,” she would think of elegant dinners of shining silver; she thought of the exquisite food served in marvellous dishes. She had neither frocks nor jewels, nothing. And she loved only those things. She had a rich friend, a schoolmate at the convent, who she did not like to visit. She suffered so much when she returned. She wept for whole days from despair and disappointment.” [CBSE-QB, 2021] (5 × 1 = 5) Q. 1. Choose the options that list the set of statements that are NOT TRUE according to the given extract: 1. Matilda was very pleased with her life. 2. Matilda envied her friend for being well-off. 3. M Loisel didn’t appreciate what Matilda cooked. 4. Matilda despised the fact that she lived a life of poverty. 5. Matilda never felt troubled, though she desired a luxurious life. 6. Matilda thought of grand dinners and silverware sitting at the dinner table. 7. Matilda felt depressed after visiting her friend. (A) Options 1, 3 & 6 (B) Options 3, 5 & 7 (C) Options 1, 3 & 5 (D) Options 2, 4 & 7 Ans. Option (C) is correct. Explanation: Matilda was highly dissatisfied and displeased with her life. M Loisel always appreciated whatever Matilda cooked even if he did not like the food at all. Matilda desired luxurious life and was troubled by the pettiness of her own life. 









(5 marks each)































































Q. 3. Choose the answer that lists the correct option of what a ‘Tureen’ is?

















Q. 2. Which word does ‘delicacies’ NOT correspond to? (A) Etherealness (B) Elegance (C) Exquisiteness (C) Robustness Ans. Option (D) is correct. Explanation: Etherealness means belonging to some other world, elegance means grace and exquisiteness and loveliness can be used to refer to delicacies or delicate items. Such delicate items cannot be robust or strong.

(A) Option (i) (B) Option (ii) (C) Option (iii) (D) Option (iv) Ans. Option (C) is correct. II. “He was silent, stupefied, in dismay, at the sight of his Wife weeping. He stammered, “What is the matter? What is the matter?” By a violent effort, she had controlled her vexation and responded in a calm voice, wiping her moist cheeks, “Nothing. Only I have no Dress and consequently I cannot go to this affair. Give your Card to some Colleague whose Wife is better fitted out than I.’’ He was grieved, but answered, “Let us see, Matilda. How much would a suitable costume cost, something that would serve for other occasions, something very simple?” She reflected for some seconds thinking of a sum that she could ask for without bringing with it an immediate refusal and a frightened exclamation from the Economical Clerk.” [CBSE-QB, 2021] (5 × 1 = 5) Q. 1. What does ‘Economical Clerk’ indicate? (A) M Loisel was a spendthrift even though he earned a lot. (B) M Loisel was thrifty as he had a meagre income. (C) M Loisel calculated money all the time as he was a Clerk. (D) M Loisel was stingy about money and didn’t spend it. Ans. Option (B) is correct. Explanation: Spendthrift is one who has money but does not want to spend it. He has not collected money. Stingy is almost same as Spendthrift. Loisel

@PROCBSE

THE NECKLACE



11 January, Monday 9:00 pm I thought Matilda would be (i) ______________ seeing the invitation in my hand. However, her reaction has left me (ii)__________________. I don’t know how I would be able to (iii)_______________ a new dress for her. (A) (i) vexed (ii) disturbed (iii) bring (B) (i) elated (ii) disturbed (iii) afford (C) (i) keen (ii) depressed (iii) bring (D) (i) elated (ii) distressed (iii) afford

Q. 2. Pick the option that correctly classifies fact/s (F) and opinion/s (O) of the people below:







Ans. Option (B) is correct. Explanation: ‘I think…’, ‘I feel…’ and ‘I really don’t think…’ are used to express opinion about someone. ‘It is important’ is a statement of fact. Q. 3. Pick the correct set that matches with the feelings of the highlighted words related to the characters: Matilda: By a violent effort, she had controlled... M Loisel: He was grieved.... (i) Matilda felt aggressive; M Loisel was troubled (ii) Matilda was irritated; M Loisel was upset and cried (iii) Matilda tried extremely hard; M Loisel felt intense sorrow (iv) Matilda was quite calm; M Loisel’s heart ached for love (A) Option (i) (B) Option (ii) (C) Option (iii) (D) Option (iv) Ans. Option (C) is correct. Q. 4. M Loisel was astonished seeing his wife’s reaction. He writes a diary entry that night. Complete the entry with the missing words, by choosing the correct option.

Q. 5. Choose the option that gives the most appropriate response to the statement made by the speaker.









(B) F-3, O-1, 2, 4 (D) F-2, 3, 4, O-1

Ans. Option (D) is correct. Explanation: When one sees an invitation, he/she feels happy (elated). The reaction may leave the other person troubled or upset (distressed). There was confusion how the funds for the dress will be managed (afforded).

















(A) F-1, 2 and O-3, 4 (C) F-2, 4, O-1, 3

















had a Meagre (low) Income and was Thrifty (One does not spend much).

279

















Ans. Option (B) is correct. Explanation: Loisel loved his wife too much and had spoiled (pampered) her.



















(A) I think Matilda was being unreasonable and unrealistic. (B) I feel that M Loisel loved Matilda and wanted her to be happy. (C) In my opinion M Loisel was being too harsh with Matilda. (D) I feel that M Loisel should not have brought the invitation home.

SUBJECTIVE TYPE QUESTIONS Short Answer Type Questions Q. 1. Briefly state how Matilda invited ‘a dreadful life of necessity’ into her family. [CBSE-QB, 2021]









Guidance The question does not require the summary of the story but needs an answer only with relevant reference to Matilda’s extreme self-indulgence and loss of the necklace. Content— Award 1 mark for full explanation of the two strands









Ans. Value Points • Matilda’s extreme self-indulgence--dreams of a luxurious life, riches and jewellery • Doesn’t pay heed to the advice of wearing natural flowers-- borrows the necklace--loses it • In paying for the necklace invited a horrible life of necessity and deprivation/ a life that resulted in a hand-to-mouth existence OR any other relevant point















(40-50 words & 3 marks each)

Award ½ mark for partial explanation. Expression – 1 mark when both aspects included

@PROCBSE 280

































































Ans. Value Points : Matilda Loisel was very pretty - considered herself born for a luxurious life. But in reality she was married to a petty clerk. The poverty of her apartment and the lack of fashionable clothes and jewellery too added to her unhappiness. (Any two) [CBSE Marking Scheme 2016] Detailed Answer : Matilda Loisel was very pretty. She considered herself to be born for a luxurious life but the reality differed from her dreams. In reality, she was married to a petty clerk and had to lead a very simple life. Her fantasy over ambitiousness made her unhappy. Matilda’s desire to belong to the rich and affluent section of society was responsible for her downfall. If she had accepted her condition with a smile and without any complaints, she would not have to live ten long years of her life in such object poverty. Q. 8. Why did Matilda not want to see her rich friends? [Board Term-II, 2016 Set 2/1/1, 2/1/2, 2/1/3] Q. 9. How did the Loisel’s react when they realised that the necklace had been lost ? [Board Term-II, 2015 Set 2/1/2] Ans. As soon as they realised that the necklace had been lost, their short-lived happiness degenerated into shock of the worst nightmare. Matilda uttered a cry. Loisel, already half undressed, arose in dismay and went immediately on foot in search of the necklace. Q. 10. Do you think M. Loisel had an enjoyable evening at the ball ? Give reasons for your answer. [Board Term-II, 2015 Set 2/1/1] Ans. I think M. Loisel didn’t enjoy much at the ball. In fact, he was not much interested in such parties. He had come there for the sake of his wife’s happiness. So when his wife was dancing with enthusiasm, intoxicated with pleasure, he waited for her (but without disturbing her) half asleep in one of the little salons past midnight.









Q. 5. Imagine Matilda got to know that M Loisel had given her four hundred francs, which he had saved to buy a gun. How do you think she would have reacted? [CBSE-QB, 2021] Q. 6. Why was Madam Loisel shocked to know at the end of the story? (CBSE 2017) Ans. Madam Loisel, who had borrowed a necklace from Madam Forestier, to wear at the minister’s ball, had lost it by the time she was back home. Both her husband and she could not find it and decided to replace it with a new one. This costed them thirty six thousand francs and ten years in toil and suffering. At the end of the story, she was shocked to know from Madam Forestier that her necklace (diamond one) was fake and did not cost over five hundred francs. Q. 7. Why was Matilda Loisel always unhappy ? [Board Term II, 2017, 2016 Set 2/1]

These questions are for practice and their solutions are available at the end of the chapter



Q. 2. Madam Loisel now knew the horrible life of necessity. Do you think Madam Loisel accepted this change willingly? Give two reasons in support of your answer. [CBSE-QB, 2021] Ans. Yes, Madam Loisel came to know the horrible life of necessity because after losing the necklace their life changed drastically. Matilda had to do all the household chores. She had to do away with her maid and carry down garbage to the street on her own. She had to wash the dishes and do all the shopping from the market with a lot of haggling. Her husband also had to work hard. Though she found it hard to accept the reality, but in the end, she knew she had to do all this in order to re-pay the money they borrowed. Q. 3. Mention two things you would have done, other than what M Loisel did, to help resolve the problem of the lost necklace. [CBSE-QB, 2021] Ans. If I would have been in M. Loisel’s place, I would have strictly asked my wife to repay the lost necklace. I would have stood by her and asked my friends and family members to help her in the hour of distress. I would have, as a responsible person, taken help of the police as well. Q. 4. Mention one of the most pleasurable things, according to you, that money can’t buy and support your choice with an appropriate reason. [CBSE-QB, 2021] Ans. Money can buy us everything but satisfaction and peace of mind. One should always be happy with what one is having and not think of what there’s to have. You as a person should always be happy in the situation God has kept you. He always thinks the best for you. The more contended you are, the more happy you would be. It is not the luxuries in life which make you a better person. You become better only by your good deeds and actions. So try to be a good human being first rather than complaining one. In Matilda’s case she always complained of more money, more comfort what she got in the end was totally opposite to what she expected out of life. So never expect, only go with natural flow in life.





Matilda was born in a middle class family but she was extremely self-indulgent and dreamt of a life full of luxuries. She ignored the advice given by her husband of wearing natural flowers. Instead borrowed the necklace and lost it. Her downfall was caused by her unhappiness with whatever life offered her. In paying for the necklace, she invited a poor life of necessity and deprivation.















 Answer organised effectively  usage of words for elaboration and cause-effect  ½ mark when either aspect is missing Deduct ½ mark from the overall score if the error density is high (more than a total of 2 spellings and grammatical errors). [CBSE Marking Scheme, 2017] Detailed Answer :



Oswaal CBSE Question Bank Chapterwise & Topicwise, ENGLISH LANG. & LIT., Class-X

@PROCBSE

THE NECKLACE 



eye admired her. But the happiness was shortlived and degenerated into shock of the worst nightmare when she discovered that the necklace was missing from her attire. Her mind was ridden with forebodings of how she would face Madame Forestier and pay up for the loss.









Q. 11. Why was Matilda sad after the ball? [Board Term-II, 2014 Code-2/1/3] Ans. The night of the ball had been the realisation of a long cherished dream for Matilda, when she was the focus of all the attention and every

Long Answer Type Questions



(100-120 words & 6 marks each)











These questions are for practice and their solutions are available at the end of the chapter

















































20th July, Monday 9:00 pm I couldn’t recognise Matilda at all. I wish she had told me the truth about the necklace. Ans. 20th July Monday 9:00 PM I couldn’t recognise Matilda at all. I wish she had told me the truth about the necklace, which she didn’t. I feel so sorry for her. The poor lady had to go through so much just for the sake of one artificial necklace worth 400 francs. I am surprised she hid this all from me fearing I would demand the real price of the necklace from her. Oh! I wish she should have at least spoken to me once, so that I could have got her out of this predicament. My innocent friend, Matilda, whom I love so dearly, had to go through this pain of having lost my necklace in the ball. She during this course of life has become a much tougher person. She no longer is that sweet face of woman she used to be. It clearly pains to see her like this. I would like to help her. Madam Forestier Q. 2. Imagine you’re one of the columnists in a magazine. People write to you about their issues, seeking your advice. Matilda writes to you about the guilt she feels for having lost a necklace and how it changed the course of her life and her husband’s. What would [CBSE-QB, 2021] be your advice to her? Ans. The Editor Mr Jay, It’s a pleasure to write of my woes in your column. You are the best adviser to everybody’s problems. You lent a patient ear to each of your guests. Now listen and be the best adviser to me. I am into depression mode as I recently lost a diamond necklace of my best friend which I had borrowed for a party. No amount of pacifying from my husband M Loisel could bring me out of this feeling. Though I was cynosure of everybody’s eyes in the party. The loss of necklace brought it all down. After losing it, I got so worried as how I will face my friend now. All through this my husband has been very supportive of me, his immense love and faith me in me sailed me through this time. He did all in his capacity to get me out of it. To repay the

necklace, we bought her another necklace which has made our lives very difficult and we now live a life of penury with meagre income to support us. How should I help myself to get out of this! Mr. Jay : Well Matilda! Whatever happens, happens for the best. So put your worries across and think of your future ahead. You have suffered enough, now think of brighter side of life and learn from your mistakes. I am sure you will be out of it soon. Q. 3. Matilda goes home after meeting Madam Forestier after ten years and shares the fact with M Loisel that the necklace was a false one. Write the dialogue between Matilda and M Loisel. Matilda: I don’t know how to tell you this but I met Madam Forestier today and she told me that her necklace was M Loisel: What! I can’t believe my ears. That huge debt was for nothing! Matilda: ......................................................................... M Loisel: ........................................................................ [CBSE-QB, 2021] Ans. Dialogue between Matilda and M. Loisel _________ Matilda : I don’t know how to tell you this but I met Madam Forestier today and she told me that her necklace was false one. M Loisel : What! I can’t believe my ears. That huge debt was for nothing! Matilda : Yes, M Loisel it is really surprising that we had to go through so much. I don’t know what God had in store for us. M Loisel : Shocking it is really! Unbelievable. She told you all this herself. Matilda : Madam Forestier was shocked, even she could believe that we had to go through so much just for the sake of this false necklace. M Loisel : Now what do you think we should do? Matilda : Well! We can ask her to give back the real diamond necklace to us. So, that we can repay our debts. M Loisel : What if she refuses? Matilda : Well! Madam Forestier is a friend of mine. She will never refuse. I am sure. Let me just speak to her. M Loisel : Do what you feel like. Q. 4. Your teacher conducted a Turn Tables in class, and asked you to put forth your views on ‘Frugality in Life is the Ideal Way to Live’. Write the debate script with points to supplement your stand, both for and against the motion and conclude with your own views. [CBSE-QB, 2021]











Q. 1. Madam Forestier writes a diary entry after having met Matilda, knowing the truth and seeing the transformation in her friend. Write a short diary entry as Madam Forestier about your encounter and how you felt. You may begin like this. [CBSE-QB, 2021]

281

@PROCBSE Oswaal CBSE Question Bank Chapterwise & Topicwise, ENGLISH LANG. & LIT., Class-X











Ans. •













A friend to Loisels –Mme Forestier helps her in their hour of need. • Mme Loisel needs necklace to wear in the party. • Generous: opens up the entire array of her jewels for Loisels to choose from. • Considerate when Loisels delay the return of the necklace. • At the end of the story honestly reveals to Matilda that necklace was fake. • Mme Forestier – like a true friend feels bad for her unnecessary sufferings. [CBSE Marking Scheme 2019] Detailed Answer : Mme Forestier was Matilda Loisel’s childhood friend. When there was a party at the education minister’s house, Matilda went to her to borrow some jewels. She quite generously asked Matilda to choose any jewel. Matilda chose a superb diamond necklace. Mme Forestier immediately agreed to lend it to Matilda. Unfortunately, the necklace was lost. Mme Forestier was considerate when Matilda could not return the jewels on time.

















Ans. Suggested Value Points : l Matilda longs for life of luxury and wealth, immerses herself in life of glamour, borrows a necklace when invited for a party, loses it, pays

heavily, she and her husband work day and night, live in poverty to repay for it, she loses all her charm, looks old and pale, it’s better to be satisfied what you have, do not show off. (Board Marking Scheme, 2020)

Detailed Answer : It is very essential for a person to live a contented life. He should believe in simple living and high thinking. Not everyone is born with a golden spoon in his mouth. With honesty and hard work, we can always achieve our best. A person will never be happy if he keeps craving for what he doesn’t have. In this way, he may ruin even his present. Matilda had the same longing and craving for a life of luxury and wealth. She immersed herself in glamour when her husband got an invitation to attend a ball. She didn’t care for her loving middle-class husband, rather she was running after her fantasies. She always thought beyond her limits. For the minister’s party, she could have gone in her simple clothes, but she insisted on buying new dress for four hundred francs which her husband gave her. Then she longed for jewellery, which she borrowed from her friend for the party. She borrowed a diamond necklace and lost it. For repaying the diamond necklace to the friend, she had to sacrifice many years of her life, as well as her husband’s whole income. Both of them lived in utter poverty to repay for it. Matilda lost all her charm and looked old and pale. Her ostentation and vanity had landed her in trouble. Thus, it is apt that it is better to be satisfied with what one has. One should not believe in false show off. Q. 8. Mme. Forestier proved to be a true friend of the Loisels. Elaborate. (CBSE OD Set-3 2019)









Thanks any ways. Q. 6. Read the following quote. “We are too involved in materialistic things, and they don’t satisfy us. The loving relationships we have, the universe around us, we take these things for granted.” - Mitch Albom Matilda was never satisfied with her life and desired more. The given quote reflects her character. Justify. [CBSE-QB, 2021] Ans. Matilda always longed for luxuries in life. Money, glamour and luxuries were a part of her life. She didn’t care for her middle class upbringing, she always kept day dreaming about materialistic pleasures in life. She didn’t even bother about her husband’s feelings that how much he cared and loved her. For the ball, she insisted on buying the most beautiful dress, so that she could look the most beautiful in the ball. She longed for appreciation and admiration from all. She kept running after her fantasies and in the end lost everything in the bargain. Through all this ordeal, it was only her husband who stood by her. Thus, it is better to understand that one should be satisfied, with what one is having and not believe in false show off. Q. 7. Ostentation and vanity often land people in trouble. Matilda is an apt example of this. Justify. (100-120 words)





Q. 5. One of your friends has to attend a class party at his / her school and wants to purchase the most expensive clothes and shows. He/ She feels that this would make him/ her stand out in the crowd. You receive a call from your friend seeking advice on this matter. Write down that telephonic conversation between your friend and yourself. You may begin this way: Friend: Hey, I want an honest opinion from you. Will you please help me? You: ................................................................................ Friend: So, there’s this class party I have to attend. It’s a big deal! You: ................................................................................. ....................................................... [CBSE-QB, 2021] Ans. You : Frankly, the ideal way to live is being as simple as possible. Buying a costly dress doesn’t mean you would look the best in the party. One can look pretty even in a simple attire. So, go for something very normal in best colours. Friend : Just suggest something........... You : May be a saree. It makes a woman look beautiful in all situations in all times. So,why don’t you go for it. There are a number of varieties like to choose from. Friend : That’s a good idea. Let me just try it. You are the best adviser when it comes to suggest something 















282

@PROCBSE

























[CBSE Marking Scheme 2018]



283

Detailed Answer : It is very essential for a person to live a contented life. He should believe in simple living and high thinking. Not everyone is born with a golden spoon in his mouth. With honesty and hardwork, we can always achieve our best. A person will never be happy if he keeps craving for what he doesn’t have. In this way, he may ruin even his present. This world is in deed made of materialistic things. But we should never run after that. In this story, Matilda always ran behind her unlimited desires and wants. As a result, she had completely ruined her life. She didn’t care for her loving middleclass husband. But she was running after her fantasies. She always thought beyond her limits. For the minister’s party, she could have gone in her simple clothes, but she insisted on buying new dress for four hundred francs which her husband gave her. Then she longed for jewellery, which she borrowed from her friend for the party. She borrowed a diamond necklace and lost it. For repaying the diamond necklace to the friend, she had to sacrifice many years of her life, as well as her husband’s whole income. She should have known that a person’s goodness and character makes one beautiful and not the beautiful dresses and attire.

Ans. (4 for Content; 2 for Fluency; 2 for Grammatical Accuracy) (i) Live a contented life (ii) Best quoted phrase is ‘Honesty is the best policy’ (iii) Simple living and high thinking (iv) we should not run after materialistic world (v) we must not think beyond our limits

Solutions for Practice Questions (Topic-1) Ans.4. For the motion : Being frugal is the quality of a prudent person who is economical in the consumption of resources available to him. These resources can be in any form be it food, clothes money or just about anything. Avoiding wastage of any kind is the greatest virtue of man. We should always remember that resources available to us by mother earth are scanty, we should make judicious use of them. Today, the whole world is hit by deadly Corona, people are out of jobs, some are working from home, some are doing nothing. In such a scenario, the prudent use of money becomes all the more important. So, always use money wisely. Against the motion : There are no two ways about it that money makes the world go round. It’s the power of money that rules the world in today’s time. The more we flaunt it, the more progressive we are thought to be. Everybody loves to possess it. So, if we have more why not just show it. After all being thrifty always doesn’t pay, one needs to look for one’s own comfort as well. Spend what you have today, do not bother for tomorrow. For no one knows what tomorrow has in store for us.







Her friends were rich and full of money, power but she was married to a simple clerk, had inferiority complex.



Long Answer Type Questions

Ans.5. I don’t think so she would have reacted any better. She was self-conscious woman who only thought of her own comforts in life. Though her husband M Loisel loved her dearly, she never reciprocated in the same way as he did throughout the story. She was highly impulsive and impatient woman who reacted very angrily to the situation. Even if she would have come to know that M. Loisel had kept that money to buy that gun she would not have bothered as she wanted to look the best in the ball. Ans.8. Value Points :

Short Answer Type Questions







[CBSE Marking Scheme 2016] Detailed Answer : Matilda did not want to see her rich friends because she suffered from inferiority complex. She was married to a simple clerk; whereas her friends were rich and full of money power.



REFLECTIONS





THE NECKLACE

At the end of the story, when she learns that Matilda had given her real jewels in place of the jewels she had lost, Mme Forestier did not hide the truth from her friend. She immediately told Matilda that her jewels were fake. She did not bother about the fact that she would have to return them. Thus, we can say that Mme Forestier was a true friend of Matilda. Q. 9. People should always try to live within their means. Aspirations have no limits but one should never forget the ground realities. Elaborate on the basis of the chapter, “The Necklace”. (CBSE SQP 2019)







Are you contented with who you are and what you have? Always remember ostentation and vanity often lead people to trouble.

@PROCBSE

CHAPTER

8

BHOLI

– By K. A. Abbas

Revision Notes Scan to know more about this topic

With passing of time, the village progressed, and a high school. When Bholi was of marriageable age, her father fixed her match with a person named Bishamber, as he had BHOLI a big shop, house and a large bank balance. Nearly fifty years old, he also walked with a limp. Not only that, he was a widower and had grown up children. Bholi was not consulted when this marriage was fixed. The day of her marriage came, and Bishamber arrived with a big baraat. He was pleased with the arrangements but when Bholi’s silken veil was removed, he was shocked to see her pock-marked face. He refused to marry her, and said that the marriage would go through only if an additional five thousand rupees were given to him. Ramlal brought the money. When Bishamber was about to garland her, Bholi flung the garland into the fire. She very clearly told her father that she would not marry that man, who was not just only old and lame but also a mean and greedy person. She took a vow never to get married. She decided that she would become a teacher and serve her parents in their old age.



‘Bholi’ is the story of Sulekha. She is a young girl neglected by all. Education transforms her. It inspires courage and confidence in her. She raises her voice for her rights.





Introduction







The main character of this story is a simple girl, named Bholi. Her real name is Sulekha and she is the fourth daughter of Numberdar Ramlal. She fell off a cot when she was ten months old and probably, a part of her brain was damaged. Pretty at the time of her birth, her body was disfigured and at the age of two, by an attack of smallpox. Only her eyes were saved. As late speaker, she could not speak clearly and stammered while speaking children made fun of her. Bholi was the weakest child, while her brother and sisters were healthy. Her sisters, who were good looking, were married off and her brothers were sent to the city to study. When a primary school was opened in their village, Bholi’s father who was a revenue officer, wanted her to study there. He sent Bholi to school even though her mother protested against it. Dressed in new clothes, she was sent to school. Happy to see other girls there, she hoped that one of them would become her friend. The teacher asked her, her name but Bholi was too scared to answer. The teacher smilingly encouraged her to speak. She also asked Bholi to come to the school daily. She gave her a book with many colourful pictures in it. She also told Bholi that she would soon be able to read it and everyone would treat her with respect. Bholi was filled with hope, to do something in her life.

Key Words



1. disfigured: defaced, damaged. 2. pock-marked: sear or marks on the skin 3. stammered: Speak with sudden involuntary pauses.







Summary

@PROCBSE

BHOLI

Extract Based Questions



Ans. Option (D) is correct. Explanation: The Teacher was sure about what she had taught to Bholi. She was confident that her teachings will make Bholi stand for herself and her dignity in the right manner. Q. 5. Pick the option that includes the correct matches of Column A with Column B.

Column A (i)

independent and confident

II.

Ramlal

(ii)

burdenless and free

(iii)

sense of contentment and accomplishment































Ans. Q. 2.















Ans.











(iv) embarrassed and anxious (A) I-(ii); II-(iv); III-(iii) (B) I-(i); II-(iv); III-(iii) (C) I-(iii); II-(ii); III-(i) (D) I-(ii); II-(iii); III-(iv) Option (B) is correct. “What’s the matter with you, you fool”? shouted Ramlal. “I am only taking you to school.” Then he told his wife, “Let her wear some decent clothes today or else what will the teachers and the other school girls think of us when they see her? New clothes had never been made for Bholi. The Old Dresses of her Sisters were passed on to her. No one cared to mend or wash her clothes. But today, she was lucky to receive a Clean Dress which had shrunk after many washings and no longer fitted Champa. She was even bathed and oil was rubbed into her dry and matted hair. Only then did she believe that she was being taken to a Place better than her Home! When they reached the School, the Children were already in their Classrooms.” [CBSE-QB, 2021] (5 × 1 = 5) Why did Ramlal call Bholi a fool? This was because: (A) Bholi had become hysterical and was screaming. (B) Bholi shouted in fear and pulled her hand away. (C) Bholi was behaving foolishly and was running away. (D) Bholi had been behaving very strangely with her Father. Option (B) is correct. Pick the sentence that brings out the meaning of ‘decent’ as used in the extract. (A) He gets a decent amount of salary. (B) One must be decent when having a conversation with strangers. (C) She was dressed in a decent manner for the interview. (D) It was very decent of him to lend me some money. Option (C) is correct. 







Q. 1.







Bholi



































Ans. II.





Q. 4. Why did the Teacher stand in one corner watching the Drama? (A) She was elated to see what was happening. (B) She wanted to see what Bholi would be doing. (C) She didn’t want to interfere in a family matter. (D) She had faith in Bholi standing up for herself.

I.

III. Teacher





Q. 3. Pick the sentence that brings out the meaning of ‘contemplating’ as used in the extract. (A) Contemplating sharing my belongings with someone is definitely tough. (B) She took some time to respond as she was contemplating what to say. (C) I was contemplating my reflection in the mirror and was speechless. (D) She was contemplating through the pages of the document that was with her. Ans. Option (B) is correct. Explanation: In the extract, contemplating is used to mean Planning or intending. This meaning is reflected in option (B).

Column B









(5 marks each)







I. “Ramlal stood rooted to the ground, his head bowed low with the weight of grief and shame. The flames of the sacred fire slowly died down. Everyone was gone. Ramlal turned to Bholi and said, “But what about you, no one will ever marry you now. What shall we do with you?” And Sulekha said in a voice that was calm and steady. “Don’t you worry, Pitaji! In your old age I will serve you and Mother and I will teach in the same School where I learnt so much. Isn’t that right, Ma’am?” The Teacher had all along stood in a corner, watching the Drama. “Yes, Bholi, of course,” she replied. And in her smiling eyes was the light of a deep satisfaction that an Artist feels when contemplating the completion of her Masterpiece.” [CBSE-QB, 2021] (5 × 1 = 5) Q. 1. Ramlal stood rooted to the ground because he: (A) was moved by what he heard. (B) was influenced by Bholi’s words. (C) was in a state of shock. (D) was in an immovable position. Ans. Option (C) is correct. Q. 2. Bholi had refused to get married as: (A) Her father couldn’t afford the dowry that was demanded. (B) The bridegroom had been greedy and was disrespectful. (C) The bridegroom had insulted her father. (D) Her father was getting her married to a man older to her. Ans. Option (B) is correct. Explanation: Bholi was a girl of self-dignity. She was against Dowry and did not want to marry a person who wanted Dowry or could not show due respect towards others – especially Women.

285

@PROCBSE Oswaal CBSE Question Bank Chapterwise & Topicwise, ENGLISH LANG. & LIT., Class-X





(D) It was unkempt and oiled. Ans. Option (B) is correct. Explanation: Bholi’s hairs were never oiled or combed earlier, so they looked matted and everyone in Class made fun of how her hairs looked. Q. 5. What fear did Bholi have when she was told about being taken to School? (A) She thought she was going to be neglected by her parents. (A) She thought her parents were going to get rid of her. (C) She felt that she would be thrown out of the house and sold. (D) She thought her parents were thinking of ways to throw her out. Ans. Option (C) is correct.







































Explanation: In the extract, decent has been used to mean proper attire, which is the same as used in option (C). Q. 3. Pick the option that best describes how Bholi felt at the end of her first day in school. (A) Lost and scared (B) Calm and Peaceful (C) Elated and Peaceful (D) Hopeful and Elated Ans. Option (D) is correct. Explanation: Bholi was hoping to make new friends and get accustomed to her new class and school very soon. Q. 4. Why was Bholi’s hair matted? (A) It was entangled and oiled. (B) It was never oiled or combed. (C) It was not combed regularly.



286

SUBJECTIVE TYPE QUESTIONS Short Answer Type Questions



























Ans. No new clothes for Bholi, old dresses passed on to her, nobody cared for or mended her clothes, parents sent her to school as they thought there was a little chance of her getting married. (Board Marking Scheme, 2020) Detailed Answer : Bholi was quite a neglected child at home and this is evident from the text. No new clothes were made for her and she was always passed on old dresses of her elder sisters. No one cared to bathe her or oil her hair or even wash or mend her clothes. The neglect was probably due to the fact that she was the youngest in a household of seven children. It might be that she failed to draw attention and affection of her parents because of her disadvantageous looks or her physical setbacks. Her parents sent her to school as they thought that there was a little chance of her getting married. Q. 7. Bholi’s heart was overflowing with a ‘new hope and a new life’. What does the phrase ‘the new hope and the new life’ means? (CBSE, SQP 2018-19)





























Ans. To serve her parents in old age and to teach the students in the same school where she had learnt [CBSE Marking Scheme, 2018] a lot. Detailed Answer : The phrase means Bholi’s heart was overflowing with a new hope and a new life when she took a vow to serve her parents in their old age and to teach the students in the same school where she had learnt a lot. Q. 8. Why was Sulekha nicknamed Bholi ? [Board Term II, 2016 Set 2/1]

These questions are for practice and their solutions are available at the end of the chapter





Q. 6. Bholi was a neglected child. Explain. (CBSE SQP 2020) [Board Term-II, 2013 Code-2/1/3] 





Q. 1. Ramlal was worried about Bholi as she didn’t have good looks. Counter the belief that it’s important for a girl to be good looking and give a reason for the same. [CBSE-QB, 2021] Ans. Bholi’s father was always worried about her because she not only looked ugly due to pock marks but also stammered while talking. But in today’s time it is not that important. Bholi very aptly proved this wrong despite her ugly looks. She turned out to be a masterpiece who fought against the evils of the society by refusing to marry Bishamber Nath. Q. 2. How did you feel when you read about Bholi being ignored and ill-treated by her parents as a child? [CBSE-QB, 2021] Ans. It is really feels sad that a dumb cow like Bholi who was so good at heart was ill-treated by her parents. Bholi was a gem of a person who deserved much better treatment from her loved ones. All she needed was love and care. Q. 3. Do you think Bholi could have confidently refused to marry Bishamber if she weren’t educated? Give reason/s. [CBSE-QB, 2021] Q. 4. How would you have reacted if you were one of the guests witnessing Bholi’s wedding when she refused to marry Bishamber Nath? [CBSE-QB, 2021] Ans. I would have fully supported Bholi if I were present there. As a mature and aware person, I feel what she did was absolutely right. She has every right to refuse to marry a person who demands dowry and is of double her age. If I would have a daughter like Bholi, I would be the first person to appreciate her boldness. Q. 5. Do you think Bholi’s father would have agreed to the match if her mother hadn’t insisted upon it? Why/ Why not? [CBSE-QB, 2021] 







(40-50 words & 3 marks each)

@PROCBSE

BHOLI















































Ans. Bholi found her school teacher to be different from her family members because the teacher was a kind and considerate lady. She had a soft and soothing tone. She gave Bholi a lot of encouragement and inspiration which she was not getting from her family. Q. 12. Bholi’s parents accepted the match for her though Bishamber Nath was nearly the same age as her father. Bholi also accepted her parents’ decision without voicing her opinion. (a) What does this tell you about the social conditions prevailing in our society? (b) How do you think these problems can be addressed? [CBSE-QB, 2021] Ans. (a) The acceptance for Bishamber Nath as Bholi’s groom says a lot about the social conditions in our society. Bishamber Nath was nearly the same age as Bholi’s father but he was accepted by her parents as he was not demanding any dowry. But later, he refused to marry her and demanded a sum of five thousand rupees as dowry else he would leave without marrying Bholi. The dowry evil is prevalent in our society. (b) Our society is facing the problems of discrimination against girl child and the evil of dowry. Both these can be addressed if the girl child gets educated. Girls should be given equal importance in a family and should not be deprived of higher education. Q. 13. You’ve read the quote: ‘A teacher affects eternity. He can never tell where his influence stops.’ In the context of this statement, comment on the role of Bholi’s teacher in her life.[CBSE-QB, 2021] Ans. Bholi found her school teacher to be different from her family members because the teacher was a kind and considerate lady. She had a soft and soothing tone. She gave Bholi a lot of encouragement and inspiration which she was not getting from her family.

















Ans. Value Points : When 10 months old, she fell off from the cot. Head and some part of her brain got damaged, stammered. She remained a backward child and came to be known as Bholi (= the simpleton) [CBSE Marking Scheme 2016] Detailed Answer : When Sulekha was 10 months old, she fell from the cot. Due to that accident, her head and some part of brain got damaged. She also stammered. She remained a backward child. Being simple, she was nicknamed Bholi. Q. 9. Why did Bishamber’s marriage with Bholi not take place ? [Board Term-II, 2015 Set 2/1/3] OR Why does the marriage not take place ? [NCERT] Ans. When Bishamber greedily demanded five thousand rupees as a condition to marry a girl with pock marks on her face, Bholi refused to allow him to garland her and told him that he was a contemptible person. Q. 10. For what unusual reasons was Bholi sent to school? (CBSE Compt. 2018 Set-1, 2, 3) [Board Term-II, 2015 Set 2/1/3] [NCERT] Ans. The Tehseeldar Sahib put the responsibility of sending the girls of the village on Ramlal, the revenue official as he was the representative of the village. Ramlal had not the courage to disobey him. He also felt that there was a little chance of Bholi’s getting married due to her ugly face and lack of sense. Q. 11. How did Bholi find her school teacher different from her family members ? OR Did she find her teacher different from the people at home ? [NCERT]

Long Answer Type Questions



(100-120 words & 6 marks each)











Ans. Value Points Students may draw upon the following: • Introductory sentence:  crucial and indelible role played by parents  parents influence their children both implicitly (indirectly) and explicitly (directly)  are role models, first teachers, friends, companions, guides • Critical examination  Richard H. Ebright’s mother- loving, caring, understanding, a friend, a companion

















 



















Q. 1. Parents play a crucial role in the upbringing of their children. Critically examine the parents of Bholi and Ebright, highlighting their impact on their children’s lives. [CBSE-QB, 2021]

287

filled the vacuum in Ebright’s life in his father’s absence invested time and energy in his upbringing which made him see heights of success Bholi’s parents – uncaring, indifferent, biased, insensitive, had a patriarchal mindset traditional outlook- did not believe in the education of girls, totally indifferent to Bholi and her needs, neglected her did not bother to groom her (oil her hair, give her good clothes to wear) sent her to school not to educate her but to save their own face despite their prosperity, they left Bholi to her own misery, worst was when they decided to marry her off to Bishamber

@PROCBSE 288

½ mark for identification of the error (CBSE Marking Scheme, 2021)









You : Yes, in a very hard way. I studied Chemistry honours in college and topped became a lecturer in college. Friend: Wow! That’s great You : Thanks After reading Bholi’s story you decide to write a blog on the importance of educating the girl child and how it empowers her. Write that blog expressing your views. [CBSE-QB, 2021] www. blogspot.com Hi friends! I am a proud female, too proud of my lineage. I find it extremely honourable that God has made woman. She is the most beautiful creation of nature. This world would not have been what it is today if women were not there. They make the world a beautiful place to live in. I, being a woman love to explore all the challenges that life throws at me. In today’s time woman are equivalent to man in all respects. They are part of all mainstream activities involving both the sexes be it male or female. Being a modern, confident women gives me immense strength and pleasure. It feels highly respected when females like Indira Nooyi, Melinda Gates and Nita Ambani are respected and accepted by one and all in society. So, friends never feel you are inferior, you are the noblest creation of God, Just love your womanhood. Good luck! Have a great day! Imagine you are Bholi and you have been invited by a girl’s secondary school in another village to address a group of students about being independent, fearless and breaking stereotypes. Write that speech. [CBSE-QB, 2021] Hello Students! It is an honour for me to be amongst you people and speak of my life experiences. Today, I teach in my village school. I am a senior teacher teaching senior classes. But it was not so with me in my initial years. While growing up, I was a very hesitant and nervous child who even could not face people. I was so shy to meet anyone. I stammered and due to my ugly looks felt so low in confidence that I thought I would not be able to do anything in life. Even my parents thought so about me. But one fine day, I landed in a school and my life changed after this. I started studying and education brought lot of changes in me. It made me what I am today, a woman of substance whom everyone loves. So believe in your abilities and let life take it’s own course. Love you all! Take care Have a good day. Bholi is a child different from others. This difference makes her an object of neglect and laughter. Elaborate. [CBSE 2019]



Ans.









Q. 4.













Q. 5.







Ans.



…………………………………………………… Friend: Hmmm. Yes, I agree, though we are in the 21st century it still continues in a blatant manner in many parts of the country. You: ……………………………… [CBSE-QB, 2021] Ans. You: Very true it is prevalent even today. After so much of heated debates, still the condition is same in every corner of the country. We, as Indians, have still not got rid of this age old system. Friend: Have you ever faced it in your life? You: Many times, being a girl child in my family and that too belonging to a Rajput community, I have faced it many times. Friend: Can you give one instance, when you felt it? You: Well! During my grown years, I always felt that my brother was given special treatment. I wanted to study Genetic Engineering but my father refused saying that he needs to give my brother a chance to study engineering. He could not afford our academic expenses. Friend: Did you fight it?



















You: ……………………………………………………





















Detailed Answer : Parents play the most crucial role in child's life. They are the child's first teacher, who provide comfort and security, which has a direct impact on the development of the child's brain Ebright's mother played the role of a friend, philosopher and guide. She fulfilled the vacuum created by the death of his father. She tirelessly assisted him in his research on butterflies and provided everything necessary to enhance his knowledge. It was only because of her that Ebright became a renowned scientist. Contrary to Ebrights mother, Bholi's parents detested her from the day of her birth on account of her gender. Their insensitive behaviour proved detrimental to Bholi's physical, and mental development. Bholi grew up to be timid child who would stammer much to her parents' annoyance. In a nutshell, love, care and guidance from parent make a difference in shaping the child's future. Q. 2. You and your friend have a conversation about prevalent discrimination between boys and girls, even today. Write the dialogue. You may begin like this: Friend: I don’t think discrimination in terms of gender is a thing of the past.











Q. 3.





• 

(Accept any other relevant content point that lends itself to critical examination) Concluding thought: The contribution and companionship of parents cannot be denied. Love, care, and guidance from parents makes a difference, sculpts children and shapes their future. 



Oswaal CBSE Question Bank Chapterwise & Topicwise, ENGLISH LANG. & LIT., Class-X

@PROCBSE BHOLI









Ans. Value Points : Bholi stammered – had pock marks on her face –was ill treated by everyone –was sent to school as she supposedly had no future – teacher’s love and affection moulded her – became confident, bold, didn’t accept to marry lame, old, greedy Bishamber.









Detailed Answer : It was the teacher’s love and her education that transformed her totally. Bholi gives out great lessons to women at large. She is a symbol of the value of education, marriage and general well-being of girls. Earlier, she was like a dumb cow. But education made her a woman of substance. Infact, her teacher was behind her selfconfidence, self-respect and self-determination. She now had the capacity to know what is right and what is wrong. She was fully aware of her rights and so she rejected the proposal of the greedy man, Bishamber. She took the right decision by telling her parents that she would serve them in their old age. In this way, she not only showed her love and affection for her parents but also became an inspiration for all the girls. Q. 5. Bholi was believed to be a ‘dumb cow‘. What turned her into a fearless, bold and confident girl?











Ans. Value Points : Hints : Self–confidence, love and affection for parents, self-respect, determination, the teacher made her aware of her rights, rejected the proposal of the greedy man, decision making power. [CBSE Marking Scheme]

289





























society does not tolerate differences very easily • slow for her age ; stammers when speaks • small pox leaves her all covered with pockmarks -- suffers a lot • parents do not even bathe her -- ignored and neglected • taken as a burden • people laughed at her • children imitate her when she speaks • hence remains silent most of the time - has no confidence or self esteem • society must accept those who are different -- treat them with the same love and respect as others [CBSE Marking Scheme] Detailed Answer : Bholi was the youngest child of the Numberdar Ramlal. When she was ten months old, she had fallen off the cot on her head and perhaps it had damaged some part of her brain. That was why she remained a backward child and came to be known as Bholi, the simpleton. When she was two years old, she had an attack of small-pox. Only the eyes were saved, but the entire body was permanently disfigured by deep black pock-marks. She could not speak till she was five, and when at last she learnt to speak, she stammered. These were the differences in Bholi that made her an object of laughter by her siblings and an object of neglect by her own parents. She was taken as a burden by her family. She remained silent most of the time as children imitated her. She had lost self-confidence But the society must accept those who are different. They should be treated with same love and respect as others. Q. 4. “Don’t you worry, Pitaji! In your old age, I will serve you and mother.‘‘ Through the statement, the narrator wants to highlight moral values Bholi was imbued with. Based on the reading of the lesson, what made Bholi aware of he rights and how did she use them ? [CBSE 2016]



Ans. •





Detailed Answer : Everyone called Sulekha as Bholi, the simpleton. She used to stammer and her face was disfigured by pock-marks. She was not wise and not beautiful. She was ill-treated by everyone. She was sent to school as she supposedly had no future. But her education changed her personality. Her teacher’s love and affection moulded her. She became confident and bold. She knew what was good or what was bad. Her father agreed to marry her to Bishamber. But she didn’t accept to marry the lame, old and greedy man. Education had, thus, turned Bholi into a fearless, bold and confident girl. She even chose her future rightly. She told her parents that she would serve them in old age. Thus she became an inspiration for all girls and she became a teacher in the same school from where she had studied. She took a very bold decision of rejecting Bishamber.





Commonly Made Error Many students got confused with the names of Ramlal and Bishamber Nath. They also mentioned the incorrect amount that Bishamber Nath demanded. It was `5000 not `500.





Answering Tip Thorough reading of the chapter is advisable.

@PROCBSE 290

Oswaal CBSE Question Bank Chapterwise & Topicwise, ENGLISH LANG. & LIT., Class-X

Solutions for Practice Questions (Topic-1) Ans.3. No, I don’t think so, Bholi would have refused to marry Bishamber in that case. In the story, it was her going to school that brought the positive change in her and stood for her rights otherwise in the story she has been portrayed as a weak girl.

Ans.5. No, he wouldn’t have agreed to it. In the story, Ramlal is shown to be a concerned and loving father who had immense love for her youngest daughter. Even though he confronted her wife and warned her to marrying off Bholi to Bishamber but it was his wife who insisted on her getting married to this old man.

Short Answer Type Questions





REFLECTION ‘No body is perfect. We all have our own flows and strengths. We should remember our strengths while facing problems’. Reflect on the above statement.

@PROCBSE

CHAPTER

9

THE BOOK THAT SAVED THE EARTH – By Claire Boiko

Revision Notes Scan to know more about this topic

Introduction

Then Think-Tank guessed that it was a communication satellite. He ordered them to take vitamins to help them transcribe the code. Omega managed to read some of the nursery rhymes The book that like—’The Cat and the Fiddle’, saved the earth ‘Mistress Mary’ and ‘HumptyDumpty’. Think-Tank interpreted the dreams in his own way and concluded that the earthlings were more powerful than them and that they were planning to attack them. Think-Tank got scared and decided to migrate to Alpha Century, which was one hundred million miles away from Mars. Noodle who had become friendly with the people on the Earth, took over as the leader. In that way, a book of nursery rhymes was able to save the world.





This play is set in the 25th century at the museum of Ancient History. A historian tells the students how the Earth had been invaded by Martians in the 21st century, but was saved by a book of nursery rhymes.

Key Words

1. Arrogant : boastful, full of oneself 2. Confided: disclosed







This play is set in the twenty-fifth century at the museum of Ancient History. A historian tells the students how the Earth had been invaded by Martians in the twenty-first century, but was saved by a book of nursery rhymes. The ruler of Mars, ‘Think-Tank’ was an arrogant fool. He planned to invade Earth. He confided his plan to Noodle and instructed him to gather information of where his crew was on the Earth. Noodle found out that they, i.e., Captain Omega, Lieutenant Iota and Sergeant Oop were in the Centerville Public Library. They were trying to identify the objects there, but as they had never seen books, could not identify them. Think-Tank thought that the books were a kind of crude refreshment—may be sandwiches. Iota and Omega were ordered to eat them.



Summary

Extract Based Questions



NOODLE: Oh, I hope so, Mighty Think-Tank. I hope so. THINK-TANK: Now, contact the space probe. I want to invade that primitive ball of mud called Earth before lunch. NOODLE: It shall be done, sir. (He adjusts levers on switchboard. Electronic buzzes and beeps are heard as the curtains open.) [CBSE-QB, 2021] (5 × 1 = 5) Q. 1. Choose the option that lists the most appropriate meaning of think-tank from those given below.









I THINK-TANK: Mirror, mirror, in my hand. Who is the most fantastically intellectually gifted being in the land? OFFSTAGE VOICE: (after a pause) You, sir. THINK-TANK: (smacking mirror) Quicker. Answer quicker next time. I hate a slow mirror. (He admires himself in the mirror.) Ah, there I am. Are we Martians not a handsome race? So much more attractive than those ugly Earthlings with their tiny heads. Noodle, you keep on exercising your mind, and someday you’ll have a balloon brain just like mine.











(5 marks each)

@PROCBSE Oswaal CBSE Question Bank Chapterwise & Topicwise, ENGLISH LANG. & LIT., Class-X













































































(A) Option (1) (B) Option (2) (C) Option (3) (D) Option (4) Ans. Option (D) is correct. II. OOP: I haven’t a clue. I’ve been to seven galaxies, but I’ve never seen anything like this. Maybe they’re hats. (He opens a book and puts it on his head.) Say, maybe this is a haberdashery! OMEGA: (bowing low) Perhaps the Great and Mighty Think-Tank will give us the benefit of his thought on the matter.





















































Q. 4. Choose the option that gives you fits the example of an irony from the ones given below. (A) “Who is the most fantastically intellectually gifted being in the land?” (B) “I hate a slow mirror.” (C) “Noodle, you keep on exercising your mind, and someday you’ll have a balloon brain just like mine. (D) “I want to invade that primitive ball of mud called Earth before lunch.” Ans. Option (C) is correct. Q. 5. Choose the option that includes the display of ‘smacking’ as used in the extract.

THINK-TANK: Elementary, my dear Omega. Hold one of the items up so that I may view it closely. (Omega holds a book on the palm of her hand.) Yes, yes, I understand now. Since Earth creatures are always eating, the place in which you find yourselves is undoubtedly a crude refreshment stand. OMEGA: (to Iota and Oop) He says we’re in a refreshment stand. OOP: Well, the Earthlings certainly have a strange diet. [CBSE-QB, 2021] (5 × 1 = 5) Q. 1. Why did Omega bow low before Think-Tank? (A) As a mark of respect and recognition of supremacy. (B) To flatter and appease the character. (C) As a sign of submission and understanding of his strength. (D) To curry favour and goodwill from the character. Ans. Option (A) is correct. Q. 2. Choose the option that associates the person to a haberdashery. (A) Jagdeep is a primary school teacher who teaches English. (B) Tanishq is a tailor who makes garments for men only. (C) Falguni is a chemist who manufactures her own medicine. (D) Asma is an engineer who works on designing space stations. Ans. Option (B) is correct. Q. 3. Choose the option that explains what Think-Tank meant by saying- Elementary, my dear Omega. (A) “It’s simple general knowledge, Omega.” (B) “It’s something that a primary school person won’t understand, Omega.” (C) “It’s quite obviously deducible, Omega” (D) “It’s a little more than complicated, Omega.” Ans. Option (C) is correct. Q. 4. The quality of being crude has been allotted to the refreshment stand because (A) Earthlings have bad eating habits according to Think-Tank. (B) the temperature of the refreshment stand was too high. (C) the refreshment stands are responsible for poor health of the Earthlings. (D) it produces food that is unrefined and unprocessed in nature. Ans. Option (D) is correct. Q. 5. Earth inhabitants are Earthlings and those from Mars are Martians, what are the inhabitants of Venus addressed as in most sci-fi stories? (A) Venusites (B) Venatians (C) Venings (D) Venusians Ans. Option (D) is correct. 











































(A) A tank that helps you think deeper than an average person, if you use its contents regularly. (B) Non-stop flow of ideas from a fixed source about one particular issue. (C) A pool of ideas and solutions to various problems via an individual, group or organisation. (D) A method by way of which people think collectively to identify stray thoughts among them. Ans. Option (C) is correct. Q. 2. “Mirror, mirror, in my hand. Who is the most fantastically intellectually gifted being in the land?” Choose the option that lists the genre having the original reference to the above lines. (A) Biography (B) Science-fiction (C) Mystery (D) Fairy tale Ans. Option (D) is correct. Q. 3. Choose the option that correctly categorizes the phrase “(after a pause)” based on the dialogue given above. (A) aside (B) stage direction (C) narration (D) setting Ans. Option (B) is correct. Explanation: The phrase ‘after a pause’ was used for stage direction indicating a Temporary stop in action or speech.



292

@PROCBSE

THE BOOK THAT SAVED THE EARTH

293

SUBJECTIVE TYPE QUESTIONS Short Answer Type Questions Q. 1. What difficulty do the crew of the space probe face on the Earth ? (CBSE 2019)

informs him that the earthlings did not eat books rather they used them for communication.





















OR Why was the twentieth century called the ‘era of books’ ? Ans. The twentieth century was called the ‘era of books’ because they were the encyclopedias of knowledge. There were books for everything—from ant-eaters to Zulus. Books were used to educate people and teach them. In fact, books were a very important part of life of the people.



Ans. Value Point : The Earthlings did not eat books but used them for communication. [CBSE Marking Scheme 2016] Detailed Answer : Initially Think-Tank considers books as a crude refreshment, may be sandwiches, but later Noodle









Q. 2. What does Noodle tell Think-Tank about the books ? [Board Term II, 2016 Set 2/1/1, 2/1/2, 2/1/3]



Q. 3. How did Think Tank misinterpret innocent nursery rhymes ? Ans. Think-Tank heard Omega transcribe certain nursery rhymes like ‘Mistress Mary’, ‘The Cat and the Fiddle’ and ‘Humpty-Dumpty had a great fall’. On hearing those poems ‘Think Tank’ concluded that the earthlings were superior to them and in fact were planning an attack on them. Q. 4. Why are books referred to as a man’s best companion ? Which book saved the Earth from Martians in invasion ? Ans. Q. 5. What did the historian speak about the books ?









Ans. Space probe lands in a public library; crew shocked because the place looked very strange as they had no idea of books; think the library to be some kind of storage barn. [CBSE Marking Scheme] Detailed Answer : Crew of the space probe was sent to the earth for doing research, since the Martians wanted to invade the earth. Their spacecraft landed in a library. They could not understand where they had landed and what the books were meant for. The place looked strange to them as they though that they had landed in some kind of storage barn.



















(40-50 words & 3 marks each)

Long Answer Type Questions



These questions are for practice and their solutions are available at the end of the chapter















they have been targeted by them. So, decides to escape from Mars. And they go to Alpha century. Q. 2. How did the book change Think Tank’s opinion about the Earthlings ? Q. 3. Who was Omega ? Why did she agree to eat the book ? Ans. Omega is the captain of a spaceship which carried, Lieutenant Iota and Sergeant Oops to Earth from Mars. On reaching Earth they took shelter in Canterville Public Library. But they were confused and were trying to identify the objects (books) there. Lt. Iota had counted them, they were two thousand in number. He thought that library was a storage barn, whereas Sergeant Oops thought them as hots. Just then Omega thinks that perhaps the great and Mighty Think-Tank will give them the benefit of his thought on the matter. After investigating, Think-Tank gives his opinion that the Earth’s creatures are always eating. It must be an eatable item called sandwich. Sandwiches are the main staple of Earth diet. He commands Omega to eat it just to confirm his opinion on books as sandwiches. And Omega obeys ThinkTank’s command and gulps it.

Q. 1. How does Think Tank compare the Martians with the people on the Earth ? What guesses are made by him about the books found on earth? Ans. Think Tank feels that Martians are more beautiful than the people on the Earth. The Earthlings have a smaller head when compared to the Martians who have great and mighty balloon brain which has made them cleverer and more intelligent than the Earthlings. Earthlings are fond of eating so they always keep eating sandwiches. Think Tank mockingly calls the Earth a meaningless place where the people are not so intelligent and brave like the Martians and himself. So, he decided to capture the Earth. Think Tank makes guesses that the books are the kind of crude refreshment stand and calls it a sandwich, or a communication satellite. On Noodle’s suggestion, chemical vitamins are taken which enable Omega to transcribe the code of nursery rhymes on ‘Mistress Mary’, ‘HumptyDumpty’ and ‘The Cat and the Fiddle’. All the poems make Think Tank come to conclusion that Earthlings have a high level of civilization and that





(100-120 words & 6 marks each)

@PROCBSE Oswaal CBSE Question Bank Chapterwise & Topicwise, ENGLISH LANG. & LIT., Class-X



 





















 















Ans. Value Points : • The Martians led by their leader launch an inter-planetary attack on the earthlings. • The Martian space probe reached Canterville Public Library. — They come across the book of nursery rhymes “Mother Goose”. — They misinterpreted the rhyme “Humpty Dumpty” - terrifies the Martian invaders.

— They abort their plan and make good their escape. • “Man is a social animal” (Aristotle) — Peaceful coexistence is fundamental to the survival of human race. — It is totally incorrect to invade another country. — “There are no winners or losers in a war” — The Martians seemed to be ignorant of this reality understood by human race-though little too late. [CBSE Marking Scheme 2016] Detailed Answer : The Martians, lead by their leader Think Tank, planned an inter-planetary attack on the Earthlings. Think Tank misinterpreted the book of nursery rhymes completely and interpreted it in a verbal manner. When Omega read the nursery rhyme ‘Mistress Mary’, he was shocked to discover that on the earth, metals can be grown in fields. He thought that the earthlings had discovered how to combine agriculture and mining and were now able to grow high explosives. When Omega transcribed ‘Hey Diddle Diddle’, he thought that the animals on earth were also ready for a space attack. When he saw the image of HumptyDumpty, he thought that it was his own brain and the earthlings were planning to capture the Mass Central Control and therefore, they got ready to save their own lives instead of invading the earth. Thus, the nursery rhyme book saved the earth from Martian invasion. We know the fact that peaceful coexistence is fundamental to the survival of human race. It is totally incorrect to invade another country. But Martians planned to do so. They seemed to be ignorant of the reality that there are no losers or winners in the war. In fact both the sides suffer. Q. 7. Justify the title ‘The Book that Saved the Earth’. Ans. Martians planned to attack on the earth. The ruler of Mars, Think Tank instructed one of his MartiansNoodle to gather information about the Earth. He saw that other three crew members—Captain Omega, Lieutenant Iota and Sergeant Oop found some books in Centerville Public Library. A book named ‘Mother Goose’ contained nursery rhymes. Omega managed to read some nursery rhymes like ‘Mistress Mary’, ‘Humpty-Dumpty’, etc. Think Tank misinterpreted the rhymes. He was shocked to discover that metals could be grown in fields. He also thought that earthlings are intelligent enough in warfare and have advanced technology to grow high exposures. On seeing the image of HumtpyDumpty, he thought its head resembling that of his own. He feared invasion on Mars by earthlings. So he decided to migrate to Alpha Centauri, one hundred million miles away from Mars. The Martians fear the book till date. Hence, we can say





Q. 4. What changes have taken place in the field of books since twentieth century ? Ans. The twentieth century is often called the ‘Era of the Books’ which had bought a lot of significance along with it. There were books about everything from ant eaters to Zulus. Books taught people how to, when to, where to and why to. The books were illustrated, educated, punctuated and even decorated. The books were supposed to be a mine of knowledge. And there were books on every topic. Then with the advent and invention of computers and electronic media, books have got replaced with e-books and they are completely vanishing. Our present generation has no time for reading books. In future, they might be mistaken for even a sandwich or a strand. This lesson gives us a message that books are always important, as they helped to save us even from a Martian attack. Books are still a man’s best companions. They are our trustworthy friends and always stand by us. They have more patience than human beings. They can never desert us when we fall prey to bad times. They are a constant source of happiness, strength and wisdom. Q. 5. Do you think books are being replaced by the electronic media? Can we do away with the books all together? Ans. The statement is true but only partially. The electronic media has made its presence felt in a big way. Today, all the information, that was earlier sourced by encyclopaedia, is available at the touch of a button on the electronic media. The visual effect is even better when we watch video pictures than those in the books. Moreover, it saves paper and there is lesser impact on the environment in the form of carbon footprints. Nevertheless, despite the conveniences offered by the electronic media, it cannot replace the printed books. Our paper books have their own special charm and will continue to exist with whatever newer option that may be available. Books are written information which guide us on correct path. They are our real friends. They don’t harm us in any way. But we can’t rely totally on electronic media as they certainly have bad effects. It misleads our new generation, they spoil their eyes, time and energy. Moreover it relies on electric power. Q. 6. It is morally incorrect to invade another country/ planet for one’s own benefit. The martians did not understand the value of peaceful coexistence. How did the book of nursery rhymes save the earth from the Martian invasion? [CBSE 2016]









294

@PROCBSE

THE BOOK THAT SAVED THE EARTH



that the title ‘The Book that Saved the Earth’ is apt. The book of the Nursery Rhymes proved to be a most trustworthy friend. It not only saved us and

295

our planet earth, but also acted as a good friend of tiny children and entertained them in its fantasy and delighted them.





Commonly Made Error Students get confused in the names of Omega, Oop and Iota and also the poem.

Read the lesson carefully and try to understand each character in the lesson.

Short Answer Type Questions

Long Answer Type Questions Ans.2. Think Tank was the mighty ruler of Mars who believed that Martians were superior creatures with handsome looks and high intellect. He had little regard for the inhabitants of the earth and planned to invade the earth.





Ans.4. Books have been called a man’s best companion as they always teach him the right things. They are the best companions because your friends may desert you in your hour of need but a book will never leave you or give you the wrong guidance. A book of nursery rhymes saved the earth from Martians in invasion.



Solutions for Practice Questions (Topic-1)







Answering Tip

Later, when he contacted his delegates who had landed on earth and were confused to see the books in the Canterville Public Library, they could not make any sense of what the objects were nor comprehend the information therein, looking at the books. Think Tank ordered the crew to consume the vitamins to enable them to decode the information. Much to the amusement of the readers, he misinterpreted the nursery rhymes, and thought that the earthlings were powerful creatures and were planning an attack on them. He assumed Humpty-Dumpty in the rhyme as himself and fled to Alpha Centauri to save his life.

@PROCBSE

SELF ASSESSMENT PAPER - 04

Max. Marks: 30

Read the given extracts below and answer the questions/complete the sentences that follow : Let me put it more clearly, since no one will believe that a thirteen-year-old girl is completely alone in the world. And I’m not. I have loving parents and a sixteen-year-old sister, and there are about thirty people I can call friends. I have a family, loving aunts and a good home. No, on the surface I seem to have everything, except my one true friend. All I think about when I’m with friends is having a good time. I can’t bring myself to talk about anything but ordinary everyday things. We don’t seem to be able to get any closer, and that’s the problem. Maybe it’s my fault that we don’t confide in each other. In any case, that’s just how things are, and unfortunately they’re not liable to change. This is why I’ve started the diary. (i) Why was Anne Frank disturbed even when she had loving parents, relatives and friends? (A) She didn’t have her own room (B) She wanted a pet (C) She felt her parents don’t understand her (D) She didn’t have a true friend (ii) Why did Anne Frank decide to write a diary? (A) To remember what happened through the day (B) To share her secrets with a trusted source (C) To improve her writing skills (D) To publish it (iii) Which word in the extract means ‘unluckily’? (A) Ordinary (B) Closer (C) Unfortunately (D) Confide (iv) To confide in somebody is to______________ . (A) to motivate someone (B) to be friends with someone (C) to tell about your past (D) tell someone your secrets or personal information (v) How many friends did Anne have? (A) 30 (B) 16 (C) 20 (D) 10 Read the given extracts below and answer the questions/complete the sentences that follow : It was a new bus, its outside painted a gleaming white with some green stripes along the sides. Inside, the overhead bars shone like silver. Directly in front of Valli, above the windshield, there was a beautiful clock. The seats were soft and luxurious. Valli devoured everything with her eyes. But when she started to look outside, she found her view cut off by a canvas blind that covered the lower part of her window. So she stood up on the seat and peered over the blind. (i) The bus was painted__________. (A) blue and white (B) black and yellow (C) green and white (D) blue and orange (ii) What was shining like silver? (A) Seats (B) Handles (C) Windows (D) Overhead bars (iii) What did Valli notice inside the bus? (A) Cushions (B) Flowers (C) Clock (D) Curtains (iv) Which word in the extract means ‘extremely comfortable’ (A) Gleaming (B) Shone (C) Luxurious (D) Canvas (v) Which word in the extract is the synonym of ‘consumed’? (A) Devoured (B) Peered (C) Blind (D) Gleaming Which qualities are common in all the three characters of the play ‘The Proposal’? The poet uses trees as a symbol for women breaking patriarchal notions. Do you agree? Why / Why not? What was the source of Vallis information about the bus? Why did Bholi refuse to marry Bishamber? What were the factors which contributed in making Ebright a scientist? Mme Loisel's disposition invites her doom Elucidate.















Q. 3. Q. 4. Q. 5. Q. 6. Q. 7. Q. 8.





































































































Q. 2.





































Q. 1.





Max. Time: 1 hour

@PROCBSE

WORDS AND EXPRESSIONS-II [Workbook in English] Rationalised Content Following units has been deleted from the latest NCERT Edition Chapters

Pg. No.

Dropped Topics/Chapters

Unit 5

55-70

Full Unit

Unit 6

71-83

Full Unit

@PROCBSE

@PROCBSE

NCERT WORKBOOK- WORDS & EXPRESSIONS-II UNIT

1

A LETTER TO GOD



Exercises

Write the correct order in numbers in the column below (a) (b) (c) (d) (e) (f) (g) (h) (i) (j) (k)

(l)

But it rained heavily and hailstorm thrashed his corn field and all the corn was destroyed. He wrote ‘To God’ on the envelope and posted it. Lencho predicted that it will rain and it did. He and his wife were happy. Lencho felt desperate but believed that God would help him. Their corn field was longing for water. The postman and the postmaster laughed at Lencho’s letter but were surprised by the faith of the man. He wrote a letter to God asking for a hundred pesos to sow his field again and to survive until the next crop came up. After some days, Lencho went to the post office to receive the money, as the postmaster watched him. He immediately went to the counter and asked for a pen and paper, wrote a few lines and posted the letter. Lencho, his wife, and their children lived in a lonely house in a valley. When the postmaster opened the letter, it said “Oh! Thanks for sending the money. But I have received only seventy pesos. Next time you please send the money directly to me. The people at the post office are a bunch of crooks. They have taken the thirty pesos.” He opened the letter and was happy to find the money. He counted and found it was only seventy pesos. The postmaster collected about seventy pesos from his colleagues and sent it to Lencho.





Ans. (i) (j), (ii) (e), (iii) (c), (iv) (a), (v) (d), (vi) (i), (vii) (g), (viii) (b), (ix) (f), (x) (m), (xi) (h), (xii) (l), (xiii) (k) 1. Create a graphic description based on the given summary of ‘A Letter to God’. A. Lencho in his corn field.







(m)

Jumbled sentences













LET’S BEGIN 1. You have read about Lencho in ‘A Letter to God’, First Flight, textbook in English for Class X. How has the story of Lencho moved you? What do you think about Lencho’s faith in God and his attitude towards the officials in the post office? Do you think he is right in asking God for support and then blaming the people at the post office? Discuss his actions with your friends. Why does he do so? Ans. Lencho had unshakable and firm faith in God. It was his faith in God that made him write a letter to God asking for one hundred pesos as help from him. It was the postmaster who did not want that his faith should get shattered. He helped him by sending seventy pesos which he and other post office employees had contributed. On the other hand, when Lencho found thirty less pesos in the envelope, he blamed the post office employees and called them a bunch of crooks. One might think that it was not right on his part to have said so, but he was so innocent that he was expecting a reply from God. He knew nothing about the help from post office employees. For him, his letter had been answered by God and that is why he was upset and wrote another letter to God. 2. The summary of the story of Lencho is given below. The sequence of the story is jumbled up. Read the story again and rearrange the jumbled sentences to make a coherent story.

@PROCBSE Oswaal CBSE Question Bank Chapterwise & Topicwise, ENGLISH LANG. & LIT., Class-X

Ans. Students develop graphic description themselves in the boxes (B, C, D) given in the NCERT Workbook page no. 3 under the guidance of teachers.



300

READING COMPREHENSION





















TEXT I Given below is a story of the tailor who has been known for his skills in cheating and stealing. Read in groups of four or in pairs. You may discuss the events and incidences of the story with your friends and write them in a sequential manner. The Trader and the Tailor There was a tailor who was known for beating others in the art of being light-fingered and about thievery. A trader swore that even with a hundred attempts, the tailor would not be able to take a coil of thread from him without his knowledge. The trader was told that many others, more intelligent than him, had been beaten by the tailor. They said he should not imagine himself so great, for his ego would only give him worse troubles in competition with the tailor. Still, the trader became more competitive and made a wager that the tailor would not be able to rob him of anything. He wagered an Arab horse with those who taunted him. If the tailor failed to steal, then they would have to give him a horse instead. That night, the trader lay awake troubled by the situation and could not sleep a wink. In the morning, he put a piece of satin cloth under his arm, went to the bazaar, and entered the shop and saluted the cunning rogue warmly. The tailor sprang up from his seat and welcomed the trader, inquiring about his health with cordiality exceeding even that of the trader, planting in his heart great feelings of affection for him. When the trader heard these songs of sweetness from the tailor, he flung down the piece of silk saying, “Cut this into a coat for me and make it wide below my navel so as not to hamper my legs and tight about it to show off my figure.” The tailor answered, “O! kindly man, I will do you a hundred services,” and accepted the order. He measured the satin, inspected the working surface, and all the time chatted away to the trader in idle gossip, about other amirs and of bounties and gifts he had received from them, and about misers and their mean ways, and made the trader laugh with hysterics. During this beguiling talk, he was snipping away with his scissors rapidly, cutting as fast as his lips moved. The trader was laughing, the tailor was cutting, the trader closed his eyes in joy, the tailor cut extra pieces, tucking them under his thighs, hidden away from all but God. From his delight at the tailor’s tales, the trader’s former boast went out of his mind. What satin? What boast? What wager? The trader was drunk on the jokes told by the tailor. Then the tailor told such an incredible story that the trader fell over on his back with laughter. The tailor swiftly stitched a swatch of the satin to the hem of his underpants while the trader was paying no attention at all, greedily sucking with every guffaw at the jests the tailor told. The tailor continued to tell funnier and funnier tales and jokes until the trader was completely within his power. With his eyes shut and his reason vanished, the bewildered and boastful trader was drunk with joy. And the tailor continued to cut, filching yet more of the beautiful cloth, with nothing to stop him now. And yet the trader begged for more.

@PROCBSE A LETTER TO GOD (WORKBOOK)

301



You who becomes the slave of the jest, no story is more laughable than you yourself, think on this at the edge of your grave. How long will you listen to the lies of this world that leave your mind and spirit unhinged? The Universal Tailor will cut and stitch the hems of a hundred travellers, silly as children. Eventually, the tailor became bored with the whole story and told the trader that he better leave before another tale was told, “For if I tell another one, the coat will be too tight for you, and you will stop laughing and weep tears of blood.” Rumi (Abridged) Did you enjoy the story of ‘The Trader and the Tailor’? The story is summarised below. Read the story again with your friend and complete the paragraph using short sentences with the given hints. Once, there was a tailor who was known for _________________(a)__________________________ (cheat and thievery). He could cheat anyone with his cunning way of deceiving people. A trader challenged that the tailor would not be able to cheat him. He ____(b)________ (go) to the tailor with _____________(c)_______ _____________________________ (satin). The tailor _____________(d)_________________ (welcome). The trader ___________(e)__________(please) by the tailor’s praise. He told the tailor to make a coat for him. The tailor ________(f)_________ (praise) the trader ______(g)________ (maximum). The trader _________(h)________ (joy). The tailor _________(i)______ (recount) funny tales and the trader forgot _____________(j)_______ (bet). The tailor started snipping __________(k)_____________ and _________________(l)_______________ (hide). Eyes of the trader ________________(m)____________________ (tears and joy) and yet the trader kept on asking for _______(n)__________ (stories). At last, the tailor told the trader that _______________(o)______________ (tight) and _______________(p)_____________________ (weep-blood). Ans. (a) cheating and thievery (i) recounted (b) went (j) his bet (c) a piece of satin cloth (k) the satin cloth (d) welcomed him (l) hid them away (e) was pleased (m) were full of tears of joy (f) praised (n) more stories (g) to the maximum (o) he should better leave otherwise the coat would be too tight (h) closed his eyes in joy/was drunk with joy (p) he would weep tears of blood





















































































VOCABULARY 1. In this story, you have come across a word “light-fingered” which means inclined to steal things. Some such words are given below. Use appropriate words and fill in the blanks.

(a) (b) (c) (d) (e) Ans. (a)











The ______________________ is a unit of distance used to measure distance in space. The farmer bought a ______________________ truck to transport their produce, as it uses less diesel. We had a fairly ______________________ discussion with our colleagues. Rita felt dizzy and ______________________ after skydiving. He is a ____________________, a slightest noise wakes him. light year, (b) light duty, (c) light headed (d) light hearted, (e) light sleeper

















light-headed, light year, light-hearted, light sleeper, light-duty





1. You have come across Relative Clauses in the lesson ‘A Letter to God’. Read about the relative clauses again. Complete the sentences given below in the table by adding the most suitable clause from the box and make it a relative clause. The first one has been done for you. contains meaning of words

need to fill in the application

who stitches clothes

is a crook

tell lies

who help themselves

won the best actor award this year

won the nobel prize

hit a century

believe in hard work











is faithful (a) A tailor is a person who stitches clothes. (b) A dictionary is a book ________________________________. (c) The book is about a scientist ___________________________. (d) What is the name of the player _________________________? (e) I don’t like people _____________________________________. (f) What is the name of the actor __________________________________________________________________?

















GRAMMAR

@PROCBSE 302

Oswaal CBSE Question Bank Chapterwise & Topicwise, ENGLISH LANG. & LIT., Class-X





























































(g) (h) (i) (j) (k) Ans. (a) (b) (c) (d) (e) (f) (g) (h) (i) (j) (k)

Students _____________________________________________ should meet the principal in her office. Friends of the trader thought the tailor ________________ and can cheat anyone. Lencho _______________________ thought God would help him. Those ____________________________ will succeed. God helps those ______________________________. who stitches clothes which contains meaning of words who won the noble prize who hit a century who tell lies who won the best actor award this year who need to fill in the application is a crook who had faith in God who believe in hard work who help themselves



































1. Suppose Lencho gets a hundred pesos in his envelope. He writes a letter to God expressing his gratitude. However, he has missed a word in each line. Help him with those words so that his letter is complete. Dear God! Thank for Thank you for (a) sending a hundred pesos help. _______________ (b) I am very for this kind gesture. _______________ (c) You were my only and your _______________ (d) help has my faith in you. _______________ (e) This money will help me until the crop _______________ (f) and I promise to hard in the future. _______________ Sincerely yours, Lencho



















EDITING

Word Before ..................

Missing Word ..................

Word After ..................

(a)

pesos

as

help

(b)

very

grateful

for

(c)

only

hope

and

(d)

has

strengthened

my

(e)

crop

regrows



(f)

to

work

hard

1. Here is an interesting story. The story has been told in two ways. Story A has all the odd sentences (1,3,5,...) and story B has the even sentences (2,4,6,...). Working in pairs, one of you take up Story A and the other take up Story B. The person with Story A will read out the first sentence and the partner (person with story B) will write in the space provided. Then the person with Story B will read sentence two and the partner will write in the space for sentence two. Likewise, complete the whole story. Once the story is completed, both of you will read out to each other to check whether you have listened carefully to your partner and written the complete story.



the banyan tree.

2.

_______________________________ _______________________________



The Race

_______________________________ _______________________________





Machlipatti used to sit under

1.

_______________________________



Every evening the animals of the

Story B

2.





1.

The Race



Story A





LISTENING

One day Kho-Kho, the rabbit, was feeling bored.

@PROCBSE

A LETTER TO GOD (WORKBOOK)

_______________________________



_______________________________ _______________________________





will run a race with you?

3.



He said, “Let’s run a race!” “Who



3.



_______________________________





mango.

5.

“No, he’s not the fastest runner.

_______________________________ _______________________________ _______________________________





said Bhalu, the bear, eating a



Choo-Choo, the rat, shouted,



5.

No one can run faster than you,”





_______________________________

4.



_______________________________



4.

6.

I can run faster than him.”

Everyone laughed at him.

7.

_______________________________

8.

_______________________________

8.

Then, it was decided to have







11. Kho-Kho and Choo-Choo started

start the race.

11. _______________________________ _______________________________



running.







_______________________________



_______________________________

10. Coocku, the cock, crowed to



point was the neem tree.



12. _______________________________ _______________________________



_______________________________





peepal tree and the finishing

_______________________________



_______________________________



9.

10. _______________________________

12. After running half the distance, Kho-Kho looked back, but ChooChoo was nowhere to be seen.

14. _______________________________

14. When he was near the finishing

_______________________________

point, Choo-Choo jumped over



16. Everyone cheered for Choo-Choo.





15. _______________________________ _______________________________





I’ve won the race!”

the neem tree first.





_______________________________

Kho-Kho’s shoulder and touched





_______________________________

16. Everyone cheered for Choo-Choo.







13. _______________________________



13. “He must be far behind.”

15. He shouted, “I’ve won the race!

Ans. Students should to do the practice under the guidance of their teachers.



Choo-Choo.





The starting point was the

a race between Kho-Kho and





_______________________________







_______________________________

9.



_______________________________

7.



6.

303



1. Lencho, the main character in the story, is a farmer. He supports his family through farming. The conflict in the story happens when a hailstorm strikes the field and destroys his farm. All the hard work, for nothing! The entire land is wrecked by catastrophe. There is no one who can help. Lencho’s complete faith in God is amazing during the calamity.





SPEAKING

So, one type of conflict dominating in the story is man versus nature. Discuss with your partner the natural

2. Try writing a small poem on the topic man versus nature. Read aloud the poem with the required stress and



calamities that play havoc in the lives of people. Collect photographs and make a visual presentation to share in the class. intonation. Given below is a poem written by Norman Littleford, which may help you think and compose. Man vs Nature The heavens roared with thunder as lightning filled the skies

@PROCBSE 304

Oswaal CBSE Question Bank Chapterwise & Topicwise, ENGLISH LANG. & LIT., Class-X

was this God getting angry or Nature’s big surprise. Is Mother Nature telling us the best way that she can to stop the interference and abuse she gets from man. We marvel at her beauty each time we look around then dig up all her treasures from their natural burial ground. We forget that Nature gives us all that keeps us living we take it all and still want more but never think of giving. We build across the countryside progressing every day but Mother Nature can’t progress 

by Norman Littleford





for man is in the way. Ans. Students should practice it under the guidance of their teacher.

1. The tailor here is a cheat, but Lencho in the story, ‘A Letter to God’, is naive and simple. He has immense faith in God. How do you distinguish between Lencho and the tailor? What makes them different? Working in pairs, list the actions and qualities of Lencho and the tailor and draw their character sketches. The given cues will help you to draw a character sketch of these characters.



WRITING

Action: What does the person do?

What quality it reveals?

Lencho lost all his crops.

____________________________________________

He is helpless. Prays to God.

____________________________________________

How does he manage the

____________________________________________

situation? _______________

____________________________________________

Does he harm anyone by

____________________________________________

his action? ______________

____________________________________________

How do others think of

____________________________________________

him? ____________________

____________________________________________

What is your judgement of

____________________________________________

him? ____________________

____________________________________________

He received seventy pesos and

____________________________________________

he thought thirty pesos had

____________________________________________

been stolen.

____________________________________________

Ans. Now, write the character sketch of Lencho and the tailor.



____________________________________________

Lencho

The tailor

Lencho is a poor but hardworking farmer who lost all his crops in a hailstrom. He has immense faith in God. He doesn’t accept the existing situation and writes a letter to God asking for a hundred pesos. His deep faith in God makes him write this letter.

The tailor was a cunning man who was well known for beating others in the art of being light-fingered and thievery. He had a sweet tongue which made the trader have a feeling of affection for him. He was such a clever rogue that he beguiled him with his idle gossiping.

@PROCBSE

A LETTER TO GOD (WORKBOOK)

The trader forgot all about him being a cheat and thoroughly enjoyed his talks. The tailor was so good at fabricating stories that the trader was engrossed in his stories and did not pay attention to his thievery. The tailor made the trader completely within his power. Thus, we can say that the tailor was a rogue who had the art of befooling others.



2. Writing a portrait or description of a person





He is such a simple man that he starts waiting for God’s reply also. The employees at the post office at first laughed a lot at his innocence, but in order not to shake his faith in God, they collected money for him and sent it to him. The situation turns ironic when he feels that the post-office employees have taken the rest of the money and calls them ‘a bunch of crooks’, thus showing that he has lesser faith in humanity.

305

Read the following description about two persons, Raavi and Mohini.

Raavi

Mohini

A portrait or a picture of a handsome young man A portrait or a picture of a middle-aged lady who in full sleeves shirt. is a singer. Raavi is an engineer in a manufacturing firm.

She studied music and became a playback singer.

He likes to make, fix, and repair things.

She likes to be with people and participates in social functions.

He loves to watch television and goes to the movies She loves music and reading books. thrice a month. He is congenial and supportive.

She is firm and a lady with a purpose.



Now, write a description of any one of the above based on the details given. You may follow the process approach to writing. First, you read the given information in the box and make a list of describable (adjectives) points used for each person, in your own words. Secondly, take the help of these points to develop an outline in sentence form. Thirdly, use the outline and prepare the first draft of the description of the person and then edit the draft. Prepare the final copy after making all changes. The table below gives you ideas on how the Process Approach is helpful for improving your writing. Writing: The Process Approach



He does not like to play but goes for a morning She does not get time to exercise. walk everyday.

The processes

What do you do?

Brainstorming

Note down as many ideas as you can about a given topic. Then arrange the ideas in a sequence.

Outlining

Create an outline of your writing — which ideas will go first, how will they be supported by evidence, will there be a picture, etc.

Drafting

Write the first draft; here, the focus is on the content and not the form.

Revising

Revise the first draft, improve it. Add or delete ideas and improve the language.

Proofreading

Proofread the draft with the help of the teacher; here, the focus is on the form and not the content.

Final draft

Write the final draft now.

@PROCBSE Oswaal CBSE Question Bank Chapterwise & Topicwise, ENGLISH LANG. & LIT., Class-X

Raavi/ Mohini Mohini—Mohini is a middle-aged lady who is a singer. She studied music and with her hard work, she became a playback singer. Not only does she loves music, but she is fond of reading books also. She is an avid reader. Apart from that, she is very social and likes to be with people. She participates actively in social functions. She remains busy most of the time and so does not get time to exercise. However, she is firm and nothing takes her away from her goal. We can say that she is a lady with a purpose who has clear vision about her purpose of life.









306

Raavi—Raavi is a handsome young man. He works as an engineer in a manufacturing firm. He likes to make, repair, and fix things. He is very congenial and amiable kind of person who is a big support to people in their times of need. His hobbies include watching television and movies. He goes to watch movies thrice a month. He does not like to play but is fond of morning walks. He goes for a morning walk every day.



@PROCBSE

@PROCBSE









(b) a convocation address at a university



(c) a speech in Indian Parliament



Q. 2. Patel feels that future generations will heap curses on us if ___________________________.

(a) we fail to protect our freedom







(b) we fail to protect the corrupt people in public life



(c) we wage war against other nations



Q. 3. The first and foremost thing Patel stressed to be done is ____________________________________.

(a) to make the country strong and united







(b) to give employment to everyone



(c) to fight a war against those who oppressed us



Q.4. One major characteristic of university education, as Sardar Patel believes, is _________________.

(a) character







(d) to make every citizen feel happy





(b) duty



(c) knowledge



(d) power

Q. 5. What did Patel not mean when he said, “You have to realise that India has to attain its rightful place in the comity of nations”? Tick the right answer.

(a) Friendly and cordial relationship with other nations



(b) Group of nations which wage war against other



(c) Community of people

(d) Nations from a continent

Q. 6. ‘India abounds in the resources of nature and manpower’ means ___________________________ ______. (a) India has a vibrant economy (b) India’s natural resources and population are a great resource





(d) we understand the need for peace

(c) India’s lack of manpower (d) Indians are great resource for development





(d) a speech delivered to civil servants





(a) an address to a public gathering





Complete the sentences given below by choosing the most appropriate word or phrase.

Q. 1. The occasion of the speech of Sardar Patel here is ____________________________________.



our legs so that we can stand erect and bear the burden that has devolved on us. Then alone would we secure the real objective of freedom. (Source: The Collected Works of Sardar Vallabhbhai Patel Volume XIII [1 January 1948 – 31 December 1948], Editor: P.N. Chopra, Konark Publishers Pvt. Ltd., New Delhi)



This is the occasion for mutual help and mutual cooperation. We must know what our duty is as true citizens. Your teacher has to impart that duty. Unless we learn to shoulder our responsibility in the true spirit of discipline and citizenship we cannot go ahead. We must strengthen our shoulders and



The most essential requisite is character. If your character is lacking and you leave the precincts of the alma mater without your character fully developed, you will be wasting an opportunity of a lifetime. You have to realise that India has to attain its rightful place in the comity of nations. The leadership of Asia would be hers if she conducts herself well and if her citizens make their fitting contribution to nation’s resurgence. Whatever trials and troubles we might have passed through, there is no denying the fact that we have a precious inheritance and the teachings we have received from the departed leaders are great assets for us. India abounds in the resources of nature and manpower. They are awaiting full use. That task must devolve on young men and women. We ourselves were trained in the school of experience. We fought for freedom and we have attained that prize for you. It is now up to you to prove yourself worthy of the same. This you can do only if you work with single-minded devotion to the cause of the country. It is only then that you can deserve this freedom and be proud of it. The real danger of India lies in our disunity. That poison of communalism was spread in the past. You should draw a curtain over it. Only then could we equip ourselves for the freedom that has come to us. There are still people in the country whose loyalty to India has not settled down. They should and will leave India.



The great university of life is full of experiences, but to make full use of those experiences, all of us students, teachers and professors — have to be cautious and on guard.















appreciate our duty. Now that foreign rulers have gone, are we in a position to sustain the whole burden that has devolved on us? When we took over from them, the administrative fabric was in tatters. We have to mend and place it. This, in itself, is a heavy responsibility. It is for our universities to train young men to shoulder these responsibilities.



Oswaal CBSE Question Bank Chapterwise & Topicwise, ENGLISH LANG. & LIT., Class-X



308

@PROCBSE







(c) India’s lack of manpower.



(d) Indians are great resource for development.

The first and foremost duty of the young nation is _________________.



Ans. 1. (a) an address to a public gathering



2. (a) we fail to protect our freedom

3. (a) to make the country strong and united











Q. 8. Complete the following statement based on your understanding of Patel’s speech.









5. (b) Group of nations which wage war against other



6. (b) India’s natural resources and population are a great resource 7. (c) India’s lack of manpower







(b) India’s natural resources and population are a great resource.

4. (a) character









(a) India has a vibrant economy.

309

8. .... to realise that freedom has come to us through sacrifices, and this freedom is yet to be consolidated. The foundation is to be well and truly laid. People should not indulge in useless controversies and unnecessary debates and disputes. Instead, they should indulge in the task of making the foundation of freedom strong. We must focus on how to keep our country strong and united. We must train our young men to shoulder this responsibility. Everyone must work with single-minded devotion for the country.

Q. 7. What is the greatest danger for India as Patel believes in his speech?



NELSON MANDELA : LONG WALK TO FREEDOM (WORKBOOK)







hospital precincts. (b) Comity of nations : Many people want to get linked to the comity of nations. (c) Unassailable : His position in this firm was unassailable. (d) Mutual cooperation : No country can prosper





1. Use the following words and phrases in your own sentences.







(b) Comity of nations ...........................





(a) Precincts ...........................









(c) Unassailable ...........................





VOCABULARY















(d) Mutual cooperation ........................... without mutual cooperation. Ans. (a) Precincts : No one is permitted to litter in the 2. Sardar Patel called upon young students to understand the meaning of freedom and ways to sustain it. Here are two key words which he stressed upon—‘Freedom’ and ‘Responsibility’. Working in pairs, find words and phrases which convey the feeling of these words and write below. Ans. Freedom Responsibility liberty

charge

facility

care

privilege

duty

exemption

undertaking







obligation You may have written above at least seven to eight words for each key word. Using the words write at least five sentences describing ‘Freedom’ and ‘Responsibility’. One has been done for you. Example : Freedom is not free. Liberty

You have the liberty to ask me anything.

Facility

My sister handled the crisis with facility.

Privilege

I have the proud privilege to express my views in front of you all.

Responsibility

I take the responsibility of teaching poor children.

Charge

He was given the charge of looking after the elderly in the old age.

GRAMMAR 1. You have come across the use of articles ‘a’, ‘an’, and ‘the’ in the lesson. The tasks in the textbook give you the idea that the use of ‘the’ with proper noun carries a special meaning.







Use of Articles

Read the following paragraph from the speech you have read above. The articles are missing in the paragraph. With the help of your partner, fill in the blanks.

@PROCBSE Oswaal CBSE Question Bank Chapterwise & Topicwise, ENGLISH LANG. & LIT., Class-X



Pronouns  When I was a kid, my English teacher looked my way and said, “Name two pronouns.” I said, “Who, me?” 3. Fill in the blanks with suitable articles.

(a) ______ boy and ______ girl were shopping in a market. ______ boy was from Kerala and ______ girl was from Nagaland.











Fun Facts





(b) Can you turn off ______ light?



(c) He has taken ______ taxi to ______ station.



(d) Is there ______ police station nearby?



(e) My sister works in ______ bank.







(f) He has been looking for ______ job for the last seven years.





(g) Would you like to eat ______ apple?





(h) He goes to the theatre once ______ month.











(f) a (h) a

(d) a

(i) a, a, The, the

(e) a

(j) a













(c) a, the



(g) an



(b) the







Fun Facts



Ans. (a) A, a , The, the





(j) We would like to have dinner at ______ good restaurant.





2. Which article most frequently occurs in the passage? Why is it so? Can you replace it with some other article? Why / Why not? Discuss with your peers and make notes.





(h) The

Ans. The article ‘the‘ has occurred most frequently in the passage. We can not replace it with any other article because ‘the’ is used with specific nouns and other articles (a, an) are used for non-specific or nonparticular nouns. Comma  Comma can be used between words in a series. I like to drink coffee, tea, orange juice and milk.







(i) This morning I bought ______ newspaper and ______ magazine. ______ newspaper is on my table and I forgot where I have placed ______ magazine.

(g) the



a the the The



















(c) (d) (e) (f)





(b) the









____(a)____ weapons of ____(b)____ world war were: violence, brute force, political and military moves, and counter moves. As ____(c)____ result of ___(d)_____ churning of the ocean, ____(e)____ world has emitted poison. That poison is spreading all over and there is none to swallow it. ____(f)____ countries which were free have managed to digest it somewhat, but we, who have just attained our freedom, it is hard to do _______(g)______same. Therefore, those who think that having attained freedom they have attained everything, have really attained nothing! ______ (h)______ freedom that we have won is yet to be consolidated. Ans. (a) The



310















































I don’t mean that some peoples are born clear headed and is therefore natural writers, whereas others are naturally fuzzy and will never wrote well. Thinking clearly was a conscious act that writers must forced on themselves, as if they were working on any other project that requires logic: makes shopping list or doing an algebra problems. Good writing do not come naturally, though most people seems to think it does. Professional writer are constantly bearded by people who say they’d like to “trying a little writing sometime” — meaning when they retire from their real profession.





























1. The following passage has an error in each line. Find the error and write the correct word or words against the line.



























EDITING

____(a)_____ ____(b)_____ ____(c)_____ ____(d)_____ ____(e)_____ ____(f)_____ ____(g)_____ ____(h)_____ ____(i)_____ ____(j)_____ ____(k)_____ ____(l)_____ ____(m)_____

@PROCBSE NELSON MANDELA : LONG WALK TO FREEDOM (WORKBOOK)



311

problem







(m) profession



making



(l) meaned

force





(k) trying

is



(j) writer



write



(i) seems



are







(h) do









(f) makes









(e) forced









(d) was

people











(c) wrote

(g) problems











(b) is

Correct









(a) peoples





Incorrect



Ans.





does seem writers try meant professions

1. Your teacher or any one student will read out the text given below to the whole class. Listen carefully. Working



LISTENING in groups of four, note down in short the important points as you listen. Discuss in your group. Recreate and write the text you have listened to. You need not write the exact text, but the meaning should be nearest to the actual text read out to you. Your teacher will read out the text again for all the groups so that you can check whether you have understood the text well.

Ans. Students to practice under the guidance of their teacher.



Text for Listening A few minutes ago, walking back from lunch, I started to cross the street when I heard the sound of a coin dropping. It wasn’t much, but as I turned my eyes, it caught the heads of several other people turning too. A woman had dropped what appeared to be a one rupee coin. The tinkling sound of a coin dropping on the pavement grabs attention. Whatever be the value of the coin no one can ignore the sound of it. It got me thinking about sounds again.

2. Prepare a speech for the morning assembly



1. You have read the speeches of Nelson Mandela

reflecting on the following questions.

on freedom and responsibilities for a new born

(a) How do you feel today about the country’s

and Sardar Patel. You know their beliefs and ideas



SPEAKING

nation. Discuss with your partner and put down

Ans. Students to practice under the guidance of their



(b) Are we free and independent? (c) Are we serious about our responsibilities as citizens?







an outline.





your ideas on what freedom means to you. Prepare

freedom?

teacher.

(b) The interviewee responds to them. The





1. Make pairs. One of you takes the role of Nelson Mandela and the other becomes the interviewer.

interviewer’s task is to note down the answers

(a) The interviewer prepares a questionnaire on

also. (c) The interviewer, through the questions, tries to

contributions to the removal of Apartheid.



Nelson Mandela—Mandela as a person and his









WRITING

get an overview of Mandela’s life.

For interviewer S. No.

Questions to be framed

1.

How do you feel now that you have become the President?

2.

You had undergone many years of struggle. What did you feel like then?

3.

What do you have to say about the struggle in South Africa ?

4.

Who inspired you for the removal of Apartheid?

5.

What kind of childhood did you have?

@PROCBSE 312

Oswaal CBSE Question Bank Chapterwise & Topicwise, ENGLISH LANG. & LIT., Class-X

For interviewee Answers by the Interviewee

1.

It feels good, as our country is free now and not under the power of any other country.

2.

It was a long and difficult struggle where many people suffered.

3.

The struggle in South Africa was a long one. Many people lost their lives ,many were imprisoned,many lost their land,loved ones. So it was not an easy one. It was a painful experience.

4.

There were many people who acted as a guiding light to me. It was a collective leadership.

5.

I was born free but in a way that I could only know. I enjoyed running in the fields, swimming in the clear streams, and rode the broad backs of slow moving bulls. So I can say that I had a wonderful childhood.

2. Make groups of four and find out the background information on Nelson Mandela, Mahatma Gandhi, and



S. No.

Martin Luther King. All three of them fought against all odds and used non-violence to achieve their aim. Choose any two of the above stated leaders and do a comparative study of their principles and lives. S. No.

Leader 1: __________________

Leader 2: ___________________

Leader 1 : MAHATMA GANDHI

Leader 2 : NELSON MANDELA

1. 2. 3. 4. 5. 6. 7. 8. 9.



10. Ans. S. No. 1.

Mahatma Gandhi is known as the Nelson Mandela was an outstanding South Father of the nation and is one of the African political leader who was affectionately greatest leaders of the world. called “Tata”which means father.

2.

He was sent to jail and suffered a lot He was arrested and sentenced to prison with but pursued his mission. Nothing hard labour. stopped him from pursuing it.

3.

Gandhiji had an iron will and was Mandela ,like Mahatma Gandhi, had an amazing fearless. will power and continued his struggle.

4.

Mahatma Gandhi independence from powers.

5.

Gandhiji believed in non violence and He believed that violence is a necessity if the decided not to violate the laws as he government tyrannizes the nation. himself practised law.

6.

He fought against beliefs and not He favoured equality between the whites and people. Gandhiji said “Hate the sin and the blacks. not the sinner.”

7.

He believed in the benefit of masses…. He believed in using sports as a means of uniting bahujan hitay-bahujan sukhay. people.

fought for Nelson Mandela started the anti apartheid the foreign movement which gathered strength with the passage of time.

@PROCBSE

NELSON MANDELA : LONG WALK TO FREEDOM (WORKBOOK)

8.

Gandhiji preached to follow the path Like Mahatma Gandhi, Nelson Mandela also of truthfulness. He felt it to be the ma- believed in following the path of truthfulness. jor component in leading a disciplined life.

9.

He never participated actively in any After his release from prison, Mandela became an elections. active politician and later served his country as the President from 1994-1999.

10.

Mahatma Gandhi was assassinated in Nelson Mndela died at the age of 95 due to respiratory infection. 1948.

313



@PROCBSE

UNIT

3

TWO STORIES ABOUT FLYING



Exercises





(b) The young seagull was hungry, so he started to fly. ( )















(b) True



(c) True



(d) False



2. You have another story, ‘The Black Aeroplane’ in the lesson. Read the story again and develop at least five multiple choice questions based on the story. Work in pairs and ask your partner to answer them. The questions may be based on the events, persons, reasons and causes of the events, and results in the story. Your questions

(c) He was hungry and he moved to fly in search of food.

1. The pilot of the Dakota aeroplane was happy because ......... . (a) he was with his friends



(d) He did not learn to fly at all.





(b) His brothers and sisters trained him to fly.







(a) His mother showed him the fish and moved away from him to make him fly.





5. (a) False





(d) He was not liked by them at all. Q.3. How did the young seagull start flying?





4. (a) He was lazy and did not want to fly.





(c) He was not accepting their request to fly with them.





(b) he was going to meet his family



Q.4. Which of the following statements is not true of the young seagull?





(c) he enjoyed flying in that weather (d) the weather was perfect

2. Which of the following statements is not true? (a) The pilot lost contact with the Paris control room. (b) The pilot of the black aeroplane helped him. (c) The narrator was not worried about the fuel. (d) The narrator landed on the runway safely.















(a) The young seagull liked to fly with his brothers and sisters. ( )





(d) His hunger made him fly. Q.5. Say whether the following statements about the seagull are true or false.







(c) His mother, father, brothers, and sisters helped him fly.









(b) He wanted to fly, but was afraid of flying.









(a) He was lazy and did not want to fly.









3. (a) His mother showed him the fish and moved away from him to make him fly.







(b) He was not listening to them.

( )

2. (c) He was not accepting their request to fly with them.



(a) He went with his sisters and brothers.



(d) He flew on his own to get the food.



Q.2. Why did his family taunt him about his cowardice?

( )

Ans. 1. (a) he felt his wings would not support him





(d) his mother and father threatened him













(c) the sea was very large





(b) other bird knew to fly very well









(a) he felt his wings would not support him





(c) He was scared of flying first.



Q.1. The young seagull was afraid of flying because __ _______________________________________ .





1. Read the story, ‘His First Flight’ again. Working in groups of four, answer the following questions selecting the correct option.



You have read two stories about flying. ‘His First Flight’ is about how a young bird learns to fly and ‘Black Aeroplane’ is a mystery.







LET’S BEGIN

@PROCBSE TWO STORIES ABOUT FLYING (WORKBOOK)





























(d) bright sky



(c) clear sky



(b) stormy weather



(a) dark night





4. The Dakota was twisting in the sky because













(d) The lady at the control room has seen two planes on the radar. ( )



Answers : 1. (b) he was going to meet his family 2. (c) The narrator was not worried about the fuel. 3. (a) False (b) True (c) True (d) False 4. (b) stormy weather 5. (c) the mysterious pilot and the Black aeroplane were nowhere to be seen.





(c) At one stage, all the instruments of the Dakota aeroplane stopped working. ( )











(b) The pilot of the Black aeroplane was a mysterious man. ( )















(a) The narrator thanked the pilot of the Black aeroplane for his help. ( )





5. When the narrator landed safely, he was surprised because .......... . (a) the compass has started working (b) the radar showed two aeroplanes (c) the mysterious pilot and the Black aeroplane were nowhere to be seen. (d) the lady at the control centre was helpful.



3. Say whether the following statements are True or False :

315

READING COMPREHENSION was his moment of triumph. He had the law on his side and the whole bus full of angry people under the harrow. His embittered soul was having a real holiday.



It was a bitterly cold night, and even at the far end of the bus the east wind that raved along the street cut like a knife. The bus stopped and two women and a man got in together and filled the vacant places. The young woman was dressed in sealskin, and carried one of those little Pekinese dogs that women in sealskin like to carry in their laps. The conductor came in and took the fares. Then his eyes rested with old malice on the beady eyed toy dog. I saw trouble brewing. This was the opportunity for which he had been waiting and he intended to make the most of it. I had marked him as the type of what Mr. Wells has called the Resentful Employee, the man with a great vague grievance against everything and a particular grievance against passengers who came and sat in his seat while he shivered at the door.



All about a Dog

“I shall certainly do nothing of the kind. You can take my name and address,” said the woman, who had evidently expected the challenge and knew the reply. “You must take the dog out— that’s my orders.” “I won’t go on the top in such weather. It would kill me,” said the woman. “Certainly not,” said her lady companion, “You’ve got a cough as it is.” “It’s nonsense,” said her male companion. The conductor pulled the bell and the bus stopped. “This bus doesn’t go until that dog is brought out.” And he stepped on to the pavement and waited. It



“You must take that dog out,” he said with sour venom.























TEXT I Let’s now read the story of a bus conductor and his passengers with a dog. The story is written by A.G Gardiner.

The storm inside rose high “Shameful!” “He’s no better than a German.” “Why isn’t he in the army?” “Call the police. Let’s all report him.” “Let’s make him give us our fares back.” Everybody was on the side of the lady and the dog. That little animal sat blinking at the dim lights in happy unconsciousness of the rumpus of which he was the cause. The conductor came to the door, “What’s your number?” said one, taking out a pocket-book with a gesture of terrible things. “There’s my number,” said the conductor imperturbably. “Give us our fares back.” “You can’t leave us here all night.” “No fares back,” said the conductor. Two or three passengers got out and disappeared into the night. The conductor took another turn on the pavement, then went and had a talk with the driver. Another bus, the last on the road, sailed by indifferent to the shouts of the passengers to stop, “They stick by each other the villains,” was the comment. Someone pulled the bell violently. That brought the driver round to the door, “Who’s conductor of this bus?” he said and paused for reply. “None coming,” he returned to his seat, and resumed beating his arms across his chest. There was no hope in that quarter. A policeman strolled up and looked in at the door. An avalanche of indignant protests and appeals burst on him. “Well, he’s got his rules, you know,” he said. “Give your name and address.” “That’s what he’s been offered, and he won’t take it.” “Oh,” said the policeman, and he went away and took his stand a few yards down the street, where he was joined by two more constables.

@PROCBSE

presently the lady with the dog stole down the stairs and re-entered the bus. When the engine was put right, the conductor came back and pulled the bell. Then his eyes fell on the dog, and his hand went to the bell-rope again. The driver looked around, the conductor pointed to the dog, the bus stopped, and the struggle recommenced with all the original features — the conductor walking the pavement, the driver smacking his arms on the box, the little dog blinking at the lights, the sealskin lady declaring that she would not go on the top and finally going.

And still the little dog blinked at the lights, and the conductor walked to and fro on the pavement, like a captain on the quarterdeck in the hour of victory. A young woman, whose voice had risen high above the gale inside, descended on him with an air of threatening and slaughter. He was immovable as cold as the night, and as hard as the pavement. She passed on in a fury of impotence to the three policemen, who stood like a group of statuary on the street watching the drama. Then she came back, imperiously beckoned to her ‘young man’ who had sat a silent witness of her rage, and vanished. Others followed. The bus was emptying. Even the dashing young fellow who had demanded the number and who had declared he would see this thing through if he sat there all night, had taken an opportunity to slip away.





Oswaal CBSE Question Bank Chapterwise & Topicwise, ENGLISH LANG. & LIT., Class-X

Meanwhile the Pekinese party was passing through every stage as of resistance to abject surrender. “I’ll go on the top.” said the sealskin lady at last. “You mustn’t.” “I will.” “You’ll have pneumonia.” “Let me take it.” (This from the man) “Certainly not.” She would die with her dog. When she had disappeared up the stairs, the conductor came back, pulled the bell, and the bus went on. He stood sourly triumphant while his conduct was savagely discussed in his face by the remnant of the party. Then the engine struck work, and the conductor went to help the driver. It was a long job and

“I’ve got my rules,” said the conductor to me when I was the last passenger left behind. He had won his victory, but felt that he would like to justify himself to somebody. “Rules,” I said,“ are necessary things, but there are rules and rules. Some are hard and fast rules, like the rules of the road, which cannot be broken without danger to life and limb. But some are only rules for guidance, which you can apply or wink at, as common sense dictates, like that rule about the dogs. They are not a whip, put in your hand to scourge your passengers with, but an authority for an emergency. They are meant to be observed in the spirit, not in the letter, for the comfort and not the discomfort of the passengers. You have kept the rule and broken its spirit. You may mix your rules with a little goodwill and good temper.” He took it very well and when I got off the bus he said “Good night” quite amiably.

Q. 1. Now, complete the box narrating the story in the correct sequence.











316

Once two women and a man got into a bus and found places to sit.

Two or three passengers got the money back and walked away.

The conductor did not give up. He sent the lady with the dog away to the top. Ans.

(a)

Once, two women and a man got into a bus and found places to sit.

(e)

Two or three passengers got the money back and walked away.

(b)

One of the women was carrying a little pekinese dog with her.

(f)

One policeman came but didn’t help them. He joined the other two standing on the pavement.

(c)

When the conductor saw the dog, he told the woman to take the dog out. The woman refused.

(g)

The bus was emptying, as everyone started getting down.

@PROCBSE TWO STORIES ABOUT FLYING (WORKBOOK)

(d)

The conductor stopped the bus by pulling the bell and stepped on to the pavement. All the passengers were on the side of the lady and the dog. They got angry and demanded the fare back.

(h)

317

The conductor did not give up. He sent the lady with the dog away to the top.





(c) Indignant — Indignantly — The young boy answered indignantly to teacher. (d) Threatening — Threateningly — Mr. Sudesh was turned out of the room threateningly by his boss.



(b) Vague — Vaguely — The dog looked vaguely at the man standing on the other side of the road.



(a) Angry — Angrily — The old man looked at the boys angrily because they had broken the glass of his window.

(e) Hurry — Hurriedly — He walked hurriedly, as he was late for school.





1. Making adverbs: bitterly, imperturbably, violently, and evidently are some words you have come across in the story. These are adverbs. A few words are given below. Make adverbs of these words and use them in sentences of your own. (Take care of the spelling when you convert these into adverbs.)







2. Now, look at the word given below in italic. He was immovable. ‘im’ is the prefix added to the word ‘movable’. The prefix ‘im’ made ‘immovable’ an antonym of ‘movable’. Now, make more words with the prefix im- which are antonyms. 1. mobile — immobile

2. pure — impure

3. patient — impatient

4. possible — impossible

5. perceptible — imperceptible

6. moral — immoral

7. mortal — immortal

8. probability — improbability

3. In groups of five, create a word-web or collocation chart of words with prefixes ‘il’ and ‘in’ .





















VOCABULARY

Example: logical — illogical

Example: secure — insecure

1. legal — illegal

1. communicative — incommunicative

2. legitimate — illegitimate

2. formal — informal

3. literate — illiterate

3. finite — infinite

4. legible — illegible

4. expensive — inexpensive

5. licit — illicit

5. effective — ineffective

(b) walk out — The members of the committee walked out in protest.





(e) take in — He took in a deep breath before entering the room.

(f) take out — The students took out their notebooks and started writing.

(g) give away — The chief guest gave away the prices to the winners.

(h) give in — The manager had to give in to the demands of the workers. (i) give up — I gave up finding the answer to the question after half an hour.





(a) walk in — He decided to walk in to the near by restaurant to have his lunch.

(d) take up — The new teacher has taken up her charge this morning.

(j) put in — You must put in more efforts to get a good score in examination.

Find out the meaning of the following phrasal verbs and use them appropriately in sentences.

(c) take away — The thieves took away everything from the house.

(k) put away — I have put away all my woollens as the days become hot.

walk away is a phrasal verb which means to move away from.



…Walk away from the old Dakota.





The sentence below is taken from the text you have read. The phrase in italic is a phrasal verb. A phrasal verb is a combination of words (a verb + a preposition, for example check in or a verb + an adverb, for example break down). When they are used together, they usually take on a different meaning to that of the original verb.

















GRAMMAR A. Phrasal verbs

@PROCBSE



(e) The ultraviolet rays / prevent / the ozone layer / entering the Earth’s surface. (Simple Present)

(f) The field / convert / playground. (Present Perfect)



Ans. (a) The TajMahal was build by Shah Jahan.

(b) Malaria virus is transmitted by the female Anopheles mosquito.

(c) The police has solved the case and arrested two persons.

(d) Many houses and lives were destroyed by the volcanic eruption in Mt. Etna.

(e) The ultraviolet rays are prevented from entering the Earth’s surface by the ozone layer.

(f) They have converted the filed into playground.

C. Unscramble the following sentences to make meaningful ones:

(a) records / flying / many / who / American aviator / was an / Amelia Earhart / set

(b) the first / across / woman / solo / to fly / the Atlantic Ocean / she / became

(c) a flight / July 1937 / the globe / Earhart / disappeared / to circumnavigate / somewhere / over the / Pacific / in / during

(d) was never / found / and / she was / officially declared / plane wreckage / lost / at sea / her

(e) century / her / disappearance / remains / of the / one of the / greatest / unsolved mysteries / twentieth.

(b) She became the first solo woman to fly across the Atlantic Ocean.

(c) Earhart disappeared somewhere over the Pacific during a flight to circumnavigate the globe in July 1937.

(d) Her plane wreckage was never found and she was officially declared lost at sea. (e) Her disappearance remains are of the greatest unsolved mysteries of the twentieth century.





















Ans. (a) Amelia Earhart was an American aviator who set many flying records.



























































(l) put out — The firemen tried their best to put out the fire. B. The Passive Compare the two sentences given below: Ramesh painted the walls. The walls were painted by Ramesh. The first sentence is in active form and the second sentence is in passive form. When the person doing the action (Ramesh) is the subject, we use the verb in the active voice. When the subject is what the action is directed at (the walls), we use the verb in the passive voice. In the first sentence, we talked about Ramesh, but in the second sentence, we talked about what happened to the walls. In the passive sentence, we use by+agent to add important information, that is by Ramesh. We can leave out by+agent if it does not provide any specific information. Example: All the students were given sweets yesterday. My room is cleaned everyday in the morning. A large number of films on wildlife conservation have been made. Use the words given below and frame sentences using the passive forms of the verbs. Examples: l Rice / cultivate / Gangetic valley and coastal region. (Simple Present) Rice is cultivated in Gangetic valley and coastal region. l The documents / sign / principal. (Present Perfect) The documents have been signed by the principal. (a) Taj Mahal / build / Shah Jahan. (Simple Past) (b) Malaria virus / transmit / female Anopheles mosquito. (Simple Present) (c) The case / solve / police and two persons arrest. (Present Perfect) (d) Many houses / and lives / destroy / the volcanic eruption in Mt. Etna. (Simple Past)



Oswaal CBSE Question Bank Chapterwise & Topicwise, ENGLISH LANG. & LIT., Class-X













































318

Ans. In some parts of our country, there are frequent reports about Unidentified Flying Objects (UFOs), which people claimed to have seen flying in the sky and which are believed to be space ships carrying aliens from other planets. Some people have given evidences to support their observations. They said that they have found mysterious objects in paddy fields. Some others, after seeing a film on space, said that they had seen unusual objects flying in the sky. Public panicked. There were arguments and discussions all over the country. Police were on red alert. Many community centres were established. People became more vigilant. Gradually, the number of sightings reduced.





1. Use capital letters, commas, and full stops to edit this paragraph. In some parts of our country, there are frequent reports about Unidentified Flying Objects (UFOs) which people claimed to have seen flying in the sky and which are believed to be space ships carrying aliens from other planets some people have given evidences to support their observations they said that they have found mysterious objects in paddy fields. Some others, after seeing a film on space said that they had seen unusual objects flying in the sky. Public panicked there were arguments and discussions all over the country. Police were on red alert Many community centres were established people become more vigilant gradually the number of sightings reduced.





EDITING

@PROCBSE TWO STORIES ABOUT FLYING (WORKBOOK)

319

1. Do you like riddles? Riddles make us think and attempt to solve puzzles through various ways and means. Read this riddle out loud and find answers to it. You can also discuss with your friends. You will notice that we speak out what we think. This is known as Think-aloud protocol. As you solve the riddles, speak out all the ideas that come to your mind to solve it.

side in this river crossing puzzle?







(c) The boat only has room for two (monkeys or humans). (d) Monkeys can jump out of the boat when it’s on the river bank.



(b) At all times, the number of humans on either side of the river must be greater or equal to the number of monkeys on that side (or else the humans will be killed by the monkeys!).



How can the farmer get everything on the other

How will they cross the river? Help them out.

Ans. Students to do it under the guidance of their teacher.

A farmer returns from the market, where he bought a goat, a cabbage and a wolf (what a crazy market). On the way home, he must cross a river. His boat is small and won’t fit more than one of his purchases. He cannot leave the goat alone with the cabbage (because the goat would eat it), nor he can leave the goat alone with the wolf (because the goat would be eaten).







Riddle 1

Three humans, one big monkey, and two small monkeys are to cross a river:

(a) Only humans and the big monkey can row the boat.







Riddle 2





LISTENING

WRITING on how the rules can be effectively implemented in the society and citizens can follow it in the true spirit. You may use any of the ideas given in the box. Follow the stages involved in the process approach to writing.

1. ‘Rules are meant to be observed in spirit not only in letter’. Explain what you understand by this statement. Write an article in about 120 – 150 words



Article Writing









(a) Use it in the context of gender (protection of women’s rights / dowry / female infanticide, etc.). (b) In the context of environment (cutting of trees / poaching / pollution). (c) Smoking in public / following traffic rules, etc. (add cartoons with fun facts, for example parking in front of no parking sign).

RULES ARE MEANT TO BE OBSERVED IN SPIRIT AND NOT ONLY IN LETTER

2. You have read and experienced how the little seagull learnt to fly and then the story of the mysterious black aeroplane. You have also read the story, ‘All about a dog’. How were these experiences? You may have also experienced some

Ans. It was a normal Monday morning. I had got ready to go to school at 6 : 30 a.m. sharp, I sat in my school auto-rickshaw and off I went to school with my younger brother. I was studying in class two at that time and my younger brother was in class one. He was feeling sleepy, as we had gone to attend a party on Sunday and had returned late. By 6 : 45, the three wheeler had collected all the students and now we were on our way to school. I was looking outside. Suddenly, I felt some movement next to me. I looked around and noticed my younger brother was not there. I shocked and the driver stopped the vehicle. We realised that he had fallen out of the vehicle, as he must have slept while sitting. The driver took a U-turn and then we saw him sitting at a distance. He had received minor injuries and there was nothing serious. We thanked God, as it

There is no denying the fact that rules are meant to be observed in spirit and not only in letter. Our government tries its best to make several changes so as to make our country move smoothly on its path of development, but there are many who do not follow the rules. For them, rules are good only in letters. Taking the example of no smoking in public, there are a few people who still smoke at railway platforms, bus stands, markets etc. strict action should be taken against them so as to teach a lesson to them as well as to others who are watching them because they are leaving a bad impression for others. Rules are made to make us feel responsible towards our society and our nation. Things may turn bad if we do not follow the traffic rules. Hence, it is important to observe rules and not take them only as letters.

such situations in your life: learning to ride on a bicycle, watching a cat or any other animal trying to jump over the wall, your own bitter experience in a difficult situation, etc. Think of such an experience and write down the points and develop the ideas into an outline. Write your experiences like a short story.



Ans.











(d) In the context of value and self discipline.

@PROCBSE 320

Oswaal CBSE Question Bank Chapterwise & Topicwise, ENGLISH LANG. & LIT., Class-X

was a busy road and anything could have happened. He was crying bitterly and was dead scared. I collected him in my arms and tried to soothe him. He didn’t take long to return to normal, but I never left his side till we reached the school. Both of us still remember that bitter day and its memory still gives us goosebumps.



@PROCBSE

UNIT

4

FROM THE DIARY OF ANNE FRANK



Exercises













cover, however, offered no protection from artillery fire: in just over six months from the start of the siege there were only thirty-two days when shells did not explode on the city’s streets.

















Foreword from The Diary of Lena Mukhina No one could possibly have known that the city would remain in the grip of siege for 872 days and nights. But Leningrad’s citizens had begun to feel the breath of war as early as September. Daily air raids and artillery bombardment of residential neighbourhoods drove the city’s inhabitants into bomb shelters, several times a day, and for hours at a time. People’s initial curiosity in the aftermath of the first bomb attacks soon gave way to fear and then, in conditions of appalling hunger and cold, to apathy. Leningraders welcomed inclement weather for the reduced visibility it brought. They came to dread sunny days and moonlit nights, when the Germans would be sure to bomb their city. Cloud

Hunger soon set in. Between September and November 1941 the bread rations that had been announced in July were reduced five times, reaching their lowest point on 20 November — 125 ‘blockade grams’ for children, dependents and white-collar workers and 250g for manual workers, technicians and engineers. Rations of meat, butter, sugar and other food items were severely reduced. The summer months and the beginning of autumn, when it was still possible to buy food without ration cards in ordinary shops and canteens were no more than a distant memory. Lena Mukhina Q. 1. Infer the meanings of the following expressions. The first has been done for you as an example. (a) to feel the breath of war : (b) appalling hunger and cold : (c) start of the siege : (d) blockade grams : Ans. (a) to feel the breath of war : there were indications that war was going to start soon. (b) appalling hunger and cold : The situation was terrible due to inclement weather and starvation.

Read the excerpt and answer the questions that follow.



The following excerpt is from the diary of Lena Mukhina, a teenage girl who had undergone the atrocities of Germans when they fortified the city of Leningrad. The siege of Leningrad is an important landmark in the history of Russia. The siege of Leningrad lasted from September 1941 to 1944. The city was cut off from the rest of Russia. The Germans continuously bombarded the city which resulted in death, destruction, and starvation of the people. However, the resolute Russians were warned in the beginning that the enemy was at the gate and it was a question of life and death. They fought bravely and failed the Germans in their evil designs.











TEXT I

@PROCBSE Oswaal CBSE Question Bank Chapterwise & Topicwise, ENGLISH LANG. & LIT., Class-X











(b) The people under siege welcomed the bad weather conditions because it reduced the visibility, and with it, the bombing was reduced the city. (c) The mind of the people were full of the fear of sunny days and moonlit nights when they were sure that the Germany would bomb the city. (d) The people have lost hope. Q. 4. Tick the correct answer ‘…when it was still possible to buy food without ration cards in ordinary shops and canteens were no more than a distant memory.’ It means: (a) There was hope of getting ration soon. (b) Citizens under siege had to show their ration cards for buying food. (c) There were shops and canteen for buying food for the people under siege. (d) The possibility of buying the food without a ration card as in the past, was no more than a memory now. Ans. (d)

































(c) start of the siege : There was a military blockade of the city to compel it to surrender. (d) blockade grams : It was the act of using force to restrict or stop the movement of goods into or out of the area. Q. 2. List the words from the passage which are related to the war, for example ‘bombardment’. Ans. (i) air raids (ii) bomb shelters (ii) artillery fire Q.3. Answer the following questions: (a) Who were the captives as stated in the above passage? (b) Why did the people under siege welcome the bad weather conditions? (c) What were the fears in the mind of the people? (d) Have the people lost hope or reconciled to the existing situation? Ans. (a) The captives were the people who lived in leningrad.



322

READING COMPREHENSION

TEXT II

We had no such thing as printed newspapers in those days to spread rumours and reports of things, and to improve them by the invention of men, as I have lived to see practiced since. But such things as these were gathered from the letters of merchants and others who corresponded abroad, and from them were handed about by word of mouth only; so things did not spread instantly over the whole nation, as they do now. But it seems that the Government had a true account of it, and several councils were held about ways to prevent its coming over, but all was kept very private. Hence, it was that this rumour died off again, and people began to forget it as a thing we were very little concerned in, and that we hoped was not true; till the latter end of November or the beginning of December 1664 when two men, said to be Frenchmen, died of the plague in Long Acre, or rather at the upper end of Drury Lane. The family they were in endeavoured to conceal it as much as possible, but as it had gotten some vent in the discourse of the neighbourhood, the Secretaries of State got knowledge of it; and concerning themselves to inquire about it, in order to be certain of the truth, two physicians and a surgeon were ordered to go to the house and make inspection. This they did and finding evident tokens of the sickness upon both the bodies that were dead, they gave their opinions publicly that they died of the plague. Whereupon, it was given in to the parish clerk, and he also returned them to the Hall, and it was printed in the weekly bill of mortality in the usual manner, thus — 1. Parishes infected, 2. Plague.

(c) it was come → it had come (d) concerned in → concerned about





Q. 2. Answer the following questions: (a) What is being talked about in the first paragraph of Daniel Defoe’s journal? (b) How was news about important events circulated during those days?







(b) plague was returned → plague had returned





Example : whence — when

Ans. (a) whither → where







Q. 1. The above passage is written in the English that was prevalent in the eighteenth century. Find some words and phrases from the text which seem different as compared to the English language usage at present. List them and find an appropriate substitute for each. Look at the example given below:





















A Journal of the Plague Year It was about the beginning of September, 1664, that I, among the rest of my neighbours, heard in ordinary discourse that the plague was returned again in Holland; for it had been very violent there, and particularly at Amsterdam and Rotterdam, in the year 1663, whither, they say, it was brought, some said from Italy, others from the Levant, among some goods which were brought home by their Turkey fleet; others said it was brought from Candia; others from Cyprus. It mattered not from whence it came, but all agreed it was come into Holland again.

@PROCBSE FROM THE DIARY OF ANNE FRANK (WORKBOOK)







(d) How was the news about the plague confirmed?





Q. 3. Tick the correct answer.



‘… the weekly bill of mortality’ means:



(a) The weekly payment of dues

(b) The weekly news about the deaths

(c) The statement about the hospital expenditure issued every week

(d) The list which was issued weekly, of persons who died due to plague Ans. (d)









(c) The rumour about the plague died off, as the Government held several councils to prevent its coming over and kept all private.





(b) During those days, the news about important events was gathered from the letters of merchants and others who corresponded abroad.







Ans. (a) The first paragraph of Daniel Defoe’s journals talked about the return of plague in Holland. It also describes the names of different places from where it might have returned.

323

(d) The news about the plague was confirmed when two physicians and a surgeon inspected the dead bodies and found evident tokens of the sickness upon both the bodies.



(c) Why did the rumour about the plague die off?































(c) The girls ____________ were helping you were probably Rahul’s friends. (which, who, that, they)





(b) One should learn to defend _____________ against bullies. (herself, itself, himself, oneself)

(d) These dresses belong to you. I am sure they are _________. (yours, your, yours’, theirs) (e) The fire caused serious ___________ to the group of houses. (damage, damages, injuries, injury) (f) The board was ________________ plans for the tournament. (concerned to, concerned about, concerned at, concerned with) Ans. (a) it (d) yours (b) oneself (e) damage (c) who (f) concerned about





(a) I can’t tell you the plan because I don’t know about it. (him, her, it, them)





1. Choose a word or expression that completes each of the following sentences. One has been done for you.











VOCABULARY

GRAMMAR

Madhvi doesn’t like her friend Sudha as she always wants to call the shots.

Q. 1. Refer to the lesson ‘From the diary of Anne Frank’ in your Class X textbook. There are some idiomatic expressions in the text. For example:





3.







4.





5.







Q. 2. Fill in the blanks using the contracted forms of the verbs. One has been done for you. This is a beautiful flower. Is not it? Isn’t it?



(a) You should not eat so much. You ________ eat so much. (b) They have written the story. ________ written the story.



Call the shots : to be the person who controls a situation.



2.







It was not good on the part of the shopkeeper to cheat his regular customers but all’s fair in love and war.

Break the ice : To make strangers feel relaxed. Our maths teacher cracked some jokes on first day of the class to break the ice.



All is fair in love and war : to deceit someone to fulfill your objectives.







Ans. 1.



6.

(c) Let us go watch a movie. _______ go watch a movie.



icing on the cake chalk and cheese breaking the ice







all’s fair in love and war call the shots raining cats and dogs

Chalk and cheese : To be completely different from each other. Me and my friend go well with each other despite being like chalk and cheese.







Now, look at the idiomatic expressions given below. Use a dictionary to find out the meanings. Make sentences using the expressions.

Icing on the cake : something that makes a good situation better. My friend was leading a happy life but her husband’s promotion was the icing on the cake.





(b) Until then we keep telling each other not to lose heart. Not to lose one’s heart: not to lose hope or not to feel demoralised



(a) Our entire class is quaking in its boots. Quake in (one’s) boot: to tremble with fear or nervousness

Raining cats and dogs : heavy rainfall. Rohit got soaked without an umbrella because it was raining cats and dogs.





Idiomatic expressions

@PROCBSE





(breakdown, find out, look for, go out, check out) Suhasini wanted to _________(a)_________ with her mother to the book shop. Her mother came back home very late. She said that her car __________(b)________ on the way back. She had to _________(c)_________ a mechanic. She ______(d)___________ several apps for roadside assistance but then _______(e)__________ that her internet was not working.

Ans. (a) go out (b) broke down (c) look for (d) checked out (e) found out









































(d) He did not play tennis. He __________ play tennis. (e) I could not find my friend in the crowd. I _________ find my friend in the crowd. (f) Here is your bag. __________ your bag. (g) I would guide him. __________ guide him. (h) They are not rich. They ____________ rich. Ans. (a) shouldn’t (b) They’ve (c) Let’s (d) didn’t (e) couldn’t (f) Here’s (g) I’d (h) aren’t Q. 3. Fill in the blanks with the phrasal verbs given below. Change the tense of the verbs wherever required.



Oswaal CBSE Question Bank Chapterwise & Topicwise, ENGLISH LANG. & LIT., Class-X

























324







(i) on the thoughts of Greeks and Romans, and of the Semitic race, the Jewish, may draw the corrective which is most want in order to make our inner life more perfect, ___________

(a) “If I was asked under what sky the human mind





(j) more comprehensive, more universal, in fact most truly human a life... ___________





again I should point to India.”









































find them pointed me has want most





(d) (e) (f) (g) (h) (i) (j)









(h) we who has been nurtured almost exclusively ___________

(c) on









(g) And if I were to ask me from what literature ___________

(b) for









(f) studied Plato and Kant, I should pointed to India. ___________

(a) was









(e) well deserve the attention even of them who have ___________

Incorrect







(d) of life, and has find solutions of some of them which ___________

Ans.







(c) has most deeply pondered on the greatest problems ___________









(b) has most fully developed some for its choicest gifts, ___________



___________





Q. 1. Read this quote from Max Müller. There are errors in the text. Find out the errors and write the correct word in the space provided.





EDITING

by Max Miiller

Correct were of over found those point myself have wanted more





1. Listen to the following YouTube link on ‘The Siege of Leningrad’ and make notes in the first box. Then write a narrative account of it in the second box. (Source : 8th September 1941: Start of ‘The Siege of Leningrad’ https://www.youtube.com/watch?v=5n2ym0LHyRU) Ans. Students to do under the guidance of their teacher.







LISTENING

(a) Ask your teacher to recite the poem. Listen to the poem attentively and identify the qualities mentioned.





(b) Discuss these qualities with your partner. (c) Think of a person who has some of these qualities and talk about them to your partner. You can highlight the incidents and actions of the person that reflect these qualities.





1. The following poem is taken from The Diary of Lena Mukhina.



SPEAKING

Lena Mukhina’s Diary 22 May 1941–25 May 1942 ‘Remember My Sad Story: Lena Mukhina’s Siege Diary’

@PROCBSE FROM THE DIARY OF ANNE FRANK (WORKBOOK)

325

Consider any day on which you do not discover anything new or learn anything useful as lost day! Anyone can become strong, smart and steadfast. There is only one essential requirement – will power! Will power can conquer anything. A strong-willed person is persistent and determined. People are not born brave, strong and smart. These qualities must be acquired through perseverance and with determination, like the ability to read and write. Ans. Students to do under the guidance of their teacher.











Ans.











1. You have read in the passage ‘A Journal of the Plague Year’ by Daniel Defoe about how the news of plague was reported in those times. What are the means of reporting today? Ans. Means of Reporting Gone are the days when it took news, months, and sometimes years to reach others. These days everything is done in the fraction of a second. The print media and electronic media, both are helpful means of reporting. The print media includes the newspapers, magazines, journals etc. where as the electronic media includes television, internet etc. Both play an important role in reporting as both of them cover masses and help in making the people aware about what is happening in and around the world. 2. Write a report on any one of the following-conservation of environment, plantation of trees, waste disposal, water harvesting, and the cleanliness drive under Swachh Bharat Abhiyan carried out in your locality.













WRITING

TREE PLANTATION By XYZ 7 July, 20XX A tree plantation was carried out in Mayur Vihar, New Delhi on 6 July, 20XX under Swachh Bharat Abhiyan. Many residents of the colony participated in the drive. The residents of the colony were informed beforehand through the posters. The drive started at 7 a.m., in the morning. Almost everyone who reached the spot had brought a sapling with him. It began with the chief guest, Mr. R.C. Khanna, the Mayor, planting the sapling of Neem tree. Then came the turn of others who planted the sapling of Banyan, Neem, Ashoka tree etc. one by one in the open area of the colony. Everyone seemed enthusiastic and excited as they all felt that they were doing a bit to save our planet. The area looked so green. The children were given the duty watching the plants. In the end, the chief guest addressed the residents and emphasised on the need to grow plants. The drive was successful in conveying the message. The programme was a huge success and ended with a note of hope and happiness.



@PROCBSE

UNIT

7

GLIMPSES OF INDIA



Exercises READING COMPREHENSION it is an exhausting exercise. The first time I made it under my grandmother’s supervision was also the last. I could hardly move my arms for the next two days! Since then, I gained a healthy respect for my dainty grandmother.

The following is an excerpt about the Parsi community in India and their food.



Teatime was special at our home. My grandmother made it a point to dress for tea; I was made to do the same. Once we were ready, out came the treats: Parsi biscuits — batasa, nan-khatai and flaky khari; patrel, rolled, steamed arbi leaves stuffed with besan masala; kumas, rich Parsi cake, and my favourite, bhakra, sweet deep fried doughnuts. That tradition has stayed with me. I certainly don’t dress up, but I still need a snack with at least three cups of tea!... (Source: ‘Down Memory Lane with the Parsis of New Delhi’, Shernaz Italiah, The Wire)



Q. 1. Infer the meaning of the following expressions

(a) Nowrosji would cajole Parsi visitors off the train.

(b) Jer Bai would occasionally object to unknown visitors, she was always overruled.

(c) ...outside catering was unheard of.

(d) I gained a healthy respect for my dainty grandmother. Ans. (a) Nowrosji would politely greet the visitors getting off the train.



Food preparations began two days in advance. Dar ni pori (rich pastry stuffed with sweetened lentils) and malido (halwa) were carried in big vatus (pots) and served with puris. Anyone who has made malido can vouch for the fact that you need strong biceps;



During festive gatherings, the women went to town with the food; outside catering was unheard of. The poorer members of the community never knew who had paid for what. This tradition continued in the northern cities of Lucknow, Kanpur, and Allahabad well into the ‘70s. Sadly, that kind of life has almost gone forever.



...Writing about Parsis in the Delhi of old is not easy. There are memories of my grandparents and parents — a collective memory of generations past and people long gone, yet here in spirit....

The menu was extensive. Breakfast would start with either sev—brown vermicelli cooked in milk and served with fresh cream—or ravo, semolina pudding, Mithu dahi or sweetened curd made with full cream milk was an absolute must. This was followed by bafellaeeda, hard-boiled eggs, and kheemo kaleji, mutton mince with liver. For lunch, there was almost always smori dal chawal and macchi no patio — white boiled rice with yellow dal offset by a tart and tangy fish curry.





“Aavoji, aavo, aavo! Welcome to Delhi! Let me show you my beautiful city.” The booming voice of Nowrosji Kapadia could be heard across the length and breadth of the platform. It was Nowrosji’s favourite pastime: a walk to the Old Delhi railway station to greet the Frontier Mail, as it chugged into Delhi from Mumbai to Peshawar. With this refrain, an eager Nowrosji would cajole Parsi visitors off the train and take them home for a meal and often persuade some to stay overnight or for a few days. He would use this opportunity to tell them about the advantages of shifting to Delhi. Though his wife Jer Bai would occasionally object to unknown visitors, she was always overruled. This was the beginning of the community of Parsis in Delhi....



Read the passage and answer the questions.

















TEXT I

@PROCBSE







(d) In the 70’s, Parsis were very helpful. They would pay for the poorer members of the community without their knowing who had paid for what. That tradition has almost gone forever.





(a) “Aavoji, aavo, aavo! Welcome to Delhi! Let me show you my beautiful city.” Who said this and why?





(e) Making of malido (halwa) was an exhausting exercise. The writer’s grandmother always made it. That’s why she became respectful to her grandmother.



(b) Why were Jer Bai’s arguments always overruled?

(f) The expensive teatime menu and the grandmother’s point to dress up at tea-time was special in the writer’s home.

( )









(c) sev

(d) mithu dahi (sweetened curd)

(e) bafellaeeda

(f) Kheemo Kaleji

(g) Smori dal chawal (h) Macchi no patio





Dar ni pori







(b) malido

Lagan Nu Custard









Q. 4. Make a list of Parsi delicacies as mentioned in the passage. A few are given above. You can add more to the list. Find the words for Mithu dahi in your language. Ans. (a) Dar-ni-pori

(d) Cooking malido, the Parsi style halwa.

Parsi Malido











Ravo

Dhansak

Ans. (d) Cooking malido, the Parsi style halwa.



(c) Writing about Parsis in old Delhi.

( )



(b) Travelling from Mumbai to Peshawar.



Patara ni Machhi





Ans. (a) Nowrosji Kapadira said this to pass visitors at Delhi Station. He wanted to use this opportunity to tell them the advantages of shifting to Delhi.

( )





(a) Preparing for the festival.



(f) What was special about the tea time in the writer’s home?

Q. 3. Tick the correct answer. What is “an exhausting exercise”?





(e) What has made the writer more respectful to her grandmother?





(d) According to the writer, what kind of life has almost gone forever?







(c) Why do you think outside catering was unheard of?































Q. 2. Answer the following questions.









(c) Outside catering was unheard of because the Parsi women would bring home-cooked food to the gatherings.

(c) No professional help was taken during festival gatherings. (d) She started giving respect to her grandmother more than ever before.

327

(b) Jer Bai’s arguments were always overruled because he wanted Parsis to visit his home and Delhi.



(b) Jer Bai would sometimes raise objection on bringing unknown persons to home, but her request was always set aside.



GLIMPSES INDIA (WORKBOOK)

( )

@PROCBSE Oswaal CBSE Question Bank Chapterwise & Topicwise, ENGLISH LANG. & LIT., Class-X













(a) in a place that is hidden (paragraph one) ____________

(b) distant or remote (paragraph two) ____________

(Source: ‘What’s Brewing in Araku Valley?’ by Diya Kohli, livemint, 23 Feb. 2019)



(b) The drive to far-flung coffee estates





(b) far-flung



(c) evrich



(b) Where is Araku valley?



(c) Underline and write below the sentences, words, and expressions which indicate that Araku coffee has moved ahead of being the only means of livelihood.



(d) State a historical fact about the coffee in the valley?

(e) There are specific terms being used in the passage for the cultivation of coffee for example, terrace farms, biodynamic farming, etc. Find more such terms and their meanings.



Ans. (a) The Araku Balloon festival in the valley held between 18-20 January every year is unique. (b) Araku valley is located at about 1,200 m above sea level in the north eastern corner of Andhra Pradesh.





(a) What is unique about the Araku valley as stated in paragraph one?





Q.3. Answer the following questions.









(d) aroma

(c) (i) globally appreciated product (ii) brand









Ans. (a) tucked away







(d) a smell specially a pleasant one (paragraph four) ____________

(iii) high





Q.1. Infer the meaning of the following: (a) traverses an unusual trajectory



(c) to improve the quality or flavour of something (paragraph four) ____________

(iv) for superior



(d) A British civil servant, N.S. Brodie introduced coffee in the valley.

Today, Araku coffee is a brand that works with 517 villages and 10,986 farmers, all of whom are estate owners and entrepreneurs with a stake in the business. The reason the coffee is of such high quality is that it follows the best practices of biodynamic farming by creating an interconnected and symbiotic ecosystem. The soil is enriched through composting and a variety of shade trees are planted, including cash-yielding fruit trees like mango and jackfruit. Terroirs are assessed and careful standard operating procedures are put in place from “sapling to savouring”, which ensures healthier plants and sweeter cherries and eventually a far superior coffee aroma and flavour.



The journey of Araku Coffee from a livelihood initiative by the Naandi Foundation to a globally appreciated product has been in the making for over a decade. It is also intrinsically tied to the history of how coffee arrived in this valley. It was a British civil servant, N.S. Brodie, who introduced coffee to these hill tracts in 1898.





The drive to far-flung coffee estates takes us through the five northern mandals of Anantha giri, Dumbriguda, Hukumpeta, Munchingi Puttu, and Araku valley. The name Araku for the coffee was chosen simply because it sounded poetic. The view from the ground level is as impressive as the one from above — the landscape changes from densely forested hills to sweeping valleys and terrace farms surrounding occasional village clusters.





Located at about 1,200m above sea level, the valley, tucked away in the north-eastern corner of Andhra Pradesh, shares a border with Odisha. For guests and participants at the Araku Balloon Festival (ABF), held between 18-20 January every year, this is sightseeing like nothing else.









Q.2. Find words from the text which means the following:







(d) a group of human settlement near rural areas.





(c) hills which had dense forests



Read the passage given below and answer the questions.

(b) The visit to the far-flung coffee estates



You have read about Coorg, its rainforests, and coffee and tea from Assam in ‘Glimpses of India’ (First Flight, Class X textbook, NCERT). The following extract is about the cultivation of coffee in Araku valley in Andhra Pradesh. The story of Araku Valley traverses an unusual trajectory through Adivasi empowerment, hot-air balloons, and some of the best coffee in the world.

Ans. (a) to cross or travel through a curved path







(d) village clusters





(j) bhakra

TEXT II



(c) densely forested hills







(i) Kumos

(e) Composting → A method of decomposing organic solid waste

328

Planting Cash yielding Trees → Growing of tree which give profit as their value keeps growing.

@PROCBSE





(d) Coffee plant can give a cup of enjoyable coffee. ( ) 





(b) Coffee saplings are very delicate hence a lot of care is being taken of them. ( )









Ans. (a) Coffee plants are taken care of from the stage of plantation till its beans are ready for an enjoyable cup of coffee.







(a) Coffee plants are taken care of from the stage of plantation till its beans are ready for an enjoyable cup of coffee. ( )

329

(c) The growth of coffee plants is dependent on shady trees. ( )





Q. 4. The expression “sapling to savouring” in paragraph four means: (Tick the right answer)





GLIMPSES INDIA (WORKBOOK)

Ans.

of an engine



(c) Puffing

(d) Clanking

(g) Banging

of leaves



(f) Rustling

of the door



(h) Gurgling (i) Howling

of chains of the wind



(e) Sighing



B of chains of leaves of feet of a match of the door of the wind of an engine of hyenas of water

of feet of a match



(b) Striking



(a) Shuffling

The meanings of some words are given below which will help you to find out the words. A (a) Shuffling (b) Striking (c) Puffing (d) clanking (e) sighing (f) rustling (g) banging (h) gurggling (i) howling

B



A

Example: rustling of leaves











VOCABULARY 1. Match the sounds in Column A with the appropriate words from Column B.

of water of hyenas





Ans. (a) proposed



(b) watching



(c) quoted



(d) infected



(e) taken/used

Her performance in the test was pleasing.

l

She appeared delighted with the gift.

l

The bored students became restless.





(infect, take, quote, use, watch, frighten, propose)

(a) People demonstrated against the changes ______________.

(b) The loud cheer went up from the students ______________.

(c) After the editor read the article, she checked all the examples ______________.

(d) Rub the antiseptic cream in the ______________ area. the

things

(f) Cyclone Fani was a ______________ experience.

Birds, bees, and butterflies are there to give you company. Macaques, Malabar squirrels, langurs, and slender loris keep a watchful eye from the tree canopy. I do, however, prefer to step aside for wild elephants. The climb to the Brahmagiri hills brings you into a panoramic view of the entire misty landscape of Coorg. A walk across the rope bridge leads to the sixty-four acre island of Nisargadhama. Running into Buddhist monks from India’s largest Tibetan settlement, at nearby Bylakuppe, is a bonus. The monks, in red, ochre, and yellow robes are amongst the many surprises that wait to be discovered by visitors searching for the heart and soul of India, right here in Coorg.

Ans. Birds, bess and butterflies were there to give you company. Macaques, Malabar squirrels, langurs and slender loris kept a watchful eye from the tree

among



(e) My wallet was ______________.



Now fill in the blanks of the sentences with past participle from one of the following words.











2. Rewrite in the past tense the following excerpt from the ‘Glimpses of India’ (First Flight, Class X textbook) and check your answers with a partner.

l



Rita gave me a welcoming cup of coffee..





l



(f) frightening







GRAMMAR 1. You have read about -ing and -ed adjectives in the textbook, the First Flight, page number 97. Read it again to understand how these adjectives are used. These are also the present participles (-ing forms) and past participles (-ed forms) of verbs and are used as adjectives. Read the following sentences to know this.

@PROCBSE 330

Oswaal CBSE Question Bank Chapterwise & Topicwise, ENGLISH LANG. & LIT., Class-X

canopy. I, however, preferred to step aside for wild elephants. The climb to the Brahmagiri hills brought you into a panoromic view of the entire misty landscape of Coorg. A walk across the rope bridge led to the sixty four acre island of Nisargadhama. Running into Buddhist marks from India’s largest

Tibetan settlement, at nearby Bylakuppe was a bonus. The monks, in red, ochre and yellow robes were amongst the many surprises that waited to be discovered by visitors searching for the heart and soul of India, right here in Coorg.

1. Some words have been wrongly spelt in the paragraph given below. Correct the words and read the paragraph aloud.

to ban all (h) mountainering, trekking, expeditions and livestock (i) grasing in the area, with effect from February 20, 1983. This is to (j) privent further damage to one of our nation’s unique repositories of natural beauty.





variety

(c) magnifiscent



magnificent

(d) cigarete



cigarette

(e) Trakking



Trekking

(f) delikate



delicate



compelled































grazing

(j) privent



prevent













(i) grasing



mountaineering



(h) mountainering –









(g) compeled







Over the past few years, thousand of visitors have ‘discovered’ this idyll amidst the mountain. However, they live like marks only by way of empty cans, waste-paper and (d) cigarete butts. (e) Trakking in the area has resulted in the trampling of these (f) delikate plants by humans and ponies. As a result, the government of India was (g) compeled



(b) varity



highway









Ans. (a) heightway







The Valley of Flowers The journey to the Valley of Flowers begins about 16 km away from Joshimath, at Govindghat (1,770 m) on the Rishikesh- Badrinath (a) heightway. The Valley of Flowers is covered with a rich (b) varity of (c) magnifiscent plants.





EDITING



Now summarise the paragraph using your notes.

Ans. Students to do it under the guidance of their teacher.

The baker made his musical entry on the scene with the ‘jhang, jhang — sound of his specially made



During our childhood in Goa, the baker used to be our friend, companion, and guide. He used to come at least twice a day. Once, when he set out in the morning on his selling round, and then again, when he returned after emptying his huge basket. The jingling thud of his bamboo woke us up from sleep and we ran to meet and greet him. Why was it so? Was it for the love of the loaf? Not at all. The loaves were bought by some Paskine or Bastine, the maid-servant of the house! What we longed for were those bread-bangles which we chose carefully. Sometimes it was sweet bread of a special kind.

bamboo staff. One hand supported the basket on his head and the other banged the bamboo on the ground. He would greet the lady of the house with a “Good morning” and then place his basket on the vertical bamboo. We kids would be pushed aside with a mild rebuke and the loaves would be delivered to the servant. But we would not give up. We would climb a bench or the parapet and peep into the basket, somehow. I can still recall the typical fragrance of those loaves, loaves for the elders and the bangles for the children. Then we did not even care to brush our teeth or wash our mouths properly. And why should we? Who would take the trouble of plucking the mango leaf for the toothbrush? And why was it necessary at all? The tiger never brushed his teeth. Hot tea could wash and clean up everything so nicely, after all!











LISTENING Ask your classmate or teacher to read aloud the following passage from the text ‘Glimpses of India’ (First Flight, Class X textbook). Listen carefully, and note down the keywords, main ideas and interesting expressions in your notebook while listening.

l

Find out the location (look up the map) and accessibility to the places to be visited.

l

Calculate the cost of the stay.





In groups of four, discuss and plan a budget trip to Araku valley. You may keep the following in mind while planning. Add more points to the following as per your requirement.





1. Excursions give opportunities to explore nature, places of historical interest, etc. Tourists are interested in meeting people, understanding their culture, and enjoying the variety in food. The Indian Railways have recently introduced glass Vistadome coach in train to provide a delightful travelling experience to the picturesque valley of Araku in Vishakhapatnam.



SPEAKING

@PROCBSE

Find out the modes of transport and booking.

(b) Six slim slick slender saplings

l



(c) The sixth sheikh’s sixth sheep’s sick







Ans. Students to do it under the guidance of their teacher.



(a) Pre shrunk shirts



(e) Double bubble gum bubbles double







(d) Tie twine to three twigs







2. Try repeating some of these tongue twisters:





Now, prepare an itinerary of your excursion to Araku valley and share with the class.

331



Search for an economic and safe accommodation, perhaps a home stay.



l





GLIMPSES INDIA (WORKBOOK)

l

chugged into

l

vouch for

l

long gone















out came the treats

Now, write an account of a travel by train using these expressions. Describe the journey by giving details of distance, booking of tickets, facilities available, and suggestions for improvement. You can add pictures of places, people, etc.

A Journey By Train





During the summer vacation last year, I decided to visit Chandigarh, the beautiful city. It is almost 425 km. from Agra. I booked the tickets in Kalka Express. Finally, the day arrived when I had to travel. I reached the platform before time. A large number of people could be seen across the length and breadth of the platform. Soon, the arrival of the train was announced. A few minutes later, Kalka Express chugged into the platform. There was a great hustle and bustle. Everyone was busy getting into or getting down the train; finally the train started towards its destination. People started adjusting their luggage and then took their seats. Out came the treats after almost half an hour. People had brought delicacies from home to be consumed during the journey. I was enjoyed everything. I looked around my coach. Some people were sleeping while others were looking out of the window, enjoying the scene outside. I also felt sleepy. I don’t remember for how long did I sleep, but I woke up when my mother told me that we had reached Chandigarh. We took our luggage and got down the train to enjoy my holidays.

2. Write an advertisement for your favourite tea or coffee.



Ans.



length and breadth



l

l





1. Infer the meaning of the following expressions in the passage on Parsi community. “Aavoji, aavo, aavo! Welcome to Delhi.”





WRITING

m fro de ality a M t qu bes beans

Coffee

Superior coffee aroma better coffee flavor keeps you fresh for a longer time









it is called in Kashmiri has variety of breads for different occasions that are baked in the Kandur’s oven. Read about some of the Kashmiri breads: Tsot-Girda : is a medium–sized round everyday bread that is a must on every breakfast table in Kashmir. Lavasa : is a puffy level bread made from maida. Tsochwor or Tilvor : is a bagel–shaped, slightly hard bread with a sprinkling of sesame seeds on top. The baker prepares ‘tsochwor’ at noon time to be enjoyed with the afternoon tea.







3. You have read about breads and bakers in Goa. (First Flight, Class X). Traditional breads are famous in Kashmir as well. It is said that a Kashmiri’s day starts with a cup of piping hot nun chai (salty pink tea) and a crisp, freshly baked bread from the Kandur (the traditional baker). In Kashmir, the Kandur shop isn’t just a place where one goes to buy the morning and evening breads, but it is a social hub. It is a place where you get to hear and participate in discussions that range from gossip to political discourses. The Kandur shop or Kandur waan as





for more details visit www.coffee.co.in

@PROCBSE





l You

can talk to peers, teachers, parents, grandparents, community members and make use of internet.

l Make an interesting collage using pictures and

write a few sentences about each bread based on the information you have gathered. 







Roath: The sweet bread with a cake like texture is baked in traditional tandoor and is topped with dry fruits. However, Ans. this one bread is made for grand

rotis in different parts of the country.

: is another type of bread from Kashmir. It is mildly sweet.

l Now, collect information about breads, parathas,



Sheermal

occasions like weddings, child birth, or engagements and served with Kehva.



These are crispy palm-sized breads sprinkled with poppy seeds. These are — mith (sweet) and namkeen (savoury). The bread is taken with some traditional Dodh Kehva (milk Kehva).





Kulcha :





Oswaal CBSE Question Bank Chapterwise & Topicwise, ENGLISH LANG. & LIT., Class-X











332



@PROCBSE

UNIT

8

MIJBIL THE OTTER



Exercises



(a) He made a body belt for him to take him to the bathroom or for a walk.



1. You have read the chapter ‘Mijbil the Otter’. Discuss with your partner the commitment the author had towards Mijbil, what arrangements he made for his pet ? What good care did he take like getting fish, taking it for a walk or exercise, providing things, and space to play etc. Make a list and then share with other friends in the class.









(c) The author made arrangements for his safe transportation to London.He packed Mijbil carefully in a box and took him to the airport.



(d) He took a parcel of fish for Mijbil for the journey and gave it to the air hostess to keep in a cool place.



Ans. The author Gavin Maxwell wanted to keep another pet when his dog Jonnie died. But he did not want to keep a dog this time. So he decided to own an Otter to keep his loneliness and sadness away. He knew that owning a pet is a lifetime commitment. He sensed his responsibility and took great care of him. He made several arrangements for his convenience which are as follows:

(b) He arranged ping pong balls,marbles,rubber fruits and a terrapin shell for his play.





(e) The author exercised him outside the house and often took him for long walks.



2. Find out which illustrations will go with your list.



Ans. All the illustration will go with the above mentioned list. 3. Mijbil is an active animal who shows many emotions. Refer to the chapter and complete the following Table.







LET’S BEGIN

What Mijbil Does

How Mijbil feels

1. Plunges, rolls in water

1. Goes wild with joy.

2.

2.

3.

3.

4.

4.

5.

5.

6.

6. What Mijbil Does

How Mijbil feels

1. Plunges, rolls in water

1. Goes wild with joy.

2. Plays with rubber balls

2. Enjoys every moment of it.

3. Bounds on author,s knees in aeroplane

3. shows affection by nuzzling his face and neck.

4. Uses a suitcase as a plaything

4. Gets engrossed in the game

5. Juggle uses small objects between two paws.

5. feels amazing

6. Plays with water in bathroom.

6. keenly intereseted in surroundings.

@PROCBSE 334

Oswaal CBSE Question Bank Chapterwise & Topicwise, ENGLISH LANG. & LIT., Class-X









My home is a little thatched hut by a river in a small village in Assam. The hut has an earthen floor and mud-splattered walls. Its doors are always open. I come and go as I please! Mama takes care of all three of us! Her name is Anjali. She cooks, cleans, and plays with me. I secretly think she loves me the most. She lets me be real naughty!



Hi! My name is Bhakat. I am a baby Civet cat. I am three month old. Shhh! I am taking a nap with my brothers. Yes, I live in a family of humans. They treat me like their own Baby!





Baby Bhakat

Mama is about to make brunch! She is making roti and vegetables. I don’t like vegetables, but I like fruits. I love meat. I am omnivorous. Mama always feeds me before anyone else! I love things dipped in milk. They are yummy! The man of the house is Dharini. If not for him, I would not be alive today! My civet mother built a nest for me in a very tall coconut tree. One night I slipped and fell to the ground! When villagers found me, they took me to Dharini. Dharini loves animals. I began to live in his home. I was very small. I could not walk. I ate and slept a lot! Do you know why my eyes are so large? That’s because I am a nocturnal animal. I sleep in the day and am active in the night. My large eyes help me see better in the dark! I also have a very long tail! It helps me balance on tree-tops and branches. I mostly live on trees, though I do come down to the ground. Animals like me are called arboreal! Inside Dharini’s house, I always snooze in the rafters. I like being high up; it reminds me of trees! One day I woke up to find a strange sight, Dharini’s house was filling with water! In the night, the river behind the house started overflowing its banks! Inside everything was helter-skelter. Things were piled on top of one another! The monsoon had arrived! Dharini and Anjali carried us babies outside the house! There was water everywhere and it was very muddy! But the sun was out and it was not raining! I stayed high on the fence and made sure even my long tail did not get wet! Dharini went to pick snails. Snails had climbed branches and walls to escape drowning in the flood! Every gardener knows that snails eat plants. By eating snails, I ensure that they do not overrun your garden. I maintain the balance of life in nature! When it floods, you find fish everywhere! Dharini cast his net in the garden and began catching fishes. They are kept in a pan of water to keep them alive and fresh to eat! Montu, my brother, tried to be naughty! He caught me and threatened to dip me in water! He got a scolding from Dharini. I hate to be in water! Back inside the house, it is time for me to play with Mama! I shall rest awhile and then be up again at night. Mama always leaves a midnight snack by the lantern! 



Civet cats are found in most parts of India. They are also called toddy cats. They live on palm trees and love to drink the sweet sap, which is collected by toddy tappers to make wine! Dharini and his family rescues and rehabilitates wild animals. These are mostly babies or animals that cannot look after themselves.



The following is a story of a baby Civet cat that fell from a tree and was raised by humans. It is a true story from Assam written by Rommel Shunmugam who is a conservation photographer. Through his photo stories, he seeks to inculcate in children a love for nature and wildlife.









TEXT I



READING COMPREHENSION

Rommel Shunmugam

@PROCBSE



MIJBIL THE OTTER (WORKBOOK)





(a) is the land owner.



(b) loves animals.



(c) is a gardener.



Q. 2. Civet cats are also called ___________________.

(a) Baby cats









(d) sleep in the rafter

Q. 5. The home for the Bhakat is _______________.

(a) a garden near a river





(c) a little hut by a river



Q. 6. Tick the statement which is not true.



(a) Dharani and Anjali treat Bhakat as their own baby.







(b) They keep the door closed to keep Bhakat safe.



(c) Anjali plays with Bhakat and lets it be naughty.



Ans. 1. (b) loves animals





(d) Anjali feeds Bhakat before anyone else.



2. (c) Toddy cats

It’s not easy having Elephants in your backyard.







Read the passage given below and answer the questions that follow.





6. (b) They keep the door closed to keep Bhakat safe.







5. (d) the nest in a coconut tree







4. (b) be in water











3. (d) meat





(d) the nest in a coconut tree











(b) the roof of the little hut







(c) play with Mama







(b) be in water





(a) eat snails





Q.4. Baby Bhakat hates to ___________________.









(d) meat







(c) fruits







(b) vegetables







(a) roti

Around the world, as communities expand, and natural wild places are reduced, people and wildlife are increasingly coming into conflict over living space and food. It might be baboons in Namibia attacking young goats or elephants, in Nepal eating crops or European bears and wolves killing livestock. The problem is universal, affects rich and poor, and is bad news for all concerned. The impacts are often huge. People lose their crops and livestock (and therefore a source of income and food security), property, and sometimes their lives — even a severe injury caused by wildlife can result in a loss of livelihood. The animals, some of which are already threatened or even endangered, are sometimes killed in retaliation or to prevent future conflicts. Human-wildlife conflict is happening more and more, affecting a lot of different species. The effects of climate change will probably make the problem worse. How we are tackling human-wildlife conflict The solutions are often specific to the wildlife or area concerned, and are often creative and simple — for instance planting a barrier of crops that repel the animals (elephants and some other wildlife don’t like chilli, for example). An important aspect of the work is that it benefits both the animals and local people and actively involves the communities concerned (in the case of chilli, it can be sold to increase income). It’s about finding solutions that lead to mutually beneficial coexistence. The work has also often led to people being more enthusiastic and supportive of conservation, and has demonstrated that people can live alongside wildlife while developing sustainable livelihoods. (Source: https://www.wwf.org.uk/what-we-do/area-ofwork/ preventing-human-wildlife-conflict) Q. 1. What happens when communities expand? Q. 2. What is the meaning of “impact” in the passage? How it is affecting people and their life? Q. 3. What will make the human-wildlife conflict more intense? Q. 4. The passage talks about one solution to tackle human-wildlife conflict. What is it? Q. 5. These types of creative and simple solutions lead to — (Tick the statement(s) which is/are not true.)





Q. 3. Baby Bhakat loves to eat ___________________.







(d) Snail cats











(c) Toddy cats





(b) Night cats







(d) is father of Bhakat.





Now read the following questions and tick the right answer.

TEXT II



335

Q. 1. Dharini —









@PROCBSE Oswaal CBSE Question Bank Chapterwise & Topicwise, ENGLISH LANG. & LIT., Class-X





to in order to help animals. Logically and morally, there is no reason to differentiate in the way we treat the animals we share our homes with and those who are farmed for food. They are all individuals, with the same capacity to feel pain and fear. Animal rights help us to look past the arbitrary distinctions between different species, to rediscover our innate compassion, and to respect all animals equally. Anyone who cares about animals can start putting these principles into practice every single day with the food they eat, the clothes they wear, and the products they buy. These choices are a form of non-violent protest that makes a real difference both by reducing the profits of corporations that harm or kill animals and by creating a growing market for cruelty-free food, fashion, services, and entertainment. (Source: https://www.peta.org.uk/action) Q. 1. What do the people who support animal rights recognise? Q. 2. As human beings, what are our moral obligations towards animals? Q. 3. What is the right of animals as mentioned in paragraph three? Q. 4. Why should we rediscover our empathy, compassion, and respect for animals? Q. 5. Anyone who cares about animals can start putting “these principles” into practice. What are “these principles”?







2. As human beings, our moral obligations towards animals is not to harm them in any way and not doing anything that would interfere with their needs. 3. Animals have the right not to suffer at the hands of humans. They have the right to live a life of freedom, a life which is free from suffering and exploration.







Ans. 1. People who support animal rights recognise that all animals have an inherent worth, a value which is completely separate from their usefulness to humans. They have the right to live a life which is free from suffering and exploration.



Read the passage given below and answer the questions that follow. Animal rights People who support animal rights recognise that all animals have an inherent worth, a value completely separate from their usefulness to humans. Every being with a will to live has the right to live free from exploitation and suffering. All animals have the ability to suffer in the same way and to the same degree that humans do. They feel pain, pleasure, fear, frustration, loneliness, and familial love. Whenever we consider doing something that would interfere with their needs, we are morally obligated to take them into account. People often ask if animal rights mean that animals should have the right to vote or drive a car. Of course, that would be silly because those aren’t rights that would benefit animals. But animals have the right not to suffer at the hands of humans and to live their lives free from suffering and exploitation because they have an interest in doing so. That is the difference between equal consideration and equal treatment. Almost everyone cares about animals in some context, whether it’s a beloved family companion, an irresistibly cute kitten or a majestic wild animal seen in a documentary. After all, we each have some built-in capacity for empathy and compassion, as can be seen from the lengths that children often go















TEXT III



















Ans. 1. There is an increasing conflict over living space and food when communities expand. 2. The meaning of ‘impact’ in the passage is ‘result’ or ‘consequence’; it is majorly affecting the people, as they lose their crops livestock, property, and sometimes their lives. In this way, they lose their source of income and ford security as well. 3. The effects of climate change will make the human wildlife conflict more intense. 4. One solution to tackle human-wildlife conflict is to plant a barrier of crops e.g., chilli which repel the animals. 5. (d) people live with wildlife happily





























active community involvement. generate income naturally. mutually beneficial coexistence. people live with wildlife happily. be more enthusiastic and supportive of conservation (f) developing sustainable livelihood.

4. We should rediscover our empathy, compassion, and respect for animals, so as to realise that they are all individuals with some capacity to feel pain and fear. 5. ‘These Principles’ refers to the principles of nonviolent protest that convey that we must reduce the use of food products and clothing that harm animals. We must grow the market for cruelty-free food, clothing, and amusement.

(a) (b) (c) (d) (e)



336





(c) hostile, unfriendly, unreceptive







Ans. (a) ridiculous



agreeable,

(e) appalling, atrocious, wonderful, awful, dreadful



(b) suitable, unfitting, appropriate, right, proper

averse,

(d) peaceful, harmonious, quiet, serene, violent





1. The following words are from the text you have read. The synonyms for each word are given. Find the odd one out. (a) serious, grave, ridiculous, stern, solemn







VOCABULARY

@PROCBSE MIJBIL THE OTTER (WORKBOOK)







Ridhima received a mail from her father. (Noun)



(e) Cable : An NGO is planning to cable the rural areas in near future. (Verb) The cable has been laid underground. (Noun)











(d) Mail : Rajesh mailed the letter to his boss. (Verb)









(f) Sack : The officer sacked the employees on their negligent work. (Verb)



He put a sack on the floor to sit. (Noun)





(g) End : The story ended on a happy note. (Verb)





I will return your book by the end of this month. (Noun)





(h) Head : Mr. Sharma was ordered to head the meeting that day. (Verb)







I am having severe throbbing in my head. (Noun)







(i) Colour : Colour this wall in light blue shade. (Verb)









(j) Lead : This road leads to Jaipur. (Verb) Lead is the long black part in the centre of a pencil. (Noun)









The rainbow has seven colours. (Noun)





337

I forgot to keep my Geography book in the bag. (Noun)





(b) unfitting (c) agreeable (d) violent (e) wonderful 2. Look at the two sentences given below. These are from the text. Look at the words in italics. These are used as verbs. They can be used as nouns as well. (a) By then, it had crossed my mind… (b) Camusfearna, ringed by water… The noun forms of these two verbs are ‘cross’ and ‘ring’. (a) I’ve put a cross on the map to show where the river Mahanadi is. (b) She put the ring back in the jewelry box. A list of words is given below. Make sentences using these words as verb and noun. (c) Book (d) Mail (e) Cable (f) Sack (g) End (h) Head (i) Colour (j) Lead Ans. (c) Book : Please book the tickets for the evening show. (Verb)























































(b) an interesting short





Ans. (a) a blue, sporty Maruti



(c) to a beautifully decorated (d) a frightfully expensive designer



(e) The quickly prepared

2. Use the correct form of the word given in the bracket and fill in the blanks. One has been done for you.

Suresh went ______________________________ ___ house.

(a) The death of 40 jawans was _______________. We were _______________ when we heard the news. (shock) (b) I met an ____________________ person in the party. He was _________________ in tasting



(c) Suresh went to a house. It was decorated beautifully.



He made _______________________________ speech.



His behaviour was disappointing. I expected him to be more polite. People present there were disappointed. (disappoint)







(b) He made a speech. The speech was short. The speech was interesting.



I bought _________________________________ ______ car.



____________________________________ food was tasty.



(a) I bought a Maruti car. The colour is blue. It is sporty.

(e) The food was tasty. It was prepared quickly.



Now, join the sentences given below and make a simple sentence with an adjective phrase.

She bought _______________________________ ______ dress.



I went to a lovely Christmas party.





Example: I went to a party. It was a Christmas party. The party was lovely.

(d) She bought a dress. It is a designer one but frightfully expensive.







1. You have read about “noun modifiers” in the chapter ‘Mijbil the Otter’, page 112. These give more information about a noun, hence are called noun modifiers. These are adjectives or adjective phrases. Let us look at more of the adjective phrases and their usage.













GRAMMAR

@PROCBSE Oswaal CBSE Question Bank Chapterwise & Topicwise, ENGLISH LANG. & LIT., Class-X









(b) I _______________ lend him my books when he was preparing for his examination.

(c) He _______________ visit his friend in the hospital every evening. (d) Each time I called him he ___________ come immediately.







(f) When I was a child, I_______________believe that fire flies were small fire balls.







(f) used to



(d) would

(e) used to



(e) disgusting, disgusted, disgusted



(c) used to







(d) amused, amusing

(b) used to





Ans. (a) would



(c) excited, exciting







(b) interesting, interested



(e) During his hostel life, he_______________eat all kinds of food without grumbling.





Ans. (a) shocking, shocked







(e) It is very humid in the Summers. I don’t like this weather. This weather is ________________. I am ________________, as I sweat a lot. Children are _______________, as they can’t play outside. (disgust)

(a) Every day she ________________ come home from work with a rose to give to her mother.







(d) She got a new guest in her house. She was _______________ by his handling the pups. His voice was more ________________ when he was calling these pups near him. (amuse)











(c) Ravi is not very _________________ to visit new places. He tells his friends no place is _________________ compared to his village in Ranikhet. (excite)

3. You have read about how to describe a repeated action in the past by using ‘would’ or ‘used to’. Go through the rules again and fill in the blanks with ‘would’ or ‘used to’ as appropriate.



each and every dish kept in the food counter. (interest)



338

























bustards, peacocks etc. They protect them from poachers and provide them plenty vegetation. They keep water in the stone vessels for the animals to drink from and hang water-filled pots from the branches for the birds. 2. Parts of sentences are given below. Rearrange the parts and write meaningful sentences in the space provided. Use appropriate punctuation marks. (a) a town in Iraq/ in large numbers/ otters are found/ in the marshes near Basra. (b) to the market/ we were fatigued/ having walked so far/ on account of (c) 40 paramilitary personnel/ the suicide car/ Pulwama district killed/ bombing in (d) india’s first semi-high speed train,/ was flagged off with excitement/ the inaugural trip of/ Vande Bharat Express, (e) the eastern hemisphere/ the longer than all/ the nile is said to be/ other rivers in Ans. (a) Otters are found in large numbers in the marshes near Basra, a town in Iraq. (b) We were fatigued on account of having walked so far to the market. (c) The suicide car bombing in Pulwama district killed 40 paramilitary personnel. (d) The inaugural trip of/ Vande Bharat Express, India’s first semi-high speed train, was flagged off with excitement. (e) The Nile is said to be the longer than all other rivers in the eastern hemisphere.

















1. Use capital letters, full-stops, commas, and inverted commas wherever necessary in the following paragraph. the land of the Bisnois in rajasthan is known for conservation of wildlife. it is a living religion to the Bishnois. It is said This is probably only one religion in the world that is founded on the principles of conserving nature. It has followers over ten lakhs. For these many people protection of living beings is a way of life for the ishnois tree is sacred and their empathy and love extend to all living beings on earth. They protect the ecosystem that surrounds the village which is a safe haven for blackbucks chinkaras vultures great Indian bustards peacocks etc they protect them from poachers and provide them plenty vegetation they keep water in the stone vessels for the animals to drink from, and hang water filled pots from the branches for the birds. Ans. The land of the Bishnois in Rajasthan is known for conservation of wildlife. It is a living religion to the Bishnois. It is said, “This is probably only one religion in the world that is founded on the principles of conserving nature.” It has followers over ten lakhs. For these many people, protection of living beings is a way of life. For the Bishnois, tree is sacred and their empathy and love extend to all living beings on Earth. They protect the ecosystem that surrounds the village which is a safe haven for blackbucks, chinkaras, vultures, great Indian







EDITING





1. Following is a story of Gangaram, a 130 years old crocodile, who died in a Chhattisgarh village. Listen to the story carefully and answer the questions that follow. You can listen to the recorded story or ask your teacher, sibling, or friend to read the story aloud.



LISTENING The incident took place in Bawamohatra, a village in Bemetra district of Chhattisgarh. The residents of Bawamohatra gathered near the community pond and started weeping after they saw that the crocodile had died. The villagers were emotionally attached to the reptile and were heartbroken after

@PROCBSE 339



































6. Give three reasons to justify Gangaram was friendly: (a) Children could __________________________. (b) Gangaram never _________________________. (c) Gangaram was very ______________________. 7. What was the new name given to the village and why? 8. What will the villagers do to remember their friends? Ans. 1. The residents of Bawamohatra gathered near the community pond to see the dead crocodile. 2. The villagers were crying on the death of the crocodile as they were emotionally attached to the reptile and were heartbroken after his death. 3. Gangaram was the crocodile. 4. He was three-meter long. 5. He was 130 years old. 6. (a) Children could swim around him (b) Gangaram never harmed or attacked anyone (c) Gangaram was very understanding 7. The new name given to the village is-Magarmachha wala gaon—the crocodile’s village. The village was named so as the villagers used to identify themselves with the crocodile. 8. The villagers now wish to build a statue of Gangaram near the pond to remember their friend, who got the village a new name.















his death. About 500 people of the small village attended the last rites of their beloved crocodile, Gangaram. The over three-metre long crocodile was buried in Chhattisgarh’s Bawamohatra village after his last rites were performed by the villagers. A forest department official estimated that the crocodile was 130 years old. The crocodile was carried to its funeral on a tractor decorated with flowers and garlands. The villagers say Gangaram was friendly. Even the kids of the village could swim around him. He had never harmed or attacked anyone. He was not a crocodile but a friend and a divine creature for them, who was worshipped in this village. Some say that he would even eat rice and dal which was served by the villagers and kids. He was very understanding and if he saw anyone swimming near him, he used to go to the other side of the pond. Villagers used to identify themselves with the crocodile as people used to call this village — Magarmachha vala gaaon, the Crocodile’s village. In fact, the villagers now wish to build a statue of Gangaram near the pond to remember their friend, who got the village a new name. It may be weird, but it could be an example of how humans and animals can coexist without harming each other. 1. Why did the residents gather near the village pond? 2. Why were the villagers crying? 3. Who was Gangaram? 4. How big was he? 5. How old was he?



MIJBIL THE OTTER (WORKBOOK)































Mijbil : ______________________________________ _________________________________________ 2. You have read a story ‘The Bond of Love’ in the class IX textbook, Beehive. In this story, the author talks about a sloth bear that his wife kept as a pet. The sloth bear was sent to the Zoo when it became too big to be kept at home. Now, discuss in a group of four, the following points, list views, and then share in the class. (a) Keeping any big wild animal, like the sloth bears, at home (b) Keeping unusual pets for pleasure (c) Using tigers, lions, dancing bears in circus (d) Making the monkeys dance, cock fighting, bull fighting for amusement Ans. Students to do under the guidance of their teacher.





























1. We all have seen hand puppets on YouTube or during story telling sessions. Try to make one hand puppet of an Otter. The situation is: Mijbil playing with water and spilling water every where. Write a few dialogues between Mijbil and Maxwell. Be the characters and exchange dialogues. Maxwell : Oh! Mijbil, what have you done? Mijbil : He, he________________________________ _________________________________________ Maxwell : ___________________________________ _________________________________________ Mijbil : ______________________________________ _________________________________________ Maxwel : ____________________________________ _________________________________________





















SPEAKING





1. Based on the above points, write a paragraph giving your points of view. You can highlight on the loss of natural habitat, cruelty, loss of freedom etc. Ans. Cruelty Towards Animals he has to share the planet with animals as well as Man is a social animal who has so many needs and plants. He has cut down trees and turned the forest desires. In order to fulfil his needs, he has become into treeless area to make buildings or to use it for self-centred and greedy. He no longer realises that any other purpose. The loss of natural habitat has







WRITING

@PROCBSE 340

Oswaal CBSE Question Bank Chapterwise & Topicwise, ENGLISH LANG. & LIT., Class-X

taken its full on wild animals. They have lost their homes. Many of the animals become the victims of humans beings when they are kept at home, used for amusement like keeping usual pets, and

keeping monkeys for their performance public to earn money. This towards animals must be checked. Everyone has a right to live freely on the Earth and man must understand this.



















2. Read these quotations and write one paragraph in the context of ‘Animal Rights’. (a) “The greatness of a nation and its moral progress can be judged by the way its animals are treated.” – Mahatma Gandhi (b) “When it comes to pain, love, joy, loneliness, and fear, a rat is a pig is a dog is a boy. Each one values his or her life and fights the knife.” – PETA founder, Ingrid E. Newkirk Ans. Animals Rights Every living being on Earth has the right to live freely. No one should intervene in their life. Many people use animals for amusement, experiment, clothing, food etc. It is wrong. They should be treated in a kind and considerate manner. One must realise that animals too have their own needs and should not become the victim of man’s greed and selfishness. They have the right to live freely and more freely. We should respect their rights and let them live a life of their own.



@PROCBSE

UNIT

9

MADAM RIDES THE BUS



Exercises

Was the excitement the same on her return journey?





l



What all exciting things did she see from the bus?



l





canal,the grasslands,the palm trees and the distant mountains. Acres and acres of green fields also amused her. The hamlet past an old wayside shop or another vehicle coming towards them also kept her engrossed.

Discuss with your classmate and write a paragraph based on your discussion on the above mentioned questions.

Ans. Valli’s desire was fulfilled when she took her first ride in the bus. She was very happy when she saw many exciting things from the bus. She enjoyed the











LET’S BEGIN 1. In the story ‘Madam Rides the Bus’, Valli took a bus ride on the bus that she wanted to for last several months. She knew that she had to buy tickets for this. So she saved money suppressing all temptation to buy things like peppermints, toys, balloons, etc., and paid for the ticket.

But one thing that she enjoyed the most was the cow which was running very fast right in the middle of the road. The more the driver honked,the more frightened the animal became and galloped faster with her tail high in the air. But her journey back to her house was not exciting because the same cow which she had seen in the morning was lying dead by the roadside. It probably had been struck by some fast moving vehicle. The beautiful charming looked horrible now. The memory of the dead cow haunted her and dampened her enthusiasm.



Read the passage given below and answer the questions that follow. Vishav Driman (65 year old) and her husband Kamal Jeet (69 year old) set out to discover the hidden treasure of Uttarakhand by bicycle. Vishav shares their post-retirement adventure. Pedalling through the Golden Years At a time when our peers were slowing down, relaxing and watching sunsets from their front porch, we found a new passion — cycling. We started out with short daily rides, but found ourselves looking forward to them more and more, so much that we decided to venture further and cover more ground. There were obvious health concerns — the risk of performing a physically strenuous activity at a high altitude, the most obvious of them. However, it helped that we had lived in Uttarakhand’s capital, Dehradun, our entire lives, and were therefore acclimatised to the cold winds and narrow winding roads that characterise this beautiful state. We were also familiar with almost all the scenic mountain trails it had to offer.











TEXT I



READING COMPREHENSION Short rides extended to longer ones, as we started falling deeper in love with not just the sport, but the very idea of discovering a world we had always been a part of but never had the time to explore. Soon, a simple hobby turned into a project of sorts, and we started participating in cyclotrons and other cycling events being held in the city. Admiration and encouragement poured in from family members, friends and neighbours along the way, sparking a new thought — was it possible to travel across the state on our bicycles? It seemed daunting at first, but we knew we had to try. Our gear in place, our bikes serviced, and our minds set, we set off on an adventure we would remember for the rest of our lives. In my opinion, there are very few ways to enjoy and appreciate the beauty of Uttarakhand and cycling is the most thrilling of them all. Sunlight filtering through the trees and a cool breeze gently caressing your faces as you ride, coupled with breathtaking views of sal forests spread like a green cover on the mountains and surrounding valleys, the experience of pedalling across the state is truly inexplicable.

@PROCBSE Oswaal CBSE Question Bank Chapterwise & Topicwise, ENGLISH LANG. & LIT., Class-X













Read the poem given below and answer the questions that follow. Going Down Hill on a Bicycle, A Boy’s Song With lifted feet, hands still, I am poised, and down the hill Dart, with heedful mind; The air goes by in a wind. Swifter and yet more swift, Till the heart with a mighty lift













Makes the lungs laugh, the throat cry:— “O bird, see; see, bird, I fly. “Is this, is this your joy? O bird, then I, though a boy, For a golden moment share Your feathery life in air!” Say, heart, is there aught like this

Q. 5. Which trail occupies a special place in the couple’s heart?









Q. 4. What, according to the author, is the most thrilling way to enjoy and appreciate Uttarakhand? Why does she say so?





Q. 3. What was the strong passion that prompted the retired couple to take up this adventurous journey?





Q. 2. What made Vishav and her husband confident to venture into Uttarakhand on their bicycles?











In a world that is full of bliss? ‘Tis more than skating, bound Steel-shod to the level ground. Speed slackens now, I float

Q. 6. What are the expectations of a cyclist from such an adventurous trip through the mountains? Did Vishav and Kamal Jeet get these experiences from the 100 km ride?





Q. 1. What is the meaning of the expression “when our peers were slowing down, relaxing and watching sunsets from their front porch” and in what context has the author used this?

TEXT II





(Source: By Vishav Driman, India Travel 360)

Yes, the couple got these experiences from this 100 km. ride.



Of the many trails we navigated, there is one that occupies a special place in our hearts. It began from Ghanta Ghar, a popular tourist attraction in Dehradun, it took us through some of the most picturesque locations of the state such as Kimadi, Hathi Paon, Mussoorie and Chaar Dukaan, right up to George Everest point. This 100km ride (approximately) is also one of the few routes in Uttarakhand that encapsulates all the element that one expects while cycling here — steep slopes, sharp curves, endless hills, lush forests, and offroad experiences.







342



1. The author has used this to tell is that all her friends has started living a relaxed life after their retirement where as they both had different plans in their mind.



Awhile in my airy boat; Till, when the wheels scarce crawl,







Must end in a vale; but still, Who climbs with toil, wheresoe’er, Shall find wings waiting there. Henry Charles Beeching





Alas, that the longest hill

Q. 1. The poem describes the ________________________ of going downhill on a bicycle.



My feet to the treadles fall.







(b) happiness (c) excitement (d) satisfaction





(a) enjoyment





6. A cyclist expects steep slopes, sharp curves, endless hills, bush forests and off road experiences from such an adventurous trip through the mountains.





5. The trail from Ghanta Ghar to George Everest Point occupies a special place in the couples heart.





4. According to the author, cycling is the most thrilling way to enjoy and appreciate Uttarakhand. It is so because of the cool breeze gently caressing the face, the sunlight filtering through the trees and of the scenic mountain valley, its green cover of Sal forests









3. The strong passion of cycling and of discovering the hidden treasure of Utharakhand by bicycle promoted the retired couple to take up their adventurous journey.





2. Vishav and her husband were confident to venture into Uttarakhand on their bicycles because they had lived there all their lives, and were acclimatised to the cold winds, narrow winding roads and the scenic mountain trails.







Answer.

@PROCBSE MADAM RIDES THE BUS (WORKBOOK)







(b) For a golden moment share





3. The boy riding a bicycle down the hills feels, as if he is flying. He is referring to the golden movements of a birds ‘feathery life in air’.



Q. 4. What is the meaning of ‘full of bliss’ here? Can the boy find this kind of bliss in the world where he lives?





Your feathery life in air!”





4. Since the boy is fully enjoying his experience of cycling, he has used ‘full of bliss’ that means extreme state of happiness. No, he finds it more blissful than anywhere and anything else.



Q. 5. There is a comparison in the third stanza. What is it? Q. 6. What is the meaning of the last two lines of the poem? Does “toil” mean dedication, determination, and will power to climb the hill? What is the reward here?

5. In the third stanza, the boy has compared his joyride to that of a bird flying free in air.







6. Yes, the word ‘toil’ in the last two lines means dedication, determination, and will power to climb the hill. The lines convey that whosoever works hard and has the will power to do it with dedication will get rewarded in the end.





Q. 3. Which is the experience the boy is sharing with the bird? What is the golden moment he is referring to?





1. (a) enjoyment 2. The lines which depict the freedom like that of a bird enjoys are :



Answer.



343

(a) O bird, see; see; birds, fly



Q. 2. Which line(s) depict the freedom like that of a bird that the poet enjoys?



















(e) Ans. (a) (b) (c) (d) (e)

















1. Find out the words from the text ‘Madam rides the Bus’ that match with the given meanings. (a) inquisitive _______________________________ (b) intricate _________________________________ (c) mesmerizing _____________________________ (d) immense _________________________________











VOCABULARY hardly __________________________________ curious elaborate fascinating unending only





























(a) It was evening. She ________________ the light.



break, drive, get, fill, out, down, off, on, in, wake, turn up





B



A

(b) Do not ______________________ Ramesh. He was studying late at night yesterday. (c) The school bus ______________________ and had to be towed to the garage for repair. (d) They got in the car and _____________________. (e) She hurt herself, as she was _____________ of the car. (f) You have to __________________________ the form yourself. Ans. (a) turned on (b) wake up (c) broke down (d) drove off (e) getting out (f) fill in















1. Read the two sentences given below. The groups of words in italics are phrasal verbs. This way she picked up various small details. Valli said, “I am the one to get on.” The verbs picked and get are combined with up and on respectively. Together they make phrasal verbs. Now, complete each sentence using a verb from box A and a word from box B (by making a phrasal verb). Use the correct form of the verb wherever necessary.











GRAMMAR





1. Use capital letters and full stops wherever necessary and correct the spelling errors as well, in the following paragraph. An exciting boat journey gliding crosswise the river from Godkhali jetty will take to a place of incredible mystic beauty of Nature.







EDITING

The sundarbans a unesco World Heritage Site is located at the South eastern tip of the 24 Paraganas district about 110 km from Kolkata it got its name from one of the mangrov plants known

as Sundari (Heritiera Minor) Sundrbans are a part of the world’s largest delta, formed by the mighty rivers Ganga, brahmaputra and meghna it is one of the most attractive and alluring places remaining on earth, a truly undiscovered paradise the Sundarbans is the largest singal block of tidal, halophytic mangrove forests in the world. It is the largest tiger reserve and National Park apart from being a unique largest mangrove eco-system of the world, the sundarbns has the world’s largest deltaic

@PROCBSE Oswaal CBSE Question Bank Chapterwise & Topicwise, ENGLISH LANG. & LIT., Class-X



largest delta, formed by the mighty rivers—Ganga, Brahmaputra, and Meghna. It is one of the most attractive and alluring places remaining on Earth, a truly undiscovered paradise. The Sundarbans is the largest single block of tidal, halophytic mangrove forests in the world. It is the largest tiger Reserve and National Park.



Ans. An exciting boat journey, gliding crosswise the river from Godkhali jetty, will take to a place of incredible mystic beauty of nature.





mangroove forests and is also home to one of India’s most iconic wildlife species - the royal Bengal Tiger. The Sundarbans also serves as a crucial protective barrier for the inhabitants in and around Kolkata against the floods that result from the cyclones which are a regular occurrence. The Sundarbans have also been enlisted amongst the finalists in the New 7 Wonders of Nature.



344

The Sundarbans, a UNESCO World Heritage Site, is located at the South-Eastern tip of the 24 Paraganas district, about 110 km from Kolkata. It got its name from one of the mangrove plants, known as Sundari (Heritiera Minor). Sundrbans are a part of the world’s

Apart from being a unique largest mangrove ecosystem of the world, the Sundarbans has the world’s largest deltaic mangrove forests and is also home to one of India’s most iconic wildlife species the Royal Bengal Tiger. The Sundarbans also serves as a crucial protective barrier for the inhabitants in and around Kolkata against the floods that result from the cyclones which are a regular occurrence. The Sundarbans have also been enlisted amongst the finalists in the New 7 Wonders of Nature.

Q. 1. The land of Mizo is a large open area of _________________. The hills are ________________ and ________________. Ans. hill, blue, green. Q. 2. The state referred here is made of two names —













(a) ‘Mi’ means _____________________________.



(b) ‘Zo’ means _____________________________.





(c) and Mizoram means ____________.





(b) hill





(c) human dweller of a hill



Ans. Agricultural activities are associated with Mizo festivals. Mizos enjoy these festivals by singing and dancing.





Q. 4. What are the special features of a Mizo dance ? Ans. (i) The dance are the expressions of the gay. carefree spirit of the Mizos.

(ii) These dances are not intended for stage performance, rather they have been evolved for community involvement and participation.





Q. 5. The following statements characterise Mizo people. Tick the one which is not true.



(a) Mizos have a beautiful culture.

( )













Q. 3. Which activities are associated with Mizo festivals? How do Mizos enjoy these festivals?











Ans. (a) human beings







(b) They live in a beautiful natural environment.



( )





(d) They are a vibrant people.

( )







(c) They are a protective people.

( )







(e) They love to dance and not so much to sing. ( ) 

1. Following is a passage on Mizoram and its people. When one visits this place, one is enchanted by its natural beauty and the vibrant culture of the state. Listen to the passage carefully and answer the questions that follow. You can listen to the recorded story or ask your teacher, sibling, or friend to read the story aloud. Mizoram Once a tourist remarked, “I visited the Eastern States this time — where the people talk in songs and they always dance.” The Eastern region of India is no doubt charmingly novel in many of its features. Mizoram, literally the land of Mizos is an expanse of blue-green hills. Mizoram is broken up lengthwise into five major mountain ranges gradually descending from the south to the north. The hills are rugged and their heights uneven. The name of the state Mizoram is made of two words. ‘Mi’ means human beings, ‘Zo’ means a hill. Therefore, a ‘Mizo’ is a human dweller of a hill; Mizoram means the land of the hill-dwellers. Aizawl is the capital town of the State built in tiers along the hill. The Mizos have three main festivals — Mim Kut, Chapchar Kut, and Pawl Kut. These festivals are associated with their agricultural activities during which they enjoy life by singing and dancing. The Mizos, blessed as they are with a beautiful environment and rich culture, are a vibrant, and social people. They love to dance as much as they love to sing. They boast a number of folk and community dances handed down from one generation to the other through the ages. The dances are expressions of the gay, carefree spirit of the Mizos. It should be noted here that these dances are not intended for stage performance, rather they have been evolved for community involvement and participation. English with a Purpose A Textbook for Class XII (Core Course) NCERT, 2003







(f) They have a number of community and folk dances. ( )







(g) The community involvement and participation is always there. ( )

Ans. (e) They love to dance and not so much to sing.



















LISTENING

@PROCBSE

MADAM RIDES THE BUS (WORKBOOK)

345







































Valli (steadfastly): Because I____________________ __________ A voice (with concern): If you__________________ ___________ Valli (irritably): I’m not________________________ ______ Ans. Conductor (smilingly): Oh. really! You don’t say so Valli (commandingly): I have to go to town Here is my money (Conductor stretched out his hand to help her up.) Valli (smartly): I can get on by myself. You don’t have to help me.” Conductor (jokingly): Oh, please don’t be angry with me, my fine madam. (The bus started to move forward with a roar. Valli stood up on the seat to see the outside world clearly.) A voice (anxiously): Listen, child “you shouldn’t stand like that. Sit down.” Valli (haughtily): There is nobody_here who’s a child,” Conductor (smilingly): Oh, sir but this is a very grown-up madam. Valli (angrily): I am not a madam. Please remember that. And you’ve not yet given me my ticket.” Conductor (mimicking her): “I’ll remember,” Valli (steadfastly): Because I want to A voice (with concern): If you stand on the seat, you may fall and hurt yourself when the bus makes a sharp turn or hits a bump. That’s why we want you to sit down, child.” Valli (irritably): I’m not a child. Q. 2. Copy full question from NCERT Page 130 Ans. Students do this activity themselves under the guidance of their teacher.

























































1. Read Part II of the story ‘Madam Rides the Bus’. There are many conversations between Valli, the conductor, and some elderly people in the bus. Convert the conversations into dialogues between the characters. Many words like commandingly, haughtily, loudly (shouted), smilingly, irritably, curtly etc. have been used in this part. These are the different ways in which the characters have spoken. These show the mood, emotion, and intention of the characters. Now, write the dialogues and then in groups of three, enact it as a skit in the class. It was a fine spring afternoon. The bus was just on the verge of leaving the village. Valli, a small eight year old girl, wanted to go on a bus ride. Q. 1. Valli (loudly): Stop the bus! Stop the bus! Conductor (without looking up): Who is it? Hurry then! Valli (loudly): It’s me, I am the one who has to get on the bus. Conductor (smilingly): Oh. really!______________ Valli (commandingly): I have to__________________ Here is_____________________________________. (Conductor stretched out his hand to help her up.) Valli (smartly): I can___________________________ __________ Conductor (jokingly): Oh, please_______________ (The bus started to move forward with a roar. Valli stood up on the seat to see the outside world clearly.) A voice (anxiously): Listen, child_______________ Valli (haughtily): There is nobody________________ Conductor (smilingly): Oh, sir__________________ Valli (angrily): I am ___________________________ Conductor (mimicking her):____________________

































SPEAKING

WRITING



l Freedom should be given to them to move



l Safety of the young children is of utmost



l What type of education should be given to



l This is a vulnerable age and children are

around.

concern. them?







innocent.

Ans. Everyone in the world wishes to lead a life of freedom where no restrictions are imposed on him especially the youngsters. The young children are innocent and sometimes easily fall into a trap laid for them. It is at this stage that they need the guidance of the elderly and their parents who are much more experienced and have a caring attitude towards them. Many children feel that the elderly are fussy and impose restrictions on them. They feel that freedom should be given to them to move around or travel alone in a bus or auto-rickshaw. In









1. In the text, seeing Valli travelling alone in the bus, some of the characters showed concerns. An elderly woman asked, “Are you alone, dear?” The conductor questioned, “Won’t your mother be looking for you?” The old woman continued asking, “Is it proper for such a young person to travel alone? Do you exactly know where you’re going in town? What’s the street? What’s the house number?” Do you think that the concerns of the elderly lady and the conductor are apt and genuine? Will you say that it is applicable to young boys as well? Discuss in groups, make note and then arrange a debate competition. You can consider the following points for discussion or debate. Then write a paragraph considering these issues and give your opinion as well. l Girls and boys of Valli’s age can travel alone in a bus or auto rickshaw, etc.

@PROCBSE 346

Oswaal CBSE Question Bank Chapterwise & Topicwise, ENGLISH LANG. & LIT., Class-X







2. You have read that on her return journey, Valli saw the young cow lying dead by the road side. This cow was running very fast in front of the bus and had given Valli unbound happiness when she was travelling to the town. Now, write a paragraph on ‘Should the Domestic Animals be let loose to walk on the Roads freely‘ where the vehicles are plying ? Who is responsible for the death of the young cow ? Ans. SHOULD THE DOMESTIC ANIMALS BE LET LOOSE ON ROADS ?









my opinion, the children should take liberties only upto an extent and should follow the advice of their parents.

Man is a selfish animal. He has his own priorities and move accordingly. When it comes to the safety

of domestic animals, many of them fail in this task. They show an irresponsible behaviour. They domesticate animals but let them loose to walk on the roads freely where the vehicle are plying. Most of them do not care about the chaos that the animal is going to create on the road nor does he feel concerned about the safety of the animal. This is not a good thing. Everyone should behave as a responsible citizen; Domestic animals should be taken good care of. If the young cow in the chapter had not been allowed to roam on the road, it would never have died. It is a humble request to all the owners of domestic animals to take care of their animals.



@PROCBSE

UNIT

10

THE SERMON AT BENARAS



Exercises READING COMPREHENSION











TEXT II











































Q. 6. What does His Holiness, the Dalai Lama mean by “spiritual development”? Q. 7. A compassionate heart brings — (a) .................. (b) .................. (c) .................. Ans. 1. According to His Holiness, Dalai lama, if we want a happier humanity, intelligence should be accompanied with human affection and feeling. 2. The ultimate cause of the problem lies within the human mind. 3. ‘It’ here refers to proper motivation or proper development. 4. Dalai Lama says that proper motivation is important because every human action whether it is verbal or physical or even any minor action, needs some motivation. 5. There is a need of correlation between the intention of an individual and the interest of the society because if the moral values and standards of society’s behaviour are negative, everyone is going to suffer because of it. 6. By “Spiritual development” His Holiness, Dalai Lama means the sense of having a good and compressional heart which further leads to happiness. 7. (a) inner strength (b) less fear (c) less doubt













Read the passage given below and answer the questions that follow. In the future, if we want a happier humanity, a happy world, we must tackle the root of the problem. Of course, the economy and political power are also causes, but the ultimate cause lies within the human mind. Every human action, verbal or physical even minor actions, have some motivation. Proper motivation or proper development is an important factor. Thus, if intelligence is accompanied with human affection and compassion, what I call human feeling, then it is very useful. If society’s moral values and standards of behaviour become negative, each of us will suffer. Therefore, the intentions of an individual are very much related to the interests of society. There is a definite correlation. The educational system and family life are two very important areas. In the educational field, one has to take care not only of the brain, but also of one’s spiritual development. I say “spiritual development” not in a religious sense but simply in the sense of having a good and compassionate heart. If one has a compassionate heart, it automatically brings inner strength and allows for less fear and less doubt. Subsequently, one becomes happier and more open-minded, thus making more friends in society. (Source: The Transformed Mind, His Holiness, The Dalai Lama) Q. 1. According to His Holiness, the Dalai Lama what should we do if we want a happier humanity? Q. 2. Where does the ultimate cause of the problem lie? Q. 3. “What I call human feeling, then ‘it’ is very useful.” What is ‘it’ referred to here? Q. 4. Why does the Dalai Lama say that proper motivation is important? Q. 5. Why is there a requirement of correlation between the intention of an individual and the interest of society?





















TEXT I

Read the poem given below and answer the questions that follow. Father Gilligan was overworked and extremely tired, as he kept attending to the sick and dying among his parishioners. He was offering them spiritual comfort day and night. One night he fell asleep on a chair. God sent his angel down to help his most beloved servant of the people.

@PROCBSE

He knelt him at that word.







Or under green sods lay.



And he began to grieve.

He knelt, prayed, fell asleep;

Stood upright on the floor. “Mavrone, Mavrone! The man has died,











(a) He was frustrated and amazed.



(b) He was ashamed and annoyed.

(c) He was full of grief and self-condemnation.

Q. 5. Why did Father Gilligan kneel down when he heard the sick man’s wife say “When you were gone...”? Q. 6. How did God help when “the least of things” was in need?

Ans. 1. Father Gilligan had been busy attending the sick and was exhausted. He thought about himself and felt-that he had no rest, joy or peace in his life. The next movent he regretted having this thought and said these words.

The sick man’s wife opened the door:



“Father! You come again!”



“And is the poor man dead?” he cried. “He died an hour ago.” The old priest Peter Gilligan “When you were gone he turned and died, As merry as a bird.”



In grief swayed to and fro.





4. (c) He was full of grief and self-condemnation.

By rocky lane and fen;

3. No, it does not mean that he had no case for the safety of the house and people on the way. Father Gilligan had slept after he had many sleepless days and nights. When he got up, he felt bad that he couldn’t attend the person who needed his help, so he took his horse and rode it, as he wanted to reach that place as early as possible. 5. On hearing the words of the sick man’s wife, Father Gilligan knelt down in gratitude, as he realised that God had sent one of his angles to help him in his need.

He rode now as he never rode,

2. (b) the passage of time

6. When the ‘least of things’ was in need, God sent one of angles to the dying man in his last hours.

And rode with little care.



He roused his horse out of its sleep



While I slept on the chair,”



The old priest Peter Gilligan



Upon the time of sparrow chirp When the moths came once more,

Q. 4. How did the priest feel when he heard that the sick man was dead? (Choose the correct option.)



And whispered to mankind.

Q. 3. Why did the poet say that the priest “rode with little care”? Does it mean that the old priest, Peter Gilligan, had no care for the safety of the horse and people on the way? What did he not care about?



And God covered the world with shade,



And leaves shook in the wind;



And the moth-hour went from the fields,

They slowly into millions grew,

(c) the actions of Father Gilligan.



And then, half-lying on the chair,

And stars began to peep.

(b) the passage of time.





My body spake, not I!”

(a) the movement of moths.



And after cried he, “God forgive!

Q. 2. “The moth-hour of eve”, “the moth hour went” and “the moths came once more” establish a link. They suggest —



For people die and die;”

To help me in my need.

Q. 1. Why did Father Gilligan cry “my body spake, not I”?



“I have no rest, nor joy, nor peace,

Sent one of His great angles down



Another poor man sent for him,

For souls who tire and bleed,



At the moth - hour of eve,



Once while he nodded on a chair,

“He who hath made the night of stars



For half his flock were in their beds,





The old priest Peter Gilligan

Was weary night and day,









The old priest Peter Gilligan



The Ballad of Father Gilligan



Oswaal CBSE Question Bank Chapterwise & Topicwise, ENGLISH LANG. & LIT., Class-X















348

@PROCBSE THE SERMON AT BENARAS (WORKBOOK)

349































2. Use a dictionary to find out four words each having the meaning nearest to the ones given below. Example: wander — stroll; dawdle; drift; roam (a) vowed — (b) preached — (c) reserved — (d) awakened — (e) enlightened — (f) reigned — Ans. (a) vowed — promised; swore; pledged (b) preached — sermonized; addressed; delivered (c) preserved — kept; conserved; maintained (d) awakened — roused; activated; enlivened (e) enlightened — acquainted; cleared; fervidness (f) reigned — influenced; controlled; commanded























































1. Antonyms for the words in the text ‘The Sermon at Benaras’ are given below. Find the words and fill in the blanks. (a) unfit ______________________ (b) joy ______________________ (c) contentment ______________________ (d) found ______________________ (e) cheered ______________________ (f) lively ______________________ (g) calm ______________________ Ans. (a) befitted (b) sorrow, grief (c) desdation (d) lost (e) moved (f) weary (g) hopeless































VOCABULARY

Some of the reporting verbs are: tell, ask, reply, warn, say, report, answer, explain, exclaim, mention, promise, suggest, order, etc.









Example (a) The doctor said to me, “Drink eight glasses of water to avoid acidity.”



(b) The doctor told me to drink eight glasses of water to avoid acidity.







(c) “Please don’t tell anybody about my illness”, Amita said to me.

1. Read the passage (“Kisa Gotami had an only son, ... a child, husband, parent or friend.”) from the text, the Sermon at Benaras (page 134). Rewrite the paragraphs changing the direct speech into reported speech in the space given below.









(d) Amita asked me not to tell anyone about her illness.





In the sentences ‘b’ and ‘d’ there are changes of person (I–he, Meena-you-she, your-her). The reporting verb ‘said’ has been changed to ‘asked’.















(d) Alexa asked Meena if she had visited her home town Puducherry.









(c) Alexa said to Meena, “Have you visited your home town Puducherry?”









(b) Riki said that he was hungry.

Ans. Kisa Gotami had an only son and he died. In her grief, she carried the dead child to all her neighbours, asking them for medicine and the people said that she had lost her senses. They further added that the boy was dead. At length, Kisa Gotami met a man who replied to her request and said that he could not give her medicine for the child, but he knew a physician who could. And the girl requested respectfully to tell her who that was. And the man told her to go to Sakyamuni, the Buddha. Kissa Gotami repaired to the Buddha and requested him give her the medicine that would cure her boy. The Buddha answered that he wanted a handful of mustard seeds. And when the girl in her joy promised to procure it, the Buddha added that the mustard seeds had be taken from a house where no one had lost a child, husband, parent or friend. 2. Read the story given below. Change the direct speech into reported speech and rewrite the story in the space provided. Use the reporting verbs given above wherever necessary. You can use any other reporting verb you find suitable. It is said, “It is not how much we have, but how much we enjoy that makes happiness.” Being happy does not require everything to be good but an eye to see good in everything. One day a crow saw a swan. “This swan is so white,” he thought, “and I am so black. This swan must be the happiest bird in the world.” The swan replied, “I was feeling that I was the happiest bird around. But I think the parrot is the happiest bird as it has two colours.” The crow then approached the parrot. The parrot said, “I have only two colours, but the peacock has multiple colours.” The crow then visited a peacock in the zoo. “Dear peacock,” the crow said, “you are so beautiful. Every day, thousands of people come to see you. You are the happiest bird on the planet.”

For example





Reported Speech We use direct speech when we repeat someone’s words and indirect speech when we use our own words to report what someone else has said. We use an appropriate reporting verb, make pronominal (relating to pronouns) changes and change certain words denoting ‘nearness’ in the direct form to words suggesting ‘remoteness’ (in time and place) in the indirect form. (a) Riki said, “I’m hungry.”







GRAMMAR

@PROCBSE Oswaal CBSE Question Bank Chapterwise & Topicwise, ENGLISH LANG. & LIT., Class-X

parrot was the happiest bird, as it has two colours. The crow then approached the parrot. The parrot said that he had only two colours, but the peacock has multiple colours.











The peacock said, “I always thought that I was the most beautiful and happy bird on the planet. But I am entrapped in this zoo. I have realised that the crow is the only bird not kept in a cage, and could happily roam everywhere.” Ans. It is said that is is not how much we have, but how much we enjoy that makes happiness. Being happy does not require everything to be good but an eye to see good in everything. One day a crow saw a swan. He thought that swan was so white and he was so black. He further though that swan must be the happiest bird in the world.









350

The swan replied that he was feeling that he was the happiest bird around, but he thought that the

The crow then visited a peacock in the zoo. He said affectionately to the peacock that he is so beautiful. He further added the thousands of people came to see him every day and he was the happiest bird on the planet. The peacock said that he always though that he was the most beautiful and happy bird on the planet, but he was entrapped in that zoo. He further added that he had realised that the crow was the only bird not kept in a cage and could happily roam everywhere.





belonged to him because he had saved its life. It did not belong to Devadutta who had tried to kill. Ans. going to the







1. There are some omissions in the paragraph given below. Edit the paragraph, use suitable prepositions and words. The Wild Geese One day, Prince Siddhartha was going the royal gardens on his way the river. A flock wild geese passed overhead. Devadatta, the Prince’s cousin, shot arrow into their midst one of the geese fell. It was wounded and fell just front of Siddhartha. His heart was filled compassion when he saw poor bird bleeding profusely. He lifted it and he drew the arrow very carefully from its body. He took the bird with him. Devadatta sent a messenger claim the bird. But Siddhartha refused to give it. He said that

























EDITING

way to the flock of wild shot an arrow just in front filled with compassion saw the poor drew out the messenger to claim that it belonged

1. Following is a narrative on Sanchi Stupa. Listen to it carefully and answer the questions that follow. You can listen to the recorded narrative or ask your teacher, sibling, or friend to read it aloud.

Pradesh, India. It is one of the most important Buddhist monuments that portrays Buddhist art and sculpture. Starting from the third century BC through the twelfth century AD, it attracts the attention of thousands of visitors round the year including national and foreign tourists, archaeologists, and historians among others.





The fascinating and world famous Sanchi Stupa is located on a hilltop in Sanchi town, Madhya









LISTENING

This stupa is the oldest stone structure in India that was built during the Mauryan period. Originally, commissioned in the third century BCE by Emperor Ashoka, this huge hemispherical dome consists of a central chamber. In this chamber, the relics of Lord Buddha are placed. The original construction work of this stupa was supervised by Ashoka. His wife, Devi, was the daughter of a merchant of nearby

@PROCBSE THE SERMON AT BENARAS (WORKBOOK)



(e) What structures were added hemisphere in first century BCE?











()



(d) Where is the central chamber in the Stupa? What is



(d) The Central chamber is in the huge hemispherical dome. The relics of Lord Buddha are kept inside it.









(b) Who are the people who visit this Stupa throughout the year?

(c) It was built during the Mauryan period, originally commissioned in the third century BCE by Emperor Ashoka







Ans. (a) Sanchi Stupa in Madhya Pradesh. It portrays Buddhist art and sculpture. (b) Thousands of visitors round the year including national and foreign tourists, archaeologists and historians among others.

(e) Four ornamental gateways or toranas facing four directions and a balustrade surrounding the stupa were added in the first century BCE.



wheel, footprints, etc.



()

(d) symbolic depiction of Lord Buddha as throne,











(c) Lord Buddha’s relics

()

(f) b-events of Lord Buddha’s life.









huge

()

(b) events of Lord Buddha’s life

(a) Which Buddhist monument is referred to here and what does it portray?

(c) In which period the Stupa was built, and who commissioned it?

the





(a) stone ornaments





The sculptures on the ornamental gateways or toranas consist of decorative illustrations of events encompassing the life of Lord Buddha. These events are explained in the Jataka tales. The tree used here symbolises Lord Buddha. The most striking features regarding the stupa is that Lord Buddha has been depicted symbolically by figures like thrones, wheels, and footprints among others.

to

(f) The toranas are decorated with: (Tick the right answer







kept inside it?



village, Vidisha. Sanchi was also her birthplace as well as the venue of her and Ashoka’s wedding. Later, four ornamental gateways or toranas facing four directions and a balustrade surrounding the stupa were added in the first century BCE. The Great Stupa at Sanchi is one of the oldest stone structures in India and counted among the best conserved ancient stupas of Central India. It has been enlisted as an UNESCO World Heritage Site since 1989.

351









l

Discuss with your friend and note down— what could be the possible reasons for

someone’s grief or sorrow how can they overcome this Share your feelings and suggestions in the class.

l



Ans. Students do it themselves.

You have experienced a loss or an intense sorrow, especially caused by someone’s death. This can also refer to the loss of physical or cognitive abilities or the loss of something that was routine in your life such as friendship, head boy or head girl, playing

for school, etc. l



It is important to know that everyone has some problem or other, and they grieve in unique ways. Your grief is different than those around you.







SPEAKING

stand by them

some others will want to talk about their thoughts and feelings — be a support to them,

let the grieving person know you care for them — be patient

l

focus on what you can do, like holding their hand, etc. — a solace or a consolation



l





Ans.





l



reach out to others in their grief, but some people may not want to accept help; some will not share their grief — understand this



l



grief is expressed through a variety of behaviours — observe these



l





1. Write a paragraph on how you can help others who are in grief. You may consider the following points:











WRITING

Help Others The path of life is full of joys and sorrows. At one stage or the other, one has to face the loss of near and dear ones. It is a tough time and one must help other in their hour of grief. The first and foremost

@PROCBSE



2. Two poems are given in the textbook, First Flight, page 138 – 139, one by Kahlil Gibran, and the other by Rabindra Nath Tagore. Read the poems, discuss in groups of four on what these two poets want to say in their respective poems. Then write the summary of each poem and share it with your classmates. Ans. Students to do it under the guidance of their teacher.

thing is to stay connected with them. There may be some people who would like to talk about their thought, but there may be others who would not like to accept help or share the grief. One must be prepared to accept the mood swings, be supportive and standby them. It is only the way you show that you care which will matter. Holding their hand, reassuring hug can also make him realise that you care. It will be a solace for that person.



Oswaal CBSE Question Bank Chapterwise & Topicwise, ENGLISH LANG. & LIT., Class-X



352



@PROCBSE

UNIT

11

THE PROPOSAL

Chapter Overview

This play is about how wealthy families seek to tie up with one another through marriages in order to increase their property and wealth. Lomov is a rich bachelor who wishes to marry Natalya, the daughter of Chubukov who himself is a rich man. The play deals with how all the three characters are very quarrelsome. They quarrel over trivial issues and call each other names. Lomov is not really in love with Natalya, but wants to marry her as he feels she is a good house-keeper, beautiful and well educated. He feels that it is the time he should settle down. Natalya, who doesn’t know he has come to propose to her, feels that he has come to claim oxen meadows as his own. This results in a quarrel with her father Chubukov joining in. After much mudslinging, when Natalya comes to know that he has come to propose her, she becomes willing to marry him. The proposal is made and accepted, with the knowledge that quarrels will take place in life, and important things should not be side-lined because of this.



Exercises





2. disgust 5. sadness



Anger happiness







Ans. 1. 4.









Let’s Begin 1. Facial expressions are non-verbal communications. These expressions play an important role in a play. They are powerful communication tools. The human face is extremely expressive. It is able to convey countless emotions without saying a word. Expressions tell us the mood of the characters in the play. Some facial expressions are given below. The facial expressions are for happiness, sadness, anger, surprise, fear, and disgust. Look at the facial expressions and write which emotion do they express.

3. fear 6. surprise

@PROCBSE Oswaal CBSE Question Bank Chapterwise & Topicwise, ENGLISH LANG. & LIT., Class-X



the facial expressions given above. l The marriage proposal is a mockery of romance and marriage in the upper class. l The pride in property, even the superiority of dogs, take priority over love and marriage. l The characters lack patience, stop listening to each other, and then contradict each other. l Do you think these quarrels will promise a happy life? l What qualities would you consider for a wellmatched couple? l Do you think all of us should know how to manage our anger? Ans. Students do this activity themselves under the guidance of their teachers.















2. The play ‘The Proposal’ is set in only one location, the drawing room of Chubukov’s house. All three characters in the play are quarrelsome people and they quarrel over petty issues. Lomov and Natalya start with the issue like a piece of land that is situated at the border between the two neighbors’ properties. The discussion turns into a quarrel and the marriage proposal is forgotten. They shout at each other. They are again dragged into an argument over the superiority of each other’s dogs. Therefore, they drag their ancestors in their foolish quarrel. They again abuse each other and call names. Now, make groups of four and discuss the following points. While conducting the discussion, try to use





354

We can never control circumstances, people, or situations, as they are constantly changing. The only thing we can control is our response. Therefore, we have to increase our capacity to tolerate, ability to understand, and learn to nurture love for others.



































TEXT II

Let us all become the managers of our own anger. When angry, take a few moments to calm down (take five deep breaths, count up to ten, drink water, change your place) before responding. More often than not we do not have control over the situations that distress us. Getting physically active reduces stress. Funny dances, clapping, thumping your feet, a walk, making funny faces at the mirror, etc., go a long way in helping us let go of the anger or the frustration inside us. Once we have our feelings under control, go through all the aspects of the situation and try to see the same situation from different perspectives. If it is a problem, try to equip yourself with a few solutions before entering the same situation. Avoid holding a grudge with people and learn when to seek help from others. Realising your own shortcomings can become one of the greatest strengths of character over the years to come.

Q. 2. List any four strategies to manage anger. Q. 3. How can you develop strength of character as given in the passage above? Q. 4. Anger management helps you in (Tick the correct answer.) — (a) remaining always happy (b) developing strength of character (c) remaining stress free (d) learning how to respond to the situation Q. 5. What is under our control? How can we make it a positive one? Ans. 1. Anger is a normal and healthy emotion. It is absence of peace with oneself, people, or situation around us. 2. (a) Take five deep breaths. (b) Get physically active like clop, thump your feet etc. (c) Count up to ten. (d) Drink water. 3. We can develop strength of character by realising our own shortcomings. 4. (b) developing strength of character 5. Our response is under our control. We can make it a positive one by increasing our capacity to tolerate, ability to understand and by learning to nurture love for others.

Anger is a normal and healthy emotion only if we know how to respond to it. Uncontrolled anger can often harm us directly or indirectly whether we realise or not. Before we learn to address the issue, let us revisit the concept of anger. Anger is nothing but absence of peace with oneself, people, or situations around us. We express it either by being assertive or aggressive.

Q. 1. What do you understand by the word ‘anger’?



How Can We Control Anger? Do you get angry when your mother switches off the television? Do you get upset when you lose a game? Do you crib when your teacher does not pay attention to you? Most of us can have “yes” as an answer to one or more of these situations. Anger sometimes gets the better of us and leaves us to regret the consequences later. The question is ‘How can we control anger?’

















TEXT I



READING COMPREHENSION

On Violence There is a great deal of violence in the world. There is physical violence and also inward violence. Physical violence is to kill, to hurt other people consciously, deliberately or without thought, to say cruel things, full of antagonism and hate and inwardly, inside the skin, to dislike people, to hate people, to criticise people. Inwardly, we are always quarrelling, battling, not only with others but with ourselves. We want people to change; we want to force them to our way of thinking.

@PROCBSE

in the name of self-importance, in the name of the security of the family. You will have to consider the outward cruelty and violence, and the inward violence which you do not yet know. (Source: ‘On Violence’, On Education, J. Krishnamurti) Q. 1. What is the physical violence that J. Krishnamurti is talking about?



355

















2. The inward violence cannot be expressed. We fight and battle with ourselves and do not express it. We want people to change and we want to force them to our way of thinking.



3. (e) all of the above

4. According to him, the rich want to keep people poor and the poor want to get rich and in the process, hate the rich.

5. Education makes a man become really beautiful, healthy, sane, and rational. It will help human beings bring about a new society, a new world. 6. We can create a new world by not following the old, brutal culture. We have to ponder upon the outward cruelty of man to man in the name of god, religion, self-importance, and security of family.































Q. 2. What is the violence that cannot be expressed? Q. 3. War, the ultimate violence, could be due to (Tick the correct options) — (a) difference of ideas (b) religious principles (c) nationalities (d) the feeling to protect a little piece of land (e) all of the above (f) none of the above Q. 4. How has J. Krishnamurti given the feelings of the rich and the poor in the given paragraph? Q. 5. What is the role of education in a human being’s life? Q. 6. How, as a student, will you create a new world? Ans. 1. The physical violence that J. Krishnamurti is talking about is to kill others, to hurt other people thought or without, to say cruel things and to criticise people.



In the world, as we grow up, we see a great deal of violence, at all levels of human existence. The ultimate violence is war — the killing for ideas, for so-called religious principles, for nationalities, the killing to preserve a little piece of land. To do that, man will kill, destroy, maim, and also be killed himself. There is enormous violence in the world; the rich wanting to keep people poor, and the poor wanting to get rich and in the process hating the rich. And you, being caught in society, are also going to contribute to this. There is violence between husband, wife, and children. There is violence, antagonism, hate, cruelty, ugly criticism, anger — all this is inherent in man, inherent in each human being. It is inherent in you. And education is supposed to help you to go beyond all that, not merely to pass an examination and get a job. You have to be educated so that you become a really beautiful, healthy, sane, and rational human being, not a brutal man with a very clever brain who can argue and defend his brutality. You are going to face all this violence as you grow up. You will forget all that you have here, and will be caught in the stream of society. You will become like the rest of the cruel, hard, bitter, angry, violent, people and you will not help to bring about a new society, a new world. However, a new world is necessary. A new culture is necessary. The old culture is dead, buried, burnt, exploded, and vaporised. You have to create a new culture. A new culture cannot be based on violence. The new culture depends on you because the older generation has built a society based on violence, based on aggressiveness and it is this that has caused all the confusion, all the misery. The older generations have produced this world and you have to change it. You cannot just sit back and say, “I will follow the rest of the people and seek success and position.” If you do, your children are going to suffer. You may have a good time, but your children are going to pay for it. Therefore, you have to take all that into account, the outward cruelty of man to man in the name of god, in the name of religion,











THE PROPOSAL (WORKBOOK)



1. Some verbs are given below. Write their nouns and adverb forms in the space provided.



VOCABULARY Ans. verb

noun

encourage

encouragement

encouragingly

excite

excitement

excitably excitedly

trouble

trouble

trouble

troublesomely

beautify

beautify

beauty

beautifully

tremble

tremble

tremble

tremblingly

verb encourage excite

noun

Adverb

Adverb

@PROCBSE 356

Oswaal CBSE Question Bank Chapterwise & Topicwise, ENGLISH LANG. & LIT., Class-X



We can use ‘ask’ with or without an indirect object. Example My mother asked (me) if I was ready for the party. Madhu asked (the tailor) whether her dresses were ready. 2. Use ‘said’, ‘told’ or ‘asked’ to fill in the blanks in the following sentences. One has been done for you. Shilpi : Father, I want to go on a trip to Uttarkashi. Shilpi told her father that she wanted to go on a trip to Uttarkashi. Archana : It is very hot in Jaisalmer. Archana said that it was very hot in Jaisalmer. (a) The Scientist ______________ that dinosaurs lived 230 million years ago. (b) The teacher _____________ the students that she was going to conduct a new experiment. (c) Could you please ___________ me where the new bookshop is located? (d) The student _____________ (the teacher) if it was possible This needs to changed; doesn’t fit (e) It is requested that he should ___________ the truth. (f) Father ____________ (his son) whether he had paid his fees or not. Ans. (a) said (d) asked (b) told (e) tell (c) tell (f) asked

































































l































Reported speech 1. Some children are taken to a health clinic for a regular checkup. Rewrite the sentences they speak in reported speech. (Use complain, say, tell, ask, inquire, mention, etc., wherever necessary.) Ashish : I feel sleepy all the time. Neha : I get pain in my legs when I run. Sonal : I’m fine. Ritu : I had fever last week. Akash : Why do I always cough? Rehman : Should I walk everyday to keep myself fit? Sumi : Oh, thank you, Doctor. Now I know the reason for my constant stomach-ache. Ans. Ashish said that he felt sleepy all the time. Neha complained that she got pain in her legs when she ran. Sonal told the doctor that she was fine. Ritu mentioned that she had fever the previous week. Aksah asked why he always coughed. Rehman inquired if he should walk every day to keep himself fit. Sumi thanked the doctor and told him that she then knew the reason for her constant stomachache. l Though ‘tell’ and ‘say’ almost mean the same, there are differences in the usage of the two words. ‘Tell’ is always followed by an indirect object but ‘say’ is not. Example I told my sister that she was right. Ashish said he worked hard to achieve success.



































GRAMMAR













(e) you must hear/to be a great leader /and pains of people/ the unspoken joys Ans. (a) The inner determination of an individual can transform everything by giving ultimate expression to the infinite potential of human being. (b) The sun teaches us that you may go down many a time but keep rising. (c) It’s not how much we give but how much love we put into giving. (d) The citizen of tomorrow would mould the world into a globe, fit for human habitation. (e) To be a great leader, you must hear the unspoken joys and pains of people.



















Jumbled Sentences 1. Join the parts of the sentences given below and write meaningful sentences. (a) by giving ultimate/individual can transform everything/ expression to /the inner determination of an/the infinite potential of human being (b) down many a time/but keep rising/that you may go/the Sun teaches us (c) but how much love/much we give/we put into giving/it’s not how (d) fit for human habitation/would mould the world/the citizens of tomorrow/into a globe











EDITING







1. You have read a play by Anton Chekov. Given below is a story by the same author. Listen and enjoy the recorded story or someone may read it aloud to you. Then answer the questions that follow. Vanka (Vanka Zhukov, a nine-year old boy, was made an assistant to Alyakhin, the shoemaker. He didn’t go to bed on Christmas Eve. When everyone in the shop had gone to Church, he began to write





LISTENING on a crumpled sheet of paper. “Dear Grandfather Konstantin, he wrote”, I’m writing a letter to you. I wish you a Merry Christmas and all good things from the Lord God. I’ve no father and mother, and you are all I have left.”) Vanka raised his eyes to the dark windowpane. In his imagination, he saw his grandfather Konstantin standing there. His grandfather was a night watchman on the estate of a rich man. Konstantin was a small, thin, and lively old man of sixty-five

@PROCBSE 357







window. He remembered it was grandfather who always went to the forest to cut down a Christmas tree for the rich people, taking Vanka with him. They had a wonderful time together, the snow falling, the trees crackling and hares springing across the trees, When the tree had been chopped down, grandfather would drag it to the big house and they would start decorating it. The young mistress Olga, Vanka’s favourite, was the busiest of all. Vanka’s mother, when alive, had worked as Olga’s maid. Then Olga had given him all the sweets and played a lot with Vanka. However, after his mother’s death, Vanka was sent to the kitchen to be with his grandfather and from there to Alyakhin, the shoemaker. “Come to me, dear grandfather,” Vanka wrote on, “Please, take me away from here, have pity on me, a poor orphan. They are always beating me. I’m terribly hungry and so miserable that I’m always crying. Remember me to all my friends. I remain your grandson, Ivan Zhukov. Dear grandfather, please come soon.” Vanka folded the sheet of paper and then put it in an envelope he had bought the previous day. He thought for a while, then wrote the address: To Grandfather in the Village. He added the words: Konstantin Makarich. He was happy that no one had disturbed him while he was writing his letter. He ran out to the street to post it. Vanka had been told by the clerks that letter were dropped in boxes from where they were carried all over the world on mail coaches drawn by horse and driven by drunken drivers, while the bells jingled. Vanka ran to the nearest mailbox and put his letter in it. An hour later, lulled by sweetest hopes, he was fast asleep. He dreamed of a stove. His grandfather was sitting beside it, reading out his letter to the cooks. Eel, the dog, was walking round the stove, wagging his tail. Anton Chekov Q. (a) Vanka went to bed early on Christmas Eve as __________. (b) Why, of all people, did he write a letter to his grandfather? (c) How was his grandfather a very lovable person? (d) Vanka was beaten _________________ times. (e) Vanka was beaten because __________. (f) What was Vanka prepared to do for grandfather if he was taken away from the shoemaker? (g) He was sent to Alyakhin, the shoemaker,____________ (h) Who does Vanka remember most at Christmas, next to his grandfather? (i) How did he address the letter? (j) Did he sleep well after posting the letter? What did he dream? Ans. (a) He wanted to write a letter to his grandfather. (b) He write a letter to his grandfather to wish Merry Christmas as he was all alone with no parents and his grandfather is the only one who left with him. (c) His grandfather was lovable person of sixty five because his face was always crinckled with laughter.



































whose face was always crinkling with laughter. In the daytime, the old man slept in the servant’s kitchen or cracked jokes with the cooks. At night, wrapped in a big sheepskin coat he made rounds of the estate, blowing his whistle at regular intervals. He was always followed by his dogs, Brownie and Eel. Eel was a particularly remarkable dog. He was extremely respectful, endearing, and looked lovingly on friends and strangers alike; yet no one trusted him. He would creep behind someone and bite his leg or run off with a peasant’s chicken. Many a time, Eel was beaten mercilessly, yet he always managed to survive. At this very moment, Vanka thought, grandfather was probably standing by the gates, looking up at the bright red windows of the village church and cracking jokes with the servants. “What about a pinch of snuff?” he would say, holding out his snuffbox to the women. The women would take a pinch and sneeze and the old man would feel happy about it. “Fine for frozen noses, eh!” he’d say. The dogs, too, were given snuff. Brownie would sneeze, shake her head, and walk away looking hurt. Eel, too polite to sneeze, only wagged his tail. Vanka looked through the window. The weather was glorious. The air was fresh. The night was very dark, but the roofs of the houses and trees were all covered with snow. Stars twinkled in the sky and it appeared that they had been washed and placed there only for the holidays. Vanka sighed, and went on writing: “Yesterday, I was given a beating. That was because when I was rocking the baby in the cradle, I unfortunately fell asleep. Another day, the mistress of the house asked me to clean the fish. I didn’t know how, so she rubbed the fish all over my face. My friends laughed at me. Then there’s nothing to eat. In the morning, they give me bread, there is porridge for dinner, and in the evening, only bread again. The master and the mistress eat all the good things themselves. I sleep in the corridor and when the baby cries, I don’t get any sleep at all because I’ve to rock the cradle. Dear Grandfather, please take me away from here, take me to the village, it’s more than I can bear.” Vanka rubbed his eyes with his fists and sobbed. “I’ll grind your snuff for you, he went on”, I’ll pray to God to keep you healthy and if I ever do anything wrong, you can beat me as you like. If you think there’s no place for me, then I can clean boots or even work as a shepherd boy. Grandfather, it’s more than I can bear. I thought of running away to the village, but I don’t have any boots. If you’ll help me now, I’ll feed you when I grow up and when you die, I’ll pray for your soul just like I do for my mother. “Dear Grandfather, when they put up the Christmas tree at the big house, please take down a golden walnut for me and hide it in the green box. Ask the young mistress Olga, and say it is for Vanka.” Vanka sighed and gazed in the direction of the



























THE PROPOSAL (WORKBOOK)

@PROCBSE 358

Oswaal CBSE Question Bank Chapterwise & Topicwise, ENGLISH LANG. & LIT., Class-X

They had a wonderful time together, the snow falling,the trees crackling and hares springing across the trees. When the tree had been chopped down,grandfather would drag it to the big house where they decorate the tree altogether. (i) He address the letter as –To the Grandfather in the Village. He added the words-Konstantin Makarich and ran to the nearest mailbox to post it. (j) He was fast asleep and dreamed of a stove. His grandfather was sitting beside it, reading out his letter to the crooks,eel,the dog was walking round the stove, wagging his tail.





(d) many.



(e) When he was rocking the baby in the cradle, he fell asleep.









(f) If Vanka was taken away from the shoemaker he will pray to God to keep his grandfather healthy and if he ever do something wrong, his grandfather can beat. Vanka will feed his grandfather and when he dies Vanka will pray for his soul.







(h) Vanka remembered that how he and his grandfather always went to the forest to cut down a Christmas tree for the rich people.







(g) to serve him



(iv) I’m sorry. I’m using it right now. Maybe later.

(v) That’s okay. Thanks any way.





1. Enact the play in the class. Keep in mind— dialogue delivery— tone, voice modulation; non-verbal communications— facial expressions, gestures, movements, etc. Many things are communicated through these non-verbal tools.





Prem : Thank you very much. I think I can manage. I am preparing a chart on children’s rights.



Role Play Chaman : Hello Prem. You look tired. Is there anything I can do to help you?







Prem : Oh, can you? That sounds wonderful.



Chaman : I can help you in illustrating some of the rights.



Chaman : It’s a lot of work. Let me help you. Prem : Thank you, but I’ve already collected all the necessary information.

Chaman : Could you tell me the themes to be illustrated?



Prem : Oh, yes. It’s a good suggestion. I’ll list their duties also.



Prem : Here you are. I’m really thankful to you. Chaman : What about children’s duties?

Chaman : Thank you.





(i) Excuse me...

(ii) Yes? (iii) I wonder if you could lend me your dictionary — I’m doing my class work.

(Discussing, disagreeing)

giving

opinions,

agreeing,

and

Role Play Smita : Well! What do you think about violence on television? Romila : If you ask me, there is too much violence on television. Children should not be exposed to it. Smita : I don’t agree with you. Why do you say so? Romila : It’s not wise to expose children to a lot of fighting, shooting, stabbing, and killing. Smita : But children find it thrilling. Romila : No, they don’t. On the contrary, it brutalises them and makes them insensitive. They look upon violence and killing as common place incidents. Smita : But the young people now-a-days are quite violent. Romila : Precisely. It is the influence of television. Wouldn’t it be better if through television we give them the message of peace and togetherness ? Pair or Group Work Discuss with your friends the following topics — people going abroad for studies, building a dispensary in place of a playground, sports cannot be a profession, or any topic that is of interest to you and your friends.

Ans. Students to do under the guidance of their teacher.















Pair or Group Work Make up conversations like the one above imagining you are in the classroom. Use the clues given below.

Makeup similar conversations imagining you are in the following situations — in the kitchen, at the railway station, asking somebody to open the door, in a hospital, in the playground. Task II



Task I (Offering, accepting and refusing Help)































2. Two communicative tasks are given below. Practice in pairs or groups.



































SPEAKING

@PROCBSE THE PROPOSAL (WORKBOOK)

359

WRITING







1. Letters in the olden times were carried by mail coaches driven by horses. How are they carried today? List the types of letters sent by different means of transport. Ans. Letters in the olden times were carried by mail coaches driven by horses. These days they are carried through postal services which can be government or private. These services include the physical transportation of letters and parcels. Letters are also e-mailed. The different types of letters sent by transport are inland letters, postcards, parcels etc. 2. You have listened to the story of Vanka. Try writing an e-mail to your grandfather describing your life. Have you ever felt as bad as Vanka did? What did you do to feel happy again?













From : [email protected] Ans. To : [email protected] Date : 15 March 20XX Dear Grandpa, I am happy today, as finally I have made some friends in the hostel. I was not happy earlier because my classmates treated me as a stranger and made fun of me. I used to get very upset. Those were terrible days. I would sit alone in the class whereas everyone would enjoy especially during the lunch break and games period. In the beginning, I was a bit nervous, but then I started enjoying their jokes. When my classmates realised that I was also a fun-loving boy, they changed their attitude towards me. Now, I am very happy here and enjoying my stay here.











Ans. Lencho : I am good. You know I wrote a letter to God asking for seventy pesos. Vanka : Did you get the pesos ? What address did you write ? Lencho : Yes, I seventy pesos, I’ve written another latter, asking for thirty more pesos. I wrote ‘To God’ on it and posted. Vanka : I also wrote a letter to my Grandfather and addressed ‘To Grandfather’ in village. That means my letter will also reach my grandpa. Oh! I am so happy that I will meet my grandfather again.

































3. (a) You have read ‘A Letter to God’ in your textbook First Flight. Lencho wrote a letter to God expressing his desire. Did it reach God? No, it did not reach God . (b) Do you think Vanka’s letter reached his grandfather? What is the correct way to address a letter? Ans. (b) No, Vanka’s letter would not have reached his grandfather because he didn’t write grandfather’s address on it. The correct way to address a letter is to write the name of the person to whom it is addressed, his house number, the street where he lives and then the city. (c) Suppose Vanka and Lencho meet. Create an interesting dialogue between the two. What encouraged them to write letters to God and grandfather? Dialogue Lencho : Hello! Vanka Vanka : Hello! How are you? Lencho : __________________________________________. Vanka : __________________________________________.



@PROCBSE

PRACTICE PAPER - 1 Max. Marks: 30



Max. Time: 1 hour



   

































































































I. Read the following passage carefully : (10 marks) (1) Music is perhaps the most popular and widely practised form of fine arts, transcending all kinds of cultural and linguistic barriers. Any form of fine art is difficult to master and almost impossible to perfect and music is no exception. (2) Nature, it is learnt, has blessed almost two-thirds of the human race with musical ability of some sort. Music has the power to bring out the deepest emotions. In fact, it is a magic medicine and many seek refuge in it when they are depressed or stressed. It is this intimacy that makes us listen to music or even hum or sing sometimes. This singing, or realistically speaking, expressing one’s emotion musically sometimes takes a serious turn. (3) The desire to sing before an audience is innocent and beautiful and indeed it is perfectly alright to have such a genuine desire. But it is also important to understand that singing is an intricate art — a highly refined one at that, which requires systematic, prolonged and strict training, to be acceptable. This is an aspect we forget in our keen desire to reach the stage and perform. It is almost like preparing a formal meal for some specially invited guests, without even having learnt and trained in the basics of cooking. (4) These days almost everyone sings and it does not stop here. Most of us want to become professional singers. Result: a complete disregard for and ignorance of the training part, as the need is never felt to go through one and the urge to get to the stage and perform overcomes even a little desire to learn, if any. If at all, somewhere along the way one feels the need to gain some knowledge and training, it leads to hurried shortcuts and halfhearted attempts, best described as ‘Crash Courses’. (5) It is observed that those who have attained the so called partial success, suddenly feel that they lack the required knowledge and are not learned enough. But it is too late by then. It should be understood here that the stage or a performance brings in a different mind-set within the artist. It is always recommended and rightly said that while on stage, cover the mistakes and weaknesses if any, and get along. But the contrary is true when it comes to acquiring knowledge and during the learning process. While under training, the student is expected to make mistakes but then rectify those mistakes under the supervision and guidance of the teacher. Therefore, it is good to make mistakes and then be corrected during the process of learning as this subsequently makes one flawless and educated. This is a different mind-set. And these two mind-set discussed above, (those of a performer and that of a student) cannot coexist. On the basis of your understanding of the above passage, answer the questions that follow: (1 × 10 = 10) Q. 1. Music is perhaps the most popular and widely practised form of ____________. (A) performing arts (B) modern arts (C) linguistic arts (D) fine arts Q. 2. Nature has blessed almost two-thirds of the human race with ____________. (A) singing ability (B) musical ability (C) dancing ability (D) all of these Q. 3. _____ are brought out by the power of music. (A) Hidden talents (B) Feelings of joy and contentment (C) Deepest emotions (D) Hidden desires Q. 4. How can we say that music is a magic medicine? (A) It can make one cry or bring a smile on one’s face. (B) It can heal people. (C) It relieves people of their pain. (D) People forget their problems. Q. 5. What is best described as ‘Crash Courses’? (A) Detailed courses (B) Dedicated learning (C) Years of learning and practice (D) Hurried shortcuts Q. 6. The mind-set of a student of music should be ____________. (A) Never to make mistakes. (B) Always be perfect.

















SECTION - A (Reading)

@PROCBSE PRACTICE PAPER

361





Ethical Tourism



Geo tourism



Pro-Poor Tourism



Responsible Tourism



DEFINITION Responsible travel to natural areas that conserves the environment, socially and economically sustains the well-being of local people, and creates knowledge and understanding through interpretation and education of all involved (including staff, travellers, and community residents). Tourism in a destination where ethical issues are the key driver, e.g. social injustice, human rights, animal welfare, or the environment. Tourism that sustains or enhances the geographical character of a place - its environment, heritage, aesthetics, culture, and well-being of its residents. Tourism that results in increased net benefit for the poor people in a destination. Tourism that maximizes the benefits to local communities, minimizes negative social or environmental impacts, and helps local people conserve fragile cultures and habitats or species. Tourism that leads to the management of all resources in such a way that economic, social, and aesthetic needs can be fulfilled while maintaining cultural integrity, essential ecological processes, biological diversity, and life-support systems.

Based on data collected by a survey by Travel Bureau, the following market profile of an eco-tourist was constructed: Age: 35 - 54 years old, although age varied with activity and other factors such as cost. Gender: 50% female and 50% male, although clear differences based on activity were found. Education: 82% were college graduates, a shift in interest in eco-tourism from those who have high levels of education to those with less education was also found, indicating an expansion into mainstream markets. Household composition: No major differences were found between general tourists and experienced eco-tourists** Party composition: A majority (60%) of experienced eco-tourism respondents stated they prefer to travel as a couple, with only 15% stating they preferred to travel with their families, and 13% preferring to travel alone. (** xperienced eco-tourists = Tourists that had been on at least one “eco-tourism” oriented trip.) Trip duration: The largest group of experienced eco-tourists- (50%) preferred trips lasting 8-14 days. Expenditure: Experienced eco-tourists were willing to spend more than general tourists, the largest group (26%). Important elements of trip: Experienced eco-tourists top three responses were: (a) wilderness setting, (b) wildlife viewing, (c) hiking/trekking. Motivations for taking next trip: Experienced eco-tourists top two responses were (a) enjoy scenery/nature, (b) new experiences/places. 















Sustainable Tourism













Eco tourism

















Read the following passage carefully : The UN’s 2017 International Year tells that sustainable tourism is an important tool for development, most importantly in poor communities and countries. Today sustainability – environmental, social, and economic – is increasingly recognised as the benchmark for all tourism business. As noted by the UN World Tourism Organisation, 57% of international tourist arrivals will be in emerging economies, by 2030. The various ‘Tourism Terms’ are defined as follows: CATEGORY













II.

































(C) Cover up the mistakes. (D) Make mistakes but then rectify those mistakes. Q. 7. In our keen desire to reach the stage and perform we forget _______________. (A) It requires prolonged and strict training. (B) It requires confidence. (C) It requires hard work. (D) It requires talent. Q. 8. While performing on stage an artist must ___________. (A) make mistakes and seek guidance (B) cover the mistake and get along (C) stop singing (D) none of these Q. 9. Any form of ________ is difficult to master. (A) fine arts (B) dance (C) cooking (D) all of these Q. 10. ‘Transcending’ in para 1 means : (A) drown under (B) rise above (C) surrender (D) fail

@PROCBSE Oswaal CBSE Question Bank Chapterwise & Topicwise, ENGLISH LANG. & LIT., Class-X 



















2. promotes landscape appreciations

3. promises luxurious travel

4. includes being environmentally responsible 





1. showcases adventure sports 









































On the basis of your understanding of the above passage, answer the questions that follow: (1 × 10 = 10) Q. 1. In the line “……recognised as the benchmark”, the word “benchmark” DOES NOT refer to: (A) a basis for something. (B) the criterion required. (C) the ability to launch something new. (D) a standard point of reference. Q. 2. The World Tourism Organisation of the UN, in an observation, shared that: (A) emerging economies of the world will gain 57% of their annual profits from International tourists. (B) countries with upcoming economies shall see maximum tourist footfall from all over the world in the next decade. (C) a large number of international tourists in 2030 will be from developing countries. (D) barely any tourist in the next decade shall travel from an economically strong nation to a weak one. Q. 3. One of the elements that is important to eco-tourists on trip is: (A) wild and untouched surroundings. (B) cultural exchange. (C) car and bus rides. (D) fully furnished flats. Q. 4. Choose the option that lists the correct answers for the following: 1. Asha Mathew, an NRI, loves animals and wishes to travel to places that safeguard their rights and inculcate awareness of their rights. What kind of tourist is she? 2. Gurdeep Singh from UK is an environmental scientist and has always chosen to travel to places that are examples of a symbiotic relationship between man and nature. What kind of tourist is he? (A) (1) is an eco-tourist and (2) is a geo tourist (B) (1) is an ethical tourist and (2) is a geo tourist (C) (1) is a sustainable tourist and (2) is a pro-poor tourist (D) (1) is a geo tourist and (2) is a responsible tourist Q. 5. Based on your understanding of the passage, choose the option that lists the inherent qualities of geo tourism.





362











































































5. believes in commercializing forests 6. initiates donations for the underprivileged (A) 1 & 2 (B) 5 & 6 (C) 2 & 4 (D) 3 & 5 Q. 6. In the market profile of an eco-tourist, the information on gender indicates that: (A) female eco-tourists were more than the male eco-tourists. (B) the activity preferences were varied in females and males. (C) the choice of things to do on a trip were quite similar for both the genders. (D) male eco-tourists were frequent travellers. Q. 7. The education aspect in the market profile of the eco-tourist revealed that: (A) mainstream market trends were popular with undergraduates. (B) eco-tourists were only those who had basic education. (C) mainstream markets were popular tourist destinations for educated eco-tourists. (D) eco-tourism was no more limited to the small group of highly educated travellers. Q. 8. According to the survey conducted by the Travel Bureau, the total percentage of experienced eco-tourists who DID NOT prefer to travel alone was: (A) 60%. (B) 75%. (C) 15%. (D) 13%. Q. 9. According to the survey, one of the most powerful driving forces leading experienced eco-tourism to invest in new trips was: (A) setting up work stations in new places. (B) the chance to go camping in the wild. (C) competing with other eco-tourists as frequent travellers. (D) the opportunity to travel to new places. Q. 10. Who isn't an experienced eco-tourist? (A) The person who has travelled as an eco – tourist once earlier. (B) The person who is yet to travel even once as an eco – tourist (C) The person who is the regular eco - tourism enthusiast and, traveller (D) The person who is not regularly travelling on eco – tourism trips.





I. Fill in the blanks in the paragraph given below by choosing the most appropriate options from the ones that follow. Write the answers in your answer-sheet against the correct blank numbers. (Any three) [3] Reading books 1. a good hobby. Books open and change our outlook 4. the World. 





SECTION - B (Grammar)

2.

vast new world to us. They increase

3.

knowledge [1 mark each]

@PROCBSE

1.

(A)

has

(B)

was

(C)

are

(D)

is

2.

(A)

the

(B)

a

(C)

an

(D)

some

3.

(A)

his

(B)

one's

(C)

our

(D)

your

363

4. (A) towards (B) for (C) by (D) from II. The following paragraph has not been edited. There is one error in each line. Write the error and its correction as shown in the example. (Any three) [3] [1 mark each] 





PRACTICE PAPER

Error

Correction

We, there in India have so many

e.g.

there

here

religions, we ate so many different

1.

___

___

foods, we wear so much different

2.

___

___

kinds of dresses. Still we are much

3.

___

___

united than any other country on the world.

4.

___

___

1.

(A)

eaten

(B)

eating

(C)

eat

(D)

eats

2.

(A)

most

(B)

more

(C)

many

(D)

few

3.

(A)

most

(B)

more

(C)

many

(D)

few

4.

(A)

of

(B)

at

(C)

in

(D)

up







III. Read the dialogue given below and then complete the Report by choosing the correct options. Write your answers in your answer sheet against the correct blank numbers. [4][1 mark each]





Ram : I want to meet the Principal.





Peon : Sir, he is not in his Office. He has gone to attend a Meeting.





Ram : When will he come back to the Office?





Peon : I think in another ten minutes.





Ram : What are the Meeting hours?



Ram went to the Principal’s Office and told the Peon (A) __________ the Principal. Respectfully, the Peon told him that (B) __________ and (C) __________. So Ram asked him (D) __________. The Peon replied that he would be back in another ten minutes. Ram again enquired from the peon what the meeting hours were.

I. You are Parvati/ Praveen, Store In-Charge of Aman Stores, 32/54, Ravi Nagar, Delhi. Your school requires school bags and accessories in bulk for the commencement of the new session. Write a letter of enquiry to the Sales Manager, Duckback Industries Ltd. 13/4, Maharajpur, Delhi enquiring about the details of goods manufactured. [100 - 120 words] OR You wish to send your aged parents for the Char Dham Yatra. Write a letter to the tour manager of a leading travel agency inquiring about a reasonable package, requesting him to provide other necessary details for the same. Sales of Three Types of Vehicles in the Indian Market. Market from the Year 2011 to 2016 5 4.5 4 3.5 3 2.5 2 1.5 1

Two wheelers Four wheelers Commercial Vehicles

2011 2012

2013

2014

2015

2016







II. Study the given line graph carefully and summarize: it into a paragraph of 100-120 words.













(Creative Writing)

[5×1=5]

@PROCBSE 364

Oswaal CBSE Question Bank Chapterwise & Topicwise, ENGLISH LANG. & LIT., Class-X



OR The pie charts show the main reasons for migration to and from the UK in 2007. Summarize the information by selecting and reporting the main features and make comparisons where relevant. Write at least 100-120 words. Main Reason For Migration To/From The UK - 2007 Immigration

Immigration 22%

29%

15%

12%

13% 26%

30%

6%

18%

11%

4% 4%

Formal study

Accompany/join

Looking for work

Definite job

No reason stated

other









[40 Marks] I. Read the extract given below and attempt questions that follow. [1×5=5] "I can hardly believe my ears. These meadows aren't worth much to me. They only come to five dessiatins and are worth perhaps 300 roubles, but I can't stand unfairness. Say what you will, I can't stand unfairness." Q. 1. Who is the speaker of these lines? (A) Natalya (B) Chubukov (C) Chubukov's Aunt (D) Lomov Q. 2. To whom are these words said to? (A) Chubukov (B) Chubukov's Aunt (C) Natalya to herself (D) Lomov Q. 3. In what context is 'unfairness' being talked about? (A) To the worthlessness of meadows (B) The worth of meadows being 300 roubles (C) The claiming of meadows by Lomov (D) The possession of meadow lands by Chubukov's for years Q. 4. What could the speaker not believe? (A) The unfairness by Lomov (B) Lomov's claim over Chubukov's meadows lands (C) The worth of meadows being 300 roubles (D) The quarrel between Lomov and Chubukov over the meadows Q. 5. Which word in the passage means 'injustice'? (A) Worth (B) Unfairness (C) Dessiatins (D) Hardly 1 OR Read the extract given below and attempt questions that follow. Poor Kisa Gotami now went from house to house, and the people pitied her and said, "Here is mustard-seed; take it!" But when she asked, "Did a son or daughter, a father or mother, die in your family?" they answered her, "Alas! The living are few, but the dead are many. Do not remind us of our deepest grief." And there was no house but some beloved one had died in it. Q. 1. The community's response to Kisa in the above extract was somewhat different from before. Why do you think that was the case? (A) They had learnt from Buddha's sermons. (B) They were able to help Kisa in some way this time. (C) They understood parental grief. (D) They liked Kisa and enjoyed talking to her. 1

































































































SECTION - C (Literature)

@PROCBSE

























































































































































365

Q. 2. Which of the following options represent the correct understanding of the word "poor" in the phrase "Poor Kisa Gotami"? (A) in need of money (B) weak (C) unfortunate (D) inferior 1 Q. 3. "Do not remind us of our deepest grief." The tone of the speaker(s) is: (A) disillusioned. (B) sceptical. (C) ironic. (D) solemn. 1 Q. 4. Pick the option that explains — '...the living few, but the dead many.' (A) It shows the high death rate and low birth rate in the city of Benares. (B) It highlights the holy status of Benares where many Hindus go to die. (C) It throws light on the numerous loved ones the villagers had lost over time. (D) It reflects that many children who had died in the village for various reasons. 1 Q. 5. Imagine you are a photo journalist visiting the city at the time Kisa Gotami went from house to house. You documented her experience given in the above extract in a photo series. Your publisher wants to publish the photo series in three parts wherein Part 1 shows Kisa's visits to the houses; Part 2 depicts her conversations with people, and Part 3 captures Kisa's reflections at the end of the day sitting by the wayside. The publisher would also like you to choose titles for the series and its three parts. Look at the titles given below, and choose the options that provide the most appropriate set of titles. 1. Series Title - From Darkness to Light. Part I -Living in Loss; Part II - A Mother's Journey; Part III-Mustard Seed 2. Series Title - Mustard Seed. Part I - A Mother's Journey; Part II - From Darkness to Light; Part III - Living in Loss 3. Series Title - A Mother's Journey. Part I -Mustard Seed; Part II - Living in Loss; Part III - From Darkness to Light 4. Series Title - Living in Loss. Part I - From Darkness to Light; Part II - Mustard Seed; Part III - A Mother's Journey (A) 1&2 (B) 2&3 (C) 3&4 (D) 4&1 II. Read the extract given below and attempt the ANY THREE questions that follow. 1 × 3=3 (I am an orphan, roaming the street. I pattern soft dust with my hushed, bare feet. The silence is golden, the freedom is sweet.) Q. 1. The tone of the given lines is: (A) analytical. (B) despairing. (C) peaceful. (D) nervous. Q. 2. Read the statements A and B given below, and choose the option that correctly evaluates these statements. Statement A - The figure 'I' imagines a less than realistic view of being an orphan. Statement B - The figure 'I' does not like the speaker. (A) A is true, B is false, according to the extract (B) A is true, B cannot be clearly inferred from the extract. (C) A cannot be clearly inferred from the extract, B is false. (D) A is true and can be inferred from the poem, B is true too. Q. 3. The golden silence is contrasted with the _________________. (A) chaos of the street. (B) constant instructions received. (C) sweetness of freedom. (D) hushed, bare feet. Q. 4. The rhyme scheme 'aaa' in the above extract is followed in all other stanzas of the poem that are written in parenthesis, i.e. (). Why? Read the reasons given below, and choose the option that lists the most accurate reasoning: (i) It shows the simplicity of the child's thoughts. (ii) It reflects the harmony and rhythm of the child's inner world. (iii) It mirrors a child's expression. (iv) It highlights the poet's aesthetic sensibility. (A) i,iv (B) i, ii (C) ii, iii (D) iii, iv. 1 Q. 5. Pick the option that lists the usage of the word 'pattern', as in the extract above. (A) That is a lovely pattern for a wallpaper. (B) Poetry is a form of pattern making. (C) She patterned her hair after her favourite celebrity. (D) None of the above



PRACTICE PAPER

@PROCBSE 366

Oswaal CBSE Question Bank Chapterwise & Topicwise, ENGLISH LANG. & LIT., Class-X

OR























































































Q. 4. “The Lomov and the Chubukov have always had the most friendly, and I might almost say the most affectionate, regard for each other.” How would you evaluate Lomov and Chubukov’s relationship as neighbours? Q. 5. What is the theme of the poem- ‘How to tell the wild Animals’? IV. Answer any Two of the following questions in about 40 – 50 words each. [3 × 2=6] Q. 1. Why is Mrs. Pumphery responsible for Tricky's condition? Q. 2. Why did not Anil hand Hari Singh over to the police? Q. 3. What was Madam Loisel shocked to know at the end of the story? V. Answer any one of the following questions in about 100-120 words. [1 × 6=6] Q. 1. Bravery is spontaneous? Explain with reference to the poem 'The Tale of custard the Dragon'? Q. 2. A noted author, Richard Bach wrote - "For most gulls it was not flying that mattered, but eating. For this gull, though, it was not eating that mattered, but flight." Evaluate this statement with respect to the young seagull.





































































Belinda tickled him, she tickled him unmerciful, [1×3=3] Ink, Blink and Mustard, they rudely called him Percival, They all sat laughing in the little red wagon At the realio, trulio, cowardly dragon. Q. 1. Who was tickled by Belinda? (A) Ink (B) Blink (C) Custard, the dragon (D) Mustard Q. 2. Why did she tickle ‘him’? (A) To tease (B) Make fun of him (C) To help (D) Both (A) & (B) Q. 3. Who are Ink, Blink and Mustard? (A) Ink-black kitten (B) Blink- grey mouse (C) Mustard- Yellow dog (D) All the above are correct Q. 4. Why did they all laugh at ‘him’? (A) Because he was a joker (B) Because he was very huge (C) Because he was spiky (D) Because he was a coward Q. 5. What do the words ‘realio, trulio’ mean? (A) Really and Truly (B) Real and True (C) Only (A) is correct (D) Both (A) & (B) are correct III. Short and Long Type Questions: [4 × 3=12] I Answer any four of the following questions in about 40 – 50 words each. Q. 1. When we think of losses, we generally think of people or possessions. Time is considered a very precious commodity. Explain why time can probably be one of the things people bitterly regret losing/wasting. [The Ball Poem] Q. 2. Why did the lady in the control room give the pilot a puzzled look? Q. 3. Write the appropriate response comprising of at least two reasons to present a counter to the following:

@PROCBSE















Q. 2. Why did Fowler become white faced at the end of the story?

367

VI. Answer any one of the following questions in about 100-120 words. [1 × 6=6] Q. 1. Imagine that one of Mr. Herriot’s partners can understand the language of dogs and listens to Tricky on his last night with them (a) What might Tricky share about his experience? (b) How would he evaluate it in comparison to his home experience?









PRACTICE PAPER

@PROCBSE 368

Oswaal CBSE Question Bank Chapterwise & Topicwise, ENGLISH LANG. & LIT., Class-X

PRACTICE PAPER - 2 Max. Marks: 30



Max. Time: 1 hour



 



 





 





















































































































I. Read the following passage carefully : [10 marks] (1) Overpowering prey is a challenge for creatures that do not have any limbs. Some species like Russell’s viper, inject poison into their prey. Some others opt for an alternative non-chemical method - rat snakes, for instance, catch and push their prey against the ground to immobilize them, while pythons use their muscle power to crush their prey to death. But snakes can’t be neatly divided into poisonous and non-poisonous categories. (2) Even species listed as non-poisonous are not completely free of poison. The common Sand Boa, for instance, produces secretions particularly poisonous and dangerous to birds. So, the species does not prefer to take any chance – it crushes its prey and injects poison as an extra step. (3) Do vipers need poison powerful enough to kill hundreds of rats with just one drop? After all, they eat only one or two at a time. (4) While hunting, animals try their best to kill as efficiently as possible while their prey will put in all the efforts, use any trick to avoid getting caught and becoming a meal, such as developing immunity to the poison. For instance, Californian ground squirrels are resistant to Northern Pacific rattle snake getting caught and poison. (5) Competition with prey is not the only thing driving snakes to evolve more and more deadly poison. Snakes also have to struggle to avoid becoming prey themselves. (6) Some snake killers are prone to have partial immunity to poison. Famously, mongooses are sure; highly resistant to the poison of cobra, and with their speed and agility, they can kill snakes fearlessly and are relatively unharmed . It would be mean definite death of cobras as a species if they did not evolve a more deadly poison to stop and resist the mongooses. (7) Poison has another very important role. It has an extreme meat softening property; their specific enzymes break up the insides of the prey. Normally, a reptile depends on the sun’s warm rays to aid it in its digestion. (8) But I wonder if we cannot use the venom in our favour. In remote parts of India, local hospitality there often involves serving leather-tough meat. I chew and chew until my jaws start aching. If I spit it out or refuse, our hosts would be offended, I swallow like a python stuffing a deer, down its throat and hope I don’t choke. If only I had poison. On the basis of your understanding of the above passage, answer the questions that follow: (1 × 10 = 10) Q. 1. How do Russell vipers attack their prey? (A) Inject poison (B) Crush their prey (C) Push their prey against the ground (D) Bite their prey Q. 2. Rat snakes attack their prey by ___________. (A) injecting poison (B) crushing their prey (C) pushing their prey against the ground (D) biting their prey Q. 3. The Sand Boa attacks its prey by __________. (A) injecting poison (B) crushing its prey (C) both (a) and (b) (D) only (a) Q. 4. How many rats’ vipers consume at a time? (A) one or two at a time. (B) three to four at a time. (C) five to six at a time. (D) seven to eight at a time. Q. 5. Why do animals try to kill efficiently? (A) as their prey will use any trick to avoid becoming a meal. (B) as their prey might attack them. (C) as they are testing their skill. (D) as they are trying to learn new tricks. Q. 6. What immunity does Californian ground squirrels have? (A) They have grown bigger in size. (B) They inject poison. (C) They have become resistant to Northern Pacific rattle snake poison. (D) They can run extremely fast. Q. 7. Poison has one important role, it ___________. (A) makes the meat sweet (B) reduces the size of the meat (C) it acts as a preservative (D) makes the meat soft



















SECTION - A (Reading)

@PROCBSE PRACTICE PAPER

369

































































On the basis of your understanding of the above passage, answer the questions that follow: 1. How many soldiers fought in the battle of Solferino on June 24? (A) 2,70,000 (B) 40,000 (C) 2,30,000 (D) 23,000 2. What is the correct order of the information given below: I. Dunant organised a camp of volunteers. II. Emperor Napoleon III and Franz Joseph I fought a battle. III. The book ‘A Memory of Solferino’ was written. IV. Dunant travelled in Solferino. (A) IV, III, II, I (B) I, II, III, IV (C) II, I, IV III (D) II, IV I, III 3. The writer of ‘A Memory of Solferino’ was: (A) Emperor Napoleon II (B) Franz Joseph I (C) Jean Henri Dunant (D) International Red Cross





Read the following passage carefully : [10 marks] On June 24,1859, Emperors Napoleon III and Franz Joseph I were engaged in the Battle of Solferino, commanding a combined total of about 2,70,000 troops onto the field for a single day of battle. Nearly 40,000 were either dead, injured, or missing and many of them were simply left to die on the battlefield. Later, spectators crowded the fields, looking for loved ones, searching for items they could sell, or for simply taking in the horrors of the battle. A Swiss businessman and social activist Jean Henri Dunant, who was travelling in Solferino, witnessed all this. Jean Henri Dunant witnessed the atrocities of war as well as the helplessness of the countries not prepared or equipped to ease the suffering of those who had been injured in the Battle of Solferino. Dunant organised a group of volunteers to help bring water and food to the injured, to assist with medical treatment, or to write letters to the families of those who were dying. He also urged the public to create an organisation which would assist the wounded, regardless of which side they fought for during the times of war. From that moment, he wrote the book, A Memory of Solferino, which urged the public to create an organisation wh:ch would assist the wounded, regardless of which side they fought for during the times of war. His writing inspired countless others to rally behind him in the creation of the International Federation of the Red Cross. The modern-day Red Cross began by devoting itself largel to the acts of disaster relief and epidemic treatment. This effort continues to this day. One of the easiest ways you can help the Red Cross is to make sure you are able to donate blood and make an appointment at the Red Cross website. But donating blood isn’t the only way you can help out—the Red Cross also encourages you to donate your time if you can. This is what the Red Cross wants everyone to know.



II.

































Q. 8. The writer wished he had poison because ____________. (A) he wanted to hurt someone. (B) he wanted to know how the poison works. (C) he wanted to make the meat soft. (D) none of these. Q. 9. The synonym of the ‘another’ in para (1) is ___________. (A) opt (B) alternative (C) a creature (D) against Q. 10. The synonym 'periocular' in para (7) is ___________. (A) aid (B) extreme (C) enzymes (D) specific

(1 × 10 = 10)

@PROCBSE 370

Oswaal CBSE Question Bank Chapterwise & Topicwise, ENGLISH LANG. & LIT., Class-X  











































SECTION - B (Grammar) I.

[10 marks]



































































4. The modern day Red Cross does not deal with : (A) Blood donation (B) Disaster Relief (C) Epidemic Treatment (D) Writing a book. 5. The most appropriate sub-heading to para 2 of the passage is: (A) How International Federation of Red Cross Evolved! (B) Functions of Red Cross. (C) The Contributions of Jean Henri Dunant. (D) The Battle of Solferino. 6. The aid provided by Dunant’s volunteers during the battle of Solferino was: I. to help bring water and food to the injured. II. to assist with medical treatment. III. to write a book. IV. to give relief in epidemic. (A) I and III (B) III and IV (C) I and II (D) II and III 7. You can donate your blood and ______to Red Cross. (A) water (B) time (C) meals (D) books 8. To donate blood one should __________. (A) make an appointment with society members (B) make an appointment at Red Cross website (C) collect information from patients (D) collect information from neighbourhood 9. And a word from para 2 which means the same as ‘an individual or group who freely gives time to benefit others’ : (A) injured (B) Volunteers (C) Public (D) organisation 10. A word which means the opposite of ‘ancient’ in para 3 is ______ . (A) devoting (B) treatment (C) modern (D) donating



Choose the correct options to fill in the blanks to complete the note about the Wangala Festival of Meghalaya. (Attempt any three)





The Wangala 1. festival for the Garo in Meghalaya, Assam and Nagaland. It is a post harvest festival 2. the end of the Agricultural Year. It is popularly known as ‘The Hundred Drums’ Festival. During the Signature dance, the Leading Warrior 3. with Synchronised dance 4. and specific Hand-head Movements. [3] 1.

(A)

is important

(B) are an important (C)

was the important (D)

is an important

2.

(A)

being celebrated for marking

(B) celebrated to mark

(C)

celebrated to marking

(D)

being celebrated for mark

3.

(A)

leads the youngsters

(B) is lead the youngsters

(C)

was leading the youngsters

(D)

had leads the youngsters

4.

(A)

step

(B) steps

(C)

foot

(D)

feet





II. The following paragraph has not been edited. There is an error in each line. Write the error along with its correction in the space provided. [3] (Attempt any three) Error

Correction

Tallam is situated in an

1.

_______

_______

altitude of about 550 feet on a Southern

2.

_______

_______

arm of a deep Bay of the Western Ghats.

3.

_______

_______

Tallam boasts of delight Forest Scenery.

4.

_______

_______

1.

(A)

at

(B)

on

(C)

of

(D)

onto

2.

(A)

an

(B)

the

(C)

a

(D)

that

3.

(A)

of

(B)

off

(C)

in

(D)

on

4.

(A)

delighted

(B)

lighted

(C)

delightful

(D)

None of the these

@PROCBSE 371

PRACTICE PAPER

Read the dialogue given below and then complete the Report by choosing the correct options. Write your answers in your answer sheet against the correct blank numbers. (Attempt any four) [1 mark each] Ram : What a cold day it is ! I think this must be the Coldest day of the Season. Shyam : I don’t mind the cold. It is the Summer that bothers me. Ram : Well ! I am only worried about My Mother. She stays alone in Shimla Ram explained that 1. __________ and he thought that 2. __________ . Shyam replied that 3. __________ and further added that 4. __________. Ram stated that 5. __________ and that she stayed alone in similar. 1. (A) it was a very cold day (B) it is a very cold day (C) it will be a very cold day (D) it has been a very cold day 2. (A) it will be the Coldest day of the Season (B) it had been the Coldest day of the Season (C) it must be the Coldest day of the Season (D) it has been be the Coldest day of the Season 3. (A) he didn’t mind the cold (B) he does mind the cold (C) he didn’t mind cold (D) he did mind the cold 4. (A) it was the Summer that bothered him. (B) it is the Summer that bothered him. (C) it has been the Summer that bothered him. (D) it will be the Summer that bothered him. 5. (A) he is worried about his Mother. (B) he was worried about his Mother. (C) he will be worried about his Mother. (D) he has been worried about his Mother.











































































































II.









OR Read the following excerpt from an article that appeared in the magazine section of a local daily: The ban on single-use plastic is impractical. The purpose of articles like bags and packaging is ultimately to make human life easier. Plastic articles do this well, so they shouldn’t be banned. Write a paragraph to analyze the given argument. You could think about what alternative explanations might weaken the given conclusion and include rationale / evidence that would strengthen / counter the given argument.

















(Creative Writing) I. Write a letter to the Sales Manager, MMS Books, Ashok Vihar, New Delhi, placing order for four titles of books that you need for educational purpose. You are Sanjeev/Sanjana, 12/CA, Model Town, Kurukshetra.(100-120 words) (5) OR You are Trisha, a student of Class X of Townvale Public School, Pune. You are eager to join the National Theatre School, Mumbai after your board exams. Write a letter to the director of the theatre school seeking information regarding admission procedure, eligibility criteria, fees structure, placement opportunities, etc. (100-120 words) II. The chart below displays data about the number of digital devices purchased in Rishu nagar across the years 2015-2019. Write a paragraph in not more than 100-120 words analyzing the given data. (5)



[100-120 Words] [5]





V. Read the extract given below and attempt questions that follow. [40 Marks] I For the first twenty-four hours Mijbil was neither hostile nor friendly; he was simply aloof and indifferent, choosing to sleep on the floor as far from my bed as possible. The second night Mijbil came on to my bed in the small hours and remained asleep in the crook of my knees until the servant brought tea in the morning, and during the day he began to lose his apathy and take a keen, much too keen, interest in his surroundings. I made a body-belt for him and took him on a lead to the bathroom, where for half an hour he went wild with joy in the water, plunging and rolling in it, shooting up and down the length of the bathtub underwater, and making enough slosh and splash for a hippo. (1 × 5 = 5) Q. 1. "Mijbil was neither hostile nor friendly; he was simply aloof and indifferent... " From the actions of the given set of friends, choose the person whose behaviour is likely to resemble Mijbil's. (A) Akanksha rudely tells her best friend to stop calling. (B) Piyush was amicable since the day he walked into his new classroom.















SECTION - C (Literature)

@PROCBSE 372

Oswaal CBSE Question Bank Chapterwise & Topicwise, ENGLISH LANG. & LIT., Class-X



















(2)







(C) (D)



























































      

(3) (4) Option (2) Option (4) OR Read the extract given below and attempt questions that follow. “Ramlal stood rooted to the ground, his head bowed low with the weight of grief and shame. The flames of the sacred fire slowly died down. Everyone was gone. Ramlal turned to Bholi and said, “But what about you, no one will ever marry you now. What shall we do with you?” And Sulekha said in a voice that was calm and steady. “Don’t you worry, Pitaji! In your old age I will serve you and Mother and I will teach in the same School where I learnt so much. Isn’t that right, Ma’am?” The Teacher had all along stood in a corner, watching the Drama. “Yes, Bholi, of course,” she replied. And in her smiling eyes was the light of a deep satisfaction that an Artist feels when contemplating the completion of her Masterpiece.” (1 × 5 = 5) Q. 1. Ramlal stood rooted to the ground because he: (A) was moved by what he heard. (B) was influenced by Bholi’s words. (C) was in a state of shock. (D) was in an immovable position. Q. 2. Bholi had refused to get married as: (A) Her father couldn’t afford the dowry that was demanded. (B) The bridegroom had been greedy and was disrespectful. (C) The bridegroom had insulted her father. (D) Her father was getting her married to a man older to her. Q. 3. Pick the sentence that brings out the meaning of ‘contemplating’ as used in the extract. (A) Contemplating sharing my belongings with someone is definitely tough. (B) She took some time to respond as she was contemplating what to say. (C) I was contemplating my reflection in the mirror and was speechless. (D) She was contemplating through the pages of the document that was with her. Q. 4. Why did the Teacher stand in one corner watching the Drama? (A) She was elated to see what was happening. (B) She wanted to see what Bholi would be doing. (C) She didn’t want to interfere in a family matter. (D) She had faith in Bholi standing up for herself. (A) Option (1) (C) Option (3)

     

(1)

    





































(C) Rishabh became nasty as soon as he started earning money. (D) Urvashi was distant for a while after losing her grandmother. Q. 2. Select the option that correctly puts the following events in correct order. 1. Maxwell planned to take Mijbil to the bathroom. 2. Mijbil slept at a significant distance from Maxwell's bed. 3. Mijbil immensely enjoyed playing in water. 4. Mijbil slept in much closer vicinity of the author as opposed to the previous night. (A) 1, 3, 4, 2 (B) 3, 1, 2, 4 (C) 2, 4, 1, 3 (D) 4, 3, 1, 2 Q. 3. The words "slosh and splash" are examples of the onomatopoeic words that go together phonetically-resembling the sound that they describe. Choose the option that DOES NOT fit the given description. (A) hip -hop (B) bow -wow (C) tick- tock (D) pitter -patter Q. 4. Choose the option that correctly describes a body-belt in the given context as explained by the four people given below. (A) A body-belt is a device especially made for swimmers and scuba divers, which helps them breathe underwater. (B) Body-belts basically resemble the waist belts and keep our body upright, in a standing position. (C) Body-belts are used for restraining while doing certain activities, in order to prevent hurt. (D) A body-belt is an accessory used by body builders for weight training as a measure against muscle and bone injuries. Q. 5. Select the option that best describes the final action of the otter, based on the given passage.

@PROCBSE Q. 5. Pick the option that includes the correct matches of Column A with Column B. Column A

Column B

I.

Bholi

(i)

independent and confident

II.

Ramlal

(ii)

burdenless and free

(iii)

sense of contentment and accomplishment

III. Teacher







(1 × 5 = 5)













(iv) embarrassed and anxious (A) I-(ii); II-(iv); III-(iii) (B) I-(i); II-(iv); III-(iii) (C) I-(iii); II-(ii); III-(i) (D) I-(ii); II-(iii); III-(iv) II. Read the extract given below and attempt questions that follow.

He should be lurking in shadow, Sliding through long grass Near the water hole Where plump deer pass. Q. 1. According to the extract, the poet wishes for the tiger to be ‘sliding’ through the foliage as this would (A) assist in keeping the prey unsuspecting of the predator’s sound. (B) aid in camouflaging the presence of the predator before it rushes in. (C) help the predator pounce on the prey comfortably without getting tired. (D) Support the predator’s vision as it eyes its prey. Q. 2. Which fact DOES NOT connect with the significance of the water hole for the tiger? (A) Many tigers chase prey into the water and holds the victim’s head under water until it drowns. (B) Prey feed in the water on water-lilies, and often wander into the middle of the water hole, where they are vulnerable and easy for the tiger to kill. (C) Prey that has quenched its thirst ensures consumption of hydrated meat for the tiger. (D) Chasing the panicked prey from shallow to deep water where the tiger grabs it. Q. 3. Pick the option that DOES NOT use ‘lurking’ correctly to fill in the blank. (A) The thug was ______ in the alley late evening, for unsuspecting passers-by. (B) The hyena was ______ in its den after a good meal. (C) The detective cautioned her team about the ______ dangers likely to impact the case. (D) The prejudices ______ beneath the surface create misunderstandings. Q. 4. ‘shadow’ here, refers to the shadow of (A) the tiger. (B) long grass. (C) water hole. (D) deer Q. 5. Pick the phrase that DOES NOT suggest that the forest in the extract is lush. (A) long grass (B) the water hole (C) plump deer (D) lurking in shadow OR Read the following extracts and answer the questions/complete the sentences that follow : (5×1=5) Some say the world will end in fire, Some say in ice. From what I’ve tasted of desire I hold with those who favor fire. But if it had to perish twice, I think I know enough of hate To say that for destruction ice Is also great And would suffice. Q. 1. Choose the CORRECT statement about the given poem. (A) Fire and ice are images — they help the readers visualise the power of nature over man. (B) Fire and ice are symbols — not of natural disasters, but of humanity’s ability to create disasters of its own. (C) Fire and ice are elements — not of Nature but man-made and possess the ability to create havoc for mankind. (D) Fire and ice are agents—they change the thinking of mankind from negative to positive and bring harmony. Q. 2. Select the option that correctly classifies the connotations for fire and ice, as suggested in the poem. (1) rage (2) violence (3) indifference (4) hate (5) greed (A) Fire- 3, 4; Ice- 1, 2, 5 (B) Fire - 2, 5; lce -1, 3, 4 (C) Fire - 1, 3, 5; Ice - 2, 4 (D) Fire - 1, 2, 4; Ice - 3, 5 Q. 3. The poem is a________, put across by the poet. (A) powerful warning (B) heart-felt apology (C) earnest appeal (D) vengeful threat

















































































































373





PRACTICE PAPER

@PROCBSE Oswaal CBSE Question Bank Chapterwise & Topicwise, ENGLISH LANG. & LIT., Class-X





Q. 4. The poet uses the phrasal verb -hold with. Choose the option that DOES NOT indicate a valid phrasal verb.

















































II. Answer any Two of the following questions in about 40 – 50 words each. [2 × 3=6] Q. 1. Bholi was a neglected child. Explain. Q. 2. How can you say that Horace Danby was good and respectable but not completely honest ? Q. 3. What difficulty do the crew of the space probe face on the Earth ? III. Answer any one of the following questions in about 100-120 words. [6 × 1=6] Q. 1. What is the epistemology of loss in this world of possessions ? How has the child learned to stand up in life? Q. 2. If Mijbil were living in the world of the poem, The Tale of Custard the Dragon, how do you think he would have (a) behaved with Custard? (b) reacted to the appearance of the pirate? Rationalise your answer. IV. Answer any one of the following questions in about 100-120 words. [6 × 1=6] Q. 1. Albert Einstein said, ‘The important thing is to never stop questioning.’ Richard was a genius who proved this quote true. Justify. Q. 2. After reading Bholi’s story you decide to write a blog on the importance of educating the girl child and how it empowers her. Write that blog expressing your views.













































































(A) Option 1 (B) Option 2 (C) Option 3 (D) Option 4 Q. 5. Pick the option that is NOT TRUE about the poet according to the extract. The poet (A) is inclined to believe that the world would most likely end with fire. (B) has heard divided opinions about the way the world would end in all likelihood. (C) preaches love and kindness to combat the spread of hate among all. (D) declares the power of ice to be as destructive as that of fire. VI Short and Long Type Questions: [4 × 3=12] I Answer any fore of the following questions in about 40 – 50 words each. Q. 1. What message might the Buddha’s story hold for those who are in positions of power and privilege? Q. 2. What legends are associated with the origin of tea? Q. 3. Is there a hidden message that the poet is wishing to convey or do you think that entertainment is the sole purpose of writing this poem ? [ The tale of Custard, the Dragon] Q. 4. What was the most fascinating thing that Valli saw on the street ? Q. 5. The crow and hemlock are usually used as negative references in literature. How is this different in this poem?





374

@PROCBSE EXCLUSIVE SCHOOL BOOKS SUPPLIERS HYDERABAD

TELANGANA

Sri Balaji Book Depot , 9676996199, (040) 27613300,

Shri Sharada Store & Stationers, 9000400069

ANDHRA PRADESH

VIJAYAWADA

Sri Vikas Book Centre, 9848571114, 9440715700

WEST KAMENG

Dutta Book Stall, 8729948473

TUMKUR

Palasandra Promoters, 9448214377, (0816 ) 2273220

CHENNAI

Book Mart, 7305151653

DEOGHAR

Bharti International, 06432359136, 9431132051

HYDERABAD VISAKHAPATHAM PORTBLAIR

ASSAM

KOLHAPUR PUNE

KARNATAKA

JALNA

TAMIL NADU

KOLKATA

JHARKHAND

ANDHRA PRADESH

DELHI

WEST BENGAL

Schoolwale & Company, 9731715655 United Book House, 9831344622

Yash Books House, 9637936999

MAHARASHTRA

Granth the Book World, 9922295522

MUMBAI

Shivam Books & Stationery, (022) 28230897, 9892935799

Krishna Book Centre, 9474205570

Vandhaman Educational, 9860574354, Renuka Book Distributor, 9765406133 NAGPUR

Book Emporium, 9675972993, 6000763186, Ashok Publication, 7896141127

BIHAR

Bokaro Student Friends, (0612) 2300600, 2618400, Gyan Ganga Ltd., 9304826651

Laxmi Pustakalay and Stationers, (0712) 2727354, Novelty Book Depot, 9657690200

Karmveer Book, 9923966466, Vijay Book, (0712) 2534217 DHULE YAVATMAL

Navjeevan Book Stall, 7020525561 Shri Ganesh Pustkalaya, 9423131275

ODISHA

Nova Publisher & Distributors, (0612) 2666404,

CUTTACK

Shri Durga Pustak Mandir, 9334477386, Vikas Book Depot, 9504780402

BHUBANESHWAR M/s Pragnya, 8847888616, 9437943777, Padmalaya, 9437026922

CHATTISGARH

KEONJHOAR

Shri Ramdev Traders, 9981761797

DELHI

Mittal Books, (011) 23288887, 9899037390, Shyam Brother, 9313619935

GUJARAT

AHMEDABAD

Patel Book, 9898184248, 9824386112, 9825900335

VAPI NAVSARI

Goutam Book Sellers, 9081790813 College Store, 8141913750, 9825099121

SURAT

Kazi Brothers, 7984880995

Shalibhadra Stationers, 9904477855, Zaveri Agency, 9979897312, 9979890330

Shopping Point, 0261-2230097

HARYANA

ROHTAK

Swami Kitab Ghar, 9255121948

BOKARO DHANBAD

Bokaro Student Friends, (0654) 2233094, 7360021503 Bokaro Student Friends, (0326) 3590527

DUMKA

Ashish Book Depot, 7798420420 Praveen Sales, 9890683475 Sai Shubham, 9975687687 Anil Paper Mart, 9422722522, (02482) 230733

KOLHAPUR

BARIPADA

BRAHMAPUR JALANDHAR

R. D. Chawla & Sons, 9899445522

DEOGHAR

MAHARASHTRA

ANDAMAN & NICOBAR ISLAND

Bokaro,Student Friends Pvt Ltd., 7004074900

RANCHI

Adithi Distributor, 9229171880

Central Book Shop, 9121152053 JBD Educatinals, 9246632691, 9246633693

Sharda Pustak Bhandar, 9334259293 RAIPUR

LATUR

Sri Balaji Book Depot, (040) 27613300, 9866355473,

Kayaan Enterprises, (0361) 2630443 PATNA

MADHYA PRADESH

OUR DISTRIBUTORS

ASSAM GUWAHATI

INDORE

KOTA BHILWARA JAIPUR

Pustak Mandir, 9431115138 Vidyanthi Pustak Bhandar, 9431310228 Bharti International, 9472754990 Navyug Sahitya Mandir, 9334375509, 9431367981

KARNATAKA

Trimurti Book World, 9437034735

Students corner, 7008435418

Sibani Book Store, 8763340822

PUNJAB

Cheap Book Store, 9872223458, 9878258592, Gaurav the Book World, 9478821183, Subhash Book Depot, 9876453625 City Book Shop, 9417440753

RAJASTHAN

Perfect Stationers & Gen. Shoppe, 9829863904 Vardhman Book Depot, 9571365020, Bhandari Stationers, 9001094271 Nakoda Book Depot, (01482) 243653, 9214983594 J K Enterprises, 9460983939, 9414782130, Ravi Enterprises, 9829060694 Saraswati Book House, (0141) 2610823, 9829811155

TAMIL NADU COIMBATORE CHENNAI

JHARKHAND

Bokaro Student Friends, 9234628152, Gyan Ganga Ltd., 9117889900, Crown Book Distributor & Publishers, (0651) 2213735,

A. K. Mishra Agencies, 9437025991, 9437081319

Majestic Book House, (0422) 2384333 Arraba Book Traders, (044) 25387868, 9841459105, M.R. Book Store (044) 25364596, Indian Book House, (044) 24327784, 9094129595, Kalaimagal Store, (044) 5544072, 9940619404, Vijaya Stores, 9381037417

PUDUCHERRY

AGRA

Sri Saraswathi Book Stall, (04132) 222283, 9092626287

UTTAR PRADESH

Sparsh Book Agency, 9412257817

Om Pustak Mandir, (0562) 2464014, 9319117771, Panchsheel Books, 9412257961, 9412257962,

HUBLI

Renuka Book Distributor, (0836) 2244124

ERNAKULAM

Asad Book Centre, (0484) 2370431, 9447314548, Academic Book House, (0484) 2376613 LUCKNOW

Rama Book Depot (Retail), (0522) 4080133, 9956922433, Vyapar Sadan, 7607102462

KOTTAYAM

Book Centre, (0481) 2566992

KANPUR

Universal Books, 8881093333, 7652043110 Raj Book Dist., 9235616506

CALICUT

Aman Book Stall, (0495) 2721282,

NOIDA

KOUAM

Surya Book Centre, 9847238314, H & C Store, 9864196344

Academic Book House, (0471) 2333349, 9447063349

Vasinnthe Book Stall, 9895072023

MADHYA PRADESH

GWALIOR

Agarwal Book Depot, 9425116210

INDORE

Sushil Prakashan,(0731) 2503333, 2535892, 9425322330

JABALPUR

Sangam General Store, (0761) 2412592, Vinay Pustak, 8962362667

MEERUT

VARANASI

GOA

Golden Heart Emporium, (0832) 2725208, 9370273479

PUNE

Natraj The Book Shopee, (020) 24485054, 9890054092,

Vidyarthi Sales Agencies, 9819776110, New Student Agencies, 7045065799

Vikas Book House, 9921331187, Sai Shubham, 9975687687, 9890043496

Akshar Books & Stationary, 7385089789,

The Wisdom Store, 9822588775, 02025433344

Pravin Sales, 9890683475, New Saraswati Granth Bhandar, 9422323859

Mehrotra Book Agency, (0532) 2266865, 9415636890 Sasta Sahitya Sadan, 9450029674

Azad Book Depot Pvt. Ltd., 7317000250, Book Seden, 9839487327,

Ideal Book Depot, (0121) 4059252, 9837066307

Prozo (Global Edu4 Share Pvt. Ltd), 9318395520 Goyal Books Oveseas, (0120) 4655555, 9873387003 Bokaro Student Friends, (0542) 2401250, 8299344607 Bookman & Company, 9935194495, Gupta Books, 9918155500

KOLKATA

WEST BENGAL

Oriental Publishers & Distributor (033) 40628367, Kath 'O' Kahini, (033) 22196313, 22419071,

New Radhika Book Palace, 9425411533-66

MARGO

MUMBAI

AZAMGARH

Saha Book House, (033) 22193671, 9333416484,

COOCH BEHAN SILIGURI

United Book House, 9831344622, Bijey Pustak Bhandar, 8961260603

S.B. Book Distributorm, Cockbchar, 9002670771 Agarwal Book House, (0353) 2535274, 9832038727

Modern Book, 8145578772

0206

TRIVANDRUM

KERALA

ALLAHABAD

Bookmark It-Books & Stat. Store, 7305151653, M.K. Store, 9840030099 Tiger Books Pvt. Ltd., 9710447000

@PROCBSE

PROCBSE THANK YOU FOR DOWNLOADING BOOK. WE HERE AT PROCBSE MAKE SURE THAT EVERY PIECE OF MATERIAL REACHES YOU WITHOUT DIVING DEEP INTO THE TABS OF YOUR BROWSER. THIS E-BOOK HAS BEEN DOWNLOADED TOTALLY FREE FROM PROCBSE ON TELEGRAM. UPI ID - PROCBSE.FAMC@IDFCBANK

CLICK TO VIEW/OPEN

@ PROCBSE JOIN US FOR QUESTIONS BANK , SAMPLE PAPER , PYQ , NOTES , MIND MAPS AND MANY MOTE THINGS ⚡

@PROCBSE

You can contribute a little to our efforts. Even a small amount can do wonders.

DONATE US PLEASE HELP US

OR

CONTRIBUTE TO OUR UPI ID PROCBSE.FAMC@IDFCBANK

@PROCBSE

@PROCBSE